Sie sind auf Seite 1von 333

STUDENT’S

SOLUTIONS MANUAL
JUDITH A. PENNA
Indiana University Purdue University Indianapolis

C OLLEGE A LGEBRA:
G RAPHS AND M ODELS
F IFTH E DITION

Marvin L. Bittinger
Indiana University Purdue University Indianapolis

Judith A. Beecher
Indiana University Purdue University Indianapolis

David J. Ellenbogen
Community College of Vermont

Judith A. Penna
Indiana University Purdue University Indianapolis

Boston Columbus Indianapolis New York San Francisco Upper Saddle River
Amsterdam Cape Town Dubai London Madrid Milan Munich Paris Montreal Toronto
Delhi Mexico City Sao Paulo Sydney Hong Kong Seoul Singapore Taipei Tokyo
The author and publisher of this book have used their best efforts in preparing this book. These efforts include the
development, research, and testing of the theories and programs to determine their effectiveness. The author and
publisher make no warranty of any kind, expressed or implied, with regard to these programs or the documentation
contained in this book. The author and publisher shall not be liable in any event for incidental or consequential
damages in connection with, or arising out of, the furnishing, performance, or use of these programs.

Reproduced by Pearson from electronic files supplied by the author.

Copyright © 2013, 2009, 2005 Pearson Education, Inc.


Publishing as Pearson, 75 Arlington Street, Boston, MA 02116.

All rights reserved. No part of this publication may be reproduced, stored in a retrieval system, or transmitted, in any
form or by any means, electronic, mechanical, photocopying, recording, or otherwise, without the prior written
permission of the publisher. Printed in the United States of America.

ISBN-13: 978-0-321-79125-2
ISBN-10: 0-321-79125-8

1 2 3 4 5 6 BRR 15 14 13 12 11

www.pearsonhighered.com
Contents
Chapter R . . . . . . . . . . . . . . . . . . . . . . . . . . . 1

Chapter 1 . . . . . . . . . . . . . . . . . . . . . . . . . . 25

Chapter 2 . . . . . . . . . . . . . . . . . . . . . . . . . 63

Chapter 3 . . . . . . . . . . . . . . . . . . . . . . . . . 93

Chapter 4 . . . . . . . . . . . . . . . . . . . . . . . . . . 129

Chapter 5 . . . . . . . . . . . . . . . . . . . . . . . . . 181

Chapter 6 . . . . . . . . . . . . . . . . . . . . . . . . . 215

Chapter 7 . . . . . . . . . . . . . . . . . . . . . . . . . 275

Chapter 8 . . . . . . . . . . . . . . . . . . . . . . . . . 307

Copyright © 2013 Pearson Education, Inc.


Chapter R
Basic Concepts of Algebra
23. The endpoint −9 is included in the interval, so we use a
Exercise Set R.1 bracket before the −9. The endpoint −4 is not included,
so we use a parenthesis after the −4. Interval notation is
2 √ 1 8 [−9, −4).
1. Rational numbers: , 6, −2.45, 18.4, −11, 3 27, 5 , − ,
√ 3 6 7
0, 16 25. Both endpoints are included in the interval, so we use
√ √ √ √ brackets. Interval notation is [x, x + h].
3. Irrational numbers: 3, 6 26, 7.151551555 . . . , − 35, 5 3
27. The endpoint p is not included in the interval, so we use a
(Although there is a pattern in 7.151551555 . . . , there is
parenthesis before the p. The interval is of unlimited ex-
no repeating block of digits.)
tent in the positive direction, so we use the infinity symbol
√ √ ∞. Interval notation is (p, ∞).
5. Whole numbers: 6, 3 27, 0, 16

7. Integers but not natural numbers: −11, 0 29. Since 6 is an element of the set of natural numbers, the
statement is true.
2 1
9. Rational numbers but not integers: , −2.45, 18.4, 5 , 31. Since 3.2 is not an element of the set of integers, the state-
3 6
8 ment is false.

7 11
33. Since − is an element of the set of rational numbers,
11. This is a closed interval, so we use brackets. Interval no- 5
tation is [−5, 5]. the statement is true.

35. Since 11 is an element of the set of real numbers, the
⫺5 0 5 statement is false.

13. This is a half-open interval. We use a parenthesis on 37. Since 24 is an element of the set of whole numbers, the
the left and a bracket on the right. Interval notation is statement is false.
(−3, −1].
39. Since 1.089 is not an element of the set of irrational num-
bers, the statement is true.
⫺3 ⫺1 0
41. Since every whole number is an integer, the statement is
15. This interval is of unlimited extent in the negative direc- true.
tion, and the endpoint −2 is included. Interval notation is
(−∞, −2]. 43. Since every rational number is a real number, the state-
ment is true.

⫺2 0 45. Since there are real numbers that are not integers, the
statement is false.
17. This interval is of unlimited extent in the positive direc-
tion, and the endpoint 3.8 is not included. Interval nota- 47. The sentence 3 + y = y + 3 illustrates the commutative
tion is (3.8, ∞). property of addition.

49. The sentence −3 · 1 = −3 illustrates the multiplicative


0 3.8 identity property.

19. {x|7 < x}, or {x|x > 7}. 51. The sentence 5 · x = x · 5 illustrates the commutative prop-
This interval is of unlimited extent in the positive direction erty of multiplication.
and the endpoint 7 is not included. Interval notation is
53. The sentence 2(a+b) = (a+b)2 illustrates the commutative
(7, ∞).
property of multiplication.

0 7
55. The sentence −6(m + n) = −6(n + m) illustrates the com-
mutative property of addition.
21. The endpoints 0 and 5 are not included in the interval, so 1
we use parentheses. Interval notation is (0, 5). 57. The sentence 8 · = 1 illustrates the multiplicative inverse
8
property.

Copyright © 2013 Pearson Education, Inc.


2 Chapter R: Basic Concepts of Algebra

59. The distance of −8.15 from 0 is 8.15, so | − 8.15| = 8.15. 9. z 0 · z 7 = z 0+7 = z 7 , or


z0 · z7 = 1 · z7 = z7
61. The distance 295 from 0 is 295, so |295| = 295.
√ √ √ √ 11. 58 · 5−6 = 58+(−6) = 52 , or 25
63. The distance of − 97 from 0 is 97, so | − 97| = 97.
13. m−5 · m5 = m−5+5 = m0 = 1
65. The distance of 0 from 0 is 0, so |0| = 0.
5 5 15. y 3 · y −7 = y 3+(−7) = y −4 , or
1
5 5  
67. The distance of from 0 is , so   = . y4
4 4 4 4
69. |14 − (−8)| = |14 + 8| = |22| = 22, or 17. (x + 3)4 (x + 3)−2 = (x + 3)4+(−2) = (x + 3)2
| − 8 − 14| = | − 22| = 22 19. 3−3 · 38 · 3 = 3−3+8+1 = 36 , or 729
71. | − 3 − (−9)| = | − 3 + 9| = |6| = 6, or 21. 2x3 · 3x2 = 2 · 3 · x3+2 = 6x5
| − 9 − (−3)| = | − 9 + 3| = | − 6| = 6
23. (−3a−5 )(5a−7 ) = −3 · 5 · a−5+(−7) = −15a−12 , or
73. |12.1 − 6.7| = |5.4| = 5.4, or 15
− 12
|6.7 − 12.1| = | − 5.4| = 5.4 a
      25. (6x−3 y 5 )(−7x2 y −9 ) = 6(−7)x−3+2 y 5+(−9) =
     
75.  − 3 − 15  =  − 6 − 15  =  − 21  = 21 , or
 4     8 42
 
8
 
8 8
 
8
   −42x−1 y −4 , or − 4
 15   15 3   15 6   21  21 xy
 − − 3 = + = + = =
8 4  8 4  8 8  8  8 27. (2x)4 (3x)3 = 24 · x4 · 33 · x3 = 16 · 27 · x4+3 = 432x7
77. | − 7 − 0| = | − 7| = 7, or 29. (−2n)3 (5n)2 = (−2)3 n3 · 52 n2 = −8 · 25 · n3+2 =
|0 − (−7)| = |0 + 7| = |7| = 7 −200n5
79. Answers may vary. One such number is y 35
0.124124412444 . . . . 31. = y 35−31 = y 4
y 31
1
81. Answers may vary. Since − = 0.0099 and b−7 1
101 33. = b−7−12 = b−19 , or 19
1 b12 b
− = −0.01, one such number is −0.00999.
100
x2 y −2 x3
35. = x2−(−1) y −2−1 = x3 y −3 , or 3
83. Since 12 + 32 = 10, the hypotenuse of a right triangle
√ with −1
x y y
legs of lengths 1 unit and 3 units has a length of 10 units.
32x−4 y 3 32 −4−(−5) 3−8 8x
37. = x y = 8xy −5 , or 5
✏ 4x−5 y 8
c ✏✏
2 2 2 4 y
c = 1 +3
✏ ✏✏ 1
✏✏ c2 = 10
√ 39. (2x2 y)4 = 24 (x2 )4 y 4 = 16x2·4 y 4 = 16x8 y 4
3 c = 10
41. (−2x3 )5 = (−2)5 (x3 )5 = (−2)5 x3·5 = −32x15

Exercise Set R.2 43. (−5c−1 d−2 )−2 = (−5)−2 c−1(−2) d−2(−2) =
c2 d 4 c2 d 4
  =
1 1 (−5) 2 25
1. 3−7 = a−m = , a = 0
37 am
45. (3m4 )3 (2m−5 )4 = 33 m12 · 24 m−20 =
3. Observe that each exponent is negative. We move each 432
factor to the other side of the fraction bar and change the 27 · 16m12+(−20) = 432m−8 , or 8
m
sign of each exponent.
 −3 7 3
x−5 y4 2x y (2x−3 y 7 )3 23 x−9 y 21
= 5 47. −1
= −1
= =
y −4 x z (z ) 3 z −3
8x−9 y 21 8y 21 z 3
5. Observe that each exponent is negative. We move each , or
factor to the other side of the fraction bar and change the z −3 x9
sign of each exponent.   −5
24a10 b−8 c7
m−1 n−12 t6 t6 49. 6 −3 5
= (2a4 b−5 c2 )−5 = 2−5 a−20 b25 c−10 ,
= , or 12a b c
t−6 m1 n12 mn12
b25
or
7. 230 = 1 (For any nonzero real number, a0 = 1.) 32a20 c10

Copyright © 2013 Pearson Education, Inc.


Exercise Set R.2 3

51. Convert 16,500,000 to scientific notation. 71. (4.2 × 107 )(3.2 × 10−2 )
We want the decimal point to be positioned between the = (4.2 × 3.2) × (107 × 10−2 )
1 and the 6, so we move it 7 places to the left. Since = 13.44 × 105 This is not scientific notation.
16,500,000 is greater than 10, the exponent must be posi-
tive. = (1.344 × 10) × 105

16, 500, 000 = 1.65 × 107 = 1.344 × 106 Writing scientific notation

73. (2.6 × 10−18 )(8.5 × 107 )


53. Convert 0.000000437 to scientific notation.
We want the decimal point to be positioned between the = (2.6 × 8.5) × (10−18 × 107 )
4 and the 3, so we move it 7 places to the right. Since = 22.1 × 10−11 This is not scientific notation.
0.000000437 is a number between 0 and 1, the exponent = (2.21 × 10) × 10−11
must be negative.
= 2.21 × 10−10
0.000000437 = 4.37 × 10−7
6.4 × 10−7 6.4 10−7
55. Convert 234,600,000,000 to scientific notation. We want 75. = ×
8.0 × 106 8.0 106
the decimal point to be positioned between the 2 and the = 0.8 × 10−13 This is not scientific
3, so we move it 11 places to the left. Since 234,600,000,000 notation.
is greater than 10, the exponent must be positive. = (8 × 10−1 ) × 10−13
234, 600, 000, 000 = 2.346 × 1011 = 8 × 10−14 Writing scientific
57. Convert 0.00104 to scientific notation. We want the deci- notation
mal point to be positioned between the 1 and the last 0, so 1.8 × 10−3
we move it 3 places to the right. Since 0.00104 is a number 77.
7.2 × 10−9
between 0 and 1, the exponent must be negative.
1.8 10−3
0.00104 = 1.04 × 10−3 = ×
7.2 10−9
59. Convert 0.00000000000000000000000000167 to scientific = 0.25 × 106 This is not scientific notation.
notation. = (2.5 × 10−1 ) × 106
We want the decimal point to be positioned between the = 2.5 × 105
1 and the 6, so we move it 27 places to the right. Since
0.00000000000000000000000000167 is a number between 0 79. The average number of pieces of trash per mile is the total
and 1, the exponent must be negative. number of pieces of trash divided by the number of miles.
0.00000000000000000000000000167 = 1.67 × 10−27 51.2 billion 5.12 × 1010
=
61. Convert 7.6 × 105 to decimal notation. 76 million 7.6 × 107
≈ 0.6737 × 103
The exponent is positive, so the number is greater than
10. We move the decimal point 5 places to the right. ≈ (6.737 × 10−1 ) × 103
7.6 × 105 = 760, 000 ≈ 6.737 × 102
On average, there are about 6.737 × 102 pieces of trash on
63. Convert 1.09 × 10−7 to decimal notation. each mile of roadway.
The exponent is negative, so the number is between 0 and
1. We move the decimal point 7 places to the left. 81. The number of people per square mile is the total number
1.09 × 10−7 = 0.000000109 of people divided by the number of square miles.
38, 000 3.8 × 104
65. Convert 3.496 × 1010 to decimal notation. =
0.75 7.5 × 10−1
The exponent is positive, so the number is greater than ≈ 0.50667 × 105
10. We move the decimal point 10 places to the right. ≈ (5.0667 × 10−1 ) × 105
3.496 × 1010 = 34, 960, 000, 000 ≈ 5.0667 × 104
67. Convert 5.41 × 10−8 to decimal notation. There are about 5.0667 × 104 people per square mile.
The exponent is negative, so the number is between 0 and
83. We multiply the number of light years by the number of
1. We move the decimal point 8 places to the left.
miles in a light year.
5.41 × 10−8 = 0.0000000541
4.22 × 5.88 × 1012 = 24.8136 × 1012
69. Convert 2.319 × 10 to decimal notation.
8
= (2.48136 × 10) × 1012
The exponent is positive, so the number is greater than = 2.48136 × 1013
10. We move the decimal point 8 places to the right. The distance from Earth to Alpha Centauri C is
2.319 × 108 = 231, 900, 000 2.48136 × 1013 mi.

Copyright © 2013 Pearson Education, Inc.


4 Chapter R: Basic Concepts of Algebra

85. First find the number of seconds in 1 hour: 93. Since interest is compounded semiannually, n = 2. Substi-
1 hour = 1 ✧ hr ×
✦ × 60 sec = 3600 sec
60 min tute $3225 for P , 3.1% or 0.031 for i, 2 for n, and 4 for t
1✧
hr ✦
1 min in the compound interest formula.
The number of disintegrations produced in 1 hour is the  i nt
number of disintegrations per second times the number of A = P 1+
n
seconds in 1 hour.  0.031 2·4
37 billion × 3600 = $3225 1 + Substituting
2
= 37, 000, 000, 000 × 3600 2·4
= $3225(1 + 0.0155) Dividing
= 3.7 × 1010 × 3.6 × 103 Writing scientific = $3225(1.0155)2·4 Adding
notation = $3225(1.0155)8 Multiplying 2 and 4
= (3.7 × 3.6) × (1010 × 103 )
≈ $3225(1.130939628) Evaluating the
= 13.32 × 1013 Multiplying exponential expression
= (1.332 × 10) × 1013 ≈ $3647.2803 Multiplying
= 1.332 × 10 14 ≈ $3647.28 Rounding to the nearest cent
One gram of radium produces 1.332 × 1014 disintegrations 95. Since interest is compounded quarterly, n = 4. Substitute
in 1 hour. $4100 for P , 2.3% or 0.023 for i, 4 for n, and 6 for t in the
compound interest formula.
87. = 5 · 3 + 8 · 32 + 4(6 − 2)  i nt
= 5 · 3 + 8 · 32 + 4 · 4 Working inside parentheses A = P 1+
n
= 5·3+8·9+4·4 Evaluating 32  0.023 4·6
= $4100 1 + Substituting
= 15 + 72 + 16 Multiplying 4
= 87 + 16 Adding in order = $4100(1 + 0.00575)4·6 Dividing
= 103 from left to right = $4100(1.00575)4·6 Adding
= $4100(1.00575)24 Multiplying 4 and 6
89. 16 ÷ 4 · 4 ÷ 2 · 256
≈ $4100(1.147521919) Evaluating the
= 4 · 4 ÷ 2 · 256 Multiplying and dividing exponential expression
in order from left to right ≈ $4704.839868 Multiplying
= 16 ÷ 2 · 256
≈ $4704.84 Rounding to the nearest cent
= 8 · 256
= 2048 97. Substitute $250 for P , 0.05 for r and 27 for t and perform
the resulting computation.
4(8 − 6) − 4 · 3 + 2 · 8
2
 
91. r 12·t
31 + 190 1+ −1
4 · 22 − 4 · 3 + 2 · 8 S=P 12
r

= Calculating in the
3+1 12
numerator and in  0.05 12·27 
the denominator 1+ −1
4·4−4·3+2·8 = $250  12 
= 0.05
4
16 − 12 + 16 12
= ≈ $170, 797.30
4
4 + 16
= 99. Substitute $120,000 for S, 0.03 for r, and 18 for t and solve
4
for P .
20  
= r 12·t
4 1+ −1
= 5 S=P 12
r

12
 0.03 12·18 
1+ −1
$120, 000 = P  12
0.03

12


(1.0025)216 − 1
$120, 000 = P
0.0025
$120, 000 ≈ P (285.94035)
$419.67 ≈ P

Copyright © 2013 Pearson Education, Inc.


Exercise Set R.3 5

101. (xt · x3t )2 = (x4t )2 = x4t·2 = x8t 19. (y − 3)(y + 5)

103. (ta+x · tx−a )4 = (t2x )4 = t2x·4 = t8x = y 2 + 5y − 3y − 15 Using FOIL


(3xa y b )3 2 27x3a y 3b 2 = y 2 + 2y − 15 Collecting like terms
105. =
(−3xa y b )2 9x2a y 2b 21. (x + 6)(x + 3)
 a b 2
= 3x y = x2 + 3x + 6x + 18 Using FOIL
2a 2b 2
= 9x y = x + 9x + 18 Collecting like terms

23. (2a + 3)(a + 5)


Exercise Set R.3
= 2a2 + 10a + 3a + 15 Using FOIL
1. 7x − 4x + 8x + 5 = 7x + (−4x ) + 8x + 5
3 2 3 2 2
= 2a + 13a + 15 Collecting like terms
Terms: 7x3 , −4x2 , 8x, 5 25. (2x + 3y)(2x + y)
The degree of the term of highest degree, 7x3 , is 3. Thus,
= 4x2 + 2xy + 6xy + 3y 2 Using FOIL
the degree of the polynomial is 3.
2 2
= 4x + 8xy + 3y
3. 3a4 b − 7a3 b3 + 5ab − 2 = 3a4 b + (−7a3 b3 ) + 5ab + (−2)
27. (x + 3)2
Terms: 3a4 b, −7a3 b3 , 5ab, −2
= x2 + 2 · x · 3 + 32
The degrees of the terms are 5, 6, 2, and, 0, respectively,
so the degree of the polynomial is 6. [(A + B)2 = A2 + 2AB + B 2 ]
2
= x + 6x + 9
5. (3ab2 − 4a2 b − 2ab + 6)+
(−ab2 − 5a2 b + 8ab + 4) 29. (y − 5)2
= (3 − 1)ab2 + (−4 − 5)a2 b + (−2 + 8)ab + (6 + 4) = y 2 − 2 · y · 5 + 52
= 2ab2 − 9a2 b + 6ab + 10 [(A − B)2 = A2 − 2AB + B 2 ]

7. (2x + 3y + z − 7) + (4x − 2y − z + 8)+ = y − 10y + 25


2

(−3x + y − 2z − 4) 31. (5x − 3)2


= (2 + 4 − 3)x + (3 − 2 + 1)y + (1 − 1 − 2)z+ = (5x)2 − 2 · 5x · 3 + 32
(−7 + 8 − 4) [(A − B)2 = A2 − 2AB + B 2 ]
= 3x + 2y − 2z − 3 = 25x2 − 30x + 9
9. (3x2 − 2x − x3 + 2) − (5x2 − 8x − x3 + 4) 33. (2x + 3y)2
= (3x − 2x − x + 2) + (−5x + 8x + x − 4)
2 3 2 3
= (2x)2 + 2(2x)(3y) + (3y)2
= (3 − 5)x + (−2 + 8)x + (−1 + 1)x + (2 − 4)
2 3
[(A+B)2 = A2 +2AB +B 2 ]
= −2x + 6x − 2
2
= 4x2 + 12xy + 9y 2
11. (x4 − 3x2 + 4x) − (3x3 + x2 − 5x + 3)
35. (2x2 − 3y)2
= (x4 − 3x2 + 4x) + (−3x3 − x2 + 5x − 3)
= (2x2 )2 − 2(2x2 )(3y) + (3y)2
= x4 − 3x3 + (−3 − 1)x2 + (4 + 5)x − 3
[(A − B)2 = A2 − 2AB + B 2 ]
= x4 − 3x3 − 4x2 + 9x − 3 = 4x − 12x y + 9y 2
4 2

13. (3a2 )(−7a4 ) = [3(−7)](a2 · a4 ) 37. (n + 6)(n − 6)


= −21a6 = n2 − 62 [(A + B)(A − B) = A2 − B 2 ]
15. (6xy )(9x y ) = (6 · 9)(x · x )(y · y )
3 4 2 4 3 2
= n2 − 36
5 5
= 54x y
39. (3y + 4)(3y − 4)
17. (a − b)(2a − ab + 3b2 )
3
= (3y)2 − 42 [(A + B)(A − B) = A2 − B 2 ]
= (a − b)(2a ) + (a − b)(−ab) + (a − b)(3b )
3 2
= 9y − 16
2

Using the distributive property


41. (3x − 2y)(3x + 2y)
= 2a4 − 2a3 b − a2 b + ab2 + 3ab2 − 3b3
= (3x)2 − (2y)2 [(A−B)(A+B) = A2 −B 2 ]
Using the distributive property
three more times = 9x2 − 4y 2
= 2a4 − 2a3 b − a2 b + 4ab2 − 3b3 Collecting like
terms

Copyright © 2013 Pearson Education, Inc.


6 Chapter R: Basic Concepts of Algebra

43. (2x + 3y + 4)(2x + 3y − 4) 15. x3 − x2 − 5x + 5


= [(2x + 3y) + 4][(2x + 3y) − 4] = x2 (x − 1) − 5(x − 1)
= (2x + 3y) − 4 2 2
= (x − 1)(x2 − 5)
= 4x2 + 12xy + 9y 2 − 16
17. a3 − 3a2 − 2a + 6
45. (x + 1)(x − 1)(x2 + 1) = a2 (a − 3) − 2(a − 3)
= (x − 1)(x + 1)
2 2
= (a − 3)(a2 − 2)
= x4 − 1
19. w2 − 7w + 10
47. (a + b )(a − b ) = (a ) − (b )
n n n n n 2 n 2
We look for two numbers with a product of 10 and a sum
= a2n − b2n of −7. By trial, we determine that they are −5 and −2.
w2 − 7w + 10 = (w − 5)(w − 2)
49. (an + bn )2 = (an )2 + 2 · an · bn + (bn )2
= a2n + 2an bn + b2n 21. x2 + 6x + 5
We look for two numbers with a product of 5 and a sum
51. (x − 1)(x2 + x + 1)(x3 + 1)
of 6. By trial, we determine that they are 1 and 5.
= [(x − 1)x2 + (x − 1)x + (x − 1) · 1](x3 + 1)
x2 + 6x + 5 = (x + 1)(x + 5)
= (x3 − x2 + x2 − x + x − 1)(x3 + 1)
= (x3 − 1)(x3 + 1) 23. t2 + 8t + 15
= (x3 )2 − 12 We look for two numbers with a product of 15 and a sum
of 8. By trial, we determine that they are 3 and 5.
= x6 − 1
t2 + 8t + 15 = (t + 3)(t + 5)
53. (xa−b )a+b
25. x2 − 6xy − 27y 2
= x(a−b)(a+b)
2
−b2
We look for two numbers with a product of −27 and a sum
= xa of −6. By trial, we determine that they are 3 and −9.
55. (a + b + c)2 x2 − 6xy − 27y 2 = (x + 3y)(x − 9y)
= (a + b + c)(a + b + c)
27. 2n2 − 20n − 48 = 2(n2 − 10n − 24)
= (a + b + c)(a) + (a + b + c)(b) + (a + b + c)(c)
Now factor n2 − 10n − 24. We look for two numbers with a
= a2 + ab + ac + ab + b2 + bc + ac + bc + c2 product of −24 and a sum of −10. By trial, we determine
= a2 + b2 + c2 + 2ab + 2ac + 2bc that they are 2 and −12. Then n2 − 10n − 24 =
(n + 2)(n − 12). We must include the common factor, 2,
to have a factorization of the original trinomial.
Exercise Set R.4 2n2 − 20n − 48 = 2(n + 2)(n − 12)

1. 3x + 18 = 3 · x + 3 · 6 = 3(x + 6) 29. y 2 − 4y − 21
We look for two numbers with a product of −21 and a sum
3. 2z 3 − 8z 2 = 2z 2 · z − 2z 2 · 4 = 2z 2 (z − 4) of −4. By trial, we determine that they are 3 and −7.
5. 4a2 − 12a + 16 = 4 · a2 − 4 · 3a + 4 · 4 = 4(a2 − 3a + 4) y 2 − 4y − 21 = (y + 3)(y − 7)

7. a(b − 2) + c(b − 2) = (b − 2)(a + c) 31. y 4 − 9y 3 + 14y 2 = y 2 (y 2 − 9y + 14)


Now factor y 2 − 9y + 14. Look for two numbers with a
9. 3x3 − x2 + 18x − 6 product of 14 and a sum of −9. The numbers are −2 and
= x2 (3x − 1) + 6(3x − 1) −7. Then y 2 − 9y + 14 = (y − 2)(y − 7). We must include
= (3x − 1)(x2 + 6) the common factor, y 2 , in order to have a factorization of
the original trinomial.
11. y 3 − y 2 + 2y − 2 y 4 − 9y 3 + 14y 2 = y 2 (y − 2)(y − 7)
= y (y − 1) + 2(y − 1)
2
33. 2x3 − 2x2 y − 24xy 2 = 2x(x2 − xy − 12y 2 )
= (y − 1)(y 2 + 2)
Now factor x2 − xy − 12y 2 . Look for two numbers with a
13. 24x − 36x + 72x − 108
3 2 product of −12 and a sum of −1. The numbers are −4 and
= 12(2x3 − 3x2 + 6x − 9) 3. Then x2 −xy−12y 2 = (x−4y)(x+3y). We must include
the common factor, 2x, in order to have a factorization of
= 12[x2 (2x − 3) + 3(2x − 3)] the original trinomial.
= 12(2x − 3)(x2 + 3) 2x3 − 2x2 y − 24xy 2 = 2x(x − 4y)(x + 3y)

Copyright © 2013 Pearson Education, Inc.


Exercise Set R.4 7

35. 2n2 + 9n − 56 4. Split the middle term using the numbers found in
We use the FOIL method. step (3):
1. There is no common factor other than 1 or −1. y = 4y − 3y
2. The factorization must be of the form 5. Factor by grouping.
(2n+ )(n+ ). 2y 2 + y − 6 = 2y 2 + 4y − 3y − 6
3. Factor the constant term, −56. The possibilities = 2y(y + 2) − 3(y + 2)
are −1 · 56, 1(−56), −2 · 28, 2(−28), −4 · 16, 4(−16), = (y + 2)(2y − 3)
−7 · 8, and 7(−8). The factors can be written in
the opposite order as well: 56(−1), −56 · 1, 28(−2), 43. 6a2 − 29ab + 28b2
−28 · 2, 16(−4), −16 · 4, 8(−7), and −8 · 7.
We use the FOIL method.
4. Find a pair of factors for which the sum of the out-
1. There is no common factor other than 1 or −1.
side and the inside products is the middle term,
9n. By trial, we determine that the factorization 2. The factorization must be of the form
is (2n − 7)(n + 8). (6x+ )(x+ ) or (3x+ )(2x+ ).
3. Factor the coefficient of the last term, 28. The pos-
37. 12x2 + 11x + 2 sibilities are 1 · 28, −1(−28), 2 · 14, −2(−14), 4 · 7,
We use the grouping method. and −4(−7). The factors can be written in the op-
1. There is no common factor other than 1 or −1. posite order as well: 28·1, −28(−1), 14·2, −14(−2),
7 · 4, and −7(−4).
2. Multiply the leading coefficient and the constant:
12 · 2 = 24. 4. Find a pair of factors for which the sum of the out-
side and the inside products is the middle term,
3. Try to factor 24 so that the sum of the factors is the −29. Observe that the second term of each bino-
coefficient of the middle term, 11. The factors we mial factor will contain a factor of b. By trial, we
want are 3 and 8. determine that the factorization is (3a−4b)(2a−7b).
4. Split the middle term using the numbers found in
step (3): 45. 12a2 − 4a − 16
11x = 3x + 8x We will use the grouping method.
5. Factor by grouping. 1. Factor out the common factor, 4.
12x2 + 11x + 2 = 12x2 + 3x + 8x + 2 12a2 − 4a − 16 = 4(3a2 − a − 4)
= 3x(4x + 1) + 2(4x + 1) 2. Now consider 3a2 − a − 4. Multiply the leading
coefficient and the constant: 3(−4) = −12.
= (4x + 1)(3x + 2)
3. Try to factor −12 so that the sum of the factors is
2
39. 4x + 15x + 9 the coefficient of the middle term, −1. The factors
we want are −4 and 3.
We use the FOIL method.
4. Split the middle term using the numbers found in
1. There is no common factor other than 1 or −1.
step (3):
2. The factorization must be of the form
−a = −4a + 3a
(4x+ )(x+ ) or (2x+ )(2x+ ).
5. Factor by grouping.
3. Factor the constant term, 9. The possibilities are
1 · 9, −1(−9), 3 · 3, and −3(−3). The first two pairs 3a2 − a − 4 = 3a2 − 4a + 3a − 4
of factors can be written in the opposite order as = a(3a − 4) + (3a − 4)
well: 9 · 1, −9(−1).
= (3a − 4)(a + 1)
4. Find a pair of factors for which the sum of the out- We must include the common factor to get a factor-
side and the inside products is the middle term, ization of the original trinomial.
15x. By trial, we determine that the factorization
is (4x + 3)(x + 3). 12a2 − 4a − 16 = 4(3a − 4)(a + 1)

41. 2y 2 + y − 6 47. z 2 − 81 = z 2 − 92 = (z + 9)(z − 9)


We use the grouping method. 49. 16x2 − 9 = (4x)2 − 32 = (4x + 3)(4x − 3)
1. There is no common factor other than 1 or −1.
51. 6x2 − 6y 2 = 6(x2 − y 2 ) = 6(x + y)(x − y)
2. Multiply the leading coefficient and the constant:
2(−6) = −12. 53. 4xy 4 − 4xz 2 = 4x(y 4 − z 2 )
3. Try to factor −12 so that the sum of the factors is = 4x[(y 2 )2 − z 2 ]
the coefficient of the middle term, 1. The factors we
= 4x(y 2 + z)(y 2 − z)
want are 4 and −3.

Copyright © 2013 Pearson Education, Inc.


8 Chapter R: Basic Concepts of Algebra

55. 7pq 4 − 7py 4 = 7p(q 4 − y 4 ) 87. x2 + 9x + 20


= 7p[(q 2 )2 − (y 2 )2 ] We look for two numbers with a product of 20 and a sum
= 7p(q 2 + y 2 )(q 2 − y 2 ) of 9. They are 4 and 5.
= 7p(q 2 + y 2 )(q + y)(q − y) x2 + 9x + 20 = (x + 4)(x + 5)

57. x2 + 12x + 36 = x2 + 2 · x · 6 + 62 89. y 2 − 6y + 5


= (x + 6)2 We look for two numbers with a product of 5 and a sum
of −6. They are −5 and −1.
59. 9z 2 − 12z + 4 = (3z)2 − 2 · 3z · 2 + 22 = (3z − 2)2 y 2 − 6y + 5 = (y − 5)(y − 1)

61. 1 − 8x + 16x2 = 12 − 2 · 1 · 4x + (4x)2 91. 2a2 + 9a + 4


= (1 − 4x) 2
We use the FOIL method.
63. a3 + 24a2 + 144a 1. There is no common factor other than 1 or −1.
= a(a2 + 24a + 144) 2. The factorization must be of the form
(2a+ )(a+ ).
= a(a2 + 2 · a · 12 + 122 )
3. Factor the constant term, 4. The possibilities are
= a(a + 12)2 1 · 4, −1(−4), and 2 · 2. The first two pairs of factors
65. 4p2 − 8pq + 4q 2 can be written in the opposite order as well: 4 · 1,
−4(−1).
= 4(p2 − 2pq + q 2 )
4. Find a pair of factors for which the sum of the out-
= 4(p − q)2 side and the inside products is the middle term,
9a. By trial, we determine that the factorization
67. x3 + 64 = x3 + 43
is (2a + 1)(a + 4).
= (x + 4)(x2 − 4x + 16)
93. 6x2 + 7x − 3
69. m3 − 216 = m3 − 63
We use the grouping method.
= (m − 6)(m2 + 6m + 36)
1. There is no common factor other than 1 or −1.
71. 8t3 + 8 = 8(t3 + 1) 2. Multiply the leading coefficient and the constant:
= 8(t3 + 13 ) 6(−3) = −18.
= 8(t + 1)(t2 − t + 1) 3. Try to factor −18 so that the sum of the factors is
the coefficient of the middle term, 7. The factors we
73. 3a5 − 24a2 = 3a2 (a3 − 8) want are 9 and −2.
= 3a2 (a3 − 23 ) 4. Split the middle term using the numbers found in
= 3a2 (a − 2)(a2 + 2a + 4) step (3):
7x = 9x − 2x
75. t6 + 1 = (t2 )3 + 13
5. Factor by grouping.
= (t2 + 1)(t4 − t2 + 1)
6x2 + 7x − 3 = 6x2 + 9x − 2x − 3
77. 18a b − 15ab = 3ab · 6a − 3ab · 5b
2 2
= 3x(2x + 3) − (2x + 3)
= 3ab(6a − 5b) = (2x + 3)(3x − 1)
79. x3 − 4x2 + 5x − 20 = x2 (x − 4) + 5(x − 4) 95. y 2 − 18y + 81 = y 2 − 2 · y · 9 + 92
= (x − 4)(x + 5) 2
= (y − 9)2
81. 8x2 − 32 = 8(x2 − 4) 97. 9z 2 − 24z + 16 = (3z)2 − 2 · 3z · 4 + 42
= 8(x + 2)(x − 2) = (3z − 4)2

83. 4y 2 − 5 99. x2 y 2 − 14xy + 49 = (xy)2 − 2 · xy · 7 + 72


There are no common factors. We might try to factor = (xy − 7)2
this polynomial as a difference of squares, but there is no
integer which yields 5 when squared. Thus, the polynomial 101. 4ax2 + 20ax − 56a = 4a(x2 + 5x − 14)
is prime. = 4a(x + 7)(x − 2)

85. m2 − 9n2 = m2 − (3n)2 103. 3z 3 − 24 = 3(z 3 − 8)


= (m + 3n)(m − 3n) = 3(z 3 − 23 )
= 3(z − 2)(z 2 + 2z + 4)

Copyright © 2013 Pearson Education, Inc.


Exercise Set R.5 9

105. 16a7 b + 54ab7 129. (x + h)3 − x3


= 2ab(8a6 + 27b6 ) = [(x + h) − x][(x + h)2 + x(x + h) + x2 ]
= 2ab[(2a ) + (3b ) ]2 3 2 3 = (x + h − x)(x2 + 2xh + h2 + x2 + xh + x2 )
= 2ab(2a2 + 3b2 )(4a4 − 6a2 b2 + 9b4 ) = h(3x2 + 3xh + h2 )

107. y 3 − 3y 2 − 4y + 12 131. (y − 4)2 + 5(y − 4) − 24


= y 2 (y − 3) − 4(y − 3) = u2 + 5u − 24 Substituting u for y − 4
= (y − 3)(y 2 − 4) = (u + 8)(u − 3)
= (y − 3)(y + 2)(y − 2) = (y − 4 + 8)(y − 4 − 3) Substituting y − 4
for u
109. x3 − x2 + x − 1 = (y + 4)(y − 7)
= x2 (x − 1) + (x − 1)
133. x2n + 5xn − 24 = (xn )2 + 5xn − 24
= (x − 1)(x2 + 1)
= (xn + 8)(xn − 3)
111. 5m4 − 20 = 5(m4 − 4)
135. x2 + ax + bx + ab = x(x + a) + b(x + a)
= 5(m2 + 2)(m2 − 2)
= (x + a)(x + b)
113. 2x + 6x − 8x − 24
3 2
137. 25y 2m − (x2n − 2xn + 1)
= 2(x3 + 3x2 − 4x − 12) = (5y m )2 − (xn − 1)2
= 2[x2 (x + 3) − 4(x + 3)] = [5y m + (xn − 1)][5y m − (xn − 1)]
= 2(x + 3)(x2 − 4) = (5y m + xn − 1)(5y m − xn + 1)
= 2(x + 3)(x + 2)(x − 2)
139. (y − 1)4 − (y − 1)2
115. 4c − 4cd − d = (2c) − 2 · 2c · d − d
2 2 2 2
= (y − 1)2 [(y − 1)2 − 1]
= (2c − d)2 = (y − 1)2 [y 2 − 2y + 1 − 1]
117. m6 + 8m3 − 20 = (m3 )2 + 8m3 − 20 = (y − 1)2 (y 2 − 2y)
We look for two numbers with a product of −20 and a sum = y(y − 1)2 (y − 2)
of 8. They are 10 and −2.
m6 + 8m3 − 20 = (m3 + 10)(m3 − 2) Exercise Set R.5
119. p − 64p4 = p(1 − 64p3 ) 1. x − 5 = 7
= p[13 − (4p)3 ] x = 12 Adding 5
= p(1 − 4p)(1 + 4p + 16p2 ) The solution is 12.
121. y − 84 + 5y
4 2
3. 3x + 4 = −8
= y 4 + 5y 2 − 84 3x = −12 Subtracting 4
= u2 + 5u − 84 Substituting u for y 2 x = −4 Dividing by 3
= (u + 12)(u − 7) The solution is −4.
= (y 2 + 12)(y 2 − 7) Substituting y 2 for u
5. 5y − 12 = 3
8 2 2 8 5y = 15 Adding 12
123. y −
2
+ y = y2 + y −
49 7 7 49 y=3 Dividing by 5
 4  2
= y+ y− The solution is 3.
7 7
 2 7. 6x − 15 = 45
9 3 3
125. x2 + 3x + = x2 + 2 · x · + 6x = 60 Adding 15
4 2 2
 2 x = 10 Dividing by 6
3
= x+
2 The solution is 10.
 2
1 1 1 9. 5x − 10 = 45
127. x −x+ = x −2·x· +
2 2
4 2 2 5x = 55 Adding 10
 2
1 x = 11 Dividing by 5
= x−
2 The solution is 11.

Copyright © 2013 Pearson Education, Inc.


10 Chapter R: Basic Concepts of Algebra

11. 9t + 4 = −5 27. 24 = 5(2t + 5)


9t = −9 Subtracting 4 24 = 10t + 25
t = −1 Dividing by 9 −1 = 10t Subtracting 25
The solution is −1. −
1
=t Dividing by 10
10
13. 8x + 48 = 3x − 12 1
The solution is − .
5x + 48 = −12 Subtracting 3x 10
5x = −60 Subtracting 48 29. 5y − 4(2y − 10) = 25
x = −12 Dividing by 5 5y − 8y + 40 = 25
The solution is −12. −3y + 40 = 25 Collecting like terms
−3y = −15 Subtracting 40
15. 7y − 1 = 23 − 5y
y=5 Dividing by −3
12y − 1 = 23 Adding 5y
The solution is 5.
12y = 24 Adding 1
y=2 Dividing by 12 31. 7(3x + 6) = 11 − (x + 2)
The solution is 2. 21x + 42 = 11 − x − 2
21x + 42 = 9 − x Collecting like terms
17. 3x − 4 = 5 + 12x
22x + 42 = 9 Adding x
−9x − 4 = 5 Subtracting 12x
22x = −33 Subtracting 42
−9x = 9 Adding 4
3
x = −1 Dividing by −9 x=− Dividing by 22
2
The solution is −1. 3
The solution is − .
2
19. 5 − 4a = a − 13
5 − 5a = −13 Subtracting a 33. 4(3y − 1) − 6 = 5(y + 2)

−5a = −18 Subtracting 5 12y − 4 − 6 = 5y + 10


18 12y − 10 = 5y + 10 Collecting like terms
a= Dividing by −5
5 7y − 10 = 10 Subtracting 5y
18 7y = 20 Adding 10
The solution is .
5 20
y= Dividing by 7
21. 3m − 7 = −13 + m 7
2m − 7 = −13 Subtracting m 20
The solution is .
7
2m = −6 Adding 7
m = −3 Dividing by 2
35. x2 + 3x − 28 = 0

The solution is −3. (x + 7)(x − 4) = 0 Factoring


x+7 = 0 or x − 4 = 0 Principle of zero products
23. 11 − 3x = 5x + 3
x = −7 or x=4
11 − 8x = 3 Subtracting 5x
The solutions are −7 and 4.
−8x = −8 Subtracting 11
x=1 37. x2 + 5x = 0
The solution is 1. x(x + 5) = 0 Factoring
x = 0 or x + 5 = 0 Principle of zero products
25. 2(x + 7) = 5x + 14
x = 0 or x = −5
2x + 14 = 5x + 14
The solutions are 0 and −5.
−3x + 14 = 14 Subtracting 5x
−3x = 0 Subtracting 14 39. y 2 + 6y + 9 = 0
x=0 (y + 3)(y + 3) = 0
The solution is 0. y+3 = 0 or y + 3 = 0
y = −3 or y = −3
The solution is −3.

Copyright © 2013 Pearson Education, Inc.


Exercise Set R.5 11

41. x2 + 100 = 20x 53. x2 − 36 = 0


x2 − 20x + 100 = 0 Subtracting 20x (x + 6)(x − 6) = 0
(x − 10)(x − 10) = 0 x+6 = 0 or x − 6 = 0
x − 10 = 0 or x − 10 = 0 x = −6 or x=6
x = 10 or x = 10 The solutions are −6 and 6.
The solution is 10.
55. z 2 = 144
43. x − 4x − 32 = 0
2
z − 144 = 0
2

(x − 8)(x + 4) = 0 (z + 12)(z − 12) = 0


x − 8 = 0 or x + 4 = 0 z + 12 = 0 or z − 12 = 0
x = 8 or x = −4 z = −12 or z = 12
The solutions are 8 and −4. The solutions are −12 and 12.

45. 3y 2 + 8y + 4 = 0 57. 2x2 − 20 = 0


(3y + 2)(y + 2) = 0 2x2 = 20
3y + 2 = 0 or y + 2 = 0 x2 = 10
√ √
3y = −2 or y = −2 x= 10 or x = − 10 Principle of square roots
√ √ √
2 The solutions are 10 and − 10, or ± 10.
y = − or y = −2
3
2 59. 6z 2 − 18 = 0
The solutions are − and −2.
3 6z 2 = 18
2
47. 12z + z = 6 z2 = 3
√ √
12z 2 + z − 6 = 0 z = 3 or z = − 3
√ √ √
(4z + 3)(3z − 2) = 0 The solutions are 3 and − 3, or ± 3.
4z + 3 = 0 or 3z − 2 = 0 1
61. A= bh
4z = −3 or 3z = 2 2
3 2 2A = bh Multiplying by 2 on both sides
z = − or z=
4 3 2A
3 2 =b Dividing by h on both sides
The solutions are − and . h
4 3
63. P = 2l + 2w
49. 12a2 − 28 = 5a P − 2l = 2w Subtracting 2l on both sides
12a2 − 5a − 28 = 0
(3a + 4)(4a − 7) = 0 P − 2l
=w Dividing by 2 on both sides
3a + 4 = 0 or 4a − 7 = 0 2
3a = −4 or 4a = 7
1
4 7 65. A= h(b1 + b2 )
a = − or a= 2
3 4
2A 2
4 7 = b 1 + b2 Multiplying by on both sides
The solutions are − and . h h
3 4
2A
− b1 = b2 , or
51. 14 = x(x − 5) h
14 = x2 − 5x 2A − b1 h
= b2
0 = x2 − 5x − 14 h
0 = (x − 7)(x + 2) 67.
4 3
V = πr
x − 7 = 0 or x + 2 = 0 3
3V = 4πr3 Multiplying by 3 on both sides
x = 7 or x = −2
3V
The solutions are 7 and −2. =π Dividing by 4r3 on both sides
4r3

Copyright © 2013 Pearson Education, Inc.


12 Chapter R: Basic Concepts of Algebra

9
69. F = C + 32 83. x − {3x − [2x − (5x − (7x − 1))]} = x + 7
5
9 x − {3x − [2x − (5x − 7x + 1)]} = x + 7
F − 32 = C Subtracting 32 on both sides x − {3x − [2x − (−2x + 1)]} = x + 7
5
5 5 x − {3x − [2x + 2x − 1]} = x + 7
(F − 32) = C Multiplying by on both sides
9 9 x − {3x − [4x − 1]} = x + 7
x − {3x − 4x + 1} = x + 7
71. Ax + By = C x − {−x + 1} = x + 7
Ax = C − By Subtracting By on both sides x+x−1 = x+7
C − By
A= Dividing by x on both sides 2x − 1 = x + 7
x
x−1 = 7
73. 2w + 2h + l = p
x=8
2h = p − 2w − l Subtracting 2w and l The solution is 8.

p − 2w − l 85. (5x2 + 6x)(12x2 − 5x − 2) = 0


h= Dividing by 2
2 x(5x + 6)(4x + 1)(3x − 2) = 0
75. 2x − 3y = 6 x = 0 or 5x+6 = 0 or 4x+1 = 0 or 3x−2 = 0
−3y = 6 − 2x Subtracting 2x x = 0 or 5x = −6 or 4x = −1 or 3x = 2
6 − 2x 6 1 2
y= , or Dividing by −3 x = 0 or x = − or x = − or x=
−3 5 4 3
2x − 6 6 1 2
The solutions are 0, − , − , and .
3 5 4 3
87. 3x3 + 6x2 − 27x − 54 = 0
77. a = b + bcd
3(x3 + 2x2 − 9x − 18) = 0
a = b(1 + cd) Factoring
a 3[x2 (x + 2) − 9(x + 2)] = 0 Factoring by grouping
=b Dividing by 1 + cd 3(x + 2)(x − 9) = 0
2
1 + cd
3(x + 2)(x + 3)(x − 3) = 0
79. z = xy − xy 2

z = x(y − y 2 ) Factoring x+2 = 0 or x + 3 = 0 or x − 3 = 0


z x = −2 or x = −3 or x=3
=x Dividing by y − y 2
y − y2 The solutions are −2, −3, and 3.
81. 3[5 − 3(4 − t)] − 2 = 5[3(5t − 4) + 8] − 26
Exercise Set R.6
3[5 − 12 + 3t] − 2 = 5[15t − 12 + 8] − 26
3[−7 + 3t] − 2 = 5[15t − 4] − 26 5
1. Since − is defined for all real numbers, the domain is
−21 + 9t − 2 = 75t − 20 − 26 3
{x|x is a real number}.
9t − 23 = 75t − 46
−66t − 23 = −46 3x − 3
3.
x(x − 1)
−66t = −23
The denominator is 0 when the factor x = 0 and
23
t= also when x − 1 = 0, or x = 1. The domain is
66
{x|x is a real number and x = 0 and x = 1}.
23
The solution is .
66 x+5 x+5
5. =
x2 + 4x − 5 (x + 5)(x − 1)
We see that x + 5 = 0 when x = −5 and x − 1 = 0
when x = 1. Thus, the domain is
{x|x is a real number and x = −5 and x = 1}.

7. We first factor the denominator completely.


7x2 − 28x + 28 7x2 − 28x + 28
=
(x −4)(x +3x−10)
2 2 (x+2)(x−2)(x+5)(x−2)
We see that x + 2 = 0 when x = −2, x − 2 = 0 when
x = 2, and x + 5 = 0 when x = −5. Thus, the domain is
{x|x is a real number and x = −2 and x = 2 and x = −5}.

Copyright © 2013 Pearson Education, Inc.


Exercise Set R.6 13

x2 − 4 ✏
(x + 2)(x−2) x + 2 27. x2 − y 2 x2 + xy + y 2
·
9.
x − 4x + 4
2
= =

(x − 2)(x−2) x − 2 x3 − y 3 x2 + 2xy + y 2

x3 − 6x2 + 9x x(x2 − 6x + 9) (x + y)(x − y)(x2 + xy + y 2 )


11. =
x − 3x
3 2
=
x2 (x − 3) (x − y)(x2 + xy + y 2 )(x + y)(x + y)

/(x−3)(x − 3)
x =
1
·
(x + y)(x − y)(x2 + xy + y 2 )
=
/ · x(x−3)
x ✏ x+y (x + y)(x − y)(x2 + xy + y 2 )
x−3 1
= = ·1 Removing a factor of 1
x x+y
1
13. 6y 2 + 12y − 48 6(y 2 + 2y − 8) =
= x+y
3y − 9y + 6
2 3(y 2 − 3y + 2)
2·3 ✏
/ · (y + 4)(y−2) (x − y)2 − z 2 x−y+z
✏ ÷
= 29.
/(y − 1)(y−2)
3 (x + y) − z
2 2 x+y−z
2(y + 4) (x − y)2 − z 2 x + y − z
= = ·
y−1 (x + y)2 − z 2 x − y + z
4−x −1(x−4) ✏ (x − y + z)(x − y − z)(x + y − z)
15.
x2 + 4x − 32
=

(x−4)(x + 8)
=
(x + y + z)(x + y − z)(x − y + z)
−1 1 (x − y + z)(x + y − z) x − y − z
= , or − = ·
x+8 x+8 (x − y + z)(x + y − z) x + y + z
r − s r2 − s2 (r − s)(r2 − s2 ) x−y−z
17. · = = 1· Removing a factor of 1
r + s (r − s)2 (r + s)(r − s)2 x+y+z
✏ ✏ ✏
(r−s)(r−s)(r+s) · 1 =
x−y−z
=
✏ ✏ ✏
(r+s)(r−s)(r−s) x+y+z
=1 7 3 7+3
31. + =
x2 + 2x − 35 9x3 − 4x 5x 5x 5x
19. · 10
3x3 − 2x2

7x + 49 =
✏ 5x


(x+7)(x − 5)(x)
✧ (3x + 2)(3x−2)
/ ·2
=
/ · x(3x−2)(7)(x+7)
x ✏ =
5
/ ·x
5
(x − 5)(3x + 2) 2
= =
7x x
4x2 + 9x + 2 x2 − 1
21. · 2 33.
4 3a 4 + 3a
x + x − 2 3x + x − 2
2 +
3a + 4 3a + 4
=
3a + 4
✏ ✏ ✏
(4x + 1)(x+2)(x+1)(x−1) =1 (4 + 3a = 3a + 4)
=
✏ ✏ ✏
(x+2)(x−1)(3x − 2)(x+1)
5 3
=
4x + 1 35. − , LCD is 8z
3x − 2 4z 8z
5 2 3
m2 − n2 m−n = · −
23. ÷ 4z 2 8z
r+s r+s 10 3
= −
m2 − n2 r + s 8z 8z
= ·
r+s m−n 7
✏ ✏
=
(m + n)(m−n)(r+s) 8z
=
✏ ✏
(r+s)(m−n)
37.
3
+
2
= m+n x + 2 x2 − 4
3x + 12 (x + 4)2 3 2
25. ÷ = +
x + 2 (x + 2)(x − 2)
, LCD is (x+2)(x−2)
2x − 8 (x − 4)2
3x + 12 (x − 4)2 3 x−2 2
· = · +
=
2x − 8 (x + 4)2 x + 2 x − 2 (x + 2)(x − 2)
✏ ✏
3(x+4)(x−4)(x − 4) =
3x − 6
+
2
=
✏ ✏
2(x−4)(x+4)(x + 4)
(x + 2)(x − 2) (x + 2)(x − 2)
3x − 4
3(x − 4) =
= (x + 2)(x − 2)
2(x + 4)

Copyright © 2013 Pearson Education, Inc.


14 Chapter R: Basic Concepts of Algebra

y 2 9x + 2 7
39. − 47. +
y 2 − y − 20 y+4 3x2 − 2x − 8 3x2 + x − 4
y 2 9x + 2 7
= − , LCD is (y + 4)(y − 5) = + ,
(y + 4)(y − 5) y+4 (3x + 4)(x − 2) (3x + 4)(x − 1)
y 2 y−5 LCD is (3x + 4)(x − 2)(x − 1)
= − ·
(y + 4)(y − 5) y+4 y−5 9x + 2 x−1 7 x−2
= · + ·
y 2y − 10 (3x+4)(x−2) x − 1 (3x+4)(x−1) x − 2
= −
(y + 4)(y − 5) (y + 4)(y − 5) 9x2 − 7x − 2 7x − 14
= +
y − (2y − 10) (3x+4)(x−2)(x−1) (3x+4)(x−1)(x−2)
=
(y + 4)(y − 5) 9x2 − 16
=
y − 2y + 10 (3x + 4)(x − 2)(x − 1)
=
(y + 4)(y − 5) ✘

(3x+4)(3x − 4)
−y + 10 =
= (3x+4)(x − 2)(x − 1)
(y + 4)(y − 5) 3x − 4
=
3 x − 5y (x − 2)(x − 1)
41. +
x + y x2 − y 2 5a ab 4b
3 x − 5y 49. + +
= + , LCD is (x + y)(x − y) a − b a2 − b2 a+b
x + y (x + y)(x − y) 5a ab 4b
3 x−y x − 5y = + + ,
= · + a − b (a + b)(a − b) a + b
x + y x − y (x + y)(x − y) LCD is (a + b)(a − b)
3x − 3y x − 5y 5a a+b ab 4b a−b
= + · ·
(x + y)(x − y) (x + y)(x − y) = + +
a − b a + b (a + b)(a − b) a + b a − b
4x − 8y
= 5a2 + 5ab ab 4ab − 4b2
(x + y)(x − y) = + +
(a+b)(a−b) (a+b)(a−b) (a+b)(a−b)

43.
y 2 5a2 + 10ab − 4b2
+ =
y−1 1−y (a + b)(a − b)
y −1 2
= + · 7 x+8 3x − 2
y − 1 −1 1 − y 51. − +
x + 2 4 − x2 4 − 4x + x2
y −2
= + 7 x+8 3x − 2
y−1 y−1 = − + ,
x + 2 (2 + x)(2 − x) (2 − x)2
y−2
= LCD is (2 + x)(2 − x)2
y−1
7 (2 − x)2 x+8 2−x
x y = · − · +
45. − 2 + x (2 − x)2 (2 + x)(2 − x) 2−x
2x − 3y 3y − 2x
3x − 2 2+x
x −1 y ·
= − · (2 − x)2 2+x
2x − 3y −1 3y − 2x
28−28x+7x2 −(16−6x−x2 )+3x2 +4x−4
x −y =
= − (2 + x)(2 − x)2
2x − 3y 2x − 3y
x+y 28 − 28x + 7x2 − 16 + 6x + x2 + 3x2 + 4x − 4
= [x − (−y) = x + y] =
2x − 3y (2 + x)(2 − x)2
11x2 − 18x + 8 11x2 − 18x + 8
= , or
(2 + x)(2 − x) 2 (x + 2)(x − 2)2

Copyright © 2013 Pearson Education, Inc.


Exercise Set R.6 15
2
8 c 8
1 x x2 + 2 c+ c· 2 + 2
53. + + 2 59. c2 = c c
x+1 2−x x −x−2 2 c 2
1+ 1· +
1 x x2 + 2 c c c
= + + c3 + 8
x + 1 2 − x (x + 1)(x − 2)
2
1 −1 x x2 + 2 = c
= + · + c+2
x + 1 −1 2 − x (x + 1)(x − 2)
c
1 −x x2 + 2 3
c +8 c
= + + , = ·
x + 1 x − 2 (x + 1)(x − 2) c2 c+2
LCD is (x + 1)(x − 2)

(c+2)(c2 − 2c + 4)c/
=
1
·
x−2
+
−x x + 1
· +
x2 + 2 =

/c · c(c+2)
x + 1 x − 2 x − 2 x + 1 (x + 1)(x − 2)
c − 2c + 4
2
x−2 −x2 − x x2 + 2 =
= + + c
(x+1)(x−2) (x+1)(x−2) (x+1)(x−2)
x − 2 − x2 − x + x2 + 2 x2 + xy + y 2 x2 + xy + y 2
= 61. = 2
(x + 1)(x − 2) x2 y2 x x y2 y
0 − · − ·
= y x y x x y
(x + 1)(x − 2)
x2 + xy + y 2
=0 =
x3 − y 3
a−b xy
a−b ab xy
55. b = · = (x2 + xy + y 2 ) ·
a2 − b2 b a2 − b2 x3 − y 3
ab (x2 + xy + y 2 )(xy)
=
a−b ab (x − y)(x2 + xy + y 2 )
= ·
b (a + b)(a − b) x2 + xy + y 2 xy
= ·

a ✧(a−b)
b
x2 + xy + y 2 x − y
=

/b (a + b)(a−b) = 1·
xy
x−y
a
= xy
a+b =
x−y
x y x y
− − 1 a 1
y x y x xy a− a· −
57. = · , LCM is xy a − a−1
1 1 1 1 xy 63. = a = a a
+ + a + a−1 1 a 1
y x y x
x y  a+ a· +
a a a
− (xy)
y x a2 − 1
=  
1 1
+ (xy) = 2a
y x a +1
x2 − y 2 a
= a2 − 1 a
x+y = · 2

(x+y)(x − y) a a +1
=

(x+y) · 1 =
a2 − 1
= x−y a2 + 1

Copyright © 2013 Pearson Education, Inc.


16 Chapter R: Basic Concepts of Algebra
1 2 1 x+3 2 x−3
+ · + ·
65. x−3 x+3 = x−3 x+3 x+3 x−3 73. (x + h)3 − x3 x3 + 3x2 h + 3xh2 + h3 − x3
=
3 4 3 x+2 4 x−1 h h
− · − ·
x−1 x+2 x−1 x+2 x+2 x−1 3x2 h + 3xh2 + h3
=
x + 3 + 2(x − 3) h
(x − 3)(x + 3) h/(3x2 + 3xh + h2 )
= =
3(x + 2) − 4(x − 1) h/ · 1
(x − 1)(x + 2) = 3x2 + 3xh + h2
x + 3 + 2x − 6
 5  
(x − 3)(x + 3) x+1 (x + 1) + (x − 1) 5
+1
=
3x + 6 − 4x + 4 75. 
 x −1  
 =  (x + 1) x−1 
x+1 − (x − 1) 
(x − 1)(x + 2) −1
x−1 x−1
3x − 3
5
2x x−1
(x − 3)(x + 3) = ·
=
−x + 10 x−1 2


(x − 1)(x + 2) ✦ 5
/x(x−1)
2
3x − 3 (x − 1)(x + 2)
=
1·2 ✦
/(x−1)
= ·
(x − 3)(x + 3) −x + 10 = x5
(3x − 3)(x − 1)(x + 2)
= , or 77. n(n + 1)(n + 2) (n + 1)(n + 2)
(x − 3)(x + 3)(−x + 10) +
2·3 2
3(x − 1)2 (x + 2) n(n + 1)(n + 2) (n + 1)(n + 2) 3
(x − 3)(x + 3)(−x + 10) = + · ,
2·3 2 3
a 1+a a a 1+a 1−a LCD is 2 · 3
+ · + ·
67. 1−a a = 1−a a a 1−a
1−a a 1−a 1+a a a n(n + 1)(n + 2) + 3(n + 1)(n + 2)
· · =
a
+
1+a a
+
1+a 1+a a 2·3

a2 + (1 − a2 ) (n + 1)(n + 2)(n + 3)
= Factoring the num-
a(1 − a) 2·3
= erator by grouping
(1 − a2 ) + a2
a(1 + a) x2 − 9 5x2 − 15x + 45 x2 + x
79. · +
1 a/(1 + a) x3 + 27 x2 − 2x − 3 4 + 2x
= ·
a/(1 − a) 1 (x + 3)(x − 3)(5)(x2 − 3x + 9) x2 + x
= +
1+a (x + 3)(x2 − 3x + 9)(x − 3)(x + 1) 4 + 2x
=
1−a (x + 3)(x − 3)(x2 − 3x + 9) 5 x2 + x
= · +
1 2 1 1 2 1 (x + 3)(x − 3)(x − 3x + 9) x + 1 4 + 2x
2
+ + + + 2 2
69. a2 ab b2 = a2 ab b2 · a b , 5 x2 + x
1 1 1 1 a2 b2 = 1· +
− 2 − 2 x + 1 4 + 2x
a2 b a2 b
5 x2 + x
LCM is a2 b2 = +
x + 1 2(2 + x)
b2 + 2ab + a2
= 5 · 2(2 + x) + (x2 + x)(x + 1)
b2 − a2 =

(b+a)(b + a) 2(x + 1)(2 + x)
=

(b+a)(b − a) =
20 + 10x + x3 + 2x2 + x
b+a 2(x + 1)(2 + x)
=
b−a x3 + 2x2 + 11x + 20
=
2(x + 1)(2 + x)

71. (x + h)2 − x2 x2 + 2xh + h2 − x2


=
h h Exercise Set R.7
2xh + h2
= 
h 1. (−21)2 = | − 21| = 21
=
h
/(2x + h)  
/·1
h 3. 9y 2 = (3y)2 = |3y| = 3|y|
= 2x + h 
5. (a − 2)2 = |a − 2|

Copyright © 2013 Pearson Education, Inc.


Exercise Set R.7 17

√  √ √ √ √
7. 3
−27x3 = 3
(−3x)3 = −3x 45. 5 2 + 3 32 = 5 2 + 3 16 · 2
√ √
√  = 5 2+3·4 2
81x8 = 4 (3x2 )4 = |3x2 | = 3x2 √ √
4
9.
= 5 2 + 12 2

5
√5 √
11. 32 = 25 = 2 = (5 + 12) 2
√ √ √ √ √ √
13. 180 = 36 · 5 = 36 · 5 = 6 5 = 17 2
√ √ √ √ √ √
√ √ √ √ √ 47. 6 20 − 4 45 + 80 = 6 4 · 5 − 4 9 · 5 + 16 · 5
15. 72 = 36 · 2 = 36 · 2 = 6 2 √ √ √
√ √ √ √ √ = 6·2 5−4·3 5+4 5
17. 3
54 = 3 27 · 2 = 3 27 · 3 2 = 3 3 2 √ √ √
= 12 5 − 12 5 + 4 5
√ √ √ √ √
19. 128c2 d4 = 64c2 d4 · 2 = |8cd2 | 2 = 8 2 |c|d2 = (12 − 12 + 4) 5

  √ =4 5
48x6 y 4 = 4 16x4 y 4 · 3x2 = |2xy| 3x2 =
4
21. 4

√ √ √ √
4
2|x||y| 3x2 49. 8 2x2 − 6 20x − 5 8x2
√ √ √
√  = 8x 2 − 6 4 · 5x − 5 4x2 · 2
23. x2 − 4x + 4 = (x − 2)2 = |x − 2| √ √ √
= 8x 2 − 6 · 2 5x − 5 · 2x 2
√ √ √ √ √ √ √ √
25. 15 35 = 15 · 35 = 3 · 5 · 5 · 7 = 52 · 3 · 7 = = 8x 2 − 12 5x − 10x 2
√ √ √ √ √
52 · 3 · 7 = 5 21 = −2x 2 − 12 5x
√ √ √ √ √ √ √ √ √ 
27. 8 10 = 8 · 10 = 2 · 4 · 2 · 5 = 22 · 4 · 5 = 51. 8+2 5 8−2 5
√ √ √ 2  √ 2
2·2 5=4 5 = 8 − 2 5
 √   √ = 8−4·5
29. 2x3 y 12xy = 24x4 y 2 = 4x4 y 2 · 6 = 2x2 y 6
= 8 − 20
 √   √
31. 3
3x2 y 3 36x = 3 108x3 y = 3 27x3 · 4y = 3x 3 4y = −12
   √ √ √ √
33. 3
2(x + 4) 3 4(x + 4)4 = 3 8(x + 4)5 53. (2 3 + 5)( 3 − 3 5)
 √ √ √ √ √ √ √ √
= 3 8(x + 4)3 · (x + 4)2 = 2 3· 3−2 3·3 5+ 5· 3− 5·3 5
 √ √
= 2(x + 4) 3 (x + 4)2 = 2 · 3 − 6 15 + 15 − 3 · 5
√ √
 = 6 − 6 15 + 15 − 15
 8 √
8 m16 n24 8 m2 n 3 m 2 n3 = −9 − 5 15
35. = =
28 2 2 √ √ √
55. ( 2 − 5)2 = ( 2)2 − 2 · 2 · 5 + 52
√  √
40xy 40xy  = 2 − 10 2 + 25
37. √ = = 5y √
8x 8x = 27 − 10 2
√   √ √ √ √ √ √
3
3x2 3x2 1 1 57. ( 5 − 6)2 = ( 5)2 − 2 5 · 6 + ( 6)2
√ √
3 3
39. = = =
24x5 8x3 = 5 − 2 30 + 6
3
24x5 2x
  √
3 64a
4
3 64 · a · a
3 = 11 − 2 30
41. =
27b 3 27 · b3 59. We use the Pythagorean theorem. We have a = 47 and
√3 √
64a3 3 a b = 25.
= √ 3
27b3 c2 = a2 + b2

4a a 3
c2 = 472 + 252
=
 
3b c2 = 2209 + 625
7x 3 7 · x2 · x c2 = 2834
43. =
36y 6 36 · y 6 c ≈ 53.2
√ √
x2 7x The distance across the pond is about 53.2 yd.
= 
36y 6

x 7x
=
6y 3

Copyright © 2013 Pearson Education, Inc.


18 Chapter R: Basic Concepts of Algebra
√ √ √ √
 a 2 1− 2 1− 2 2 3+ 6
61. a) h2 + = a2 Pythagorean theorem 73. √ √ = √ √ · √ √
2 2 3− 6 2 3− 6 2 3+ 6
√ √ √ √
a2 2 3 + 6 − 2 6 − 12
h2 + = a2 =
4 4·3−6
√ √ √ √
3a2 2 3+ 6−2 6−2 3
h2 = =
4 12 − 6
 √ √
3a2 − 6 6
h= = , or −
4 6 6
a√ √ √
h= 3 6 6 m+ n
2 75. √ √ = √ √ ·√ √
m− n m− n m+ n
b) Using the result of part (a) we have √ √
6( m + n)
1 = √ 2 √
A= · base · height ( m) − ( n)2
2 √ √
1 a√ a a  6 m+6 n
=
A= a· 3 + =a m−n
2 2 2 2
2√
√ √ √ √
a 50 50 2 100 10
A= 3 77. = ·√ = √ = √
4 3 3 2 3 2 3 2
63.    √
x 3 2 3 2 4 8 3
8 2
· = = √ = √
3
79. =
❅ 5 5 4 20 3
20 3
20
❅ √
❅8 2 √ √ √ √
x ❅ x 11 11 11 121 11
81. √ = √ ·√ = √ =√
❅ 3 3 11 33 33
❅ √ √ √
❅ 9− 5 9− 5 9+ 5
x 83. √ = √ · √
3− 3 3− 3 9+ 5
√ √
x2 + x2 = (8 2)2 Pythagorean theorem 92 − ( 5)2
= √ √ √
2
2x = 128 27 + 3 5 − 9 3 − 15
x2 = 64 81 − 5
= √ √ √
x=8 27 + 3 5 − 9 3 − 15
76
   √ √ = √ √ √
3 3 7 21 21 21 27 + 3 5 − 9 3 − 15
65. = · = =√ =
7 7 7 49 49 7 √ √ √ √ √ √
a+ b a+ b a− b
√ √ √ √ √ 85. = ·√ √
3
7 3
7 3
5 3
35 3
35 3a 3a a− b
67. √ = √ · √ = √ = √ 2 √ 2
( a) − ( b)
3 3 3 3
25 25 5 125 5
   √ = √ √
3 3a( a − b)
3 16 3 16 3 3 48 48
69. = · = = √ = a−b

3
9 9 3 27 27 = √
√ √ 3a a − 3a b
3
8·6 236
=
3 3 
√ 87. y 5/6 = 6
y5
2 2 3+1
71. √ = √ ·√ √ 3
3−1 3−1 3+1 89. 163/4 = (161/4 )3 = 4
16 = 23 = 8

2( 3 + 1)
= 1 1 1
3−1 91. 125−1/3 = = √ =
√ 1251/3 3
125 5
2( 3 + 1)
= √
4 √ 
2 5/4 −3/4 a5/4 a5 a4a a
√ 93. a b = 3/4 = 4 √ √
= 4 , or a 4 3
2/( 3 + 1) b b3 b3 b
=
2/ · 1 √3

3

3

3
√ 95. m5/3 n7/3 = m5 n7 = m5 n7 = mn2 m2 n
= 3+1 √5
97. 173 = 173/5
√ 4
99. 5
12 = 124/5

Copyright © 2013 Pearson Education, Inc.


Chapter R Review Exercises 19

 √ √ 1/3  √a
101.
3
11 = 11 = (111/2 )1/3 = 111/6  √  √ √ a
a/2
127. a a = a = aa/2
√ √
103. 5 3 5 = 51/2 · 51/3 = 51/2+1/3 = 55/6
√5
105. 322 = 322/5 = (321/5 )2 = 22 = 4
Chapter R Review Exercises
3/2 1/2 3/2+1/2 2
107. (2a )(4a ) = 8a = 8a
 x6 1/2  x6 1/2 x3 x3 b2 1. True
109. = 2 −4 = −2 , or
9b−4 3 b 3b 3
3. True
x2/3 y 5/6 √
111. = x2/3−(−1/3) y 5/6−1/2 = xy 1/3 = x 3 y 4 √ 3 12
x−1/3 y 1/2 5. Rational numbers: −7, 43, − , 0, 3 64, 4 , , 102
9 4 7
113. (m1/2 n5/2 )2/3 = m 2 · 3 n 2 · 3 = m1/3 n5/3 =
1 2 5 2 √
√ √ √ 7. Integers: −7, 43, 0, 3 64, 102
√ 3 3 3
3
m n5 = mn5 = n mn2 √
9. Natural numbers: 43, 3 64, 102
115. a3/4 (a2/3 + a4/3 ) = a3/4+2/3 + a3/4+4/3 =

12

12 11. (−4, 7]
a17/12 + a25/12 = a17 + a25 =
√ √  
12
a a5 + a2 12 a 7 7  7  7
13. The distance of − from 0 is , so  − = .
√ √ 8 8 8 8
117. 3
6 2 = 61/3 21/2 = 62/6 23/6
15. 3 · 2 − 4 · 22 + 6(3 − 1)
= (62 23 )1/6
√ = 3 · 2 − 4 · 22 + 6 · 2 Working inside
= 6 36 · 8 parentheses

= 6 288 = 3·2−4·4+6·2 Evaluating 22
√  = 6 − 16 + 12 Multiplying
119. 4 xy 3
x2 y = (xy)1/4 (x2 y)1/3 = (xy)3/12 (x2 y)4/12
 1/12 = −10 + 12 Adding in order
= (xy)3 (x2 y)4
 1/12 =2 from left to right
= x3 y 3 x8 y 4
 17. Convert 8.3 × 10−5 to decimal notation.
= 12 x11 y 7
 The exponent is negative, so the number is between 0 and
√  √ 1/3
121. 3
a4 a3 = a4 a3 = (a4 a3/2 )1/3 1. We move the decimal point 5 places to the left.
= (a11/2 )1/3 8.3 × 10−5 = 0.000083
= a11/6 19. Convert 405,000 to scientific notation.
√6
= a11 We want the decimal point to be positioned between the
√6
= a a5 4 and the first 0, so we move it 5 places to the left. Since
405,000 is greater than 10, the exponent must be positive.
 
123. (a + x)3 3 (a + x)2 (a + x)3/2 (a + x)2/3 405, 000 = 4.05 × 105
√ =
4
a+x (a + x)1/4
21. (3.1 × 105 )(4.5 × 10−3 )
(a + x)26/12
=
(a + x)3/12 = (3.1 × 4.5) × (105 × 10−3 )

= (a + x)23/12 = 13.95 × 102 This is not scientific notation.


 = (1.395 × 10) × 102
= 12 (a + x)23
 = 1.395 × 103 Writing scientific notation
= (a + x) 12 (a + x)11
 1 23. (−3x4 y −5 )(4x−2 y) = −3(4)x4+(−2) y −5+1 =
125. 1 + x2 + √
1 + x2 −12x2 12x2
√ −12x2 y −4 , or , or − 4
 1 + x2 1 1 + x2 y 4 y
= 1+x · 2 +√ ·√
1 + x2 1+x 2 1 + x2 √
√ √ √
4 4
2
(1 + x ) 1 + x 2 1+x 2 25. 81 = 34 = 3
= +
1 + x2 1 + x2

(2 + x2 ) 1 + x2
=
1 + x2

Copyright © 2013 Pearson Education, Inc.


20 Chapter R: Basic Concepts of Algebra
1
b − a−1 b−
27. = a 49. 6(2x − 1) = 3 − (x + 10)
a − b−1 1
a− 12x − 6 = 3 − x − 10
b
a 1 12x − 6 = −x − 7
b· −
a a
=
b 1 13x − 6 = −7
a· − 13x = −1
b b
ab 1 1
− x=−
13
= a a
ab 1 1
− The solution is − .
b b 13
ab − 1 51. x2 − x = 20
a
=
ab − 1 x − x − 20 = 0
2

b (x − 5)(x + 4) = 0
ab − 1 b x − 5 = 0 or x + 4 = 0
= ·
a ab − 1 x = −4

(ab−1)b
x = 5 or
=

a(ab−1)
The solutions are 5 and −4.

b 53. x(x − 2) = 3
=
a x2 − 2x = 3
√ √ √ √ √ √ x2 − 2x − 3 = 0
29. ( 3 − 7)( 3 + 7) = ( 3)2 − ( 7)2
(x + 1)(x − 3) = 0
= 3−7
x+1 = 0 or x − 3 = 0
= −4
√ x = −1 or x=3
√ 25 √ 25 5
31. 8 5+ √ = 8 5+ √ · √ The solutions are −1 and 3.
5 5 5

√ 25 5 55. n2 − 7 = 0
= 8 5+
5 n2 = 7
√ √ √ √
= 8 5+5 5 n = 7 or n = − 7
√ √ √ √
= 13 5 The solutions are 7 and − 7, or ± 7.
33. (5a + 4b)(2a − 3b)
57. xy = x + 3
= 10a2 − 15ab + 8ab − 12b2
xy − x = 3
= 10a2 − 7ab − 12b2
x(y − 1) = 3
35. 32x4 − 40xy 3 = 8x · 4x3 − 8x · 5y 3 = 8x(4x3 − 5y 3 ) x=
3
y−1
37. 24x + 144 + x2
x 4
= x2 + 24x + 144 59. −
x2 + 9x + 20 x2 + 7x + 12
= (x + 12)2 x 4
= −
39. 9x2 − 30x + 25 = (3x − 5)2 (x + 5)(x + 4) (x + 4)(x + 3)
LCD is (x + 5)(x + 4)(x + 3)
41. 18x2 − 3x + 6 = 3(6x2 − x + 2)
x x+3 4 x+5
= · − ·
43. 6x3 + 48 (x + 5)(x + 4) x + 3 (x + 4)(x + 3) x + 5
= 6(x3 + 8) x(x + 3) − 4(x + 5)
=
= 6(x + 2)(x2 − 2x + 4) (x + 5)(x + 4)(x + 3)
x2 + 3x − 4x − 20
=
45. 2x2 + 5x − 3 = (2x − 1)(x + 3) (x + 5)(x + 4)(x + 3)
x2 − x − 20
47. 5x − 7 = 3x − 9 =
(x + 5)(x + 4)(x + 3)
2x − 7 = −9 ✏
(x − 5)(x+4)
2x = −2 =

(x + 5)(x+4)(x + 3)
x = −1 x−5
=
The solution is −1. (x + 5)(x + 3)

Copyright © 2013 Pearson Education, Inc.


Chapter R Test 21

 √
(a + b)3 3 a + b
61. 6
(a + b)7 Chapter R Test
(a + b)3/2 (a + b)1/3
=
(a + b)7/6 √
3
= (a + b)3/2+1/3−7/6 1. a) Whole numbers: 0, 8, 29

= (a + b)9/6+2/6−7/6 b) Irrational numbers: 12
c) Integers but not natural numbers: 0, −5
= (a + b)2/3
 6 13
= 3 (a + b)2 d) Rational numbers but not integers: 6 , − , −1.2
7 4
  1/8
8 m32 n16 m32 n16 m4 n2 2. | − 17.6| = 17.6
63. = =  
38 38 3  15  15
3.   =
11 11
65. a = 8 and b = 17. Find c.
c2 = a2 + b2 4. |0| = 0
c2 = 82 + 172 5. (−3, 6]
c2 = 64 + 289
c2 = 353
c ≈ 18.8
6. | − 9 − 6| = | − 15| = 15, or
The guy wire is about 18.8 ft long.
|6 − (−9)| = |6 + 9| = |15| = 15
67. 9x2 − 36y 2 = 9(x2 − 4y 2 ) 7. 32 ÷ 23 − 12 ÷ 4 · 3
= 9[x2 − (2y)2 ] = 32 ÷ 8 − 12 ÷ 4 · 3
= 9(x + 2y)(x − 2y) = 4 − 12 ÷ 4 · 3
Answer C is correct. = 4−3·3
69. Substitute $124, 000 − $20, 000, or $104,000 for P , 0.0575 = 4−9
for r, and 12 · 30, or 360, for n and perform the resulting = −5
computation.
  n  8. Position the decimal point 6 places to the left, between the
r r
 12 1 +  4 and the 5. Since 4,509,000 is a number greater than 10,
M =P  12
n

 the exponent must be positive.
r
1+ −1 4, 509, 000 = 4.509 × 106
12
  360 
0.0575 0.0575 9. The exponent is negative, so the number is between 0 and
 12 1 + 
12
= $104, 000 
  360 

1. We move the decimal point 5 places to the left.
0.0575 8.6 × 10−5 = 0.000086
1+ −1
12
≈ $606.92 2.7 × 104
10. = 0.75 × 107
3.6 × 10−3
71. (an −bn )3 = (an −bn )(an −bn )2
= (7.5 × 10−1 ) × 107
= (an − bn )(a2n − 2an bn + b2n )
= 7.5 × 106
= a3n−2a2n bn +an b2n−a2n bn +2an b2n−b3n
= a3n − 3a2n bn + 3an b2n − b3n 1
11. x−8 · x5 = x−8+5 = x−3 , or
x3
73. m6n − m3n = m3n (m3n − 1)
12. (2y 2 )3 (3y 4 )2 = 23 y 6 · 32 y 8 = 8 · 9 · y 6+8 = 72y 14
= m3n [(mn )3 − 13 ]
= m3n (mn − 1)(m2n + mn + 1) 13. (−3a5 b−4 )(5a−1 b3 )
= −3 · 5 · a5+(−1) · b−4+3
15a4
= −15a4 b−1 , or −
b
14. (5xy 4 − 7xy 2 + 4x2 −3)−(−3xy 4 + 2xy 2 −2y+ 4)
= (5xy 4 −7xy 2 + 4x2 − 3)+(3xy 4 −2xy 2 + 2y − 4)
= (5 + 3)xy 4 + (−7 − 2)xy 2 + 4x2 + 2y + (−3 − 4)
= 8xy 4 − 9xy 2 + 4x2 + 2y − 7

Copyright © 2013 Pearson Education, Inc.


22 Chapter R: Basic Concepts of Algebra

15. (y − 2)(3y + 4) = 3y 2 + 4y − 6y − 8 = 3y 2 − 2y − 8 29. 3(y − 5) + 6 = 8 − (y + 2)


3y − 15 + 6 = 8 − y − 2
16. (4x − 3)2 = (4x)2 − 2 · 4x · 3 + 32 = 16x2 − 24x + 9
3y − 9 = −y + 6
x y x x y y
− · − · 4y − 9 = 6
17. y x y x x y
=
x+y x+y 4y = 15
x 2
y 2 15
− y=
xy xy 4
= 15
x+y The solution is .
4
x2 − y 2
xy 30. 2x2 + 5x + 3 = 0
=
x+y (2x + 3)(x + 1) = 0
x −y
2 2
1 2x + 3 = 0 or x + 1 = 0
= ·
xy x+y 2x = −3 or x = −1

(x+y)(x − y) 3
=
xy(x+y) ✏ x = − or
2
x = −1
x−y 3
= The solutions are − and −1.
xy 2
√ √ √ √ √ 31. z 2 − 11 = 0
18. 45 = 9 · 5 = 9 5 = 3 5
√ √ √ √ √ z 2 = 11
19. 3 56 = 3 8 · 7 = 3 8 3 7 = 2 3 7 √ √
z = 11 or z = − 11
√ √ √ √ √ √ √
20. 3 75 + 2 27 = 3 25 · 3 + 2 9 · 3 The solutions are 11 and − 11, or ± 11.
√ √
= 3·5 3+2·3 3 32. 8x + 3y = 24
√ √
= 15 3 + 6 3 3y = −8x + 24

= 21 3 −8x + 24 8
y= , or − x + 8
√√ √ √ 3 3
21. 18√ 10 = √ 18 · 10 = 2 · 3 · 3 · 2 · 5 =
2·3 5=6 5 x2 + x − 6 x2 − 25
33. · 2
√ √ x + 8x + 15 x − 4x + 4
2
22. (2 + 3)(5 − 2 3) (x2 + x − 6)(x2 − 25)
√ √ = 2
= 2·5−4 3+5 3−2·3 (x + 8x + 15)(x2 − 4x + 4)
√ √
= 10 − 4 3 + 5 3 − 6 ✏ ✏ ✏
(x+3)(x−2)(x+5)(x − 5)

= 4+ 3

=
✏ ✏ ✏
(x+3)(x+5)(x−2)(x − 2)
x−5
=
23. 8x2 − 18 = 2(4x2 − 9) = 2(2x + 3)(2x − 3) x−2
x 3
24. y 2 − 3y − 18 = (y + 3)(y − 6) 34. −
x2 − 1 x2 + 4x − 5
25. 2n2 + 5n − 12 = (2n − 3)(n + 4) =
x

3
(x + 1)(x − 1) (x − 1)(x + 5)
26. x3 + 10x2 + 25x = x(x2 + 10x + 25) = x(x + 5)2 LCD is (x + 1)(x − 1)(x + 5)
27. m3 − 8 = (m − 2)(m2 + 2m + 4) x x+ 5 3 x+ 1
= · − ·
(x+ 1)(x−1) x + 5 (x−1)(x + 5) x+ 1
28. 7x − 4 = 24 x(x + 5) − 3(x + 1)
=
7x = 28 (x + 1)(x − 1)(x + 5)
x=4 x2 + 5x − 3x − 3
=
The solution is 4. (x + 1)(x − 1)(x + 5)
x2 + 2x − 3
=
(x + 1)(x − 1)(x + 5)

(x + 3)(x−1)
=

(x + 1)(x−1)(x + 5)
x+3
=
(x + 1)(x + 5)

Copyright © 2013 Pearson Education, Inc.


Chapter R Test 23

√ √
5 5 7+ 3 35 + 5 3
35. √ = √ · √ = =
7− 3 7− 3 7+ 3 49 − 3

35 + 5 3
46
√8
3/8
36. m = m3

6
37. 35 = 35/6

38. a = 5 and b = 12. Find c.


c2 = a2 + b2
c2 = 52 + 122
c2 = 25 + 144
c2 = 169
c = 13
The guy wire is 13 ft long.

39. (x − y − 1)2
= [(x − y) − 1]2
= (x − y)2 − 2(x − y)(1) + 12
= x2 − 2xy + y 2 − 2x + 2y + 1

Copyright © 2013 Pearson Education, Inc.


Copyright © 2013 Pearson Education, Inc.
Chapter 1
Graphs, Functions, and Models
To graph (2, −1) we move from the origin 2 units to the
Exercise Set 1.1 right of the y-axis. Then we move 1 unit down from the
x-axis.
1. Point A is located 5 units to the left of the y-axis and To graph (0, 1) we do not move to the right or the left of
4 units up from the x-axis, so its coordinates are (−5, 4). the y-axis since the first coordinate is 0. From the origin
we move 1 unit up.
Point B is located 2 units to the right of the y-axis and
2 units down from the x-axis, so its coordinates are (2, −2). y
Point C is located 0 units to the right or left of the y-axis
and 5 units down from the x-axis, so its coordinates are 4
(0, −5). (2, 3)
2
Point D is located 3 units to the right of the y-axis and (5, 1) (0, 1) (5, 1)
4 2 4 x
5 units up from the x-axis, so its coordinates are (3, 5).
2 (2, 1)
Point E is located 5 units to the left of the y-axis and
4
4 units down from the x-axis, so its coordinates are
(−5, −4).
Point F is located 3 units to the right of the y-axis and 7. The first coordinate represents the year and the cor-
0 units up or down from the x-axis, so its coordinates are responding second coordinate represents the percentage
(3, 0). of the U.S. population that is foreign-born. The or-
dered pairs are (1970, 4.7%), (1980, 6.2%), (1990, 8.0%),
3. To graph (4, 0) we move from the origin 4 units to the right (2000, 10.4%), (2010, 12.8%).
of the y-axis. Since the second coordinate is 0, we do not
move up or down from the x-axis. 9. To determine whether (−1, −9) is a solution, substitute
To graph (−3, −5) we move from the origin 3 units to the −1 for x and −9 for y.
left of the y-axis. Then we move 5 units down from the y = 7x − 2
x-axis.
−9 ? 7(−1) − 2
To graph (−1, 4) we move from the origin 1 unit to the left 
 −7 − 2
of the y-axis. Then we move 4 units up from the x-axis. 
−9  −9 TRUE
To graph (0, 2) we do not move to the right or the left of
the y-axis since the first coordinate is 0. From the origin The equation −9 = −9 is true, so (−1, −9) is a solution.
we move 2 units up. To determine whether (0, 2) is a solution, substitute 0 for
To graph (2, −2) we move from the origin 2 units to the x and 2 for y.
right of the y-axis. Then we move 2 units down from the y = 7x − 2
x-axis.
2 ? 7 · 0 − 2

y  0−2

2  −2 FALSE
(1, 4) 4
The equation 2 = −2 is false, so (0, 2) is not a solution.
2 (0, 2)
(4, 0) 2 3 2
4 2 2 4 x 11. To determine whether , is a solution, substitute
3 4 3
2 (2, 2) 3
for x and for y.
(3, 5) 4 4
6x − 4y = 1
5. To graph (−5, 1) we move from the origin 5 units to the 2 3
6· −4· ? 1
left of the y-axis. Then we move 1 unit up from the x-axis. 3 4 

To graph (5, 1) we move from the origin 5 units to the right 4−3 

of the y-axis. Then we move 1 unit up from the x-axis. 1  1 TRUE
To graph (2, 3) we move from the origin 2 units to the right 2 3
of the y-axis. Then we move 3 units up from the x-axis. The equation 1 = 1 is true, so , is a solution.
3 4

Copyright © 2013 Pearson Education, Inc.


26 Chapter 1: Graphs, Functions, and Models

 3
To determine whether 1, is a solution, substitute 1 for 5x − 3 · 0 = −15
2 5x = −15
3
x and for y.
2 x = −3
6x − 4y = 1 The x-intercept is (−3, 0).
3 To find the y-intercept we replace x with 0 and solve for
6·1−4· ? 1 y.
2 
 5 · 0 − 3y = −15
6−6 
 −3y = −15
0  1 FALSE
 3 y=5
The equation 0 = 1 is false, so 1, is not a solution. The y-intercept is (0, 5).
2
 1 4 We plot the intercepts and draw the line that contains
13. To determine whether − , − is a solution, substitute them. We could find a third point as a check that the
2 5
1 4 intercepts were found correctly.
− for a and − for b.
2 5
y
2a + 5b = 3 (0, 5)
 1  4 4
2 − +5 − ? 3 5x  3y  15
2 5  2
(3, 0)

−1 − 4  4 2 2 4
 x
−5  3 FALSE 2
 1 4 4
The equation −5 = 3 is false, so − , − is not a solu-
2 5
tion.
 3 19. Graph 2x + y = 4.
To determine whether 0, is a solution, substitute 0 for
5 To find the x-intercept we replace y with 0 and solve for
3 x.
a and for b.
5 2x + 0 = 4
2a + 5b = 3 2x = 4
3 x=2
2·0+5· ? 3
5  The x-intercept is (2, 0).

0+3  To find the y-intercept we replace x with 0 and solve for

3  3 TRUE y.
 3 2·0+y = 4
The equation 3 = 3 is true, so 0, is a solution.
5 y=4
15. To determine whether (−0.75, 2.75) is a solution, substi- The y-intercept is (0, 4).
tute −0.75 for x and 2.75 for y. We plot the intercepts and draw the line that contains
x2 − y 2 = 3 them. We could find a third point as a check that the
intercepts were found correctly.
(−0.75)2 − (2.75)2 ? 3

0.5625 − 7.5625  y

−7  3 FALSE 2x  y  4 4 (0, 4)
The equation −7 = 3 is false, so (−0.75, 2.75) is not a 2
solution. (2, 0)
4 2 2 4
To determine whether (2, −1) is a solution, substitute 2 x
2
for x and −1 for y.
4
x2 − y 2 = 3
22 − (−1)2 ? 3
 21. Graph 4y − 3x = 12.
4−1 
 To find the x-intercept we replace y with 0 and solve for
3  3 TRUE x.
The equation 3 = 3 is true, so (2, −1) is a solution. 4 · 0 − 3x = 12
17. Graph 5x − 3y = −15. −3x = 12
To find the x-intercept we replace y with 0 and solve for x = −4
x. The x-intercept is (−4, 0).

Copyright © 2013 Pearson Education, Inc.


Exercise Set 1.1 27

To find the y-intercept we replace x with 0 and solve for y


y. 4 xy3
4y − 3 · 0 = 12 2
4y = 12
4 2 2 4 x
y=3 2
The y-intercept is (0, 3). 4

We plot the intercepts and draw the line that contains


them. We could find a third point as a check that the 3
intercepts were found correctly. 27. Graph y = − x + 3.
4
By choosing multiples of 4 for x, we can avoid fraction
y
values for y. Make a table of values, plot the points in the
table, and draw the graph.
4y  3x  12 4
2 (0, 3)
(4, 0) x y (x, y)
4 2 2 4 x −4 6 (−4, 6)
2

4 0 3 (0, 3)

4 0 (4, 0)
23. Graph y = 3x + 5.
We choose some values for x and find the corresponding y
y-values.
4
When x = −3, y = 3x + 5 = 3(−3) + 5 = −9 + 5 = −4. 2
When x = −1, y = 3x + 5 = 3(−1) + 5 = −3 + 5 = 2.
4 2 2 4 x
When x = 0, y = 3x + 5 = 3 · 0 + 5 = 0 + 5 = 5 2
3
We list these points in a table, plot them, and draw the 4
y  x
4 3
graph.

x y (x, y) 29. Graph 5x − 2y = 8.


−3 −4 (−3, −4) We could solve for y first.
5x − 2y = 8
−1 2 (−1, 2)
−2y = −5x + 8 Subtracting 5x on both sides
0 5 (0, 5) 5 1
y = x−4 Multiplying by − on both
2 2
y sides
By choosing multiples of 2 for x we can avoid fraction
6
values for y. Make a table of values, plot the points in the
y  3x  5
table, and draw the graph.
2
x y (x, y)
4 2 4 x
2 0 −4 (0, −4)

2 1 (2, 1)
25. Graph x − y = 3.
4 6 (4, 6)
Make a table of values, plot the points in the table, and
draw the graph.
y
x y (x, y)
4

−2 −5 (−2, −5) 2

0 −3 (0, −3) 4 2 2 4 x
2
3 0 (3, 0) 4
5x  2y  8

Copyright © 2013 Pearson Education, Inc.


28 Chapter 1: Graphs, Functions, and Models

31. Graph x − 4y = 5. y

Make a table of values, plot the points in the table, and 4 2 2 4 x


draw the graph. 2

x y (x, y) 4 y  x 2
6
−3 −2 (−3, −2)
8
1 −1 (1, −1)

5 0 (5, 0) 37. Graph y = x2 − 3.


Make a table of values, plot the points in the table, and
y draw the graph.
4 x  4y  5 x y (x, y)
2
−3 6 (−3, 6)
2 4 6 x
2 −1 −2 (−1, −2)
4 0 −3 (0, −3)

1 −2 (1, −2)
33. Graph 2x + 5y = −10.
In this case, it is convenient to find the intercepts along 3 6 (3, 6)
with a third point on the graph. Make a table of values,
plot the points in the table, and draw the graph. y

x y (x, y) 6

4
−5 0 (−5, 0)
2 y  x2  3
0 −2 (0, −2) 4 2 2 4 x
2
5 −4 (5, −4)

y 39. Graph y = −x2 + 2x + 3.


4 2x  5y  10 Make a table of values, plot the points in the table, and
2
draw the graph.

6 2 2 x
x y (x, y)

−2 −5 (−2, −5)
4
−1 0 (−1, 0)

35. Graph y = −x2 . 0 3 (0, 3)


Make a table of values, plot the points in the table, and 1 4 (1, 4)
draw the graph.
2 3 (2, 3)
x y (x, y)
3 0 (3, 0)
−2 −4 (−2, −4)
4 −5 (4, −5)
−1 −1 (−1, −1)

0 0 (0, 0) y
y  x 2  2x  3
1 −1 (1, −1) 4

2 −4 (2, −4) 8 4 4 8 x
4

8

12

Copyright © 2013 Pearson Education, Inc.


Exercise Set 1.1 29

41. Graph (b) is the graph of y = 3 − x. 53. Enter the equation, select the standard window, and graph
the equation as described in the Graphing Calculator Man-
43. Graph (a) is the graph of y = x2 + 2x + 1.
ual that accompanies the text.
45. Enter the equation, select the standard window, and graph
the equation as described in the Graphing Calculator Man- y  x2  6
ual that accompanies the text. 10

y  2x  1
10 10 10

10 10 10

55. Enter the equation, select the standard window, and graph
10 the equation as described in the Graphing Calculator Man-
ual that accompanies the text.
47. First solve the equation for y: y = −4x + 7. Enter the
equation in this form, select the standard window, and y  2  x2
graph the equation as described in the Graphing Calcula- 10
tor Manual that accompanies the text.

4x  y  7 10 10
10

10
10 10
57. Enter the equation, select the standard window, and graph
the equation as described in the Graphing Calculator Man-
10 ual that accompanies the text.

49. Enter the equation, select the standard window, and graph y  x 2  4x  2
the equation as described in the Graphing Calculator Man- 10
ual that accompanies the text.

y  ax  2 10 10
10

10
10 10
59. Standard window:

10

51. First solve the equation for y.


2x + 3y = −5
3y = −2x − 5
−2x − 5 1
y= , or (−2x − 5)
3 3
[−4, 4, −4, 4]
Enter the equation in “y =” form, select the standard win-
dow, and graph the equation as described in the Graphing
Calculator Manual that accompanies the text.

2x  3y  5
10

10 10
We see that the standard window is a better choice for this
graph.
10

Copyright © 2013 Pearson Education, Inc.


30 Chapter 1: Graphs, Functions, and Models

61. Standard window: 79. First we find the distance between each pair of points.
For (−4, 5) and (6, 1):

d = (−4 − 6)2 + (5 − 1)2
 √
= (−10)2 + 42 = 116
For (−4, 5) and (−8, −5):

d = (−4 − (−8))2 + (5 − (−5))2
√ √
= 42 + 102 = 116
[−1, 1, −0.3, 0.3], Xscl = 0.1, Yscl = 0.1 For (6, 1) and (−8, −5):

d = (6 − (−8))2 + (1 − (−5))2
√ √
= 142 + 62 = 232
√ √ √
Since ( 116)2 + ( 116)2 = ( 232)2 , the points could be
the vertices of a right triangle.

81. First we find the distance between each pair of points.


For (−4, 3) and (0, 5):
We see that [−1, 1, −0.3, 0.3] is a better choice for this 
d = (−4 − 0)2 + (3 − 5)2
graph.  √
= (−4)2 + (−2)2 = 20
63. Either point can be considered as (x1 , y1 ).
 For (−4, 3) and (3, −4):
d = (4 − 5)2 + (6 − 9)2 
 √ d = (−4 − 3)2 + [3 − (−4)]2
= (−1)2 + (−3)2 = 10 ≈ 3.162  √
= (−7)2 + 72 = 98
65. Either point can be considered as (x1 , y1 ). For (0, 5) and (3, −4):
 
d = (−13 − (−8))2 + (1 − (−11))2 d = (0 − 3)2 + [5 − (−4)]2
 √  √
= (−5)2 + 122 = 169 = 13 = (−3)2 + 92 = 90

67. Either point can be considered as (x1 , y1 ). The greatest distance is 98, so if the points are the ver-

d = (6 − 9)2 + (−1 − 5)2 tices
√ of a right
√ triangle,
√ then it is the hypotenuse. But
 √ ( 20)2 + ( 90)2 = ( 98)2 , so the points are not the ver-
= (−3)2 + (−6)2 = 45 ≈ 6.708 tices of a right triangle.
69. Either point can be considered as (x1 , y1 ). 83. We use the midpoint formula.
  2    
7 7 4 + (−12) −9 + (−3) 8 12
d = (−8 − 8)2 + − , = − ,− = (−4, −6)
11 11 2 2 2 2

= (−16)2 + 02 = 16 85. We use the midpoint formula.
  
  2  2
0 + − 2 1
2 1  
3 3 2 −0 −
71. d = − − − + −4− 5
, 2 = 5 , 2 1 1
= − ,
5 5 3 2 2 2 2 5 4
  2
14 14
= 02 + − = 87. We use the midpoint formula.
3 3    
6.1 + 3.8 −3.8 + (−6.1) 9.9 9.9
, = ,− =
73. Either point can be considered as (x1 , y1 ). 2 2 2 2

d = (−4.2 − 2.1)2 + [3 − (−6.4)]2 (4.95, −4.95)
 √
= (−6.3)2 + (9.4)2 = 128.05 ≈ 11.316 89. We use the midpoint formula.
     
−6 + (−6) 5 + 8 12 13 13
75. Either point can be considered as (x1 , y1 ). , = − , = − 6,
 √ 2 2 2 2 2
d = (0 − a)2 + (0 − b)2 = a2 + b2
91. We use the midpoint formula.
77. First we find the length of the diameter:
1  2 3 5
5 13
 − + − − + −
d = (−3 − 9)2 + (−1 − 4)2 6 3 , 5 4 = 6 , 20 =
 √ 2 2 2 2
= (−12)2 + (−5)2 = 169 = 13  
5 13
The length of the radius is one-half the length of the di- − ,
1 12 40
ameter, or (13), or 6.5.
2

Copyright © 2013 Pearson Education, Inc.


Exercise Set 1.1 31

93. 101. The length of a radius is the distance between (−1, 4) and
y
(3, 7):
8 
6 r = (−1 − 3)2 + (4 − 7)2
 √
4 = (−4)2 + (−3)2 = 25 = 5
2
(x − h)2 + (y − k)2 = r2
4 2 4 6 x
[x − (−1)]2 + (y − 4)2 = 52
2
(x + 1)2 + (y − 4)2 = 25
For the side with vertices (−3, 4) and (2, −1): 103. The center is the midpoint of the diameter:
     
−3 + 2 4 + (−1) 1 3 7 + (−3) 13 + (−11)
, = − , , = (2, 1)
2 2 2 2 2 2
For the side with vertices (2, −1) and (5, 2): Use the center and either endpoint of the diameter to find
   
2 + 5 −1 + 2 7 1 the length of a radius. We use the point (7, 13):
, = , 
2 2 2 2 r = (7 − 2)2 + (13 − 1)2
√ √
For the side with vertices (5, 2) and (0, 7): = 52 + 122 = 169 = 13
   
5+0 2+7 5 9
, = , (x − h)2 + (y − k)2 = r2
2 2 2 2
For the side with vertices (0, 7) and (−3, 4): (x − 2)2 + (y − 1)2 = 132
   
0 + (−3) 7 + 4 3 11 (x − 2)2 + (y − 1)2 = 169
, = − ,
2 2 2 2 105. Since the center is 2 units to the left of the y-axis and the
For the quadrilateral whose vertices are the points found circle is tangent to the y-axis, the length of a radius is 2.
above, the diagonals have endpoints (x − h)2 + (y − k)2 = r2
       
1 3 5 9 7 1 3 11 [x − (−2)]2 + (y − 3)2 = 22
− , , , and , , − , .
2 2 2 2 2 2 2 2
(x + 2)2 + (y − 3)2 = 4
We find the length of each of these diagonals.
    107. x2 + y 2 = 4
1 3 5 9
For − , , , : (x − 0) + (y − 0)2 = 22
2
2 2 2 2
 2  2 Center: (0, 0); radius: 2
1 5 3 9
d= − − + −
2 2 2 2 y
 √
= (−3)2 + (−3)2 = 18
    4
x2  y 2  4
7 1 3 11
For , , − , : 2
2 2 2 2
  2  2 4 2 2 4 x
7 3 1 11
d= − − + − 2
2 2 2 2
 √
4
= 52 + (−5)2 = 50
Since the diagonals do not have the same lengths, the mid-
points are not vertices of a rectangle.
109. x2 + (y − 3)2 = 16
95. We use the midpoint formula. (x − 0)2 + (y − 3)2 = 42
√ √  √ √ 
7 + 2 −4 + 3 7+ 2 1 Center: (0, 3); radius: 4
, = ,−
2 2 2 2
y
97. Square the viewing window. For the graph shown, one 8
possibility is [−12, 9, −4, 10].
6
99. (x − h)2 + (y − k)2 = r2
 2 4
5
(x − 2)2 + (y − 3)2 = Substituting 2
3
25
(x − 2)2 + (y − 3)2 = 4 2 2 4 x
9 2

x 2  (y  3)2  16

Copyright © 2013 Pearson Education, Inc.


32 Chapter 1: Graphs, Functions, and Models

111. (x − 1)2 + (y − 5)2 = 36 Then we have:


(x − 1) + (y − 5) = 6
2 2 2 (x − h)2 + (y − k)2 = r2
Center: (1, 5); radius: 6 (x − 2)2 + [y − (−7)]2 = 36
(x − 2)2 + (y + 7)2 = 36
y
125. Let (0, y) be the required point. We set the distance from
12 (−2, 0) to (0, y) equal to the distance from (4, 6) to (0, y)
and solve for y.
8  
[0 − (−2)]2 + (y − 0)2 = (0 − 4)2 + (y − 6)2
4  
4 + y 2 = 16 + y 2 − 12y + 36
8 4 4 8 x 4 + y 2 = 16 + y 2 − 12y + 36
4 Squaring both sides
−48 = −12y
(x  1)2  (y  5)2  36 4=y
The point is (0, 4).
113. (x + 4)2 + (y + 5)2 = 9
[x − (−4)]2 + [y − (−5)]2 = 32 127. a) When the circle is positioned on a coordinate system
as shown in the text, the center lies on the y-axis
Center: (−4, −5); radius: 3 and is equidistant from (−4, 0) and (0, 2).
y Let (0, y) be the coordinates of the center.
 
2 (−4−0)2 +(0−y)2 = (0−0)2 +(2−y)2
42 + y 2 = (2 − y)2
6 4 2 2 x
2
16 + y 2 = 4 − 4y + y 2
12 = −4y
4
−3 = y
6
The center of the circle is (0, −3).
8
b) Use the point (−4, 0) and the center (0, −3) to find
(x  4)2  (y  5)2  9 the radius.
(−4 − 0)2 + [0 − (−3)]2 = r2
115. From the graph we see that the center of the circle is 25 = r2
(−2, 1) and the radius is 3. The equation of the circle
5=r
is [x − (−2)]2 + (y − 1)2 = 32 , or (x + 2)2 + (y − 1)2 = 32 .
The radius is 5 ft.
117. From the graph we see that the center of the circle is
(5, −5) and the radius is 15. The equation of the circle 129. x2 + y 2 = 1
is (x − 5)2 + [y − (−5)]2 = 152 , or (x − 5)2 + (y + 5)2 = 152 .  √ 2  2
3 1
+ − ? 1
119. If the point (p, q) is in the fourth quadrant, then p > 0 2 2 
and q < 0. If p > 0, then −p < 0 so both coordinates of 
3 1 
the point (q, −p) are negative and (q, −p) is in the third + 
4 4 
quadrant. 
1  1 TRUE
121. Use the distance formula. Either point can be considered √ 
3 1
as (x1 , y1 ). ,− lies on the unit circle.
√ 2 2

d = (a + h − a)2 + ( a + h − a)2
 √ 131. x2 + y 2 = 1
= h2 + a + h − 2 a2 + ah + a  √ 2  √ 2
 √ 2 2
= h2 + 2a + h − 2 a2 + ah − + ? 1
2 2 

Next we use the midpoint formula.
 √ √   √ √  2 2 
+ 
a+a+h a+ a+h 2a+h a+ a+h 4 4 
, = , 
2 2 2 2 1  1 TRUE
123. First use the formula for the area of a circle to find r2 :  √ √ 
2 2
A = πr2 − , lies on the unit circle.
2 2
36π = πr2
133. See the answer section in the text.
36 = r2

Copyright © 2013 Pearson Education, Inc.


Exercise Set 1.2 33

23. g(x) = x3
Exercise Set 1.2 a) g(2) = 23 = 8
b) g(−2) = (−2)3 = −8
1. This correspondence is a function, because each member
c) g(−x) = (−x)3 = −x3
of the domain corresponds to exactly one member of the
range. d) g(3y) = (3y)3 = 27y 3
e) g(2 + h) = (2 + h)3 = 8 + 12h + 6h2 + h3
3. This correspondence is a function, because each member
of the domain corresponds to exactly one member of the x−4
range. 25. g(x) =
x+3
5. This correspondence is not a function, because there is a 5−4 1
a) g(5) = =
member of the domain (m) that corresponds to more than 5+3 8
one member of the range (A and B). 4−4
b) g(4) = =0
4+7
7. This correspondence is a function, because each member −3 − 4 −7
of the domain corresponds to exactly one member of the c) g(−3) = =
−3 + 3 0
range.
Since division by 0 is not defined, g(−3) does not
9. This correspondence is a function, because each car has exist.
exactly one license number. −16.25 − 4 −20.25 81
d) g(−16.25) = = = ≈ 1.5283
11. This correspondence is a function, because each integer −16.25 + 3 −13.25 53
less than 9 corresponds to exactly one multiple of 5. x+h−4
e) g(x + h) =
x+h+3
13. This correspondence is not a function, because at least one x
27. g(x) = √
student will have more than one neighboring seat occupied 1 − x2
by another student. 0 0 0
g(0) = √ = √ = =0
15. The relation is a function, because no two ordered pairs 1−0 2 1 1
have the same first coordinate and different second coor- −1 −1 −1 −1
g(−1) =  =√ =√ =
dinates. 1 − (−1)2 1−1 0 0
The domain is the set of all first coordinates: Since division by 0 is not defined, g(−1) does not exist.
{2, 3, 4}. 5 5 5
g(5) = √ =√ =√
The range is the set of all second coordinates: {10, 15, 20}. 1−5 2 1 − 25 −24

17. The relation is not a function, because the ordered pairs Since −24 is not defined as a real number, g(5) does not
(−2, 1) and (−2, 4) have the same first coordinate and dif- exist as a real number.
ferent second coordinates.   1 1 1
1 2 2 2
The domain is the set of all first coordinates: g =  2 = = =
2 1 3
{−7, −2, 0}. 1−
1 1−
2 4 4
The range is the set of all second coordinates: {3, 1, 4, 7}.
1 √
1 2 1·2 1 3
19. The relation is a function, because no two ordered pairs √2 = · √ = √ = √ , or
have the same first coordinate and different second coor- 3 2 3 2 3 3 3
dinates. 2
The domain is the set of all first coordinates: 29.
{−2, 0, 2, 4, −3}.
The range is the set of all second coordinates: {1}.

21. g(x) = 3x2 − 2x + 1


a) g(0) = 3 · 02 − 2 · 0 + 1 = 1
b) g(−1) = 3(−1)2 − 2(−1) + 1 = 6
c) g(3) = 3 · 32 − 2 · 3 + 1 = 22 Rounding to the nearest tenth, we see that g(−2.1) ≈
d) g(−x) = 3(−x)2 − 2(−x) + 1 = 3x2 + 2x + 1 −21.8, g(5.08) ≈ −130.4, and g(10.003) ≈ −468.3.

e) g(1 − t) = 3(1 − t)2 − 2(1 − t) + 1 = 31. Graph f (x) =


1
x + 3.
2
3(1−2t+t2 )−2(1−t)+1 = 3−6t+3t2 −2+2t+1 =
We select values for x and find the corresponding values
3t2 − 4t + 2 of f (x). Then we plot the points and connect them with
a smooth curve.

Copyright © 2013 Pearson Education, Inc.


34 Chapter 1: Graphs, Functions, and Models

x f (x) (x, f (x)) output is 0, so s(−2) = 0. When the input is 0, the output
is −3, so s(0) = −3.
−4 1 (−4, 1)
41. From the graph we see that, when the input is −1, the
0 3 (0, 3) output is 2, so f (−1) = 2. When the input is 0, the output
is 0, so f (0) = 0. When the input is 1, the output is −2,
2 4 (2, 4) so f (1) = −2.

y 43. This is not the graph of a function, because we can find a


vertical line that crosses the graph more than once.
4

2
y
4 2 2 4 x
2

4

f (x)  1 x  3 x
2

33. Graph f (x) = −x2 + 4.


We select values for x and find the corresponding values
of f (x). Then we plot the points and connect them with
a smooth curve. 45. This is the graph of a function, because there is no vertical
line that crosses the graph more than once.
x f (x) (x, f (x))
47. This is the graph of a function, because there is no vertical
−3 −5 (−3, −5)
line that crosses the graph more than once.
−2 0 (−2, 0) y 49. This is not the graph of a function, because we can find a
−1 3 (−1, 3) 4 vertical line that crosses the graph more than once.
2 51. We can substitute any real number for x. Thus, the do-
0 4 (0, 4)
4 2 2 4
main is the set of all real numbers, or (−∞, ∞).
x
1 3 (1, 3) 2
53. We can substitute any real number for x. Thus, the do-
2 0 (2, 0) 4 main is the set of all real numbers, or (−∞, ∞).

3 −5 (3, −5) 55. The input 0 results in a denominator of 0. Thus, the do-
f (x)  x 2  4 main is {x|x = 0}, or (−∞, 0) ∪ (0, ∞).

35. Graph f (x) = x − 1. 57. We can substitute any real number in the numerator, but
We select values for x and find the corresponding values we must avoid inputs that make the denominator 0. We
of f (x). Then we plot the points and connect them with find these inputs.
a smooth curve. 2−x = 0
x f (x) (x, f (x)) y 2=x
4 The domain is {x|x = 2}, or (−∞, 2) ∪ (2, ∞).
1 0 (1, 0)
2 59. We find the inputs that make the denominator 0:
2 1 (2, 1)
4 2 2 4 x x2 − 4x − 5 = 0
2
4 1.7 (4, 1.7) (x − 5)(x + 1) = 0
4
5 2 (5, 2) x − 5 = 0 or x + 1 = 0
x = 5 or x = −1
f (x )  兹x  1
The domain is {x|x = 5 and x = −1}, or
(−∞, −1) ∪ (−1, 5) ∪ (5, ∞).
37. From the graph we see that, when the input is 1, the output
is −2, so h(1) = −2. When the input is 3, the output is 61. We can substitute any real number for x. Thus, the do-
2, so h(3) = 2. When the input is 4, the output is 1, so main is the set of all real numbers, or (−∞, ∞).
h(4) = 1.
63. We can substitute any real number in the numerator, but
39. From the graph we see that, when the input is −4, the we must avoid inputs that make the denominator 0. We
output is 3, so s(−4) = 3. When the input is −2, the find these inputs.

Copyright © 2013 Pearson Education, Inc.


Exercise Set 1.2 35

x2 − 7x = 0 79.
x(x − 7) = 0
x = 0 or x − 7 = 0
x = 0 or x=7
The domain is {x|x = 0 and x = 7}, or (−∞, 0) ∪ (0, 7) ∪
(7, ∞).
65. We can substitute any real number for x. Thus, the do-
We see that each point on the x-axis except 3 corresponds
main is the set of all real numbers, or (−∞, ∞).
to a point on the graph, so the domain is (−∞, 3) ∪ (3, ∞).
67. The inputs on the x-axis that correspond to points on the We also see that each point on the y-axis except 0 corre-
graph extend from 0 to 5, inclusive. Thus, the domain is sponds to an output, so the range is (−∞, 0) ∪ (0, ∞).
{x|0 ≤ x ≤ 5}, or [0, 5].
81.
The outputs on the y-axis extend from 0 to 3, inclusive.
Thus, the range is {y|0 ≤ y ≤ 3}, or [0, 3].
69. The inputs on the x-axis that correspond to points on the
graph extend from −2π to 2π inclusive. Thus, the domain
is {x| − 2π ≤ x ≤ 2π}, or [−2π, 2π].
The outputs on the y-axis extend from −1 to 1, inclusive.
Thus, the range is {y| − 1 ≤ y ≤ 1}, or [−1, 1].
Each point on the x-axis corresponds to a point on the
71. The graph extends to the left and to the right without graph, so the domain is the set of all real numbers, or
bound. Thus, the domain is the set of all real numbers, or (−∞, ∞).
(−∞, ∞).
Each point on the y-axis also corresponds to a point on the
The only output is −3, so the range is {−3}. graph, so the range is the set of all real numbers, (−∞, ∞).
73. The inputs on the x-axis extend from −5 to 3, inclusive. 83.
Thus, the domain is [−5, 3].
The outputs on the y-axis extend from −2 to 2, inclusive.
Thus, the range is [−2, 2].
75.

The largest input on the x-axis is 7 and every number less


than 7 is also an input. Thus, the domain is (−∞, 7].
The number 0 is the smallest output, and every number
greater than 0 is also an output. Thus, the range is [0, ∞).
To find the domain we look for the inputs on the x-axis
85.
that correspond to a point on the graph. We see that each
point on the x-axis corresponds to a point on the graph so
the domain is the set of all real numbers, or (−∞, ∞).
To find the range we look for outputs on the y-axis. The
number 0 is the smallest output, and every number greater
than 0 is also an output. Thus, the range is [0, ∞).
77.
Each point on the x-axis corresponds to a point on the
graph, so the domain is the set of all real numbers, or
(−∞, ∞).
The largest output is 3 and every number less than 3 is
also an output. Thus, the range is (−∞, 3].

87. a) In 2018, x = 2018 − 1985 = 33.


We see that each point on the x-axis corresponds to a point V (33) = 0.4652(33) + 10.87 ≈ $26.22
on the graph so the domain is the set of all real numbers,
or (−∞, ∞). We also see that each point on the y-axis In 2025, x = 2025 − 1985 = 40.
corresponds to an output so the range is the set of all real V (40) = 0.4652(40) + 10.87 ≈ $29.48
numbers, or (−∞, ∞).

Copyright © 2013 Pearson Education, Inc.


36 Chapter 1: Graphs, Functions, and Models

b) Substitute 40 for V (x) and solve for x. 95. Graph −2x − 5y = 10.
40 = 0.4652x + 10.87 Make a table of values, plot the points in the table, and
29.13 = 0.4652x draw the graph.
y
63 ≈ x x y (x, y)
4
It will take approximately $40 to equal the value of −5 0 (−5, 0)
$1 in 1913 about 63 yr after 1985, or in 2048. 2

0 −2 (0, −2)
89. E(t) = 1000(100 − t) + 580(100 − t)2 4 2 2 4 x
2
a) E(99.5) = 1000(100−99.5)+580(100−99.5)2 5 −4 (5, −4)
4

= 1000(0.5) + 580(0.5)2 2x  5y  10


= 500 + 580(0.25) = 500 + 145
= 645 m above sea level 97. We find the inputs for which 2x + 5 is nonnegative.
b) E(100) = 1000(100 − 100) + 580(100 − 100)2 2x + 5 ≥ 0
2x ≥ −5
= 1000 · 0 + 580(0)2 = 0 + 0
5
= 0 m above sea level, or at sea level x≥−
2
    
91. To determine whether (0, −7) is a solution, substitute 0  5 5

Thus, the domain is xx ≥ − , or − , ∞ .
for x and −7 for y. 2 2
y = 0.5x + 7 √
99. x + 6 is not defined for values of x for which x + 6 is
−7 ? 0.5(0) + 7 negative. We find the inputs for which x+6 is nonnegative.
 x+6 ≥ 0
 0+7

−7  7 FALSE x ≥ −6
The equation −7 = 7 is false, so (0, −7) is not a solution. We must also avoid inputs that make the denominator 0.
To determine whether (8, 11) is a solution, substitute 8 for (x + 2)(x − 3) = 0
x and 11 for y. x+2 = 0 or x − 3 = 0
y = 0.5x + 7 x = −2 or x=3
11 ? 0.5(8) + 7 Then the domain is {x|x ≥ −6 and x = −2 and x = 3},
 or [−6, −2) ∪ (−2, 3) ∪ (3, ∞).
 4+7

11  11 TRUE 101. Answers may vary. Two possibilities are f (x) = x, g(x) =
x + 1 and f (x) = x2 , g(x) = x2 − 4.
The equation 11 = 11 is true, so (8, 11) is a solution.
103. First find the value of x for which x + 3 = −1.
93. Graph y = (x − 1)2 . x + 3 = −1
Make a table of values, plot the points in the table, and x = −4
draw the graph.
Then we have:
x y (x, y) g(x + 3) = 2x + 1
−1 4 (−1, 4) y g(−1) = g(−4 + 3) = 2(−4) + 1 = −8 + 1 = −7

0 1 (0, 1) 4

2
Exercise Set 1.3
1 0 (1, 0)
4 2 2 4 x 1. a) Yes. Each input is 1 more than the one that pre-
2 1 (2, 1) 2
cedes it.
y  (x 1)2
4
3 4 (3, 4) b) Yes. Each output is 3 more than the one that pre-
cedes it.
c) Yes. Constant changes in inputs result in constant
changes in outputs.
3. a) Yes. Each input is 15 more than the one that pre-
cedes it.
b) No. The change in the outputs varies.
c) No. Constant changes in inputs do not result in
constant changes in outputs.

Copyright © 2013 Pearson Education, Inc.


Exercise Set 1.3 37

5. Two points on the line are (−4, −2) and (1, 4). 39. The graph of y = 0.7 is a horizontal line, so the slope is
y2 − y1 4 − (−2) 6 0. (We also see this if we write the equation in the form
m= = = y = 0x + 0.7).
x2 − x1 1 − (−4) 5
7. Two points on the line are (0, 3) and (5, 0). 41. We have the points (2009, 945) and (1998, 425). We find
y2 − y1 0−3 −3 3 the average rate of change, or slope.
m= = = , or − 945 − 425 520
x2 − x1 5−0 5 5 m= = ≈ 47.3
2009 − 1998 11
y2 − y1 3−3 0
9. m = = = =0 The average rate of change in the revenue of the fireworks
x2 − x1 3−0 3 industry in the U.S. from 1998 to 2009 was about $47.3 mil-
y2 − y 1 2−4 −2 1 lion per year.
11. m = = = =
x2 − x1 −1 − 9 −10 5
43. We have the points (2000, 348, 194) and (2010, 319, 294).
y2 − y1 6 − (−9) 15 We find the average rate of change, or slope.
13. m = = =
x2 − x1 4−4 0 348, 194 − 319, 294 28, 900
m= = = −2890
Since division by 0 is not defined, the slope is not defined. 2000 − 2010 −10
y2 − y1 −0.4 − (−0.1) −0.3 The average rate of change in the population in St. Louis,
15. m = = = = 0.3 Missouri, from 2000 to 2010 was −2890 people per year.
x2 − x1 −0.3 − 0.7 −1
y 2 − y1 −2 − (−2) 0 45. We have the points (2009, 21.0) and (2012, 25.0). We find
17. m = = = =0 the average rate of change, or slope.
x2 − x1 4−2 2
  21.0 − 25.0 −4.0
3 3 6 m= = ≈ 1.3
− − 2009 − 2012 −3
y2 − y1 5 5 6
19. m = = = 5 =− The average rate of change in the sales of electric bikes in
x2 − x1 1 1 −1 5
− − China from 2009 to 2012 was about 1.3 million bikes per
2 2
year.
y 2 − y1 −5 − (−13) 8 1
21. m = = = =− 47. We have the points (1990, 3.1) and (2008, 5.5). We find
x2 − x1 −8 − 16 −24 3
the average rate of change, or slope.
7 − (−7) 14 5.5 − 3.1 2.4
23. m = = m= = ≈ 0.13
−10 − (−10) 0 2008 − 1990 18
Since division by 0 is not defined, the slope is not defined. The average rate of change in the consumption of broccoli
per capita from 1990 to 2008 was about 0.13 lb per year.
25. We have the points (4, 3) and (−2, 15).
y2 − y1 15 − 3 12 3
m= = = = −2 49. y = x−7
x2 − x1 −2 − 4 −6 5
    3
1 1 11 The equation is in the form y = mx + b where m =
27. We have the points , and − 1, − . 5
5 2 2 3
and b = −7. Thus, the slope is , and the y-intercept is
11 1   5
y2 − y1 − − (0, −7).
m= = 2 2 = −6 = −6 · − 5 = 5
x2 − x1 1 6 6 2
−1 − − 51. x = −
5 5 5
   
4 4 2
29. We have the points − 6, and 0, . This is the equation of a vertical line unit to the left
5 5 5
of the y-axis. The slope is not defined, and there is no
4 4
y2 − y1 − 0
y-intercept.
m= = 5 5 = =0
x2 − x1 −6 − 0 −6 1 1
53. f (x) = 5 − x, or f (x) = − x + 5
2 2
31. y = 1.3x − 5 is in the form y = mx + b with m = 1.3, so The second equation is in the form y = mx + b where
the slope is 1.3. 1 1
m = − and b = 5. Thus, the slope is − and the y-
2 2
33. The graph of x = −2 is a vertical line, so the slope is not intercept is (0, 5).
defined.
55. Solve the equation for y.
1 1
35. f (x) = − x + 3 is in the form y = mx + b with m = − , 3x + 2y = 10
2 2
1 2y = −3x + 10
so the slope is − .
2 3
37. y = 9 − x can be written as y = −x + 9, or y = −1 · x + 9. y = − x+5
2
Now we have an equation in the form y = mx + b with 3
m = −1, so the slope is −1. Slope: − ; y-intercept: (0, 5)
2

Copyright © 2013 Pearson Education, Inc.


38 Chapter 1: Graphs, Functions, and Models

57. y = −6 = 0 · x − 6 67. First solve the equation for y.


Slope: 0; y-intercept: (0, −6) 3x − 4y = 20
−4y = −3x + 20
59. Solve the equation for y.
3
5y − 4x = 8 y = x−5
4
5y = 4x + 8 3
4 8 Plot the y-intercept, (0, −5). Then using the slope, ,
y = x+ 4
5 5 start at (0, −5) and find another point by moving up
  3 units and right 4 units. We have the point (4, −2). We
4 8
Slope: ; y-intercept: 0, can move from the point (4, −2) in a similar manner to get
5 5
a third point, (8, 1). Connect the three points to draw the
61. Solve the equation for y. graph.
4y − x + 2 = 0
y
4y = x − 2
1 1 3x  4y  20
y = x− 2
4 2
  4 2 2 4 x
1 1
Slope: ; y-intercept: 0, − 2
4 2
4
1
63. Graph y = − x − 3. 6
2
Plot the y-intercept, (0, −3). We can think of the slope
−1 69. First solve the equation for y.
as . Start at (0, −3) and find another point by moving
2 x + 3y = 18
down 1 unit and right 2 units. We have the point (2, −4).
1 3y = −x + 18
We could also think of the slope as . Then we can start 1
−2 y = − x+6
at (0, −3) and get another point by moving up 1 unit and 3
left 2 units. We have the point (−2, −2). Connect the Plot the y-intercept, (0, 6). We can think of the slope as
three points to draw the graph. −1
. Start at (0, 6) and find another point by moving down
3
y 1 unit and right 3 units. We have the point (3, 5). We can
move from the point (3, 5) in a similar manner to get a
4 1 third point, (6, 4). Connect the three points and draw the
y  x
2 3
2
graph.

4 2 2 4 y
x
2

4 6

2 x  3y  18
65. Graph f (x) = 3x − 1.
Plot the y-intercept, (0, −1). We can think of the slope 4 2 2 4 x
3 2
as . Start at (0, −1) and find another point by moving
1
up 3 units and right 1 unit. We have the point (1, 2). We
71. a) 1
can move from the point (1, 2) in a similar manner to get y ⫽ ⫺x ⫹ 1
a third point, (2, 5). Connect the three points to draw the 33
300
graph.

4
0 8000
2 0
1
4 2 2 4 x b) P (0) = · 0 + 1 = 1 atm
2
33
f(x)  3x  1 1
4 P (33) = · 33 + 1 = 2 atm
33
1 10
P (1000) = · 1000 + 1 = 31 atm
33 33

Copyright © 2013 Pearson Education, Inc.


Chapter 1 Mid-Chapter Mixed Review 39

1 17 87. False. For example, let f (x) = x + 1. Then f (c − d) =


P (5000) = · 5000 + 1 = 152 atm
33 33 c − d + 1, but f (c) − f (d) = c + 1 − (d + 1) = c − d.
1 4
P (7000) = · 7000 + 1 = 213 atm 89. f (x) = mx + b
33 33
11 1 f (x + 2) = f (x) + 2
73. a) D(r) = r+
10 2 m(x + 2) + b = mx + b + 2
11 mx + 2m + b = mx + b + 2
The slope is .
10
2m = 2
11
For each mph faster the car travels, it takes ft m=1
10
longer to stop. Thus, f (x) = 1 · x + b, or f (x) = x + b.
b) 11 1
y ⫽ ⫺x ⫹ ⫺
10 2 Chapter 1 Mid-Chapter Mixed Review
100

1. The statement is false. The x-intercept of a line that passes


through the origin is (0, 0).

0 100 3. The statement is false. The line parallel to the y-axis that
0
passes through (−5, 25) is x = −5.
11 1 11 1 12
c) D(5) = ·5+ = + = = 6 ft 5. Distance:

10 2 2 2 2
d = (−8 − 3)2 + (−15 − 7)2
11 1 1 1 
D(10) = · 10 + = 11 + = 11 , or 11.5 ft = (−11)2 + (−22)2
10 2 2 2 √
11 1 1 1 = 121 + 484
D(20) = · 20 + = 22 + = 22 , or 22.5 ft √
10 2 2 2 = 605 ≈ 24.6
11 1 1 1    
D(50) = · 50 + = 55 + = 55 , or 55.5 ft −8 + 3 −15 + 7 −5 −8
10 2 2 2 Midpoint: , = , =
2 2 2 2
11 1 143 1 144  
D(65) = · 65 + = + = = 72 ft 5
10 2 2 2 2 − , −4
1 2
d) The speed cannot be negative. D(0) = which
2 7. (x − h)2 + (y − k)2 = r2
1
says that a stopped car travels ft before stop-
2 (x − (−5))2 + (y − 2)2 = 132
ping. Thus, 0 is not in the domain. The speed can
(x + 5)2 + (y − 2)2 = 169
be positive, so the domain is {r|r > 0}, or (0, ∞).
9. Graph 3x − 6y = 6.
75. C(t) = 89 + 114.99t
We will find the intercepts along with a third point on the
C(24) = 89 + 114.99(24) = $2848.76
graph. Make a table of values, plot the points, and draw
77. C(x) = 750 + 15x the graph.
y
C(32) = 750 + 15 · 32 = $1230 x y (x, y)
79. f (x) = x2 − 3x 2 0 (2, 0) 3x  6y  6
   2 4
1 1 1 1 3 5
f = −3· = − =− 0 −1 (0, −1) 2
2 2 2 4 2 4
4 1 (4, 1)
81. f (x) = x2 − 3x 4 2 2 4 x
f (−5) = (−5) − 3(−5) = 25 + 15 = 40
2 2

83. f (x) = x2 − 3x 4

f (a + h) = (a + h)2 − 3(a + h) = a2 + 2ah + h2 − 3a − 3h


85. y2 − y1 (a+h)2 −a2 a2 +2ah+h2 −a2 11. Graph y = 2 − x2 .
m= = = =
x2 − x1 a+h−a h
We choose some values for x and find the corresponding
2ah + h2 h(2a + h) y-values. We list these points in a table, plot them, and
= = 2a + h
h h draw the graph.

Copyright © 2013 Pearson Education, Inc.


40 Chapter 1: Graphs, Functions, and Models

y 27. 3x − 16y + 1 = 0
x y (x, y)
3x + 1 = 16y
−2 −2 (−2, −2) 4 y  2  x2 3 1
x+ =y
−1 1 (−1, 1) 2 16 16
 
3 1
0 2 (0, 2) Slope: ; y-intercept: 0,
4 2 2 4 x 16 16
1 1 (1, 1) 2 29. A vertical line (x = a) crosses the graph more than once.
2 −2 (2, −2) 4
31. Let A = (a, b) and B = (c, d). The coordinates  of a point

a+c b+d
C one-half of the way from A to B are , .
2 2
13. f (x) = x − 2x2 A point D that is one-half of the way from C to B is
1 1 1 3
f (−4) = −4 − 2(−4)2 = −4 − 2 · 16 = −4 − 32 = −36 + · , or of the way from A to B. Its coordinates
2 2 2 4
f (0) = 0 − 2 · 02 = 0 − 0 = 0  a+c b+d   
2 +c +d a + 3c b + 3d
f (1) = 1 − 2 · 12 = 1 − 2 · 1 = 1 − 2 = −1 are , 2 , or , . Then a
2 2 4 4
point E that is one-half of the way from D to B is
15. We can substitute any real number for x. Thus, the do- 3 1 1 7
+ · , or of the way from A to B. Its coordinates
main is the set of all real numbers, or (−∞, ∞). 4 2 4 8   
a+3c b+3d
4 +c +d a + 7c b + 7d
17. We find the inputs for which the denominator is 0. are , 4 , or , .
2 2 8 8
x2 + 2x − 3 = 0
(x + 3)(x − 1) = 0
Exercise Set 1.4
x+3 = 0 or x − 1 = 0
x = −3 or x=1 1. We see that the y-intercept is (0, −2). Another point on
The domain is {x|x
= −3 and x
= 1}, or the graph is (1, 2). Use these points to find the slope.
(−∞, −3) ∪ (−3, 1) ∪ (1, ∞). y2 − y1 2 − (−2) 4
m= = = =4
19. Graph g(x) = x2 − 1. x2 − x1 1−0 1
We have m = 4 and b = −2, so the equation is y = 4x − 2.
Make a table of values, plot the points in the table, and
draw the graph. 3. We see that the y-intercept is (0, 0). Another point on the
graph is (3, −3). Use these points to find the slope.
x g(x) (x, g(x))
y2 − y1 −3 − 0 −3
m= = = = −1
−2 3 (−2, 3) x2 − x1 3−0 3
y We have m = −1 and b = 0, so the equation is
−1 0 (−1, 0)
y = −1 · x + 0, or y = −x.
0 −1 (0, −1) 4 5. We see that the y-intercept is (0, −3). This is a horizontal
1 0 (1, 0) line, so the slope is 0. We have m = 0 and b = −3, so the
2
equation is y = 0 · x − 3, or y = −3.
2 3 (2, 3)
4 2 2 4 x 2
7. We substitute for m and 4 for b in the slope-intercept
2 9
equation.
4 g (x )  x 2  1
y = mx + b
2
y = x+4
y2 − y1 −5 − 13 −18 9
21. m = = =
x2 − x1 −2 − (−2) 0 9. We substitute −4 for m and −7 for b in the slope-intercept
Since division by 0 is not defined, the slope is not defined. equation.
y = mx + b
1 1
y2 − y1 − y = −4x − 7
23. m = = 3 3 = 0 =0
x2 − x1 2 5 3
− − 3
7 7 7 11. We substitute −4.2 for m and for b in the slope-intercept
4
25. We can write y = −6 as y = 0x − 6, so the slope is 0 and equation.
the y-intercept is (0, −6). y = mx + b
3
y = −4.2x +
4

Copyright © 2013 Pearson Education, Inc.


Exercise Set 1.4 41

13. Using the point-slope equation: Using the slope-intercept equation:


y − y1 = m(x − x1 ) y = mx + b
2 3
y − 7 = (x − 3) Substituting −1 = − (−4) + b
9 5
2 2 12
y−7 = x− −1 = +b
9 3 5
2 19 17
y = x+ Slope-intercept equation − =b
9 3 5
Using the slope-intercept equation: 3 17
Then we have y = − x − .
2 5 5
Substitute for m, 3 for x, and 7 for y in the slope-
9 19. First we find the slope.
intercept equation and solve for b.
−4 − 5 −9
y = mx + b m= = = −3
2 − (−1) 3
2
7 = ·3+b Using the point-slope equation:
9
2 Using the point (−1, 5), we get
7 = +b
3 y − 5 = −3(x − (−1)), or y − 5 = −3(x + 1).
19 Using the point (2, −4), we get
=b
3
y − (−4) = −3(x − 2), or y + 4 = −3(x − 2).
2 19
Now substitute for m and for b in y = mx + b. In either case, the slope-intercept equation is
9 3
2 19 y = −3x + 2.
y = x+
9 3 Using the slope-intercept equation and the point (−1, 5):
15. The slope is 0 and the second coordinate of the given point y = mx + b
is 8, so we have a horizontal line 8 units above the x-axis. 5 = −3(−1) + b
Thus, the equation is y = 8.
5 = 3+b
We could also use the point-slope equation or the slope-
2=b
intercept equation to find the equation of the line.
Then we have y = −3x + 2.
Using the point-slope equation:
y − y1 = m(x − x1 ) 21. First we find the slope.
y − 8 = 0(x − (−2)) Substituting 4−0 4 1
m= = =−
−1 − 7 −8 2
y−8 = 0
Using the point-slope equation:
y=8
Using the point (7, 0), we get
Using the slope-intercept equation:
1
y = mx + b y − 0 = − (x − 7).
2
y = 0(−2) + 8 Using the point (−1, 4), we get
y=8 1
y − 4 = − (x − (−1)), or
2
17. Using the point-slope equation:
1
y − y1 = m(x − x1 ) y − 4 = − (x + 1).
2
3 In either case, the slope-intercept equation is
y − (−1) = − (x − (−4)) 1 7
5 y =− x+ .
3 2 2
y + 1 = − (x + 4)
5 Using the slope-intercept equation and the point (7, 0):
3 12 1
y+1 = − x− 0 = − ·7+b
5 5 2
3 17 7
y = − x− Slope-intercept =b
5 5 2
equation 1 7
Then we have y = − x + .
2 2
23. First we find the slope.
−4 − (−6) 2
m= =
3−0 3

Copyright © 2013 Pearson Education, Inc.


42 Chapter 1: Graphs, Functions, and Models

We know the y-intercept is (0, −6), so we substitute in the 26 3


35. The slopes are and − . Their product is −1, so the
slope-intercept equation. 3 26
lines are perpendicular.
y = mx + b
2 2
2 37. The slopes are and − . The slopes are not the same and
y = x−6 5 5
3 their product is not −1, so the lines are neither parallel nor
25. First we find the slope. perpendicular.
7.3 − 7.3 0
m= = =0 39. We solve each equation for y.
−4 − 0 −4
x + 2y = 5 2x + 4y = 8
We know the y-intercept is (0, 7.3), so we substitute in
1 5 1
the slope-intercept equation. y = − x+ y = − x+2
2 2 2
y = mx + b 1 1
y = 0 · x + 7.3 We see that m1 = − and m2 = − . Since the slopes are
2 2
5
y = 7.3 the same and the y-intercepts, and 2, are different, the
2
27. The equation of the horizontal line through (0, −3) is of lines are parallel.
the form y = b where b is −3. We have y = −3. 41. We solve each equation for y.
The equation of the vertical line through (0, −3) is of the y = 4x − 5 4y = 8 − x
form x = a where a is 0. We have x = 0.
  1
2 y = − x+2
29. The equation of the horizontal line through , −1 is 4
11 1
of the form y = b where b is −1. We have y = −1. We see that m1 = 4 and m2 = − . Since
    4
2 1
The equation of the vertical line through , −1 is of m1 m 2 = 4 − = −1, the lines are perpendicular.
11 4
2 2 2 2
the form x = a where a is . We have x = . 43. y = x + 1; m =
11 11 7 7
31. We have the points (1, 4) and (−2, 13). First we find the 2
The line parallel to the given line will have slope . We
slope. 7
13 − 4 9 2
m= = = −3 use the point-slope equation for a line with slope and
−2 − 1 −3 7
containing the point (3, 5):
We will use the point-slope equation, choosing (1, 4) for
y − y1 = m(x − x1 )
the given point.
2
y − 4 = −3(x − 1) y − 5 = (x − 3)
7
y − 4 = −3x + 3 2 6
y = −3x + 7, or y−5 = x−
7 7
h(x) = −3x + 7 2 29
y = x+ Slope-intercept form
Then h(2) = −3 · 2 + 7 = −6 + 7 = 1. 7 7
The slope of the line perpendicular to the given line is the
33. We have the points (5, 1) and (−5, −3). First we find the 2 7
opposite of the reciprocal of , or − . We use the point-
slope. 7 2
−3 − 1 −4 7
m= = =
2 slope equation for a line with slope − and containing the
−5 − 5 −10 5 2
point (3, 5):
We will use the slope-intercept equation, choosing (5, 1)
y − y1 = m(x − x1 )
for the given point.
7
y = mx + b y − 5 = − (x − 3)
2
2
1 = ·5+b 7 21
5 y−5 = − x+
2 2
1 = 2+b
7 31
−1 = b y = − x+ Slope-intercept form
2 2
2
Then we have f (x) = x − 1. 45. y = −0.3x + 4.3; m = −0.3
5
Now we find f (0). The line parallel to the given line will have slope −0.3. We
2 use the point-slope equation for a line with slope −0.3 and
f (0) = · 0 − 1 = −1. containing the point (−7, 0):
5

Copyright © 2013 Pearson Education, Inc.


Exercise Set 1.4 43

y − y1 = m(x − x1 ) 55. x = −1 and x = 1 are both vertical lines, so it is false that


y − 0 = −0.3(x − (−7)) they are perpendicular.
y = −0.3x − 2.1 Slope-intercept form 57. No. The data points fall faster from 0 to 2 than after 2
The slope of the line perpendicular to the given line is the (that is, the rate of change is not constant), so they cannot
1 10 be modeled by a linear function.
opposite of the reciprocal of −0.3, or = .
0.3 3
10 59. Yes. The rate of change seems to be constant, so the data
We use the point-slope equation for a line with slope points might be modeled by a linear function.
3
and containing the point (−7, 0):
61. a) Answers may vary depending on the data points
y − y1 = m(x − x1 ) used. We will use (2, 785) and (8, 1858).
y−0 =
10
(x − (−7)) 1858 − 785 1073
m= = ≈ 178.83
3 8−2 6
10 70 We will use the point-slope equation, letting
y= x+ Slope-intercept form
3 3 (x1 , y1 ) = (2, 785).
47. 3x + 4y = 5 y − 785 = 178.83(x − 2)
4y = −3x + 5 y − 785 = 178.83x − 357.66
3 5 3 y = 178.83x + 427.34,
y = − x+ ; m=−
4 4 4 where x is the number of years after 2001 and y is
3
The line parallel to the given line will have slope − . We in millions.
4
3 b) In 2013, x = 2013 − 2001 = 12.
use the point-slope equation for a line with slope − and y = 178.83(12) + 427.34 ≈ 2573
4
containing the point (3, −2): We estimate the number of world Internet users in
y − y1 = m(x − x1 ) 2013 to be about 2573 million.
3
y − (−2) = − (x − 3) In 2018, x = 2018 − 2001 = 17.
4
y = 178.83(17) + 427.34 ≈ 3467
3 9
y+2 = − x+ We estimate the number of world Internet users in
4 4
2018 to be about 3467 million.
3 1
y = − x+ Slope-intercept form
4 4 63. Answers may vary depending on the data points used. We
The slope of the line perpendicular to the given line is the will use (2, 16.3) and (5, 19.0).
3 4 19.0 − 16.3
opposite of the reciprocal of − , or . We use the point- m= =
2.7
= 0.9
4 3 5−2 3
4
slope equation for a line with slope and containing the We will use the slope-intercept equation with (5, 19.0).
3
point (3, −2): 19.0 = 0.9(5) + b
y − y1 = m(x − x1 ) 19.0 = 4.5 + b
4
y − (−2) = (x − 3) 14.5 = b
3
We have y = 0.9x + 14.5 where x is the number of years
4
y+2 = x−4 after 2005 and y is in billions of dollars.
3
In 2015, x = 2015 − 2005 = 10. Then we have
4
y = x − 6 Slope-intercept form y = 0.9(10) + 14.5 = 23.5.
3
49. x = −1 is the equation of a vertical line. The line parallel We predict the net sales will be $23.5 billion in 2015.
to the given line is a vertical line containing the point 65. Answers may vary depending on the data points used. We
(3, −3), or x = 3. will use (3, 24.9) and (15, 56.9).
The line perpendicular to the given line is a horizontal line 56.9 − 24.9 32
containing the point (3, −3), or y = −3. m= = ≈ 2.67
15 − 3 12
51. x = −3 is a vertical line and y = 5 is a horizontal line, so We will use the point-slope equation with (3, 24.9).
it is true that the lines are perpendicular. y − 24.9 = 2.67(x − 3)
2 y − 24.9 = 2.67x − 8.01
53. The lines have the same slope, , and different y-
5 y = 2.67x + 16.89
intercepts, (0, 4) and (0, −4), so it is true that the lines are
parallel. We have y = 2.67x + 16.89 where x is the number of years
after 1991 and y is in billions of dollars.

Copyright © 2013 Pearson Education, Inc.


44 Chapter 1: Graphs, Functions, and Models

In 2005, x = 2005 − 1991 = 14. We have The slopes must be equal in order for the lines to be par-
y = 2.67(14) + 16.89 ≈ $54.27 billion allel:
8−k 9
In 2015, x = 2015 − 1991 = 24. We have 7
=
4
y = 2.67(24) + 16.89 ≈ $80.97 billion 32 − 4k = 63 Multiplying by 28
67. a) Using the linear regression feature on a graphing −4k = 31
calculator, we have y = 171.2606061x+430.5272727, 31
where x is the number of years after 2001 and y is k = − , or − 7.75
4
in millions. 79. The slope of the line containing (−1, 3) and (2, 9) is
b) In 2013, x = 2013 − 2001 = 12. We have 9−3 6
= = 2.
y = 171.2606061(12) + 430.5272727 ≈ 2486 million 2 − (−1) 3
This is 87 million less than the value found in Ex- 1
Then the slope of the desired line is − . We find the
ercise 61. 2
equation of that line:
c) r ≈ 0.9890; the line fits the data fairly well. 1
y − 5 = − (x − 4)
2
69. a) Using the linear regression feature on a graphing
calculator, we have y = 2.628571429x+17.41428571, 1
y−5 = − x+2
where x is the number of years after 1991 and y is 2
in billions of dollars. 1
y = − x+7
b) In 2015, x = 2015 − 1991 = 24. We have 2
y = 2.628571429(24) + 17.41428571 ≈ $80.5 billion.
This value is $0.47 billion less than the value found Exercise Set 1.5
in Exercise 65.
1. 4x + 5 = 21
c) r ≈ 0.9950; the line fits the data well. 4x = 16 Subtracting 5 on both sides
71. a) Using the linear regression feature on a graphing x = 4 Dividing by 4 on both sides
calculator, we get M = 0.2H + 156. The solution is 4.
b) For H = 40: M = 0.2(40) + 156 = 164 beats per 2 2
minute 3. 23 − x = − x + 23
5 5
For H = 65: M = 0.2(65) + 156 = 169 beats per 2
23 = 23 Adding x on both sides
minute 5
We get an equation that is true for any value of x, so the
For H = 76: M = 0.2(76) + 156 ≈ 171 beats per
solution set is the set of real numbers,
minute
{x|x is a real number}, or (−∞, ∞).
For H = 84: M = 0.2(84) + 156 ≈ 173 beats per
minute 5. 4x + 3 = 0
c) r = 1; all the data points are on the regression line 4x = −3 Subtracting 3 on both sides
so it should be a good predictor. 3
x=− Dividing by 4 on both sides
y2 − y1 4
73. m= 3
x2 − x1 The solution is − .
4
−1 − (−8) −1 + 8
= = 7. 3 − x = 12
−5 − 2 −7
7 −x = 9 Subtracting 3 on both sides
= = −1 x = −9 Multiplying (or dividing) by −1
−7
on both sides
75. (x − h) + (y − k) = r
2 2 2
 2 The solution is −9.
9
[x − (−7)]2 + [y − (−1)]2 = 1 3
5 9. 3− x = The LCD is 4.
4 2
2 2 81  
(x + 7) + (y + 1) = 1 3
25 4 3− x = 4· Multiplying by the LCD
4 2 to clear fractions
77. The slope of the line containing (−3, k) and (4, 8) is
12 − x = 6
8−k 8−k
= . −x = −6 Subtracting 12 on both sides
4 − (−3) 7
x=6 Multiplying (or dividing) by −1
The slope of the line containing (5, 3) and (1, −6) is
on both sides
−6 − 3 −9 9
= = . The solution is 6.
1−5 −4 4

Copyright © 2013 Pearson Education, Inc.


Exercise Set 1.5 45

2 9
11. − 4x = −4x + 27. 7(3x + 6) = 11 − (x + 2)
11 11
2 9 21x + 42 = 11 − x − 2 Using the distributive
= Adding 4x on both sides property
11 11
We get a false equation. Thus, the original equation has 21x + 42 = 9 − x Collecting like terms
no solution. 21x + x = 9 − 42 Adding x and subtract-
ing 42 on both sides
13. 8 = 5x − 3 22x = −33 Collecting like terms
11 = 5x Adding 3 on both sides 33
x=− Dividing by 22 on both
11 22
=x Dividing by 5 on both sides sides
5 3
11 x=− Simplifying
The solution is . 2
5 3
2 1 The solution is − .
15. y−2 = The LCD is 15. 2
5 3
  29. 3(x + 1) = 5 − 2(3x + 4)
2 1
15 y − 2 = 15 · Multiplying by the LCD 3x + 3 = 5 − 6x − 8 Removing parentheses
5 3
to clear fractions 3x + 3 = −6x − 3 Collecting like terms
6y − 30 = 5
9x + 3 = −3 Adding 6x
6y = 35 Adding 30 on both sides
9x = −6 Subtracting 3
35
y= Dividing by 6 on both sides 2
6 x=− Dividing by 9
3
35
The solution is . 2
6 The solution is − .
3
17. y + 1 = 2y − 7
31. 2(x − 4) = 3 − 5(2x + 1)
1 = y−7 Subtracting y on both sides
2x − 8 = 3 − 10x − 5 Using the distributive
8=y Adding 7 on both sides property
The solution is 8. 2x − 8 = −10x − 2 Collecting like terms
19. 2x + 7 = x + 3 12x = 6 Adding 10x and 8 on both sides
x+7 = 3 Subtracting x on both sides 1
x= Dividing by 12 on both sides
2
x = −4 Subtracting 7 on both sides 1
The solution is −4. The solution is .
2
21. 3x − 5 = 2x + 1 33. Familiarize. Let s = the number of foreign students from
x−5 = 1 Subtracting 2x on both sides India. Then 22% more than this number is s + 0.22s, or
1.22s.
x=6 Adding 5 on both sides
Translate.
The solution is 6.
Number of 22% more than
23. 4x − 5 = 7x − 2 students is number of students
−5 = 3x − 2 Subtracting 4x on both sides from China from India
 
   

−3 = 3x Adding 2 on both sides
−1 = x Dividing by 3 on both sides 128, 000 = 1.22s
The solution is −1. Carry out. We solve the equation.
128, 000 = 1.22s
25. 5x − 2 + 3x = 2x + 6 − 4x
105, 000 ≈ s
8x − 2 = 6 − 2x Collecting like terms
Check. 105, 000 + 0.22(105, 000) = 128, 100 ≈ 128, 000.
8x + 2x = 6 + 2 Adding 2x and 2 on The answer checks. (Remember that we rounded the value
both sides of s.)
10x = 8 Collecting like terms
8 State. There were about 105,000 students from India in
x= Dividing by 10 on both the U.S. in the 2009-2010 school year.
10
sides
4
x= Simplifying
5
4
The solution is .
5

Copyright © 2013 Pearson Education, Inc.


46 Chapter 1: Graphs, Functions, and Models

35. Familiarize. Let x = the number of metric tons of olive Carry out. We solve the equation.
oil consumed in the U.S. in 2009/2010. 2w + 2130 + 2w − 5 + w = 11, 150
Translate. 5w + 2125 = 11, 150
Consumption consumption
was 2.5 times plus 60, 000 5w = 9025
in Italy in U.S.
 
     
   w = 1805

If w = 1805, then 2w − 5 = 2 · 1805 − 5 = 3605 and
710, 000 = 2.5 · x + 60, 000
2w + 2130 = 2 · 1805 + 2130 = 5740.
Carry out. We solve the equation.
Check. 1805+3605+5740 = 11, 150, so the answer checks.
710, 000 = 2.5x + 60, 000
State. The Tundra weighs 5740 lb, the Mustang weighs
650, 000 = 2.5x 3605 lb, and the Smart For Two weighs 1805 lb.
260, 000 = x
41. Familiarize. Let v = the number of ABC viewers, in
Check. 2.5(260, 000) = 650, 000 and 650, 000 + 60, 000 = millions. Then v + 1.7 = the number of CBS viewers and
710, 000. The answer checks. v − 1.7 = the number of NBC viewers.
State. 260,000 metric tons of olive oil were consumed in Translate.
the U.S. in 2009 - 2010. ABC CBS NBC total
plus plus is
37. Familiarize. Let d = the average depth of the Atlantic viewers viewers viewers viewers.
             
4
Ocean, in feet. Then d − 272 = the average depth of the
5 v + (v + 1.7) + (v − 1.7) = 29.1
Indian Ocean.
Translate. Carry out.
Average Average Average v + (v + 1.7) + (v − 1.7) = 29.1
depth of depth of depth of 3v = 29.1
is plus less 8890 ft
Pacific Atlantic Indian
v = 9.7
Ocean Ocean Ocean
            Then v+1.7 = 9.7+1.7 = 11.4 and v−1.7 = 9.7−1.7 = 8.0.

4 Check. 9.7 + 11.4 + 8.0 = 29.1, so the answer checks.
14, 040 = d + d − 272 − 8890 State. ABC had 9.7 million viewers, CBS had 11.4 million
5
Carry out. We solve the equation. viewers, and NBC had 8.0 million viewers.
4
14, 040 = d + d − 272 − 8890 43. Familiarize. Let P = the amount Tamisha borrowed. We
5 will use the formula I = P rt to find the interest owed. For
9 r = 5%, or 0.05, and t = 1, we have I = P (0.05)(1), or
14, 040 = d − 9162
5 0.05P .
9 Translate.
23, 202 = d
5 Amount borrowed plus interest is $1365.
 
    
5
· 23, 202 = d
9 P + 0.05P = 1365
12, 890 = d
Carry out. We solve the equation.
If d = 12, 890, then the average depth of the Indian Ocean
4 P + 0.05P = 1365
is · 12, 890 − 272 = 10, 040.
5 1.05P = 1365 Adding
Check. 12, 890 + 10, 040 − 8890 = 14, 040, so the answer P = 1300 Dividing by 1.05
checks.
Check. The interest due on a loan of $1300 for 1 year at
State. The average depth of the Indian Ocean is 10,040 ft. a rate of 5% is $1300(0.05)(1), or $65, and $1300 + $65 =
$1365. The answer checks.
39. Familiarize. Let w = the weight of a Smart for Two car,
in pounds. Then 2w − 5 = the weight of a Mustang and State. Tamisha borrowed $1300.
2w − 5 + 2135, or 2w + 2130 = the weight of a Tundra.
45. Familiarize. Let s = Ryan’s sales for the month. Then
Translate. his commission is 8% of s, or 0.08s.
Weight weight weight
total Translate.
of plus of plus of is
weight Base salary plus commission is total pay.
Tundra Mustang Smart Car    
      
    
   
  

1500 + 0.08s = 2284
2w + 2130 + 2w − 5 + w = 11, 150

Copyright © 2013 Pearson Education, Inc.


Exercise Set 1.5 47

Carry out. We solve the equation. State. The measure of angles A, B, and C are 26◦ , 130◦ ,
1500 + 0.08s = 2284 and 24◦ , respectively.
0.08s = 784 Subtracting 1500 51. Familiarize. Using the labels on the drawing in the text,
s = 9800 we let w = the width of the test plot and w + 25 = the
length, in meters. Recall that for a rectangle, Perime-
Check. 8% of $9800, or 0.08($9800), is $784 and $1500 +
ter = 2 · length + 2 · width.
$784 = $2284. The answer checks.
Translate.
State. Ryan’s sales for the month were $9800.
Perimeter
  = 2 · length + 2 · width
47. Familiarize. Let w = Soledad’s regular hourly wage. She       


earned 40w for working the first 40 hr. She worked 48−40,
322 = 2(w + 25) + 2·w
or 8 hr, of overtime. She earned 8(1.5w) for working 8 hr
of overtime. Carry out. We solve the equation.
Translate. The total earned was $442, so we write an 322 = 2(w + 25) + 2 · w
equation. 322 = 2w + 50 + 2w
40w + 8(1.5w) = 442 322 = 4w + 50
Carry out. We solve the equation. 272 = 4w
40w + 8(1.5)w = 442 68 = w
40w + 12w = 442 When w = 68, then w + 25 = 68 + 25 = 93.
52w = 442 Check. The length is 25 m more than the width: 93 =
w = 8.5 68 + 25. The perimeter is 2 · 93 + 2 · 68, or 186 + 136, or
322 m. The answer checks.
Check. 40($8.50) + 8[1.5($8.50)] = $340 + $102 = $442,
so the answer checks. State. The length is 93 m; the width is 68 m.
State. Soledad’s regular hourly wage is $8.50. 53. Familiarize. Let l = the length of the soccer field and
l − 35 = the width, in yards.
49. Familiarize. We make a drawing.
Translate. We use the formula for the perimeter of a
B rectangle. We substitute 330 for P and l − 35 for w.
❝❝
✔5x P = 2l + 2w
✔ ❝ 330 = 2l + 2(l − 35)
✔ ❝
✔ ❝ Carry out. We solve the equation.


✔x x − 2❝❝ 330 = 2l + 2(l − 35)
A C 330 = 2l + 2l − 70
We let x = the measure of angle A. Then 5x = the measure 330 = 4l − 70
of angle B, and x − 2 = the measure of angle C. The sum
400 = 4l
of the angle measures is 180◦ .
100 = l
Translate.
Measure Measure Measure If l = 100, then l − 35 = 100 − 35 = 65.
of + of + of = 180. Check. The width, 65 yd, is 35 yd less than the length,
angle A angle B angle C 100 yd. Also, the perimeter is
         
2 · 100 yd + 2 · 65 yd = 200 yd + 130 yd = 330 yd.
x + 5x + x−2 = 180 The answer checks.
Carry out. We solve the equation. State. The length of the field is 100 yd, and the width is
x + 5x + x − 2 = 180 65 yd.
7x − 2 = 180 55. Familiarize. Let w = the number of pounds of Lily’s
7x = 182 body weight that is water.
x = 26 Translate.
If x = 26, then 5x = 5 · 26, or 130, and x − 2 = 26 − 2, or 55% of body weight is water.
 
24. ↓ ↓ ↓ ↓ ↓
Check. The measure of angle B, 130◦ , is five times the 0.55 × 135 = w
measure of angle A, 26◦ . The measure of angle C, 24◦ , is Carry out. We solve the equation.
2◦ less than the measure of angle A, 26◦ . The sum of the
0.55 × 135 = w
angle measures is 26◦ + 130◦ + 24◦ , or 180◦ . The answer
checks. 74.25 = w

Copyright © 2013 Pearson Education, Inc.


48 Chapter 1: Graphs, Functions, and Models

Check. Since 55% of 135 is 74.25, the answer checks. 61. Familiarize. Let t = the number of hours it will take the
State. 74.25 lb of Lily’s body weight is water. plane to travel 1050 mi into the wind. The speed into the
headwind is 450 − 30, or 420 mph.
57. Familiarize. We make a drawing. Let t = the number Translate. We use the formula d = rt.
of hours the passenger train travels before it overtakes the
freight train. Then t + 1 = the number of hours the freight 1050 = 420 · t
train travels before it is overtaken by the passenger train. Carry out. We solve the equation.
Also let d = the distance the trains travel. 1050 = 420 · t
2.5 = t
Freight train
t + 1 hr d Check. At a rate of 420 mph, in 2.5 hr the plane travels
q 60 mph ✲
420(2.5), or 1050 mi. The answer checks.
Passenger train State. It will take the plane 2.5 hr to travel 1050 mi into
q 80 mph t hr d ✲ the wind.

We can also organize the information in a table. 63. Familiarize. Let x = the amount invested at 3% interest.
d = r · t Then 5000 − x = the amount invested at 4%. We organize
the information in a table, keeping in mind the simple
Distance Rate Time interest formula, I = P rt.
Amount Interest Amount
Freight invested rate Time of interest
d 60 t+1
train 3% 3%, or x(0.03)(1),
invest- x 1 yr
Passenger ment 0.03 or 0.03x
d 80 t
train 4% 4%, or (5000−x)(0.04)(1),
invest- 5000−x 1 yr
Translate. Using the formula d = rt in each row of the ment 0.04 or 0.04(5000−x)
table, we get two equations. Total 5000 176
d = 60(t + 1) and d = 80t. Translate.
Since the distances are the same, we have the equation Interest on interest on
plus is $176.
60(t + 1) = 80t. 3% investment 4% investment
 
   
  
Carry out. We solve the equation.
60(t + 1) = 80t 0.03x + 0.04(5000 − x) = 176
60t + 60 = 80t Carry out. We solve the equation.
60 = 20t 0.03x + 0.04(5000 − x) = 176

3=t 0.03x + 200 − 0.04x = 176

When t = 3, then t + 1 = 3 + 1 = 4. −0.01x + 200 = 176

Check. In 4 hr the freight train travels 60 · 4, or 240 mi. −0.01x = −24


In 3 hr the passenger train travels 80 · 3, or 240 mi. Since x = 2400
the distances are the same, the answer checks. If x = 2400, then 5000 − x = 5000 − 2400 = 2600.
State. It will take the passenger train 3 hr to overtake the Check. The interest on $2400 at 3% for 1 yr is
freight train. $2400(0.03)(1) = $72. The interest on $2600 at 4% for
1 yr is $2600(0.04)(1) = $104. Since $72 + $104 = $176,
59. Familiarize. Let t = the number of hours it takes the
the answer checks.
kayak to travel 36 mi upstream. The kayak travels up-
stream at a rate of 12 − 4, or 8 mph. State. $2400 was invested at 3%, and $2600 was invested
at 4%.
Translate. We use the formula d = rt.
36 = 8 · t 65. Familiarize. Let a = the number of unique ama-
zon.com visitors. Then a + 42, 826, 225 = the number of
Carry out. We solve the equation.
youtube.com visitors.
36 = 8 · t
Translate.
4.5 = t Number of number of
Check. At a rate of 8 mph, in 4.5 hr the kayak travels facebook.com is amazon.com plus
8(4.5), or 36 mi. The answer checks. visitors visitors
 
  
 
State. It takes the kayak 4.5 hr to travel 36 mi upstream.
134, 078, 221 = a +

Copyright © 2013 Pearson Education, Inc.


Exercise Set 1.5 49

number of 71. x+5 = 0 Setting f (x) = 0


youtube.com less 51, 604, 956
x+5−5 = 0−5 Subtracting 5 on both sides
visitors
 
    x = −5
  
The zero of the function is −5.
a + 42, 826, 225 − 51, 604, 956
Carry out. We solve the equation. 73. −2x + 11 = 0 Setting f (x) = 0
134, 078, 221 = a + a + 42, 826, 225 − 51, 604, 956 −2x + 11 − 11 = 0 − 11 Subtracting 11 on both
sides
134, 078, 221 = 2a − 8, 778, 731
−2x = −11
142, 856, 952 = 2a 11
x= Dividing by −2 on both sides
71, 428, 476 = a 2
Then a + 42, 826, 225 = 71, 428, 476 + 42, 826, 225 =
11
114, 254, 701. The zero of the function is .
2
Check. The total number of amazon and youtube vis-
75. 16 − x = 0 Setting f (x) = 0
itors was 71, 428, 476 + 114, 254, 701, or 185,683,177 and
185, 683, 177 − 51, 604, 956 = 134, 078, 221. The answer 16 − x + x = 0 + x Adding x on both sides
checks. 16 = x
State. In February, 2011, amazon.com had 71,428,476 The zero of the function is 16.
unique visitors, and youtube.com had 114,254,701 unique
visitors. 77. x + 12 = 0 Setting f (x) = 0
x + 12 − 12 = 0 − 12 Subtracting 12 on
67. Familiarize. Let l = the elevation of Lucas Oil Stadium, both sides
in feet. x = −12
Translate. The zero of the function is −12.
Elevation of
Elevation of more 79. −x + 6 = 0 Setting f (x) = 0
is 247
 ft than 7 times Lucas Oil
Invesco Field
      Stadium −x + 6 + x = 0 + x Adding x on both sides
       
 
       6=x
5210 = 247 + 7 · l The zero of the function is 6.
Carry out. We solve the equation. 81. 20 − x = 0 Setting f (x) = 0
5210 = 247 + 7l
20 − x + x = 0 + x Adding x on both sides
4963 = 7l
20 = x
709 = l The zero of the function is 20.
Check. 247 more than 7 times 709 is 247 + 7 · 709 =
2
247 + 4963 = 5210. The answer checks. 83. x − 10 = 0 Setting f (x) = 0
5
State. The elevation of Lucas Oil Stadium is 709 ft.
2
x = 10 Adding 10 on both sides
69. Familiarize. Let n = the number of people in the ur- 5
ban population of the Dominican Republic for whom bot- 5 2 5 5
tled water was the primary source of drinking water in · x = · 10 Multiplying by on both
2 5 2 2
2009. We are told that the urban population is 66.8% of sides
9,650,054, or 0.668(9, 650, 054). x = 25
Translate. The zero of the function is 25.
Number for whom
urban 85. −x + 15 = 0 Setting f (x) = 0
bottled water is is 67% of
population 15 = x Adding x on both sides
primary source   
 
      The zero of the function is 15.
    
n = 0.67 · 0.668(9, 650, 054) 87. a) The graph crosses the x-axis at (4, 0). This is the
Carry out. We carry out the calculation. x-intercept.
n = 0.67 · 0.668(9, 650, 054) ≈ 4, 318, 978 b) The zero of the function is the first coordinate of
the x-intercept. It is 4.
Check. We can repeat the calculation. The answer
checks. 89. a) The graph crosses the x-axis at (−2, 0). This is the
State. Bottled water was the primary source of drinking x-intercept.
water for about 4,318,978 people in the urban population b) The zero of the function is the first coordinate of
of the Dominican Republic in 2009. the x-intercept. It is −2.

Copyright © 2013 Pearson Education, Inc.


50 Chapter 1: Graphs, Functions, and Models

91. a) The graph crosses the x-axis at (−4, 0). This is the 105. The size of the cup was reduced 8 oz − 6 oz, or 2 oz, and
x-intercept. 2 oz
= 0.25, so the size was reduced 25%. The price per
b) The zero of the function is the first coordinate of 8 oz
89/
c
the x-intercept. It is −4. ounce of the 8 oz cup was , or 11.125/c/oz. The price
8 oz
93. First find the slope of the given line. 71/c
per ounce of the 6 oz cup is c/oz. Since the
, or 11.83/
3x + 4y = 7 6 oz
price per ounce was not reduced, it is clear that the price
4y = −3x + 7 per ounce was not reduced by the same percent as the size
3 7 of the cup. The price was increased by 11.83 − 11.125/ c, or
y = − x+ 0.7083/
c
4 4 0.7083/c per ounce. This is an increase of ≈ 0.064,
3 11.125/
c
The slope is − . Now write a slope-intercept equation of or about 6.4% per ounce.
4
3
the line containing (−1, 4) with slope − . 107. We use a proportion to determine the number of calories
4
3 c burned running for 75 minutes, or 1.25 hr.
y − 4 = − [x − (−1)] 720 c
4 =
3 1 1.25
y − 4 = − (x + 1) 720(1.25) = c
4
3 3 900 = c
y−4 = − x−
4 4 Next we use a proportion to determine how long the person
3 13 would have to walk to use 900 calories. Let t represent this
y = − x+
4 4 time, in hours. We express 90 min as 1.5 hr.
 1.5 t
95. d = (x2 − x1 )2 + (y2 − y1 )2 =
 480 900
= (−10 − 2)2 + (−3 − 2)2
√ √ 900(1.5)
=t
= 144 + 25 = 169 = 13 480
2.8125 = t
x Then, at a rate of 4 mph, the person would have to walk
97. f (x) =
x−3 4(2.8125), or 11.25 mi.
−3 −3 1
f (−3) = = =
−3 − 3 −6 2 Exercise Set 1.6
0 0
f (0) = = =0
0−3 −3 1. 4x − 3 > 2x + 7
3 3 2x − 3 > 7 Subtracting 2x
f (3) = =
3−3 0
2x > 10 Adding 3
Since division by 0 is not defined, f (3) does not exist.
x>5 Dividing by 2
3 3 The solution set is {x|x > 5}, or (5, ∞). The graph is
99. f (x) = 7 − x = − x + 7
2 2 shown below.
The function can be written in the form y = mx + b, so it
is a linear function. 0 5

101. f (x) = x2 +1 cannot be written in the form f (x) = mx+b,


3. x + 6 < 5x − 6
so it is not a linear function.
6 + 6 < 5x − x Subtracting x and adding 6
103. 2x − {x − [3x − (6x + 5)]} = 4x − 1 on both sides
2x − {x − [3x − 6x − 5]} = 4x − 1 12 < 4x
2x − {x − [−3x − 5]} = 4x − 1 12
<x Dividing by 4 on both sides
4
2x − {x + 3x + 5} = 4x − 1
3<x
2x − {4x + 5} = 4x − 1
This inequality could also be solved as follows:
2x − 4x − 5 = 4x − 1
x + 6 < 5x − 6
−2x − 5 = 4x − 1
x − 5x < −6 − 6 Subtracting 5x and 6 on
−6x − 5 = −1 both sides
−6x = 4 −4x < −12
2 −12
x=− x> Dividing by −4 on both sides and
3 −4
2 reversing the inequality symbol
The solution is − . x>3
3

Copyright © 2013 Pearson Education, Inc.


Exercise Set 1.6 51

3 5 2
The solution set is {x|x > 3}, or (3, ∞). The graph is 13. − x ≥− + x
shown below. 4 8 3
5 3 2
≥ x+ x
0 3 8 4 3
5 9 8
5. 4 − 2x ≤ 2x + 16 ≥ x+ x
8 12 12
4 − 4x ≤ 16 Subtracting 2x 5 17
−4x ≤ 12 Subtracting 4 ≥ x
8 12
x ≥ −3 Dividing by −4 and reversing 12 5 12 17
the inequality symbol · ≥ · x
17 8 17 12
The solution set is {x|x ≥ −3}, or [−3, ∞). The graph is 15
shown below. ≥x
34

 
15 15
⫺3 0 The solution set is x x ≤ , or − ∞, . The
34 34
14 − 5y ≤ 8y − 8 graph is shown below.
7.
14 + 8 ≤ 8y + 5y 15


34
22 ≤ 13y
0
22
≤y
13
3
This inequality could also be solved as follows: 

14

14 − 5y ≤ 8y − 8 0

−5y − 8y ≤ −8 − 14
15. 4x(x − 2) < 2(2x − 1)(x − 3)
−13y ≤ −22
22 4x(x − 2) < 2(2x2 − 7x + 3)
y≥ Dividing by −13 on 4x2 − 8x < 4x2 − 14x + 6
13 both sides and reversing
the inequality symbol −8x < −14x + 6

−8x + 14x < 6
22 22
The solution set is y y ≥ , or , ∞ . The graph
13 13 6x < 6
is shown below. 6
x<
6
0 22


13 x<1
The solution set is {x|x < 1}, or (−∞, 1). The graph is
9. 7x − 7 > 5x + 5
shown below.
2x − 7 > 5 Subtracting 5x
2x > 12 Adding 7 0 1

x>6 Dividing by 2
17. The radicand must be nonnegative, so we solve the in-
The solution set is {x|x > 6}, or (6, ∞). The graph is equality x − 7 ≥ 0.
shown below.
x−7 ≥ 0
0 6 x≥7

3x − 3 + 2x ≥ 1 − 7x − 9 The domain is {x|x ≥ 7}, or [7, ∞).


11.
5x − 3 ≥ −7x − 8 Collecting like terms 19. The radicand must be nonnegative, so we solve the in-
5x + 7x ≥ −8 + 3 Adding 7x and 3 equality 1 − 5x ≥ 0.
on both sides 1 − 5x ≥ 0
12x ≥ −5
1 ≥ 5x
5
x≥− Dividing by 12 on both sides 1
12 ≥x
5
5

 
5

The solution set is x x ≥ − , or − , ∞ . The 1 1
12 12 The domain is x x ≤ , or − ∞, .
graph is shown below. 5 5

5


12

Copyright © 2013 Pearson Education, Inc.


52 Chapter 1: Graphs, Functions, and Models

21. The radicand must be positive, so we solve the inequality 35. 3x ≤ −6 or x − 1 > 0
4 + x > 0. x ≤ −2 or x>1
4+x > 0
The solution set is (−∞, −2] ∪ (1, ∞). The graph is shown
x > −4 below.
The domain is {x|x > −4}, or (−4, ∞).
2
−2 ≤ x + 1 < 4
0 1
23.
−3 ≤ x < 3 Subtracting 1 37. 2x + 3 ≤ −4 or 2x + 3 ≥ 4
The solution set is [−3, 3). The graph is shown below. 2x ≤ −7 or 2x ≥ 1
7 1
x ≤ − or x≥
2 2
3 0 3
 
7 1
25. 5 ≤ x−3≤7 The solution set is − ∞, − ∪ , ∞ . The graph is
2 2
8 ≤ x ≤ 10 Adding 3 shown below.
The solution set is [8, 10]. The graph is shown below. 7 1

2 
2

0
0 8 10

39. 2x − 20 < −0.8 or 2x − 20 > 0.8


27. −3 ≤ x + 4 ≤ 3
2x < 19.2 or 2x > 20.8
−7 ≤ x ≤ −1 Subtracting 4
x < 9.6 or x > 10.4
The solution set is [−7, −1]. The graph is shown below.
The solution set is (−∞, 9.6) ∪ (10.4, ∞). The graph is
shown below.
7 1 0

29. −2 < 2x + 1 < 5 0 9.6 10.4

−3 < 2x < 4 Adding −1


1 1
3 1 41. x + 14 ≤ − or x + 14 ≥
− <x<2 Multiplying by 4 4
2 2
 57 55
3 x≤− or x≥−
The solution set is − , 2 . The graph is shown below. 4 4
2  
57 55
The solution set is − ∞, − ∪ − ,∞ . The
4 4
3

2
0 2 graph is shown below.

31. −4 ≤ 6 − 2x < 4 57 55 0

4 
4
−10 ≤ −2x < −2 Adding −6
1
5≥x>1 Multiplying by − 43. Familiarize and Translate. Spending is given by the
2
equation y = 31.7x+487. We want to know when spending
or 1<x≤5 will be more than $775 billion, so we have
The solution set is (1, 5]. The graph is shown below. 31.7x + 487 > 775.
Carry out. We solve the inequality.
0 1 5
31.7x + 487 > 775
1 31.7x > 288
33. −5 < (3x + 1) < 7
2 x>9 Rounding
−10 < 3x + 1 < 14 Multiplying by 2
Check. When x = 9, spending is 31.7(9) + 487 =
−11 < 3x < 13 Adding −1 772.3 ≈ 775. (Remember that we rounded the an-
11 13 1 swer.) As a partial check we try a value of x less than
− <x< Multiplying by
3 3 3 9 and a value greater than 9. When x = 8.9, we have

11 13 y = 31.7(8.9) + 487 = 769.13 < 775; when x = 9.1,
The solution set is − , . The graph is shown be- y = 31.7(9.1) + 487 = 775.47 > 775. Since y ≈ 775 when
3 3
low. x = 9 and y > 775 when x = 9.1 > 9, the answer is
probably correct.
11

 0 13


State. Spending will be more than $775 billion more than
3 3
9 yr after 2007, or in years after 2016.

Copyright © 2013 Pearson Education, Inc.


Exercise Set 1.6 53

45. Familiarize. Let t = the number of hours worked. Then −0.015x + 2750 ≥ 2660.
Acme Movers charge 100 + 30t and Hank’s Movers charge Carry out. We solve the inequality.
55t.
−0.015x + 2750 ≥ 2660
Translate.
−0.015x ≥ −90
Hank’s charge is less than Acme’s charge.
 
   
  
  x ≤ 6000
  
55t < 100 + 30t If $6000 is invested at 7%, then 2 · $6000, or $12,000 is
invested at 4%.
Carry out. We solve the inequality.
Check. If $6000 is invested at 7% and $12,000 is invested
55t < 100 + 30t at 4%, the amount invested at 5.5% is $50, 000 − $6000 −
25t < 100 $12, 000, or $32,000. The interest earned is 0.07 · $6000 +
t<4 0.04 · $12, 000 + 0.055 · $32, 000, or $420 + $480 + $1760, or
$2660.
Check. When t = 4, Hank’s Movers charge 55 · 4, or $220
and Acme Movers charge 100 + 30 · 4 = 100 + 120 = $220, As a partial check, we determine the total interest when
so the charges are the same. As a partial check, we find more than $12,000 is invested at 4%. Suppose $12,001 is
the charges for a value of t < 4. When t = 3.5, Hank’s invested at 4%. Then $12, 001/2, or $6000.50 is invested
Movers charge 55(3.5) = $192.50 and Acme Movers charge at 7% and $50, 000 − $6000.50 − $12, 001, or $31,998.50, is
100 + 30(3.5) = 100 + 105 = $205. Since Hank’s charge is invested at 5.5%. The interest earned is 0.07($6000.50) +
less than Acme’s, the answer is probably correct. 0.04($12, 001) + 0.055($31, 998.50), or $2659.99. Since this
is less than $2660, the answer is probably correct.
State. For times less than 4 hr it costs less to hire Hank’s
Movers. State. The most that can be invested at 4% is $12,000.

47. Familiarize. Let x = the amount invested at 4%. Then 51. Familiarize. Let s = the monthly sales. Then the amount
7500 − x = the amount invested at 5%. Using the simple- of sales in excess of $8000 is s − 8000.
interest formula, I = P rt, we see that in one year the Translate.
4% investment earns 0.04x and the 5% investment earns Income from is greater income from
0.05(7500 − x). plan B than plan A.
 
   
  
 
Translate.   
Interest at 4% plus interest at 5% is least $325.
 at
 
  
    1200 + 0.15(s − 8000) > 900 + 0.1s
    
Carry out. We solve the inequality.
0.04x + 0.05(7500 − x) ≥ 325
1200 + 0.15(s − 8000) > 900 + 0.1s
Carry out. We solve the inequality.
1200 + 0.15s − 1200 > 900 + 0.1s
0.04x + 0.05(7500 − x) ≥ 325
0.15s > 900 + 0.1s
0.04x + 375 − 0.05x ≥ 325
0.05s > 900
−0.01x + 375 ≥ 325
s > 18, 000
−0.01x ≥ −50
Check. For sales of $18,000 the income from plan A is
x ≤ 5000 $900+0.1($18, 000), or $2700, and the income from plan B
Check. When $5000 is invested at 4%, then $7500−$5000, is 1200 + 0.15(18, 000 − 8000), or $2700 so the incomes are
or $2500, is invested at 5%. In one year the 4% invest- the same. As a partial check we can compare the incomes
ment earns 0.04($5000), or $200, in simple interest and for an amount of sales greater than $18,000. For sales of
the 5% investment earns 0.05($2500), or $125, so the total $18,001, for example, the income from plan A is $900 +
interest is $200 + $125, or $325. As a partial check, we 0.1($18, 001), or $2700.10, and the income from plan B is
determine the total interest when an amount greater than $1200 + 0.15($18, 001 − $8000), or $2700.15. Since plan B
$5000 is invested at 4%. Suppose $5001 is invested at 4%. is better than plan A in this case, the answer is probably
Then $2499 is invested at 5%, and the total interest is correct.
0.04($5001) + 0.05($2499), or $324.99. Since this amount State. Plan B is better than plan A for monthly sales
is less than $325, the answer is probably correct. greater than $18,000.
State. The most that can be invested at 4% is $5000.
53. Function; domain; range; domain; exactly one; range
49. Familiarize and Translate. Let x = the amount in-
vested at 7%. Then 2x = the amount invested at 4%, 55. x-intercept
and 50, 000 − x − 2x, or 50, 000 − 3x = the amount in-
vested at 5.5%. The interest earned is 0.07x + 0.04 · 2x +
0.055(50, 000 − 3x), or 0.07x + 0.08x + 2750 − 0.165x, or
−0.015x + 2750. The foundation wants the interest to be
at least $2660 so we have

Copyright © 2013 Pearson Education, Inc.


54 Chapter 1: Graphs, Functions, and Models

57. 2x ≤ 5 − 7x < 7 + x 9. 2x − 3y = 6
2x ≤ 5 − 7x and 5 − 7x < 7 + x To find the x-intercept we replace y with 0 and solve for
9x ≤ 5 and −8x < 2 x.
5 1 2x − 3 · 0 = 6
x≤ x>−
and
9 4 2x = 6
 
1 5 x=3
The solution set is − , .
4 9 The x-intercept is (3, 0).
59. 3y < 4 − 5y < 5 + 3y To find the y-intercept we replace x with 0 and solve for
0 < 4 − 8y < 5 Subtracting 3y y.
−4 < −8y < 1 Subtracting 4 2 · 0 − 3y = 6
1 1 −3y = 6
>y>− Dividing by −8 and reversing
2 8
the inequality symbols y = −2
 The y-intercept is (0, −2).
1 1
The solution set is − , . We plot the intercepts and draw the line that contains
8 2
them. We could find a third point as a check that the
intercepts were found correctly.
Chapter 1 Review Exercises
y

4
1. First we solve each equation for y.
2
ax + y = c x − by = d (3, 0)
4 2 2 4 x
y = −ax + c −by = −x + d
2 (0, 2)
1 d
y = x− 4
b b
If the lines are perpendicular, the product of their slopes is
1 a a 2x  3y  6
−1, so we have −a · = −1, or − = −1, or = 1. The
b b b
statement is true. 11.
 √ y
3 − (−3) 6
3. f (−3) = = , so −3 is in the domain of 4
−3 −3 2
f (x). Thus, the statement is false. 2 y  3 x  1
5. The statement is true. See page 133 in the text.
4 2 2 4 x
 2
24
7. For 3,
9
: 2x − 9y = −18
4
24
2·3−9· ? −18
9
13.
6 − 24 y

−18 −18 TRUE
 4
y  2  x2
24
3, is a solution. 2
9

For (0, −9): 2x − 9y = −18 4 2 2 4 x


2(0) − 9(−9) ? −18 2
4
0 + 81

81 −18 FALSE
(0, −9) is not a solution. 
15. m = x1 + x2 y1 + y2
,
2 2

3 + (−2) 7 + 4
= ,
2 2

1 11
= ,
2 2

Copyright © 2013 Pearson Education, Inc.


Chapter 1 Review Exercises 55

17. (x − h)2 + (y − k)2 = r2 35. y


 2
3 5 f (x ) = 16 – x 2
(x − 0)2 + [y − (−4)]2 = Substituting 4
2
3
9 2
x2 + (y + 4)2 =
4 1
x
-5 -4 -3 -2 -1 1 2 3 4 5
19. The center is the midpoint of the diameter: -1
 
−3 + 7 5 + 3 4 8 -2
, = , = (2, 4) -3
2 2 2 2 -4

Use the center and either endpoint of the diameter to find -5

the radius. We use the point (7, 3).


 
r = (7 − √ 2)2 + (3 − 4)2 = 52 + (−1)2 = The inputs on the x axis extend from −4 to 4, so the
√ domain is [−4, 4].
25 + 1 = 26
√ The outputs on the y-axis extend from 0 to 4, so the range
The equation of the circle is (x−2)2 + (y−4)2 = ( 26)2 ,
or (x − 2) + (y − 4) = 26.
2 2 is [0, 4].

21. The correspondence is a function because each member 37.


of the domain corresponds to exactly one member of the y
range.
f (x ) = x 3 –7 10
8
23. The relation is a function, because no two ordered pairs
6
have the same first coordinate and different second co- 4
ordinates. The domain is the set of first coordinates: 2

{−2, 0, 1, 2, 7}. The range is the set of second coordinates: –10 –8 –6 –4 –2 2 4 6 8 10 x


{−7, −4, −2, 2, 7}. –2
–4

x−7 –6
25. f (x) = –8
x+5 –10

7−7 0
a) f (7) = = =0
7+5 12 Every point on the x-axis corresponds to a point on the
x+1−7 x−6 graph, so the domain is the set of all real numbers, or
b) f (x + 1) = =
x+1+5 x+6 (−∞, ∞).
−5 − 7 −12
c) f (−5) = = Each point on the y-axis also corresponds to a point on
−5 + 5 0 the graph, so the range is the set of all real numbers, or
Since division by 0 is not defined, f (−5) does not exist. (−∞, ∞).

 1 15 39. a) Yes. Each input is 1 more than the one that pre-
1 − −7 −
d) f − = 2 = 2 = − 15 · 2 = cedes it.
2 1 9 2 9
− +5 b) No. The change in the output varies.
2 2
3/ · 5 · /2 5 c) No. Constant changes in inputs do not result in
− =− constant changes in outputs.
2/ · /3 · 3 3
27. This is not the graph of a function, because we can find a y 2 − y1
41. m =
vertical line that crosses the graph more than once. x2 − x1
−6 − (−11) 5
29. This is not the graph of a function, because we can find a = =
5−2 3
vertical line that crosses the graph more than once.
y 2 − y1
43. m =
31. We can substitute any real number for x. Thus, the do- x2 − x1
main is the set of all real numbers, or (−∞, ∞). 0−3 −3
= =
1 1 0
33. Find the inputs that make the denominator zero: −
2 2
x2 − 6x + 5 = 0 The slope is not defined.
(x − 1)(x − 5) = 0
7
x − 1 = 0 or x − 5 = 0 45. y = − x−6
11
x = 1 or x=5 7
The equation is in the form y = mx + b. The slope is − ,
The domain is {x|x = 1 and x = 5}, or 11
and the y-intercept is (0, −6).
(−∞, 1) ∪ (1, 5) ∪ (5, ∞).

Copyright © 2013 Pearson Education, Inc.


56 Chapter 1: Graphs, Functions, and Models

1 We use the slope-intercept equation to find the y-intercept.


47. Graph y = − x + 3.
4 y = mx + b
Plot the y-intercept, (0, 3). We can think of the slope as
−1 3
. Start at (0, 3) and find another point by moving down −1 = · 1 + b
4 2
1 unit and right 4 units. We have the point (4, 2). 3
−1 = + b
1 2
We could also think of the slope as . Then we can start
−4 5
− =b
at (0, 3) and find another point by moving up 1 unit and 2
left 4 units. We have the point (−4, 4). Connect the three 3 5
points and draw the graph. Then the equation of the desired line is y = x − .
2 2
y
61. 4y − 5 = 1
4y = 6
4
3
2 y=
2
4 2 2 4
3
x The solution is
.
2 2
1
4 y  x
4 3 63. 5(3x + 1) = 2(x − 4)
15x + 5 = 2x − 8

49. a) T (d) = 10d + 20 13x = −13

T (5) = 10(5) + 20 = 70◦ C x = −1


The solution is −1.
T (20) = 10(20) + 20 = 220◦ C
T (1000) = 10(1000) + 20 = 10, 020◦ C 3 3
65. y−2= The LCD is 40
5 8
b) 5600 km is the maximum depth. Domain: [0, 5600]. 
3 3
40 y−2 = 40 · Multiplying to clear fractions
51. y − y1 = m(x − x1 ) 5 8
y − (−1) = 3(x − (−2)) 24y − 80 = 15
y + 1 = 3(x + 2) 24y = 95
y + 1 = 3x + 6 95
y=
y = 3x + 5 24
 95
2 2 The solution is .
53. The horizontal line that passes through − 4, is unit 24
5 5 67. x − 13 = −13 + x
2
above the x-axis. An equation of the line is y = . −13 = −13 Subtracting x
5
 We have an equation that is true for any real num-
2
The vertical line that passes through − 4, is 4 units ber, so the solution set is the set of all real numbers,
5
to the left of the y-axis. An equation of the line is x = −4. {x|x is a real number}, or (−∞, ∞).

55. 3x − 2y = 8 6x − 4y = 2 69. Familiarize. Let a = the amount originally invested. Us-


ing the simple interest formula, I = P rt, we see that the
3 3 1
y = x−4 y = x− interest earned at 5.2% interest for 1 year is a(0.052) · 1 =
2 2 2 0.052a.
3
The lines have the same slope, , and different Translate.
 2
1 Amount interest
y-intercepts, (0, −4) and 0, − , so they are parallel. invested
plus
earned
is $2419.60
2
         
3 3 2     
57. The slope of y = x + 7 is and the slope of y = − x − 4 a + 0.052a = 2419.60
2 2 3
2 3 2 Carry out. We solve the equation.
is − . Since − = −1, the lines are perpendicular.
3 2 3 a + 0.052a = 2419.60
59. From Exercise 58 we know that the slope of the given line 1.052a = 2419.60
2
is − . The slope of a line perpendicular to this line is the a = 2300
3
2 3 Check. 5.2% of $2300 is 0.052($2300), or $119.60, and
negative reciprocal of − , or .
3 2 $2300 + $119.60 = $2419.60. The answer checks.
State. $2300 was originally invested.

Copyright © 2013 Pearson Education, Inc.


Chapter 1 Test 57

71. 6x − 18 = 0 State. In years more than about 14 years after 1999, or


6x = 18 in years after 2013, the number of homeschooled children
will exceed 2.0 million.
x=3
x+3
The zero of the function is 3. 83. f (x) =
8 − 4x
73. 2 − 10x = 0 When x = 2, the denominator is 0, so 2 is not in the
−10x = −2 domain of the function. Thus, the domain is
(−∞, 2) ∪ (2, ∞) and answer B is correct.
1
x= , or 0.2 1 1
5 85. The graph of f (x) = − x − 2 has slope − , so it slants
1 2 2
The zero of the function is , or 0.2. down from left to right. The y-intercept is (0, −2). Thus,
5 graph C is the graph of this function.
75. 2x − 5 < x + 7 √
1−x
x < 12 87. f (x) =
x − |x|
The solution set is {x|x < 12}, or (−∞, 12). We cannot find the square root of a negative number, so
x ≤ 1. Division by zero is undefined, so x < 0.
0 12 Domain of f is {x|x < 0}, or (−∞, 0).
77. −3 ≤ 3x + 1 ≤ 5 −3/5 1/2
89. Think of the slopes as and . The graph of
−4 ≤ 3x ≤ 4 1 1
3
4 4 f (x) changes unit vertically for each unit of horizon-
− ≤x≤ 5
3 3 1
 tal change while the graph of g(x) changes unit ver-
4 4 2
− , 3 1
3 3 tically for each unit of horizontal change. Since > ,
5 2
3
the graph of f (x) = − x + 4 is steeper than the graph of
5
1
g(x) = x − 6.
2
79. 2x < −1 or x − 3 > 0
91. The solution set of a disjunction is a union of sets, so it is
1
x < − or x>3 not possible for a disjunction to have no solution.
2


1 93. By definition, the notation 3 < x < 4 indicates that


The solution set is x x < − or x > 3 , or
2 3 < x and x < 4. The disjunction x < 3 or x > 4 cannot

1 be written 3 > x > 4, or 4 < x < 3, because it is not
− ∞, − ∪ (3, ∞). possible for x to be greater than 4 and less than 3.
2

1

2 Chapter 1 Test
0 3

81. Familiarize and Translate. The number of home- 1. 5y − 4 = x


schooled children in the U.S., in millions, is estimated by
9 1
the equation y = 0.08x + 0.83, where x is the number of 5· −4 ?
10 2
years after 1999. We want to know for what year this
9
number will exceed 2.0 million, so we have −4
2

0.08x + 0.83 > 2. 1 1
TRUE
Carry out. We solve the inequality. 2 2

0.08x + 0.83 > 2 1 9
, is a solution.
0.08x > 1.17 2 10
x > 14.625 2. 5x − 2y = −10
Check. When x = 14.625, y = 0.08(14.625) + 0.83 = 2.0. To find the x-intercept we replace y with 0 and solve for
As a partial check, we could try a value less than 14.625 x.
and a value greater than 14.625. When x = 14, we have 5x − 2 · 0 = −10
y = 0.08(14) + 0.83 = 1.95 < 2.0; when x = 15, we have 5x = −10
y = 0.08(15) + 0.83 = 2.03 > 2.0. Since y = 2.0 when
x = 14.625 and y > 2.0 when x = 15 > 14, the answer is x = −2
probably correct. The x-intercept is (−2, 0).

Copyright © 2013 Pearson Education, Inc.


58 Chapter 1: Graphs, Functions, and Models

To find the y-intercept we replace x with 0 and solve for 11. a) This is not the graph of a function, because we can
y. find a vertical line that crosses the graph more than
5 · 0 − 2y = −10 once.
−2y = −10 b) This is the graph of a function, because there is no
vertical line that crosses the graph more than once.
y=5
The y-intercept is (0, 5). 12. The input 4 results in a denominator of 0. Thus the do-
main is {x|x = 4}, or (−∞, 4) ∪ (4, ∞).
We plot the intercepts and draw the line that contains
them. We could find a third point as a check that the 13. We can substitute any real number for x. Thus the domain
intercepts were found correctly. is the set of all real numbers, or (−∞, ∞).

y 14. We cannot find the square root of a negative number. Thus


(0, 5) 25 − x2 ≥ 0 and the domain is {x| − 5 ≤ x ≤ 5}, or [−5, 5].
3
2 15. a)
(2, 0) y
4 2 2 4 x 4
2

4
2 f (x)  兩x  2兩  3
4 2 2 4 x
5x  2y  10 2

 √ √ 4
3. d = (5 − (−1))2 + (8 − 5)2 = 62 + 32 = 36 + 9 =

45 ≈ 6.708 b) Each point on the x-axis corresponds to a point on
   the graph, so the domain is the set of all real num-
−2 + (−4) 6 + 3 −6 9 9
4. m = , = , = − 3, bers, or (−∞, ∞).
2 2 2 2 2
c) The number 3 is the smallest output on the y-axis
5. (x + 4)2 + (y − 5)2 = 36
and every number greater than 3 is also an output,
[x − (−4)]2 + (y − 5)2 = 62 so the range is [3, ∞).
Center: (−4, 5); radius: 6 2 13
√ 5−
6. [x − (−1)] + (y − 2) = ( 5)2
2 2
16. m = 3 = 3
−2 − (−2) 0
(x + 1)2 + (y − 2)2 = 5
The slope is not defined.
7. a) The relation is a function, because no two ordered 12 − (−10) 22 11
pairs have the same first coordinate and different 17. m = = =−
−8 − 4 −12 6
second coordinates.
6−6 0
b) The domain is the set of first coordinates: 18. m = = =0
3 23
{−4, 0, 1, 3}. − (−5)
4 4
c) The range is the set of second coordinates: {0, 5, 7}.
19. We have the points (1995,21.6) and (2008,11.4).
8. f (x) = 2x2 − x + 5 11.4 − 21.6 −10.2
m= = ≈ −0.8
a) f (−1) = 2(−1)2 − (−1) + 5 = 2 + 1 + 5 = 8 2008 − 1995 13
b) f (a + 2) = 2(a + 2)2 − (a + 2) + 5 The average rate of change in the percent of 12th graders
who smoke daily was about −0.8% per year from 1995 to
= 2(a2 + 4a + 4) − (a + 2) + 5 2008.
= 2a2 + 8a + 8 − a − 2 + 5
20. −3x + 2y = 5
= 2a2 + 7a + 11
2y = 3x + 5
1−x 3 5
9. f (x) = y = x+
x 2 2
1−0 1 
a) f (0) = = 3 5
0 0 Slope: ; y-intercept: 0,
2 2
Since the division by 0 is not defined, f (0) does not
exist. 21. C(t) = 80 + 49.95t
1−1 0 2 yr = 2 · 1 yr = 2 · 12 months = 24 months
b) f (1) = = =0
1 1
C(24) = 80 + 49.95(24) = $1278.80
10. From the graph we see that when the input is −3, the
output is 0, so f (−3) = 0.

Copyright © 2013 Pearson Education, Inc.


Chapter 1 Test 59

22. y = mx + b Using the slope-intercept equation:


5 y = mx + b
y = − x−5
8 3 = 2(−1) + b
23. First we find the slope: 3 = −2 + b
−2 − 4 −6 3
m= = =− 5=b
3 − (−5) 8 4
The equation is y = 2x + 5.
Use the point-slope equation.
3 Using the point-slope equation.
Using (−5, 4): y − 4 = − (x − (−5)), or y − y1 = m(x − x1 )
4
3 y − 3 = 2(x − (−1))
y − 4 = − (x + 5)
4 y − 3 = 2(x + 1)
3
Using (3, −2): y − (−2) = − (x − 3), or y − 3 = 2x + 2
4
3 y = 2x + 5
y + 2 = − (x − 3)
4 28. a) Answers may vary depending on the data points
3 1
In either case, we have y = − x + . used. We will use (1, 12, 485) and (3, 11, 788).
4 4
 11, 788 − 12, 485 −697
m= = = −348.5
3
24. The vertical line that passes through − , 11 is unit
3 3−1 2
8 8 We will use the point-slope equation with
3 (1, 12, 485).
to the left of the y-axis. An equation of the line is x = − .
8
y − 12, 485 = −348.5(x − 1)
25. 2x + 3y = −12 2y − 3x = 8
y − 12, 485 = −348.5x + 348.5
2 3
y = − x−4 y = x+4 y = −348.5x + 12, 833.5,
3 2
2 3 where x is the number of years after 2005.
m1 = − , m2 = ; m1 m2 = −1. In 2010, x = 2010 − 2005 = 5.
3 2
The lines are perpendicular. y = −348.5(5) + 12, 833.5 = 11, 091 mi
26. First find the slope of the given line. In 2013, x = 2013 − 2005 = 8.
x + 2y = −6 y = −348.5(8) + 12, 833.5 = 10, 045.5 mi
2y = −x − 6 b) y = −234.7x + 12, 623.8, where x is the number of
1 1 years after 2005.
y = − x − 3; m = −
2 2 In 2010, y ≈ 11, 450 mi.
1 In 2013, y ≈ 10, 746 mi.
A line parallel to the given line has slope − . We use the
2
point-slope equation. r ≈ −0.9036
1
y − 3 = − (x − (−1)) 29. 6x + 7 = 1
2
1 6x = −6
y − 3 = − (x + 1) x = −1
2
1 1 The solution is −1.
y−3 = − x−
2 2
1 5 30. 2.5 − x = −x + 2.5
y = − x+
2 2 2.5 = 2.5 True equation
27. First we find the slope of the given line. The solution set is {x|x is a real number}, or (−∞, ∞).
x + 2y = −6 3 5
31. y−4 = y+6 The LCD is 6.
2y = −x − 6 2 3
1 1  
y = − x − 3, m = − 3 5
2 2 6 y−4 = 6 y+6
2 3
The slope of a line perpendicular to this line is the negative 9y − 24 = 10y + 36
1
reciprocal of − , or 2. Now we find an equation of the line −24 = y + 36
2
with slope 2 and containing (−1, 3). −60 = y
The solution is −60.

Copyright © 2013 Pearson Education, Inc.


60 Chapter 1: Graphs, Functions, and Models

32. 2(4x + 1) = 8 − 3(x − 5) 35. 3x + 9 = 0 Setting f (x) = 0


8x + 2 = 8 − 3x + 15 3x = −9
8x + 2 = 23 − 3x x = −3
11x + 2 = 23 The zero of the function is −3.
11x = 21 36. 5 − x ≥ 4x + 20
21
x= 5 − 5x ≥ 20
11
21 −5x ≥ 15
The solution is .
11 x ≤ −3 Dividing by −5 and reversing
3 the inequality symbol
33. Familiarize. Let l = the length, in meters. Then l =
4 The solution set is {x|x ≤ −3}, or (−∞, −3].
the width. Recall that the formula for the perimeter P of
a rectangle with length l and width w is P = 2l + 2w.
3 0
Translate.
The perimeter is 210
   
m. 37. −7 < 2x + 3 < 9
 
  
   −10 < 2x < 6 Subtracting 3
3 −5 < x < 3
2l + 2 · l = 210 Dividing by 2
4
Carry out. We solve the equation. The solution set is (−5, 3).
3
2l + 2 · l = 210
4
3
2l + l = 210 38. 2x − 1 ≤ 3 or 5x + 6 ≥ 26
2
7 2x ≤ 4 or 5x ≥ 20
l = 210
2 x ≤ 2 or x≥4
l = 60 The solution set is (−∞, 2] ∪ [4, ∞).
3 3
If l = 60, then l = · 60 = 45.
4 4
Check. The width, 45 m, is three-fourths of the length,
60 m. Also, 2 · 60 m + 2 · 45 m = 210 m, so the answer 39. Familiarize. Let t = the number of hours a move requires.
checks. Then Morgan Movers charges 90+25t to make a move and
State. The length is 60 m and the width is 45 m. McKinley Movers charges 40t.
Translate.
34. Familiarize. Let p = the wholesale price of the juice.
Morgan Movers’ McKinley Movers’
Translate. We express 25/c as $0.25. is
 less
than
charge charge.
 
    
 
50% of   
Wholesale
plus wholesale plus $0.25 is $2.95.
price 90 + 25t < 40t
  price
    
       Carry out. We solve the inequality.
p + 0.5p + 0.25 = 2.95 90 + 25t < 40t
Carry out. We solve the equation. 90 < 15t
p + 0.5p + 0.25 = 2.95 6<t
1.5p + 0.25 = 2.95 Check. For t = 6, Morgan Movers charge 90 + 25 · 6,
1.5p = 2.7 or $240, and McKinley Movers charge 40 · 6, or $240, so
the charge is the same for 6 hours. As a partial check, we
p = 1.8
can find the charges for a value of t greater than 6. For
Check. 50% of $1.80 is $0.90 and $1.80 + $0.90 + $0.25 = instance, for 6.5 hr Morgan Movers charge 90 + 25(6.5),
$2.95, so the answer checks. or $252.50, and McKinley Movers charge 40(6.5), or $260.
State. The wholesale price of a bottle of juice is $1.80. Since Morgan Movers cost less for a value of t greater than
6, the answer is probably correct.
State. It costs less to hire Morgan Movers when a move
takes more than 6 hr.

Copyright © 2013 Pearson Education, Inc.


Chapter 1 Test 61

1
40. The slope is − , so the graph slants down from left to
2
right. The y-intercept is (0, 1). Thus, graph B is the graph
1
of g(x) = 1 − x.
2
41. First we find the value of x for which x + 2 = −2:
x + 2 = −2
x = −4
Now we find h(−4 + 2), or h(−2).
1
h(−4 + 2) = (−4) = −2
2

Copyright © 2013 Pearson Education, Inc.


Copyright © 2013 Pearson Education, Inc.
Chapter 2
More on Functions
15. From the graph we see that a relative maximum value of
Exercise Set 2.1 the function is 2.370. It occurs at x = −0.667. We also
see that a relative minimum value of 0 occurs at x = 2.
1. a) For x-values from −5 to 1, the y-values increase from The graph starts rising, or increasing, from the left and
−3 to 3. Thus the function is increasing on the stops increasing at the relative maximum. From this point
interval (−5, 1). it decreases to the relative minimum and then increases
again. Thus the function is increasing on (−∞, −0.667)
b) For x-values from 3 to 5, the y-values decrease from
and on (2, ∞). It is decreasing on (−0.667, 2).
3 to 1. Thus the function is decreasing on the inter-
val (3, 5). 17. y

c) For x-values from 1 to 3, y is 3. Thus the function 5


is constant on (1, 3). 4
3
3. a) For x-values from −3 to −1, the y-values increase 2 f (x ) = x 2
from −4 to 4. Also, for x-values from 3 to 5, the 1

y-values increase from 2 to 6. Thus the function is –5 –4 –3 –2 –1 1 2 3 4 5 x


–1
increasing on (−3, −1) and on (3, 5). –2
–3
b) For x-values from 1 to 3, the y-values decrease from –4
3 to 2. Thus the function is decreasing on the inter- –5
val (1, 3).
c) For x-values from −5 to −3, y is 1. Thus the func- The function is increasing on (0, ∞) and decreasing on
tion is constant on (−5, −3). (−∞, 0). We estimate that the minimum is 0 at x = 0.
There are no maxima.
5. a) For x-values from −∞ to −8, the y-values increase
from −∞ to 2. Also, for x-values from −3 to −2, the 19. y
y-values increase from −2 to 3. Thus the function 5
is increasing on (−∞, −8) and on (−3, −2). 4
f (x ) = 5 — | x |

b) For x-values from −8 to −6, the y-values decrease 3

from 2 to −2. Thus the function is decreasing on


2
1
the interval (−8, −6). x
–5 –4 –3 –2 –1 1 2 3 4 5
c) For x-values from −6 to −3, y is −2. Also, for x- –1
–2
values from −2 to ∞, y is 3. Thus the function is –3
constant on (−6, −3) and on (−2, ∞). –4
–5
7. The x-values extend from −5 to 5, so the domain is [−5, 5].
The y-values extend from −3 to 3, so the range is [−3, 3]. The function is increasing on (−∞, 0) and decreasing on
(0, ∞). We estimate that the maximum is 5 at x = 0.
9. The x-values extend from −5 to −1 and from 1 to 5, so
There are no minima.
the domain is [−5, −1] ∪ [1, 5].
The y-values extend from −4 to 6, so the range is [−4, 6]. 21. y

5
11. The x-values extend from −∞ to ∞, so the domain is 4
(−∞, ∞). 3

The y-values extend from −∞ to 3, so the range is (−∞, 3].


2
1

x
13. From the graph we see that a relative maximum value of –5 –4 –3 –2 –1
–1
1 2 3 4 5

the function is 3.25. It occurs at x = 2.5. There is no –2 f (x ) = x 2 — 6x + 10


relative minimum value. –3
–4
The graph starts rising, or increasing, from the left and –5
stops increasing at the relative maximum. From this point,
the graph decreases. Thus the function is increasing on The function is decreasing on (−∞, 3) and increasing on
(−∞, 2.5) and is decreasing on (2.5, ∞). (3, ∞). We estimate that the minimum is 1 at x = 3.
There are no maxima.

Copyright © 2013 Pearson Education, Inc.


64 Chapter 2: More on Functions

23. 35. After t minutes, the balloon has risen 120t ft. We use the
Pythagorean theorem.
[d(t)]2 = (120t)2 + 4002

d(t) = (120t)2 + 4002
We considered only the positive square root since distance
must be nonnegative.
37. Let w = the width of the rectangle. Then the
40 − 2w
Beginning at the left side of the window, the graph first length = , or 20 − w. Divide the rectangle into
drops as we move to the right. We see that the function is 2
quadrants as shown below.
decreasing on (−∞, 1). We then find that the function is
increasing on (1, 3) and decreasing again on (3, ∞). The
MAXIMUM and MINIMUM features also show that the
relative maximum is −4 at x = 3 and the relative minimum
is −8 at x = 1.
20 – w
25.

In each quadrant there are two congruent triangles. One


triangle is part of the rhombus and both are part of the
rectangle. Thus, in each quadrant the area of the rhombus
We find that the function is increasing on (−1.552, 0) and is one-half the area of the rectangle. Then, in total, the
on (1.552, ∞) and decreasing on (−∞, −1.552) and on area of the rhombus is one-half the area of the rectangle.
(0, 1.552). The relative maximum is 4.07 at x = 0 and 1
A(w) = (20 − w)(w)
the relative minima are −2.314 at x = −1.552 and −2.314 2
at x = 1.552.
w2
A(w) = 10w −
27. a) y⫽ ⫺x 2 ⫹ 300x ⫹ 6 2
50,000 39. We will
 use similar triangles, expressing all distances
 in
1 s
feet. 6 in. = ft, s in. = ft, and d yd = 3d ft We
2 12
have
1
3d 2
0 300 = s
0 7
12
b) 22, 506 at a = 150 s 1
· 3d = 7 ·
c) The greatest number of baskets will be sold when 12 2
$150 thousand is spent on advertising. For that sd 7
amount, 22,506 baskets will be sold. =
4 2
8x 4 7
29. Graph y = . d = · , so
x2 + 1 s 2
Increasing: (−1, 1) 14
d(s) = .
Decreasing: (−∞, −1), (1, ∞) s
√ 41. a) If the length = x feet, then the width = 30 − x feet.
31. Graph y = x 4 − x2 , for −2 ≤ x ≤ 2.
A(x) = x(30 − x)
Increasing: (−1.414, 1.414)
A(x) = 30x − x2
Decreasing: (−2, −1.414), (1.414, 2)
b) The length of the rectangle must be positive and
33. If x = the length of the rectangle, in meters, then the less than 30 ft, so the domain of the function is
480 − 2x {x|0 < x < 30}, or (0, 30).
width is , or 240 − x. We use the formula Area =
2 c) We see from the graph that the maximum value of
length × width:
the area function on the interval (0, 30) appears to
A(x) = x(240 − x), or be 225 when x = 15. Then the dimensions that yield
A(x) = 240x − x2 the maximum area are length = 15 ft and width
= 30 − 15, or 15 ft.

Copyright © 2013 Pearson Education, Inc.


Exercise Set 2.1 65

x + 4, for x ≤ 1,
43. a) If the height of the file is x inches, then the width 47. g(x) =
8 − x, for x > 1
is 14 − 2x inches and the length is 8 in. We use the
formula Volume = length × width × height to find Since −4 ≤ 1, g(−4) = −4 + 4 = 0.
the volume of the file. Since 0 ≤ 1, g(0) = 0 + 4 = 4.
V (x) = 8(14 − 2x)x, or
Since 1 ≤ 1, g(1) = 1 + 4 = 5.
V (x) = 112x − 16x2
Since 3 > 1, g(3) = 8 − 3 = 5.
b) The height of the file must be positive and less than
half of the measure of the longside
 of the piece
 of 
 14 −3x − 18, for x < −5,

plastic. Thus, the domain is x0 < x < , or 49. h(x) = 1, for −5 ≤ x < 1,
2
{x|0 < x < 7}. x + 2, for x ≥ 1
Since −5 is in the interval [−5, 1), h(−5) = 1.
c) y ⫽ 112x ⫺ 16x 2
250 Since 0 is in the interval [−5, 1), h(0) = 1.
Since 1 ≥ 1, h(1) = 1 + 2 = 3.
Since 4 ≥ 1, h(4) = 4 + 2 = 6.

1
0 7  x, for x < 0,
0
51. f (x) = 2

d) Using the MAXIMUM feature, we find that the x + 3, for x ≥ 0
maximum value of the volume function occurs when 1
x = 3.5, so the file should be 3.5 in. tall. We create the graph in two parts. Graph f (x) = x for
2
inputs x less than 0. Then graph f (x) = x + 3 for inputs
45. a) The length of a diameter of the circle (and a di- x greater than or equal to 0.
agonal of the rectangle) is 2 · 8, or 16 ft. Let l =
the length of the rectangle. Use the Pythagorean y
theorem to write l as a function of x.
4
x2 + l2 = 162
2
x2 + l2 = 256
l2 = 256 − x2 4 2 2 4 x
 2
l = 256 − x2 4
Since the length must be positive, we considered
only the positive square root.
 3
Use the formula Area = length × width to find the  − x + 2, for x < 4,
area of the rectangle: 53. f (x) = 4
√ 
A(x) = x 256 − x2 −1, for x ≥ 4
b) The width of the rectangle must be positive and less We create the graph in two parts. Graph
3
than the diameter of the circle. Thus, the domain f (x) = − x + 2 for inputs x less than 4. Then graph
of the function is {x|0 < x < 16}, or (0, 16). 4
f (x) = −1 for inputs x greater than or equal to 4.
c)
y ⫽ x 256 ⫺ x 2
y
150
4

4 2 2 4 x
2
0 16
0 4

d) Using the MAXIMUM feature, we find that the max-


imum area√ occurs when x is about 11.314. When x ≈
11.314, 256 − x2 ≈ 256 − (11.314)2 ≈ 11.313.
Thus, the dimensions that maximize the area are
about 11.314 ft by 11.313 ft. (Answers may vary
slightly due to rounding differences.)

Copyright © 2013 Pearson Education, Inc.


66 Chapter 2: More on Functions



 x + 1, for x ≤ −3, y




 8
55. f (x) = −1, for −3 < x < 4




2


1
for x ≥ 4 8 4
 2 x, 4 8 x
4
We create the graph in three parts. Graph f (x) = x + 1 8
for inputs x less than or equal to −3. Graph f (x) = −1
for inputs greater than −3 and less than 4. Then graph
1
f (x) = x for inputs greater than or equal to 4. 61. f (x) = [[x]]
2
See Example 9.
y

4 2 2 4 x
2

4

1

 x − 1, for x < 0, 63. f (x) = 1 + [[x]]

2

This function can be defined by a piecewise function with
57. g(x) = 3, for 0 ≤ x ≤ 1

 an infinite

  number of statements:
 .
−2x, 

for x > 1 


 .
1 

We create the graph in three parts. Graph g(x) = x−1 
 .
2 

for inputs less than 0. Graph g(x) = 3 for inputs greater 
 −1, for −2 ≤ x < −1,

 0,
than or equal to 0 and less than or equal to 1. Then graph for −1 ≤ x < 0,
f (x) = 1, for 0 ≤ x < 1,
g(x) = −2x for inputs greater than 1. 


 2, for 1 ≤ x < 2,



 .
y 


 .


4 

.
2

4 2 2 4 x y
2
4
4
2

 4 2 4 x


2, for x = 5, 2
59. f (x) = x − 25
2 4 g(x)  1  冀x冁

 , for x = 5
x−5
When x = 5, the denominator of (x2 − 25)/(x − 5) is 65. From the graph we see that the domain is (−∞, ∞) and
nonzero so we can simplify: the range is (−∞, 0) ∪ [3, ∞).
x2 − 25 (x + 5)(x − 5)
= = x + 5. 67. From the graph we see that the domain is (−∞, ∞) and
x−5 x−5
the range is [−1, ∞).
Thus, f (x) = x + 5, for x = 5.
The graph of this part of the function consists of a line 69. From the graph we see that the domain is (−∞, ∞) and
with a “hole” at the point (5, 10), indicated by an open the range is {y|y ≤ −2 or y = −1 or y ≥ 2}.
dot. At x = 5, we have f (5) = 2, so the point (5, 2) is 71. From the graph we see that the domain is (−∞, ∞) and
plotted below the open dot. the range is {−5, −2, 4}. An equation for the function is:

−2, for x < 2,
f (x) = −5, for x = 2,
4, for x > 2

Copyright © 2013 Pearson Education, Inc.


Exercise Set 2.1 67

73. From the graph we see that the domain is (−∞, ∞) and 83. a) The function C(t) can be defined piecewise.
the range is (−∞, −1] ∪ [2, ∞). Finding the slope of each 


2, for 0 < t < 1,
segment and using the slope-intercept or point-slope for- 
 for 1 ≤ t < 2,


4,
mula, we find that an equation for the function is: 
 6, for 2 ≤ t < 3,
 C(t) = .
x, for x ≤ −1, 


g(x) = 2, for −1 < x ≤ 2, 

.


x, for x > 2 .
This can also be expressed as follows:
 We graph this function.
x, for x ≤ −1,
g(x) = 2, for −1 < x < 2, C
x, for x ≥ 2 8

6
75. From the graph we see that the domain is [−5, 3] and the
range is (−3, 5). Finding the slope of each segment and 4
using the slope-intercept or point-slope formula, we find 2
that an equation for the function is:
2 4 t

x + 8, for −5 ≤ x < −3,
h(x) = 3, for −3 ≤ x ≤ 1, b) From the definition of the function in part (a),
3x − 6, for 1 < x ≤ 3 we see that it can be written as

77. f (x) = 5x2 − 7 C(t) = 2[[t]] + 1, t > 0.

a) f (−3) = 5(−3)2 − 7 = 5 · 9 − 7 = 45 − 7 = 38 85. If [[x]]2 = 25, then [[x]] = −5 or [[x]] = 5. For


−5 ≤ x < −4, [[x]] = −5. For 5 ≤ x < 6, [[x]] = 5.
b) f (3) = 5 · 32 − 7 = 5 · 9 − 7 = 45 − 7 = 38
Thus, the possible inputs for x are
c) f (a) = 5a2 − 7 {x| − 5 ≤ x < −4 or 5 ≤ x < 6}.
d) f (−a) = 5(−a)2 − 7 = 5a2 − 7 87. a) We add labels to the drawing in the text.
79. First find the slope of the given line. E
8x − y = 10
8x = y + 10
8x − 10 = y
D
The slope of the given line is 8. The slope of a line per- 10
pendicular to this line is the opposite of the reciprocal of
1 h
8, or − .
8
y − y1 = m(x − x1 ) C
A r B 6–r
1
y − 1 = − [x − (−1)] 6
8
1 We write a proportion involving the lengths of the
y − 1 = − (x + 1) sides of the similar triangles BCD and ACE. Then
8
1 1 we solve it for h.
y−1 = − x− h 10
8 8 =
1 7 6−r 6
y = − x+ 10 5
8 8 h= (6 − r) = (6 − r)
6 3
81. Graph y = x4 + 4x3 − 36x2 − 160x + 400
30 − 5r
Increasing: (−5, −2), (4, ∞) h=
3
Decreasing: (−∞, −5), (−2, 4) 30 − 5r
Thus, h(r) = .
Relative maximum: 560 at x = −2 3
Relative minima: 425 at x = −5, −304 at x = 4
b) V = πr2 h
 
2 30 − 5r
V (r) = πr Substituting for h
3

Copyright © 2013 Pearson Education, Inc.


68 Chapter 2: More on Functions

c) We first express r in terms of h. 9. (g − f )(−1) = g(−1) − f (−1)


30 − 5r = [2(−1) + 1] − [(−1)2 − 3]
h=
3 = (−2 + 1) − (1 − 3)
3h = 30 − 5r
= −1 − (−2)
5r = 30 − 3h
= −1 + 2
30 − 3h
r= =1
5
V = πr2 h
 2
30 − 3h 11. (h − g)(−4) = h(−4) − g(−4)
V (h) = π h √
5 = (−4 + 4) − −4 − 1

Substituting for r = 0 − −5
 2 √
30 − 3h Since −5 is not a real number, (h−g)(−4) does not exist.
We can also write V (h) = πh .
5
13. (g/h)(1) = g(1)
h(1)
Exercise Set 2.2 √
1−1
=
1+4
1. (f + g)(5) = f (5) + g(5) √
0
= (52 − 3) + (2 · 5 + 1) =
5
= 25 − 3 + 10 + 1 0
= 33 = =0
5
3. (f − g)(−1) = f (−1) − g(−1) 15. (g + h)(1) = g(1) + h(1)
= ((−1)2 − 3) − (2(−1) + 1) √
= 1 − 1 + (1 + 4)
= −2 − (−1) = −2 + 1 √
= 0+5
= −1
= 0+5=5
1
  f −
1 17. f (x) = 2x + 3, g(x) = 3 − 5x
5. (f /g) − = 12
2 a) The domain of f and of g is the set of all real numbers,
g −
2 or (−∞, ∞). Then the domain of f + g, f − g, f f ,
1 2 3
− −3 and f g is also (−∞, ∞). For f /g we must exclude
3 5
= 21
2 − +1 since g = 0. Then the domain of f /g is
2 5
3 3
1 − ∞, ∪ , ∞ . For g/f we must exclude
−3 5 5
= 4 3 3
−1 + 1 − since f − = 0. The domain of g/f is
2 2
11
− − ∞, −
3 3
∪ − ,∞ .
= 4 2 2
0
  b) (f + g)(x) = f (x) + g(x) = (2x + 3) + (3 − 5x) =
1
Since division by 0 is not defined, (f /g) − does not −3x + 6
2
exist. (f − g)(x) = f (x) − g(x) = (2x + 3) − (3 − 5x) =
      2x + 3 − 3 + 5x = 7x
7. (f g) − 1 = f − 1 · g − 1 (f g)(x) = f (x) · g(x) = (2x + 3)(3 − 5x) =
2 2 2
 2     6x − 10x2 + 9 − 15x = −10x2 − 9x + 9
1 1 (f f )(x) = f (x) · f (x) = (2x + 3)(2x + 3) =
= − −3 2 − +1
2 2 4x2 + 12x + 9
11
= − ·0=0 f (x) 2x + 3
4 (f /g)(x) = =
g(x) 3 − 5x
g(x) 3 − 5x
(g/f )(x) = =
f (x) 2x + 3

Copyright © 2013 Pearson Education, Inc.


Exercise Set 2.2 69


19. f (x) = x − 3, g(x) = x+4 of the domains of f and g, or [3, ∞). The domain
of f f is the same as the domain of f , or [3, ∞). For
a) Any number can be an input in f , so the domain of
f /g, we must exclude −3 since g(−3) = 0. This is
f is the set of all real numbers, or (−∞, ∞).
not in [3, ∞), so the domain of f /g is [3, ∞). For
The domain of g consists of all values of x for which g/f , we must exclude 3 since f (3) = 0. The domain
x+4 is nonnegative, so we have x+4 ≥ 0, or x ≥ −4. of g/f is (3, ∞).
Thus, the domain of g is [−4, ∞). √ √
b) (f + g)(x) = f (x) + g(x) = x − 3 + x + 3
The domain of f + g, f − g, and f g is the set of all √ √
numbers in the domains of both f and g. This is (f − g)(x) = f (x) − g(x) = x − 3 − x + 3
√ √ √
[−4, ∞). (f g)(x) = f (x) · g(x) = x−3 · x + 3 = x2 −9
√ √
The domain of f f is the domain of f , or (−∞, ∞). (f f )(x) = f (x) · f (x) = x − 3 · x − 3 = |x − 3|

The domain of f /g is the set of all numbers in x−3
the domains of f and g, excluding those for which (f /g)(x) = √
x+3
g(x) = 0. Since g(−4) = 0, the domain of f /g is √
(−4, ∞). x+3
(g/f )(x) = √
x−3
The domain of g/f is the set of all numbers in
the domains of g and f , excluding those for which 25. f (x) = x + 1, g(x) = |x|
f (x) = 0. Since f (3) = 0, the domain of g/f is a) The domain of f and of g is (−∞, ∞). Then the
[−4, 3) ∪ (3, ∞). domain of f + g, f − g, f g, and f f is (−∞, ∞).
√ For f /g, we must exclude 0 since g(0) = 0. The
b) (f + g)(x) = f (x) + g(x) = x − 3 + x + 4
√ domain of f /g is (−∞, 0) ∪ (0, ∞). For g/f , we
(f − g)(x) = f (x) − g(x) = x − 3 − x + 4 must exclude −1 since f (−1) = 0. The domain of

(f g)(x) = f (x) · g(x) = (x − 3) x + 4 g/f is (−∞, −1) ∪ (−1, ∞).
 2
(f f )(x) = f (x) = (x − 3)2 = x2 − 6x + 9 b) (f + g)(x) = f (x) + g(x) = x + 1 + |x|
f (x) x−3 (f − g)(x) = f (x) − g(x) = x + 1 − |x|
(f /g)(x) = =√
g(x) x+4 (f g)(x) = f (x) · g(x) = (x + 1)|x|

g(x) x+4 (f f )(x) = f (x)·f (x) = (x+1)(x+1) = x2 + 2x + 1
(g/f )(x) = =
f (x) x−3 x+1
(f /g)(x) =
21. f (x) = 2x − 1, g(x) = −2x2 |x|
a) The domain of f and of g is (−∞, ∞). Then the |x|
(g/f )(x) =
domain of f + g, f − g, f g, and f f is (−∞, ∞). x+1
For f /g, we must exclude 0 since g(0) = 0. The 27. f (x) = x3 , g(x) = 2x2 + 5x − 3
domain of f /g is (−∞, 0) ∪ (0, ∞). For g/f , we
1 1 a) Since any number can be an input for either f or g,
must exclude since f = 0. The domain of the domain of f , g, f + g, f − g, f g, and f f is the set
2 2
1 1 of all real numbers, or (−∞, ∞).
g/f is − ∞, ∪ ,∞ .  
2 2 1
Since g(−3) = 0 and g = 0, the domain of f /g
b) (f + g)(x) = f (x) + g(x) = (2x − 1) + (−2x2 ) = 2
−2x2 + 2x − 1 1 1
is (−∞, −3) ∪ − 3, ∪ ,∞ .
(f − g)(x) = f (x) − g(x) = (2x − 1) − (−2x2 ) = 2 2
2x2 + 2x − 1 Since f (0) = 0, the domain of g/f is
(f g)(x) = f (x) · g(x) = (2x − 1)(−2x2 ) = (−∞, 0) ∪ (0, ∞).
−4x3 + 2x2
b) (f + g)(x) = f (x) + g(x) = x3 + 2x2 + 5x − 3
(f f )(x) = f (x) · f (x) = (2x − 1)(2x − 1) =
4x2 − 4x + 1 (f − g)(x) = f (x)−g(x) = x3 −(2x2 +5x−3) =
f (x) 2x − 1 x3 − 2x2 − 5x + 3
(f /g)(x) = =
g(x) −2x2 (f g)(x) = f (x) · g(x) = x3 (2x2 + 5x − 3) =
g(x) −2x 2
2x5 + 5x4 − 3x3
(g/f )(x) = =
f (x) 2x − 1 (f f )(x) = f (x) · f (x) = x3 · x3 = x6
√ √
23. f (x) = x − 3, g(x) = x + 3 f (x) x3
(f /g)(x) = = 2
a) Since f (x) is nonnegative for values of x in [3, ∞), g(x) 2x + 5x − 3
this is the domain of f . Since g(x) is nonnegative g(x) 2x2 + 5x − 3
(g/f )(x) = =
for values of x in [−3, ∞), this is the domain of g. f (x) x3
The domain of f +g, f −g, and f g is the intersection

Copyright © 2013 Pearson Education, Inc.


70 Chapter 2: More on Functions

4 1 3 √
29. f (x) = , g(x) = 33. f (x) = , g(x) = x − 1
x+1 6−x x−2
a) Since x + 1 = 0 when x = −1, we must exclude a) Since f (2) is not defined, the domain of f is
−1 from the domain of f . It is (−∞, −1) ∪ (−1, ∞). (−∞, 2) ∪ (2, ∞). Since g(x) is nonnegative for val-
Since 6 − x = 0 when x = 6, we must exclude 6 from ues of x in [1, ∞), this is the domain of g. The
the domain of g. It is (−∞, 6)∪(6, ∞). The domain domain of f + g, f − g, and f g is the intersection
of f + g, f − g, and f g is the intersection of the of the domains of f and g, or [1, 2) ∪ (2, ∞). The
domains of f and g, or (−∞, −1) ∪ (−1, 6) ∪ (6, ∞). domain of f f is the same as the domain of f , or
The domain of f f is the same as the domain of f , (−∞, 2) ∪ (2, ∞). For f /g, we must exclude 1 since
or (−∞, −1) ∪ (−1, ∞). Since there are no values g(1) = 0, so the domain of f /g is (1, 2) ∪ (2, ∞).
of x for which g(x) = 0 or f (x) = 0, the domain of There are no values of x for which f (x) = 0, so the
f /g and g/f is (−∞, −1) ∪ (−1, 6) ∪ (6, ∞). domain of g/f is [1, 2) ∪ (2, ∞).
4 1 3 √
b) (f + g)(x) = f (x) + g(x) = + b) (f + g)(x) = f (x) + g(x) = + x−1
x+1 6−x x−2
4 1 3 √
(f − g)(x) = f (x) − g(x) = − (f − g)(x) = f (x) − g(x) = − x−1
x+1 6−x x−2

4 1 4 3 √ 3 x−1
(f g)(x) = f (x)·g(x) = · = (f g)(x) = f (x) · g(x) = ( x − 1), or
x+1 6−x (x+1)(6−x) x−2 x−2
4 4 16 3 3 9
(f f )(x) = f (x)·f (x) = · = , or (f f )(x) = f (x) · f (x) = · ·
x + 1 x + 1 (x + 1)2 x − 2 x − 2 (x − 2)2
16 3
x2 + 2x + 1 f (x) x − 3
(f /g)(x) = =√ 2 = √
4 g(x) x−1 (x − 2) x − 1
x + 1 = 4 6−x 4(6 − x) √ √
(f /g)(x) = · = g(x) x−1 (x − 2) x − 1
1 x+1 1 x+1 (g/f )(x) = = =
f (x) 3 3
6−x
1 x−2

(g/f )(x) = 6 − x =
1 x+1 x+1 35. From the graph we see that the domain of F is [2, 11] and
· =
4 6−x 4 4(6 − x) the domain of G is [1, 9]. The domain of F + G is the set
x+1 of numbers in the domains of both F and G. This is [2, 9].
1 37. The domain of G/F is the set of numbers in the domains of
31. f (x) = , g(x) = x − 3
x both F and G (See Exercise 33.), excluding those for which
a) Since f (0) is not defined, the domain of f is F = 0. Since F (3) = 0, the domain of G/F is [2, 3) ∪ (3, 9].
(−∞, 0) ∪ (0, ∞). The domain of g is (−∞, ∞).
Then the domain of f + g, f − g, f g, and f f is 39.
y
(−∞, 0) ∪ (0, ∞). Since g(3) = 0, the domain of 6
f /g is (−∞, 0) ∪ (0, 3) ∪ (3, ∞). There are no values 4 G F
of x for which f (x) = 0, so the domain of g/f is
2
(−∞, 0) ∪ (0, ∞).
1 2 4 6 8 10 x
b) (f + g)(x) = f (x) + g(x) = + x − 3 2
x
1 1 4
(f −g)(x) = f (x)−g(x) = −(x−3) = −x + 3
x x
1 x−3 3
(f g)(x) = f (x)·g(x) = ·(x−3) = , or 1 − 41. From the graph, we see that the domain of F is [0, 9] and
x x x the domain of G is [3, 10]. The domain of F + G is the set
1 1 1 of numbers in the domains of both F and G. This is [3, 9].
(f f )(x) = f (x) · f (x) = · = 2
x x x
1 43. The domain of G/F is the set of numbers in the domains
f (x) 1 1 1 of both F and G (See Exercise 39.), excluding those for
(f /g)(x) = = x = · = which F = 0. Since F (6) = 0 and F (8) = 0, the domain
g(x) x−3 x x−3 x(x − 3)
of G/F is [3, 6) ∪ (6, 8) ∪ (8, 9].
g(x) x−3 x
(g/f )(x) = = = (x−3) · = x(x−3), or
f (x) 1 1
x
x2 − 3x

Copyright © 2013 Pearson Education, Inc.


Exercise Set 2.2 71

45. 1
y 55. f (x) =
3x
6 1
G⫺F f (x + h) =
4 3(x + h)
2 1 1

f (x + h) − f (x) 3(x + h) 3x
2 4 6 8 10 x =
⫺2
h h
1 x 1 x+h
· − ·
3(x + h) x 3x x + h
47. a) P (x) = R(x) − C(x) = 60x − 0.4x2 − (3x + 13) = =
h
60x − 0.4x2 − 3x − 13 = −0.4x2 + 57x − 13 x x+h

b) R(100) = 60·100−0.4(100)2 = 6000−0.4(10, 000) = 3x(x + h) 3x(x + h)
=
6000 − 4000 = 2000 h
C(100) = 3 · 100 + 13 = 300 + 13 = 313 x − (x + h) x−x−h
3x(x + h) 3x(x + h)
P (100) = R(100) − C(100) = 2000 − 313 = 1687 = =
h h
49. f (x) = 3x − 5 −h
f (x + h) = 3(x + h) − 5 = 3x + 3h − 5 3x(x + h) −h 1
= = ·
f (x + h) − f (x) 3x + 3h − 5 − (3x − 5) h 3x(x + h) h
=
h h −h −1 · h
/
= =
3x + 3h − 5 − 3x + 5 3x(x + h) · h 3x(x + h) · h
/
= −1 1
h = , or −
3h 3x(x + h) 3x(x + h)
= =3
h 1
57. f (x) = −
51. f (x) = 6x + 2 4x
1
f (x + h) = 6(x + h) + 2 = 6x + 6h + 2 f (x + h) = −
4(x + h)
f (x + h) − f (x) 6x + 6h + 2 − (6x + 2)  
= 1 1
h h − − −
6x + 6h + 2 − 6x − 2 f (x + h) − f (x) 4(x + h) 4x
= =
h h h
 
6h 1 x 1 x+h
= =6 − · − − ·
h 4(x + h) x 4x x+h
=
1 h
53. f (x) = x+1 x x+h
3 − +
1 1 1 4x(x + h) 4x(x + h)
f (x + h) = (x + h) + 1 = x + h + 1 =
3 3 3 h
 
1 1 1 −x + x + h h
x+ h+1− x+1
f (x + h) − f (x) 3 3 3 4x(x + h) 4x(x + h)
= = =
h h h h
1 1 1 h 1 / ·1
h 1
x+ h+1− x−1 = · = =
= 3 3 3 4x(x+h) h 4x(x+h)·h/ 4x(x+h)
h
1
h 1
= 3 =
h 3

Copyright © 2013 Pearson Education, Inc.


72 Chapter 2: More on Functions

59. f (x) = x2 + 1 67. f (x) = x3


f (x + h) = (x + h)2 + 1 = x2 + 2xh + h2 + 1 f (x + h) = (x + h)3 = x3 + 3x2 h + 3xh2 + h3
f (x + h) − f (x) x + 2xh + h + 1 − (x + 1)
2 2 2
f (x) = x3
=
h h f (x + h) − f (x) x3 + 3x2 h + 3xh2 + h3 − x3
= =
x + 2xh + h + 1 − x − 1
2 2 2 h h
=
h 3x2 h + 3xh2 + h3 h(3x2 + 3xh + h2 )
= =
2xh + h2 h h·1
= h 3x2 + 3xh + h2
h · = 3x2 + 3xh + h2
h 1
h(2x + h)
= x−4
h 69. f (x) =
h 2x + h x+3
= · x+h−4 x−4
h 1
f (x + h) − f (x) −
= 2x + h = +h+3 x+3 =
x
h h
61. f (x) = 4 − x2 x+h−4 x−4

f (x + h) = 4 − (x + h)2 = 4 − (x2 + 2xh + h2 ) = x + h + 3 x + 3 · (x + h + 3)(x + 3) =
4 − x2 − 2xh − h2 h (x + h + 3)(x + 3)
f (x + h) − f (x) 4 − x2 − 2xh − h2 − (4 − x2 ) (x + h − 4)(x + 3) − (x − 4)(x + h + 3)
= =
h h h(x + h + 3)(x + 3)
4 − x2 − 2xh − h2 − 4 + x2 x2+hx−4x+3x+3h−12−(x2+hx+3x−4x−4h−12)
= =
h h(x+h+3)(x+3)
−2xh − h2 /(−2x − h)
h x2 + hx − x + 3h − 12 − x2 − hx + x + 4h + 12
= = =
h h/ h(x + h + 3)(x + 3)
= −2x − h 7h h 7
= · =
63. f (x) = 3x2 − 2x + 1 h(x + h + 3)(x + 3) h (x + h + 3)(x + 3)
7
f (x + h) = 3(x + h)2 − 2(x + h) + 1 =
(x + h + 3)(x + 3)
3(x2 + 2xh + h2 ) − 2(x + h) + 1 =
71. Graph y = 3x − 1.
3x2 + 6xh + 3h2 − 2x − 2h + 1
We find some ordered pairs that are solutions of the equa-
f (x) = 3x2 − 2x + 1
tion, plot these points, and draw the graph.
f (x + h) − f (x)
= When x = −1, y = 3(−1) − 1 = −3 − 1 = −4.
h
(3x + 6xh + 3h −2x−2h + 1)−(3x2 − 2x + 1)
2 2 When x = 0, y = 3 · 0 − 1 = 0 − 1 = −1.
=
h When x = 2, y = 3 · 2 − 1 = 6 − 1 = 5.
3x + 6xh + 3h − 2x − 2h + 1 − 3x + 2x − 1
2 2 2 y
= x y
h
6xh + 3h − 2h
2
h(6x + 3h − 2) −1 −4 4
= =
h h·1 0 −1 2
h 6x + 3h − 2
· = 6x + 3h − 2 2 5 4 2 2 4 x
h 1 2
65. f (x) = 4 + 5|x| 4 y  3x  1
f (x + h) = 4 + 5|x + h|
f (x + h) − f (x) 4 + 5|x + h| − (4 + 5|x|) 73. Graph x − 3y = 3.
=
h h
First we find the x- and y-intercepts.
4 + 5|x + h| − 4 − 5|x|
= x−3·0 = 3
h
x=3
5|x + h| − 5|x|
= The x-intercept is (3, 0).
h
0 − 3y = 3
−3y = 3
y = −1
The y-intercept is (0, −1).

Copyright © 2013 Pearson Education, Inc.


Exercise Set 2.3 73

We find a third point as a check. We let x = −3 and solve 17. (f ◦ g)(x) = f (g(x)) = f (x − 3) = x − 3 + 3 = x
for y. (g ◦ f )(x) = g(f (x)) = g(x + 3) = x + 3 − 3 = x
−3 − 3y = 3 The domain of f and of g is (−∞, ∞), so the domain of
−3y = 6 f ◦ g and of g ◦ f is (−∞, ∞).
y = −2
Another point on the graph is (−3, −2). We plot the points 19. (f ◦ g)(x) = f (g(x)) = f (3x2 −2x−1) = 3x2 −2x−1+1 =
and draw the graph. 3x2 − 2x
y (g ◦ f )(x) = g(f (x)) = g(x+1) = 3(x+1)2 −2(x+1)−1 =
3(x2 +2x+1)−2(x+1)−1 = 3x2 +6x+3−2x−2−1 =
4 3x2 + 4x
2 x  3y  3 The domain of f and of g is (−∞, ∞), so the domain of
f ◦ g and of g ◦ f is (−∞, ∞).
4 2 2 4 x
2 21. (f ◦ g)(x) = f (g(x)) = f (4x−3) = (4x−3)2 −3 =
4 16x2 − 24x + 9 − 3 = 16x2 − 24x + 6
(g ◦ f )(x) = g(f (x)) = g(x2 −3) = 4(x2 −3)−3 =
4x2 − 12 − 3 = 4x2 − 15
1 1
75. Answers may vary; f (x) = , g(x) = The domain of f and of g is (−∞, ∞), so the domain of
x+7 x−3
  f ◦ g and of g ◦ f is (−∞, ∞).
 7
77. The domain of h(x) is xx = , and the domain of g(x) 1
3 4 4
7 23. (f ◦ g)(x) = f (g(x)) = f = = =
is {x|x = 3}, so and 3 are not in the domain of (h/g)(x). x 1 5
3 1−5· 1−
We must also exclude the value of x for which g(x) = 0. x x
4 x 4x
=4· =
x4 − 1 x−5 x−5 x−5
=0
5x − 15 x 4 1
x4 − 1 = 0 Multiplying by 5x − 15 (g ◦ f )(x) = g(f (x)) = g = =
1 − 5x 4
4
x =1 1 − 5x
1 − 5x 1 − 5x
x = ±1 1· =
4 4
  
Then the domain of (h/g)(x) is
    1
 7 The domain of f is xx = and the domain of g is
xx = and x = 3 and x = −1 and x = 1 , or 5
3 7 7 {x|x = 0}. Consider the domain of f ◦ g. Since 0 is not in
(−∞, −1) ∪ (−1, 1) ∪ 1, ∪ , 3 ∪ (3, ∞). 1
3 3 the domain of g, 0 is not in the domain of f ◦ g. Since
5
1
is not in the domain of f , we know that g(x) cannot be .
Exercise Set 2.3 5
1
We find the value(s) of x for which g(x) = .
5
1. (f ◦ g)(−1) = f (g(−1)) = f ((−1)2 − 2(−1) − 6) = 1 1
=
f (1 + 2 − 6) = f (−3) = 3(−3) + 1 = −9 + 1 = −8 x 5
5=x Multiplying by 5x
3. (h ◦ f )(1) = h(f (1)) = h(3 · 1 + 1) = h(3 + 1) =
Thus 5 is also not in the domain of f ◦ g. Then the domain
h(4) = 43 = 64
of f ◦g is {x|x = 0 and x = 5}, or (−∞, 0)∪(0, 5)∪(5, ∞).
1
5. (g ◦ f )(5) = g(f (5)) = g(3 · 5 + 1) = g(15 + 1) = Now consider the domain of g ◦ f . Recall that is not in
5
g(16) = 162 − 2 · 16 − 6 = 218 the domain of f , so it is not in the domain of g ◦ f . Now 0
is not in the domain of g but f (x) is never 0, so the domain
  1 1 1
7. (f ◦ h)(−3) = f (h(−3)) = f ((−3)3 ) = f (−27) = 
of g ◦ f is xx = , or − ∞, ∪ ,∞ .
3(−27) + 1 = −81 + 1 = −80 5 5 5
 
9. (g ◦ g)(−2) = g(g(−2)) = g((−2)2 − 2(−2) − 6) = x+7
25. (f ◦ g)(x) = f (g(x)) = f =
g(4 + 4 − 6) = g(2) = 22 − 2 · 2 − 6 = 4 − 4 − 6 = −6 3
 
x+7
11. (h ◦ h)(2) = h(h(2)) = h(23 ) = h(8) = 83 = 512 3 −7=x+7−7=x
3
(f ◦ f )(−4) = f (f (−4)) = f (3(−4) + 1) = f (−12 + 1) = (3x − 7) + 7
13. (g ◦ f )(x) = g(f (x)) = g(3x − 7) = =
3
f (−11) = 3(−11) + 1 = −33 + 1 = −32
3x
=x
15. (h ◦ h)(x) = h(h(x)) = h(x3 ) = (x3 )3 = x9 3

Copyright © 2013 Pearson Education, Inc.


74 Chapter 2: More on Functions

 
The domain of f and of g is (−∞, ∞), so the domain of 1−x
(g ◦ f )(x) = g(f (x)) = g =
f ◦ g and of g ◦ f is (−∞, ∞). x
√ √ 1 1
27. (f ◦ g)(x) = f (g(x)) = f ( x) = 2 x + 1  = =
√ 1−x x+1−x
(g ◦ f )(x) = g(f (x)) = g(2x + 1) = 2x + 1 1+
x
x
The domain of f is (−∞, ∞) and the domain of g is 1 x
{x|x ≥ 0}. Thus the domain of f ◦ g is {x|x ≥ 0}, or =1· =x
1 1
[0, ∞).
x
Now consider the domain of g ◦f . There are no restrictions The domain of f is {x|x = 0} and the domain of g is
on the domain of f , but the domain of g is {x|x ≥ 0}. Since {x|x = −1}, so we know that −1 is not in the domain
  1
1  of f ◦ g. Since 0 is not in the domain of f , values of x
f (x) ≥ 0 for x ≥ − , the domain of g ◦ f is xx ≥ − ,
 1  2 2 for which g(x) = 0 are not in the domain of f ◦ g. But
or − , ∞ . g(x) is never 0, so the domain of f ◦ g is {x|x = −1}, or
2
(−∞, −1) ∪ (−1, ∞).
29. (f ◦ g)(x) = f (g(x)) = f (0.05) = 20
Now consider the domain of g ◦ f . Recall that 0 is not in
(g ◦ f )(x) = g(f (x)) = g(20) = 0.05 the domain of f . Since −1 is not in the domain of g, we
The domain of f and of g is (−∞, ∞), so the domain of know that g(x) cannot be −1. We find the value(s) of x
f ◦ g and of g ◦ f is (−∞, ∞). for which f (x) = −1.
1−x
31. (f ◦ g)(x) = f (g(x)) = f (x2 − 5) = = −1
√ √ x
x2 − 5 + 5 = x2 = |x| 1 − x = −x Multiplying by x

(g ◦ f )(x) = g(f (x)) = g( x + 5) = 1=0 False equation

( x + 5)2 − 5 = x + 5 − 5 = x We see that there are no values of x for which f (x) = −1,
The domain of f is {x|x ≥ −5} and the domain of g is so the domain of g ◦ f is {x|x = 0}, or (−∞, 0) ∪ (0, ∞).
(−∞, ∞). Since x2 ≥ 0 for all values of x, then x2 −5 ≥ −5
for all values of x and the domain of g ◦ f is (−∞, ∞). 37. (f ◦ g)(x) = f (g(x)) = f (x + 1) =
Now consider the domain of f ◦g. There are no restrictions (x + 1)3 − 5(x + 1)2 + 3(x + 1) + 7 =
on the domain of g, so the domain of f ◦ g is the same as x3 + 3x2 + 3x + 1 − 5x2 − 10x − 5 + 3x + 3 + 7 =
the domain of f , {x|x ≥ −5}, or [−5, ∞).
x3 − 2x2 − 4x + 6
√ √
33. (f ◦ g)(x) = f (g(x)) = f ( 3 − x) = ( 3 − x)2 + 2 =
(g ◦ f )(x) = g(f (x)) = g(x3 − 5x2 + 3x + 7) =
3−x+2=5−x
 x3 − 5x2 + 3x + 7 + 1 = x3 − 5x2 + 3x + 8
(g ◦ f )(x) = g(f (x)) = g(x2 + 2) = 3 − (x2 + 2) =
√ √ The domain of f and of g is (−∞, ∞), so the domain of
3 − x2 − 2 = 1 − x2 f ◦ g and of g ◦ f is (−∞, ∞).
The domain of f is (−∞, ∞) and the domain of g is
{x|x ≤ 3}, so the domain of f ◦ g is {x|x ≤ 3}, or (−∞, 3]. 39. h(x) = (4 + 3x)5

Now consider the domain of g ◦f . There are no restrictions This is 4 + 3x to the 5th power. The most obvious answer
on the domain of f and the domain of g is {x|x ≤ 3}, so is f (x) = x5 and g(x) = 4 + 3x.
we find the values of x for which f (x) ≤ 3. We see that 1
x2 + 2 ≤ 3 for −1 ≤ x ≤ 1, so the domain of g ◦ f is 41. h(x) =
(x − 2)4
{x| − 1 ≤ x ≤ 1}, or [−1, 1].
This is 1 divided by (x − 2) to the 4th power. One obvious
  1
35. (f ◦ g)(x) = f (g(x)) = f
1
= answer is f (x) = 4 and g(x) = x − 2. Another possibility
1+x x
1
is f (x) = and g(x) = (x − 2)4 .
  x
1−
1 1+x−1
1+x x−1
= 1+x = 43. f (x) = , g(x) = x3
1 1 x+1
1+x 1+x 2 + x3
45. f (x) = x6 , g(x) =
x 1+x 2 − x3
· =x
1+x 1 √ x−5
47. f (x) = x, g(x) =
x+2
49. f (x) = x3 − 5x2 + 3x − 1, g(x) = x + 2

Copyright © 2013 Pearson Education, Inc.


Chapter 2 Mid-Chapter Mixed Review 75

51. a) Use the distance formula, distance = rate × y


time. Substitute 3 for the rate and t for time.
4
r(t) = 3t
2
b) Use the formula for the area of a circle.
A(r) = πr2 ⫺4 ⫺2 2 4 x
⫺2
c) (A ◦ r)(t) = A(r(t)) = A(3t) = π(3t)2 = 9πt2
⫺4
This function gives the area of the ripple in terms
of time t.
11. (f g)(0) = f (0) · g(0)
53. The manufacturer charges m + 2 per drill. The chain store
sells each drill for 150%(m + 2), or 1.5(m + 2), or 1.5m + 3. = (3 · 0 − 1) · (02 + 4)
Thus, we have P (m) = 1.5m + 3. = −1 · 4
55. Equations (a) − (f ) are in the form y = mx + b, so we can = −4
read the y-intercepts directly from the equations. Equa-  
1
2   g
tions (g) and (h) can be written in this form as y = x − 2 1 3
3 13. (g/f ) =  
and y = −2x + 3, respectively. We see that only equa- 3 1
tion (c) has y-intercept (0, 1). f
3
 2
57. If a line slopes down from left to right, its slope is negative. 1
The equations y = mx + b for which m is negative are (b), +4
3
(d), (f), and (h). (See Exercise 55.) =
1
3· −1
59. The only equation that has (0, 0) as a solution is (a). 3
1
61. Only equations (c) and (g) have the same slope and differ- +4
= 9
ent y-intercepts. They represent parallel lines 1−1
63. Only the composition (c ◦ p)(a) makes sense. It represents 37
the cost of the grass seed required to seed a lawn with area = 9
a. 0
 
1
Since division by 0 is not defined, (g/f ) does not exist.
3
Chapter 2 Mid-Chapter Mixed Review

15. f (x) = x − 1, g(x) = x+2
a) Any number can be an input for f , so the domain
1. The statement is true. See page 162 in the text.
of f is the set of all real numbers, or (−∞, ∞).
3. The statement is true. See Example 2 on page 185 in the The domain of g consists of all values for which x+2
text, for instance. is nonnegative, so we have x + 2 ≥ 0, or x ≥ −2, or
[−2, ∞). Then the domain of f + g, f − g, and f g
5. From the graph we see that a relative maximum value of
is [−2, ∞).
6.30 occurs at x = −1.29. We also see that a relative
minimum value of −2.30 occurs at x = 1.29. The domain of f f is (−∞, ∞).
The graph starts rising, or increasing, from the left and Since g(−2) = 0, the domain of f /g is (−2, ∞).
stops increasing at the relative maximum. From this point Since f (1) = 0, the domain of g/f is [−2, 1)∪(1, ∞).
it decreases to the relative minimum and then increases √
b) (f + g)(x) = f (x) + g(x) = x − 1 + x + 2
again. Thus the function is increasing on (−∞, −1.29) √
and on (1.29, ∞). It is decreasing on (−1.29, 1.29). (f − g)(x) = f (x) − g(x) = x − 1 − x + 2

1 (f g)(x) = f (x) · g(x) = (x − 1) x + 2
7. A(h) = (h − 2)h
2 (f f )(x) = f (x) · f (x) = (x − 1)(x − 1) =
1
A(h) = h2 − h x2 − x − x + 1 = x2 − 2x + 1
2 f (x) x−1
 (f /g)(x) = =√
x + 2, for x < −4, g(x) x+2
9. g(x) = √
−x, for x ≥ −4 g(x) x+2
(g/f )(x) = =
We create the graph in two parts. Graph g(x) = x + 2 f (x) x−1
for inputs less than −4. Then graph g(x) = −x for inputs
greater than or equal to −4.

Copyright © 2013 Pearson Education, Inc.


76 Chapter 2: More on Functions

17. f (x) = 6 − x2 5. If the graph were folded on the x-axis, the parts above and
f (x + h) − f (x) 6 − (x + h)2 − (6 − x2 ) below the x-axis would not coincide, so the graph is not
= = symmetric with respect to the x-axis.
h h
6−(x2 +2xh+h2 )−6+x2 6−x2 −2xh−h2 −6 + x2 If the graph were folded on the y-axis, the parts to the left
= = and right of the y-axis would not coincide, so the graph is
h h
not symmetric with respect to the y-axis.
−2xh − h2 /(−2x − h)
h
= = −2x − h If the graph were rotated 180◦ , the resulting graph would
h /·1
h coincide with the original graph, so it is symmetric with
19. (g ◦ h)(2) = g(h(2)) = g(22 − 2 · 2 + 3) = g(4 − 4 + 3) = respect to the origin.
g(3) = 33 + 1 = 27 + 1 = 28
7. y
21. (h ◦ f )(−1) = h(f (−1)) = h(5(−1) − 4) = h(−5 − 4) =
h(−9) = (−9)2 − 2(−9) + 3 = 81 + 18 + 3 = 102 5
4
√ √
23. (f ◦ g)(x) = f (g(x)) = f ( x) = 3 x + 2 3

√ 2
(g ◦ f )(x) = g(f (x)) = g(3x + 2) = 3x + 2 1

The domain of f is (−∞, ∞) and the domain of g is [0, ∞). —5 —4 —3 —2 —1


—1
1 2 3 4 5 x

Consider the domain of f ◦ g. Since any number can be an —2

input for f , the domain of f ◦ g is the same as the domain


—3
y = |x | – 2
—4
of g, [0, ∞). —5

Now consider the domain of g ◦ f . Since the inputs of g


2 The graph is symmetric with respect to the y-axis. It is
must be nonnegative, we must have 3x+2 ≥ 0, or x ≥ − .
  3 not symmetric with respect to the x-axis or the origin.
2
Thus the domain of g ◦ f is − , ∞ . Test algebraically for symmetry with respect to the x-axis:
3
y = |x| − 2 Original equation
25. Under the given conditions, (f + g)(x) and (f /g)(x) have
−y = |x| − 2 Replacing y by −y
different domains if g(x) = 0 for one or more real numbers
x. y = −|x| + 2 Simplifying
The last equation is not equivalent to the original equation,
27. This approach is not valid. Consider Exercise 23 on page
4x so the graph is not symmetric with respect to the x-axis.
188 in the text, for example. Since (f ◦ g)(x) = ,
x−5 Test algebraically for symmetry with respect to the y-axis:
an examination of only this composed function would lead y = |x| − 2 Original equation
to the incorrect conclusion that the domain of f ◦ g is
(−∞, 5) ∪ (5, ∞). However, we must also exclude from the y = | − x| − 2 Replacing x by −x
domain of f ◦g those values of x that are not in the domain y = |x| − 2 Simplifying
of g. Thus, the domain of f ◦ g is (−∞, 0) ∪ (0, 5) ∪ (5, ∞). The last equation is equivalent to the original equation, so
the graph is symmetric with respect to the y-axis.
Exercise Set 2.4 Test algebraically for symmetry with respect to the origin:
y = |x| − 2 Original equation
1. If the graph were folded on the x-axis, the parts above and
below the x-axis would not coincide, so the graph is not −y = | − x| − 2 Replacing x by −x and
symmetric with respect to the x-axis. y by −y
If the graph were folded on the y-axis, the parts to the −y = |x| − 2 Simplifying
left and right of the y-axis would coincide, so the graph is y = −|x| + 2
symmetric with respect to the y-axis. The last equation is not equivalent to the original equation,
If the graph were rotated 180◦ , the resulting graph would so the graph is not symmetric with respect to the origin.
not coincide with the original graph, so it is not symmetric
with respect to the origin. 9. y

3. If the graph were folded on the x-axis, the parts above and 5
4
below the x-axis would coincide, so the graph is symmetric
5y = 4x + 5 3
with respect to the x-axis. 2

If the graph were folded on the y-axis, the parts to the left 1

and right of the y-axis would not coincide, so the graph is —5 —4 —3 —2 —1


—1
1 2 3 4 5 x

not symmetric with respect to the y-axis. —2

If the graph were rotated 180◦ , the resulting graph would —3


—4
not coincide with the original graph, so it is not symmetric —5
with respect to the origin.

Copyright © 2013 Pearson Education, Inc.


Exercise Set 2.4 77

The graph is not symmetric with respect to the x-axis, the Test algebraically for symmetry with respect to the origin:
y-axis, or the origin. 5y = 2x2 − 3 Original equation
Test algebraically for symmetry with respect to the x-axis: 5(−y) = 2(−x)2 − 3 Replacing x by −x and
5y = 4x + 5 Original equation y by −y
5(−y) = 4x + 5 Replacing y by −y −5y = 2x2 − 3 Simplifying
−5y = 4x + 5 Simplifying 5y = −2x + 32

5y = −4x − 5 The last equation is not equivalent to the original equation,


The last equation is not equivalent to the original equation, so the graph is not symmetric with respect to the origin.
so the graph is not symmetric with respect to the x-axis. 13. y
Test algebraically for symmetry with respect to the y-axis:
5
5y = 4x + 5 Original equation 4
1
5y = 4(−x) + 5 Replacing x by −x y= —
x
3
2
5y = −4x + 5 Simplifying 1

x
The last equation is not equivalent to the original equation, —5 —4 —3 —2 —1
—1
1 2 3 4 5

so the graph is not symmetric with respect to the y-axis. —2


—3
Test algebraically for symmetry with respect to the origin: —4
5y = 4x + 5 Original equation —5

5(−y) = 4(−x) + 5 Replacing x by −x


and The graph is not symmetric with respect to the x-axis or
y by −y the y-axis. It is symmetric with respect to the origin.
−5y = −4x + 5 Simplifying Test algebraically for symmetry with respect to the x-axis:
5y = 4x − 5 1
y= Original equation
x
The last equation is not equivalent to the original equation,
so the graph is not symmetric with respect to the origin. 1
−y = Replacing y by −y
x
11. y 1
y=− Simplifying
5
x
4 The last equation is not equivalent to the original equation,
3 so the graph is not symmetric with respect to the x-axis.
2
1 Test algebraically for symmetry with respect to the y-axis:
—5 —4 —3 —2 —1 1 2 3 4 5 x 1
—1 y= Original equation
—2 5y = 2x 2 – 3
x
1
Replacing x by −x
—3
y=
—4 −x
—5
1
y=− Simplifying
The graph is symmetric with respect to the y-axis. It is x
not symmetric with respect to the x-axis or the origin. The last equation is not equivalent to the original equation,
so the graph is not symmetric with respect to the y-axis.
Test algebraically for symmetry with respect to the x-axis:
Test algebraically for symmetry with respect to the origin:
5y = 2x2 − 3 Original equation
1
5(−y) = 2x2 − 3 Replacing y by −y y= Original equation
x
−5y = 2x2 − 3 Simplifying 1
−y = Replacing x by −x and y by −y
5y = −2x + 3 2 −x
The last equation is not equivalent to the original equation, 1
y= Simplifying
so the graph is not symmetric with respect to the x-axis. x
Test algebraically for symmetry with respect to the y-axis: The last equation is equivalent to the original equation, so
the graph is symmetric with respect to the origin.
5y = 2x2 − 3 Original equation
5y = 2(−x) − 3 2
Replacing x by −x 15. Test for symmetry with respect to the x-axis:
5y = 2x − 3
2 5x − 5y = 0 Original equation
The last equation is equivalent to the original equation, so 5x − 5(−y) = 0 Replacing y by −y
the graph is symmetric with respect to the y-axis. 5x + 5y = 0 Simplifying

Copyright © 2013 Pearson Education, Inc.


78 Chapter 2: More on Functions

The last equation is not equivalent to the original equation, Test for symmetry with respect to the origin:
so the graph is not symmetric with respect to the x-axis. y = |2x| Original equation
Test for symmetry with respect to the y-axis: −y = |2(−x)| Replacing x by −x and y by −y
5x − 5y = 0 Original equation −y = | − 2x| Simplifying
5(−x) − 5y = 0 Replacing x by −x −y = |2x|
−5x − 5y = 0 Simplifying y = −|2x|
5x + 5y = 0 The last equation is not equivalent to the original equation,
The last equation is not equivalent to the original equation, so the graph is not symmetric with respect to the origin.
so the graph is not symmetric with respect to the y-axis.
21. Test for symmetry with respect to the x-axis:
Test for symmetry with respect to the origin: 2x4 + 3 = y 2 Original equation
5x − 5y = 0 Original equation 4
2x + 3 = (−y) 2
Replacing y by −y
5(−x) − 5(−y) = 0 Replacing x by −x and
2x4 + 3 = y 2 Simplifying
y by −y
The last equation is equivalent to the original equation, so
−5x + 5y = 0 Simplifying
the graph is symmetric with respect to the x-axis.
5x − 5y = 0
Test for symmetry with respect to the y-axis:
The last equation is equivalent to the original equation, so
2x4 + 3 = y 2 Original equation
the graph is symmetric with respect to the origin.
4
2(−x) + 3 = y 2
Replacing x by −x
17. Test for symmetry with respect to the x-axis: 2x4 + 3 = y 2 Simplifying
3x2 − 2y 2 = 3 Original equation The last equation is equivalent to the original equation, so
3x2 − 2(−y)2 = 3 Replacing y by −y the graph is symmetric with respect to the y-axis.
3x2 − 2y 2 = 3 Simplifying Test for symmetry with respect to the origin:
The last equation is equivalent to the original equation, so 2x4 + 3 = y 2 Original equation
the graph is symmetric with respect to the x-axis. 4
2(−x) + 3 = (−y) 2
Replacing x by −x
Test for symmetry with respect to the y-axis: and y by −y
3x2 − 2y 2 = 3 Original equation 2x4 + 3 = y 2 Simplifying
3(−x) − 2y = 3
2 2
Replacing x by −x The last equation is equivalent to the original equation, so
3x2 − 2y 2 = 3 Simplifying the graph is symmetric with respect to the origin.
The last equation is equivalent to the original equation, so 23. Test for symmetry with respect to the x-axis:
the graph is symmetric with respect to the y-axis.
3y 3 = 4x3 + 2 Original equation
Test for symmetry with respect to the origin: 3
3(−y) = 4x + 2 3
Replacing y by −y
3x2 − 2y 2 = 3 Original equation
−3y 3 = 4x3 + 2 Simplifying
3(−x)2 − 2(−y)2 = 3 Replacing x by −x
3y = −4x − 2
3 3
and y by −y
The last equation is not equivalent to the original equation,
3x2 − 2y 2 = 3 Simplifying
so the graph is not symmetric with respect to the x-axis.
The last equation is equivalent to the original equation, so
Test for symmetry with respect to the y-axis:
the graph is symmetric with respect to the origin.
3y 3 = 4x3 + 2 Original equation
19. Test for symmetry with respect to the x-axis:
3y 3 = 4(−x)3 + 2 Replacing x by −x
y = |2x| Original equation
3y 3 = −4x3 + 2 Simplifying
−y = |2x| Replacing y by −y
The last equation is not equivalent to the original equation,
y = −|2x| Simplifying so the graph is not symmetric with respect to the y-axis.
The last equation is not equivalent to the original equation, Test for symmetry with respect to the origin:
so the graph is not symmetric with respect to the x-axis.
3y 3 = 4x3 + 2 Original equation
Test for symmetry with respect to the y-axis: 3
3(−y) = 4(−x) + 2 3
Replacing x by −x
y = |2x| Original equation and y by −y
y = |2(−x)| Replacing x by −x −3y 3 = −4x3 + 2 Simplifying
y = | − 2x| Simplifying 3y 3 = 4x3 − 2
y = |2x| The last equation is not equivalent to the original equation,
The last equation is equivalent to the original equation, so so the graph is not symmetric with respect to the origin.
the graph is symmetric with respect to the y-axis.

Copyright © 2013 Pearson Education, Inc.


Exercise Set 2.5 79

25. Test for symmetry with respect to the x-axis: 45. f (x) = x − |x|
xy = 12 Original equation f (−x) = (−x) − |(−x)| = −x − |x|
x(−y) = 12 Replacing y by −y −f (x) = −(x − |x|) = −x + |x|
−xy = 12 Simplifying f (x) = f (−x), so f is not even.
xy = −12 f (−x) = −f (x), so f is not odd.
The last equation is not equivalent to the original equation, Thus, f (x) = x − |x| is neither even nor odd.
so the graph is not symmetric with respect to the x-axis.
47. f (x) = 8
Test for symmetry with respect to the y-axis:
xy = 12 Original equation f (−x) = 8

−xy = 12 Replacing x by −x f (x) = f (−x), so f is even.

xy = −12 Simplifying 49. y

The last equation is not equivalent to the original equation, 4


so the graph is not symmetric with respect to the y-axis. 2
Test for symmetry with respect to the origin:
4 2 2 4 x
xy = 12 Original equation 2
−x(−y) = 12 Replacing x by −x and y by −y 4
xy = 12 Simplifying
The last equation is equivalent to the original equation, so √
51. f (x) = x 10 − x2
the graph is symmetric with respect to the origin.  √
f (−x) = −x 10 − (−x)2 = −x 10 − x2
27. x-axis: Replace y with −y; (−5, −6) √
−f (x) = −x 10 − x2
y-axis: Replace x with −x; (5, 6)
Since f (−x) = −f (x), f is odd.
Origin: Replace x with −x and y with −y; (5, −6)
29. x-axis: Replace y with −y; (−10, 7) 53. If the graph were folded on the x-axis, the parts above and
below the x-axis would coincide, so the graph is symmetric
y-axis: Replace x with −x; (10, −7)
with respect to the x-axis.
Origin: Replace x with −x and y with −y; (10, 7)
If the graph were folded on the y-axis, the parts to the left
31. x-axis: Replace y with −y; (0, 4) and right of the y-axis would not coincide, so the graph is
not symmetric with respect to the y-axis.
y-axis: Replace x with −x; (0, −4)
If the graph were rotated 180◦ , the resulting graph would
Origin: Replace x with −x and y with −y; (0, 4)
not coincide with the original graph, so it is not symmetric
33. The graph is symmetric with respect to the y-axis, so the with respect to the origin.
function is even.
55. See the answer section in the text.
35. The graph is symmetric with respect to the origin, so the
57. a), b) See the answer section in the text.
function is odd.
59. Let f (x) and g(x) be even functions. Then by definition,
37. The graph is not symmetric with respect to either the y-
f (x) = f (−x) and g(x) = g(−x). Thus, (f + g)(x) =
axis or the origin, so the function is neither even nor odd.
f (x) + g(x) = f (−x) + g(−x) = (f + g)(−x) and f + g is
39. f (x) = −3x3 + 2x even. The statement is true.
f (−x) = −3(−x)3 + 2(−x) = 3x3 − 2x
−f (x) = −(−3x3 + 2x) = 3x3 − 2x Exercise Set 2.5
f (−x) = −f (x), so f is odd.
1. Shift the graph of f (x) = x2 right 3 units.
41. f (x) = 5x2 + 2x4 − 1
f (−x) = 5(−x)2 + 2(−x)4 − 1 = 5x2 + 2x4 − 1 y

f (x) = f (−x), so f is even.


4

17 2
43. f (x) = x
f (−x) = (−x)17 = −x17 4 2 2 4 x
2
−f (x) = −x 17
4
f(x)  (x  3)2
f (−x) = −f (x), so f is odd.

Copyright © 2013 Pearson Education, Inc.


80 Chapter 2: More on Functions

3. Shift the graph of g(x) = x down 3 units. 11. First shrink the graph of h(x) = |x| vertically by multiply-
1
y
ing each y-coordinate by . Then shift it down 2 units.
2
y
4

2
4 1
h(x)  |x|
2 2
4 2 2 4 x 2
2
4 2 4 x
g(x)  x  3 2

4

5. Reflect the graph of h(x) = x across the x-axis.

y 13. Shift the graph of g(x) = x3 right 2 units and reflect it


across the x-axis.
4
h(x)  √x y
2

4
4 2 2 4 x g(x)  (x  2)3
2 2

4
4 2 2 4 x
2
1 4
7. Shift the graph of h(x) = up 4 units.
x
y 15. Shift the graph of g(x) = x2 left 1 unit and down 1 unit.
8
y
6

4 4
1
2 h(x)  
x 4
2

4 2 2 4 x 4 2 2 4 x
2 2

4
g(x)  (x  1)2  1
9. First stretch the graph of h(x) = x vertically by multi-
plying each y-coordinate by 3. Then reflect it across the
x-axis and shift it up 3 units. 17. First shrink the graph of g(x) = x3 vertically by multiply-
1
ing each y-coordinate by . Then shift it up 2 units.
y 3

h(x)  3x  3 y

2
4
4 2 2 4 x 2
2
4 2 4 x
4
2
1 3
4
g(x)  x
3 2

Copyright © 2013 Pearson Education, Inc.


Exercise Set 2.5 81


19. Shift the graph of f (x) = x left 2 units. 35. Think of the graph of f (x) = x2 . Since h(x) = −f (x−3)+
5, the graph of h(x) = −(x − 3)2 + 5 is the graph of f (x) =
y x2 shifted right 3 units, reflected across the x-axis, and
shifted up 5 units.
4
37. The graph of y = g(x) is the graph of y = f (x) shrunk
2
1
vertically by a factor of . Multiply the y-coordinate by
4 2 2 4 x 2
1
2 : (−12, 2).
f(x)  √
x 2 2
4
39. The graph of y = g(x) is the graph of y = f (x) reflected
√ across the y-axis, so we reflect the point across the y-axis:
3
21. Shift the graph of f (x) = x down 2 units. (12, 4).

y 41. The graph of y = g(x) is the graph of y = f (x) shifted


down 2 units. Subtract 2 from the y-coordinate: (−12, 2).
4 3
f(x)  √x  2 43. The graph of y = g(x) is the graph of y = f (x) stretched
2 vertically by a factor of 4. Multiply the y-coordinate by 4:
(−12, 16).
4 2 2 x
2 45. g(x) = x2 + 4 is the function f (x) = x2 + 3 shifted up
4 1 unit, so g(x) = f (x) + 1. Answer B is correct.
47. If we substitute x − 2 for x in f , we get (x − 2)3 + 3, so
23. Think of the graph of f (x) = |x|. Since g(x) = f (x − 2). Answer A is correct.
g(x) = f (3x), the graph of g(x) = |3x| is the graph
of f (x) = |x| shrunk horizontally by dividing each x- 49. Shape: h(x) = x2
 1 Turn h(x) upside-down (that is, reflect it across the x-
coordinate by 3 or multiplying each x-coordinate by .
3 axis): g(x) = −h(x) = −x2
1 Shift g(x) right 8 units: f (x) = g(x − 8) = −(x − 8)2
25. Think of the graph of f (x) = . Since h(x) = 2f (x),
x
2 1 51. Shape: h(x) = |x|
the graph of h(x) = is the graph of f (x) = stretched
x x Shift h(x) left 7 units: g(x) = h(x + 7) = |x + 7|
vertically by multiplying each y-coordinate by 2.
√ Shift g(x) up 2 units: f (x) = g(x) + 2 = |x + 7| + 2
√ = x. Since f (x) = 3g(x) −√5,
27. Think of the graph of g(x)
1
the graph of f (x) = 3 x − 5 is the graph of g(x) = x 53. Shape: h(x) =
stretched vertically by multiplying each y-coordinate by 3 x
and then shifted down 5 units. 1 
Shrink h(x) vertically by a factor of that is,
2
29. Think of the graph of f (x) = |x|. Since g(x) = 1
    multiply each function value by :
1 1  2
f x − 4, the graph of g(x) =  x − 4 is the graph of 1 1 1 1
3 3 g(x) = h(x) = · , or
2 2 x 2x
f (x) = |x| stretched horizontally by multiplying each x- 1
coordinate by 3 and then shifted down 4 units. Shift g(x) down 3 units: f (x) = g(x) − 3 = −3
2x
1
31. Think of the graph of g(x) = x2 . Since f (x) = − g(x − 5), 55. Shape: m(x) = x2
4
1 Turn m(x) upside-down (that is, reflect it across the x-
the graph of f (x) = − (x − 5) is the graph of g(x) = x2
2
4 axis): h(x) = −m(x) = −x2
shifted right 5 units, shrunk vertically by multiplying each
1 Shift h(x) right 3 units: g(x) = h(x − 3) = −(x − 3)2
y-coordinate by , and reflected across the x-axis.
4 Shift g(x) up 4 units: f (x) = g(x) + 4 = −(x − 3)2 + 4
1 √
33. Think of the graph of g(x) = . Since f (x) = 57. Shape: m(x) = x
x √
1 Reflect m(x) across the y-axis: h(x) = m(−x) = −x
g(x + 3) + 2, the graph of f (x) = + 2 is the graph 
x+3 Shift h(x) left 2 units: g(x) = h(x + 2) = −(x + 2)
1
of g(x) = shifted left 3 units and up 2 units.
x Shift g(x) down 1 unit: f (x) = g(x) − 1 =

−(x + 2) − 1

Copyright © 2013 Pearson Education, Inc.


82 Chapter 2: More on Functions

59. Each y-coordinate is multiplied by −2. We plot and con- h(x)  g(x  2)  1
y
nect (−4, 0), (−3, 4), (−1, 4), (2, −6), and (5, 0). 4 (3, 3)
(9, 1) (2, 1) 2 (1, 0)
y (3, 0)
(⫺1, 4) 10 8 6 4 2 2 4 6 x
(⫺3, 4) 4 g(x) ⫽ ⫺2f(x) 2
(7, 3) (4, 3) 4 (0, 3)
(⫺4, 0) (5, 0)
⫺4 ⫺2 2 4 6 x 6 (5, 5)
⫺2
⫺4 69. The graph is shrunk horizontally. The x-coordinates of
⫺6
y = h(x) are one-half the corresponding x-coordinates of
(2, ⫺6)
y = g(x).
61. The graph is reflected across the y-axis and stretched hor- y
izontally by a factor of 2. That is, each x-coordinate is h(x)  g (2x)
 1
8
multiplied by −2 or divided by − . We plot and con- (1, 4) 6 (,
7
2 6)
2 (1, 4)
nect (8, 0), (6, −2), (2, −2), (−4, 3), and (−10, 0). (,
5
2 4) 4
1
7 2 (,
2 1)
y (,
2 0)
4 2 2 4 x
(⫺4, 3) 4 g (x) ⫽ f 冢⫺qx冣 1 2
(,
2 1) 5
(⫺10, 0) 2
4 2 2)
(,
(8, 0)
(0, 0)
⫺10 ⫺8 ⫺6 ⫺4 ⫺2 2 4 6 8 x
⫺2 (6, ⫺2)
(2, ⫺2) 71. g(x) = f (−x) + 3
⫺4
The graph of g(x) is the graph of f (x) reflected across the
y-axis and shifted up 3 units. This is graph (f).
63. The graph is shifted right 1 unit so each x-coordinate is 73. g(x) = −f (x) + 3
increased by 1. The graph is also reflected across the x-
The graph of g(x) is the graph of f (x) reflected across the
axis, shrunk vertically by a factor of 2, and shifted up 3
1 x-axis and shifted up 3 units. This is graph (f).
units. Thus, each y-coordinate is multiplied by − and
2 1
then increased by 3. We plot and connect (−3, 3), (−2, 4), 75. g(x) = f (x − 2)
3
(0, 4), (3, 1.5), and (6, 3).
The graph of g(x)  is the graph of f (x) shrunk vertically
y by a factor of 3 that is, each y-coordinate is multiplied
6
(0, 4) 1
(⫺2, 4) by and then shifted right 2 units. This is graph (d).
(1, 3) (6, 3) 3
(⫺3, 3) 2 1
(3, 1.5) 77. g(x) = f (x + 2)
⫺4 ⫺2 2 4 6 x 3
⫺2 The graph of g(x)  is the graph of f (x) shrunk vertically
g (x) ⫽ ⫺qf (x ⫺ 1) ⫹ 3
⫺4 by a factor of 3 that is, each y-coordinate is multiplied
1
by and then shifted left 2 units. This is graph (c).
65. The graph is reflected across the y-axis so each 3
x-coordinate is replaced by its opposite. 79. f (−x) = 2(−x)4 − 35(−x)3 + 3(−x) − 5 =
2x4 + 35x3 − 3x − 5 = g(x)
y
(2, 3) 4 g(x)  f (x ) 81. The graph of f (x) = x3 − 3x2 is shifted up 2 units. A
2 formula for the transformed function is g(x) = f (x) + 2,
(5, 0) (4, 0) or g(x) = x3 − 3x2 + 2.
6 4 2 2 4 6 x
2
(3, 2) 83. The graph of f (x) = x3 − 3x2 is shifted left 1 unit. A
4 (1, 2) formula for the transformed function is k(x) = f (x + 1),
or k(x) = (x + 1)3 − 3(x + 1)2 .
67. The graph is shifted left 2 units so each x-coordinate is 85. Test for symmetry with respect to the x-axis.
decreased by 2. It is also reflected across the x-axis so each
y-coordinate is replaced with its opposite. In addition, the y = 3x4 − 3 Original equation
graph is shifted up 1 unit, so each y-coordinate is then −y = 3x − 34
Replacing y by −y
increased by 1. y = −3x4 + 3 Simplifying

Copyright © 2013 Pearson Education, Inc.


Exercise Set 2.5 83

The last equation is not equivalent to the original equation, Check. 6% of $83.3 billion is 0.06($83.3 billion) =
so the graph is not symmetric with respect to the x-axis. $4.998 billion ≈ $5 billion. (Remember that we rounded
Test for symmetry with respect to the y-axis. the value of g.) The answer checks.
y = 3x4 − 3 Original equation State. About $83.3 billion was spent on gift cards.
y = 3(−x) − 3 Replacing x by −x
4
91. Each point for which f (x) < 0 is reflected across the x-axis.
y = 3x4 − 3 Simplifying
y
The last equation is equivalent to the original equation, so
the graph is symmetric with respect to the y-axis. 4
(1, 2) (1, 2)
Test for symmetry with respect to the origin: 2 (4, 2)
(4, 0)
y = 3x − 3
4
4 2 2 4 x
−y = 3(−x)4 − 3 Replacing x by −x and 2
y by −y 4 |f(x)|
−y = 3x4 − 3
y = −3x4 + 3 Simplifying
93. The graph of y = g(|x|) consists of the points of y = g(x)
The last equation is not equivalent to the original equation, for which x ≥ 0 along with their reflections across the
so the graph is not symmetric with respect to the origin. y-axis.
87. Test for symmetry with respect to the x-axis: y
2x − 5y = 0 Original equation
4
2x − 5(−y) = 0 Replacing y by −y
(2, 1) 2 (2, 1)
2x + 5y = 0 Simplifying (4, 0) (4, 0)
4 2 2 4 x
The last equation is not equivalent to the original equation,
2
so the graph is not symmetric with respect to the x-axis.
4 g(|x|)
Test for symmetry with respect to the y-axis:
2x − 5y = 0 Original equation
2(−x) − 5y = 0 Replacing x by −x 95. Think of the graph of g(x) = int(x). Since
   
−2x − 5y = 0 Simplifying 1 1
f (x) = g x − , the graph of f (x) = int x − is the
2 2
The last equation is not equivalent to the original equation, 1
so the graph is not symmetric with respect to the y-axis. graph of g(x) = int(x) shifted right unit. The domain
2
Test for symmetry with respect to the origin: is the set of all real numbers; the range is the set of all
integers.
2x − 5y = 0 Original equation
2(−x) − 5(−y) = 0 Replacing x by −x and
y by −y
−2x + 5y = 0
2x − 5y = 0 Simplifying
The last equation is equivalent to the original equation, so
the graph is symmetric with respect to the origin.

89. Familiarize. Let g = the total amount spent on gift cards,


in billions of dollars. 97. On the graph of y = 2f (x) each y-coordinate of y = f (x) is
Translate. multiplied by 2, so (3, 4 · 2), or (3, 8) is on the transformed
$5 graph.
 billion


is 6% of total amount spent


    

On the graph of y = 2+f (x), each y-coordinate of y = f (x)


is increased by 2 (shifted up 2 units), so (3, 4 + 2), or (3, 6)
5 = 0.06 · g
is on the transformed graph.
Carry out. We solve the equation.
On the graph of y = f (2x), each x-coordinate of y =
5 = 0.06 · g 1 1
5 f (x) is multiplied by (or divided by 2), so · 3, 4 , or
=g   2 2
0.06 3
83.3 ≈ g 2
, 4 is on the transformed graph.

Copyright © 2013 Pearson Education, Inc.


84 Chapter 2: More on Functions

13. Let S = the sales tax and p = the purchase price.


Exercise Set 2.6 S = kp S varies directly as p.
17.50 = k · 260 Substituting
1. y = kx 0.067 ≈ k Variation constant
54 = k · 12
54 9 S = 0.067p Equation of variation
= k, or k =
12 2
9 S = 0.067(21) Substituting
The variation constant is , or 4.5. The equation of vari- S ≈ 1.41
2
9
ation is y = x, or y = 4.5x. The sales tax is $1.41.
2
k
k 15. W = W varies inversely as L.
3. y= L
x k
k 1200 = Substituting
3= 8
12 9600 = k Variation constant
36 = k
The variation constant is 36. The equation of variation is 9600
36 W = Equation of variation
y= . L
x 9600
W = Substituting
14
5. y = kx
W ≈ 686
1
1 = k·
4 A 14-m beam can support about 686 kg.
4=k
The variation constant is 4. The equation of variation is 17. Let F = the number of grams of fat and w = the weight.
y = 4x. F = kw F varies directly as w.
k 60 = k · 120 Substituting
7. y=
x 60
= k, or Solving for k
k 120
32 =
1 1
=k Variation constant
8 2
1
· 32 = k
8 1
F = w Equation of variation
4=k 2
The variation constant is 4. The equation of variation is 1
4 F = · 180 Substituting
y= . 2
x F = 90
9. y = kx
3 The maximum daily fat intake for a person weighing 180
= k·2 lb is 90 g.
4
1 3 k
· =k 19. T = T varies inversely as P .
2 4 P
3 k
=k 5= Substituting
8 7
3 35 = k Variation constant
The variation constant is . The equation of variation is
8
3
y = x. 35
8 T = Equation of variation
P
k 35
11. y= T = Substituting
x 10
k T = 3.5
1.8 =
0.3 It will take 10 bricklayers 3.5 hr to complete the job.
0.54 = k
The variation constant is 0.54. The equation of variation
0.54
is y = .
x

Copyright © 2013 Pearson Education, Inc.


Exercise Set 2.6 85

21. d = km d varies directly as m. 31. y = kxz 2


40 = k · 3 Substituting 105 = k · 14 · 52 Substituting
40 105 = 350k
=k Variation constant
3 105
=k
350
40
d= m Equation of variation 3
3 =k
10
40 200
d= ·5= Substituting The equation of variation is y =
3 2
xz .
3 3 10
2 xz
d = 66 33. y=k
3 wp
2 3 3 · 10
A 5-kg mass will stretch the spring 66 cm. =k Substituting
3 28 7·8
k
23. P = P varies inversely as W . 3 30
W = k·
28 56
k
330 = Substituting 3 56
3.2 · =k
28 30
1056 = k Variation constant 1
=k
5
1056
P = Equation of variation 1 xz xz
W The equation of variation is y = , or .
5 wp 5wp
1056 k
550 = Substituting 35. I = 2
W d
550W = 1056 Multiplying by W k
90 = 2 Substituting
5
1056 k
W = Dividing by 550 90 =
550 25
W = 1.92 Simplifying 2250 = k
A tone with a pitch of 550 vibrations per second has a 2250
The equation of variation is I = .
wavelength of 1.92 ft. d2
k Substitute 40 for I and find d.
25. y= 2 2250
x 40 = 2
d
k
0.15 = Substituting 40d2 = 2250
(0.1)2
d2 = 56.25
k
0.15 = d = 7.5
0.01
0.15(0.01) = k The distance from 5 m to 7.5 m is 7.5 − 5, or 2.5 m, so it is
2.5 m further to a point where the intensity is 40 W/m2 .
0.0015 = k
0.0015 37. d = kr2
The equation of variation is y = .
x2 200 = k · 602 Substituting
2
27. y = kx 200 = 3600k
0.15 = k(0.1)2 Substituting 200
=k
0.15 = 0.01k 3600
0.15 1
=k =k
0.01 18
1 2
15 = k The equation of variation is d = r .
18
The equation of variation is y = 15x2 . Substitute 72 for d and find r.
29. y = kxz 1 2
72 = r
18
56 = k · 7 · 8 Substituting 2
1296 = r
56 = 56k
36 = r
1=k A car can travel 36 mph and still stop in 72 ft.
The equation of variation is y = xz.

Copyright © 2013 Pearson Education, Inc.


86 Chapter 2: More on Functions

39. E =
kR 5. a) For x-values from −4 to −2, the y-values increase
I from 1 to 4. Thus the function is increasing on the
We first find k. interval (−4, −2).
k · 93
3.89 = Substituting b) For x-values from 2 to 5, the y-values decrease from
215.2 4 to 3. Thus the function is decreasing on the inter-

215.2 215.2 val (2, 5).
3.89 =k Multiplying by
93 93 c) For x-values from −2 to 2, y is 4. Thus the function
9≈k is constant on the interval (−2, 2).
9R 7. y
The equation of variation is E = .
I
5
Substitute 3.89 for E and 238 for I and solve for R. 4
9R 3
3.89 =
238 2
1
3.89(238) 238
=R Multiplying by –5 –4 –3 –2 –1 1 2 3 4 5 x
9 9 –1
f (x ) = x 2 – 1
103 ≈ R –2
–3
Bronson Arroyo would have given up about 103 earned –4

runs if he had pitched 238 innings. –5

41. parallel
The function is increasing on (0, ∞) and decreasing on
43. relative minimum (−∞, 0). We estimate that the minimum value is −1 at
x = 0. There are no maxima.
45. inverse variation
9.
47. Let V represent the volume and p represent the price of a
jar of peanut butter.
V = kp V varies directly as p.
 2
3
π (5) = k(2.89) Substituting
2
3.89π = k Variation constant
V = 3.89πp Equation of variation
We find that the function is increasing on (2, ∞) and de-
π(1.625)2 (5.5) = 3.89πp Substituting creasing on (−∞, 2). The relative minimum is −1 at x = 2.
3.73 ≈ p There are no maxima.
If cost is directly proportional to volume, the larger jar 11.
should cost $3.73.
Now let W represent the weight and p represent the price
of a jar of peanut butter.
W = kp
18 = k(2.89) Substituting
6.23 ≈ k Variation constant
W = 6.23p Equation of variation We find that the function is increasing on (−∞, −1.155)
28 = 6.23p Substituting and on (1.155, ∞) and decreasing on (−1.155, 1.155). The
relative maximum is 3.079 at x = −1.155 and the relative
4.49 ≈ p
minimum is −3.079 at x = 1.155.
If cost is directly proportional to weight, the larger jar
should cost $4.49. (Answers may vary slightly due to 13. If l = the length of the tablecloth, then the width is
rounding differences.) 20 − 2l
, or 10 − l. We use the formula Area = length ×
2
width.
Chapter 2 Review Exercises A(l) = l(10 − l), or
A(l) = 10l − l2
1. This statement is true by the definition of the greatest 15. a) If the length of the side parallel to the garage is
integer function. x feet long, then the length of each of the other
66 − x x
3. The graph of y = f (x − d) is the graph of y = f (x) shifted two sides is , or 33 − . We use the formula
2 2
right d units, so the statement is true. Area = length × width.

Copyright © 2013 Pearson Education, Inc.


Chapter 2 Review Exercises 87

 
A(x) = x 33 −
x
, or 21. f (x) = [[x − 3]]
2 This function could be defined by a piecewise function with
x2 an infinite number of statements.
A(x) = 33x − 
2 

 .
b) The length of the side parallel to the garage must 


 .
be positive and less than 66 ft, so the domain of the 

.

function is {x|0 < x < 66}, or (0, 66). 


 −4, for −1 ≤ x < 0,

 −3, for 0 ≤ x < 1,
x
c) y1 ⫽ x 冸33 ⫺ ⫺冹 f (x) = −2, for 1 ≤ x < 2,

2 

600 
 −1, for 2 ≤ x < 3,



 .



 .



.

0 80
0

d) By observing the graph or using the MAXIMUM


feature, we see that the maximum value of the func-
tion occurs when x = 33. When x = 33, then
x 33
33 − = 33 − = 33 − 16.5 = 16.5. Thus the di-
2 2
mensions that yield the maximum area are 33 ft by
16.5 ft.
  2
 −x, for x ≤ −4,  x − 1, for x = −1,
17. f (x) = 1 x + 1, for x > −4 23. f (x) = x+1
2 
3, for x = −1
We create the graph in two parts. Graph f (x) = −x for (−2)2 − 1 4−1 3
1 Since −2 = −1, f (−2) = = = = −3.
inputs less than or equal to −4. Then graph f (x) = x + 1 −2 + 1 −1 −1
2 Since x = −1, we have f (−1) = 3.
for inputs greater than −4.
02 − 1 −1
y Since 0 = −1, f (0) = = = −1.
0+1 1
4 42 − 1 16 − 1 15
Since 4 = −1, f (4) = = = = 3.
2 4+1 5 5
25. (f g)(2) = f (2) · g(2)
4 2 2 4 x √
2 = 2 − 2 · (22 − 1)
4 = 0 · (4 − 1)
=0
 2
 x − 1, for x = −1, 27. f (x) =
4
, g(x) = 3 − 2x
19. f (x) = x+1 x2
 a) Division by zero is undefined, so the domain of f is
3, for x = −1
{x|x = 0}, or (−∞, 0) ∪ (0, ∞). The domain of g is
x2 − 1 the set of all real numbers, or (−∞, ∞).
We create the graph in two parts. Graph f (x) =
x+1
for all inputs except −1. Then graph f (x) = 3 for x = −1. The domain of f + g, f − g and f g is {x|x = 0},
3
or (−∞, 0) ∪ (0, ∞). Since g = 0, the domain
y   2

 3
4 of f /g is xx = 0 and x = , or
   2
2 3 3
(−∞, 0) ∪ 0, ∪ ,∞ .
4 2 2 4 2 2
x
 
4 4
4 b) (f + g)(x) = + (3 − 2x) = 2 + 3 − 2x
x2 x
 
4 4
(f − g)(x) = − (3 − 2x) = 2 − 3 + 2x
x2 x

Copyright © 2013 Pearson Education, Inc.


88 Chapter 2: More on Functions

 
4 12 8 45. x2 + y 2 = 4
(f g)(x) = (3 − 2x) = 2 −
x2 x x
  y
4
x2 4 5
(f /g)(x) = = 2 4
(3 − 2x) x (3 − 2x)
3 x2 + y2 = 4
29. P (x) = R(x) − C(x) 2
1
= (120x − 0.5x2 ) − (15x + 6) x
-5 -4 -3 -2 -1 1 2 3 4 5
= 120x − 0.5x − 15x − 6
2 -1
-2
= −0.5x2 + 105x − 6 -3
-4
31. f (x) = 3 − x2 -5

f (x + h) = 3 − (x + h)2 = 3 − (x2 + 2xh + h2 ) =


3 − x2 − 2xh − h2 The graph is symmetric with respect to the x-axis, the
f (x + h) − f (x) 3 − x2 − 2xh − h2 − (3 − x2 ) y-axis, and the origin.
=
h h Replace y with −y to test algebraically for symmetry with
respect to the x-axis.
3 − x − 2xh − h − 3 + x
2 2 2
= x2 + (−y)2 = 4
h
−2xh − h2 h(−2x − h) x2 + y 2 = 4
= = The resulting equation is equivalent to the original equa-
h h
h −2x − h tion, so the graph is symmetric with respect to the x-axis.
= · = −2x − h Replace x with −x to test algebraically for symmetry with
h 1
respect to the y-axis.
33. (f ◦ g)(1) = f (g(1)) = f (1 + 4) = f (1 + 4) = f (5) =
2
(−x)2 + y 2 = 4
2 · 5 − 1 = 10 − 1 = 9
x2 + y 2 = 4
35. (h ◦ f )(−2) = h(f (−2)) = h(2(−2) − 1) =
The resulting equation is equivalent to the original equa-
h(−4 − 1) = h(−5) = 3 − (−5)3 = 3 − (−125) = tion, so the graph is symmetric with respect to the y-axis.
3 + 125 = 128 Replace x and −x and y with −y to test for symmetry
with respect to the origin.
37. (f ◦ h)(−1) = f (h(−1)) = f (3 − (−1) ) = 3
(−x)2 + (−y)2 = 4
f (3 − (−1)) = f (3 + 1) = f (4) = 2 · 4 − 1 = 8 − 1 = 7
x2 + y 2 = 4
39. (f ◦ f )(x) = f (f (x)) = f (2x − 1) = 2(2x − 1) − 1 =
The resulting equation is equivalent to the original equa-
4x − 2 − 1 = 4x − 3
tion, so the graph is symmetric with respect to the origin.
4
41. a) f ◦ g(x) = f (3 − 2x) =
(3 − 2x)2 47. x + y = 3
   
4 4 8
g ◦ f (x) = g 2 = 3 − 2 2 = 3 − 2 y

x x x 5
x+y=3
b) The domain of f is {x|x = 0} and the domain of g 4
3
is the set of all real numbers. To find the domain
2
of f ◦ g, we find the values of x for which g(x) = 0. 1
3
Since 3 − 2x = 0 when x = , the domain of f ◦ g –5 –4 –3 –2 –1 1 2 3 4 5 x
    2   –1
 3 3 3

is xx = , or − ∞, ∪ , ∞ . Since any
–2
–3
2 2 2
–4
real number can be an input for g, the domain of
–5
g ◦ f is the same as the domain of f , {x|x = 0}, or
(−∞, 0) ∪ (0, ∞).
The graph is not symmetric with respect to the x-axis, the
√ y-axis, or the origin.
43. f (x) = x, g(x) = 5x + 2. Answers may vary.
Replace y with −y to test algebraically for symmetry with
respect to the x-axis.
x−y =3
The resulting equation is not equivalent to the original
equation, so the graph is not symmetric with respect to
the x-axis.

Copyright © 2013 Pearson Education, Inc.


Chapter 2 Review Exercises 89

Replace x with −x to test algebraically for symmetry with 53. The graph is symmetric with respect to the origin, so the
respect to the y-axis. function is odd.
−x + y = 3 55. f (x) = 9 − x2
The resulting equation is not equivalent to the original f (−x) = 9 − (−x2 ) = 9 − x2
equation, so the graph is not symmetric with respect to
the y-axis. f (x) = f (−x), so f is even.

Replace x and −x and y with −y to test for symmetry 57. f (x) = x7 − x5


with respect to the origin. f (−x) = (−x)7 − (−x)5 = −x7 + x5
−x − y = 3 f (x) = f (−x), so f is not even.
x + y = −3 −f (x) = −(x7 − x5 ) = −x7 + x5
The resulting equation is not equivalent to the original f (−x) = −f (x), so f is odd.
equation, so the graph is not symmetric with respect to √
the origin. 59. f (x) = 16 − x2
 √
49. y = x3 f (−x) = 16 − (−x2 ) = 16 − x2
f (x) = f (−x), so f is even.
y

5 61. Shape: g(x) = x2


4
3 y = x3 Shift g(x) left 3 units: f (x) = g(x + 3) = (x + 3)2
2
1 63. Shape: h(x) = |x|
–5 –4 –3 –2 –1 1 2 3 4 5 x
Stretch h(x) vertically by a factor of 2 (that is, multiply
–1
–2 each function value by 2): g(x) = 2h(x) = 2|x|.
–3
–4
Shift g(x) right 3 units: f (x) = g(x − 3) = 2|x − 3|.
–5
65. The graph is shrunk horizontally by a factor of 2. That
is, each x-coordinate
   is divided by 2. We plot and connect
The graph is symmetric with respect to the origin. It is 5 3
not symmetric with respect to the x-axis or the y-axis. − , 3 , − , 0 , (0, 1) and (2, −2).
2 2
Replace y with −y to test algebraically for symmetry with
respect to the x-axis.
−y = x3
y = −x3
The resulting equation is not equivalent to the original
equation, so the graph is not symmetric with respect to
the x-axis.
Replace x with −x to test algebraically for symmetry with
respect to the y-axis.
67. Each y-coordinate is increased by 3. We plot and connect
y = (−x)3
(−5, 6), (−3, 3), (0, 4) and (4, 1).
y = −x3
The resulting equation is not equivalent to the original
equation, so the graph is not symmetric with respect to
the y-axis.
Replace x and −x and y with −y to test for symmetry
with respect to the origin.
−y = (−x)3
−y = −x3
y = x3
69. y = kx
The resulting equation is equivalent to the original equa-
tion, so the graph is symmetric with respect to the origin. 6 = 9x
2
51. The graph is symmetric with respect to the y-axis, so the = x Variation constant
3
function is even. 2
Equation of variation: y = x
3

Copyright © 2013 Pearson Education, Inc.


90 Chapter 2: More on Functions

71. k 85. If all of the exponents are even numbers, then f (x) is an
y=
x even function. If a0 = 0 and all of the exponents are odd
k numbers, then f (x) is an odd function.
6=
9 k2 k2 k1 k2
54 = k Variation constant 87. Let y = k1 x and x = . Then y = k1 · , or y = ,
z z z
54 so y varies inversely as z.
Equation of variation: y =
x

73. kxz 2 Chapter 2 Test


y=
w  
2
1
k(16)
2 1. a) For x-values from −5 to −2, the y-values increase
2= from −4 to 3. Thus the function is increasing on
0.2
 
1 the interval (−5, −2).
k(16)
4 b) For x-values from 2 to 5, the y-values decrease from
2=
0.2 2 to −1. Thus the function is decreasing on the
4k interval (2, 5).
2=
0.2 c) For x-values from −2 to 2, y is 2. Thus the function
2= 20k is constant on the interval (−2, 2).
1
=k 2. y
10
1 xz 2 5
y= 4
10 w 3 f (x ) = 2 – x 2
2
75. N = ka 1

87 = k · 29 –5 –4 –3 –2 –1 1 2 3 4 5 x
–1
3=k –2
–3
N = 3a –4
–5

N = 3 · 25
N = 75 The function is increasing on (−∞, 0) and decreasing on
(0, ∞). The relative maximum is 2 at x = 0. There are no
Ellen’s score would have been 75 if she had answered 25 minima.
questions correctly.
√ 3.
77. f (x) = x + 1, g(x) = x
The domain of f is (−∞, ∞), and the domain of g is [0, ∞).
To find the domain of (g ◦ f )(x), we find the values of x
for which f (x) ≥ 0.
x+1 ≥ 0
x ≥ −1
Thus the domain of (g ◦ f )(x) is [−1, ∞). Answer A is
We find that the function is increasing on (−∞, −2.667)
correct.
and on (0, ∞) and decreasing on (−2.667, 0). The relative
1
79. The graph of g(x) = − f (x) + 1 is the graph of y = f (x) maximum is 9.481 at −2.667 and the relative minimum is
2 0 at x = 0.
1
shrunk vertically by a factor of , then reflected across the
2 4. If b = the length of the base, in inches, then the height =
x-axis, and shifted up 1 unit. The correct graph is B. 4b − 6. We use the formula for the area of a triangle,
1
81. Reflect the graph of y = f (x) across the x-axis and then A = bh.
2
across the y-axis.
1
A(b) = b(4b − 6), or
83. In the graph of y = f (cx), the constant c stretches or 2
shrinks the graph of y = f (x) horizontally. The constant A(b) = 2b2 − 3b
c in y = cf (x) stretches or shrinks the graph of y = f (x)
vertically. For y = f (cx), the x-coordinates of y = f (x) are
divided by c; for y = cf (x), the y-coordinates of y = f (x)
are multiplied by c.

Copyright © 2013 Pearson Education, Inc.


Chapter 2 Test 91

 2

x , for x < −1, 1
21. f (x) = x+4
5. f (x) = |x|, for −1 ≤ x ≤ 1, 2

√ 1 1 1
x − 1, for x > 1 f (x + h) = (x + h) + 4 = x + h + 4
2 2 2
 
1 1 1
y x+ h+4− x+4
f (x + h) − f (x) 2 2 2
4 =
h h
2
1 1 1
x+ h+4− x−4
4 2 2 4 x = 2 2 2
2 h
4 1
h 1 1 1 h 1
= 2 = h· = · =
    h 2 h 2 h 2
7 7  7 7
6. Since −1 ≤ − ≤ 1, f − =  −  = . 22. f (x) = 2x2 − x + 3
8 8 8 8
√ √ f (x+h) = 2(x+h)2 −(x+h)+3 = 2(x2 +2xh+h2 )−x − h+3 =
Since 5 > 1, f (5) = 5 − 1 = 4 = 2.
2x2 + 4xh + 2h2 − x − h + 3
Since −4 < −1, f (−4) = (−4)2 = 16.
7. (f + g)(−6) = f (−6) + g(−6) = f (x+h)−f (x) 2x2+4xh+2h2−x−h + 3−(2x2 −x + 3)
=
 h h
(−6)2 − 4(−6) + 3 + 3 − (−6) =
√ √ 2x2 +4xh+2h2−x−h+3−2x2 +x−3
36 + 24 + 3 + 3 + 6 = 63 + 9 = 63 + 3 = 66 =
h
8. (f − g)(−1) = f (−1) − g(−1) = 4xh + 2h2 − h
 =
(−1)2 − 4(−1) + 3 − 3 − (−1) = h
√ √
1+4+3− 3+1=8− 4=8−2=6 h/(4x + 2h − 1)
=
√ h/
9. (f g)(2) = f (2) · g(2) = (22 − 4 · 2 + 3)( 3 − 2) =
√ = 4x + 2h − 1
(4 − 8 + 3)( 1) = −1 · 1 = −1
23. (g ◦ h)(2) = g(h(2)) = g(3 · 22 + 2 · 2 + 4) =
f (1) 1 −4·1+3
2
1−4+3 0 g(3 · 4 + 4 + 4) = g(12 + 4 + 4) = g(20) = 4 · 20 + 3 =
10. (f /g)(1) = = √ = √ = √ =0
g(1) 3−1 2 2 80 + 3 = 83
11. Any real number can be an input for f (x) = x2 , so the
24. (f ◦ g)(−1) = f (g(−1)) = f (4(−1) + 3) = f (−4 + 3) =
domain is the set of real numbers, or (−∞, ∞).
√ f (−1) = (−1)2 − 1 = 1 − 1 = 0
12. The domain of g(x) = x − 3 is the set of real numbers for
which x − 3 ≥ 0, or x ≥ 3. Thus the domain is {x|x ≥ 3}, 25. (h ◦ f )(1) = h(f (1)) = h(12 − 1) = h(1 − 1) = h(0) =
or [3, ∞). 3 · 02 + 2 · 0 + 4 = 0 + 0 + 4 = 4
13. The domain of f + g is the intersection of the domains of 26. (g ◦ g)(x) = g(g(x)) = g(4x + 3) = 4(4x + 3) + 3 =
f and g. This is {x|x ≥ 3}, or [3, ∞). 16x + 12 + 3 = 16x + 15
14. The domain of f − g is the intersection of the domains of √
27. (f ◦ g)(x) = f (g(x)) = f (x2 + 1) = x2 + 1 − 5 =
f and g. This is {x|x ≥ 3}, or [3, ∞). √
x2 − 4
15. The domain of f g is the intersection of the domains of f √ √
(g ◦ f )(x) = g(f (x)) = g( x − 5) = ( x − 5)2 + 1 =
and g. This is {x|x ≥ 3}, or [3, ∞).
x−5+1=x−4
16. The domain of f /g is the intersection of the domains of f
and g, excluding those x-values for which g(x) = 0. Since 28. The inputs for f (x) must be such that x − 5 ≥ 0, or x ≥ 5.
x − 3 = 0 when x = 3, the domain is (3, ∞). Then for (f ◦g)(x) we must have g(x) ≥ 5, or x2 +1 ≥ 5, or
√ x2 ≥ 4. Then the domain of (f ◦g)(x) is (−∞, −2]∪[2, ∞).
17. (f + g)(x) = f (x) + g(x) = x2 + x − 3 Since we can substitute any real number for x in g, the
√ domain of (g ◦ f )(x) is the same as the domain of f (x),
18. (f − g)(x) = f (x) − g(x) = x2 − x − 3
√ [5, ∞).
19. (f g)(x) = f (x) · g(x) = x2 x − 3
29. Answers may vary. f (x) = x4 , g(x) = 2x − 7
f (x) x2
20. (f /g)(x) = =√
g(x) x−3

Copyright © 2013 Pearson Education, Inc.


92 Chapter 2: More on Functions

k
30. y = x4 − 2x2 35. y=
x
Replace y with −y to test for symmetry with respect to k
the x-axis. 5=
6
−y = x4 − 2x2
30 = k Variation constant
y = −x4 + 2x2 30
The resulting equation is not equivalent to the original Equation of variation: y =
x
equation, so the graph is not symmetric with respect to
36. y = kx
the x-axis.
60 = k · 12
Replace x with −x to test for symmetry with respect to
the y-axis. 5=k Variation constant
y = (−x)4 − 2(−x)2 Equation of variation: y = 5x
y = x − 2x
4 2
kxz 2
37. y=
The resulting equation is equivalent to the original equa- w
tion, so the graph is symmetric with respect to the y-axis. k(0.1)(10)2
100 =
Replace x with −x and y with −y to test for symmetry 5
with respect to the origin. 100 = 2k
−y = (−x)4 − 2(−x)2 50 = k Variation constant
−y = x4 − 2x2 50xz 2
y= Equation of variation
y = −x4 + 2x2 w
The resulting equation is not equivalent to the original 38. d = kr2
equation, so the graph is not symmetric with respect to 200 = k · 602
the origin.
1
=k Variation constant
2x 18
31. f (x) =
x2 + 1 1 2
d= r Equation of variation
2(−x) 2x 18
f (−x) = =− 2
(−x)2 + 1 x +1 1
d= · 302
f (x) = f (−x), so f is not even. 18
2x d = 50 ft
−f (x) = − 2
x +1 39. The graph of g(x) = 2f (x) − 1 is the graph of y = f (x)
f (−x) = −f (x), so f is odd. stretched vertically by a factor of 2 and shifted down 1 unit.
The correct graph is C.
32. Shape: h(x) = x2
Shift h(x) right 2 units: g(x) = h(x − 2) = (x − 2)2 40. Each x-coordinate on the graph of y = f (x) is
 divided
 by
−3
Shift g(x) down 1 unit: f (x) = (x − 2) − 1
2 3 on the graph of y = f (3x). Thus the point , 1 , or
3
33. Shape: h(x) = x2 (−1, 1) is on the graph of f (3x).
Shift h(x) left 2 units: g(x) = h(x + 2) = (x + 2)2
Shift g(x) down 3 units: f (x) = (x + 2)2 − 3
1
34. Each y-coordinate is multiplied by − . We plot and con-
2
nect (−5, 1), (−3, −2), (1, 2) and (4, −1).

Copyright © 2013 Pearson Education, Inc.


Chapter 3
Quadratic Functions and Equations;
Inequalities
25. (6 − 4i) − (−5 + i)
Exercise Set 3.1 = [6 − (−5)] + (−4i − i)
√ √ √ √ √ √ = (6 + 5) + (−4i − i)
1. −3 = −1 · 3 = −1 · 3 = i 3, or 3i
√ √ √ √ = 11 − 5i
3. −25 = −1 · 25 = −1 · 25 = i · 5 = 5i
√ √ √ √ √ √ 27. (−5 + 2i) − (−4 − 3i)
5. − −33 = − −1 · 33 = − −1 · 33 = −i 33, or − 33i = [−5 − (−4)] + [2i − (−3i)]
√ √ √ √
7. − −81 = − −1 · 81 = − −1 · 81 = −i · 9 = −9i = (−5 + 4) + (2i + 3i)
√ √ √ √ √ = −1 + 5i
9. −98 = −1 · 98 = −1 · 98 = i 49 · 2 =
√ √ √ 29. (4 − 9i) − (2 + 3i)
i · 7 2 = 7i 2, or 7 2i
= (4 − 2) + (−9i − 3i)
11. (−5 + 3i) + (7 + 8i)
= 2 − 12i
= (−5 + 7) + (3i + 8i) Collecting the real parts √ √
and the imaginary parts 31. −4 · −36 = 2i · 6i = 12i2 = 12(−1) = −12
√ √
33. −81 · −25 = 9i · 5i = 45i2 = 45(−1) = −45
= 2 + (3 + 8)i
35. 7i(2 − 5i)
= 2 + 11i
= 14i − 35i2 Using the distributive law
13. (4 − 9i) + (1 − 3i)
= 14i + 35 i2 = −1
= (4 + 1) + (−9i − 3i) Collecting the real parts
= 35 + 14i Writing in the form a + bi
and the imaginary parts
37. −2i(−8 + 3i)
= 5 + (−9 − 3)i = 16i − 6i2 Using the distributive law
= 5 − 12i = 16i + 6 i2 = −1
15. (12 + 3i) + (−8 + 5i) = 6 + 16i Writing in the form a + bi
= (12 − 8) + (3i + 5i) 39. (1 + 3i)(1 − 4i)
= 4 + 8i = 1 − 4i + 3i − 12i2 Using FOIL
17. (−1 − i) + (−3 − i) = 1 − 4i + 3i − 12(−1) i2 = −1

= (−1 − 3) + (−i − i) = 1 − i + 12

= −4 − 2i = 13 − i
√ √ 41. (2 + 3i)(2 + 5i)
19. (3 + −16) + (2 + −25) = (3 + 4i) + (2 + 5i)
= 4 + 10i + 6i + 15i2 Using FOIL
= (3 + 2) + (4i + 5i)
= 4 + 10i + 6i − 15 i2 = −1
= 5 + 9i
= −11 + 16i
21. (10 + 7i) − (5 + 3i)
43. (−4 + i)(3 − 2i)
= (10 − 5) + (7i − 3i) The 5 and the 3i are
both being subtracted. = −12 + 8i + 3i − 2i2 Using FOIL
= 5 + 4i = −12 + 8i + 3i + 2 i2 = −1
23. (13 + 9i) − (8 + 2i) = −10 + 11i
= (13 − 8) + (9i − 2i) The 8 and the 2i are 45. (8 − 3i)(−2 − 5i)
both being subtracted. = −16 − 40i + 6i + 15i2
= 5 + 7i
= −16 − 40i + 6i − 15 i2 = −1
= −31 − 34i

Copyright © 2013 Pearson Education, Inc.


94 Chapter 3: Quadratic Functions and Equations; Inequalities

√ √
47. (3 + −16)(2 + −25) 63. (3 + 4i)2
= (3 + 4i)(2 + 5i) = 9 + 2 · 3 · 4i + (4i)2
= 6 + 15i + 8i + 20i 2 = 9 + 24i + 16i2
= 6 + 15i + 8i − 20 i2 = −1 = 9 + 24i − 16 i2 = −1
= −14 + 23i = −7 + 24i
49. (5 − 4i)(5 + 4i) = 52 − (4i)2 3
65.
= 25 − 16i 2 5 − 11i
i2 = −1 3 5 + 11i
= 25 + 16 = · 5 − 11i is the conjugate
5 − 11i 5 + 11i of 5 + 11i.
= 41
3(5 + 11i)
51. (3 + 2i)(3 − 2i) =
(5 − 11i)(5 + 11i)
= 9 − 6i + 6i − 4i2 15 + 33i
=
= 9 − 6i + 6i + 4 i2 = −1 25 − 121i2
= 13 15 + 33i
= i2 = −1
25 + 121
53. (7 − 5i)(7 + 5i)
15 + 33i
= 49 + 35i − 35i − 25i2 =
146
= 49 + 35i − 35i + 25 i = −1
2
15 33
= + i Writing in the form a + bi
= 74 146 146
55. (4 + 2i)2 5
67.
= 16 + 2 · 4 · 2i + (2i) 2
Recall (A + B) = 2 2 + 3i
A2 + 2AB + B 2 5 2 − 3i
= · 2 − 3i is the conjugate
= 16 + 16i + 4i2 2 + 3i 2 − 3i of 2 + 3i.
= 16 + 16i − 4 i2 = −1 5(2 − 3i)
=
= 12 + 16i (2 + 3i)(2 − 3i)
10 − 15i
57. (−2 + 7i)2 =
4 − 9i2
2 2 2
= (−2) + 2(−2)(7i) + (7i) Recall (A + B) = 10 − 15i
A2 + 2AB + B 2 = i2 = −1
4+9
= 4 − 28i + 49i 2
10 − 15i
= 4 − 28i − 49 i2 = −1 =
13
= −45 − 28i 10 15
= − i Writing in the form a + bi
59. (1 − 3i)2 13 13

= 12 − 2 · 1 · (3i) + (3i)2 4+i


69.
−3 − 2i
= 1 − 6i + 9i2
4+i −3 + 2i
= 1 − 6i − 9 i2 = −1 = · −3 + 2i is the conjugate
−3 − 2i −3 + 2i of the divisor.
= −8 − 6i (4 + i)(−3 + 2i)
=
61. (−1 − i) 2 (−3 − 2i)(−3 + 2i)
= (−1)2 − 2(−1)(i) + i2 −12 + 5i + 2i2
=
= 1 + 2i + i2 9 − 4i2
= 1 + 2i − 1 i2 = −1 −12 + 5i − 2
= i2 = −1
9+4
= 2i
−14 + 5i
=
13
14 5
=− + i Writing in the form a + bi
13 13

Copyright © 2013 Pearson Education, Inc.


Exercise Set 3.1 95

5 − 3i 4 − 2i 2 − 5i
71. 77. +
4 + 3i 1+i 1+i
5 − 3i 4 − 3i 6 − 7i
= · 4 − 3i is the conjugate = Adding
4 + 3i 4 − 3i of 4 + 3i. 1+i
(5 − 3i)(4 − 3i) 6 − 7i 1−i
= = · 1 − i is the conjugate
(4 + 3i)(4 − 3i) 1+i 1−i of 1 + i.
20 − 27i + 9i2 (6 − 7i)(1 − i)
= =
16 − 9i2 (1 + i)(1 − i)
20 − 27i − 9 6 − 13i + 7i2
= i2 = −1 =
16 + 9 1 − i2
11 − 27i 6 − 13i − 7
=
25 = i2 = −1
1+1
=
11 27
− i Writing in the form a + bi −1 − 13i
25 25 =
√ 2
2 + 3i 1 13
73. =− − i
5 − 4i 2 2

2 + 3i 5 + 4i
= · 5 + 4i is the conjugate 79. i11 = i10 · i = (i2 )5 · i = (−1)5 · i = −1 · i = −i
5 − 4i 5 + 4i of the divisor.

(2 + 3i)(5 + 4i) 81. i35 = i34 · i = (i2 )17 · i = (−1)17 · i = −1 · i = −i
=
(5 − 4i)(5 + 4i) 83. i64 = (i2 )32 = (−1)32 = 1
√ √
10 + 8i + 5 3i + 4 3i2
= 85. (−i)71 = (−1 · i)71 = (−1)71 · i71 = −i70 · i =
25 − 16i2
√ √ −(i2 )35 · i = −(−1)35 · i = −(−1)i = i
10 + 8i + 5 3i − 4 3
= i2 = −1
25 + 16 87. (5i)4 = 54 · i4 = 625(i2 )2 = 625(−1)2 = 625 · 1 = 625
√ √
10 − 4 3 + (8 + 5 3)i 89. First find the slope of the given line.
=
41 3x − 6y = 7
√ √
10 − 4 3 8 + 5 3 −6y = −3x + 7
= + i Writing in the
41 41 form a + bi 1 7
y = x−
2 6
1+i
75. 1
(1 − i)2 The slope is . The slope of the desired line is the oppo-
2
1+i 1
= site of the reciprocal of , or −2. Write a slope-intercept
1 − 2i + i2 2
equation of the line containing (3, −5) with slope −2.
1+i
= i2 = −1 y − (−5) = −2(x − 3)
1 − 2i − 1
1+i y + 5 = −2x + 6
=
−2i y = −2x + 1
1 + i 2i
= · 2i is the conjugate 91. The domain of f is the set of all real numbers as is the
−2i 2i of −2i. 5
domain of g. When x = − , g(x) = 0, so the domain of
 3 
5  5
(1 + i)(2i)
=
(−2i)(2i) f /g is − ∞, − ∪ − ,∞ .
3 3
2i + 2i2
= f (2) 22 + 4 4+4 8
−4i2 93. (f /g)(2) = = = =
g(2) 3·2+5 6+5 11
2i − 2
= i2 = −1
4 95. (a+bi)+(a−bi) = 2a, a real number. Thus, the statement
2 2 is true.
=− + i
4 4
1 1 97. (a + bi)(c + di) = (ac − bd) + (ad + bc)i. The conjugate of
=− + i the product is (ac − bd) − (ad + bc)i =
2 2
(a − bi)(c − di), the product of the conjugates of the indi-
vidual complex numbers. Thus, the statement is true.

99. zz = (a + bi)(a − bi) = a2 − b2 i2 = a2 + b2

Copyright © 2013 Pearson Education, Inc.


96 Chapter 3: Quadratic Functions and Equations; Inequalities

101. [x − (3 + 4i)][x − (3 − 4i)] 13. x2 + 16 = 0


= [x − 3 − 4i][x − 3 + 4i] x2 = −16
√ √
= [(x − 3) − 4i][(x − 3) + 4i] x = −16 or x = − −16
= (x − 3)2 − (4i)2 x = 4i or x = −4i
= x2 − 6x + 9 − 16i2 The solutions are 4i and −4i.
= x2 − 6x + 9 + 16 i2 = −1 15. 2x2 = 6x
= x − 6x + 25
2
2x2 − 6x = 0 Subtracting 6x on both sides
2x(x − 3) = 0
Exercise Set 3.2 2x = 0 or x − 3 = 0
x = 0 or x=3
1. (2x − 3)(3x − 2) = 0 The solutions are 0 and 3.
2x − 3 = 0 or 3x − 2 = 0 Using the principle
17. 3y 3 − 5y 2 − 2y = 0
of zero products
2x = 3 or 3x = 2 y(3y 2 − 5y − 2) = 0
3 2 y(3y + 1)(y − 2) = 0
x= or x=
2 3 y = 0 or 3y + 1 = 0 or y − 2 = 0
3 2 1
The solutions are and . y = 0 or y = − or y=2
2 3 3
3. x2 − 8x − 20 = 0 1
The solutions are − , 0 and 2.
(x − 10)(x + 2) = 0 Factoring 3
19. 7x3 + x2 − 7x − 1 = 0
x − 10 = 0 or x + 2 = 0 Using the principle
of zero products x2 (7x + 1) − (7x + 1) = 0
x = 10 or x = −2 (x2 − 1)(7x + 1) = 0
The solutions are 10 and −2. (x + 1)(x − 1)(7x + 1) = 0
5. 3x + x − 2 = 0
2 x+1 = 0 or x − 1 = 0 or 7x + 1 = 0
(3x − 2)(x + 1) = 0 1
Factoring x = −1 or x = 1 or x=−
7
3x − 2 = 0 or x + 1 = 0 Using the principle 1
of zero products The solutions are −1, − , and 1.
7
2
x= or x = −1 21. a) The graph crosses the x-axis at (−4, 0) and at (2, 0).
3
2 These are the x-intercepts.
The solutions are and −1.
3 b) The zeros of the function are the first coordinates of
the x-intercepts of the graph. They are −4 and 2.
7. 4x2 − 12 = 0
4x2 = 12 23. a) The graph crosses the x-axis at (−1, 0) and at (3, 0).
These are the x-intercepts.
x2 = 3
√ √ b) The zeros of the function are the first coordinates of
x = 3 or x = − 3 Using the principle the x-intercepts of the graph. They are −1 and 3.
of square roots
√ √ 25. a) The graph crosses the x-axis at (−2, 0) and at (2, 0).
The solutions are 3 and − 3. These are the x-intercepts.
9. 3x2 = 21 b) The zeros of the function are the first coordinates of
x2 = 7 the x-intercepts of the graph. They are −2 and 2.
√ √
x = 7 or x = − 7 Using the principle 27. a) The graph has only one x-intercept, (1, 0).
of square roots b) The zero of the function is the first coordinate of
√ √ the x-intercept of the graph, 1.
The solutions are 7 and − 7.
29. x2 + 6x = 7
11. 5x2 + 10 = 0
x2 + 6x + 9 = 7 + 9 Completing the square:
5x2 = −10
2 · 6 = 3 and 3 = 9
1 2

x2 = −2 (x + 3)2 = 16 Factoring
√ √
x = 2i or x = − 2i x + 3 = ±4 Using the principle
√ √
The solutions are 2i and − 2i. of square roots
x = −3 ± 4

Copyright © 2013 Pearson Education, Inc.


Exercise Set 3.2 97

x = −3 − 4 or x = −3 + 4 39. 5m2 + 3m = 2
x = −7 or x = 1 5m2 + 3m − 2 = 0
The solutions are −7 and 1. (5m − 2)(m + 1) = 0 Factoring
31. x = 8x − 9
2
5m − 2 = 0 or m + 1 = 0
x2 − 8x = −9 Subtracting 8x 2
m= or m = −1
x2 − 8x + 16 = −9 + 16 Completing the square: 5
2
1
= −4 and (−4)2 = 16 The solutions are and −1.
2 (−8) 5
(x − 4)2 = 7 Factoring 41. 3x2 + 6 = 10x
√ 3x2 − 10x + 6 = 0
x−4 = ± 7 Using the principle
of square roots We use the quadratic formula. Here a = 3, b = −10, and

x = 4±
7 c = 6.

√ √ √ −b ± b2 − 4ac
The solutions are 4 − 7 and 4 + 7, or 4 ± 7. x=
2a
33. x2 + 8x + 25 = 0 
−(−10)± (−10)2 −4·3·6
x2 + 8x = −25 Subtracting 25 = Substituting
2·3
x + 8x + 16 = −25 + 16
2
Completing the √ √
10 ± 28 10 ± 2 7
square: = =
6 6
1
2 · 8 = 4 and 42 = 16 √ √
2(5 ± 7) 5± 7
(x + 4)2 = −9 Factoring = =
2·3 3
x + 4 = ±3i Using the principle √ √ √
5− 7 5+ 7 5± 7
of square roots The solutions are and , or .
3 3 3
x = −4 ± 3i
The solutions are −4 − 3i and −4 + 3i, or −4 ± 3i. 43. x2 + x + 2 = 0
We use the quadratic formula. Here a = 1, b = 1, and
35. 3x2 + 5x − 2 = 0
c = 2.
3x2 + 5x = 2 Adding 2 √
−b ± b2 − 4ac
5 2 x=
x2 + x = Dividing by 3 2a
3 3 √
−1 ± 12 − 4 · 1 · 2
5 25 2 25 = Substituting
x2 + x + = + Completing the 2·1
3 36 3 36 √
square: −1 ± −7
=
1
· 5
= 5
and ( 56 )2 = 25 2
 2
2 3 6 36
√ √
5 49 −1 ± 7i 1 7
x+ = Factoring and = =− ± i
6 36 2 2 2
simplifying √ √
1 7 1 7
5
x+=±
7
Using the principle The solutions are − − i and − + i, or
6 6 √ 2 2 2 2
of square roots 1 7
5 7 − ± i.
x=− ± 2 2
6 6
5 7 5 7 45. 5t2 − 8t = 3
x=− − or x = − +
6 6 6 6 5t − 8t − 3 = 0
2

12 2 We use the quadratic formula. Here a = 5, b = −8, and


x=− or x =
6 6 c = −3.

1 −b ± b2 − 4ac
x = −2 or x = t=
3 2a
1 
The solutions are −2 and . −(−8) ± (−8)2 − 4 · 5(−3)
3 =
2·5
37. x2 − 2x = 15 √ √
x2 − 2x − 15 = 0 8 ± 124 8 ± 2 31
= =
10 10
(x − 5)(x + 3) = 0 Factoring √ √
2(4 ± 31) 4 ± 31
x − 5 = 0 or x + 3 = 0 = =
2·5 5
x = 5 or x = −3 √ √
4 − 31 4 + 31
The solutions are 5 and −3. The solutions are
5
and
5
, or

Copyright © 2013 Pearson Education, Inc.


98 Chapter 3: Quadratic Functions and Equations; Inequalities

√ √ √ √
4± 31 5− 17 5 + 17 5 ± 17
. The solutions are and , or .
5 4 4 4
47. 3x2 + 4 = 5x 55. 5x2 + 2x = −2
3x2 − 5x + 4 = 0 2
5x + 2x + 2 = 0
We use the quadratic formula. Here a = 3, b = −5, and We use the quadratic formula. We have a = 5, b = 2, and
c = 4. √ c = 2.
−b ± b2 − 4ac √
x= −b ± b2 − 4ac
2a x=
 2a
−(−5) ± (−5)2 − 4 · 3 · 4 √
= −2 ± 22 − 4 · 5 · 2
2·3 =
√ √ 2·5
5 ± −23 5 ± 23i √
= = −2 ± −36 −2 ± 6i
6 6 = =
√ 10 10

= ±
5 23
i 2(−1 ± 3i) −1 ± 3i
= =
6 6 √ √ 2·5 5
5 23 5 23 1 3
The solutions are − i and + i, or =− ± i
√ 6 6 6 6 5 5
5 23
± i. 1 3 1 3 1 3
The solutions are − − i and − + i, or − ± i.
6 6 5 5 5 5 5 5
49. x2 − 8x + 5 = 0 57. 4x2 = 8x + 5
We use the quadratic formula. Here a = 1, b = −8, and 4x − 8x − 5 = 0
2
c = 5. √
−b ± b2 − 4ac a = 4, b = −8, c = −5
x= b2 − 4ac = (−8)2 − 4 · 4(−5) = 144
2a

−(−8) ± (−8)2 − 4 · 1 · 5 Since b2 − 4ac > 0, there are two different real-number
= solutions.
2·1
√ √
8 ± 44 8 ± 2 11 59. x2 + 3x + 4 = 0
= =
2 2 a = 1, b = 3, c = 4

2(4 ± 11) √ b2 − 4ac = 32 − 4 · 1 · 4 = −7
= = 4 ± 11
2 √ √ √ Since b2 − 4ac < 0, there are two different imaginary-
The solutions are 4 − 11 and 4 + 11, or 4 ± 11. number solutions.
51. 3x2 + x = 5 61. 5t2 − 7t = 0
3x + x − 5 = 0
2
a = 5, b = −7, c = 0
We use the quadratic formula. We have a = 3, b = 1, and b2 − 4ac = (−7)2 − 4 · 5 · 0 = 49
c = −5.
√ Since b2 − 4ac = 49, there are two real-number solutions.
−b ± b2 − 4ac
x=
2a 63. Graph y = x2 − 8x + 12 and use the Zero feature twice.

−1 ± 12 − 4 · 3 · (−5)
=
2·3

−1 ± 61
=
6 √ √
−1 − 61 −1 + 61
The solutions are and , or
√ 6 6
−1 ± 61
.
6
53. 2x2 + 1 = 5x
2x2 − 5x + 1 = 0
We use the quadratic formula. We have a = 2, b = −5,
and c = 1. √
−b ± b2 − 4ac
x=
2a
 √
−(−5) ± (−5)2 − 4 · 2 · 1 5 ± 17 The solutions are 2 and 6.
= =
2·2 4

Copyright © 2013 Pearson Education, Inc.


Exercise Set 3.2 99

65. Graph y = 7x2 − 43x + 6 and use the Zero feature twice. 71. x2 + 6x + 5 = 0 Setting f (x) = 0
(x + 5)(x + 1) = 0 Factoring
x+5 = 0 or x + 1 = 0
x = −5 or x = −1
The zeros of the function are −5 and −1.
73. x2 − 3x − 3 = 0
a = 1, b = −3, c = −3

−b ± b2 − 4ac
x=
2a

−(−3) ± (−3)2 − 4 · 1 · (−3)
=
2·1

3 ± 9 + 12
=
2

3 ± 21
One solution is approximately 0.143 and the other is 6. =
2
√ √
67. Graph y1 = 6x + 1 and y2 = 4x2 and use the Intersect 3 − 21 3 + 21
The zeros of the function are and , or
feature twice. √ 2 2
3 ± 21
.
2
We use a calculator to find decimal approximations for the
zeros:
√ √
3 + 21 3 − 21
≈ 3.791 and ≈ −0.791.
2 2
75. x2 − 5x + 1 = 0
a = 1, b = −5, c = 1

−b ± b2 − 4ac
x=
2a

−(−5) ± (−5)2 − 4 · 1 · 1
=
2·1

5 ± 25 − 4
=
2

The solutions are approximately −0.151 and 1.651. 5 ± 21
=
2
69. Graph y = 2x2 − 5x − 4 and use the Zero feature twice. √ √
5 − 21 5 + 21
The zeros of the function are and , or
√ 2 2
5 ± 21
.
2
We use a calculator to find decimal approximations for the
zeros:
√ √
5 + 21 5 − 21
≈ 4.791 and ≈ 0.209.
2 2
77. x2 + 2x − 5 = 0
a = 1, b = 2, c = −5

−b ± b2 − 4ac
x=
2a

−2 ± 22 − 4 · 1 · (−5)
=
2·1
√ √
−2 ± 4 + 20 −2 ± 24
The zeros are approximately −0.637 and 3.137. = =
2 2

−2 ± 2 6 √
= = −1 ± 6
2

Copyright © 2013 Pearson Education, Inc.


100 Chapter 3: Quadratic Functions and Equations; Inequalities

√ √ √ √
√ of the function are −1 +
The zeros 6 and −1 − 6, or 1+ 6 1− 6
≈ 0.690 and ≈ −0.290.
−1 ± 6. 5 5
We use a calculator to find decimal approximations for the 85. 4x2 + 3x − 3 = 0
zeros: a = 4, b = 3, c = −3
√ √
−1 + 6 ≈ 1.449 and −1 − 6 ≈ −3.449 √
−b ± b2 − 4ac
x=
79. 2x2 − x + 4 = 0 2a

a = 2, b = −1, c = 4 −3 ± 32 − 4 · 4 · (−3)
√ =
−b ± b2 − 4ac 2·4
x= √
2a −3 ± 57
 =
−(−1) ± (−1)2 − 4 · 2 · 4 8
= √ √
2·2 −3 − 57 −3 + 57
√ √ The zeros of the function are and ,
1 ± −31 1 ± 31i √ 8 8
= = −3 ± 57
4 4 or .
√ 8
1 31 We use a calculator to find decimal approximations for the
= ± i zeros:
4 4 √ √
√ √
1 31 1 31 −3 + 57 −3 − 57
The zeros of the function are − i and + i, or ≈ 0.569 and ≈ −1.319.
√ 4 4 4 4 8 8
1 31
± i. 87. Graph y = 3x2 + 2x − 4 and use the Zero feature twice.
4 4
81. 3x2 − x − 1 = 0
a = 3, b = −1, c = −1

−b ± b2 − 4ac
x=
2a

−(−1) ± (−1)2 − 4 · 3 · (−1)
=
2·3

1 ± 13
=
6
√ √
1 − 13 1 + 13
The zeros of the function are and , or
√ 6 6
1 ± 13
.
6
We use a calculator to find decimal approximations for the
zeros: The zeros are approximately −1.535 and 0.869.
√ √
1 + 13 1 − 13
≈ 0.768 and ≈ −0.434. 89. Graph y = 5.02x2 − 4.19x − 2.057 and use the Zero feature
6 6
twice.
83. 5x2 − 2x − 1 = 0
a = 5, b = −2, c = −1

−b ± b2 − 4ac
x=
2a

−(−2) ± (−2)2 − 4 · 5 · (−1)
=
2·5
√ √
2 ± 24 2±2 6
= =
10 10
√ √
2(1 ± 6) 1± 6
= =
2·5 5
√ √
1− 6 1+ 6
The zeros of the function are and , or
√ 5 5
1± 6
.
5
We use a calculator to find decimal approximations for the The zeros are approximately −0.347 and 1.181.
zeros:

Copyright © 2013 Pearson Education, Inc.


Exercise Set 3.2 101

91. x4 − 3x2 + 2 = 0 99. m2/3 − 2m1/3 − 8 = 0


Let u = x2 . Let u = m1/3 .
u2 − 3u + 2 = 0 Substituting u for x 2
u2 − 2u − 8 = 0 Substituting u for m1/3
(u − 1)(u − 2) = 0 (u + 2)(u − 4) = 0
u − 1 = 0 or u − 2 = 0 u+2 = 0 or u − 4 = 0
u = 1 or u=2 u = −2 or u=4
Now substitute x2 for u and solve for x. Now substitute m 1/3
for u and solve for m.
x2 = 1 or x2 = 2 m1/3 = −2 or m1/3 = 4

x = ±1 or x = ± 2 (m1/3 )3 = (−2)3 or (m1/3 )3 = 43 Using the
√ √ principle of powers
The solutions are −1, 1, − 2, and 2.
m = −8 or m = 64
93. x4 + 3x2 = 10 The solutions are −8 and 64.
x + 3x2 − 10 = 0
4
101. x1/2 − 3x1/4 + 2 = 0
Let u = x2 .
Let u = x1/4 .
u2 + 3u − 10 = 0 Substituting u for x2
u2 − 3u + 2 = 0 Substituting u for x1/4
(u + 5)(u − 2) = 0
(u − 1)(u − 2) = 0
u+5 = 0 or u − 2 = 0
u − 1 = 0 or u − 2 = 0
u = −5 or u=2
u = 1 or u=2
Now substitute x2 for u and solve for x. 1/4
Now substitute x for u and solve for x.
x2 = −5 or x2 = 2
√ √ x1/4 = 1 or x1/4 = 2
x = ± 5i or x = ± 2
√ √ √ √ (x1/4 )4 = 14 or (x1/4 )4 = 24
The solutions are − 5i, 5i, − 2, and 2.
x=1 or x = 16
95. y 4 + 4y 2 − 5 = 0 The solutions are 1 and 16.
Let u = y 2 .
103. (2x − 3)2 − 5(2x − 3) + 6 = 0
u2 + 4u − 5 = 0 Substituting u for y 2
Let u = 2x − 3.
(u + 5)(u − 1) = 0 u2 − 5u + 6 = 0 Substituting u for 2x − 3
u+5 = 0 or u − 1 = 0 (u − 2)(u − 3) = 0
u = −5 or u=1 u − 2 = 0 or u − 3 = 0
2
Now substitute y for u and solve for y. u = 2 or u=3
y 2 = −5 or y 2 = 1 Now substitute 2x − 3 for u and solve for x.

y = ± 5i or y = ±1 2x − 3 = 2 or 2x − 3 = 3
√ √
The solutions are − 5i, 5i, −1, and 1. 2x = 5 or 2x = 6
√ 5
97. x − 3 x − 4 = 0 x= or x=3
√ 2
Let u = x. 5
√ The solutions are and 3.
u2 − 3u − 4 = 0 Substituting u for x 2
(u + 1)(u − 4) = 0 105. (2t2 + t)2 − 4(2t2 + t) + 3 = 0
u+1 = 0 or u − 4 = 0 Let u = 2t2 + t.
u = −1 or u=4 u2 − 4u + 3 = 0 Substituting u for 2t2 + t

Now substitute x for u and solve for x. (u − 1)(u − 3) = 0
√ √
x = −1 or x=4 u − 1 = 0 or u − 3 = 0
No solution x = 16 u = 1 or u=3
√ √
Note that x must be nonnegative, so x = −1 has no 2
Now substitute 2t + t for u and solve for t.
solution. The number 16 checks and is the solution. The 2t2 + t = 1 or 2t2 + t = 3
solution is 16.
2t2 + t − 1 = 0 or 2t2 + t − 3 = 0
(2t − 1)(t + 1) = 0 or (2t + 3)(t − 1) = 0

Copyright © 2013 Pearson Education, Inc.


102 Chapter 3: Quadratic Functions and Equations; Inequalities

2t−1=0 or t+1=0 or 2t+3=0 or t−1 = 0 Translate. We use the Pythagorean equation.


1 3 w2 + (w + 1)2 = 52
t= or t=−1 or t=− or t=1
2 2
Carry out. We solve the equation.
1 3
The solutions are , −1, − and 1. w2 + (w + 1)2 = 52
2 2
107. Substitute 40 for h(x) and solve for x. w + w2 + 2w + 1 = 25
2

40 = 0.012x2 − 0.583x + 35.727 2w2 + 2w + 1 = 25

0 = 0.012x2 − 0.583x − 4.273 2w2 + 2w − 24 = 0


2(w + 4)(w − 3) = 0
a = 0.012, b = −0.583, c = −4.273
w+4 = 0 or w − 3 = 0

−b ± b2 − 4ac w = −4 or w=3
x=
2a Since the width cannot be negative, we consider only 3.

−(−0.583) ± (−0.583)2 − 4(0.012)(−4.273) When w = 3, w + 1 = 3 + 1 = 4.
=
2(0.012) Check. The length, 4 ft, is 1 ft more than the width, 3 ft.
√ The length of√a diagonal of
0.583 ± 0.544993 √ a rectangle
√ with width 3 ft and
= length 4 ft is 32 + 42 = 9 + 16 = 25 = 5. The answer
0.024
checks.
x ≈ −6.5 or x ≈ 55.0
State. The length is 4 ft, and the width is 3 ft.
Since we are looking for a year after 1940, we use the
positive solution. There were 40 million multigenerational 115. Familiarize. Let n = the smaller number. Then n + 5 =
households about 55 yr after 1940, or in 1995. the larger number.
109. Substitute 130 for t(x) and solve for x. Translate.
The product of the numbers is 36.
130 = 0.16x2 + 0.46x + 21.36  
0 = 0.16x2 + 0.46x − 108.64 ↓ ↓ ↓
n(n + 5) = 36
a = 0.16, b = 0.46, c = −108.64 Carry out.
√ n(n + 5) = 36
−b ± b2 − 4ac
x=
2a n2 + 5n = 36

−0.46 ± (0.46)2 − 4(0.16)(−108.64) n2 + 5n − 36 = 0
=
2(0.16) (n + 9)(n − 4) = 0

−0.46 ± 69.7412 n+9 = 0 or n − 4 = 0
=
0.32 n = −9 or n=4
x ≈ −28 or x ≈ 25 If n = −9, then n + 5 = −9 + 5 = −4. If n = 4, then
We use the positive solution because a negative number has n + 5 = 4 + 5 = 9.
no meaning in this situation. The average U.S. household Check. The number −4 is 5 more than −9 and
received 130 TV channels about 25 yr after 1985, or in (−4)(−9) = 36, so the pair −9 and −4 check. The number
2010. 9 is 5 more than 4 and 9 · 4 = 36, so the pair 4 and 9 also
111. Familiarize and Translate. We will use the formula check.
s = 16t2 , substituting 1670 for s. State. The numbers are −9 and −4 or 4 and 9.
2
1670 = 16t 117. Familiarize. We add labels to the drawing in the text.
Carry out. We solve the equation. We let x represent the length of a side of the square in
1670 = 16t2 each corner. Then the length and width of the resulting
104.375 = t2 Dividing by 16 on both sides base are represented by 20 − 2x and 10 − 2x, respectively.
Recall that for a rectangle, Area = length × width.
10.216 ≈ t Taking the square root on
both sides
x x
Check. When t = 10.216, s = 16(10.216)2 ≈ 1670. The
x x
answer checks.
10 cm 10 − 2x
State. It would take an object about 10.216 sec to reach x 20 − 2x x
the ground.
x x
113. Familiarize. Let w = the width of the rug. Then w + 1 = 20 cm
the length.

Copyright © 2013 Pearson Education, Inc.


Exercise Set 3.2 103

Translate. 125. f (x) = 1.2x − (3.6)2


The
 area 
of the base is 96 cm2 .
  The function is in the form f (x) = mx + b, so it is a linear
(20 − 2x)(10 − 2x) = 96 function.
Carry out. We solve the equation. 127. In 2010, x = 2010 − 2004 = 6.
200 − 60x + 4x2 = 96 a(6) = 1.24(6) + 9.24 = 7.44 + 9.24 = 16.68
4x2 − 60x + 104 = 0 In 2010, $16.68 billion was spent on antipsychotic drugs.
x2 − 15x + 26 = 0
129. Test for symmetry with respect to the x-axis:
(x − 13)(x − 2) = 0
3x2 + 4y 2 = 5 Original equation
x − 13 = 0 or x − 2 = 0
3x2 + 4(−y)2 = 5 Replacing y by −y
x = 13 or x=2 2 2
3x + 4y = 5 Simplifying
Check. When x = 13, both 20 − 2x and 10 − 2x are
The last equation is equivalent to the original equation, so
negative numbers, so we only consider x = 2. When x = 2,
the graph is symmetric with respect to the x-axis.
then 20 − 2x = 20 − 2 · 2 = 16 and 10 − 2x = 10 − 2 · 2 = 6,
and the area of the base is 16 · 6, or 96 cm2 . The answer Test for symmetry with respect to the y-axis:
checks. 3x2 + 4y 2 = 5 Original equation
State. The length of the sides of the squares is 2 cm. 2
3(−x) + 4y = 5 2
Replacing x by −x
119. Familiarize. We have P = 2l + 2w, or 28 = 2l + 2w. 3x2 + 4y 2 = 5 Simplifying
Solving for w, we have The last equation is equivalent to the original equation, so
28 = 2l + 2w the equation is symmetric with respect to the y-axis.
14 = l + w Dividing by 2 Test for symmetry with respect to the origin:
14 − l = w. 3x2 + 4y 2 = 5 Original equation
Then we have l = the length of the rug and 14 − l = the 3(−x)2 + 4(−y)2 = 5 Replacing x by −x
width, in feet. Recall that the area of a rectangle is the and y by −y
product of the length and the width. 3x2 + 4y 2 = 5 Simplifying
Translate. The last equation is equivalent to the original equation, so
The area is 48 ft2 . the equation is symmetric with respect to the origin.
    
131. f (x) = 2x3 − x
l(14 − l) = 48 f (−x) = 2(−x)3 − (−x) = −2x3 + x
Carry out. We solve the equation. −f (x) = −2x3 + x
l(14 − l) = 48 f (x) = f (−x) so f is not even
14l − l2 = 48 f (−x) = −f (x), so f is odd.
0 = l2 − 14l + 48
133. a) kx2 − 17x + 33 = 0
0 = (l − 6)(l − 8)
k(3)2 − 17(3) + 33 = 0 Substituting 3 for x
l − 6 = 0 or l − 8 = 0
9k − 51 + 33 = 0
l = 6 or l=8
9k = 18
If l = 6, then 14 − l = 14 − 6 = 8.
k=2
If l = 8, then 14 − l = 14 − 8 = 6.
b) 2x − 17x + 33 = 0
2
Substituting 2 for k
In either case, the dimensions are 8 ft by 6 ft. Since we
usually consider the length to be greater than the width, (2x − 11)(x − 3) = 0
we let 8 ft = the length and 6 ft = the width. 2x − 11 = 0 or x − 3 = 0
Check. The perimeter is 2·8 ft+2·6 ft = 16 ft +12 ft = 11
28 ft. The answer checks. x= or x=3
2
State. The length of the rug is 8 ft, and the width is 6 ft. 11
The other solution is .
2
121. f (x) = 4 − 5x = −5x + 4
135. a) (1 + i)2 − k(1 + i) + 2 = 0 Substituting
The function can be written in the form y = mx + b, so it
1 + i for x
is a linear function.
1 + 2i − 1 − k − ki + 2 = 0
123. f (x) = 7x2 2 + 2i = k + ki
The function is in the form f (x) = ax2 + bx + c, a = 0, so 2(1 + i) = k(1 + i)
it is a quadratic function. 2=k

Copyright © 2013 Pearson Education, Inc.


104 Chapter 3: Quadratic Functions and Equations; Inequalities

√ √
b) x2 − 2x + 2 = 0 Substituting 2 for k 145. x−3− 4
x−3=2

 Substitute u for 4 x − 3.
−(−2) ± (−2)2 − 4 · 1 · 2
x= u2 − u − 2 = 0
2·1
√ (u − 2)(u + 1) = 0
2 ± −4
= u − 2 = 0 or u + 1 = 0
2
2 ± 2i u = 2 or u = −1
= =1±i √
2 Substitute x − 3 for u and solve for x.
4

The other solution is 1 − i. √ √


4
x − 3 = 2 or 4 x − 3 = 1
137. (x − 2)3 = x3 − 2 x − 3 = 16 No solution
x − 6x + 12x − 8 = x3 − 2
3 2
x = 19
0 = 6x2 − 12x + 6 The value checks. The solution is 19.
0 = 6(x2 − 2x + 1)  2
147. 2 6
0 = 6(x − 1)(x − 1) y+ + 3y + = 4
y y
x − 1 = 0 or x − 1 = 0  2  
2 2
x = 1 or x=1 y+ +3 y+ −4 = 0
y y
The solution is 1. 2
Substitute u for y + .
139. (6x3 + 7x2 − 3x)(x2 − 7) = 0 y
x(6x2 + 7x − 3)(x2 − 7) = 0 u2 + 3u − 4 = 0
x(3x − 1)(2x + 3)(x2 − 7) = 0 (u + 4)(u − 1) = 0
u = −4 or u = 1
x=0 or 3x − 1=0 or 2x + 3=0 or x2 − 7 = 0
2
1 3 √ Substitute y + for u and solve for y.
x=0 or x= or x=− or x = 7 or y
3 2 √
x=− 7 2
y + = −4 or y+ =1
2
√ 3 1 √ y y
The exact solutions are − 7, − , 0, , and 7. y 2 + 2 = −4y or y2 + 2 = y
2 3
√ y 2 + 4y + 2 = 0 or y 2 − y + 2 = 0
141. x + x −
2
2=0 √
√ −4± 42 −4·1·2
−b ±b2 − 4ac y= or
x= 2·1
2a 
−(−1)± (−1)2 −4·1·2
√  √ y=
−1 ± 12 − 4 · 1(− 2) −1 ± 1 + 4 2 2·1
= = √ √
2·1 2 −4 ± 8 1 ± −7
 √ y= or y =
−1 ± 1 + 4 2 2 2
The solutions are . √ √
2 −4 ± 2 2 1 ± 7i
y= or y =
143. 2t2 + (t − 4)2 = 5t(t − 4) + 24 2 2

2t + t2 − 8t + 16 = 5t2 − 20t + 24
2 √ 1 7
y = −2 ± 2 or y = ± i
0 = 2t2 − 12t + 8 2 2

√ 1 7
0 = t2 − 6t + 4 Dividing by 2 The solutions are −2 ± 2 and ± i.
2 2
Use the quadratic formula.

−b ± b2 − 4ac Exercise Set 3.3
t=
2a
 1. a) The minimum function value
−(−6) ± (−6)2 − 4 · 1 · 4   occurs at the vertex,
= 1 9
2·1 so the vertex is − , − .
√ √ 2 4
6 ± 20 6±2 5 b) The axis of symmetry is a vertical line through the
= =
2 2 1
√ vertex. It is x = − .
2(3 ± 5) √ 2
= =3± 5 9
2
√ c) The minimum value of the function is − .
4
The solutions are 3 ± 5.

Copyright © 2013 Pearson Education, Inc.


Exercise Set 3.3 105

3. f (x) = x2 − 8x + 12 16 completes the x f (x) y


square for x2 − 8x.
= x − 8x + 16 − 16 + 12 Adding 16−16
2 7 1
− 8
on the right side 2 4
6
= (x2 − 8x + 16) − 16 + 12 4 0
4
= (x − 4) − 4
2
Factoring and
5 2 2
simplifying
= (x − 4)2 + (−4) Writing in the form 3 0 2 2 4 6 x
f (x) = a(x − h)2 + k
1 6
a) Vertex: (4, −4) f (x)  x 2  7x  12

b) Axis of symmetry: x = 4
7. f (x) = x2 + 4x + 5 4 completes the
c) Minimum value: −4
square for x2 + 4x
d) We plot the vertex and find several points on either = x2 + 4x + 4 − 4 + 5 Adding 4 − 4
side of it. Then we plot these points and connect on the right side
them with a smooth curve.
= (x + 2)2 + 1 Factoring and simplifying
x f (x) = [x − (−2)]2 + 1 Writing in the form
y
4 −4 f (x) = a(x − h)2 + k
4 a) Vertex: (−2, 1)
2 0
2 b) Axis of symmetry: x = −2
1 5 2 2 4 6 8 c) Minimum value: 1
x
2
5 −3 d) We plot the vertex and find several points on either
4 side of it. Then we plot these points and connect
6 0 them with a smooth curve.
y
f (x)  x 2  8x  12 x f (x)

49 −2 1 16
5. f (x) = x2 − 7x + 12 completes the
4 12
square for x2 − 7x. −1 2
8
49 49 0 5
= x − 7x +
2
− + 12 Adding 4
4 4
−3 2 6 4 2 2 x
49 49
− on the right side
4 4 −4 5
  f (x)  x 2  4x  5
49 49
= x2 − 7x + − + 12
4 4
 2 x2
7 1 9. g(x) = + 4x + 6
= x− − Factoring and 2
2 4
1 1
simplifying = (x2 + 8x) + 6 Factoring out of the
 2   2 2
7 1 first two terms
= x− + − Writing in the 1 2
2 4 =(x +8x+16−16)+6 Adding 16 − 16 inside
2
form f (x) = a(x − h)2 + k the parentheses
  1 2 1
7 1 = (x +8x+16)− ·16+6 Removing −16 from
a) Vertex: ,− 2 2
2 4 within the parentheses
7 1
b) Axis of symmetry: x = = (x + 4) − 2 Factoring and simplifying
2

2 2
1 1
c) Minimum value: − = [x − (−4)]2 + (−2)
4 2
d) We plot the vertex and find several points on either a) Vertex: (−4, −2)
side of it. Then we plot these points and connect b) Axis of symmetry: x = −4
them with a smooth curve.
c) Minimum value: −2
d) We plot the vertex and find several points on either
side of it. Then we plot these points and connect
them with a smooth curve.

Copyright © 2013 Pearson Education, Inc.


106 Chapter 3: Quadratic Functions and Equations; Inequalities

x g(x) y 13. f (x) = −x2 − 6x + 3


= −(x2 + 6x) + 3 9 completes the square
−4 −2 8 for x2 + 6x.
−2 0 4 = −(x + 6x + 9 − 9) + 3
2

8 4 4 8
= −(x + 3)2 − (−9) + 3 Removing −9 from
0 6 x
4
the parentheses
−6 0 = −(x + 3)2 + 9 + 3
8
= −[x − (−3)]2 + 12
−8 6
x2
a) Vertex: (−3, 12)
g(x)    4x  6
2 b) Axis of symmetry: x = −3
c) Maximum value: 12
11. g(x) = 2x2 + 6x + 8 d) We plot the vertex and find several points on either
= 2(x2 + 3x) + 8 Factoring 2 out of side of it. Then we plot these points and connect
the first two terms them with a smooth curve.
 
9 9 y
= 2 x2 + 3x + − +8 Adding x f (x)
4 4
9 9 −3 12
− inside the parentheses
4 4 4
  0 3
9 9
= 2 x2 + 3x + − 2 · + 8 Removing 8 4 4 8 x
4 4 1 −4
4
9
− from within the parentheses −6 3 8
4
 2
3 7 −7 −4
= 2 x+ + Factoring and f (x)  x 2  6x  3
2 2
simplifying
 2 15. g(x) = −2x2 + 2x + 1
3 7 = −2(x2 − x) + 1 Factoring −2 out of the
= 2 x− − +
2 2
  first two terms
3 7  
a) Vertex: − , 1 1 1 1
2 2 = −2 x2 −x+ − +1 Adding −
4 4 4 4
3 inside the parentheses
b) Axis of symmetry: x = −    
2 1 1
7 = −2 x2 −x+ −2 − +1
c) Minimum value: 4 4
2
1
d) We plot the vertex and find several points on either Removing − from within the parentheses
side of it. Then we plot these points and connect 4
 2
them with a smooth curve. 1 3
= −2 x − +
x f (x) 2 2
y
 
3 7 1 3
− 16
a) Vertex: ,
2 2
2 2
12 1
−1 4 b) Axis of symmetry: x =
8 2
0 8 3
4 c) Maximum value:
2
−2 4 4 2 2 4 x
−3 8
g(x)  2x 2  6x  8

Copyright © 2013 Pearson Education, Inc.


Exercise Set 3.3 107

d) We plot the vertex and find several points on either b) Since a = 2 > 0, the graph opens up so the second
side of it. Then we plot these points and connect coordinate of the vertex, −18, is the minimum value
them with a smooth curve. of the function.
x f (x) c) The range is [−18, ∞).
y
1 3 d) Since the graph opens up, function values decrease
2 2 4 to the left of the vertex and increase to the right of
2
the vertex. Thus, f (x) is increasing on (−1, ∞) and
1 1 decreasing on (−∞, −1).
4 2 2 4 x
2 −3 1
2 35. f (x) = − x2 + 5x − 8
2
0 1 4
a) The x-coordinate of the vertex is
−1 −3 b 5
− =−   = 5.
g(x)  2x 2  2x  1 2a 1
2 −
2
17. The graph of y = (x + 3)2 has vertex (−3, 0) and opens 1 2 9
up. It is graph (f). Since f (5) = − · 5 + 5 · 5 − 8 = , the vertex is
  2 2
9
19. The graph of y = 2(x − 4)2 − 1 has vertex (4, −1) and 5, .
2
opens up. It is graph (b). 1
b) Since a = − < 0, the graph opens down so the sec-
1 2
21. The graph of y = − (x + 3)2 + 4 has vertex (−3, 4) and 9
2 ond coordinate of the vertex, , is the maximum
opens down. It is graph (h). 2
value of the function.
 
23. The graph of y = −(x + 3)2 + 4 has vertex (−3, 4) and c) The range is − ∞, 9 .
opens down. It is graph (c). 2
25. The function f (x) = −3x2 + 2x + 5 is of the form d) Since the graph opens down, function values in-
f (x) = ax2 + bx + c with a < 0, so it is true that it has a crease to the left of the vertex and decrease to the
maximum value. right of the vertex. Thus, f (x) is increasing on
(−∞, 5) and decreasing on (5, ∞).
27. The statement is false. The graph of h(x) = (x + 2)2 can
be obtained by translating the graph of h(x) = x2 two 37. f (x) = 3x2 + 6x + 5
units to the left. a) The x-coordinate of the vertex is
b 6
29. The function f (x) = −(x + 2)2 − 4 can be written as − =− = −1.
2a 2·3
f (x) = −[x − (−2)]2 − 4, so it is true that the axis of
Since f (−1) = 3(−1)2 + 6(−1) + 5 = 2, the vertex
symmetry is x = −2.
is (−1, 2).
31. f (x) = x2 − 6x + 5 b) Since a = 3 > 0, the graph opens up so the second
a) The x-coordinate of the vertex is coordinate of the vertex, 2, is the minimum value of
b −6 the function.
− =− = 3.
2a 2·1 c) The range is [2, ∞).
Since f (3) = 32 −6·3+5 = −4, the vertex is (3, −4).
d) Since the graph opens up, function values decrease
b) Since a = 1 > 0, the graph opens up so the second to the left of the vertex and increase to the right of
coordinate of the vertex, −4, is the minimum value the vertex. Thus, f (x) is increasing on (−1, ∞) and
of the function. decreasing on (−∞, −1).
c) The range is [−4, ∞).
39. g(x) = −4x2 − 12x + 9
d) Since the graph opens up, function values decrease a) The x-coordinate of the vertex is
to the left of the vertex and increase to the right of b −12 3
the vertex. Thus, f (x) is increasing on (3, ∞) and − =− =− .
2a 2(−4) 2
decreasing on (−∞, 3).  3  3 2  3
Since g − = −4 − −12 − + 9 = 18,
33. f (x) = 2x2 + 4x − 16 2  2
 2
3
a) The x-coordinate of the vertex is the vertex is − , 18 .
2
b 4
− =− = −1. b) Since a = −4 < 0, the graph opens down so the
2a 2·2
second coordinate of the vertex, 18, is the maximum
Since f (−1) = 2(−1)2 + 4(−1) − 16 = −18, the value of the function.
vertex is (−1, −18).
c) The range is (−∞, 18].

Copyright © 2013 Pearson Education, Inc.


108 Chapter 3: Quadratic Functions and Equations; Inequalities

d) Since the graph opens down, function values in- Check. When x = 4.5, then 18 − 2x = 9 and V (x) =
crease to the left of the vertex and decrease to the 10 · 9(4.5), or 405. As a partial check, we can find V (x) for
right of the vertex. Thus, g(x) is increasing on a value of x less than 4.5 and for a value of x greater than
 3  3 
− ∞, − and decreasing on − , ∞ . 4.5. For instance, V (4.4) = 404.8 and V (4.6) = 404.8.
2 2 Since both of these values are less than 405, our result
41. Familiarize and Translate. The function appears to be correct.
s(t) = −16t2 + 20t + 6 is given in the statement of the State. The file should be 4.5 in. tall in order to maximize
problem. the volume.
Carry out. The function s(t) is quadratic and the coef-
47. Familiarize. Let b = the length of the base of the triangle.
ficient of t2 is negative, so s(t) has a maximum value. It
Then the height = 20 − b.
occurs at the vertex of the graph of the function. We find
1
the first coordinate of the vertex. This is the time at which Translate. Since the area of a triangle is × base×height,
the ball reaches its maximum height. 2
we have
b 20 1 1
t=− =− = 0.625 A(b) = b(20 − b), or − b2 + 10b.
2a 2(−16) 2 2
The second coordinate of the vertex gives the maximum Carry out. Since A(b) is a quadratic function with
height. 1
a = − < 0, the maximum function value occurs at the
s(0.625) = −16(0.625)2 + 20(0.625) + 6 = 12.25 2
vertex of the graph of the function. The first coordinate
Check. Completing the square, we write the function in of the vertex is
the form s(t) = −16(t − 0.625)2 + 12.25. We see that the b 10
− =−   = 10.
coordinates of the vertex are (0.625, 12.25), so the answer 2a 1
checks. 2 −
2
State. The ball reaches its maximum height after When b = 10, then 20 − b = 20 − 10 = 10, and the area is
0.625 seconds. The maximum height is 12.25 ft. 1
· 10 · 10 = 50 cm2 .
2
43. Familiarize and Translate. The function
Check. As a partial check, we can find A(b) for a value
s(t) = −16t2 + 120t + 80 is given in the statement of the
of b less than 10 and for a value of b greater than 10. For
problem.
instance, V (9.9) = 49.995 and V (10.1) = 49.995. Since
Carry out. The function s(t) is quadratic and the coef- both of these values are less than 50, our result appears to
ficient of t2 is negative, so s(t) has a maximum value. It be correct.
occurs at the vertex of the graph of the function. We find
State. The area is a maximum when the base and the
the first coordinate of the vertex. This is the time at which
height are both 10 cm.
the rocket reaches its maximum height.
b 120 49. C(x) = 0.1x2 − 0.7x + 1.625
t=− =− = 3.75
2a 2(−16) Since C(x) is a quadratic function with a = 0.1 > 0, a
The second coordinate of the vertex gives the maximum minimum function value occurs at the vertex of the graph
height. of C(x). The first coordinate of the vertex is
s(3.75) = −16(3.75)2 + 120(3.75) + 80 = 305 b −0.7
− =− = 3.5.
Check. Completing the square, we write the function in 2a 2(0.1)
the form s(t) = −16(t − 3.75)2 + 305. We see that the Thus, 3.5 hundred, or 350 chairs should be built to mini-
coordinates of the vertex are (3.75, 305), so the answer mize the average cost per chair.
checks.
51. P (x) = R(x) − C(x)
State. The rocket reaches its maximum height after
P (x) = (50x − 0.5x2 ) − (10x + 3)
3.75 seconds. The maximum height is 305 ft.
P (x) = −0.5x2 + 40x − 3
45. Familiarize. Using the label in the text, we let x = the
Since P (x) is a quadratic function with a =
height of the file. Then the length = 10 and the width =
−0.5 < 0, a maximum function value occurs at the vertex
18 − 2x.
of the graph of the function. The first coordinate of the
Translate. Since the volume of a rectangular solid is vertex is
length × width × height we have b 40
− =− = 40.
V (x) = 10(18 − 2x)x, or −20x2 + 180x. 2a 2(−0.5)
Carry out. Since V (x) is a quadratic function with a = P (40) = −0.5(40)2 + 40 · 40 − 3 = 797
−20 < 0, the maximum function value occurs at the vertex Thus, the maximum profit is $797. It occurs when 40 units
of the graph of the function. The first coordinate of the are sold.
vertex is
b 180
− =− = 4.5.
2a 2(−20)

Copyright © 2013 Pearson Education, Inc.


Exercise Set 3.3 109

53. Familiarize. Using the labels on the drawing in the text, 57. f (x) = 3x − 7
we let x = the width of each corral and 240 − 3x = the f (x + h) − f (x) 3(x + h) − 7 − (3x − 7)
total length of the corrals. =
h h
Translate. Since the area of a rectangle is length × width, 3x + 3h − 7 − 3x + 7
we have =
h
A(x) = (240 − 3x)x = −3x2 + 240x. 3h
= =3
Carry out. Since A(x) is a quadratic function with a = h
−3 < 0, the maximum function value occurs at the vertex 59. The graph of f (x) is stretched vertically and reflected
of the graph of A(x). The first coordinate of the vertex is across the x-axis.
b 240
− =− = 40. y
2a 2(−3) (3, 4)
4
A(40) = −3(40)2 + 240(40) = 4800
2
Check. As a partial check we can find A(x) for a value (5, 0) (5, 0)
of x less than 40 and for a value of x greater than 40. For 4 2 2 4 x
instance, A(39.9) = 4799.97 and A(40.1) = 4799.97. Since 2
both of these values are less than 4800, our result appears (0, 4)
to be correct. (3, 4)

State. The largest total area that can be enclosed is g(x)  2f(x)
4800 yd2 .
61. f (x) = −0.2x2 − 3x + c
55. Familiarize. We let s = the height of the elevator shaft, b
t1 = the time it takes the screwdriver to reach the bottom The x-coordinate of the vertex of f (x) is − =
2a
of the shaft, and t2 = the time it takes the sound to reach −3
the top of the shaft. − = −7.5. Now we find c such that f (−7.5) =
2(−0.2)
Translate. We know that t1 + t2 = 5. Using the informa- −225.
tion in Example 7 we also know that −0.2(−7.5)2 − 3(−7.5) + c = −225

s −11.25 + 22.5 + c = −225
s = 16t21 , or t1 = and
4
c = −236.25
s
s = 1100t2 , or t2 = .
1100 63.
√ f(x)  (|x |  5)2  3
s s
Then + = 5.
4 1100 y
Carry out. We solve the last equation above.
√ 24
s s
+ =5 16
4 1100
√ 8
275 s + s = 5500 Multiplying by 1100

s + 275 s − 5500 = 0 8 4 4 8 x
√ 8
Let u = s and substitute.
u2 + 275u − 5500 = 0
√ 65. First we find the radius r of a circle with circumference x:
−b + b2 − 4ac
u= We only want the 2πr = x
2a positive solution. x
 r=
−275 + 275 − 4 · 1(−5500)
2 2π
= Then we find the length s of a side of a square with perime-
2·1
√ ter 24 − x:
−275 + 97, 625
= ≈ 18.725 4s = 24 − x
2
√ 24 − x
Since u ≈ 18.725, we have s = 18.725, so s ≈ 350.6. s=
√ √ 4
s 350.6
Check. If s ≈ 350.6, then t1 = = ≈ Then S = area of circle + area of square
4 4
s 350.6 S= πr2 + s2
4.68 and t2 = = ≈ 0.32, so t1 + t2 = 4.68 +  2  2
1100 1100 x 24 − x
0.32 = 5. S(x) = π +
2π 4
The result checks.  
1 1
State. The elevator shaft is about 350.6 ft tall. S(x) = + x2 − 3x + 36
4π 16

Copyright © 2013 Pearson Education, Inc.


110 Chapter 3: Quadratic Functions and Equations; Inequalities

Since S(x) is a quadratic function with 21. 4x2 − 8x − 3 = 0


1 1
a= + > 0, the minimum function value occurs at 4x2 − 8x = 3
4π 16
the vertex of the graph of S(x). The first coordinate of 3
x2 − 2x =
the vertex is 4
b −3 24π 3
− =−  = . x2 − 2x + 1 = + 1 Completing the square:
2a 1 1 4+π 4 1
2 + (−2) = −1 and (−1)2 = 1
4π 16 2
24π 7
Then the string should be cut so that one piece is in., (x − 1)2 =
4+π 4
24π √
or about 10.56 in. The other piece will be 24 − , or 7
4+π x−1 = ±
96 2
in., or about 13.44 in. √
4+π 7
x = 1+
2

Chapter 3 Mid-Chapter Mixed Review 2± 7
x=
2
√ √ √
2+ 7 2− 7 2± 7
1. The statement is true. See page 240 in the text. The zeros are and , or .
2 4 2
3. The statement is true. See page 251 in the text. 23. 4x2 − 12x + 9 = 0
√ √ √ √ a) b2 − 4ac = (−12)2 − 4 · 4 · 9 = 144 − 144 = 0
5. −36 = −1 · 36 = −1 · 36 = i · 6 = 6i
√ √ √ √ There is one real-number solution.
7. − −16 = − −1 · 16 = − −1 · 16 = −i · 4 = −4i
b) 4x2 − 12x + 9 = 0
9. (3 − 2i) + (−4 + 3i) = (3 − 4) + (−2i + 3i) = −1 + i
(2x − 3)2 = 0
11. (2 + 3i)(4 − 5i) = 8 − 10i + 12i − 15i 2
2x − 3 = 0
= 8 + 2i + 15 2x = 3
= 23 + 2i 3
x=
2
13. i13 = i12 · i = (i2 )6 · i = (−1)6 · i = i 3
The solution is .
15. (−i)5 = (−1 · i)5 = (−1)5 i5 = −i4 · i = −(i2 )2 · i = 2
−(−1)2 · i = −i 25. x4 + 5x2 − 6 = 0
Let u = x2 .
17. x2 + 3x − 4 = 0
u2 + 5u − 6 = 0 Substituting
(x + 4)(x − 1) = 0
(u + 6)(u − 1) = 0
x+4 = 0 or x − 1 = 0
u+6 = 0 or u − 1 = 0
x = −4 or x=1
u = −6 or u=1
The solutions are −4 and 1.
x = −6
2
or x2 = 1
19. 2
4x = 24 √
x = ± 6i or x = ±1
x2 = 6 √
√ √ The solutions are ± 6i and ±1.
x = 6 or x = − 6
√ √ √ 27. Familiarize. Let x = the smaller number. Then x + 2 =
The solutions are 6 and − 6, or ± 6. the larger number.
Translate.
The product of the numbers is 35.
 
↓ ↓ ↓
x(x + 2) = 35
Carry out.
x(x + 2) = 35
x2 + 2x = 35
x2 + 2x − 35 = 0
(x + 7)(x − 5) = 0

Copyright © 2013 Pearson Education, Inc.


Exercise Set 3.4 111

x+7 = 0 or x − 5 = 0 Check:
x = −7 or x=5 1 1
+ =
1
If x = −7, then x + 2 = −7 + 2 = −5; if x = 5, then 4 5 t
x + 2 = 5 + 2 = 7. 1 1 1
+ ?
Check. −5 is 2 more than −7, and (−7)(−5) = 35. Also, 4 5  20

7 is 2 more than 5, and 5 · 7 = 35. The numbers check.  9
5 4  9
State. The numbers are 5 and 7 or −7 and −5. + 1·
20 20  20
9  9
29. f (x) = −2x2 − 4x − 5 TRUE
20  20
= −2(x2 + 2x) − 5
20
= −2(x2 + 2x + 1 − 1) − 5 The solution is .
9
= −2(x2 + 2x + 1) − 2(−1) − 5 x+2 x−1
3. − = 15, LCD is 20
= −2(x + 1)2 − 3 4 5
x + 2 x − 1
= −2[x − (−1)]2 + (−3)
20 − = 20 · 15
a) Vertex: (−1, −3) 4 5
5(x + 2) − 4(x − 1) = 300
b) Axis of symmetry: x = −1
5x + 10 − 4x + 4 = 300
c) Maximum value: −3
x + 14 = 300
d) Range: (−∞, −3]
x = 286
e) Increasing: (−∞, −3); decreasing: (−3, ∞)
The solution is 286.
f)
x f (x) 1 2 1 3
y 5. + = + , LCD is 6x
−1 −3 2 x 3 x
1 2 1 3 
2
−3 −11 6x +
2 x
= 6x +
3 x
6 4 2 2 x
−2 −5 2
3x + 12 = 2x + 18

0 −5 4 3x − 2x = 18 − 12
6 x=6
1 −11
Check:
f (x)  2x 2  4x  5 1 2 1 3
+ = +
2 x 3 x
31. Use the discriminant. If b2 − 4ac < 0, there are no x- 1 2 1 3
+ ? +
intercepts. If b2 − 4ac = 0, there is one x-intercept. If 2 6 3 6
b2 − 4ac > 0, there are two x-intercepts. |
1 1  1 1
+ + TRUE
33. The x-intercepts of g(x) are also (x1 , 0) and (x2 , 0). This is 2 3  3 2
true because f (x) and g(x) have the same zeros. Consider The solution is 6.
g(x) = 0, or −ax2 − bx − c = 0. Multiplying by −1 on
both sides, we get an equivalent equation ax2 + bx + c = 0, 5 3
7. = , LCD is 2x(3x + 2)
or f (x) = 0. 3x + 2 2x
5 3
2x(3x + 2) · = 2x(3x + 2) ·
3x + 2 2x
Exercise Set 3.4 2x · 5 = 3(3x + 2)
1 1 1 10x = 9x + 6
1. + = , LCD is 20t
4 5 t x=6
1 1 1 6 checks, so the solution is 6.
20t + = 20t ·
4 5 t
9. y2 16
1 1 1 = , LCD is y + 4
20t · + 20t · = 20t · y+4 y+4
4 5 t 2
y 16
5t + 4t = 20 (y + 4) · = (y + 4) ·
y+4 y+4
9t = 20 y 2 = 16
20 y = 4 or y = −4
t=
9
Only 4 checks. It is the solution.

Copyright © 2013 Pearson Education, Inc.


112 Chapter 3: Quadratic Functions and Equations; Inequalities

6 2 1 16
11. x+ = 5, LCD is x 17. + = 2
x x+5 x−5 x − 25
 
6 2 1 16
x x+ = x·5 + = ,
x x+5 x−5 (x + 5)(x − 5)
x2 + 6 = 5x LCD is (x + 5)(x − 5)
x2 − 5x + 6 = 0  
2 1 16
(x+5)(x−5) + = (x+5)(x−5)·
(x − 2)(x − 3) = 0 x+5 x−5 (x+5)(x−5)
x − 2 = 0 or x − 3 = 0 2(x − 5) + x + 5 = 16
x = 2 or x=3 2x − 10 + x + 5 = 16
Both numbers check. The solutions are 2 and 3. 3x − 5 = 16
3x = 21
6 2 5y − 3
13. + = 2 x=7
y+3 y y −9
6 2 5y − 3 7 checks, so the solution is 7.
+ = ,
y+3 y (y + 3)(y − 3) 3x 6 12
19. + =
LCD is y(y+3)(y−3) x+2 x x2 + 2x
  3x 6 12
6 2 5y−3 + = , LCD is x(x+2)
y(y+3)(y−3) + = y(y+3)(y−3)· x+2 x x(x+2)
y+3 y (y+3)(y−3)
 3x 6
6y(y−3)+2(y+3)(y−3) = y(5y − 3) x(x + 2) + = x(x + 2) ·
12
6y 2 − 18y + 2(y 2 − 9) = 5y 2 − 3y x+2 x x(x + 2)
3x · x + 6(x + 2) = 12
6y 2 − 18y + 2y 2 − 18 = 5y 2 − 3y
3x2 + 6x + 12 = 12
8y 2 − 18y − 18 = 5y 2 − 3y
3x2 + 6x = 0
3y 2 − 15y − 18 = 0
3x(x + 2) = 0
y 2 − 5y − 6 = 0
(y − 6)(y + 1) = 0 3x = 0 or x + 2 = 0
x = 0 or x = −2
y − 6 = 0 or y + 1 = 0
Neither 0 nor −2 checks, so the equation has no solution.
y = 6 or y = −1
Both numbers check. The solutions are 6 and −1. 1 1 3
21. − =
5x + 20 x2 − 16 x−4
2x 5
15. = , LCD is (x − 1)(x − 3) 1 1 3
x−1 x−3 − = ,
5(x + 4) (x + 4)(x − 4) x−4
2x 5
(x − 1)(x − 3) ·
x−1
= (x − 1)(x − 3) ·
x−3   5(x + 4)(x − 4)
LCD is
1 1 3
2x(x − 3) = 5(x − 1) 5(x+4)(x−4) − = 5(x+4)(x−4)·
5(x+4) (x+4)(x−4) x−4
2x2 − 6x = 5x − 5 x − 4 − 5 = 15(x + 4)
2x − 11x + 5 = 0
2
x − 9 = 15x + 60
(2x − 1)(x − 5) = 0 −14x − 9 = 60
2x − 1 = 0 or x − 5 = 0 −14x = 69
2x = 1 or x=5 69
x=−
1 14
x= or x=5 69 69
2 − checks, so the solution is − .
1 14 14
Both numbers check. The solutions are and 5.
2

Copyright © 2013 Pearson Education, Inc.


Exercise Set 3.4 113

2 3 4 1 1 6
23. − = 29. − = 2
5x + 5 x2 − 1 x−1 x−6 x x − 6x
2 3 4 1 1 6
− = , − = , LCD is x(x−6)
5(x + 1) (x + 1)(x − 1) x−1 x−6 x x(x−6)
 
 LCD is  5(x + 1)(x − 1) x(x−6)
1

1
= x(x−6) ·
6
2 3 4 x−6 x x(x−6)
5(x+1)(x−1) − = 5(x+1)(x−1)·
5(x+1) (x+1)(x−1) x−1 x − (x − 6) = 6
2(x − 1) − 5 · 3 = 20(x + 1)
x−x+6 = 6
2x − 2 − 15 = 20x + 20
6=6
2x − 17 = 20x + 20
We get an equation that is true for all real numbers.
−18x − 17 = 20 Note, however, that when x = 6 or x = 0, division
−18x = 37 by 0 occurs in the original equation. Thus, the solution
37 set is {x|x is a real number and x = 6 and x = 0}, or
x=− (−∞, 0) ∪ (0, 6) ∪ (6, ∞).
18

37 37
checks, so the solution is − . √
18 18 31. 3x − 4 = 1

8 x 24 ( 3x − 4)2 = 12
25. = + ,
x2 − 2x + 4 x + 2 x3 + 8 3x − 4 = 1
LCD is (x + 2)(x − 2x + 4)
2
3x = 5
8 5
(x+2)(x2 −2x+4) · 2 = x=
x −2x+4 3
 
x 24 Check:
(x+2)(x2− 2x+4) + √
x+2 (x+2)(x2 −2x+4) 3x − 4 =1
8(x + 2) = x(x −2x+4)+24
2 
5
8x + 16 = x3 −2x2 +4x+24 3· −4 ? 1
3 
0 = x −2x −4x+8
3 2 √ 
5−4 
√ 
0 = x2 (x−2) − 4(x−2) 
1 
0 = (x−2)(x2 −4) 
1  1 TRUE
0 = (x−2)(x+2)(x−2)
5
The solution is .
x − 2 = 0 or x + 2 = 0 or x − 2 = 0 3

x = 2 or x = −2 or x=2 33. 2x − 5 = 2

Only 2 checks. The solution is 2. ( 2x − 5)2 = 22
x 4 32 2x − 5 = 4
27. − = 2
x−4 x+4 x − 16 2x = 9
x 4 32 9
− = , x=
x−4 x+4 (x+4)(x−4) 2
LCD is (x+4)(x−4) Check:

 x 4  32 2x − 5 =2
(x+4)(x−4) − = (x+4)(x−4)· 
x−4 x+4 (x+4)(x−4)
9
x(x + 4) − 4(x − 4) = 32 2· −5 ? 2
2 
x + 4x − 4x + 16 = 32
2
√ 
9−5 
x2 + 16 = 32 √ 

4 
x2 = 16 
2  2 TRUE
x = ±4
9
Neither 4 nor −4 checks, so the equation has no solution. The solution is .
2

35. 7−x = 2

( 7 − x)2 = 22
7−x = 4
−x = −3
x=3

Copyright © 2013 Pearson Education, Inc.


114 Chapter 3: Quadratic Functions and Equations; Inequalities

Check: If we do not observe the above fact, we can continue and



7−x =2 reach the same answer.
√ √
7−3 ? 2 ( y − 1)2 = (−4)2
√  y − 1 = 16
4 

2  2 TRUE y = 17
The solution is 3. Check:

√ y−1+4 =0
37. 1 − 2x = 3 √
√ 17 − 1 + 4 ? 0
( 1 − 2x)2 = 32 √ 
1 − 2x = 9
16 + 4 

4 + 4 
−2x = 8
8  0 FALSE
x = −4
Since 17 does not check, there is no solution.
Check: √

1 − 2x =3 45. b+3−2 = 1
 √
b+3 = 3
1 − 2(−4) ? 3 
√  2
1+8  ( b + 3) = 32
√ 
9  b+3 = 9

3  3 TRUE b=6
The solution is −4. Check:
√ √
39. 3
5x − 2 = −3 b+3−2 =1
√ √
( 5x − 2)3 = (−3)3
3
6+3−2 ? 1
√ 
5x − 2 = −27 9−2 

5x = −25 3−2 

1  1 TRUE
x = −5
Check: The solution is 6.
√ √
3
5x − 2 = −3 47. z+2+3 = 4
 √
3
5(−5) − 2 ? −3 z+2 = 1
√  √
3
−25 − 2  ( z + 2)2 = 12
√ 
3
−27  z+2 = 1

−3  −3 TRUE z = −1
The solution is −5. Check:
√ √
41. 4
x2 − 1 = 1 z+2+3 =4
√ √
( x − 1)4 = 14
4 2
−1 + 2 + 3 ? 4
√ 
x −1 = 1
2
1+3 

x2 = 2 1+3 
√ 
4  4 TRUE
x=± 2
Check: The solution is −1.
√ √
4
x2 − 1 = 1 49. 2x + 1 − 3 = 3
 √ √
2x + 1 = 6
4
(± 2)2 − 1 ? 1 √
√ 
 ( 2x + 1)2 = 62
4
2−1 
√  2x + 1 = 36


4
1
 2x = 35
1  1 TRUE
√ 35
The solutions are ± 2. x=
2

43. y−1+4 = 0

y − 1 = −4
The principal square root is never negative. Thus, there is
no solution.

Copyright © 2013 Pearson Education, Inc.


Exercise Set 3.4 115

Check: Check:

2x + 1 − 3 =3 For 0:

 x+4+2 = x
35 √
2· +1−3 ? 3
2  0 + 4 + 2 ? 0
√  
35 + 1 − 3  2+2 
√  
36 − 3  4  0 FALSE

 For 5:
6−3  √
 x+4+2 =x
3  3 TRUE

35 5+4+2 ? 5
The solution is . √ 
2 9+2 
√ 
51. 2−x−4 = 6 3 + 2 
√ 5  5 TRUE
2 − x = 10
√ The number 5 checks but 0 does not. The solution is 5.
( 2 − x)2 = 102

2 − x = 100 57. x−3+5 =x

−x = 98 x−3 = x−5

x = −98 ( x − 3)2 = (x − 5)2
Check:
√ x − 3 = x2 − 10x + 25
2−x−4 =6 0 = x2 − 11x + 28

2 − (−98) − 4 ? 6 0 = (x − 4)(x − 7)
√ 
100 − 4  x − 4 = 0 or x − 7 = 0

10 − 4 
 x = 4 or x=7
6  6 TRUE Check:
The solution is −98. For 4:
√ √
53. 3 6x + 9 + 8 = 5 x−3+5 =x
√ √
3
6x + 9 = −3 4−3+5 ? 4
√ √ 
( 6x + 9)3 = (−3)3 
3
1+5

6x + 9 = −27 1+5 

6  4 FALSE
6x = −36
x = −6 For 7:

x−3+5 =x
Check:

√ 7−3+5 ? 7

3
6x + 9 + 8 = 5 
 4+5 


3
6(−6) + 9 + 8 ? 5
√ 
2+5 
3
−27 + 8  7  7 TRUE

−3 + 8 
 The number 7 checks but 4 does not. The solution is 7.
 5
5 TRUE √
59. x+7 = x+1
The solution is −6. √
√ ( x + 7)2 = (x + 1)2
55. x+4+2 = x
√ x + 7 = x2 + 2x + 1
x+4 = x−2
√ 0 = x2 + x − 6
( x + 4)2 = (x − 2)2
0 = (x + 3)(x − 2)
x + 4 = x2 − 4x + 4
x+3 = 0 or x − 2 = 0
0 = x2 − 5x
x = −3 or x=2
0 = x(x − 5)
x = 0 or x − 5 = 0
x = 0 or x=5

Copyright © 2013 Pearson Education, Inc.


116 Chapter 3: Quadratic Functions and Equations; Inequalities

Check: For 7:

For −3: 5x + 1 = x−1
√ √
x+7 = x+1 5·7+1 ? 7 − 1
√ √ 
−3 + 7 ? −3 + 1 36  6
√  
4  −2 6  6 TRUE

2  −2 FALSE The number 7 checks but 0 does not. The solution is 7.
For 2: √ √
√ 65. x−3+ x+2 = 5
x+7 = x+1 √ √
x+2 = 5− x−3
√ √ √
2+7

? 2 + 1 ( x + 2)2 = (5 − x − 3)2
 √
9  3 x + 2 = 25 − 10 x − 3 + (x − 3)
 √
3  3 TRUE x + 2 = 22 − 10 x − 3 + x
The number 2 checks but −3 does not. The solution is 2. √
10 x − 3 = 20
√ √
61. 3x + 3 = x + 1 x−3 = 2
√ √
( 3x + 3)2 = (x + 1)2 ( x − 3)2 = 22
3x + 3 = x2 + 2x + 1 x−3 = 4
0 = x2 − x − 2 x=7
0 = (x − 2)(x + 1) Check:
√ √
x − 2 = 0 or x + 1 = 0 x−3+ x+2 =5
√ √
x = 2 or x = −1 7−3+ 7+2 ? 5
√ √ |
Check: 4+ 9 

2+3 
For 2: 
√ 5  5 TRUE
3x + 3 = x+1
√ The solution is 7.
3·2+3 ? 2 + 1 √ √
√  67. 3x − 5 + 2x + 3 + 1 = 0
9  3 √ √

3  3 TRUE 3x − 5 + 2x + 3 = −1
For −1: The principal square root is never negative. Thus the sum
√ of two principal square roots cannot equal −1. There is no
3x + 3 = x+1
 solution.
3(−1) + 3 ? −1 + 1 √ √
√  69. x − 3x − 3 = 1
0  0 √ √
 x = 3x − 3 + 1
0  0 TRUE √ √
( x)2 = ( 3x − 3 + 1)2
Both numbers check. The solutions are 2 and −1. √
√ x = (3x − 3) + 2 3x − 3 + 1
63. 5x + 1 = x − 1 √
√ 2 − 2x = 2 3x − 3
( 5x + 1)2 = (x − 1)2 √
1 − x = 3x − 3
5x + 1 = x2 − 2x + 1 √
(1 − x)2 = ( 3x − 3)2
0 = x2 − 7x 1 − 2x + x2 = 3x − 3
0 = x(x − 7) x2 − 5x + 4 = 0
x = 0 or x − 7 = 0 (x − 4)(x − 1) = 0
x = 0 or x=7 x = 4 or x = 1
Check: The number 4 does not check, but 1 does. The solution is
For 0: 1.

5x + 1 = x−1

5·0+1 ? 0 − 1
√ 
1  −1

1  −1 FALSE

Copyright © 2013 Pearson Education, Inc.


Exercise Set 3.4 117

√ √ 1
71. 2y − 5 − y − 3 = 1 83. W =
√ √ LC
2y − 5 = y − 3 + 1  2
√ √ 1
( 2y − 5)2 = ( y − 3 + 1)2 W2 = Squaring both sides
√ LC
2y − 5 = (y − 3) + 2 y − 3 + 1 1
√ W2 =
y−3 = 2 y−3 LC

(y − 3)2 = (2 y − 3)2 1
CW 2 = Multiplying by C
y 2 − 6y + 9 = 4(y − 3) L
y 2 − 6y + 9 = 4y − 12 1
C = Dividing by W 2
LW 2
y − 10y + 21 = 0
2

(y − 7)(y − 3) = 0 1 1 1
85. = +
y = 7 or y = 3 R R1 R2
 
Both numbers check. The solutions are 7 and 3. 1 1 1
RR1 R2 · = RR1 R2 +
√ √ R R1 R2
73. y+4− y−1 = 1 Multiplying by RR1 R2 on both sides
√ √
y+4 = y−1+1
√ √ R1 R2 = RR2 + RR1
( y + 4)2 = ( y − 1 + 1)2
√ R1 R2 − RR2 = RR1 Subtracting RR2 on
y+4 = y−1+2 y−1+1 both sides

4 = 2 y−1 R2 (R1 − R) = RR1 Factoring

2 = y−1 Dividing by 2 R2 =
RR1
Dividing by
√ R1 − R
22 = ( y − 1)2
R1 − R on both sides
4 = y−1

5=y A
87. I = −1
The answer checks. The solution is 5. P

√ √ A
75. x+5+ x+2 = 3 I +1 = Adding 1
√ √ P
x+5 = 3− x+2  2
√ √ A
( x + 5)2 = (3 − x + 2)2 (I + 1)2 =
√ P
x+5 = 9−6 x+2+x+2
√ I 2 + 2I + 1 =
A
−6 = −6 x + 2 P

1 = x+2 Dividing by −6 P (I 2 +2I +1) = A Multplying by P

12 = ( x + 2)2 A
P = 2 Dividing by I 2 +2I +1
1 = x+2 I +2I +1
−1 = x A
We could also express this result as P = .
(I + 1)2
The answer checks. The solution is −1.
77. x1/3 = −2 1 1 1
89. = +
√ F m p
(x1/3 )3 = (−2)3 (x1/3 = 1 1
3
x)
1
x = −8 F mp · = F mp + Multiplying by
F m p F mp on both sides
The value checks. The solution is −8.
79. t1/4 = 3 mp = F p + F m

(t 1/4 4
) =3 4
(t1/4
= 4
t) mp − F p = F m Subtracting F p on
both sides
t = 81
p(m − F ) = F m Factoring
The value checks. The solution is 81. Fm
p= Dividing by m − F on
P1 V1 P2 V 2 m−F
81. = both sides
T1 T2
P1 V1 T2 = P2 V2 T1 Multiplying by T1 T2 on
both sides
P 1 V1 T 2
= T1 Dividing by P2 V2 on
P2 V 2 both sides

Copyright © 2013 Pearson Education, Inc.


118 Chapter 3: Quadratic Functions and Equations; Inequalities

91. 15 − 2x = 0 Setting f (x) = 0 99. x2/3 = x


15 = 2x (x2/3 )3 = x3
15 x2 = x3
= x, or
2 0 = x3 − x2
7.5 = x
0 = x2 (x − 1)
15
The zero of the function is , or 7.5. x = 0 or x − 1 = 0
2
2
93. Familiarize. Let f = the number of highway fatalities x = 0 or x=1
involving distracted driving in 2004. Both numbers check. The solutions are 0 and 1.
Translate.
Fatalities
plus 18%
more is
fatalities Exercise Set 3.5
in 2004 in 2008


   

     1. |x| = 7
f + 0.18f = 5870 The solutions are those numbers whose distance from 0 on
Carry out. We solve the equation. a number line is 7. They are −7 and 7. That is,
f + 0.18f = 5870 x = −7 or x = 7.
1.18f = 5870 The solutions are −7 and 7.
f ≈ 4975 3. |x| = 0
Check. 18% of 4975 is 0.18(4975) ≈ 896 and 4975+896 = The distance of 0 from 0 on a number line is 0. That is,
5871 ≈ 5870. The answer checks. (Remember that we
rounded the value of f .) x = 0.

State. About 4975 highway fatalities involved distracted The solution is 0.


driving in 2004. 5
5. |x| =
6
95. (x − 3)2/3 = 2 5 5
x = − or x =
[(x − 3)2/3 ]3 = 23 6 6
(x − 3)2 = 8 5 5
The solutions are − and .
6 6
x − 6x + 9 = 8
2

x2 − 6x + 1 = 0 7. |x| = −10.7

a = 1, b = −6, c = 1 The absolute value of a number is nonnegative. Thus, the


√ equation has no solution.
−b ± b2 − 4ac
x=
2a 9. |3x| = 1

−(−6) ± (−6)2 − 4 · 1 · 1 3x = −1 or 3x = 1
=
2·1 1 1
√ √ x = − or x =
6 ± 32 6±4 2 3 3
= = 1 1
2 2 The solutions are − and .

2(3 ± 2 2) √ 3 3
= =3±2 2
2 11. |8x| = 24

Both values check. The solutions are 3 ± 2 2. 8x = −24 or 8x = 24
√ 6 √ x = −3 or x=3
97. x+5+1 = √ , LCD is x + 5 The solutions are −3 and 3.
x+5
√ √
x+5+ x+5 = 6 Multiplying by x + 5 13. |x − 1| = 4

x+5 = 1−x x − 1 = −4 or x − 1 = 4
x + 5 = 1 − 2x + x2 x = −3 or x=5
0 = x2 − 3x − 4 The solutions are −3 and 5.
0 = (x − 4)(x + 1)
15. |x + 2| = 6
x = 4 or x = −1
x + 2 = −6 or x + 2 = 6
Only −1 checks. The solution set is −1.
x = −8 or x=4
The solutions are −8 and 4.

Copyright © 2013 Pearson Education, Inc.


Exercise Set 3.5 119

17. |3x + 2| = 1 31. 7 − |2x − 1| = 6


3x + 2 = −1 or 3x + 2 = 1 −|2x − 1| = −1
3x = −3 or 3x = −1 |2x − 1| = 1 Multiplying by −1
1 2x − 1 = −1 or 2x − 1 = 1
x = −1 or x=−
3
2x = 0 or 2x = 2
1
The solutions are −1 and − . x=0 or x=1
3
1  The solutions are 0 and 1.
 
19.  x − 5 = 17
2 33. |x| < 7
1 1
x − 5 = −17 or x − 5 = 17 To solve we look for all numbers x whose distance from
2 2
0 is less than 7. These are the numbers between −7 and
1 1 7. That is, −7 < x < 7. The solution set is (−7, 7). The
x = −12 or x = 22
2 2 graph is shown below.
x = −24 or x = 44
The solutions are −24 and 44. ( )
7 0 7

21. |x − 1| + 3 = 6
35. |x| ≤ 2
|x − 1| = 3
−2 ≤ x ≤ 2
x − 1 = −3 or x − 1 = 3
The solution set is [−2, 2]. The graph is shown below.
x = −2 or x=4
The solutions are −2 and 4.
⫺2 0 2

23. |x + 3| − 2 = 8
37. |x| ≥ 4.5
|x + 3| = 10
To solve we look for all numbers x whose distance from 0 is
x + 3 = −10 or x + 3 = 10 greater than or equal to 4.5. That is, x ≤ −4.5 or x ≥ 4.5.
x = −13 or x=7 The solution set and its graph are as follows.
The solutions are −13 and 7. {x|x ≤ −4.5 or x ≥ 4.5}, or (−∞, −4.5] ∪ [4.5, ∞)

25. |3x + 1| − 4 = −1
|3x + 1| = 3
] [
3x + 1 = −3 or 3x + 1 = 3 4.5 0 4.5

3x = −4 or 3x = 2
39. |x| > 3
4 2
x = − or x= x < −3 or x > 3
3 3
4 2 The solution set is (−∞, −3) ∪ (3, ∞). The graph is shown
The solutions are − and . below.
3 3
27. |4x − 3| + 1 = 7
⫺3 0 3
|4x − 3| = 6
4x − 3 = −6 or 4x − 3 = 6 41. |3x| < 1
4x = −3 or 4x = 9 −1 < 3x < 1
3 9 1 1
x = − or x= − <x< Dividing by 3
3 3
4 4  
3 9 1 1
The solutions are − and . The solution set is − , . The graph is shown below.
4 4 3 3
29. 12 − |x + 6| = 5
−|x + 6| = −7 1 0 1
⫺⫺
3 ⫺
3

|x + 6| = 7 Multiplying by −1
43. |2x| ≥ 6
x + 6 = −7 or x + 6 = 7
2x ≤ −6 or 2x ≥ 6
x = −13 or x=1
x ≤ −3 or x≥3
The solutions are −13 and 1.
The solution set is (−∞, −3] ∪ [3, ∞). The graph is shown
below.

Copyright © 2013 Pearson Education, Inc.


120 Chapter 3: Quadratic Functions and Equations; Inequalities

   
1 7
⫺3 0 3 The solution set is − ∞, − ∪ , ∞ . The graph is
2 2
45. |x + 8| < 9 shown below.
−9 < x + 8 < 9 ⫺ 12 7
2
−17 < x < 1 Subtracting 8
0
The solution set is (−17, 1). The graph is shown below.  
 2  5
57. x + ≤
3 3
⫺17 01
5 2 5
− ≤ x+ ≤
47. |x + 8| ≥ 9 3 3 3
7 2
x + 8 ≤ −9 or x + 8 ≥ 9 − ≤x≤1 Subtracting
3   3
x ≤ −17 or x≥1 Subtracting 8 7
The solution set is (−∞, −17]∪[1, ∞). The graph is shown The solution set is − , 1 . The graph is shown below.
3
below.
7 0 1
⫺⫺
3
⫺17 01

   
 1  1  2x + 1 
49. x − 59.  >5
3 
 <
4 2
1 1 1 2x + 1 2x + 1
− <x− < < −5 or >5
2 4 2 3 3
1 3 1 2x + 1 < −15 or 2x + 1 > 15 Multiplying by 3
− <x< Adding
4 4 4 2x < −16 or 2x > 14 Subtracting 1
 
1 3
The solution set is − , . The graph is shown below. x < −8 or x > 7 Dividing by 2
4 4 The solution set is {x|x < −8 or x > 7}, or
(−∞, −8) ∪ (7, ∞). The graph is shown below.
1 0 3 0
⫺⫺
4 ⫺
4
⫺8 0 7

51. |2x + 3| ≤ 9
−9 ≤ 2x + 3 ≤ 9 61. |2x − 4| < −5
−12 ≤ 2x ≤ 6 Subtracting 3 Since |2x − 4| ≥ 0 for all x, there is no x such that |2x − 4|
would be less than −5. There is no solution.
−6 ≤ x ≤ 3 Dividing by 2
The solution set is [−6, 3]. The graph is shown below. 63. |7 − x| ≥ −4
Since absolute value is nonnegative, for any real-number
value of x we have |7 − x| ≥ 0 ≥ −4. Thus the solution set
⫺6 0 3
is (−∞, ∞). The graph is the entire number line.
53. |x − 5| > 0.1 65. y-intercept
x − 5 < −0.1 or x − 5 > 0.1
67. relation
x < 4.9 or x > 5.1 Adding 5
69. horizontal lines
The solution set is (−∞, 4.9) ∪ (5.1, ∞). The graph is
shown below. 71. decreasing
73. |3x − 1| > 5x − 2
0 4.9 5.1
3x − 1 < −(5x − 2) or 3x − 1 > 5x − 2
55. |6 − 4x| ≥ 8 3x − 1 < −5x + 2 or
1 > 2x
6 − 4x ≤ −8 or 6 − 4x ≥ 8 1
8x < 3 or >x
−4x ≤ −14 or −4x ≥ 2 Subtracting 6 2
14 2 3 1
x≥ or x ≤ − Dividing by −4 and x< or >x
4 4 8  2
  
reversing the inequality symbols 3 1
The solution set is − ∞, ∪ − ∞, . This is equiv-
7 1   8 2
x≥ or x ≤ − Simplifying 1
2 2 alent to − ∞, .
2

Copyright © 2013 Pearson Education, Inc.


Chapter 3 Review Exercises 121

 
75. |p − 4| + |p + 4| < 8 − ∞, −
8
∪ (−2, ∞). This is the solution set of
If p < −4, then |p − 4| = −(p − 4) and |p + 4| = −(p + 4). 3
|x − 3| + |2x + 5| > 6.
Solve: −(p − 4) + [−(p + 4)] < 8
−p + 4 − p − 4 < 8
Chapter 3 Review Exercises
−2p < 8
p > −4
Since this is false for all values of p in the interval (−∞, −4) 1. The statement is true. See page 248 in the text.
there is no solution in this interval.
3. The statement is false. For example, 32 = (−3)2 , but
If p ≥ −4, then |p + 4| = p + 4. 3 = −3.
Solve: |p − 4| + p + 4 < 8
5. (2y + 5)(3y − 1) = 0
|p − 4| < 4 − p
2y + 5 = 0 or 3y − 1 = 0
p − 4 > −(4 − p) and p − 4 < 4 − p
2y = −5 or 3y = 1
p−4 > p−4 and 2p < 8 5 1
y = − or y=
−4 > −4 and p<4 2 3
Since −4 > −4 is false for all values of p, there is no 5 1
The solutions are − and .
solution in the interval [−4, ∞). 2 3
Thus, |p − 4| + |p + 4| < 8 has no solution. 7. 3x2 + 2x = 8
3x2 + 2x − 8 = 0
77. |x − 3| + |2x + 5| > 6
(x + 2)(3x − 4) = 0
Divide the set of real numbers into three intervals:
   
5 5 x+2 = 0 or 3x − 4 = 0
− ∞, − , − , 3 , and [3, ∞).
2 2 x = −2 or 3x = 4
Find the solution set of |x − 3| + |2x + 5| > 6 in each 4
x = −2 or x=
interval. Then find the union of the three solution sets. 3
5 4
If x < − , then |x−3| = −(x−3) and |2x+5| = −(2x+5). The solutions are −2 and .
2 3
5 9. x2 + 10 = 0
Solve: x < − and −(x − 3) + [−(2x + 5)] > 6
2
x2 = −10
5
x < − and −x + 3 − 2x − 5 > 6 √ √
2 x = − −10 or x = −10
√ √
5 x = − 10i or x = 10i
x < − and −3x > 8 √ √
2 The solutions are − 10i and 10i.
5 8
x < − and x<− 11. x2 + 2x − 15 = 0
2 3
 
8 (x + 5)(x − 3) = 0
The solution set in this interval is − ∞, − .
3 x+5 = 0 or x − 3 = 0
5 x = −5 or x=3
If − ≤ x < 3, then |x−3| = −(x−3) and |2x+5| = 2x+5.
2
The zeros of the function are −5 and 3.
5
Solve: − ≤ x < 3 and −(x − 3) + 2x + 5 > 6
2 13. 3x2 + 2x + 3 = 0
5 a = 3, b = 2, c = 3
− ≤ x < 3 and −x + 3 + 2x + 5 > 6 √
2 −b ± b2 − 4ac
5 x=
− ≤ x < 3 and x > −2 2a
2 √
−2 ± 22 − 4 · 3 · 3
The solution set in this interval is (−2, 3). x=
2·3
If x ≥ 3, then |x − 3| = x − 3 and |2x + 5| = 2x + 5. √ √ √
−2 ± −32 −2 ± −16 · 2 −2 ± 4i 2
Solve: x ≥ 3 and x − 3 + 2x + 5 > 6 = = =
2·3 2·3 2·3
x ≥ 3 and 3x > 4 √ √
2(−1 ± 2i 2) −1 ± 2i 2
4 = =
x ≥ 3 and x> 2·3 3
3 √
The solution set in this interval is [3, ∞). −1 ± 2i 2
The zeros of the function are .
3
The union of the above solution sets is

Copyright © 2013 Pearson Education, Inc.


122 Chapter 3: Quadratic Functions and Equations; Inequalities

3 8 √ √ √ √ √
15. + =1 25. − −40 = − −1 · 4 · 10 = −2 10i
8x + 1 2x + 5 √
 LCD is(8x+1)(2x+5) −49 7i 7
27. √ = =−
3 8 − −64 −8i 8
(8x+1)(2x+5) + = (8x+1)(2x+5)·1
8x+1 2x+5
29. (3 − 5i) − (2 − i) = (3 − 2) + [−5i − (−i)]
3(2x + 5) + 8(8x + 1) = (8x+1)(2x+5)
= 1 − 4i
6x + 15 + 64x + 8 = 16x2 + 42x + 5
70x + 23 = 16x2 + 42x + 5 31. 2 − 3i 2 − 3i 1 + 3i
= ·
0 = 16x2 −28x−18 1 − 3i 1 − 3i 1 + 3i
0 = 2(8x2 −14x−9) 2 + 3i − 9i2
=
0 = 2(2x+1)(4x−9) 1 − 9i2
2 + 3i + 9
2x + 1 = 0 or 4x − 9 = 0 =
1+9
2x = −1 or 4x = 9 11 + 3i
=
1 9 10
x = − or x=
2 4 11 3
= + i
1 9 10 10
Both numbers check. The solutions are − and .
2 4 33. x2 − 3x = 18
√ √   2 
17. x−1− x−4 = 1 9 9 1 3 3 9
√ √ x −3x + = 18+
2
(−3) = − and − =
x−1 = x−4+1 4 4 2 2 2 4
√ √  2
( x − 1)2 = ( x − 4 + 1)2 3 81
√ x− =
x−1 = x−4+2 x−4+1 2 4

x−1 = x−3+2 x−4 3 9
√ x− = ±
2 = 2 x−4 2 2
√ 3 9
1 = x−4 Dividing by 2 x= ±
√ 2 2
1 = ( x − 4)
2 2
3 9 3 9
x= − or x = +
1 = x−4 2 2 2 2
5=x x = −3 or x = 6
This number checks. The solution is 5. The solutions are −3 and 6.

19. |2y + 7| = 9 35. 3x2 + 10x = 8


2y + 7 = −9 or 2y + 7 = 9 3x + 10x − 8 = 0
2

2y = −16 or 2y = 2 (x + 4)(3x − 2) = 0
y = −8 or y=1 x+4 = 0 or 3x − 2 = 0
The solutions are −8 and 1. x = −4 or 3x = 2
2
21. |3x + 4| < 10 x = −4 or x=
3
−10 < 3x + 4 < 10 2
The solutions are −4 and .
−14 < 3x < 6 3
14 37. x2 = 10 + 3x
− <x<2
3 x2 − 3x − 10 = 0
 
14
The solution set is − , 2 . The graph is shown below. (x + 2)(x − 5) = 0
3
x+2 = 0 or x − 5 = 0
0 2
x = −2 or x=5
⫺ 14
The solutions are −2 and 5.
3

23. |x + 4| ≥ 2 39. y 4 − 3y 2 + 1 = 0
x + 4 ≤ −2 or x + 4 ≥ 2 Let u = y 2 .
x ≤ −6 or x ≥ −2 u2 − 3u + 1 = 0
 √
The solution is (−∞, −6] ∪ [−2, ∞). −(−3) ± (−3)2 − 4 · 1 · 1 3± 5
u= =
2·1 2

Copyright © 2013 Pearson Education, Inc.


Chapter 3 Review Exercises 123

Substitute y 2 for u and solve for y. Translate. We use the Pythagorean theorem.

3± 5 x2 + (x + 10)2 = 502
y2 =
2 Carry out. We solve the equation.
 √
3± 5 x2 + (x + 10)2 = 502
y=± x + x2 + 20x + 100 = 2500
2
2
 √ 2x2 + 20x − 2400 = 0
3± 5
The solutions are ± . 2(x2 + 10x − 1200) = 0
2
2(x + 40)(x − 30) = 0
41. (p − 3)(3p + 2)(p + 2) = 0
x + 40 = 0 or x − 30 = 0
p − 3 = 0 or 3p + 2 = 0 or p + 2 = 0
x = −40 or x = 30
p = 3 or 3p = −2 or p = −2
Check. Since the length cannot be negative, we need to
2
p = 3 or p = − or p = −2 check only 30. If x = 30, then x+10 = 30+10 = 40. Since
3 302 + 402 = 900 + 1600 = 2500 = 502 , the answer checks.
2
The solutions are −2, − and 3. State. The lengths of the legs are 30 ft and 40 ft.
3
43. f (x) = −4x2 + 3x − 1 51. Familiarize. Using the drawing in the textbook, let w =
  the width of the sidewalk, in ft. Then the length of the
3
= −4 x2 − x − 1 new parking lot is 80 − 2w, and its width is 60 − 2w.
4
  Translate. We use the formula for the area of a rectangle,
3 9 9
= −4 x − x +
2
− −1 A = lw.
4 64 64 2
    New

area is of old

area
3 9 9 3
= −4 x2 − x + −4 − −1 ↓ ↓ ↓ ↓ ↓
4 64 64
  2
3 9 9 (80 − 2w)(60 − 2w) = · 80 · 60
= −4 x2 − x + + −1 3
4 64 16 Carry out. We solve the equation.
 2
3 7 2
= −4 x − − (80 − 2w)(60 − 2w) = · 80 · 60
8 16 3
  2
a) Vertex:
3
,−
7 4800 − 280w + 4w = · 80 · 3 · 20
2

8 16 3
4w2 − 280w + 4800 = 3200
3
b) Axis of symmetry: x = 4w2 − 280w + 1600 = 0
8
7 w2 − 70w + 400 = 0 Dividing by 4
c) Maximum value: −
16 We use the quadratic formula.
  √
7
d) Range: − ∞, − −b ± b2 − 4ac
16 w=
2a
e) 
y −(−70) ± (−70)2 − 4 · 1 · 400
=
2·1

4
70 ± 3300
2
=
2
√ √
⫺4 ⫺2 2 4 x 70 ± 33 · 100 70 ± 10 33
⫺2
= =
√2 2
⫺4 = 35 ± 5 33
√ √
35 + 5 33 ≈ 63.7 and 35 − 5 33 ≈ 6.3
f (x) ⫽ ⫺4x 2 ⫹ 3x ⫺ 1 Check. The width of the sidewalk cannot be 63.7 ft be-
cause this width exceeds the √ width of the original parking
45. The graph of y = (x − 2)2 has vertex (2, 0) and opens up. lot, 60 ft. We check 35 − 5 33 ≈ 6.3. If the width of the
It is graph (d). sidewalk in about 6.3 ft, then the length of the new park-
ing lot is 80 − 2(6.3), or 67.4, and the width is 60 − 2(6.3),
47. The graph of y = −2(x + 3)2 + 4 has vertex (−3, 4) and or 47.4. The area of a parking lot with these dimensions is
opens down. It is graph (b). (67.4)(47.4) = 3194.76. Two-thirds of the area of the origi-
2
49. Familiarize. Using the labels in the textbook, the legs of nal parking lot is · 80 · 60 = 3200. Since 3194.76 ≈ 3200,
the right triangle are represented by x and x + 10. 3
this answer checks.

Copyright © 2013 Pearson Education, Inc.


124 Chapter 3: Quadratic Functions and Equations; Inequalities

√ 

State. The width of the sidewalk is 35 − 5 33 ft, or about 57. x =2
6.3 ft.  
√ 2
53. Familiarize. Using the labels in the textbook, let x = the x = 22
length of the sides of the squares, in cm. Then the length √
x =4
of the base of the box is 20 − 2x and the width of the base √ 2
is 10 − 2x. x = 42

Translate. We use the formula for the area of a rectangle, x = 16
A = lw. √ 2
( x) = 162
90 = (20 − 2x)(10 − 2x)
x = 256
90 = 200 − 60x + 4x2
The answer checks. The solution is 256.
0 = 4x2 − 60x + 110
59. (x − 1)2/3 = 4
0 = 2x2 − 30x + 55 Dividing by 2
(x − 1)2 = 43
We use the quadratic formula. √
x − 1 = ± 64
Carry out.
√ x − 1 = ±8
−b ± b2 − 4ac
x= x − 1 = −8 or x − 1 = 8
2a
 x = −7 or x=9
−(−30) ± (−30)2 − 4 · 2 · 55
= Both numbers check. The solutions are −7 and 9.
2·2
√ √ √
30 ± 460 61. x + 2 + 4 x + 2 − 2 = 0
= √ √ √
4 Let u = 4 x + 2, so u2 = ( 4 x + 2)2 = x + 2.
√ √
30 ± 4 · 115 30 ± 2 115 u2 + u − 2 = 0
= =
4 4 (u + 2)(u − 1) = 0

15 ± 115 u = −2 or u = 1
= √
2 Substitute 4 x + 2 for u and solve for x.
√ √ √ √
15 + 115 15 − 115 4
x + 2 = −2 or 4 x + 2 = 1
≈ 12.9 and ≈ 2.1.
2 2 No real solution x+2 = 1
Check. The length of the sides of the squares cannot
be 12.9 cm because this length exceeds the width of the x = −1
piece of aluminum. We check 2.1 cm. If the sides of the This number checks. The solution is −1.
squares are 2.1 cm, then the length of the base of the box is
20 − 2(2.1) = 15.8, and the width is 10 − 2(2.1) = 5.8. The  coordi-
63. The maximum value occurs at the vertex. The first 
b b b b
area of the base is 15.8(5.8) = 91.64 ≈ 90. This answer nate of the vertex is − =− = and f = 2.
2a 2(−3) 6 6
checks.  2  
b b
State. The length of the sides of the squares is −3 +b −1 = 2
√ 6 6
15 − 115
cm, or about 2.1 cm. b2 b2
2 − + −1 = 2
√ √ 12 6
55. 4x + 1 + 2x = 1 −b2 + 2b2 − 12 = 24
√ √
4x + 1 = 1 − 2x b2 = 36
√ √
( 4x + 1)2 = (1 − 2x)2 b = ±6

4x + 1 = 1 − 2 2x + 2x 65. No; consider the quadratic formula
√ √
2x = −2 2x −b ± b2 − 4ac −b
√ x= . If b2 − 4ac = 0, then x = , so
x = − 2x 2a √ 2a
√ there is one real zero. If b − 4ac > 0, then b − 4ac is a
2 2
x2 = (− 2x)2
real number
√ and there are two real zeros. If b2 − 4ac < 0,
x2 = 2x then b2 − 4ac is an imaginary number and there are two
x − 2x = 0
2 imaginary zeros. Thus, a quadratic function cannot have
x(x − 2) = 0 one real zero and one imaginary zero.
x = 0 or x = 2 67. When both sides of an equation are multiplied by the LCD,
Only 0 checks, so answer B is correct. the resulting equation might not be equivalent to the orig-
inal equation. One or more of the possible solutions of
the resulting equation might make a denominator of the
original equation 0.

Copyright © 2013 Pearson Education, Inc.


Chapter 3 Test 125


69. Absolute value is nonnegative. 7. x + 5 x − 36 = 0

Let u = x.
Chapter 3 Test u2 + 5u − 36 = 0
(u + 9)(u − 4) = 0
1. (2x − 1)(x + 5) = 0 u+9 = 0 or u − 4 = 0
2x − 1 = 0 or x + 5 = 0 u = −9 or u=4

2x = 1 or x = −5 Substitute x for u and solve for x.
1 √ √
x= or x = −5 x = −9 or x=4
2
No solution x = 16
1
The solutions are and −5. The number 16 checks. It is the solution.
2
2. 6x2 − 36 = 0 3 2
8. + = 2, LCD is (3x+4)(x−1)
2 3x + 4 x−1
6x = 36  
3 2
x2 = 6 (3x+4)(x−1) + = (3x+4)(x−1)(2)
√ √ 3x+4 x−1
x = − 6 or x = 6 3(x − 1) + 2(3x + 4) = 2(3x2 + x − 4)
√ √
The solutions are − 6 and 6. 3x − 3 + 6x + 8 = 6x2 + 2x − 8
3. 2
x +4 = 0 9x + 5 = 6x2 + 2x − 8
x = −4 2
0 = 6x2 − 7x − 13

x = ± −4 0 = (x + 1)(6x − 13)
x = −2i or x = 2i x+1 = 0 or 6x − 13 = 0
The solutions are −2i and 2i. x = −1 or 6x = 13
4. x − 2x − 3 = 0
2 13
x = −1 or x=
6
(x + 1)(x − 3) = 0
13
x+1 = 0 or x − 3 = 0 Both numbers check. The solutions are −1 and .
6
x = −1 or x=3 √
9. x+4−2 = 1
The solutions are −1 and 3. √
x+4 = 3

5. x2 − 5x + 3 = 0 ( x + 4)2 = 32
a = 1, b = −5, c = 3 x+4 = 9

−b ± b2 − 4ac x=5
x=
2a This number checks. The solution is 5.

−(−5) ± (−5)2 − 4 · 1 · 3 √ √
x= 10. x+4− x−4 = 2
2·1 √ √
√ x+4 = x−4+2
5 ± 13 √ √
= ( x + 4)2 = ( x − 4 + 2)2
2
√ √ √
5 + 13 5 − 13 x+4 = x−4+4 x−4+4
The solutions are and . √
2 2 4= 4 x−4

6. 2t − 3t + 4 = 0
2 1= x−4

a = 2, b = −3, c = 4 12 = ( x − 4)2

−b ± b2 − 4ac 1 = x−4
x=
2a 5=x

−(−3) ± (−3)2 − 4 · 2 · 4 This number checks. The solution is 5.
x=
2·2 11. |x + 4| = 7
√ √
3 ± −23 3 ± i 23 x + 4 = −7 or x + 4 = 7
= =
4 4
√ x = −11 or x=3
3 23 The solutions are −11 and 3.
= ± i
4 4
√ √
3 23 3 23
The solutions are + i and − i.
4 4 4 4

Copyright © 2013 Pearson Education, Inc.


126 Chapter 3: Quadratic Functions and Equations; Inequalities

12. |4y − 3| = 5 19. x2 + 4x = 1


 
4y − 3 = −5 or 4y − 3 = 5 1
x2 + 4x + 4 = 1 + 4 (4) = 2 and 22 = 4
4y = −2 or 4y = 8 2
1 (x + 2)2 = 5
y = − or y=2 √
2 x+2 = ± 5
1 √
The solutions are − and 2. x = −2 ± 5
2 √ √
The solutions are −2 + 5 and −2 − 5.
13. |x + 3| ≤ 4
−4 ≤ x + 3 ≤ 4 20. Familiarize and Translate. We will use the formula
s = 16t2 , substituting 2063 for s.
−7 ≤ x ≤ 1
2063 = 16t2
The solution set is [−7, 1].
Carry out. We solve the equation.
2063 = 16t2
2063
= t2
14. |2x − 1| < 5 16
−5 < 2x − 1 < 5 11.4 ≈ t
−4 < 2x < 6 Check. When t = 11.4, s = 16(11.4)2 = 2079.36 ≈ 2063.
The answer checks.
−2 < x < 3
State. It would take an object about 11.4 sec to reach the
The solution set is (−2, 3).
ground.
√ √ √ √ √
⫺2 0 3 21. −43 = −1 · 43 = i 43, or 43i
√ √ √
15. |x + 5| > 2 22. − −25 = − −1 · 25 = −5i

x + 5 < −2 or x + 5 > 2 23. (5 − 2i) − (2 + 3i) = (5 − 2) + (−2i − 3i)


x < −7 or x > −3 = 3 − 5i
The solution set is (−∞, −7) ∪ (−3, ∞). 24. (3 + 4i)(2 − i) = 6 − 3i + 8i − 4i2
= 6 + 5i + 4 (i2 = −1)
= 10 + 5i
16. |3 − 2x| ≥ 7 1−i 1 − i 6 − 2i
25. = ·
3 − 2x ≤ −7 or 3 − 2x ≥ 7 6 + 2i 6 + 2i 6 − 2i
−2x ≤ −10 or −2x ≥ 4 6 − 2i − 6i + 2i2
=
x≥5 or x ≤ −2 36 − 4i2
The solution set is (−∞, −2] ∪ [5, ∞). 6 − 8i − 2
=
36 + 4
4 − 8i
2 0 5 =
40
1 1 1 4 8
17. + = = − i
A B C 40 40
  1 1
ABC
1
+
1
= ABC ·
1 = − i
A B C 10 5
BC + AC = AB 26. i33 = (i2 )16 · i = (−1)16 · i = 1 · i = i
AC = AB − BC 27. 4x2 − 11x − 3 = 0
AC = B(A − C) (4x + 1)(x − 3) = 0
AC
=B 4x + 1 = 0 or x − 3 = 0
A−C
√ 4x = −1 or x=3
18. R= 3np
√ 1
x = − or x=3
R = ( 3np)2
2
4
2 1
R = 3np
The zeros of the functions are − and 3.
R 2 4
=n
3p

Copyright © 2013 Pearson Education, Inc.


Chapter 3 Test 127

28. 2x2 − x − 7 = 0 can find A(w) for a value of w less than 20 and for a value
a = 2, b = −1, c = −7 of w greater than 20. For instance, A(19.9) = 799.98 and
√ A(20.1) = 799.98. Since both of these values are less than
−b ± b2 − 4ac 800, the result appears to be correct.
x=
2a
 State. The dimensions for which the area is a maximum
−(−1) ± (−1)2 − 4 · 2 · (−7) are 20 ft by 40 ft.
x=
2·2
√ 31. f (x) = x2 − 2x − 1
1 ± 57
= = (x2 − 2x + 1 − 1) − 1 Completing the square
4
√ √ = (x − 2x + 1) − 1 − 1
2
1 + 57 1 − 57
The solutions are and .
4 4 = (x − 1)2 − 2
29. f (x) = −x2 + 2x + 8 The graph of this function opens up and has vertex (1, −2).
= −(x − 2x) + 8
2 Thus the correct graph is C.
= −(x2 − 2x + 1 − 1) + 8  coordi-
32. The maximum value occurs at the vertex. The first
b (−4) 2 2
= −(x − 2x + 1) − (−1) + 8
2
nate of the vertex is − =− = and f = 12.
2a 2a a a
= −(x2 − 2x + 1) + 1 + 8 Then we have:
 2  
= −(x − 1)2 + 9 2 2
a −4 + 3 = 12
a) Vertex: (1, 9) a a
b) Axis of symmetry: x = 1 4 8
a · 2 − + 3 = 12
c) Maximum value: 9 a a
4 8
d) Range: (−∞, 9] − + 3 = 12
a a
e) 4
y − + 3 = 12
a
4
8 − =9
a
6
−4 = 9a
4
4
2 − =a
9
⫺4 ⫺2 2 4 x

f (x) ⫽ ⫺x 2 ⫹ 2x ⫹ 8

30. Familiarize. We make a drawing, letting w = the width


of the rectangle, in ft. This leaves 80 − w − w, or 80 − 2w ft
of fencing for the length.
House

w w

80 − 2w

Translating. The area of a rectangle is given by length


times width.
A(w) = (80 − 2w)w
= 80w − 2w2 , or − 2w2 + 80w
Carry out. This is a quadratic function with a < 0, so
it has a maximum value that occurs at the vertex of the
graph of the function. The first coordinate of the vertex is
b 80
w=− =− = 20.
2a 2(−2)
If w = 20, then 80 − 2w = 80 − 2 · 20 = 40.
Check. The area of a rectangle with length 40 ft and
width 20 ft is 40 · 20, or 800 ft2 . As a partial check, we

Copyright © 2013 Pearson Education, Inc.


Copyright © 2013 Pearson Education, Inc.
Chapter 4
Polynomial and Rational Functions
19. f (x) = −x6 + 2x5 − 7x2
Exercise Set 4.1 The leading term is −x6 . The degree, 6, is even and the
leading coefficient, −1, is negative. Thus, (c) is the correct
1 3 graph.
1. g(x) = x − 10x + 8
2 1
1 1 21. f (x) = x5 + x−3
The leading term is x3 and the leading coefficient is . 10
2 2 The leading term is x5 . The degree, 5, is odd and the
The degree of the polynomial is 3, so the polynomial is
cubic. leading coefficient, 1, is positive. Thus, (d) is the correct
graph.
3. h(x) = 0.9x − 0.13
23. f (x) = x3 − 9x2 + 14x + 24
The leading term is 0.9x and the leading coefficient is 0.9.
The degree of the polynomial is 1, so the polynomial is f (4) = 43 − 9 · 42 + 14 · 4 + 24 = 0
linear. Since f (4) = 0, 4 is a zero of f (x).
4
5. g(x) = 305x + 4021 f (5) = 53 − 9 · 52 + 14 · 5 + 24 = −6
4
The leading term is 305x and the leading coefficient is Since f (5) = 0, 5 is not a zero of f (x).
305. The degree of the polynomial is 4, so the polynomial
is quartic. f (−2) = (−2)3 − 9(−2)2 + 14(−2) + 24 = −48
Since f (−2) = 0, −2 is not a zero of f (x).
7. h(x) = −5x2 + 7x3 + x4 = x4 + 7x3 − 5x2
The leading term is x4 and the leading coefficient is 1 (x4 = 25. g(x) = x4 − 6x3 + 8x2 + 6x − 9
1·x4 ). The degree of the polynomial is 4, so the polynomial g(2) = 24 − 6 · 23 + 8 · 22 + 6 · 2 − 9 = 3
is quartic.
Since g(2) = 0, 2 is not a zero of g(x).
1
9. g(x) = 4x − x2 + 8
3
g(3) = 34 − 6 · 33 + 8 · 32 + 6 · 3 − 9 = 0
2
The leading term is 4x3 and the leading coefficient is 4. Since g(3) = 0, 3 is a zero of g(x).
The degree of the polynomial is 3, so the polynomial is
g(−1) = (−1)4 − 6(−1)3 + 8(−1)2 + 6(−1) − 9 = 0
cubic.
Since g(−1) = 0, −1 is a zero of g(x).
11. f (x) = −3x3 − x + 4
27. f (x) = (x + 3)2 (x − 1) = (x + 3)(x + 3)(x − 1)
The leading term is −3x3 . The degree, 3, is odd and the
leading coefficient, −3, is negative. Thus the end behavior To solve f (x) = 0 we use the principle of zero products,
of the graph is like that of (d). solving x + 3 = 0 and x − 1 = 0. The zeros of f (x) are −3
and 1.
3
13. f (x) = −x6 + x4 The factor x + 3 occurs twice. Thus the zero −3 has a
4
multiplicity of two.
The leading term is −x6 . The degree, 6, is even and the
leading coefficient, −1, is negative. Thus the end behavior The factor x − 1 occurs only one time. Thus the zero 1 has
of the graph is like that of (b). a multiplicity of one.

15. f (x) = −3.5x4 + x6 + 0.1x7 = 0.1x7 + x6 − 3.5x4 29. f (x) = −2(x − 4)(x − 4)(x − 4)(x + 6) = −2(x − 4)3 (x + 6)
The leading term is 0.1x7 . The degree, 7, is odd and the To solve f (x) = 0 we use the principle of zero products,
leading coefficient, 0.1, is positive. Thus the end behavior solving x − 4 = 0 and x + 6 = 0. The zeros of f (x) are 4
of the graph is like that of (c). and −6.

1 4 2 3 1 4 2 3 The factor x − 4 occurs three times. Thus the zero 4 has


17. f (x) = 10 + x − x = x − x + 10 a multiplicity of 3.
10 5 10 5
1 4 The factor x + 6 occurs only one time. Thus the zero −6
The leading term is x . The degree, 4, is even and the has a multiplicity of 1.
10
1
leading coefficient, , is positive. Thus the end behavior
10
of the graph is like that of (a).

Copyright © 2013 Pearson Education, Inc.


130 Chapter 4: Polynomial and Rational Functions

31. f (x) = (x2 − 9)3 = [(x + 3)(x − 3)]3 = (x + 3)3 (x − 3)3 45. Graph y = x4 − 2x2 and use the Zero feature three times.
To solve f (x) = 0 we use the principle of zero products, The real-number zeros are about −1.414, 0, and 1.414.
solving x + 3 = 0 and x − 3 = 0. The zeros of f (x) are −3 47. Graph y = x3 − x and use the Zero feature three times.
and 3. The real-number zeros are −1, 0, and 1.
The factors x + 3 and x − 3 each occur three times so each
zero has a multiplicity of 3. 49. Graph y = x8 + 8x7 − 28x6 − 56x5 + 70x4 + 56x3 − 28x2 −
8x + 1 and use the Zero feature eight times. The real-
33. f (x) = x3 (x − 1)2 (x + 4) number zeros are about −10.153, −1.871, −0.821, −0.303,
To solve f (x) = 0 we use the principle of zero products, 0.098, 0.535, 1.219, and 3.297.
solving x = 0, x − 1 = 0, and x + 4 = 0. The zeros of f (x) 51. g(x) = x3 − 1.2x + 1
are 0, 1, and −4.
Graph the function and use the Zero, Maximum, and Min-
The factor x occurs three times. Thus the zero 0 has a imum features.
multiplicity of three.
Zero: −1.386
The factor x − 1 occurs twice. Thus the zero 1 has a
multiplicity of two. Relative maximum: 1.506 when x ≈ −0.632
The factor x + 4 occurs only one time. Thus the zero −4 Relative minimum: 0.494 when x ≈ 0.632
has a multiplicity of one. Range: (−∞, ∞)
35. f (x) = −8(x − 3) (x + 4) x
2 3 4
53. f (x) = x6 − 3.8
To solve f (x) = 0 we use the principle of zero products, Graph the function and use the Zero and Minimum fea-
solving x − 3 = 0, x + 4 = 0, and x = 0. The zeros of f (x) tures.
are 3, −4, and 0. Zeros: −1.249, 1.249
The factor x − 3 occurs twice. Thus the zero 3 has a There is no relative maximum.
multiplicity of 2.
Relative minimum: −3.8 when x = 0
The factor x + 4 occurs three times. Thus the zero −4 has
a multiplicity of 3. Range: [−3.8, ∞)
The factor x occurs four times. Thus the zero 0 has a 55. f (x) = x2 + 10x − x5
multiplicity of 4. Graph the function and use the Zero, Maximum, and Min-
37. f (x) = x4 − 4x2 + 3 imum features.
We factor as follows: Zeros: −1.697, 0, 1.856
f (x) = (x2 − 3)(x2 − 1) Relative maximum: 11.012 when x ≈ 1.258
√ √
= (x − 3)(x + 3)(x − 1)(x + 1) Relative minimum: −8.183 when x ≈ −1.116
√ √ Range: (−∞, ∞)
The zeros of the function are 3, − 3, 1, and −1. Each
has a multiplicity of 1. 57. Graphing the function, we see that the graph touches the
39. f (x) = x + 3x − x − 3
3 2 x-axis at (3, 0) but does not cross it, so the statement is
false.
We factor by grouping:
f (x) = x2 (x + 3) − (x + 3) 59. Graphing the function, we see that the statement is true.
= (x − 1)(x + 3)
2
61. For 1995, x = 1995 − 1990 = 5.
= (x − 1)(x + 1)(x + 3) f (5) = −0.056316(5)4 − 19.500154(5)3 + 584.892054(5)2 −
The zeros of the function are 1, −1, and −3. Each has a 1518.5717(5)+94, 299.1990 ≈ 98, 856 twin births
multiplicity of 1. For 2005, x = 2005 − 1990 = 15.
41. f (x) = 2x − x − 8x + 4
3 2 f (15) = −0.056316(15)4−19.500154(15)3+584.892054(15)2−
1518.5717(15)+94, 299.1990 ≈ 134, 457 twin births
= x2 (2x − 1) − 4(2x − 1)
= (2x − 1)(x2 − 4) 63. d(3) = 0.010255(3)3 −0.340119(3)2 +7.397499(3)+
= (2x − 1)(x + 2)(x − 2) 6.618361 ≈ 26 yr
1 d(12) = 0.010255(12)3 −0.340119(12)2 +7.397499(12)+
The zeros of the function are , −2, and 2. Each has a 6.618361 ≈ 64 yr
2
multiplicity of 1. d(16) = 0.010255(16)3 −0.340119(16)2 +7.397499(16)+
6.618361 ≈ 80 yr
43. Graph y = x3 − 3x − 1 and use the Zero feature three
times. The real-number zeros are about −1.532, −0.347,
and 1.879.

Copyright © 2013 Pearson Education, Inc.


Exercise Set 4.1 131

65. We substitute 294 for s(t) and solve for t. Quartic: y = 3.968531469x4 − 103.3181818x3 +
489.0064103x2 + 502.8350816x + 18, 108.04196;
294 = 4.9t2 + 34.3t
R2 ≈ 0.9815
0 = 4.9t2 + 34.3t − 294
√ Using the R2 -values, we see that the quartic func-
−b ± b2 − 4ac tion provides the best fit.
t=
2a b) In 2010, x = 2010 − 2000 = 10. Evaluating the

−34.3 ± (34.3)2 − 4(4.9)(−294) quartic function for x = 10, we estimate that there
= were about 8404 U.S. foreign adoptions in 2010.
2(4.9)

−34.3 ± 6938.89 79. a) For each function x represents the number of years
=
9.8 after 1975 and y is in billions of dollars.
t = 5 or t = −12 Cubic: y = −0.0029175919x3 + 0.1195127076x2 −
Only the positive number has meaning in the situation. It 0.5287827297x + 3.087289783; R2 ≈ 0.9023
will take the stone 5 sec to reach the ground. Quartic: y = −0.0001884254x4 + 0.0099636973x3 −
0.1550252422x2 + 1.300895521x + 1.77274528;
67. For 2002, x = 2002 − 2000 = 2.
R2 ≈ 0.9734
h(2) = 56.8328(2)4 − 1554.7494(2)3 + 10, 451.8211(2)2 −
Using the R2 -values, we see that the quartic func-
5655.7692(2) + 140, 589.1608 ≈ $159, 556
tion provides the better fit.
For 2005, x = 2005 − 2000 = 5.
b) In 1988, x = 1988 − 1975 = 13. Evaluating the
h(5) = 56.8328(5)4 − 1554.7494(5)3 + 10, 451.8211(5)2 − quartic function for x = 13, we estimate that the
5655.7692(5) + 140, 589.1608 ≈ $214, 783 revenue was about $8.994 billion in 1988.
For 2008, x = 2008 − 2000 = 8. In 2002, x = 2002 − 1975 = 27. Evaluating the
h(8) = 56.8328(8)4 − 1554.7494(8)3 + 10, 451.8211(8)2 − quartic function for x = 27, we estimate that the
5655.7692(8) + 140, 589.1608 ≈ $201, 015 revenue was about $19.862 billion in 2002.
For 2009, x = 2009 − 2000 = 9. In 2010, x = 2010 − 1975 = 35. Evaluating the
quartic function for x = 35, we estimate that the
h(9) = 56.8328(9)4 − 1554.7494(9)3 + 10, 451.8211(9)2 −
revenue was about $1.836 billion in 2010.
5655.7692(9) + 140, 589.1608 ≈ $175, 752

81. d = (x2 − x1 )2 + (y2 − y1 )2
69. A = P (1 + i)t 
9039.75 = 8000(1 + i)2 Substituting = [−1 − (−5)]2 + (0 − 3)2
 √
9039.75 = 42 + (−3)2 = 16 + 9
= (1 + i)2 √
8000 = 25 = 5
±1.063 ≈ 1 + i Taking the square root
83. (x − 3)2 + (y + 5)2 = 49
on both sides
−1 ± 1.063 ≈ i
(x − 3)2 + [y − (−5)]2 = 72
−1 + 1.063 ≈ i or −1 − 1.063 ≈ i Center: (3, −5); radius: 7
0.063 ≈ i or −2.063 ≈ i
85. 2y − 3 ≥ 1 − y + 5
Only the positive result has meaning in this application.
The interest rate is about 0.063, or 6.3%. 2y − 3 ≥ 6 − y Collecting like terms
3y − 3 ≥ 6 Adding y
71. The sales drop and then rise. This suggests that a
quadratic function that opens up might fit the data. Thus, 3y ≥ 9 Adding 3
we choose (b). y≥3 Dividing by 3

73. The sales rise and then drop. This suggests that a The solution set is {y|y ≥ 3}, or [3, ∞).
quadratic function that opens down might fit the data. 87. |x + 6| ≥ 7
Thus, we choose (c).
x + 6 ≤ −7 or x + 6 ≥ 7
75. The sales fall steadily. Thus, we choose (a), a linear func- x ≤ −13 or x≥1
tion.
The solution set is {x|x ≤ −13 or x ≥ 1}, or
77. a) For each function, x represents the number of years (−∞, −13] ∪ [1, ∞).
after 2000.
89. f (x) = (x5 − 1)2 (x2 + 2)3
Quadratic: y = −342.2575758x2 + 2687.051515x +
17, 133.10909; R2 ≈ 0.9388 The leading term of (x5 − 1)2 is (x5 )2 , or x10 . The leading
term of (x2 + 2)3 is (x2 )3 , or x6 . Then the leading term of
Cubic: y = −31.88461538x3 + 88.18473193x2 + f (x) is x10 · x6 , or x16 , and the degree of f (x) is 16.
1217.170746x + 17, 936.6014; R2 ≈ 0.9783

Copyright © 2013 Pearson Education, Inc.


132 Chapter 4: Polynomial and Rational Functions

−x2 = 0 or x + 2 = 0
Exercise Set 4.2 x2 = 0 or x = −2
x = 0 or x = −2
1. f (x) = x5 − x2 + 6 The zeros of the function are 0 and −2, so the x-
a) This function has degree 5, so its graph can have at intercepts of the graph are (0, 0) and (−2, 0).
most 5 real zeros. 3. The zeros divide the x-axis into 3 intervals, (−∞, −2),
b) This function has degree 5, so its graph can have at (−2, 0), and (0, ∞). We choose a value for x from each
most 5 x-intercepts. interval and find f (x). This tells us the sign of f (x)
for all values of x in that interval.
c) This function has degree 5, so its graph can have at
In (−∞, −2), test −3:
most 5 − 1, or 4, turning points.
f (−3) = −(−3)3 − 2(−3)2 = 9 > 0
3. f (x) = x10 − 2x5 + 4x − 2
In (−2, 0), test −1:
a) This function has degree 10, so its graph can have
f (−1) = −(−1)3 − 2(−1)2 = −1 < 0
at most 10 real zeros.
In (0, ∞), test 1:
b) This function has degree 10, so its graph can have
at most 10 x-intercepts. f (1) = −13 − 2 · 12 = −3 < 0
c) This function has degree 10, so its graph can have Thus the graph lies above the x-axis on (−∞, −2)
at most 10 − 1, or 9, turning points. and below the x-axis on (−2, 0) and (0, ∞). We also
know the points (−3, 9), (−1, −1), and (1, −3) are
5. f (x) = −x − x3 = −x3 − x on the graph.
a) This function has degree 3, so its graph can have at 4. From Step 2 we see that the y-intercept is (0, 0).
most 3 real zeros.
5. We find additional points on the graph and then
b) This function has degree 3, so its graph can have at draw the graph.
most 3 x-intercepts.
x f (x) y
c) This function has degree 3, so its graph can have at
most 3 − 1, or 2, turning points. −2.5 3.125 4
f(x )  x 3  2x 2
2
1 −1.5 −1.125
7. f (x) = x2 − 5
4 4 2 2 4 x
1 1.5 −7.875 2
The leading term is x2 . The sign of the leading coef-
4 4
1
ficient, , is positive and the degree, 2, is even, so we
4
would choose either graph (b) or graph (d). Note also
that f (0) = −5, so the y-intercept is (0, −5). Thus, graph 6. Checking the graph as described on page 317 in the
(d) is the graph of this function. text, we see that it appears to be correct.

9. f (x) = x5 − x4 + x2 + 4 15. h(x) = x2 + 2x − 3


The leading term is x5 . The sign of the leading coefficient, 1. The leading term is x2 . The degree, 2, is even
1, is positive and the degree, 5, is odd. Thus, graph (f) is and leading coefficient, 1, is positive so as x → ∞,
the graph of this function. h(x) → ∞ and as x → −∞, h(x) → ∞.
2. We solve h(x) = 0.
11. f (x) = x4 − 2x3 + 12x2 + x − 20
x2 + 2x − 3 = 0
The leading term is x4 . The sign of the leading coefficient,
1, is positive and the degree, 4, is even, so we would choose (x + 3)(x − 1) = 0
either graph (b) or graph (d). Note also that f (0) = −20, x+3 = 0 or x − 1 = 0
so the y-intercept is (0, −20). Thus, graph (b) is the graph x = −3 or x=1
of this function.
The zeros of the function are −3 and 1, so the x-
13. f (x) = −x3 − 2x2 intercepts of the graph are (−3, 0) and (1, 0).
1. The leading term is −x3 . The degree, 3, is odd and 3. The zeros divide the x-axis into 3 intervals, (−∞, −3),
the leading coefficient, −1, is negative so as x → ∞, (−3, 1), and (1, ∞). We choose a value for x from each
f (x) → −∞ and as x → −∞, f (x) → ∞. interval and find h(x). This tells us the sign of h(x)
for all values of x in that interval.
2. We solve f (x) = 0.
In (−∞, −3), test −4:
−x3 − 2x2 = 0
h(−4) = (−4)2 + 2(−4) − 3 = 5 > 0
−x2 (x + 2) = 0
In (−3, 1), test 0:
h(0) = 02 + 2 · 0 − 3 = −3 < 0

Copyright © 2013 Pearson Education, Inc.


Exercise Set 4.2 133

In (1, ∞), test 2: 5. We find additional points on the graph and then
h(2) = 22 + 2 · 2 − 3 = 5 > 0 draw the graph.
y
Thus the graph lies above the x-axis on (−∞, −3) x h(x)
and on (1, ∞). It lies below the x-axis on (−3, 1). 8
We also know the points (−4, 5), (0, −3), and (2, 5) −2.5 −35.2
4
are on the graph.
−1.5 5.9
4 2 2 4
4. From Step 3 we see that the y-intercept is (0, −3). x
4
1.5 −5.9
5. We find additional points on the graph and then
8
draw the graph. 2.5 35.2
x h(x) y h(x)  x 5  4x 3

−2 −3 4
6. Checking the graph as described on page 317 in the
2
−1 −4 text, we see that it appears to be correct.
4 2 2 4 x 19. h(x) = x(x − 4)(x + 1)(x − 2)
3 12 2
1. The leading term is x · x · x · x, or x4 . The degree, 4,
4 h(x)  x 2  2x  3 is even and the leading coefficient, 1, is positive so
as x → ∞, h(x) → ∞ and as x → −∞, h(x) → ∞.
2. We see that the zeros of the function are 0, 4, −1,
6. Checking the graph as described on page 317 in the and 2 so the x-intercepts of the graph are (0, 0),
text, we see that it appears to be correct. (4, 0), (−1, 0), and (2, 0).
17. h(x) = x5 − 4x3 3. The zeros divide the x-axis into 5 intervals, (−∞, −1),
(−1, 0), (0, 2), (2, 4), and (4, ∞). We choose a value
1. The leading term is x5 . The degree, 5, is odd and for x from each interval and find h(x). This tells us
the leading coefficient, 1, is positive so as x → ∞, the sign of h(x) for all values of x in that interval.
h(x) → ∞ and as x → −∞, h(x) → −∞. In (−∞, −1), test −2:
2. We solve h(x) = 0. h(−2) = −2(−2 − 4)(−2 + 1)(−2 − 2) = 48 > 0
x5 − 4x3 = 0 In (−1, 0), test −0.5:
x3 (x2 − 4) = 0 h(−0.5) = (−0.5)(−0.5 − 4)(−0.5 + 1)(−0.5 − 2) =
x (x + 2)(x − 2) = 0
3
−2.8125 < 0
x3 = 0 or x + 2 = 0 or x − 2 = 0 In (0, 2), test 1:
x = 0 or x = −2 or x=2 h(1) = 1(1 − 4)(1 + 1)(1 − 2) = 6 > 0
The zeros of the function are 0, −2, and 2 so the x- In (2, 4), test 3:
intercepts of the graph are (0, 0), (−2, 0), and (2, 0).
h(3) = 3(3 − 4)(3 + 1)(3 − 2) = −12 < 0
3. The zeros divide the x-axis into 4 intervals, (−∞, −2),
In (4, ∞), test 5:
(−2, 0), (0, 2), and (2, ∞). We choose a value for x
from each interval and find h(x). This tells us the h(5) = 5(5 − 4)(5 + 1)(5 − 2) = 90 > 0
sign of h(x) for all values of x in that interval. Thus the graph lies above the x-axis on (−∞, −1),
In (−∞, −2), test −3: (0, 2), and (4, ∞). It lies below the x-axis on (−1, 0)
and on (2, 4). We also know the points (−2, 48),
h(−3) = (−3)5 − 4(−3)3 = −135 < 0
(−0.5, −2.8125), (1, 6), (3, −12), and (5, 90) are on
In (−2, 0), test −1: the graph.
h(−1) = (−1)5 − 4(−1)3 = 3 > 0 4. From Step 2 we see that the y-intercept is (0, 0).
In (0, 2), test 1: 5. We find additional points on the graph and then
h(1) = 15 − 4 · 13 = −3 < 0 draw the graph.
y
In (2, ∞), test 3: x h(x)
h(3) = 35 − 4 · 33 = 135 > 0 −1.5 14.4
8

Thus the graph lies below the x-axis on (−∞, −2) 4


and on (0, 2). It lies above the x-axis on (−2, 0) and 1.5 4.7
4 2 2 4 6 x
on (2, ∞). We also know the points (−3, −135), 4
2.5 −6.6
(−1, 3), (1, −3), and (3, 135) are on the graph.
8
4. From Step 2 we see that the y-intercept is (0, 0). 4.5 30.9
12

h(x)  x(x  4)(x  1)(x  2)

Copyright © 2013 Pearson Education, Inc.


134 Chapter 4: Polynomial and Rational Functions

6. Checking the graph as described on page 317 in the 2. We solve f (x) = 0.


text, we see that it appears to be correct.
−x4 − 2x3 = 0
1 3 −x3 (x + 2) = 0
21. g(x) = − x3 − x2
4 4
1 −x3 = 0 or x + 2 = 0
1. The leading term is − x3 . The degree, 3, is odd
4 x = 0 or x = −2
1
and the leading coefficient, − , is negative so as The zeros of the function are 0 and −2, so the x-
4
x → ∞, g(x) → −∞ and as x → −∞, g(x) → ∞. intercepts of the graph are (0, 0) and (−2, 0).
2. We solve g(x) = 0. 3. The zeros divide the x-axis into 3 intervals, (−∞, −2),
(−2, 0), and (0, ∞). We choose a value for x from each
1 3
− x3 − x2 = 0 interval and find g(x). This tells us the sign of g(x)
4 4 for all values of x in that interval.
1 2 In (−∞, −2), test −3:
− x (x + 3) = 0
4
g(−3) = −(−3)4 − 2(−3)3 = −27 < 0
1 In (−2, 0), test −1:
− x2 = 0 or x + 3 = 0
4
g(−1) = −(−1)4 − 2(−1)3 = 1 > 0
x2 = 0 or x = −3
In (0, ∞), test 1:
x = 0 or x = −3
g(1) = −(1)4 − 2(1)3 = −3 < 0
The zeros of the function are 0 and −3, so the x-
intercepts of the graph are (0, 0) and (−3, 0). Thus the graph lies below the x-axis on (−∞, −2)
and (0, ∞) and above the x-axis on (−2, 0). We also
3. The zeros divide the x-axis into 3 intervals, (−∞, −3),
know the points (−3, −27), (−1, 1), and (1, −3) are
(−3, 0), and (0, ∞). We choose a value for x from each
on the graph.
interval and find g(x). This tells us the sign of g(x)
for all values of x in that interval. 4. From Step 2 we see that the y-intercept is (0, 0).
In (−∞, −3), test −4: 5. We find additional points on the graph and then
1 3 draw the graph.
g(−4) = − (−4)3 − (−4)2 = 4 > 0
4 4
In (−3, 0), test −1: x g(x) y

1 3
g(−1) = − (−1)3 − (−1)2 = − < 0
1 −2.5 −7.8 4
4 4 2 2
In (0, ∞), test 1: −1.5 1.7
1 3 4 2 2 4 x
g(1) = − · 13 − · 12 = −1 < 0 0.5 −0.3
2
4 4
Thus the graph lies above the x-axis on (−∞, −3) 1.5 −11.8 4
and below the x-axis on (−3, 0)and on  (0, ∞).
1 2 −32
We also know the points (−4, 4), − 1, − , and g(x)  x 4  2x 3
2
(1, −1) are on the graph. 6. Checking the graph as described on page 317 in the
4. From Step 2 we see that the y-intercept is (0, 0). text, we see that it appears to be correct.
5. We find additional points on the graph and then 1
draw the graph. 25. f (x) = − (x − 2)(x + 1)2 (x − 1)
y 2
x g(x) 1 1
4 1. The leading term is − · x · x · x · x, or − x4 . The
1 3
g (x)  x 3 2
 x
2 2
−2 −1 2 4 4 1
degree, 4, is even and the leading coefficient, − , is
2
2 −5 4 2 2 4 x negative so as x → ∞, f (x) → −∞ and as x → −∞,
2 f (x) → −∞.
2.5 −8.6
4 2. We solve f (x) = 0.
1
− (x − 2)(x + 1)2 (x − 1) = 0
6. Checking the graph as described on page 317 in the 2
text, we see that it appears to be correct. x − 2 = 0 or (x + 1)2 = 0 or x − 1 = 0
23. g(x) = −x4 − 2x3 x = 2 or x+1 = 0 or x=1
x = 2 or x = −1 or x=1
1. The leading term is −x4 . The degree, 4, is even and
the leading coefficient, −1, is negative so as x → ∞, The zeros of the function are 2, −1, and 1, so the x-
g(x) → −∞ and as x → −∞, g(x) → −∞. intercepts of the graph are (2, 0), (−1, 0), and (1, 0).

Copyright © 2013 Pearson Education, Inc.


Exercise Set 4.2 135

3. The zeros divide the x-axis into 4 intervals, (−∞, −1), In (−∞, −4), test −5:
(−1, 1), (1, 2), and (2, ∞). We choose a value for x g(−5) = −(−5)(−5 − 1)2 (−5 + 4)2 = 180 > 0
from each interval and find f (x). This tells us the
In (−4, 0), test −1:
sign of f (x) for all values of x in that interval.
g(−1) = −(−1)(−1 − 1)2 (−1 + 4)2 = 36 > 0
In (−∞, −1), test −2:
1 In (0, 1), test 0.5:
f (−2) = − (−2 − 2)(−2 + 1)2 (−2 − 1) = −6 < 0
2 g(0.5) = −0.5(0.5 − 1)2 (−0.5 + 4)2 = −2.53125 < 0
In (−1, 1), test 0: In (1, ∞), test 2:
1 g(2) = −2(2 − 1)2 (2 + 4)2 = −72 < 0
f (0) = − (0 − 2)(0 + 1)2 (0 − 1) = −1 < 0
2
Thus the graph lies above the x-axis on (−∞, −4)
In (1, 2), test 1.5:
and on (−4, 0) and below the x-axis on (0, 1) and
1 (1, ∞). We also know the points (−5, 180), (−1, 36),
f (1.5) = − (1.5 − 2)(1.5 + 1)2 (1.5 − 1) = 0.78125 > 0
2 (0.5, −2.53125), and (2, −72) are on the graph.
In (2, ∞), test 3:
4. From Step 2 we see that the y-intercept is (0,0).
1
f (3) = − (3 − 2)(3 + 1)2 (3 − 1) = −16 < 0 5. We find additional points on the graph and then
2
draw the graph.
Thus the graph lies below the x-axis on (−∞, −1), y
(−1, 1), and (2, ∞) and above the x-axis on (1, 2). We x g(x)
also know the points (−2, −6), (0, −1), (1.5, 0.78125), 80
and (3, −16) are on the graph. −3 4.8 40

4. From Step 2 we know that f (0) = −1 so the y- −2 72 6 4 2 2 4 x


intercept is (0, −1). 40
5. We find additional points on the graph and then draw 1.5 −11.3
80
the graph.
y
x f (x) g(x)  x(x  1)2(x  4)2
4
−3 −40 6. Checking the graph as described on page 317 in the
2

−0.5 −0.5 text, we see that it appears to be correct.


4 2 2 4 x
0.5 −0.8 2 29. f (x) = (x − 2)2 (x + 1)4
4 1. The leading term is x · x · x · x · x · x, or x6 . The degree,
1.5 0.8 6, is even and the leading coefficient, 1, is positive so
1
f (x)  (x  2)(x  1)2(x  1) as x → ∞, f (x) → ∞ and as x → −∞, f (x) → ∞.
2
2. We see that the zeros of the function are 2 and −1 so
the x-intercepts of the graph are (2, 0) and (−1, 0).
6. Checking the graph as described on page 317 in the
3. The zeros divide the x-axis into 3 intervals, (−∞, −1),
text, we see that it appears to be correct.
(−1, 2), and (2, ∞). We choose a value for x from each
27. g(x) = −x(x − 1)2 (x + 4)2 interval and find f (x). This tells us the sign of f (x)
for all values of x in that interval.
1. The leading term is −x · x · x · x · x, or −x5 . The
In (−∞, −1), test −2:
degree, 5, is odd and the leading coefficient, −1, is
negative so as x → ∞, g(x) → −∞ and as x → −∞, f (−2) = (−2 − 2)2 (−2 + 1)4 = 16 > 0
g(x) → ∞. In (−1, 2), test 0:
2. We solve g(x) = 0. f (0) = (0 − 2)2 (0 + 1)4 = 4 > 0
−x(x − 1) (x + 4) = 0
2 2 In (2, ∞), test 3:
−x = 0 or (x − 1)2 = 0 or (x + 4)2 = 0 f (3) = (3 − 2)2 (3 + 1)4 = 256 > 0
x = 0 or x − 1 = 0 or x+4 = 0 Thus the graph lies above the x-axis on all 3 intervals.
We also know the points (−2, 16), (0, 4), and (3, 256)
x = 0 or x = 1 or x = −4
are on the graph.
The zeros of the function are 0, 1, and −4, so the
x-intercepts are (0, 0), (1, 0), and (−4, 0). 4. From Step 3 we know that f (0) = 4 so the y-intercept
is (0, 4).
3. The zeros divide the x-axis into 4 intervals, (−∞, −4),
(−4, 0), (0, 1), and (1, ∞). We choose a value for x 5. We find additional points on the graph and then draw
from each interval and find g(x). This tells us the the graph.
sign of g(x) for all values of x in that interval.

Copyright © 2013 Pearson Education, Inc.


136 Chapter 4: Polynomial and Rational Functions

y 2. We solve h(x) = 0.
x f (x)
30 x3 + 3x2 − x − 3 = 0
−1.5 0.8
20 x2 (x + 3) − (x + 3) = 0
−0.5 0.4 10 (x + 3)(x2 − 1) = 0
1 16 4 2 2 4 x (x + 3)(x + 1)(x − 1) = 0
10
1.5 9.8 x+3 = 0 or x + 1 = 0 or x − 1 = 0
x = −3 or x = −1 or x=1
f(x)  (x  2)2 (x  1)4
The zeros of the function are −3, −1, and 1 so the x-
intercepts of the graph are (−3, 0), (−1, 0), and (1, 0).
6. Checking the graph as described on page 317 in the
3. The zeros divide the x-axis into 4 intervals, (−∞, −3),
text, we see that it appears to be correct. (−3, −1), (−1, 1), and (1, ∞). We choose a value for
31. g(x) = −(x − 1)4 x from each interval and find h(x). This tells us the
sign of h(x) for all values of x in that interval.
1. The leading term is −1 · x · x · x · x, or −x4 . The
In (−∞, −3), test −4:
degree, 4, is even and the leading coefficient, −1, is
negative so as x → ∞, g(x) → −∞ and as x → −∞, h(−4) = (−4)3 + 3(−4)2 − (−4) − 3 = −15 < 0
g(x) → −∞. In (−3, −1), test −2:
2. We see that the zero of the function is 1, so the x- h(−2) = (−2)3 + 3(−2)2 − (−2) − 3 = 3 > 0
intercept is (1, 0).
In (−1, 1), test 0:
3. The zero divides the x-axis into 2 intervals, (−∞, 1)
and (1, ∞). We choose a value for x from each interval h(0) = 03 + 3 · 02 − 0 − 3 = −3 < 0
and find g(x). This tells us the sign of g(x) for all In (1, ∞), test 2:
values of x in that interval. h(2) = 23 + 3 · 22 − 2 − 3 = 15 > 0
In (−∞, 1), test 0:
Thus the graph lies below the x-axis on (−∞, −3) and
g(0) = −(0 − 1)4 = −1 < 0 on (−1, 1) and above the x-axis on (−3, −1) and on
In (1, ∞), test 2: (1, ∞). We also know the points (−4, −15), (−2, 3),
g(2) = −(2 − 1)4 = −1 < 0 (0, −3), and (2, 15) are on the graph.
Thus the graph lies below the x-axis on both intervals. 4. From Step 3 we know that h(0) = −3 so the y-
We also know the points (0, −1) and (2, −1) are on intercept is (0, −3).
the graph. 5. We find additional points on the graph and then draw
4. From Step 3 we know that g(0) = −1 so the y- the graph.
intercept is (0, −1). y
x h(x)
5. We find additional points on the graph and then draw 4
the graph. −4.5 −28.9
2
y
x g(x) −2.5 2.6
4 4 2 2 4 x
2
−1 −16 0.5 −2.6
4 2 2 4 x 4
4
−0.5 −5.1 2.5 28.9
8
1.5 0.1 h(x)  x 3  3x 2  x  3
12

3 −16 16
6. Checking the graph as described on page 317 in the
g(x)  (x  1)4
text, we see that it appears to be correct.

35. f (x) = 6x3 − 8x2 − 54x + 72


6. Checking the graph as described on page 317 in the 1. The leading term is 6x3 . The degree, 3, is odd and
text, we see that it appears to be correct. the leading coefficient, 6, is positive so as x → ∞,
33. h(x) = x3 + 3x2 − x − 3 f (x) → ∞ and as x → −∞, f (x) → −∞.
1. The leading term is x3 . The degree, 3, is odd and
the leading coefficient, 1, is positive so as x → ∞,
h(x) → ∞ and as x → −∞, h(x) → −∞.

Copyright © 2013 Pearson Education, Inc.


Exercise Set 4.2 137

2. We solve f (x) = 0. 37. We graph g(x) = −x+3 for x ≤ −2, g(x) = 4 for 2 < x < 1,
1
6x − 8x − 54x + 72 = 0
3 2
and g(x) = x3 for x ≥ 1.
2
2(3x3 − 4x2 − 27x + 36) = 0
y
2[x2 (3x − 4) − 9(3x − 4)] = 0
4
2(3x − 4)(x2 − 9) = 0
2
2(3x − 4)(x + 3)(x − 3) = 0
4 2 2 4
3x − 4 = 0 or x + 3 = 0 or x − 3 = 0 x
2
4
x= or x = −3 or x=3 4
3
4
The zeros of the function are , −3, and 3, so the x- x  3, for x  2,
 3 g (x )  4, for 2  x  1,
4 1 3,
intercepts of the graph are , 0 , (−3, 0), and (3, 0). x
2 for x  1
3
The zerosdivide
3.    x-axis into 4 intervals, (−∞, −3),
the
39. f (−5) = (−5)3 + 3(−5)2 − 9(−5) − 13 = −18
4 4
− 3, , , 3 , and (3, ∞). We choose a value for
3 3 f (−4) = (−4)3 + 3(−4)2 − 9(−4) − 13 = 7
x from each interval and find f (x). This tells us the By the intermediate value theorem, since f (−5) and f (−4)
sign of f (x) for all values of x in that interval. have opposite signs then f (x) has a zero between −5 and
In (−∞, −3), test −4: −4.
f (−4) = 6(−4)3 − 8(−4)2 − 54(−4) + 72 = −224 < 0 41. f (−3) = 3(−3)2 − 2(−3) − 11 = 22
 
4
In − 3, , test 0: f (−2) = 3(−2)2 − 2(−2) − 11 = 5
3
Since both f (−3) and f (−2) are positive, we cannot use
f (0) = 6 · 03 − 8 · 02 − 54 · 0 + 72 = 72 > 0 the intermediate value theorem to determine if there is a
 
4 zero between −3 and −2.
In , 3 , test 2:
3
43. f (2) = 24 − 2 · 22 − 6 = 2
f (2) = 6 · 23 − 8 · 22 − 54 · 2 + 72 = −20 < 0
f (3) = 34 − 2 · 32 − 6 = 57
In (3, ∞), test 4:
Since both f (2) and f (3) are positive, we cannot use the
f (4) = 6 · 43 − 8 · 42 − 54 · 4 + 72 = 112 > 0 intermediate value theorem to determine if there is a zero
Thus thegraph lies below the x-axis  on (−∞,−3) and between 2 and 3.
4 4
on , 3 and above the x-axis on − 3, and on 45. f (4) = 43 − 5 · 42 + 4 = −12
3 3
(3, ∞). We also know the points (−4, −224), (0, 72), f (5) = 53 − 5 · 52 + 4 = 4
(2, −20), and (4, 112) are on the graph. By the intermediate value theorem, since f (4) and f (5)
4. From Step 3 we know that f (0) = 72 so the y- have opposite signs then f (x) has a zero between 4 and 5.
intercept is (0, 72).
47. The graph of y = x, or y = x + 0, has y-intercept (0, 0), so
5. We find additional points on the graph and then draw
(d) is the correct answer.
the graph.
y 49. The graph of y − 2x = 6, or y = 2x + 6, has y-intercept
x f (x) (0, 6), so (e) is the correct answer.
100
−1 112 50 51. The graph of y = 1 − x, or y = −x + 1, has y-intercept
1 16 (0, 1), so (b) is the correct answer.
4 2 2 4 x
50 1
3.5 42.25 53. 2x − = 4 − 3x
100 2
1
5x − = 4 Adding 3x
f(x)  6x 3  8x 2  54x  72 2
9 1
5x = Adding
6. Checking the graph as described on page 317 in the 2 2
1 9 1
text, we see that it appears to be correct. x= · Multiplying by
5 2 5
9
x=
10
9
The solution is .
10

Copyright © 2013 Pearson Education, Inc.


138 Chapter 4: Polynomial and Rational Functions

55. 6x2 − 23x − 55 = 0 b) x2 + x − 8


(3x + 5)(2x − 11) = 0 x − 3 x3 − 2x2 − 11x + 12
x3 − 3x2
3x + 5 = 0 or 2x − 11 = 0 x2 − 11x
3x = −5 or 2x = 11 x2 − 3x
5 11 − 8x + 12
x = − or x= − 8x + 24
3 2
5 11 − 12
The solutions are − and .
3 2 Since the remainder is not 0, x − 3 is not a factor of
g(x).
c) x2 − x − 12
Exercise Set 4.3 x − 1 x3 − 2x2 − 11x + 12
x3 − x2
1. a) x3 − 7x2 + 8x + 16 − x2 − 11x
x + 1 x4 − 6x3 + x2 + 24x − 20 − x2 + x
x4 + x3 − 12x + 12
− 7x3 + x2 − 12x + 12
− 7x3 − 7x2 0
8x2 + 24x Since the remainder is 0, x − 1 is a factor of g(x).
8x2 + 8x
16x − 20 5. x2 − 2x + 4
16x + 16 x + 2 x3 + 0x2 + 0x − 8
− 4 x3 + 2x2
− 2x2 + 0x
Since the remainder is not 0, x + 1 is not a factor of
− 2x2 − 4x
f (x).
4x − 8
b) x3 − 4x2 − 7x + 10 4x + 8
x − 2 x4 − 6x3 + x2 + 24x − 20 −16
x4 − 2x3
x3 − 8 = (x + 2)(x2 − 2x + 4) − 16
− 4x3 + x2
− 4x3 + 8x2 7. x2 − 3x + 2
− 7x2 + 24x x + 9 x3 + 6x2 − 25x + 18
− 7x2 + 14x x3 + 9x2
10x − 20 − 3x2 − 25x
10x − 20 − 3x2 − 27x
0 2x + 18
Since the remainder is 0, x − 2 is a factor of f (x). 2x + 18
0
c) x3 − 11x2 + 56x − 256
x + 5 x4 − 6x3 + x2 + 24x − 20 x3 + 6x2 − 25x + 18 = (x + 9)(x2 − 3x + 2) + 0
x4 + 5x3 9. x3 − 2x2 + 2x − 4
− 11x3 + x2 x + 2 x4 + 0x3 − 2x2 + 0x + 3
− 11x3 − 55x2 x4 + 2x3
56x2 + 24x − 2x3 − 2x2
56x2 + 280x − 2x3 − 4x2
− 256x − 20 2x2 + 0x
− 256x − 1280 2x2 + 4x
1260 − 4x + 3
Since the remainder is not 0, x + 5 is not a factor of − 4x − 8
f (x). 11
x4 − 2x2 + 3 = (x + 2)(x3 − 2x2 + 2x − 4) + 11
3. a) x2 + 2x − 3
x − 4 x3 − 2x2 − 11x + 12 11. (2x4 + 7x3 + x − 12) ÷ (x + 3)
x3 − 4x2
= (2x4 + 7x3 + 0x2 + x − 12) ÷ [x − (−3)]
2x2 − 11x 
2x2 − 8x −3  2 7 0 1 −12
− 3x + 12 −6 −3 9 −30
− 3x + 12 2 1 −3 10 −42
0 The quotient is 2x3 + x2 − 3x + 10. The remainder is −42.
Since the remainder is 0, x − 4 is a factor of g(x).

Copyright © 2013 Pearson Education, Inc.


Exercise Set 4.3 139

13. (x3 − 2x2 − 8) ÷ (x + 2) 25. f (x) = x4 − 3x3 + 2x + 8


= (x3 − 2x2 + 0x − 8) ÷ [x − (−2)] Find f (−1).
 
−2  1 −2 0 −8 −1  1 −3 0 2 8
−2 8 −16 −1 4 −4 2
1 −4 8 −24 1 −4 4 −2 10
The quotient is x2 − 4x + 8. The remainder is −24. f (−1) = 10
15. (3x − x + 4x − 10) ÷ (x + 1)
3 2 Find f (4).

= (3x − x + 4x − 10) ÷ [x − (−1)]
3 2 4  1 −3 0 2 8

−1  3 −1 4 −10
4 4 16 72
1 1 4 18 80
−3 4 −8
3 −4 8 −18 f (4) = 80
The quotient is 3x − 4x + 8. The remainder is −18.
2 Find f (−5).

17. (x + x − x) ÷ (x − 3)
5 3 −5  1 −3 0 2 8
−5 40 −200 990
= (x + 0x + x + 0x − x + 0) ÷ (x − 3)
5 4 3 2
 1 −8 40 −198 998
3 1 0 1 0 −1 0
f (−5) = 998
3 9 30 90 267
1 3 10 30 89 267 27. f (x) = 2x5 − 3x4 + 2x3 − x + 8
The quotient is x4 + 3x3 + 10x2 + 30x + 89. Find f (20).

The remainder is 267. 20  2 −3 2 0 −1 8
19. (x4 − 1) ÷ (x − 1) 40 740 14, 840 296, 800 5, 935, 980
2 37 742 14, 840 296, 799 5, 935, 988
= (x4 + 0x3 + 0x2 + 0x − 1) ÷ (x − 1)
 f (20) = 5, 935, 988
1  1 0 0 0 −1
1 1 1 1 Find f (−3).
1 1 1 1 0 
−3  2 −3 2 0 −1 8
The quotient is x3 + x2 + x + 1. The remainder is 0. −6 27 −87 261 −780
 
1 2 −9 29 −87 260 −772
21. (2x4 + 3x2 − 1) ÷ x −
2 f (−3) = −772
 
1
(2x + 0x + 3x + 0x − 1) ÷ x −
4 3 2
29. f (x) = x4 − 16
2

1 
2 0 3 0 −1 Find f (2).
2 
1 12 47 2  1 0 0 0 −16
7
8
2 1 7 7
− 18 2 4 8 16
2 4
1 2 4 8 0
7 7 1
The quotient is 2x3 + x2 + x + . The remainder is − . f (2) = 0
2 4 8
23. f (x) = x3 − 6x2 + 11x − 6 Find f (−2).

Find f (1). −2  1 0 0 0 −16
 −2 4 −8
1  1 −6 11 −6 16
1 −5 6 1 −2 4 −8 0
1 −5 6 0 f (−2) = 0
f (1) = 0
Find f (3).

Find f (−2).
 3 1 0 0 0 −16
−2  1 −6 11 −6 3 9 27 81
−2 16 −54 1 3 9 27 65
1 −8 27 −60 f (3) = 65
f (−2) = −60 √
Find f (1 − 2).
Find f (3). √ 
 1− 2  1 0√ 0√ 0√ −16√
3  1 −6 11 −6
1− √2 3−2√2 7−5√2 17−12√ 2
3 −9 6
1 1− 2 3−2 2 7−5 2 1−12 2
1 −3 2 0 √ √
f (3) = 0 f (1 − 2) = 1 − 12 2

Copyright © 2013 Pearson Education, Inc.


140 Chapter 4: Polynomial and Rational Functions

 
31. f (x) = 3x3 + 5x2 − 6x + 18 Find f
1
using synthetic division.
If −3 is a zero of f (x), then f (−3) = 0. Find f (−3) using 2

1 
2  1 −2 1 − 32
synthetic division. 7

−3  3 5 −6 18 1
− 2 − 14
3
2
−9 12 −18
1 −3 − 12 − 74
3 −4 6 0
 
Since f (−3) = 0, −3 is a zero of f (x). 1 1
Since f = 0, is not a zero of f (x).
If 2 is a zero of f (x), then f (2) = 0. Find f (2) using 2 2
synthetic division.
 39. f (x) = x3 + 4x2 + x − 6
2 3 5 −6 18
Try x − 1. Use synthetic division to see whether f (1) = 0.
6 22 32 
3 11 16 50 1  1 4 1 −6
1 5 6
Since f (2) = 0, 2 is not a zero of f (x).
1 5 6 0
33. h(x) = x4 + 4x3 + 2x2 − 4x − 3 Since f (1) = 0, x − 1 is a factor of f (x). Thus f (x) =
If −3 is a zero of h(x), then h(−3) = 0. Find h(−3) using (x − 1)(x2 + 5x + 6).
synthetic division. Factoring the trinomial we get

−3  1 4 2 −4 −3 f (x) = (x − 1)(x + 2)(x + 3).
−3 −3 3 3
To solve the equation f (x) = 0, use the principle of zero
1 1 −1 −1 0
products.
Since h(−3) = 0, −3 is a zero of h(x).
(x − 1)(x + 2)(x + 3) = 0
If 1 is a zero of h(x), then h(1) = 0. Find h(1) using
x − 1 = 0 or x + 2 = 0 or x + 3 = 0
synthetic division.
 x = 1 or x = −2 or x = −3
1  1 4 2 −4 −3
1 5 7 3 The solutions are 1, −2, and −3.
1 5 7 3 0
41. f (x) = x3 − 6x2 + 3x + 10
Since h(1) = 0, 1 is a zero of h(x).
Try x − 1. Use synthetic division to see whether f (1) = 0.

35. g(x) = x3 − 4x2 + 4x − 16 1  1 −6 3 10
If i is a zero of g(x), then g(i) = 0. Find g(i) using syn- 1 −5 −2
thetic division. Keep in mind that i2 = −1. 1 −5 −2 8

i 1 −4 4 −16 Since f (1) = 0, x − 1 is not a factor of P (x).
i −4i − 1 3i + 4 Try x+1. Use synthetic division to see whether f (−1) = 0.
1 −4 + i 3 − 4i −12 + 3i 
−1  1 −6 3 10
Since g(i) = 0, i is not a zero of g(x). −1 7 −10
If −2i is a zero of g(x), then g(−2i) = 0. Find g(−2i) 1 −7 10 0
using synthetic division. Keep in mind that i2 = −1. Since f (−1) = 0, x + 1 is a factor of f (x).

−2i  1 −4 4 −16 Thus f (x) = (x + 1)(x2 − 7x + 10).
−2i 8i − 4 16
Factoring the trinomial we get
1 −4 − 2i 8i 0
f (x) = (x + 1)(x − 2)(x − 5).
Since g(−2i) = 0, −2i is a zero of g(x).
To solve the equation f (x) = 0, use the principle of zero
7 3 products.
37. f (x) = x − x2 + x −
3
2 2 (x + 1)(x − 2)(x − 5) = 0
If −3 is a zero of f (x), then f (−3) = 0. Find f (−3) using x+1 = 0 or x − 2 = 0 or x − 5 = 0
synthetic division.
 x = −1 or x = 2 or x=5

−3  1 − 72 1 − 32
The solutions are −1, 2, and 5.
−3 2 − 123
39
2
43. f (x) = x3 − x2 − 14x + 24
1 − 13
2
41
2 −63
Try x + 1, x − 1, and x + 2. Using synthetic division we
Since f (−3) = 0, −3 is not a zero of f (x). find that f (−1) = 0, f (1) = 0 and f (−2) = 0. Thus
  x + 1, x − 1, and x + 2, are not factors of f (x).
1 1
If is a zero of f (x), then f = 0.
2 2 Try x − 2. Use synthetic division to see whether f (2) = 0.

Copyright © 2013 Pearson Education, Inc.


Exercise Set 4.3 141

 
2 1 −1 −14 24 2 1 0 −19 30
2 2 −24 2 4 −30
1 1 −12 0 1 2 −15 0
Since f (2) = 0, x − 2 is a factor of f (x). Thus f (x) = Since g(2) = 0, x − 2 is a factor of x3 − 19x + 30.
(x − 2)(x2 + x − 12). Thus f (x) = (x − 1)(x − 2)(x2 + 2x − 15).
Factoring the trinomial we get Factoring the trinomial we get
f (x) = (x − 2)(x + 4)(x − 3) f (x) = (x − 1)(x − 2)(x − 3)(x + 5).
To solve the equation f (x) = 0, use the principle of zero To solve the equation f (x) = 0, use the principle of zero
products. products.
(x − 2)(x + 4)(x − 3) = 0 (x − 1)(x − 2)(x − 3)(x + 5) = 0
x − 2 = 0 or x + 4 = 0 or x − 3 = 0 x−1 = 0 or x−2 = 0 or x−3 = 0 or x+5 = 0
x = 2 or x = −4 or x=3 x = 1 or x = 2 or x = 3 or x = −5
The solutions are 2, −4, and 3. The solutions are 1, 2, 3, and −5.
45. f (x) = x4 − 7x3 + 9x2 + 27x − 54 49. f (x) = x4 − x3 − 7x2 + x + 6
Try x + 1 and x − 1. Using synthetic division we find that 1. The leading term is x4 . The degree, 4, is even and
f (−1) = 0 and f (1) = 0. Thus x + 1 and x − 1 are not the leading coefficient, 1, is positive so as x → ∞,
factors of f (x). Try x + 2. Use synthetic division to see f (x) → ∞ and as x → −∞, f (x) → ∞.
whether f (−2) = 0.
 2. Find the zeros of the function. We first use synthetic
−2  1 −7 9 27 −54 division to determine if f (1) = 0.
−2 18 −54 54 
1 −9 27 −27 0 1  1 −1 −7 1 6
1 0 −7 −6
Since f (−2) = 0, x + 2 is a factor of f (x). Thus f (x) = 1 0 −7 −6 0
(x + 2)(x3 − 9x2 + 27x − 27).
1 is a zero of the function and we have
We continue to use synthetic division to factor g(x) = f (x) = (x − 1)(x3 − 7x − 6).
x3 − 9x2 + 27x − 27. Trying x + 2 again and x − 2 we find
Synthetic division shows that −1 is a zero of g(x) =
that g(−2) = 0 and g(2) = 0. Thus x + 2 and x − 2 are
x3 − 7x − 6.
not factors of g(x). Try x − 3. 
 −1  1 0 −7 −6
3  1 −9 27 −27
3 −18 27 −1 1 6
1 −6 9 0 1 −1 −6 0

Since g(3) = 0, x − 3 is a factor of x3 − 9x2 + 27x − 27. Then we have f (x) = (x − 1)(x + 1)(x2 − x − 6).
To find the other zeros we solve the following equa-
Thus f (x) = (x + 2)(x − 3)(x2 − 6x + 9).
tion:
Factoring the trinomial we get x2 − x − 6 = 0
f (x) = (x + 2)(x − 3)(x − 3)2 , or f (x) = (x + 2)(x − 3)3 . (x − 3)(x + 2) = 0
To solve the equation f (x) = 0, use the principle of zero x − 3 = 0 or x + 2 = 0
products.
x = 3 or x = −2
(x + 2)(x − 3)(x − 3)(x − 3) = 0
The zeros of the function are 1, −1, 3, and −2 so the
x+2 = 0 or x − 3 = 0 or x − 3 = 0 or x − 3 = 0 x-intercepts of the graph are (1, 0), (−1, 0), (3, 0), and
x = −2 or x = 3 or x = 3 or x=3 (−2, 0).
The solutions are −2 and 3. 3. The zeros divide the x-axis into five intervals,
(−∞, −2), (−2, −1), (−1, 1), (1, 3), and (3, ∞). We
47. f (x) = x4 − x3 − 19x2 + 49x − 30 choose a value for x from each interval and find f (x).
Try x − 1. Use synthetic division to see whether f (1) = 0. This tells us the sign of f (x) for all values of x in the

1  1 −1 −19 49 −30 interval.
1 0 −19 30 In (−∞, −2), test −3:
1 0 −19 30 0 f (−3) = (−3)4 − (−3)3 − 7(−3)2 + (−3) + 6 = 48 > 0
Since f (1) = 0, x − 1 is a factor of f (x). Thus f (x) = In (−2, −1), test −1.5:
(x − 1)(x3 − 19x + 30). f (−1.5) = (−1.5)4 −(−1.5)3 −7(−1.5)2 +(−1.5)+6 =
We continue to use synthetic division to factor g(x) = −2.8125 < 0
x3 − 19x + 30. Trying x − 1, x + 1, and x + 2 we find that
g(1) = 0, g(−1) = 0, and g(−2) = 0. Thus x − 1, x + 1, In (−1, 1), test 0:
and x + 2 are not factors of x3 − 19x + 30. Try x − 2. f (0) = 04 − 03 − 7 · 02 + 0 + 6 = 6 > 0

Copyright © 2013 Pearson Education, Inc.


142 Chapter 4: Polynomial and Rational Functions

In (1, 3), test 2: In (2, ∞), test 3:


f (2) = 24 − 23 − 7 · 22 + 2 + 6 = −12 < 0 f (3) = 33 − 7 · 3 + 6 = 12 > 0
In (3, ∞), test 4: Thus the graph lies below the x-axis on (−∞, −3)
and on (1, 2). It lies above the x-axis on (−3, 1) and
f (4) = 44 − 43 − 7 · 42 + 4 + 6 = 90 > 0
on (2, ∞). We also know the points (−4, −30), (0, 6),
Thus the graph lies above the x-axis on (−∞, −2), (1.5, −1.125), and (3, 12) are on the graph.
on (−1, 1), and on (3, ∞). It lies below the x-axis
4. From Step 3 we see that f (0) = 6 so the y-intercept
on (−2, −1) and on (1, 3). We also know the points is (0, 6).
(−3, 48), (−1.5, −2.8125), (0, 6), (2, −12), and (4, 90)
are on the graph. 5. We find additional points on the graph and draw the
graph.
4. From Step 3 we see that f (0) = 6 so the y-intercept y
is (0, 6). x f (x)
5. We find additional points on the graph and draw the 20
−3.5 −12.4
graph. 10
y −2 12
x f (x) 4 2 2 4 x
8
2.5 4.1 10
−2.5 14.3 4
20
4 42
−0.5 3.9 4 2 2 4 x
4
f(x)  x 3  7x  6
0.5 4.7 8

2.5 −11.8 12 6. Checking the graph as described on page 317 in the
text, we see that it appears to be correct.
f (x )  x 4  x 3  7x 2  x  6
53. f (x) = −x3 + 3x2 + 6x − 8
6. Checking the graph as described on page 317 in the 1. The leading term is −x3 . The degree, 3, is odd and
text, we see that it appears to be correct. the leading coefficient, −1, is negative so as x → ∞,
f (x) → −∞ and as x → −∞, f (x) → ∞.
51. f (x) = x3 − 7x + 6
2. Find the zeros of the function. We first use synthetic
1. The leading term is x3 . The degree, 3, is odd and division to determine if f (1) = 0.
the leading coefficient, 1, is positive so as x → ∞, 
1  −1 3 6 −8
f (x) → ∞ and as x → −∞, f (x) → −∞.
−1 2 8
2. Find the zeros of the function. We first use synthetic −1 2 8 0
division to determine if f (1) = 0.
 1 is a zero of the function and we have
1  1 0 −7 6 f (x) = (x − 1)(−x2 + 2x + 8). To find the other zeros
1 1 −6 we solve the following equation.
1 1 −6 0
−x2 + 2x + 8 = 0
1 is a zero of the function and we have
x2 − 2x − 8 = 0
f (x) = (x − 1)(x2 + x − 6). To find the other zeros
we solve the following equation. (x − 4)(x + 2) = 0
x2 + x − 6 = 0 x − 4 = 0 or x + 2 = 0
(x + 3)(x − 2) = 0 x = 4 or x = −2
x+3 = 0 or x − 2 = 0 The zeros of the function are 1, 4, and −2 so the x-
intercepts of the graph are (1, 0), (4, 0), and (−2, 0).
x = −3 or x=2
3. The zeros divide the x-axis into four intervals,
The zeros of the function are 1, −3, and 2 so the x- (−∞, −2), (−2, 1), (1, 4), and (4, ∞). We choose a
intercepts of the graph are (1, 0), (−3, 0), and (2, 0). value for x from each interval and find f (x). This tells
3. The zeros divide the x-axis into four intervals, us the sign of f (x) for all values of x in the interval.
(−∞, −3), (−3, 1), (1, 2), and (2, ∞). We choose a
In (−∞, −2), test −3:
value for x from each interval and find f (x). This tells
us the sign of f (x) for all values of x in the interval. f (−3) = −(−3)3 + 3(−3)2 + 6(−3) − 8 = 28 > 0
In (−∞, −3), test −4: In (−2, 1), test 0:
f (−4) = (−4)3 − 7(−4) + 6 = −30 < 0 f (0) = −03 + 3 · 02 + 6 · 0 − 8 = −8 < 0
In (−3, 1), test 0: In (1, 4), test 2:
f (0) = 0 − 7 · 0 + 6 = 6 > 0
3 f (2) = −23 + 3 · 22 + 6 · 2 − 8 = 8 > 0
In (1, 2), test 1.5: In (4, ∞), test 5:
f (1.5) = (1.5)3 − 7(1.5) + 6 = −1.125 < 0 f (5) = −53 + 3 · 52 + 6 · 5 − 8 = −28 < 0

Copyright © 2013 Pearson Education, Inc.


Exercise Set 4.3 143

Thus the graph lies above the x-axis on (−∞, −2) 1 1 1


A= bh = (30 − h)h = − h2 + 15h
and on (1, 4). It lies below the x-axis on (−2, 1) and 2 2 2
on (4, ∞). We also know the points (−3, 28), (0, −8), Find the value of h for which A is a maximum:
(2, 8), and (5, −28) are on the graph. −15
h= = 15
4. From Step 3 we see that f (0) = −8 so the y-intercept 2(−1/2)
is (0, −8). When h = 15, b = 30 − 15 = 15.
5. We find additional points on the graph and draw the The area is a maximum when the base and the height are
graph. each 15 in.
y
x f (x) 63. a) −4, −3, 2, and 5 are zeros of the function, so x + 4,
8 x + 3, x − 2, and x − 5 are factors.
−2.5 11.4
4 b) We first write the product of the factors:
−1 −10 4 2 2 4 P (x) = (x + 4)(x + 3)(x − 2)(x − 5)
x
4 Note that P (0) = 4 · 3(−2)(−5) > 0 and the graph
3 10
8 shows a positive y-intercept, so this function is a
4.5 −11.4 correct one.
f (x )  x 3  3x 2  6x  8 c) Yes; two examples are f (x) = c · P (x) for any non-
zero constant c and g(x) = (x − a)P (x).
6. Checking the graph as described on page 317 in the d) No; only the function in part (b) has the given
text, we see that it appears to be correct. graph.

55. 2x2 + 12 = 5x 65. Divide x3 − kx2 + 3x + 7k by x + 2.



2x2 − 5x + 12 = 0 −2  1 −k 3 7k
−2 2k + 4 −4k − 14
a = 2, b = −5, c = 12
√ 1 −k − 2 2k + 7 3k − 14
−b ± b2 − 4ac
x= Thus P (−2) = 3k − 14.
2a
 We know that if x + 2 is a factor of f (x), then f (−2) = 0.
−(−5) ± (−5)2 − 4 · 2 · 12
x=
2·2 We solve 0 = 3k − 14 for k.
√ 0 = 3k − 14
5 ± −71
= 14
4 =k
√ √ 3
5 ± i 71 5 71
= = ± i 2x2 4 12x − 4
4 4 4 67.
√ √ √ x2
+
−1 x+3
= 3
x + 3x2 − x − 3
,
5 71 5 71 5 71
The solutions are + i and − i, or ± i. LCM is (x + 1)(x − 1)(x + 3)
4 4 4 4 4 4  
57. We substitute −14 for g(x) and solve for x. 2x2 4
(x + 1)(x − 1)(x + 3) + =
(x + 1)(x − 1) x + 3
−14 = x2 + 5x − 14
12x − 4
0 = x2 + 5x (x + 1)(x − 1)(x + 3) ·
(x + 1)(x − 1)(x + 3)
0 = x(x + 5)
2x2 (x + 3) + 4(x + 1)(x − 1) = 12x − 4
x = 0 or x + 5 = 0 2x3 + 6x2 + 4x2 − 4 = 12x − 4
x = 0 or x = −5 2x3 + 10x2 − 12x = 0
When the output is −14, the input is 0 or −5. x3 + 5x2 − 6x = 0
59. We substitute −20 for g(x) and solve for x. x(x2 + 5x − 6) = 0
−20 = x2 + 5x − 14 x(x + 6)(x − 1) = 0
0 = x2 + 5x + 6 x = 0 or x + 6 = 0 x−1 = 0
0 = (x + 3)(x + 2) x = 0 or x = −6 x =1
x+3 = 0 or x + 2 = 0 Only 0 and −6 check. They are the solutions.
x = −3 or x = −2
69. Answers may vary. One possibility is P (x) = x15 − x14 .
When the output is −20, the input is −3 or −2. 
71. −i  1 3i −4i −2
61. Let b and h represent the length of the base and the height −i 2 −4 − 2i
of the triangle, respectively. 1 2i 2 − 4i −6 − 2i
b + h = 30, so b = 30 − h. Q(x) = x2 + 2ix + (2 − 4i), R(x) = −6 − 2i

Copyright © 2013 Pearson Education, Inc.


144 Chapter 4: Polynomial and Rational Functions

 

73. i 1 −3 7 (i2 = −1) 19. 1
2  20 −40 0
i −3i − 1 10 −15
1 −3 + i 6 − 3i
20 −30 −15
The answer is x − 3 + i, R 6 − 3i.
 
1
f = −15
Chapter 4 Mid-Chapter Mixed Review 2
21. f (x) = x3 − 4x2 + 9x − 36
If −3i is a zero of f (x), then f (−3i) = 0. We find f (−3i).
1. P (0) = 5 − 2 · 03 = 5, so the y-intercept is (0, 5). The given 
statement is false. −3i  1 −4 9 −36
−3i −9 + 12i 36
3. f (8) = (8 + 7)(8 − 8) = 15 · 0 = 0 1 −4 − 3i 12i 0
The given statement is true. Since f (−3i) = 0, −3i is a zero of f (x).
5. f (x) = (x2 − 10x + 25)3 = [(x − 5)2 ]3 = (x − 5)6 If 3 is a zero of f (x), then f (3) = 0. We find f (3).

Solving (x − 5)6 = 0, we get x = 5. 3  1 −4 9 −36
The factor x − 5 occurs 6 times, so the zero has a multi- 3 −3 18
plicity of 6. 1 −1 6 −18
Since f (3) = 0, 3 is not a zero of f (x).
7. g(x) = x4 −3x2 +2 = (x2 −1)(x2 −2) = (x+1)(x−1)(x2 −2)
23. h(x) = x3 − 2x2 − 55x + 56
√(x+1)(x−1)(x −2) = 0, we get x = −1 or x = 1 or
2
Solving
x = ± 2. Try x − 1.

Each factor occurs 1 time, so the multiplicity of each zero 1  1 −2 −55 56
is 1. 1 −1 −56
1 −1 −56 0
9. f (x) = x4 − x3 − 6x2
Since h(1) = 0, x − 1 is a factor of h(x). Then
The sign of the leading coefficient, 1, is positive and the h(x) = (x − 1)(x2 − x − 56). Factoring the trinomial, we
degree, 4, is even. Thus, graph (d) is the graph of the get h(x) = (x − 1)(x − 8)(x + 7).
function.
Now we solve h(x) = 0.
11. f (x) = 6x3 + 8x2 − 6x − 8 (x − 1)(x − 8)(x + 7) = 0
The sign of the leading coefficient, 6, is positive and the x − 1 = 0 or x − 8 = 0 or x + 7 = 0
degree, 3, is odd. Thus, graph (b) is the graph of the
function. x = 1 or x = 8 or x = −7
The solutions are 1, 8, and −7.
13. f (−2) = (−2)3 − 2(−2)2 + 3 = −13
f (0) = 03 − 2 · 02 + 3 = 3 25. The range of a polynomial function with an odd degree
is (−∞, ∞). The range of a polynomial function with an
By the intermediate value theorem, since f (−2) and f (0) even degree is [s, ∞) for some real number s if an > 0 and
have opposite signs, f (x) has a zero between −2 and 0. is (−∞, s] for some real number s if an < 0.

15. x3 − 5x2 − 5x − 4 27. If function values change from positive to negative or from
x − 1 x4 − 6x3 + 0x2 + x − 2 negative to positive in an interval, there would have to be
x4 − x3 a zero in the interval. Thus, between a pair of consecutive
− 5x3 + 0x2 zeros, all the function values must have the same sign.
− 5x3 + 5x2
− 5x2 + x
Exercise Set 4.4
− 5x2 + 5x
− 4x − 2
− 4x + 4 1. Find a polynomial function of degree 3 with −2, 3, and 5
−6 as zeros.
6 Such a function has factors x + 2, x − 3, and x − 5, so we
P (x) = (x − 1)(x3 − 5x2 − 5x − 4) − have f (x) = an (x + 2)(x − 3)(x − 5).
x−1
The number an can be any nonzero number. The simplest
17. (x5 − 5) ÷ (x + 1) = (x5 − 5) ÷ [x − (−1)]
 polynomial will be obtained if we let it be 1. Multiplying
−1  1 0 0 0 0 −5 the factors, we obtain
−1 1 −1 1 −1
f (x) = (x + 2)(x − 3)(x − 5)
1 −1 1 −1 1 −6
= (x2 − x − 6)(x − 5)
Q(x) = x4 − x3 + x2 − x + 1, R(x) = −6
= x3 − 6x2 − x + 30.

Copyright © 2013 Pearson Education, Inc.


Exercise Set 4.4 145

3. Find a polynomial function of degree 3 with −3, 2i, and 1


11. Find a polynomial function of degree 3 with − , 0, and 2
−2i as zeros. as zeros.
3
Such a function has factors x + 3, x − 2i, and x + 2i, so we 1
Such a function has factors x + , x − 0 (or x), and x − 2
have f (x) = an (x + 3)(x − 2i)(x + 2i). 3
so we have 
The number an can be any nonzero number. The simplest 1
polynomial will be obtained if we let it be 1. Multiplying f (x) = an x + (x)(x − 2).
3
the factors, we obtain
The number an can be any nonzero number. The simplest
f (x) = (x + 3)(x − 2i)(x + 2i)
polynomial will be obtained if we let it be 1. Multiplying
= (x + 3)(x2 + 4) the factors, we obtain
 
= x3 + 3x2 + 4x + 12. 1
f (x) = x + (x)(x − 2)
√ √ 3
5. Find a polynomial function of degree 3 with 2, − 2, and  
1
3 as zeros. = x2 + x (x − 2)
√ √ 3
√ x − √2, x + 2, and x − 3, so
Such a function has factors 5 2 2
we have f (x) = an (x − 2)(x + 2)(x − 3). = x − x − x.
3
3 3
The number an can be any nonzero number. The simplest
polynomial will be obtained if we let it be 1. Multiplying 13. A polynomial function of degree 5 has at most 5 real zeros.
the factors, we obtain Since 5 zeros are given, these are all of the zeros of the
√ √ desired function. We proceed as in Exercises 1-11, letting
f (x) = (x − 2)(x + 2)(x − 3)
an = 1.
= (x2 − 2)(x − 3) f (x) = (x + 1)3 (x − 0)(x − 1)
= x3 − 3x2 − 2x + 6. = (x3 + 3x2 + 3x + 1)(x2 − x)
√ √
7. Find a polynomial function of degree 3 with 1− 3, 1+ 3, = x5 + 2x4 − 2x2 − x
and −2 as zeros.
√ √ 15. A polynomial function of degree 4 has at most 4 real zeros.
Such a function has factors x − (1 − 3), x − (1 + 3), and Since 4 zeros are given, these are all of the zeros of the
x + 2, so we have desired function. We proceed as in Exercises 1-11, letting
√ √
f (x) = an [x − (1 − 3)][x − (1 + 3)](x + 2). an = 1.
The number an can be any nonzero number. The simplest f (x) = (x + 1)3 (x − 0)
polynomial will be obtained if we let it be 1. Multiplying = (x3 + 3x2 + 3x + 1)(x)
the factors, we obtain = x4 + 3x3 + 3x2 + x
√ √
f (x) = [x − (1 − 3)][x − (1 + 3)](x + 2)
√ √ 17. A polynomial function of degree 4 can have at most 4 zeros.
= [(x − 1) + 3][(x − 1) − 3](x + 2)
√ Since f (x) has rational coefficients, in addition√to the three

= [(x − 1)2 − ( 3)2 ](x + 2) zeros given, the other zero is the conjugate of 3, or − 3.
= (x2 − 2x + 1 − 3)(x + 2)
19. A polynomial function of degree 4 can have at most 4 zeros.
= (x2 − 2x − 2)(x + 2) Since f (x) has rational coefficients, the other zeros√are the
= x3 − 2x2 − 2x + 2x2 − 4x − 4 conjugates of the given zeros. They are i and 2 + 5.
= x3 − 6x − 4. 21. A polynomial function of degree 4 can have at most 4 zeros.
9. Find a polynomial function of degree 3 with 1 + 6i, 1 − 6i, Since f (x) has rational coefficients, in addition to the three
and −4 as zeros. zeros given, the other zero is the conjugate of 3i, or −3i.
Such a function has factors x − (1 + 6i), x − (1 − 6i), and 23. A polynomial function of degree 4 can have at most 4 zeros.
x + 4, so we have Since f (x) has rational coefficients, the other zeros are √
the
f (x) = an [x − (1 + 6i)][x − (1 − 6i)](x + 4). conjugates of the given zeros. They are −4+3i and 2+ 3.

The number an can be any nonzero number. The simplest 25. A polynomial function f (x) of degree 5 has at most 5 zeros.
polynomial will be obtained if we let it be 1. Multiplying Since f (x) has rational coefficients, in addition to √ the 3
the factors, we obtain given zeros,
√ the other zeros are the conjugates of 5 and
f (x) = [x − (1 + 6i)][x − (1 − 6i)](x + 4) −4i, or − 5 and 4i.
= [(x − 1) − 6i][(x − 1) + 6i](x + 4) 27. A polynomial function f (x) of degree 5 has at most 5 zeros.
= [(x − 1) − (6i) ](x + 4)
2 2 Since f (x) has rational coefficients, the other zero is the
conjugate of 2 − i, or 2 + i.
= (x2 − 2x + 1 + 36)(x + 4)
29. A polynomial function f (x) of degree 5 has at most 5 zeros.
= (x2 − 2x + 37)(x + 4)
Since f (x) has rational coefficients, in addition to the 3
= x3 − 2x2 + 37x + 4x2 − 8x + 148 given zeros, √ conjugates of −3 + 4i
√ the other zeros are the
= x3 + 2x2 + 29x + 148. and 4 − 5, or −3 − 4i and 4 + 5.

Copyright © 2013 Pearson Education, Inc.


146 Chapter 4: Polynomial and Rational Functions

31. A polynomial function f (x) of degree 5 has at most 5 zeros. If we let an = 1, we obtain
Since f (x) has rational coefficients, the other zero is the f (x) = (x2 − 5)(x2 + 9)
conjugate of 4 − i, or 4 + i.
= x4 + 4x2 − 45
33. Find a polynomial function of lowest degree with rational
coefficients that has 1 + i and 2 as some of its zeros. 1 − i 43. f (x) = x3 + 5x2 − 2x − 10
is also a zero. Since −5 is a zero of f (x), we have f (x) = (x + 5) · Q(x).
We use synthetic division to find Q(x).
Thus the polynomial function is 
f (x) = an (x − 2)[x − (1 + i)][x − (1 − i)]. −5  1 5 −2 −10
−5 0 10
If we let an = 1, we obtain 1 0 −2 0
f (x) = (x − 2)[(x − 1) − i][(x − 1) + i] Then f (x) = (x + 5)(x2 − 2). To find the other zeros we
= (x − 2)[(x − 1)2 − i2 ] solve x2 − 2 = 0.
= (x − 2)(x2 − 2x + 1 + 1) x2 − 2 = 0
= (x − 2)(x2 − 2x + 2) x2 = 2

= x3 − 4x2 + 6x − 4. x=± 2
√ √
35. Find a polynomial function of lowest degree with rational The other zeros are − 2 and 2.
coefficients that has 4i as one of its zeros. −4i is also a
45. If −i is a zero of f (x) = x4 − 5x3 + 7x2 − 5x + 6, i is also
zero.
a zero. Thus x + i and x − i are factors of the polynomial.
Thus the polynomial function is Since (x + i)(x − i) = x2 + 1, we know that f (x) =
f (x) = an (x − 4i)(x + 4i). (x2 + 1) · Q(x). Divide x4 − 5x3 + 7x2 − 5x + 6 by x2 + 1.
If we let an = 1, we obtain x2 − 5x + 6
x2 + 1 x4 − 5x3 + 7x2 − 5x + 6
f (x) = (x − 4i)(x + 4i) = x + 16.
2
x4 + x2
37. Find a polynomial function of lowest degree with rational −5x + 6x2 − 5x
3

coefficients that has −4i and 5 as some of its zeros. −5x3 − 5x


6x2 +6
4i is also a zero.
6x2 +6
Thus the polynomial function is 0
f (x) = an (x − 5)(x + 4i)(x − 4i). Thus
If we let an = 1, we obtain x4 −5x3 +7x2 −5x+6 = (x+i)(x−i)(x2 −5x+6)
f (x) = (x − 5)[x2 − (4i)2 ] = (x+i)(x−i)(x−2)(x−3)
= (x − 5)(x + 16) 2 Using the principle of zero products we find the other zeros
= x − 5x + 16x − 80
3 2 to be i, 2, and 3.

39. Find a polynomial function of lowest 47. x3 − 6x2 + 13x − 20 = 0


√ degree with rational
coefficients√that has 1 − i and − 5 as some of its zeros. If 4 is a zero, then x − 4 is a factor. Use synthetic division
1 + i and 5 are also zeros. to find another factor.
Thus the polynomial function is 4  1 −6 13 −20
√ √ 4 −8 20
f (x) = an [x − (1 − i)][x − (1 + i)](x + 5)(x − 5).
1 −2 5 0
If we let an = 1, we obtain
√ √ (x − 4)(x2 − 2x + 5) = 0
f (x) = [x − (1 − i)][x − (1 + i)](x + 5)(x −
5)
√ √ x−4 = 0 or x2 −2x+5 = 0 Principle of
= [(x − 1) + i][(x − 1) − i](x + 5)(x − 5) zero products

= (x2 − 2x + 1 + 1)(x2 − 5) 2± 4−20
x = 4 or x=
= (x2 − 2x + 2)(x2 − 5) 2
Quadratic formula
= x4 − 2x3 + 2x2 − 5x2 + 10x − 10
2±4i
= x4 − 2x3 − 3x2 + 10x − 10 x = 4 or x= = 1 ± 2i
2
41. Find a polynomial function of lowest degree with rational The other zeros are 1 + 2i and 1 − 2i.

coefficients that has 5 and −3i as some of its zeros.
√ 49. f (x) = x5 − 3x2 + 1
− 5 and 3i are also zeros.
According to the rational zeros theorem, any rational zero
Thus the polynomial function is of f must be of the form p/q, where p is a factor of the
√ √
f (x) = an (x − 5)(x + 5)(x + 3i)(x − 3i). constant term, 1, and q is a factor of the coefficient of x5 ,
1.

Copyright © 2013 Pearson Education, Inc.


Exercise Set 4.4 147


Possibilities for p ±1 1  3 −1 −15 5
: 3
Possibilities for q ±1 1 0 −5
Possibilities for p/q: 1, −1 3 0 −15 0
 
1
51. f (x) = 2x4 − 3x3 − x + 8 Then we have f (x) = x− (3x2 − 15), or
  3
According to the rational zeros theorem, any rational zero 1
of f must be of the form p/q, where p is a factor of the 3 x− (x2 − 5).
3
constant term, 8, and q is a factor of the coefficient of x4 , √
2. Now x2 − 5 = 0 for x = ± 5. Thus, there is
1
Possibilities for p ±1, ±2, ±4, ±8 only one rational zero, . The other zeros are
: √ 3
Possibilities for q ±1, ±2 ± 5. (Note that we could have used factoring
1 1 by grouping to find this result.)
Possibilities for p/q: 1, −1, 2, −2, 4, −4, 8, −8, , −  
2 2 1 √ √
b) f (x) = 3 x − (x + 5)(x − 5)
53. f (x) = 15x6 + 47x2 + 2 3
According to the rational zeros theorem, any rational zero
59. f (x) = x3 − 3x + 2
of f must be of the form p/q, where p is a factor of 2 and
q is a factor of 15. Possibilities for p ±1, ±2
a) :
Possibilities for p ±1, ±2 Possibilities for q ±1
: Possibilities for p/q: 1, −1, 2, −2
Possibilities for q ±1, ±3, ±5, ±15
1 1 2 2 1 We use synthetic division to find a zero. We find
Possibilities for p/q: 1, −1, 2, −2, , − , , − , ,
3 3 3 3 5 that −2 is a zero as shown below.

1 2 2 1 1 2 2 −2  1 0 −3 2
− , ,− , ,− , ,−
5 5 5 15 15 15 15 −2 4 −2
1 −2 1 0
55. f (x) = x3 + 3x2 − 2x − 6
Possibilities for p ±1, ±2, ±3, ±6 Then we have f (x) = (x + 2)(x2 − 2x + 1) =
a) : (x + 2)(x − 1)2 .
Possibilities for q ±1
Possibilities for p/q: 1, −1, 2, −2, 3, −3, 6, −6 Now (x − 1)2 = 0 for x = 1. Thus, the rational
zeros are −2 and 1. (The zero 1 has a multiplicity
We use synthetic division to find a zero. We find of 2.) These are the only zeros.
that one zero is −3 as shown below.
 b) f (x) = (x + 2)(x − 1)2
−3  1 3 −2 −6
−3 0 6 61. f (x) = 2x3 + 3x2 + 18x + 27
1 0 −2 0 Possibilities for p ±1, ±3, ±9, ±27
a) :
Then we have f (x) = (x + 3)(x2 − 2). Possibilities for q ±1, ±2
We find the other zeros: Possibilities for p/q: 1, −1, 3, −3, 9, −9, 27, −27,
x2 − 2 = 0 1 1 3 3 9 9 27 27
,− , ,− , ,− , ,−
2 2 2 2 2 2 2 2
x2 = 2
√ We use synthetic division to find a zero. We find
x = ± 2. 3
that − is a zero as shown below.
√ one rational zero, −3. The other
There is only  2
zeros are ± 2. (Note that we could have used − 32  2 3 18 27
factoring by grouping to find this result.) −3 0 −27
√ √ 2 0 18 0
b) f (x) = (x + 3)(x − 2)(x + 2)  
3
57. f (x) = 3x3 − x2 − 15x + 5 Then we have f (x) = x + (2x2 + 18), or
  2
Possibilities for p ±1, ±5 3
a) : 2 x+ (x2 + 9).
Possibilities for q ±1, ±3 2
1 1 5 5 Now x2 + 9 = 0 for x = ±3i. Thus, the
Possibilities for p/q: 1, −1, 5, −5, , − , , −
3 3 3 3 3
only rational zero is − . The other zeros are
We use synthetic division to find a zero. We find 2
1 ±3i. (Note that we could have used factoring by
that one zero is as shown below. grouping to find this result.)
3
 
3
b) f (x) = 2 x + (x + 3i)(x − 3i)
2

Copyright © 2013 Pearson Education, Inc.


148 Chapter 4: Polynomial and Rational Functions

√ √
63. f (x) = 5x4 − 4x3 + 19x2 − 16x − 4 b) f (x) = (x+2)(x+1)[x−(3+ 13)][x−(3− 13)]
√ √
Possibilities for p ±1, ±2, ±4 = (x+2)(x+1)(x−3− 13)(x−3+ 13)
a) :
Possibilities for q ±1, ±5
67. f (x) = x3 − 4x2 + 2x + 4
1 1
Possibilities for p/q: 1, −1, 2, −2, 4, −4, , − Possibilities for p ±1, ±2, ±4
5 5 a) :
2 2 4 4 Possibilities for q ±1
,− , ,−
5 5 5 5 Possibilities for p/q: 1, −1, 2, −2, 4, −4
We use synthetic division to find a zero. We find Synthetic division shows that neither −1 nor 1
that
 1 is a zero as shown below. is a zero. Try 2.

1  5 −4 19 −16 −4 2  1 −4 2 4
5 1 20 4 2 −4 −4
5 1 20 4 0 1 −2 −2 0
Then we have
Then we have f (x) = (x − 2)(x2 − 2x − 2). Use
f (x) = (x − 1)(5x3 + x2 + 20x + 4) the quadratic formula to find the other zeros.
= (x − 1)[x2 (5x + 1) + 4(5x + 1)] x2 − 2x − 2 = 0
= (x − 1)(5x + 1)(x + 4).
2 
−(−2) ± (−2)2 − 4 · 1 · (−2)
We find the other zeros: x=
2·1
5x + 1 = 0 or x2 + 4 = 0 √ √
2 ± 12 2±2 3
5x = −1 or x2 = −4 =
2
=
2
1 √
x = − or x = ±2i = 1± 3
5
1 The√only rational zero is 2. The other zeros are
The rational zeros are − and 1. The other zeros 1 ± 3.
5 √ √
are ±2i. b) f (x) = (x − 2)[x − (1 + 3)][x − (1 − 3)]
√ √
b) From part (a) we see that = (x − 2)(x − 1 − 3)(x − 1 + 3)
(x) = (5x + 1)(x − 1)(x + 2i)(x − 2i), or
f
69. f (x) = x3 + 8
1
5 x+ (x − 1)(x + 2i)(x − 2i). Possibilities for p ±1, ±2, ±4, ±8
5 a) :
Possibilities for q ±1
65. f (x) = x4 − 3x3 − 20x2 − 24x − 8
Possibilities for p/q: 1, −1, 2, −2, 4, −4, 8, −8
Possibilities for p ±1, ±2, ±4, ±8
a) : We use synthetic division to find a zero. We find
Possibilities for q ±1 that −2 is a zero as shown below.
Possibilities for p/q: 1, −1, 2, −2, 4, −4, 8, −8 
−2  1 0 0 8
We use synthetic division to find a zero. We find −2 4 −8
that −2 is a zero as shown below. 1 −2 4 0

−2  1 −3 −20 −24 −8 We have f (x) = (x + 2)(x2 − 2x + 4). Use the
−2 10 20 8 quadratic formula to find the other zeros.
1 −5 −10 −4 0
x2 − 2x + 4 = 0
Now we determine whether −1 is a zero. 
 −(−2) ±
(−2)2 − 4 · 1 · 4
−1  1 −5 −10 −4 x=
2·1
−1 6 4 √ √
1 −6 −4 0 2 ± −12 2 ± 2 3i
= =
Then we have f (x) = (x + 2)(x + 1)(x2 − 6x − 4). 2 2

= 1 ± 3i
Use the quadratic formula to find the other zeros.
x2 − 6x − 4 = 0
The only√ rational zero is −2. The other zeros
 are 1 ± 3i.
−(−6) ± (−6)2 − 4 · 1 · (−4) √ √
x= b) f (x) = (x + 2)[x − (1 + 3i)][x − (1 − 3i)]
2·1 √ √
√ √ = (x + 2)(x − 1 − 3i)(x − 1 + 3i)
6 ± 52 6 ± 2 13
= = 1 3 1 2 1 1
2 2 71. f (x) = x − x − x+
√ 3 2 6 6
= 3 ± 13
1
The rational
√ zeros are −2 and −1. The other zeros = (2x3 − 3x2 − x + 1)
are 3 ± 13. 6

Copyright © 2013 Pearson Education, Inc.


Exercise Set 4.4 149

a) The second form of the equation is equivalent to 79. f (x) = x5 − 5x4 + 5x3 + 15x2 − 36x + 20
the first and has the advantage of having integer According to the rational zeros theorem, the possible ra-
coefficients. Thus, we can use the rational zeros tional zeros are ±1, ±2, ±4, ±5, ±10, and ±20. We try
theorem for g(x) = 2x3 − 3x2 − x + 1. The zeros −2.
of g(x) are the same as the zeros of f (x). We 
−2  1 −5 5 15 −36 20
find the zeros of g(x).
−2 14 −38 46 −20
Possibilities for p ±1 1 −7 19 −23 10 0
:
Possibilities for q ±1, ±2
Thus, −2 is a zero. Now try 1.
1 1 
Possibilities for p/q: 1, −1, , − 1  1 −7 19 −23 10
2 2
1 −6 13 −10
1
Synthetic division shows that − is not a zero. 1 −6 13 −10 0
2
1 1 is also a zero. Try 2.
Try . 
2 2  1 −6 13 −10
 2 −8
1  10
2  2 −3 −1 1 1 −4 5 0
1 −1 −1
2 is also a zero.
2 −2 −2 0
  We have f (x) = (x + 2)(x − 1)(x − 2)(x2 − 4x + 5). The
1 discriminant of x2 − 4x + 5 is (−4)2 − 4 · 1 · 5, or 4 < 0,
We have g(x) = x − (2x2 − 2x − 2) =
  2 so x2 − 4x + 5 has two nonreal zeros. Thus, the rational
1 zeros are −2, 1, and 2.
x− (2)(x2 − x − 1). Use the quadratic for-
2
mula to find the other zeros. 81. f (x) = 3x5 − 2x2 + x − 1
x2 − x − 1 = 0 The number of variations in sign in f (x) is 3. Then the

−(−1) ± (−1)2 − 4 · 1 · (−1) number of positive real zeros is either 3 or less than 3 by
x= 2, 4, 6, and so on. Thus, the number of positive real zeros
2·1
√ is 3 or 1.
1± 5 f (−x) = 3(−x)5 − 2(−x)2 + (−x) − 1
=
2
1 = −3x5 − 2x2 − x − 1
The only rational zero is . The other zeros are There are no variations in sign in f (−x), so there are 0
√ 2
1± 5 negative real zeros.
.
2
83. h(x) = 6x7 + 2x2 + 5x + 4
1
b) f (x) = g(x) There are no variations in sign in h(x), so there are 0
6
   √  √  positive real zeros.
1 1 1+ 5 1− 5
= x− (2) x− x− h(−x) = 6(−x)7 + 2(−x)2 + 5(−x) + 4
6 2 2 2
  √  √  = −6x7 + 2x2 − 5x + 4
1 1 1+ 5 1− 5
= x− x− x− The number of variations in sign in h(−x) is 3. Thus, there
3 2 2 2
are 3 or 1 negative real zeros.
73. f (x) = x4 + 2x3 − 5x2 − 4x + 6 85. F (p) = 3p18 + 2p4 − 5p2 + p + 3
According to the rational zeros theorem, the possible ra- There are 2 variations in sign in F (p), so there are 2 or 0
tional zeros are ±1, ±2 , ±3, and ±6. Synthetic division positive real zeros.
shows that only 1 and −3 are zeros. F (−p) = 3(−p)18 + 2(−p)4 − 5(−p)2 + (−p) + 3
75. f (x) = x − x − 4x + 3
3 2 = 3p18 + 2p4 − 5p2 − p + 3
According to the rational zeros theorem, the possible ra- There are 2 variations in sign in F (−p), so there are 2 or
tional zeros are ±1 and ±3. Synthetic division shows that 0 negative real zeros.
none of these is a zero. Thus, there are no rational zeros.
87. C(x) = 7x6 + 3x4 − x − 10
77. f (x) = x + 2x + 2x − 4x − 8
4 3 2
There is 1 variation in sign in C(x), so there is 1 positive
According to the rational zeros theorem, the possible ra- real zero.
tional zeros are ±1, ±2, ±4, and ±8. Synthetic division C(−x) = 7(−x)6 + 3(−x)4 − (−x) − 10
shows that none of the possibilities is a zero. Thus, there = 7x6 + 3x4 + x − 10
are no rational zeros.
There is 1 variation in sign in C(−x), so there is 1 negative
real zero.

Copyright © 2013 Pearson Education, Inc.


150 Chapter 4: Polynomial and Rational Functions

89. h(t) = −4t5 − t3 + 2t2 + 1  the x-axis


3. The zeros divide  into
 4 intervals,

√ √ 1 1 √
There is 1 variation in sign in h(t), so there is 1 positive (−∞, − 2), − 2, − , − , 2 , and
real zero. √ 4 4
h(−t) = −4(−t)5 − (−t)3 + 2(−t)2 + 1 ( 2, ∞). We choose a value for x from each interval
and find f (x). This tells us the sign of f (x) for all
= 4t5 + t3 + 2t2 + 1 values of x in that interval.

There are no variations in sign in h(−t), so there are 0 In (−∞, − 2), test −2:
negative real zeros.
f (−2) = 4(−2)3 + (−2)2 − 8(−2) − 2 = −14 < 0
 
91. f (y) = y 4 + 13y 3 − y + 5 √ 1
In − 2, − , test −1:
There are 2 variations in sign in f (y), so there are 2 or 0 4
positive real zeros. f (−1) = 4(−1)3 + (−1)2 − 8(−1) − 2 = 3 > 0
 
f (−y) = (−y)4 + 13(−y)3 − (−y) + 5 1 √
In − , 2 , test 0:
= y 4 − 13y 3 + y + 5 4
There are 2 variations in sign in f (−y), so there are 2 or f (0) = 4 · 03 + 02 − 8 · 0 − 2 = −2 < 0
0 negative real zeros. √
In ( 2, ∞), test 2:
93. r(x) = x4 − 6x2 + 20x − 24 f (2) = 4 · 23 + 22 − 8 · 2 − 2 = 18 > 0

There are 3 variations in sign in r(x), so there are 3 or 1 Thus thegraph liesbelow the x-axis on (−∞, − 2)
positive real zeros. 1 √
and on − , 2 . It lies above the x-axis on
r(−x) = (−x)4 − 6(−x)2 + 20(−x) − 24  4
√ 1 √
= x4 − 6x2 − 20x − 24 − 2, − and on ( 2, ∞). We also know the
4
There is 1 variation in sign in r(−x), so there is 1 negative points (−2, −14), (−1, 3), (0, −2), and (2, 18) are on
real zero. the graph.
95. R(x) = 3x5 − 5x3 − 4x 4. From Step 3 we see that f (0) = −2 so the y-intercept
is (0, −2).
There is 1 variation in sign in R(x), so there is 1 positive
real zero. 5. We find additional points on the graph and then draw
the graph.
R(−x) = 3(−x)5 − 5(−x)3 − 4(−x)
y
= −3x5 + 5x3 + 4x x f (x)
8
There is 1 variation in sign in R(−x), so there is 1 negative −1.5 −1.25
real zero. 4
−0.5 1.75
97. f (x) = 4x3 + x2 − 8x − 2 4 2 2 4 x
1 −5 4
3
1. The leading term is 4x . The degree, 3, is odd and
8
the leading coefficient, 4, is positive so as x → ∞, 1.5 1.75
f (x) → ∞ and x → −∞, f (x) → −∞.
2. We find the rational zeros p/q of f (x). f (x )  4x 3  x 2  8x  2

Possibilities for p ±1, ±2 6. Checking the graph as described on page 317 in the
:
Possibilities for q ±1, ±2, ±4 text, we see that it appears to be correct.
1 1 1 1
Possibilities for p/q: 1, −1 , 2, −2, , − , , − 99. f (x) = 2x4 − 3x3 − 2x2 + 3x
2 2 4 4
1. The leading term is 2x4 . The degree, 4, is even and
1 the leading coefficient, 2, is positive so as x → ∞,
Synthetic division shows that − is a zero.
 4 f (x) → ∞ and as x → −∞, f (x) → ∞.
− 14  4 1 −8 −2 2. We find the rational zeros p/q of f (x). First note that
−1 0 2 f (x) = x(2x3 − 3x2 − 2x + 3), so 0 is a zero. Now
4 0 −8 0 consider g(x) = 2x3 − 3x2 − 2x + 3.
 
We have f (x) = x +
1
(4x2 − 8) = Possibilities for p ±1, ±3
4 :
  Possibilities for q ±1, ±2
1
4 x+ (x2 − 2). Solving x2 − 2 = 0 we get 1 1 3
Possibilities for p/q: 1, −1 , 3, −3, , − , , −
3
4 2 2 2 2
√ 1
x = ± 2. Thus the zeros of the function are − , We try 1.
√ √ 4 
− 2, and
  2 so the x-intercepts of the graph are 1  2 −3 −2 3
1 √ √
− , 0 , (− 2, 0), and ( 2, 0). 2 −1 −3
4 2 −1 −3 0

Copyright © 2013 Pearson Education, Inc.


Exercise Set 4.4 151

Then f (x) = x(x−1)(2x2 −x−3). Using the principle 101. f (x) = x2 − 8x + 10


3
of zero products to solve 2x2 −x−3 = 0, we get x = b −8
2 a) − =− = −(−4) = 4
or x = −1. 2a 2·1
3 f (4) = 42 − 8 · 4 + 10 = −6
Thus the zeros of the function are 0, 1, , and −1 so
2   The vertex is (4, −6).
3
the x-intercepts of the graph are (0, 0), (1, 0), ,0 , b) The axis of symmetry is x = 4.
2
and (−1, 0). c) Since the coefficient of x2 is positive, there is a
3. The zeros dividethe x-axis
 into
 5 intervals,
 (−∞, −1), minimum function value. It is the second coor-
3 3 dinate of the vertex, −6. It occurs when x = 4.
(−1, 0), (0, 1), 1, , and , ∞ . We choose a
2 2 4
value for x from each interval and find f (x). This 103. − x+8 =0
5
tells us the sign of f (x) for all values of x in that
4
interval. − x = −8 Subtracting 8
In (−∞, −1), test −2: 5
 
5 4 5 5
f (−2) = 2(−2)4 − 3(−2)3 − 2(−2)2 + 3(−2) = 42 > 0 − − x = − (−8) Multiplying by −
4 5 4 4
In (−1, 0), test −0.5:
x = 10
f (−0.5) = 2(−0.5)4 −3(−0.5)3 −2(−0.5)2 +3(−0.5) = The zero is 10.
−1.5 < 0
In (0, 1), test 0.5: 105. g(x) = −x3 − 2x2

f (0.5) = 2(0.5)4 −3(0.5)3 −2(0.5)2 +3(0.5) = 0.75 > 0 Leading term: −x3 ; leading coefficient: −1
  The degree is 3, so the function is cubic.
3
In 1, , test 1.25:
2 Since the degree is odd and the leading coefficient is nega-
tive, as x → ∞, g(x) → −∞ and as x → −∞, g(x) → ∞.
f (1.25) = 2(1.25) − 3(1.25) − 2(1.25) + 3(1.25) =
4 3 2

−0.3515625 < 0 4
  107. f (x) = −
3 9
In , ∞ , test 2: 4 4
2 Leading term: − ; leading coefficient: − ;
9 9
f (2) = 2 · 24 − 3 · 23 − 2 · 22 + 3 · 2 = 6 > 0 4
for all x, f (x) = −
Thus the graph lies above  the x-axis on (−∞, −1), 9
3 The degree is 0, so this is a constant function.
on (0, 1), and on , ∞ . It lies below the x-axis
2 
3 109. g(x) = x4 − 2x3 + x2 − x + 2
on (−1, 0) and on 1, . We also know the points
2 Leading term: x4 ; leading coefficient: 1
(−2, 42), (−0.5, −1.5), (0.5, 0.75), (1.25, −0.3515625),
The degree is 4, so the function is quartic.
and (2, 6) are on the graph.
Since the degree is even and the leading coefficient is pos-
4. From Step 2 we know that f (0) = 0 so the y-intercept
itive, as x → ∞, g(x) → ∞ and as x → −∞, g(x) → ∞.
is (0, 0).
5. We find additional points on the graph and then draw 111. f (x) = 2x3 − 5x2 − 4x + 3
the graph. a) 2x3 − 5x2 − 4x + 3 = 0
y Possibilities for p ±1, ±3
x f (x) :
4 Possibilities for q ±1, ±2
−1.5 11.25 1 1 3 3
2 Possibilities for p/q: 1, −1, 3, −3, ,− , ,−
2 2 2 2
2.5 26.25 4 2 2 4 x The first possibility that is a solution of f (x) = 0 is
2 −1:
3 72 
4 −1  2 −5 −4 3
−2 7 −3
f (x )  2x 4  3x 3  2x 2  3x 2 −7 3 0
Thus, −1 is a solution.
6. Checking the graph as described on page 317 in the Then we have:
text, we see that it appears to be correct. (x + 1)(2x2 − 7x + 3) = 0
(x + 1)(2x − 1)(x − 3) = 0
1
The other solutions are and 3.
2

Copyright © 2013 Pearson Education, Inc.


152 Chapter 4: Polynomial and Rational Functions

b) The graph of y = f (x − 1) is the graph of y = f (x) x+1


3. f (x) =
shifted 1 unit right. Thus, we add 1 to each solution x2 − 6x + 5
of f (x) = 0 to find the solutions of f (x − 1) = 0. We find the value(s) of x for which the denominator is 0.
1 3 x2 − 6x + 5 = 0
The solutions are −1 + 1, or 0; + 1, or ; and
2 2
3 + 1, or 4. (x − 1)(x − 5) = 0
c) The graph of y = f (x + 2) is the graph of y = x − 1 = 0 or x − 5 = 0
f (x) shifted 2 units left. Thus, we subtract 2 from x = 1 or x=5
each solution of f (x) = 0 to find the solutions of
f (x + 2) = 0. The solutions are −1 − 2, or −3; The domain is {x|x = 1 and x = 5}, or (−∞, 1) ∪ (1, 5) ∪
1 3 (5, ∞).
− 2, or − ; and 3 − 2, or 1.
2 2 3x − 4
d) The graph of y = f (2x) is a horizontal shrinking of 5. f (x) =
3x + 15
the graph of y = f (x) by a factor of 2. We divide We find the value(s) of x for which the denominator is 0.
each solution of f (x) = 0 by 2 to find the solutions
−1 1 1/2 3x + 15 = 0
of f (2x) = 0. The solutions are or − ; , or
2 2 2 3x = −15
1 3
; and . x = −5
4 2
The domain is {x|x = −5}, or (−∞, −5) ∪ (−5, ∞).
113. P (x) = 2x5 − 33x4 − 84x3 + 2203x2 − 3348x − 10, 080
8
2x5 − 33x4 − 84x3 + 2203x2 − 3348x − 10, 080 = 0 7. Graph (d) is the graph of f (x) = .
x2 − 4
Trying some of the many possibilities for p/q, we find that
x2 − 4 = 0 when x = ±2, so x = −2 and x = 2 are vertical
4 is a zero.
asymptotes.
4  2 −33 −84 2203 −3348 −10, 080
8 −100 −736 5868 10, 080 The x-axis, y = 0, is the horizontal asymptote because
2 −25 −184 1467 2520 0 the degree of the numerator is less than the degree of the
denominator.
Then we have:
There is no oblique asymptote.
(x − 4)(2x4 − 25x3 − 184x2 + 1467x + 2520) = 0
We now use the fourth degree polynomial above to find 8x
9. Graph (e) is the graph of f (x) = .
another zero. Synthetic division shows that 4 is not a x2 − 4
double zero, but 7 is a zero. As in Exercise 7, x = −2 and x = 2 are vertical asymp-

7  2 −25 −184
totes.
1467 2520
14 −77 −1827 −2520 The x-axis, y = 0, is the horizontal asymptote because
2 −11 −261 −360 0 the degree of the numerator is less than the degree of the
Now we have: denominator.

(x − 4)(x − 7)(2x3 − 11x2 − 261x − 360) = 0 There is no oblique asymptote.


Use the third degree polynomial above to find a third zero. 8x3
11. Graph (c) is the graph of f (x) = .
Synthetic division shows that 7 is not a double zero, but −4x2
15 is a zero. As in Exercise 7, x = −2 and x = 2 are vertical asymp-
15  2 −11 −261 −360 totes.
30 285 360 The degree of the numerator is greater than the degree of
2 19 24 0 the denominator, so there is no horizontal asymptote but
We have: there is an oblique asymptote. To find it we first divide to
P (x) = (x − 4)(x − 7)(x − 15)(2x2 + 19x + 24) find an equivalent expression.
= (x − 4)(x − 7)(x − 15)(2x + 3)(x + 8) 8x
3 x2 − 4 8x3
The rational zeros are 4, 7, 15, − , and −8. 8x3 −32x
2
32x
Exercise Set 4.5 8x3 32x
= 8x + 2
x2 −4 x −4
x2 Now we multiply by 1, using (1/x2 )/(1/x2 ).
1. f (x) =
2−x 1 32
We find the value(s) of x for which the denominator is 0. 32x x2 x
· =
x2 − 4 1 4
2−x = 0 1− 2
x2 x
2=x As |x| becomes very large, each expression with x in the
The domain is {x|x = 2}, or (−∞, 2) ∪ (2, ∞). denominator tends toward zero.

Copyright © 2013 Pearson Education, Inc.


Exercise Set 4.5 153

Then, as |x| → ∞, we have x4 − 2


29. h(x) =
32 x3 + 1
x 0
→ , or 0. x
4 1−0 x3 + 1 x4 + 0x3 + 0x2 + 0x
1− 2
x x4 + x
Thus, as |x| becomes very large, the graph of f (x) gets − x
very close to the graph of y = 8x, so y = 8x is the oblique −x
asymptote. Then h(x) = x + 3 . The oblique asymptote is
x +1
y = x.
1
13. g(x) =
x2 x3 − x2 + x − 4
31. f (x) =
The numerator and the denominator have no common fac- x2 + 2x − 1
tors. The zero of the denominator is 0, so the vertical
x− 3
asymptote is x = 0.
x2 + 2x − 1 x3 − x2 + x − 4
x+7 x3 + 2x2 − x
15. h(x) =
2−x − 3x2 + 2x − 4
The numerator and the denominator have no common fac- − 3x2 − 6x + 3
tors. 2 − x = 0 when x = 2, so the vertical asymptote is 8x − 7
x = 2. 8x − 7
Then f (x) = x − 3 + . The oblique asymptote
3−x x2 + 2x − 1
17. f (x) = is y = x − 3.
(x − 4)(x + 6)
The numerator and the denominator have no common fac- 1
33. f (x) =
tors. The zeros of the denominator are 4 and −6, so the x
vertical asymptotes are x = 4 and x = −6. 1. The numerator and the denominator have no com-
mon factors. 0 is the zero of the denominator, so
x2 x2 the domain excludes 0. It is (−∞, 0) ∪ (0, ∞). The
19. g(x) = =
2x2 − x − 3 (2x − 3)(x + 1) line x = 0, or the y-axis, is the vertical asymptote.
The numerator and the denominator have no common fac- 2. Because the degree of the numerator is less than
3
tors. The zeros of the denominator are and −1, so the the degree of the denominator, the x-axis, or y = 0,
2
3 is the horizontal asymptote. There are no oblique
vertical asymptotes are x = and x = −1. asymptotes.
2
3x2 + 5 3. The numerator has no zeros, so there is no x-
21. f (x) = intercept.
4x2 − 3
The numerator and the denominator have the same degree 4. Since 0 is not in the domain of the function, there
3 3 is no y-intercept.
and the ratio of the leading coefficients is , so y = is
4 4
the horizontal asymptote. 5. Find other function values to determine the shape
of the graph and then draw it.
x2 − 4
23. h(x) =
2x4 + 3 y
The degree of the numerator is less than the degree of the 4
denominator, so y = 0 is the horizontal asymptote. f (x)  1
x
y0 2
x3 − 2x2 + x − 1
25. g(x) = 4 2 2 4 x
x2 − 16 x0
2
The degree of the numerator is greater than the degree of
4
the denominator, so there is no horizontal asymptote.
x2 + 4x − 1
27. g(x) = 4
x+3 35. h(x) = −
x2
x+ 1
1. The numerator and the denominator have no com-
x + 3 x2 + 4x − 1
mon factors. 0 is the zero of the denominator, so
x2 + 3x
the domain excludes 0. It is (−∞, 0) ∪ (0, ∞). The
x − 1
line x = 0, or the y-axis, is the vertical asymptote.
x + 3
−4 2. Because the degree of the numerator is less than
the degree of the denominator, the x-axis, or y = 0,
−4
Then g(x) = x + 1 + . The oblique asymptote is is the horizontal asymptote. There is no oblique
x+3
y = x + 1. asymptote.

Copyright © 2013 Pearson Education, Inc.


154 Chapter 4: Polynomial and Rational Functions

3. The numerator has no zeros, so there is no x- 2. Because the degree of the numerator is less than
intercept. the degree of the denominator, the x-axis, or y = 0,
4. Since 0 is not in the domain of the function, there is the horizontal asymptote. There is no oblique
is no y-intercept. asymptote.

5. Find other function values to determine the shape 3. The numerator has no zeros, so there is no x-
of the graph and then draw it. intercept.
 
−2 2 2
y 4. f (0) = = , so 0, is the y-intercept.
0−5 5 5
4 5. Find other function values to determine the shape
x0
y0 2 of the graph and then draw it.
4 2 2 4 x y
2 2
8 f(x) 
h (x)   42 x5
4 x 6

x − 4x + 3
2
(x − 1)(x − 3) 2 y0
37. g(x) = =
x+1 x+1 2 2 4 6 x
1. The numerator and the denominator have no com- 2
x5
mon factors. The denominator, x + 1, is 0 when x = 4
−1, so the domain excludes −1. It is (−∞, −1) ∪
(−1, ∞). The line x = −1 is the vertical asymptote. 2x + 1
41. f (x) =
2. The degree of the numerator is 1 greater than the x
degree of the denominator, so we divide to find the 1. The numerator and the denominator have no com-
oblique asymptote. mon factors. 0 is the zero of the denominator, so
the domain excludes 0. It is (−∞, 0) ∪ (0, ∞). The
x− 5
line x = 0, or the y-axis, is the vertical asymptote.
x + 1 x2 − 4x + 3
x2 + x 2. The numerator and denominator have the same de-
− 5x + 3 gree, so the horizontal asymptote is determined by
− 5x − 5 the ratio of the leading coefficients, 2/1, or 2. Thus,
8 y = 2 is the horizontal asymptote. There is no
oblique asymptote.
The oblique asymptote is y = x − 5. There is no
horizontal asymptote.  solution
3. The zero of the numerator is the  of 2x+1 =
1 1
3. The zeros of the numerator are 1 and 3. Thus the 0, or − . The x-intercept is − , 0 .
x-intercepts are (1, 0) and (3, 0). 2 2
4. Since 0 is not in the domain of the function, there
02 − 4 · 0 + 3 is no y-intercept.
4. g(0) = = 3, so the y-intercept is (0, 3).
0+1 5. Find other function values to determine the shape
5. Find other function values to determine the shape of the graph and then draw it.
of the graph and then draw it.
y
y 6 2x  1
f(x) 
x  1 4 4 x
y2
8 4 4 8 x
4 yx5 4 2 2 4 x
8
x2  4x  3 冢q, 0冣 x0
g(x) 
12 x1
16
x+3 x+3
43. f (x) = =
x2 − 9 (x + 3)(x − 3)
−2 1. The domain of the function is (−∞, −3) ∪ (−3, 3) ∪
39. f (x) =
x−5 (3, ∞). The numerator and denominator have the
1. The numerator and the denominator have no com- common factor x + 3. The zeros of the denominator
mon factors. 5 is the zero of the denominator, so are −3 and 3, and the zero of the numerator is −3.
the domain excludes 5. It is (−∞, 5) ∪ (5, ∞). The Since 3 is the only zero of the denominator that
line x = 5 is the vertical asymptote. is not a zero of the numerator, the only vertical
asymptote is x = 3.

Copyright © 2013 Pearson Education, Inc.


Exercise Set 4.5 155

2. Because the degree of the numerator is less than 3. The numerator has no zeros, so there is no x-
the degree of the denominator, the x-axis, or y = 0, intercept.
 
is the horizontal asymptote. There are no oblique 1 1 1
asymptotes. 4. f (0) = = , so 0, is the y-
(0 − 2)2 4 4
3. The zero of the numerator, −3, is not in the domain intercept.
of the function, so there is no x-intercept. 5. Find other function values to determine the shape
 
0+3 1 1 of the graph and then draw it.
4. f (0) = = − , so the y-intercept is 0, − .
02 − 9 3 3
y
5. Find other function values to determine the shape x2
4
of the graph and then draw it.
3
y 2
冢0, ~冣1
4 y0
x 3 1 1 2 3 4
2 x
(3, 0) 1 1
4 2 2 4 x f(x) 
2 (x  2)2
f (x )  x2  3 2 y 0
x 9
4 x2 + 2x − 3 (x + 3)(x − 1)
49. f (x) = =
x2 + 4x + 3 (x + 3)(x + 1)
x x 1. The zeros of the denominator are −3 and −1, so
45. f (x) = =
x2 + 3x x(x + 3) the domain is (−∞, −3) ∪ (−3, −1) ∪ (−1, ∞). The
1. The zeros of the denominator are 0 and −3, so the zeros of the numerator are −3 and 1. Since −1 is
domain is (−∞, −3) ∪ (−3, 0) ∪ (0, ∞). The zero the only zero of the denominator that is not also a
of the numerator is 0. Since −3 is the only zero zero of the numerator, the only vertical asymptote
of the denominator that is not also a zero of the is x = −1.
numerator, the only vertical asymptote is x = −3. 2. The numerator and the denominator have the same
2. Because the degree of the numerator is less than degree, so the horizontal asymptote is determined
the degree of the denominator, the x-axis, or y = 0, by the ratio of the leading coefficients, 1/1, or 1.
is the horizontal asymptote. There is no oblique Thus, y = 1 is the horizontal asymptote. There is
asymptote. no oblique asymptote.
3. The zero of the numerator is 0, but 0 is not in the 3. The only zero of the numerator that is in the domain
domain of the function, so there is no x-intercept. of the function is 1, so the only x-intercept is (1, 0).
4. Since 0 is not in the domain of the function, there 02 + 2 · 0 − 3 −3
4. f (0) = 2 = = −1, so the
is no y-intercept. 0 +4·0+3 3
y-intercept is (0, −1).
5. Find other function values to determine the shape
5. Find other function values to determine the shape
of the graph and then draw it, indicating the “hole”
when x = 0 with an open circle. of the graph and then draw it, indicating the “hole”
when x = −3 with an open circle.
y
x y
f (x ) 
4 x 2  3x
4
x  3 2 (0, 13 ) (3, 2)
2 y 1
6 4 2 2 x 6 4 2 2 4 x
2 y0
2
4
4
x  1
1
47. f (x) = x 2  2x  3
(x − 2)2 f (x ) 
x 2  4x  3
1. The numerator and the denominator have no comm-
non factors. 2 is the zero of the denominator, so the 1
51. f (x) =
domain excludes 2. It is (−∞, 2) ∪ (2, ∞). The line x2+3
x = 2 is the vertical asymptote. 1. The numerator and the denominator have no com-
mon factors. The denominator has no real-number
2. Because the degree of the numerator is less than
zeros, so the domain is (−∞, ∞) and there is no
the degree of the denominator, the x-axis, or y = 0,
vertical asymptote.
is the horizontal asymptote. There is no oblique
asymptote.

Copyright © 2013 Pearson Education, Inc.


156 Chapter 4: Polynomial and Rational Functions

2. Because the degree of the numerator is less than y


the degree of the denominator, the x-axis, or y = 0, 4
is the horizontal asymptote. There is no oblique
asymptote.
2 y1

3. The numerator has no zeros, so there is no x- 4 2 4 x


intercept. x  2 x1
  f(x) 
1 1 1 4 x2
4. f (0) = = , so 0, is the y-intercept.
02 + 3 3 3
5. Find other function values to determine the shape x2 +3x x(x+3) x(x+3)
57. f (x) = = =
of the graph and then draw it. 2x3 −5x2 −3x x(2x2 −5x−3) x(2x+1)(x−3)
1
y 1. The zeros of the denominator are 0, − , and 3, so
    2
1 1
冢0, a冣 0.5 the domain is − ∞,− ∪ − , 0 ∪(0, 3)∪(3, ∞).
2 2
The zeros of the numerator are 0 and −3. Since
0.5
1
0.5 x − and 3 are the only zeros of the denominator
y0 2
0.5 1 that are not also zeros of the numerator, the vertical
f(x)  2
x 3 1
asymptotes are x = − and x = 3.
2
x2 − 4 (x + 2)(x − 2) 2. Because the degree of the numerator is less than
53. f (x) = = = x + 2, x = 2 the degree of the denominator, the x-axis, or y = 0,
x−2 x−2
is the horizontal asymptote. There is no oblique
The graph is the same as the graph of f (x) = x + 2 ex- asymptote.
cept at x = 2, where there is a hole. Thus the domain is
(−∞, 2) ∪ (2, ∞). The zero of f (x) = x + 2 is −2, so the 3. The only zero of the numerator that is in the do-
x-intercept is (−2, 0); f (0) = 2, so the y-intercept is (0, 2). main of the function is −3 so the only x-intercept is
(−3, 0).
y 4. 0 is not in the domain of the function, so there is no
4 y-intercept.
(2, 4)
5. Find other function values to determine the shape
of the graph and then draw it, indicating the “hole”
4 2 4 x when x = 0 with an open circle.
2
4
x2
4 f(x)  y
x2
1 y0
x−1
55. f (x) = 4 2 2 4 x
x+2 x  q x3
1. The numerator and the denominator have no com- (0,  1)
mon factors. −2 is the zero of the denominator, so
the domain excludes −2. It is (−∞, −2) ∪ (−2, ∞).
x 2  3x
The line x = −2 is the vertical asymptote. f (x)  3
2x  5x 2  3x
2. The numerator and denominator have the same de-
gree, so the horizontal asymptote is determined by x2 − 9 (x + 3)(x − 3)
59. f (x) = =
the ratio of the leading coefficients, 1/1, or 1. Thus, x+1 x+1
y = 1 is the horizontal asymptote. There is no 1. The numerator and the denominator have no com-
oblique asymptote. mon factors. −1 is the zero of the denominator, so
3. The zero of the numerator is 1, so the x-intercept is the domain is (−∞, −1)∪(−1, ∞). The line x = −1
(1, 0). is the vertical asymptote.
 
0−1 1 1 2. Because the degree of the numerator is one greater
4. f (0) = = − , so 0, − is the y- than the degree of the denominator, there is an
0+2 2 2
intercept. oblique asymptote. Using division, we find that
x2 − 9 −8
5. Find other function values to determine the shape =x−1+ . As |x| becomes very large,
x+1 x+1
of the graph and then draw it. the graph of f (x) gets close to the graph of y = x−1.
Thus, the line y = x − 1 is the oblique asymptote.
3. The zeros of the numerator are −3 and 3. Thus, the
x-intercepts are (−3, 0) and (3, 0).

Copyright © 2013 Pearson Education, Inc.


Exercise Set 4.5 157

4. f (0) = 0 − 9 = −9, so (0, −9) is the y-


2
y
0+1 (1, 5)
intercept. 4

5. Find other function values to determine the shape 2


of the graph and then draw it. 4 2 2 4 x
2
y 3x 2  x  2
yx1 4 g(x) 
4
x1
x  1
2
x−1 x−1
65. f (x) = =
4 2 2 4 x x2 − 2x − 3 (x + 1)(x − 3)
1. The numerator and the denominator have no com-
mon factors. The zeros of the denominator are −1
x2  9 and 3. Thus, the domain is (−∞, −1) ∪ (−1, 3) ∪
f (x) 
x1 (3, ∞) and the lines x = −1 and x = 3 are the
vertical asymptotes.
x2 + x − 2 (x + 2)(x − 1)
61. f (x) = = 2. Because the degree of the numerator is less than
2x2 + 1 2x2 + 1 the degree of the denominator, the x-axis, or y = 0,
1. The numerator and the denominator have no com- is the horizontal asymptote. There is no oblique
mon factors. The denominator has no real-number asymptote.
zeros, so the domain is (−∞, ∞) and there is no
vertical asymptote. 3. 1 is the zero of the numerator, so (1, 0) is the x-
intercept.
2. The numerator and the denominator have the same  
0−1 1 1
degree, so the horizontal asymptote is determined 4. f (0) = = , so 0, is the
by the ratio of the leading coefficients, 1/2. Thus, 02 − 2 · 0 − 3 3 3
y-intercept.
y = 1/2 is the horizontal asymptote. There is no
oblique asymptote. 5. Find other function values to determine the shape
of the graph and then draw it.
3. The zeros of the numerator are −2 and 1. Thus, the
x-intercepts are (−2, 0) and (1, 0). y
4. f (0) = 0 + 0 − 2 = −2, so (0, −2) is the
2
4
2 · 02 + 1
y-intercept. y⫽0
x ⫽ ⫺1 2
5. Find other function values to determine the shape
⫺4 4 x
of the graph and then draw it.
x⫽3
y

4 x⫺1
f(x) ⫽
2 x 2 ⫺ 2x ⫺ 3
yq
4 2 2 4 x 3x2 + 11x − 4 (3x − 1)(x + 4)
67. f (x) = =
x2 + 2x − 8 (x + 4)(x − 2)
4 x2  x  2 1. The domain of the function is (−∞, −4) ∪ (−4, 2) ∪
f (x) 
2x 2  1
(2, ∞). The numerator and the denominator have
3x2 − x − 2 (3x + 2)(x − 1) the common factor x + 4. The zeros of the denomi-
63. g(x) = = = 3x + 2, x = 1 nator are −4 and 2, and the zeros of the numerator
x−1 x−1 1
The graph is the same as the graph of g(x) = 3x + 2 are and −4. Since 2 is the only zero of the de-
3
except at x = 1, where there is a hole. Thus the domain nominator that is not a zero of the numerator, the
is (−∞, 1) ∪ (1, ∞). only vertical asymptote is x = 2.
2 2. The numerator and the denominator have the same
The zero of g(x) = 3x + 2 is − , so the x-intercept is
  3 degree, so the horizontal asymptote is determined
2
− , 0 ; g(0) = 2, so the y-intercept is (0, 2). by the ratio of the leading coefficients, 3/1, or 3.
3 Thus, y = 3 is the horizontal asymptote. There is
no oblique asymptote.
3. The only zero of the numerator that is in the
 domain

1 1
of the function is , so the x-intercept is ,0 .
3 3

Copyright © 2013 Pearson Education, Inc.


158 Chapter 4: Polynomial and Rational Functions

4. f (0) = 3 · 0 + 11 · 0 − 4 = −4 = 1 , so the
2
y
0 +2·0−8
2 −8 2
  4
1
y-intercept is 0, . 2
2
5. Find other function values to determine the shape 4 2 2 4 x
of the graph and then draw it. x0
x3  1
y f(x) 
x

8 x3 + 2x2 − 15x x(x + 5)(x − 3)


y 3 73. f (x) = =
4 x2 − 5x − 14 (x + 2)(x − 7)
冢4, 6 冣
13
8 4 4 8 1. The numerator and the denominator have no com-
x
4 mon factors. The zeros of the denominator are −2
3x2  11x  4
f (x )  x 2 and 7. Thus, the domain is (−∞, −2) ∪ (−2, 7) ∪
x 2  2x  8 8
(7, ∞) and the lines x = −2 and x = 7 are the
vertical asymptotes.
x−3
69. f (x) = 2. Because the degree of the numerator is one greater
(x + 1)3
than the degree of the denominator, there is an
1. The numerator and the denominator have no com- oblique asymptote. Using division, we find that
mon factors. −1 is the zero of the denominator, so x3 + 2x2 − 15x 34x + 98
the domain excludes −1. It is (−∞, −1) ∪ (−1, ∞). =x+7+ 2 . As |x| be-
x2 − 5x − 14 x − 5x − 14
The line x = −1 is the vertical asymptote. comes very large, the graph of f (x) gets close to the
2. Because the degree of the numerator is less than graph of y = x + 7. Thus, the line y = x + 7 is the
the degree of the denominator, the x-axis, or y = 0, oblique asymptote.
is the horizontal asymptote. There is no oblique 3. The zeros of the numerator are 0, −5, and 3. Thus,
asymptote. the x-intercepts are (−5, 0), (0, 0), and (3, 0).
3. 3 is the zero of the numerator, so (3, 0) is the x- 4. From part (3) we see that (0, 0) is the y-intercept.
intercept. 5. Find other function values to determine the shape
0−3 of the graph and then draw it.
4. f (0) = = −3, so (0, −3) is the y-
(0 + 1)3
intercept. y
50
5. Find other function values to determine the shape x7
x  2
of the graph and then draw it.

y
yx7
4

2
20 20 x
4 2 2 4 x
y0
x3 x3  2x2  15x
x  1 f (x)  f (x) 
(x  1)3 x2  5x  14

x3 + 1 5x4
71. f (x) = 75. f (x) =
x x4+1
1. The numerator and the denominator have no com- 1. The numerator and the denominator have no com-
mon factors. 0 is the zero of the denominator, so mon factors. The denominator has no real-number
the domain excludes 0. It is (−∞, 0) ∪ (0, ∞). The zeros, so the domain is (−∞, ∞) and there is no
line x = 0, or the y-axis, is the vertical asymptote. vertical asymptote.
2. Because the degree of the numerator is more than 2. The numerator and denominator have the same de-
one greater than the degree of the denominator, gree, so the horizontal asymptote is determined by
there is no horizontal or oblique asymptote. the ratio of the leading coefficients, 5/1, or 5. Thus,
y = 5 is the horizontal asymptote. There is no
3. The real-number zero of the numerator is −1, so the oblique asymptote.
x-intercept is (−1, 0).
3. The zero of the numerator is 0, so (0, 0) is the x-
4. Since 0 is not in the domain of the function, there intercept.
is no y-intercept.
4. From part (3) we see that (0, 0) is the y-intercept.
5. Find other function values to determine the shape
of the graph and then draw it.

Copyright © 2013 Pearson Education, Inc.


Exercise Set 4.5 159

5. Find other function values to determine the shape 83. a) The horizontal asymptote of N (t) is the ratio of the
of the graph and then draw it. leading coefficients of the numerator and denomina-
tor, 0.8/5, or 0.16. Thus, N (t) → 0.16 as t → ∞.
y
y5 b) The medication never completely disappears from
4 the body; a trace amount remains.
2
85. a) P (t)
4 2 2 4 x 60
5x 4 50
P(t) 
500t
f (x)  4 2t 2  9
x 1 40
30
x2 x2 20
77. f (x) = = 10
x2 − x − 2 (x + 1)(x − 2)
1. The numerator and the denominator have no com- 0 5 10 15 20 25 30 t
mon factors. The zeros of the denominator are −1 b) P (0) = 0; P (1) = 45.455 thousand, or 45, 455;
and 2. Thus, the domain is (−∞, −1) ∪ (−1, 2) ∪
(2, ∞) and the lines x = −1 and x = 2 are the P (3) = 55.556 thousand, or 55, 556;
vertical asymptotes. P (8) = 29.197 thousand, or 29, 197
2. The numerator and denominator have the same de- c) The degree of the numerator is less than the degree
gree, so the horizontal asymptote is determined by of the denominator, so the x-axis is the horizontal
the ratio of the leading coefficients, 1/1, or 1. Thus, asymptote. Thus, P (t) → 0 as t → ∞.
y = 1 is the horizontal asymptote. There is no d) Eventually, no one will live in this community.
oblique asymptote.
e) If we graph the function on a graphing calculator
3. The zero of the numerator is 0, so the x-intercept is and use the Maximum feature, we see that the max-
(0, 0). imum population is 58.926 thousand, or 58,926, and
4. From part (3) we see that (0, 0) is the y-intercept. it occurs when t ≈ 2.12 months.
5. Find other function values to determine the shape 87. domain, range, domain, range
of the graph and then draw it.
89. slope-intercept equation
y
91. x-intercept
4
y1 93. vertical lines
2

4 2 4
95. y-intercept
x
x5 + 2x3 + 4x2 −8
x  1 x2 97. f (x) = = x3 + 4 + 2
x2 + 2 x +2
−8
As |x| → ∞, 2 → 0 and the value of f (x) → x3 + 4.
x2 x +2
f (x) 
x2  x  2 Thus, the nonlinear asymptote is y = x3 + 4.

99. x  3 y x3
79. Answers may vary. The numbers −4 and 5 must be zeros of
the denominator. A function that satisfies these conditions 4
is
1 1 y2
f (x) = , or f (x) = 2 .
(x + 4)(x − 5) x − x − 20 4 4 x

81. Answers may vary. The numbers −4 and 5 must be zeros of


the denominator and −2 must be a zero of the numerator.
In addition, the numerator and denominator must have x  1
the same degree and the ratio of their leading coefficients 2x 3  x 2  8x  4
must be 3/2. A function that satisfies these conditions is f(x) 
x 3  x 2  9x  9
3x(x + 2) 3x2 + 6x
f (x) = , or f (x) = 2 .
2(x + 4)(x − 5) 2x − 2x − 40
Another function that satisfies these conditions is
3(x + 2)2 3x2 + 12x + 12
g(x) = , or g(x) = .
2(x + 4)(x − 5) 2x2 − 2x − 40

Copyright © 2013 Pearson Education, Inc.


160 Chapter 4: Polynomial and Rational Functions

7x
15. Solve < 0.
Exercise Set 4.6 (x − 1)(x + 5)
From our work in Exercise 12 we see that the solution set
is (−∞, −5) ∪ (0, 1).
1. x2 + 2x − 15 = 0
(x + 5)(x − 3) = 0 17. Solve x5 − 9x3 < 0.
x+5 = 0 or x − 3 = 0 From Exercise 16 we know the solutions of the related
equation are −3, 0, and 3. These numbers divide the x-axis
x = −5 or x=3
into the intervals (−∞, −3), (−3, 0), (0, 3), and (3, ∞). We
The solution set is {−5, 3}. test a value in each interval.
3. Solve x2 + 2x − 15 ≤ 0. (−∞, −3): g(−4) = −448 < 0
2 (−3, 0): g(−1) = 8 > 0
From Exercise 2 we know the solution set of x +2x−15 < 0
is (−5, 3). The solution set of x2 +2x−15 ≤ 0 includes the (0, 3): g(1) = −8 < 0
endpoints of this interval. Thus the solution set is [−5, 3].
(3, ∞): g(4) = 448 > 0
5. Solve x2 + 2x − 15 ≥ 0. Function values are negative on (−∞, −3) and on (0, 3).
From Exercise 4 we know the solution set of x2 +2x−15 > 0 The solution set is (−∞, −3) ∪ (0, 3).
is (−∞, −5) ∪ (3, ∞). The solution set of x2 + 2x − 15 ≥ 0
19. Solve x5 − 9x3 > 0.
includes the endpoints −5 and 3. Thus the solution set is
(−∞, −5] ∪ [3, ∞). From our work in Exercise 17 we see that the solution set
is (−3, 0) ∪ (3, ∞).
x−2
7. Solve > 0. 21. First we find an equivalent inequality with 0 on one side.
x+4
x3 + 6x2 < x + 30
The denominator tells us that g(x) is not defined when
x = −4. From Exercise 6 we know that g(2) = 0. The x + 6x − x − 30 < 0
3 2

critical values of −4 and 2 divide the x-axis into three From the graph we see that the x-intercepts of the related
intervals (−∞, −4), (−4, 2), and (2, ∞). We test a value function occur at x = −5, x = −3, and x = 2. They divide
in each interval. the x-axis into the intervals (−∞, −5), (−5, −3), (−3, 2),
(−∞, −4): g(−5) = 7 > 0 and (2, ∞). From the graph we see that the function has
1 negative values only on (−∞, −5) and (−3, 2). Thus, the
(−4, 2): g(0) = − < 0 solution set is (−∞, −5) ∪ (−3, 2).
2
1 23. By observing the graph or the denominator of the function,
(2, ∞): g(3) = > 0
7 we see that the function is not defined for x = −2 or x = 2.
Function values are positive on (−∞, −4) and on (2, ∞). We also see that 0 is a zero of the function. These num-
The solution set is (−∞, −4) ∪ (2, ∞). bers divide the x-axis into the intervals (−∞, −2), (−2, 0),
(0, 2), and (2, ∞). From the graph we see that the function
x−2
9. Solve ≥ 0. has positive values only on (−2, 0) and (2, ∞). Since the
x+4
inequality symbol is ≥, 0 must be included in the solution
x−2
From Exercise 7 we know that the solution set of >0 set. It is (−2, 0] ∪ (2, ∞).
x+4
is (−∞, −4) ∪ (2, ∞). We include the zero of the function, 25. (x − 1)(x + 4) < 0
2, since the inequality symbol is ≥. The critical value
The related equation is (x − 1)(x + 4) = 0. Using the
−4 is not included because it is not in the domain of the
principle of zero products, we find that the solutions of
function. The solution set is (−∞, −4) ∪ [2, ∞).
the related equation are 1 and −4. These numbers divide
7x the x-axis into the intervals (−∞, −4), (−4, 1), and (1, ∞).
11. =0 We let f (x) = (x − 1)(x + 4) and test a value in each
(x − 1)(x + 5)
7x = 0 Multiplying by (x − 1)(x + 5) interval.
x=0 (−∞, −4): f (−5) = 6 > 0
The solution set is {0}. (−4, 1): f (0) = −4 < 0
(1, ∞): f (2) = 6 > 0
7x
13. Solve ≥ 0. Function values are negative only in the interval (−4, 1).
(x − 1)(x + 5)
The solution set is (−4, 1).
From our work in Exercise 12 we see that function values
are positive on (−5, 0) and on (1, ∞). We also include 27. x2 + x − 2 > 0 Polynomial inequality
the zero of the function, 0, in the solution set because the
x2 + x − 2 = 0 Related equation
inequality symbol is ≥. The critical values −5 and 1 are
not included because they are not in the domain of the (x + 2)(x − 1) = 0 Factoring
function. The solution set is (−5, 0] ∪ (1, ∞).

Copyright © 2013 Pearson Education, Inc.


Exercise Set 4.6 161

Using the principle of zero products, we find that the solu- (2, ∞): f (3) = −5 < 0
tions of the related equation are −2 and 1. These numbers Function values are negative on (−∞, −2) and
divide the x-axis into the intervals (−∞, −2), (−2, 1), and (2, ∞). Since the inequality symbol is ≤, the endpoints
(1, ∞). We let f (x) = x2 + x − 2 and test a value in each of the intervals must be included in the solution set. It is
interval. (−∞, −2] ∪ [2, ∞).
(−∞, −2): f (−3) = 4 > 0
35. 6x − 9 − x2 < 0 Polynomial inequality
(−2, 1): f (0) = −2 < 0
6x − 9 − x2 = 0 Related equation
(1, ∞): f (2) = 4 > 0
−(x − 6x + 9) = 0
2
Factoring out −1 and
Function values are positive on (−∞, −2) and (1, ∞). The rearranging
solution set is (−∞, −2) ∪ (1, ∞). −(x − 3)(x − 3) = 0 Factoring
29. x2 − x − 5 ≥ x − 2 Using the principle of zero products, we find that the so-
lution of the related equation is 3. This number divides
x2 − 2x − 3 ≥ 0 Polynomial inequality the x-axis into the intervals (−∞, 3) and (3, ∞). We let
x2 − 2x − 3 = 0 Related equation f (x) = 6x − 9 − x2 and test a value in each interval.
(x + 1)(x − 3) = 0 Factoring (−∞, 3): f (−4) = −49 < 0
Using the principle of zero products, we find that the solu- (3, ∞): f (4) = −1 < 0
tions of the related equation are −1 and 3. The numbers Function values are negative on both intervals. The solu-
divide the x-axis into the intervals (−∞, −1), (−1, 3), and tion set is (−∞, 3) ∪ (3, ∞).
(3, ∞). We let f (x) = x2 − 2x − 3 and test a value in each
interval. 37. x2 + 12 < 4x Polynomial inequality
(−∞, −1): f (−2) = 5 > 0 x − 4x + 12 < 0
2
Equivalent inequality with 0
on one side
(−1, 3): f (0) = −3 < 0
x2 − 4x + 12 = 0 Related equation
(3, ∞): f (4) = 5 > 0 Using the quadratic formula, we find that the related
Function values are positive on (−∞, −1) and on (3, ∞). equation has no real-number solutions. The graph lies
Since the inequality symbol is ≥, the endpoints of the entirely above the x-axis, so the inequality has no so-
intervals must be included in the solution set. It is lution. We could determine this algebraically by letting
(−∞, −1] ∪ [3, ∞). f (x) = x2 − 4x + 12 and testing any real number (since
there are no real-number solutions of f (x) = 0 to divide
31. x2 > 25 Polynomial inequality the x-axis into intervals). For example, f (0) = 12 > 0,
x − 25 > 0
2
Equivalent inequality with so we see algebraically that the inequality has no solution.
0 on one side The solution set is ∅.
x2 − 25 = 0 Related equation
39. 4x3 − 7x2 ≤ 15x Polynomial
(x + 5)(x − 5) = 0 Factoring inequality
Using the principle of zero products, we find that the solu- 4x3 − 7x2 − 15x ≤ 0 Equivalent
tions of the related equation are −5 and 5. These numbers inequality with 0
divide the x-axis into the intervals (−∞, −5), (−5, 5), and on one side
(5, ∞). We let f (x) = x2 − 25 and test a value in each 4x3 − 7x2 − 15x = 0 Related equation
interval.
x(4x + 5)(x − 3) = 0 Factoring
(−∞, −5): f (−6) = 11 > 0
Using the principle of zero products, we find that the so-
(−5, 5): f (0) = −25 < 0 5
lutions of the related equation are 0, − , and 3. These
(5, ∞): f (6) = 11 > 0 4  
5
Function values are positive on (−∞, −5) and (5, ∞). The numbers divide the x-axis into the intervals − ∞, − ,
  4
solution set is (−∞, −5) ∪ (5, ∞). 5
− , 0 , (0, 3), and (3, ∞). We let f (x) = 4x − 7x2 −
3
33. 4 − x2 ≤ 0 Polynomial inequality 4
15x
 and test  a value in each interval.
4−x = 0
2
Related equation 5
− ∞, − : f (−2) = −30 < 0
(2 + x)(2 − x) = 0 Factoring 4
 
Using the principle of zero products, we find that the solu- 5
tions of the related equation are −2 and 2. These numbers − , 0 : f (−1) = 4 > 0
4
divide the x-axis into the intervals (−∞, −2), (−2, 2), and (0, 3) f (1) = −18 < 0
(2, ∞). We let f (x) = 4 − x2 and test a value in each
interval. (3, ∞): f (4) = 84 > 0
 
5
(−∞, −2): f (−3) = −5 < 0 Function values are negative on − ∞, − and (0, 3).
4
(−2, 2): f (0) = 4 > 0 Since the inequality symbol is ≤, the endpoints of the

Copyright © 2013 Pearson Education, Inc.


162 Chapter 4: Polynomial and Rational Functions


 
intervals must be included in the solution set. It is (−∞, − 2): f (−2) = −14 < 0
5 √
− ∞, − ∪ [0, 3]. (− 2, −1): f (−1.3) ≈ 0.37107 > 0
4 √
(−1, 2): f (0) = −2 < 0
41. x3 + 3x2 − x − 3 ≥ 0 Polynomial √
( 2, ∞): f (2) = 18 > 0
inequality √
x3 + 3x2 − x − 3 = 0 Related equation Function
√ values are positive on (− 2, −1) and
( 2, ∞). Since the inequality symbol is ≥, the endpoints
x (x + 3) − (x + 3) = 0 Factoring
2
of √
the intervals
√ must be included in the solution set. It is
(x2 − 1)(x + 3) = 0 [− 2, −1] ∪ [ 2, ∞).
(x + 1)(x − 1)(x + 3) = 0 47. 2x3 + 6 ≤ 5x2 + x Polynomial
Using the principle of zero products, we find that the solu- inequality
tions of the related equation are −1, 1, and −3. These
2x3 − 5x2 − x + 6 ≤ 0 Equivalent
numbers divide the x-axis into the intervals (−∞, −3),
inequality with 0
(−3, −1), (−1, 1), and (1, ∞). We let f (x) = x3 +3x2 −x−3
on one side
and test a value in each interval.
2x3 − 5x2 − x + 6 = 0 Related equation
(−∞, −3): f (−4) = −15 < 0 Using the techniques of Section 3.3, we find that the solu-
(−3, −1): f (−2) = 3 > 0 3
tions of the related equation are −1, , and 2. We can also
(−1, 1): f (0) = −3 < 0 2
use the graph of y = 2x3 − 5x2 − x + 6 to find these solu-
(1, ∞): f (2) = 15 > 0 tions.
 They
 divide  the x-axis into the intervals (−∞, −1),
Function values are positive on (−3, −1) and (1, ∞). Since 3 3
− 1, , , 2 , and (2, ∞). Let f (x) = 2x3 −5x2 −x+6
the inequality symbol is ≥, the endpoints of the intervals 2 2
must be included in the solution set. It is [−3, −1]∪[1, ∞). and test a value in each interval.
(−∞, −1): f (−2) = −28 < 0
43. x3 − 2x2 < 5x − 6 Polynomial  
3
inequality − 1, : f (0) = 6 > 0
2
x3 − 2x2 − 5x + 6 < 0 Equivalent  
inequality with 0 3
, 2 : f (1.6) = −0.208 < 0
on one side 2
x3 − 2x2 − 5x + 6 = 0 Related equation (2, ∞): f (3) = 12 > 0
 values are negative in (−∞, −1) and
Using the techniques of Section 3.3, we find that the solu- Function

tions of the related equation are −2, 1, and 3. They divide 3
the x-axis into the intervals (−∞, −2), (−2, 1), (1, 3), and , 2 . Since the inequality symbol is ≤, the endpoints
2
(3, ∞). Let f (x) = x3 − 2x2 − 5x + 6 and test a value in   in the solution set. The
of the intervals must be included
each interval. 3
solution set is (−∞, −1] ∪ , 2 .
(−∞, −2): f (−3) = −24 < 0 2

(−2, 1): f (0) = 6 > 0 49. x3 + 5x2 − 25x ≤ 125 Polynomial


inequality
(1, 3): f (2) = −4 < 0
x3 + 5x2 − 25x − 125 ≤ 0 Equivalent
(3, ∞): f (4) = 18 > 0
inequality with 0
Function values are negative on (−∞, −2) and (1, 3). The on one side
solution set is (−∞, −2) ∪ (1, 3).
x3 + 5x2 − 25x − 125 = 0 Related equation
45. x + x ≥ 2x + 2
5 2 3
Polynomial x2 (x + 5) − 25(x + 5) = 0 Factoring
inequality
(x − 25)(x + 5) = 0
2
x5 − 2x3 + x2 − 2 ≥ 0 Related
(x + 5)(x − 5)(x + 5) = 0
inequality with
0 on one side Using the principle of zero products, we find that the solu-
x5 − 2x3 + x2 − 2 = 0 Related tions of the related equation are −5 and 5. These numbers
equation divide the x-axis into the intervals (−∞, −5), (−5, 5), and
(5, ∞). We let f (x) = x3 + 5x2 − 25x − 125 and test a
x3 (x2 − 2) + x2 − 2 = 0 Factoring
value in each interval.
(x + 1)(x − 2) = 0
3 2
(−∞, −5): f (−6) = −11 < 0
Using the principle of zero products, we find that the
√ real- (−5, 5): f (0) = −125 < 0
number
√ solutions of the related equation are −1, − 2, and
(5, ∞): f (6) = 121 > 0
2. These
√ numbers
√ √ x-axis into
divide the √ the intervals
(−∞, − 2), (− 2, −1), (−1, 2), and ( 2, ∞). We let Function values are negative on (−∞, −5) and
f (x) = x5 − 2x3 + x2 − 2 and test a value in each interval. (−5, 5). Since the inequality symbol is ≤, the endpoints

Copyright © 2013 Pearson Education, Inc.


Exercise Set 4.6 163

 
of the intervals must be included in the solution set. It is 5
Function values are negative on − , ∞ . The solution
(−∞, −5] ∪ [−5, 5] or (−∞, 5].   2
5
51. 0.1x3 − 0.6x2 − 0.1x + 2 < 0 Polynomial set is − , ∞ .
2
inequality
2x
0.1x3 − 0.6x2 − 0.1x + 2 = 0 Related equation 57. ≥0 Rational inequality
x−4
After trying all the possibilities, we find that the re- 2x
lated equation has no rational zeros. Using the graph of =0 Related equation
x−4
y = 0.1x3 − 0.6x2 − 0.1x + 2, we find that the only real-
2x
number solutions of the related equation are approximately The denominator of f (x) = is 0 when x = 4, so the
−1.680, 2.154, and 5.526. These numbers divide the x−4
function is not defined for x = 4.
x-axis into the intervals (−∞, −1.680), (−1.680, 2.154),
(2.154, 5.526), and (5.526, ∞). We let f (x) = 0.1x3 − We solve the related equation f (x) = 0.
0.6x2 − 0.1x + 2 and test a value in each interval. 2x
=0
(−∞, −1.680): f (−2) = −1 < 0 x−4
2x = 0 Multiplying by x − 4
(−1.680, 2.154): f (0) = 2 > 0
x=0
(2.154, 5.526): f (3) = −1 < 0
The critical values are 0 and 4. They divide the x-axis into
(5.526, ∞): f (6) = 1.4 > 0 the intervals (−∞, 0), (0, 4), and (4, ∞). We test a value
Function values are negative on (−∞, −1.680) and in each interval.
(2.154, 5.526). The graph can also be used to determine 2
(−∞, 0): f (−1) = > 0
this. The solution set is (−∞, −1.680) ∪ (2.154, 5.526). 5
2
1 (0, 4): f (1) = − < 0
53. >0 Rational inequality 3
x+4 (4, ∞): f (5) = 10 > 0
1
=0 Related equation Function values are positive on (−∞, 0) and (4, ∞). Since
x+4
the inequality symbol is ≥ and f (0) = 0, then 0 must be
1
The denominator of f (x) = is 0 when x = −4, so the included in the solution set. And since 4 is not in the
x+4 domain of f (x), 4 is not included in the solution set. It is
function is not defined for x = −4. The related equation
(−∞, 0] ∪ (4, ∞).
has no solution. Thus, the only critical value is −4. It
divides the x-axis into the intervals (−∞, −4) and (−4, ∞). x−4 x+2
59. − ≤0
We test a value in each interval. x+3 x−1
(−∞, −4): f (−5) = −1 < 0 x−4 x+2
The denominator of f (x) = − is 0 when x =
1 x+3 x−1
(−4, ∞): f (0) = > 0 −3 or x = 1, so the function is not defined for these values
4
of x. We solve the related equation f (x) = 0.
Function values are positive on (−4, ∞). This can also be
1 x−4 x+2
determined from the graph of y = . The solution set − =0
x+4 x+3 x−1
is (−4, ∞).  
x−4 x+2
(x+3)(x−1) − = (x+3)(x−1)·0
−4 x+3 x−1
55. <0 Rational inequality (x − 1)(x − 4) − (x + 3)(x + 2) = 0
2x + 5
−4 x2 − 5x + 4 − (x2 + 5x + 6) = 0
=0 Related equation
2x + 5 −10x − 2 = 0
−4 5 −10x = 2
The denominator of f (x) = is 0 when x = − ,
2x + 5 2 1
5 x=−
so the function is not defined for x = − . The related 5
2
equation has no solution. Thus, the only critical
 value 
is 1
The critical values are −3, − , and 1. They divide the x-
5
− . It divides the x-axis into the intervals − ∞, −
5 5    
2  2 1 1
 axis into the intervals (−∞, −3), − 3, − , − ,1 ,
5 5 5
and − , ∞ . We test a value in each interval. and (1, ∞). We test a value in each interval.
2
  (−∞, −3): f (−4) = 7.6 > 0
5  
− ∞, − : f (−3) = 4 > 0
2 1
  − 3, − : f (−1) = −2 < 0
5 4 5
− , ∞ : f (0) = − < 0  
2 5 1 2
− , 1 : f (0) = > 0
5 3

Copyright © 2013 Pearson Education, Inc.


164 Chapter 4: Polynomial and Rational Functions

(1, ∞): f (2) = −4.4 < 0 x+1


  −3 = 0
x−2
Function values are negative on
1
− 3, − and (1, ∞).  
5 x+1
(x − 2) − 3 = (x − 2) · 0
Note
 that  since the inequality symbol is ≤ and x−2
f −
1 1
= 0, then − must be included in the solution x + 1 − 3(x − 2) = 0
5 5 x + 1 − 3x + 6 = 0
set. Note also that since neither −3 nor 1 is in the domain
−2x + 7 = 0
 f (x), they
of  are not included in the solution set. It is
1
− 3, − ∪ (1, ∞). −2x = −7
5 7
x+6 x−8 x=
61. > Rational inequality 2
x−2 x−5 7
The critical values are 2 and . They divide the x-axis
x+6 x−8  2  
− >0 Equivalent inequality 7 7
x−2 x−5 with 0 on one side into the intervals (−∞, 2), 2, , and , ∞ . We test
2 2
x+6 x−8 a value in each interval.
The denominator of f (x) = − is 0 when x = 2
x−2 x−5 (−∞, 2): f (0) = −3.5 < 0
or x = 5, so the function is not defined for these values of  
7
x. We solve the related equation f (x) = 0. 2, : f (3) = 1 > 0
2
x+6 x−8  
− =0 7
x−2 x−5 , ∞ : f (4) = −0.5 < 0
  2
x+6 x−8  
(x − 2)(x − 5) − = (x − 2)(x − 5) · 0 7
x−2 x−5 Function values are positive on 2, . Note that since
(x − 5)(x + 6) − (x − 2)(x − 8) = 0  2
7 7
x2 + x − 30 − (x2 − 10x + 16) = 0 the inequality symbol is ≥ and f = 0, then must
2 2
x2 + x − 30 − x2 + 10x − 16 = 0 be included in the solution set. Note also that since 2 is
11x − 46 = 0  of f (x), it is not included in the solution
not in thedomain
7
11x = 46 set. It is 2, .
2
46
x=
11 1
65. x−2 > Rational inequality
46 x
The critical values are 2, , and 5. They divide the x-axis
11    1
46 46 x−2− >0 Equivalent inequality
into the intervals (−∞, 2), 2, , , 5 , and (5, ∞). x with 0 on one side
11 11
We test a value in each interval. 1
The denominator of f (x) = x − 2 − is 0 when x = 0, so
(−∞, 2): f (0) = −4.6 < 0 x
  the function is not defined for this value of x. We solve
46 the related equation f (x) = 0.
2, : f (4) = 1 > 0
11 1
  x−2− = 0
46 x
, 5 : f (4.5) = −2.8 < 0  
11 1
x x−2− = x·0
(5, ∞): f (6) = 5 > 0 x
 
46 1
Function values are positive on 2, and (5, ∞). The x2 − 2x − x · = 0
  11 x
solution set is 2,
46
∪ (5, ∞). x2 − 2x − 1 = 0
11 √
√ we find that√x = 1 ± 2. The
Using the quadratic formula
x+1 critical values are 1 − 2, 0, and 1 + √2. They √ divide
63. ≥ 3 Rational inequality
x−2 the x-axis
√ into the intervals
√ (−∞, 1 − 2), (1 − 2, 0),
x+1 (0, 1 + 2), and (1 + 2, ∞). We test a value in each
− 3 ≥ 0 Equivalent inequality interval.
x−2 with 0 on one side √
(−∞, 1 − 2): f (−1) = −2 < 0
x+1 √
The denominator of f (x) = − 3 is 0 when x = 2, so (1 − 2, 0): f (−0.1) = 7.9 > 0
x−2 √
the function is not defined for this value of x. We solve (0, 1 + 2): f (1) = −2 < 0
the related equation f (x) = 0. √ 2
(1 + 2, ∞): f (3) = > 0
3

Copyright © 2013 Pearson Education, Inc.


Exercise Set 4.6 165

√ 3 6
√ values are positive on (1 −
Function 2, 0) and − =0
(1 + √2, ∞). The √
solution set is x2 + 1 5x2 + 2
 
(1 − 2, 0) ∪ (1 + 2, ∞). 3 6
(x2 + 1)(5x2 + 2) 2 − =
2 5 x + 1 5x2 + 2
67. ≤ 2 (x2 + 1)(5x2 + 2) · 0
x2 − 4x + 3 x −9
2 5 3(5x2 + 2) − 6(x2 + 1) = 0
− ≤0
x2 − 4x + 3 x2 − 9 15x2 + 6 − 6x2 − 6 = 0
2 5 9x2 = 0
− ≤0
(x − 1)(x − 3) (x + 3)(x − 3) x2 = 0
The denominator of f (x) = x=0
2 5
− is 0 when x = 1, 3, or −3, The only critical value is 0. It divides the x-axis into the
(x − 1)(x − 3) (x + 3)(x − 3)
so the function is not defined for these values of x. We intervals (−∞, 0) and (0, ∞). We test a value in each in-
solve the related equation f (x) = 0. terval.
2 5 (−∞, 0): f (−1) ≈ 0.64286 > 0
− =0
(x − 1)(x − 3) (x + 3)(x − 3) (0, ∞): f (1) ≈ 0.64286 > 0
 
2 5 Function values are positive on both intervals. Note that
(x−1)(x−3)(x+3) −
(x−1)(x−3) (x+3)(x−3) since the inequality symbol is ≥ and f (0) = 0, then 0 must
= (x − 1)(x − 3)(x + 3) · 0 be included in the solution set. It is (−∞, 0] ∪ [0, ∞), or
(−∞, ∞).
2(x + 3) − 5(x − 1) = 0
2x + 6 − 5x + 5 = 0 71.
5
<
3
−3x + 11 = 0 x2
+ 3x 2x + 1
5 3
−3x = −11 − <0
x2 + 3x 2x + 1
11
x= 5 3
3 − <0
11 x(x + 3) 2x + 1
The critical values are −3, 1, 3, and . They divide the x- 5 3
3   The denominator of f (x) = − is 0 when
11 x(x + 3) 2x + 1
axis into the intervals (−∞, −3), (−3, 1), (1, 3), 3, ,
3 1
  x = 0, −3, or − , so the function is not defined for these
11 2
and , ∞ . We test a value in each interval. values of x. We solve the related equation f (x) = 0.
3
5 3
(−∞, −3): f (−4) ≈ −0.6571 < 0 − =0
x(x + 3) 2x + 1
(−3, 1): f (0) ≈ 1.2222 > 0  
5 3
(1, 3): f (2) = −1 < 0 x(x+3)(2x+1) − =
  x(x+3) 2x+1
11
3, : f (3.5) ≈ 0.6154 > 0 x(x+3)(2x+1)·0
3
  5(2x + 1) − 3x(x + 3) = 0
11
, ∞ : f (4) ≈ −0.0476 < 0 10x + 5 − 3x2 − 9x = 0
3
−3x2 + x + 5 = 0
Function
  values are negative on (−∞, −3), (1, 3), and
11 Using the√quadratic formula we find that √
, ∞ . Note that since the inequality symbol is ≤ and 1 ± 61 1 − 61 1
3  x= . The critical values are −3, , − ,
11 11 6 √ 6 2
f = 0, then must be included in the solution set. 1 + 61
3 3 0, and . They divide the x-axis into
Note also that since −3, 1, and 3 are not in the domain
6  √ 
1 − 61
of f (x), they are not  included
 in the solution set. It is the intervals (−∞, −3), − 3, ,
√ 6 √
11      
(−∞, −3) ∪ (1, 3) ∪ ,∞ . 1 − 61 1 1 1 + 61
3 , − , − , 0 , 0, , and
6√ 2 2 6
3 6  
69. ≥ 1 + 61
x2 + 1 5x2 + 2 ,∞ .
6
3 6
− ≥0 We test a value in each interval.
x2 + 1 5x2 + 2
(−∞, −3): f (−4) ≈ 1.6786 > 0
The denominator of f (x) =
3

6
has no real-  √ 
x2 + 1 5x2 + 2 1 − 61
− 3, : f (−2) = −1.5 < 0
number zeros. We solve the related equation f (x) = 0. 6

Copyright © 2013 Pearson Education, Inc.


166 Chapter 4: Polynomial and Rational Functions

 √  x 3
1− 61 1 75. + ≤
,− : f (−1) = 0.5 > 0 x2 +4x−5 x2 −25
6 2
  2x
1
− , 0 : f (−0.1) ≈ −20.99 < 0 x2 −6x+5
2
 √  x 3 2x
1 + 61 + − ≤0
0, : f (1) = 0.25 > 0 x2 +4x−5 x2 −25 x2 −6x+5
6 x 3 2x
 √  + − ≤0
1 + 61 (x+5)(x−1) (x+5)(x−5) (x−5)(x−1)
, ∞ : f (2) = −0.1 < 0
6 The denominator of
 √ 
1 − 61 x 3 2x
Function values are negative on − 3, , f (x) = + −
6 (x+5)(x−1) (x+5)(x−5) (x−5)(x−1)
   √  is 0 when x = −5, 1, or 5, so the function is not defined for
1 1 + 61
− , 0 and , ∞ . The solution set is these values of x. We solve the related equation f (x) = 0.
2 6
 √     √  x
+
3

2x
=0
1 − 61 1 1 + 61 (x+5)(x−1) (x+5)(x−5) (x−5)(x−1)
− 3, ∪ − ,0 ∪ ,∞ .
6 2 6 x(x − 5) + 3(x − 1) − 2x(x + 5) = 0

73.
5x x Multiplying by (x + 5)(x − 1)(x − 5)
>
7x − 2 x+1 x2 − 5x + 3x − 3 − 2x2 − 10x = 0
5x x −x2 − 12x − 3 = 0
− >0
7x − 2 x + 1
x2 + 12x + 3 = 0
5x x √
The denominator of f (x) = − is 0
7x − 2 x + 1 Using the quadratic formula,√we find that x√= −6 ± 33.
2
when x = or x = −1, so the function is not defined for The critical values are −6 − 33, −5, −6 + 33, 1, and√ 5.
7 They divide
√ the x-axis into√the intervals√(−∞, −6 − 33),
these values of x. We solve the related equation f (x) = 0. (−6 − 33, −5), (−5, −6 + 33), (−6 + 33, 1), (1, 5), and
5x

x (5, ∞). We test a value in each interval.
7x − 2 x + 1
=0 √
  (−∞, −6 − 33): f (−12) ≈ 0.00194 > 0
5x x √
(7x−2)(x+1) − = (7x−2)(x+1) · 0 (−6 − 33, −5): f (−6) ≈ −0.4286 < 0
7x−2 x+1 √
5x(x + 1) − x(7x − 2) = 0 (−5, −6 + 33): f (−1) ≈ 0.16667 > 0

5x2 + 5x − 7x2 + 2x = 0 (−6 + 33, 1): f (0) = −0.12 < 0

−2x2 + 7x = 0 (1, 5): f (2) ≈ 1.4762 > 0

−x(2x − 7) = 0 (5, ∞): f (6) ≈ −2.018 < 0



7 √ values are negative on (−6 − 33, −5),
Function
x = 0 or x = (−6 + 33, 1), and (5, ∞).√Note that since the inequality

2
2 7 symbol√ is ≤ and f (−6 ± 33) = 0, then −6 − 33 and
The critical values are −1, 0, , and . They divide the x- −6 + 33 must be included in the solution set. Note also
7 2    
2 2 7 that since −5, 1, and 5 are not in the domain of√f (x), they
axis into the intervals (−∞, −1), (−1, 0), 0, , , , are not√included in the solution set. It is [−6 − 33, −5) ∪
  7 7 2
7 [−6 + 33, 1) ∪ (5, ∞).
and , ∞ . We test a value in each interval.
2
77. We write and solve a rational inequality.
(−∞, −1): f (−2) = −1.375 < 0
4t
(−1, 0): f (−0.5) ≈ 1.4545 > 0 2
+ 98.6 > 100
  t +1
2 4t
0, : f (0.1) ≈ −0.4755 < 0 − 1.4 > 0
7 t2 + 1
 
2 7 4t
, : f (1) = 0.5 > 0 The denominator of f (t) = 2 − 1.4 has no real-
7 2 t +1
  number zeros. We solve the related equation f (t) = 0.
7
, ∞ : f (4) ≈ −0.0308 < 0 4t
2 − 1.4 = 0
  t2 + 1
2 7
Function values are positive on (−1, 0) and ,
7 2
. The 4t − 1.4(t2 + 1) = 0 Multiplying by t2 + 1
 
2 7 4t − 1.4t2 − 1.4 = 0
solution set is (−1, 0) ∪ , .
7 2 Using the
√ quadratic formula, we find that
4 ± 8.16
t= ; that is, t ≈ 0.408 or t ≈ 2.449. These
2.8

Copyright © 2013 Pearson Education, Inc.


Exercise Set 4.6 167

numbers divide the t-axis into the intervals (−∞, 0.408), Function values are positive on (9, ∞). Since the inequality
(0.408, 2.449), and (2.449, ∞). We test a value in each symbol is ≥, 9 must also be included in the solution set
interval. for this portion of the inequality. It is {n|n ≥ 9}.
(−∞, 0.408): f (0) = −1.4 < 0
Now solve the second inequality.
(0.408, 2.449): f (1) = 0.6 > 0
n2 − 3n ≤ 460
(2.449, ∞): f (3) = −0.2 < 0 n2 − 3n − 460 ≤ 0
Function values are positive on (0.408, 2.449). The solution n2 − 3n − 460 = 0 Related equation
set is (0.408, 2.449).
(n + 20)(n − 23) = 0
79. a) We write and solve a polynomial inequality. n = −20 or n = 23
−3x2 + 630x − 6000 > 0 (x ≥ 0) We consider only positive values of n as above. Thus,
We first solve the related equation. we consider the intervals (0, 23) and (23, ∞). Let f (n) =
−3x2 + 630x − 6000 = 0 n2 − 3n − 460 and test a value in each interval.
x2 − 210x + 2000 = 0 Dividing by −3 (0, 23): f (1) = −462 < 0
(x − 10)(x − 200) = 0 Factoring (23, ∞): f (24) = 44 > 0
Using the principle of zero products or by observ- Function values are negative on (0, 23). Since the inequal-
ing the graph of y = −3x2 + 630 − 6000, we see ity symbol is ≤, 23 must also be included in the solution
that the solutions of the related equation are 10 and set for this portion of the inequality. It is {n|0 < n ≤ 23}.
200. These numbers divide the x-axis into the in-
tervals (−∞, 10), (10, 200), and (200, ∞). Since we The solution set of the original inequality is
are restricting our discussion to nonnegative values {n|n ≥ 9 and 0 < n ≤ 23}, or {n|9 ≤ n ≤ 23}.
of x, we consider the intervals [0, 10), (10, 200), and 83. (x − h)2 + (y − k)2 = r2
(200, ∞).
[x − (−2)]2 + (y − 4)2 = 32
We let f (x) = −3x2 + 630x − 6000 and test a value
in each interval. (x + 2)2 + (y − 4)2 = 9

[0, 10): f (0) = −6000 < 0 85. h(x) = −2x2 + 3x − 8


(10, 200): f (11) = 567 > 0 b 3 3
a) − =− =
(200, ∞): f (201) = −573 < 0 2a 2(−2) 4
   2
Function values are positive only on (10, 200). The 3 3 3 55
h = −2 +3· −8=−
solution set is {x|10 < x < 200}, or (10, 200). 4 4 4 8
 
b) From part (a), we see that function values are neg- 3 55
The vertex is ,− .
ative on [0, 10) and (200, ∞). Thus, the solution set 4 8
is {x|0 < x < 10 or x > 200}, or (0, 10) ∪ (200, ∞). b) The coefficient of x2 is negative, so there is a maxi-
mum value. It is the second coordinate of
81. We write an inequality.
55 3
n(n − 3) the vertex, − . It occurs at x = .
27 ≤ ≤ 230 8  4
2 55
54 ≤ n(n − 3) ≤ 460 Multiplying by 2 c) The range is − ∞, − .
8
54 ≤ n2 − 3n ≤ 460
87. |x2 − 5| = |5 − x2 | = 5 − x2 when 5 − x2 ≥ 0. Thus we
We write this as two inequalities. solve 5 − x2 ≥ 0.
54 ≤ n2 − 3n and n2 − 3n ≤ 460 5 − x2 ≥ 0
Solve each inequality. 5 − x2 = 0 Related equation
n2 − 3n ≥ 54 5 = x2
n2 − 3n − 54 ≥ 0 √
± 5=x
n2 − 3n − 54 = 0 Related equation Let f (x) = 5 − x2 and test a value in each of the intervals
(n + 6)(n − 9) = 0 determined by the solutions of the related equation.

n = −6 or n = 9 (−∞, − 5): f (−3) = −4 < 0
√ √
Since only positive values of n have meaning in this ap- (− 5, 5): f (0) = 5 > 0
plication, we consider the intervals (0, 9) and (9, ∞). Let √
( 5, ∞): f (3) = −4 < 0
f (n) = n2 − 3n − 54 and test a value in each interval. √ √
Function values are positive on (− 5, 5). Since the in-
(0, 9): f (1) = −56 < 0
equality symbol is ≥, the endpoints of the
√ interval
√  must
(9, ∞): f (10) = 16 > 0 be included in the solution set. It is − 5, 5 .

Copyright © 2013 Pearson Education, Inc.


168 Chapter 4: Polynomial and Rational Functions

89. 2|x|2 − |x| + 2 ≤ 5 1 1


Next solve 1 + < 3, or − 2 < 0. The denominator of
x x
2|x|2 − |x| − 3 ≤ 0 1
f (x) = − 2 is 0 when x = 0, so the function is not de-
2|x|2 − |x| − 3 = 0 Related equation x
fined for this value of x. Now solve the related equation.
(2|x| − 3)(|x| + 1) = 0 Factoring
1
2|x| − 3 = 0 or |x| + 1 = 0 −2 = 0
x
|x| =
3
or |x| = −1 1 − 2x = 0 Multiplying by x
2 1
3 3 x=
The solution of the first equation is x = − or x = . 2
2 2 1
The second equation has no solution. Let f (x) = 2|x|2 − The critical values are 0 and . Test a value in each of the
2
|x| − 3 and test a value in each interval determined by the intervals determined by them.
solutions of the related equation.
  (−∞, 0): f (−1) = −3 < 0
3  
− ∞, − : f (−2) = 3 > 0 1
2 0, : f (0.1) = 8 > 0
  2
3 3  
− , : f (0) = −3 < 0 1
2 2 , ∞ : f (1) = −1 < 0
  2
3
, ∞ : f (2) = 3 > 0 The solution setfor this portion of the inequality is
2 1
  (−∞, 0) ∪ ,∞ .
3 3 2
Function values are negative on − , . Since the in-
2 2 The solution set of the original inequality is
equality symbol is ≤, the endpoints of the  interval
 must  
 

3 3 1 1
also be included in the solution set. It is − , . − ∞, − ∪ (0, ∞) and (−∞, 0) ∪ ,∞ ,
2 2 4 2
     
 1 
1 1
91.  or − ∞, − ∪ ,∞ .
1 +  < 3 4 2
 x
1 93. First find a quadratic equation with solutions −4 and 3.
−3 < 1 + < 3
x (x + 4)(x − 3) = 0
1 1
−3 < 1 + and 1 + < 3 x2 + x − 12 = 0
x x
1 Test a point in each of the three intervals determined by
First solve −3 < 1 + . −4 and 3.
x
1 1 (−∞, −4): (−5 + 4)(−5 − 3) = 8 > 0
0 < 4 + , or +4>0
x x (−4, 3): (0 + 4)(0 − 3) = −12 < 0
1
The denominator of f (x) = + 4 is 0 when x = 0, so the (3, ∞): (4 + 4)(4 − 3) = 8 > 0
x
function is not defined for this value of x. Now solve the Then a quadratic inequality for which the solution set is
related equation. (−4, 3) is x2 + x − 12 < 0. Answers may vary.
1
+4 = 0 72
x 95. f (x) =
x2 − 4x − 21
1 + 4x = 0 Multiplying by x
The radicand must be nonnegative and the denominator
1
x=− must be nonzero. Thus, the values of x for which x2 −4x−
4 21 > 0 comprise the domain. By inspecting the graph of
1 y = x2−4x−21 we see that the domain is {x|x < −3 or x > 7},
The critical values are − and 0. Test a value in each of
4 or (−∞, −3) ∪ (7, ∞).
the intervals determined by them.
 
1
− ∞, − : f (−1) = 3 > 0
4 Chapter 4 Review Exercises
 
1
− , 0 : f (−0.1) = −6 < 0
4
1. f (−b) = (−b+a)(−b+b)(−b−c) = (−b+a)·0·(−b−c) = 0,
(0, ∞): f (1) = 5 > 0
so the statement is true.
The
 solution
 set for this portion of the inequality is
1 3. In addition to the given possibilities, 9 and −9 are also
− ∞, − ∪ (0, ∞). possible rational zeros. The statement is false.
4

Copyright © 2013 Pearson Education, Inc.


Chapter 4 Review Exercises 169

5. The domain of the function is the set of all real numbers 21. g(x) = (x − 1)3 (x + 2)2
except −2 and 3, or {x|x = −2 and x = 3}. The statement 1. The leading term is x · x · x · x · x, or x5 . The degree,
is false.
5, is odd and the leading coefficient, 1, is positive so
7. f (x) = x3 + 3x2 − 2x − 6 as x → ∞, g(x) → ∞ and as x → −∞, g(x) → −∞.
Graph the function and use the Zero, Maximum and Min- 2. We see that the zeros of the function are 1 and
imum features. −2, so the x-intercepts of the graph are (1, 0) and
(−2, 0).
a) Zeros: −3, −1.414, 1.414
3. The zeros divide the x-axis into 3 intervals,
b) Relative maximum: 2.303 at x = −2.291
(−∞, −2), (−2, 1), and (1, ∞). We choose a value
c) Relative minimum: −6.303 at x = 0.291 for x from each interval and find g(x). This tells us
d) Domain: all real numbers; range: all real numbers the sign of g(x) for all values of x in that interval.
In (−∞, −2), test −3:
9. f (x) = 7x2 − 5 + 0.45x4 − 3x3
g(−3) = (−3 − 1)3 (−3 + 2)2 = −64 < 0
= 0.45x4 − 3x3 + 7x2 − 5
In (−2, 1), test 0:
The leading term is 0.45x4 and the leading coefficient is
0.45. The degree of the polynomial is 4, so the polynomial g(0) = (0 − 1)3 (0 + 2)2 = −4 < 0
is quartic. In (1, ∞), test 2:
g(2) = (2 − 1)3 (2 + 2)2 = 16 > 0
11. g(x) = 6 − 0.5x
Thus the graph lies below the x-axis on (−∞, −2)
= −0.5x + 6
and on (−2, 1) and above the x-axis on (1, ∞). We
The leading term is −0.5x and the leading coefficient is also know that the points (−3, −64), (0, −4), and
−0.5. The degree of the polynomial is 1, so the polynomial (2, 16) are on the graph.
is linear.
4. From Step 3 we know that g(0) = −4, so the y-
1 intercept is (0, −4).
13. f (x) = − x4 + 3x2 + x − 6
2
5. We find additional points on the graph and then
1
The leading term is − x4 . The degree, 4, is even and the draw the graph.
2
1 y
leading coefficient, − , is negative. As x → ∞, x g(x)
2 4
f (x) → −∞, and as x → −∞, f (x) → −∞. −2.5 −10.7
  4 2 2 4 x
2
15. g(x) = x − (x + 2)3 (x − 5)2 −1 −8 4
3
8
2 −0.5 −7.6
, multiplicity 1; 12
3 0.5 −0.8
−2, multiplicity 3; 16

5, multiplicity 2
g(x)  (x  1)3 (x  2)2
17. h(x) = x + 4x − 9x − 36
3 2
6. Checking the graph as described on page 317 in the
= x2 (x + 4) − 9(x + 4) text, we see that it appears to be correct.
= (x + 4)(x2 − 9)
23. f (x) = x4 − 5x3 + 6x2 + 4x − 8
= (x + 4)(x + 3)(x − 3)
1. The leading term is x4 . The degree, 4, is even and
−4, ±3; each has multiplicity 1 the leading coefficient, 1, is positive so as x → ∞,
19. a) Linear: f (x) = 0.5408695652x − 30.30434783; f (x) → ∞ and as x → −∞, f (x) → ∞.
quadratic: f (x) = 0.0030322581x2 − 2. We solve f (x) = 0, or x4 − 5x3 + 6x2 + 4x − 8 = 0.
0.5764516129x + 57.53225806; The possible rational zeros are ±1, ±2, ±4, and ±8.
cubic: f (x) = 0.0000247619x3 − 0.0112857143x2 + We try −1.
2.002380952x − 82.14285714 
−1  1 −5 6 4 −8
b) For the linear function, f (300) ≈ 132; for the −1 6 −12 8
quadratic function, f (300) ≈ 158; for the cubic 1 −6 12 −8 0
function, f (300) ≈ 171. Since the cubic function
Now we have (x + 1)(x3 − 6x2 + 12x − 8) = 0. We
yields the result that is closest to the actual num-
use synthetic division to determine if 2 is a zero of
ber of 180, it makes the best prediction.
x3 − 6x2 + 12x − 8 = 0.
c) Using the cubic function, we have: f (350) ≈ 298 
2  1 −6 12 −8
and f (400) ≈ 498. 2 −8 8
1 −4 4 0

Copyright © 2013 Pearson Education, Inc.


170 Chapter 4: Polynomial and Rational Functions


We have (x + 1)(x − 2)(x2 − 4x + 4) = 0, or 31. −1  1 0 0 0 −2 0
(x + 1)(x − 2)(x − 2)2 = 0. Thus the zeros of f (x) −1 1 −1 1 1
are −1 and 2 and the x-intercepts of the graph are 1 −1 1 −1 −1 1
(−1, 0) and (2, 0).
The quotient is x4 − x3 + x2 − x − 1; the remainder is 1.
3. The zeros divide the x-axis into 3 intervals, 
(−∞, −1), (−1, 2), and (2, ∞). We choose a value 33. −2  1 0 0 0 −16
for x from each interval and find f (x). This tells us −2 4 −8 16
the sign of f (x) for all values of x in that interval. 1 −2 4 −8 0
In (−∞, −1), test −2: f (−2) = 0

f (−2) = (−2) − 5(−2) + 6(−2) + 4(−2) − 8 =
4 3 2 35. −i  1 −5 1 −5
64 > 0 −i −1 + 5i 5
In (−1, 2), test 0: 1 −5 − i 5i 0
f (−i) = 0, so −i is a zero of f (x).
f (0) = 04 − 5 · 03 + 6 · 02 + 4 · 0 − 8 = −8 < 0 
−5  1 −5 1 −5
In (2, ∞), test 3:
−5 50 −255
f (3) = 34 − 5 · 33 + 6 · 32 + 4 · 3 − 8 = 4 > 0 1 −10 51 −260
Thus the graph lies above the x-axis on (−∞, −1) f (−5) = 0, so −5 is not a zero of f (x).
and on (2, ∞) and below the x-axis on (−1, 2). We 
1 
also know that the points (−2, 64), (0, −8), and 37. 3 1 − 43 − 53 2
3
(3, 4) are on the graph. 1
3 − 13 − 23
4. From Step 3 we know that f (0) = −8, so the y- 1 −1 −2 0
intercept is (0, −8).  
5. We find additional points on the graph and then 1 1
f = 0, so is a zero of f (x).
draw the graph. 3 3

x f (x) y 1  1 − 43 − 53 2
3

−1.5 21.4 8 1 − 13 −2
4 1 − 13 −2 − 43
−0.5 −7.8
4 2 2 4 x f (1) = 0, so 1 is not a zero of f (x).
1 −2 4
39. f (x) = x3 + 2x2 − 7x + 4
4 40 8
Try x + 1 and x + 2. Using synthetic division we find that
f (−1) = 0 and f (−2) = 0. Thus x + 1 and x + 2 are not
f (x )  x 4  5x 3  6x 2  4x  8 factors of f (x). Try x + 4.

6. Checking the graph as described on page 317 in the −4  1 2 −7 4
text, we see that it appears to be correct. −4 8 −4
1 −2 1 0
25. f (1) = 4 · 12 − 5 · 1 − 3 = −4
Since f (−4) = 0, x + 4 is a factor of f (x). Thus f (x) =
f (2) = 4 · 22 − 5 · 2 − 3 = 3 (x + 4)(x2 − 2x + 1) = (x + 4)(x − 1)2 .
By the intermediate value theorem, since f (1) and f (2) Now we solve f (x) = 0.
have opposite signs, f (x) has a zero between 1 and 2.
x+4 = 0 or (x − 1)2 = 0
27. 6x2 + 16x + 52 x = −4 or x−1 = 0
x − 3 6x3 − 2x2 + 4x − 1 x = −4 or x=1
6x3 − 18x2 The solutions of f (x) = 0 are −4 and 1.
16x2 + 4x
16x2 − 48x 41. f (x) = x4 − 4x3 − 21x2 + 100x − 100
52x − 1 Using synthetic division we find that f (2) = 0:
52x − 156 
2  1 −4 −21 100 −100
155
2 −4 −50 100
Q(x) = 6x2 + 16x + 52; R(x) = 155; 1 −2 −25 50 0
P (x) = (x − 3)(6x2 + 16x + 52) + 155 Then we have:
 f (x) = (x − 2)(x3 − 2x2 − 25x + 50)
29. 5  1 2 −13 10
5 35 110 = (x − 2)[x2 (x − 2) − 25(x − 2)]
1 7 22 120 = (x − 2)(x − 2)(x2 − 25)
The quotient is x2 + 7x + 22; the remainder is 120. = (x − 2)2 (x + 5)(x − 5)

Copyright © 2013 Pearson Education, Inc.


Chapter 4 Review Exercises 171

Now solve f (x) = 0. 53. 1 − i is also a zero.


(x − 2) = 0 or x + 5 = 0
2
or x − 5 = 0 f (x) = (x + 1)(x − 4)[x − (1 + i)][x − (1 − i)]
x − 2 = 0 or x = −5 or x=5 = (x2 − 3x − 4)(x2 − 2x + 2)
x = 2 or x = −5 or x=5 = x4 − 2x3 + 2x2 − 3x3 + 6x2 − 6x − 4x2 + 8x − 8
The solutions of f (x) = 0 are 2, −5, and 5. = x4 − 5x3 + 4x2 + 2x − 8
 
43. A polynomial function of degree 3 with −4, −1, and 2 55. f (x) = x − 1 (x − 0)(x + 3)
as zeros has factors x + 4, x + 1, and x − 2 so we have 3
 
f (x) = an (x + 4)(x + 1)(x − 2). 1
= x2 − x (x + 3)
The simplest polynomial is obtained if we let an = 1. 3
f (x) = (x + 4)(x + 1)(x − 2) 8 2
= x + x −x
3

= (x2 + 5x + 4)(x − 2) 3
= x3 − 2x2 + 5x2 − 10x + 4x − 8 57. g(x) = 3x4 − x3 + 5x2 − x + 1
= x3 + 3x2 − 6x − 8 Possibilities for p ±1
:
Possibilities for q ±1, ±3
1 √
45. A polynomial function of degree 3 with , 1 − 2, and Possibilities for p/q: ±1, ±
1
2
√ 1 √ 3
1 + 2 as zeros has factors x − , x − (1 − 2), and
√ 2 59. f (x) = 3x5 + 2x4 − 25x3 − 28x2 + 12x
x − (1 + 2) so we  have
1 √ √ a) We know that 0 is a zero since
f (x) = an x − [x − (1 − 2)][x − (1 + 2)]. f (x) = x(3x4 + 2x3 − 25x2 − 28x + 12).
2
Let an = 1. Now consider g(x) = 3x4 + 2x3 − 25x2 − 28x + 12.
 
1 √ √ Possibilities for p/q: ±1, ±2, ±3, ±4, ±6, ±12,
f (x) = x − [x − (1 − 2)][x − (1 + 2)]
2 1 2 4
  ± ,± ,±
1 √ √ 3 3 3
= x− [(x − 1) + 2][(x − 1) − 2]
2 From the graph of y = 3x4 +2x3 −25x2 −28x+12, we
  1
1 see that, of all the possibilities above, only −2, ,
= x− (x2 − 2x + 1 − 2) 3
2
  2
1 , and 3 might be zeros. We use synthetic division
= x− (x2 − 2x − 1) 3
2 to determine if −2 is a zero.

1 1 −2  3 2 −25 −28 12
= x3 − 2x2 − x − x2 + x +
2 2 −6 8 34 −12
5 1 3 −4 −17 6 0
= x3 − x2 +
2 2 Now try 3 in the quotient above.
If we let an = 2, we obtain f (x) = 2x3 − 5x2 + 1. 
3  3 −4 −17 6
47. A polynomial function of degree 5 has at most 5 real ze- 9 15 −6
ros. Since 5 zeros are given, these are all of the zeros of 3 5 −2 0
the desired function. We proceed as in Exercise 39 above, We have f (x) = (x + 2)(x − 3)(3x2 + 5x − 2).
letting an = 1. We find the other zeros.
f (x) = (x + 3)2 (x − 2)(x − 0)2 3x3 + 5x − 2 = 0
= (x2 + 6x + 9)(x3 − 2x2 ) (3x − 1)(x + 2) = 0
= x5 + 6x4 + 9x3 − 2x4 − 12x3 − 18x2 3x − 1 = 0 or x + 2 = 0
= x5 + 4x4 − 3x3 − 18x2 3x = 1 or x = −2
49. A polynomial function of degree 5 can have at most 5 zeros. 1
x= or x = −2
Since f (x) has rational coefficients, in addition to the√3 3
given zeros,
√ the other
√ zeros √ are the conjugates of 1 + 3 The rational zeros of g(x) = 3x4 +2x3 −25x2 −28x+
and − 3, or 1 − 3 and 3. 1
12 are −2, 3, and . Since 0 is also a zero of f (x),
√ 3
51. − 11 is also a zero. 1
√ √ the zeros of f (x) are −2, 3, , and 0. (The zero −2
f (x) = (x − 11)(x + 11) 3
has multiplicity 2.) These are the only zeros.
= x2 − 11
b) From our work above we see
f (x) = x(x + 2)(x − 3)(3x − 1)(x + 2), or
x(x + 2)2 (x − 3)(3x − 1).

Copyright © 2013 Pearson Education, Inc.


172 Chapter 4: Polynomial and Rational Functions

61. f (x) = x4 − 6x3 + 9x2 + 6x − 10 We find the other zeros.


x3 − 3x2 − 16x + 48 = 0
a) Possibilities for p/q: ±1, ±2, ±5, ±10
x2 (x − 3) − 16(x − 3) = 0
From the graph of f (x), we see that −1 and 1 might
be zeros. (x − 3)(x2 − 16) = 0

−1  1 −6 9 6 −10 (x − 3)(x + 4)(x − 4) = 0
−1 7 −16 10 x − 3 = 0 or x + 4 = 0 or x − 4 = 0
1 −7 16 −10 0
 x = 3 or x = −4 or x=4
1  1 −7 16 −10
The rational zeros are 0, −4, 3, and 4. (The zeros
1 −6 10
0 and −4 each have multiplicity 2.) These are the
1 −6 10 0
only zeros.
f (x) = (x + 1)(x − 1)(x2 − 6x + 10)
b) f (x) = x2 (x + 4)2 (x − 3)(x − 4)
Using the quadratic formula, we find that the other
zeros are 3 ± i. 67. f (x) = 2x6 − 7x3 + x2 − x
The rational zeros are −1 and 1. The other zeros There are 3 variations in sign in f (x), so there are 3 or 1
are 3 ± i. positive real zeros.
b) f (x) = (x + 1)(x − 1)[x − (3 + i)][x − (3 − i)] f (−x) = 2(−x)6 − 7(−x)3 + (−x)2 − (−x)
= (x + 1)(x − 1)(x − 3 − i)(x − 3 + i) = 2x6 + 7x3 + x2 + x
There are no variations in sign in f (−x), so there are no
63. f (x) = 3x3 − 8x2 + 7x − 2 negative real zeros.
1 2
a) Possibilities for p/q: ±1, ±2, ± , ± 69. g(x) = 5x5 − 4x2 + x − 1
3 3
2 There are 3 variations in sign in g(x), so there are 3 or 1
From the graph of f (x), we see that and 1 might
3 positive real zeros.
be zeros.
g(−x) = 5(−x)5 − 4(−x)2 + (−x) − 1
1  3 −8 7 −2
3 −5 2 = −5x5 − 4x2 − x − 1
3 −5 2 0 There is no variation in sign in g(−x), so there are 0 neg-
We have f (x) = (x − 1)(3x − 5x + 2).
2 ative real zeros.
We find the other zeros. 5
71. f (x) =
3x2 − 5x + 2 = 0 (x − 2)2
1. The numerator and the denominator have no com-
(3x − 2)(x − 1) = 0
mon factors. The denominator is zero when x = 2,
3x − 2 = 0 or x − 1 = 0 so the domain excludes 2. It is (−∞, 2) ∪ (2, ∞).
2 The line x = 2 is the vertical asymptote.
x= or x=1
3 2. Because the degree of the numerator is less than
2 the degree of the denominator, the x-axis, or y = 0,
The rational zeros are 1 and . (The zero 1 has
3 is the horizontal asymptote. There is no oblique
multiplicity 2.) These are the only zeros.
asymptote.
b) f (x) = (x − 1)2 (3x − 2)
3. The numerator has no zeros, so there is no x-
65. f (x) = x6 + x5 − 28x4 − 16x3 + 192x2 intercept.
 
a) We know that 0 is a zero since 5 5 5
4. f (0) = = , so the y-intercept is 0, .
f (x) = x2 (x4 + x3 − 28x2 − 16x + 192). (0 − 2)2 4 4
Consider g(x) = x4 + x3 − 28x2 − 16x + 192. 5. Find other function values to determine the shape
of the graph and then draw it.
Possibilities for p/q: ±1, ±2, ±3, ±4, ±6, ±8, ±12,
±16, ±24, ±32, ±48, ±64, ±96, y
x2
10
±192
5 8
From the graph of y = g(x), we see that −4, 3 and f(x) 
(x  2)2
6
4 might be zeros.

−4  1
4
1 −28 −16 192
y0
−4 12 64 −192 2
1 −3 −16 48 0 4 2 2 4 x
We have f (x) = x2 · g(x) =
x2 (x + 4)(x3 − 3x2 − 16x + 48).

Copyright © 2013 Pearson Education, Inc.


Chapter 4 Review Exercises 173

x−2 x−2 The solutions of the related equation are 1, −4 and


73. f (x) = =
x2 − 2x − 15 (x + 3)(x − 5) 2. These numbers divide the x-axis into the intervals
1. The numerator and the denominator have no com- (−∞, −4), (−4, 1), (1, 2) and (2, ∞).
mon factors. The denominator is zero when x = −3, We let f (x) = (1 − x)(x + 4)(x − 2) and test a value in
or x = 5, so the domain excludes −3 and 5. It is each interval.
(−∞, −3) ∪ (−3, 5) ∪ (5, ∞). The lines x = −3 and (−∞, −4) : f (−5) = 42 > 0
x = 5 are vertical asymptotes.
(−4, 1) : f (0) = −8 < 0
2. Because the degree of the numerator is less than  
the degree of the denominator, the x-axis, or y = 0, 3 11
(1, 2) : f = >0
is the horizontal asymptote. There is no oblique 2 8
asymptote. (2, ∞) : f (3) = −14 < 0
3. The numerator is zero when x = 2, so the x- Function values are negative on (−4, 1) and (2, ∞). Since
intercept is (2, 0). the inequality symbol is ≤, the endpoints of the intervals
must be included in the solution set. It is [−4, 1] ∪ [2, ∞).
0−2 2
4. f (0) = 2 = , so the y-intercept is
 0 − 2 · 0 − 15 15
2 81. a) We write and solve a polynomial equation.
0, . −16t2 + 80t + 224 = 0
15
5. Find other function values to determine the shape −16(t2 − 5t − 14) = 0
of the graph and then draw it. −16(t + 2)(t − 7) = 0
y The solutions are t = −2 and t = 7. Only t = 7 has
meaning in this application. The rocket reaches the
4 ground at t = 7 seconds.
2 y0 b) We write and solve a polynomial inequality.
2 2 x −16t2 + 80t + 224 > 320 Polynomial inequality
2
−16t2 + 80t − 96 > 0 Equivalent inequality
4
−16t + 80t − 96 = 0
2
Related equation
x  3 x5 −16(t2 − 5t + 6) = 0
x2
f (x)  −16(t − 2)(t − 3) = 0
x 2  2x  15
The solutions of the related equation are 2 and 3.
75. Answers may vary. The numbers −2 and 3 must be zeros These numbers divide the t-axis into the intervals
of the denominator, and −3 must be zero of the numerator. (−∞, 2), (2, 3) and (3, ∞). We restrict our discus-
In addition, the numerator and denominator must have the sion to values of t such that 0 ≤ t ≤ 7 since we know
same degree and the ratio of the leading coefficients must from part (a) the rocket is in the air for 7 sec. We
be 4. consider the intervals [0, 2), (2, 3) and (3, 7]. We let
4x(x + 3) 4x2 + 12x f (t) = −16t2 + 80t − 96 and test a value in each
f (x) = , or f (x) = 2 interval.
(x + 2)(x − 3) x −x−6
[0, 2): f (1) = −32 < 0
77. x2 − 9 < 0 Polynomial inequality  
5
x −9 = 0
2
Related equation (2, 3): f =4>0
2
(x + 3)(x − 3) = 0 Factoring (3, 7]: f (4) = −32 < 0
The solutions of the related equation are −3 and 3. These Function values are positive on (2, 3). The solution
numbers divide the x-axis into the intervals (−∞, −3), set is (2, 3)
(−3, 3), and (3, ∞).
x2 + 2x − 3 x2 + 2x − 3
We let f (x) = (x + 3)(x − 3) and test a value in each 83. g(x) = =
x2 − 5x + 6 (x − 2)(x − 3)
interval.
The values of x that make the denominator 0 are 2 and
(−∞, −3): f (−4) = 7 > 0 3, so the domain is (−∞, 2) ∪ (2, 3) ∪ (3, ∞). Answer A is
(−3, 3): f (0) = −9 < 0 correct.
(3, ∞): f (4) = 7 > 0 1
85. f (x) = − x4 + x3 + 1
Function values are negative only on (−3, 3). The solution 2
set is (−3, 3). The degree of the function is even and the leading coef-
ficient is negative, so as x → ∞, f (x) → −∞ and as
79. (1 − x)(x + 4)(x − 2) ≤ 0 Polynomial inequality x → −∞, f (x) → −∞. In addition, f (0) = 1, so the
(1 − x)(x + 4)(x − 2) = 0 Related equation y-intercept is (0, 1). Thus B is the correct graph.

Copyright © 2013 Pearson Education, Inc.


174 Chapter 4: Polynomial and Rational Functions

 
  (x + 1)(x − 1)
87. 1 − 1  < 3 −3 = 0
 x2  x2
1 (x + 1)(x − 1) − 3x2 = 0 Multiplying by x2
−3 < 1 − 2 < 3
x x2 − 1 − 3x2 = 0
x −12
2x2 = −1
−3 < <3
x2 1
x2 = −
(x + 1)(x − 1) 2
−3 < <3
x2 There are no real solutions for this portion of the inequal-
(x + 1)(x − 1) (x + 1)(x − 1) ity.
 The solution
  set ofthe original inequality is
−3 < and <3 1 1
x2 x2 − ∞, − ∪ ,∞ .
First, solve 2 2
(x + 1)(x − 1) 89. (x − 2)−3 < 0
−3 <
x2 1
<0
(x + 1)(x − 1) (x − 2)3
0< +3
x2 1
(x + 1)(x − 1) The denominator of f (x) = is zero when x = 2,
The denominator of f (x) = +3 is zero when (x − 2)3
x2 so the function is not defined for this value of x. The
x = 0, so the function is not defined for this value of x. 1
related equation = 0 has no solution, so 2 is the
Solve the related equation. (x − 2)3
(x + 1)(x − 1) only critical point. Test a value in each of the intervals
+3 = 0 determined by this critical point.
x2
(x + 1)(x − 1) + 3x2 = 0 Multiplying by x2 (−∞, 2) : f (1) = −1 < 0
x − 1 + 3x = 0
2 2 (2, ∞) : f (3) = 1 > 0
4x2 = 1 Function values are negative on (−∞, 2). The solution set
1 is (−∞, 2).
x2 =
4 91. Divide x3 + kx2 + kx − 15 by x + 3.

−3  1
1
x=± k k −15
2 −3 9 − 3k −27 + 6k
1 1 1 −3 + k 9 − 2k −42 + 6k
The critical values are − , 0 and . Test a value in each
2 2
of the intervals determined by them. Thus f (−3) = −42 + 6k.
 
1 We know that if x + 3 is a factor of f (x), then f (−3) = 0.
− ∞, − : f (−1) = 3 > 0 We solve −42 + 6k = 0 for k.
2
    −42 + 6k = 0
1 1
− ,0 : f − = −12 < 0
2 4 6k = 42
   
1 1 k=7
0, : f = −12 < 0 √
2 4
  93. f (x) = x2 + 3x − 10
1
, ∞ : f (1) = 3 > 0 Since we cannot take the square root of a negative number,
2 then x2 + 3x − 10 ≥ 0.
The
 solution set for this portion of the inequality is x2 + 3x − 10 ≥ 0 Polynomial inequality
1 1
− ∞, − ∪ ,∞ . x2 + 3x − 10 = 0 Related equation
2 2
Next, solve (x + 5)(x − 2) = 0 Factoring
(x + 1)(x − 1) The solutions of the related equation are −5 and 2. These
<3 numbers divide the x-axis into the intervals (−∞, −5),
x2
(x + 1)(x − 1) (−5, 2) and (2, ∞).
−3 < 0 We let g(x) = (x + 5)(x − 2) and test a value in each
x2
(x + 1)(x − 1) interval.
The denominator of f (x) = − 3 is zero (−∞, −5) : g(−6) = 8 > 0
x2
when x = 0, so the function is not defined for this value of (−5, 2) : g(0) = −10 < 0
x. Now solve the related equation.
(2, ∞) : g(3) = 8 > 0
Functions values are positive on (−∞, −5) and (2, ∞).
Since the equality symbol is ≥, the endpoints of the
intervals must be included in the solution set. It is
(−∞, −5] ∪ [2, ∞).

Copyright © 2013 Pearson Education, Inc.


Chapter 4 Test 175

1 In 2000, x = 2000 − 1900 = 100.


95. f (x) = 
5 − |7x + 2| f (100) = −0.0000007623221(100)4
We cannot take the square root of a negative number; + 0.00021189064(100)3 − 0.016314058(100)2
neither can the denominator be zero. Thus we have + 0.2440779643(100) + 13.59260684 ≈ 10.5%
5 − |7x + 2| > 0.
5. f (x) = x3 − 5x2 + 2x + 8
5 − |7x + 2| > 0 Polynomial inequality
1. The leading term is x3 . The degree, 3, is odd and
|7x + 2| < 5
the leading coefficient, 1, is positive so as x → ∞,
−5 < 7x + 2 < 5 f (x) → ∞ and as x → −∞, f (x) → −∞.
−7 < 7x < 3
2. We solve f (x) = 0. By the rational zeros theorem,
3 we know that the possible rational zeros are 1, −1,
−1 < x <

7
 2, −2, 4, −4, 8, and −8. Synthetic division shows
The solution set is − 1,
3 that 1 is not a zero. We try −1.
. 
7 −1  1 −5 2 8
97. No; since imaginary zeros of polynomials with rational co- −1 6 −8
efficients occur in conjugate pairs, a third-degree polyno- 1 −6 8 0
mial with rational coefficients can have at most two imag- We have f (x) = (x + 1)(x2 − 6x + 8) =
inary zeros. Thus, there must be at least one real zero. (x + 1)(x − 2)(x − 4).
99. If P (x) is an even function, then P (−x) = P (x) and Now we find the zeros of f (x).
thus P (−x) has the same number of sign changes as P (x). x+1 = 0 or x − 2 = 0 or x − 4 = 0
Hence, P (x) has one negative real zero also.
x = −1 or x = 2 or x=4
101. A quadratic inequality ax2 +bx+c ≤ 0, a > 0, or ax2 +bx+ The zeros of the function are −1, 2, and 4, so the
c ≥ 0, a < 0, has a solution set that is a closed interval. x-intercepts are (−1, 0), (2, 0), and (4, 0).
3. The zeros divide the x-axis into 4 intervals,
Chapter 4 Test (−∞, −1), (−1, 2), (2, 4), and (4, ∞). We choose a
value for x in each interval and find f (x). This tells
us the sign of f (x) for all values of x in that interval.
1. f (x) = 2x3 + 6x2 − x4 + 11
In (−∞, −1), test −3:
= −x4 + 2x3 + 6x2 + 11
f (−3) = (−3)3 − 5(−3)2 + 2(−3) + 8 = −70 < 0
The leading term is −x4 and the leading coefficient is −1.
The degree of the polynomial is 4, so the polynomial is In (−1, 2), test 0:
quartic. f (0) = 03 − 5(0)2 + 2(0) + 8 = 8 > 0

2. h(x) = −4.7x + 29 In (2, 4), test 3:


The leading term is −4.7x and the leading coefficient is f (3) = 33 − 5(3)2 + 2(3) + 8 = −4 < 0
−4.7. The degree of the polynomial is 1, so the polynomial In (4, ∞), test 5:
is linear. f (5) = 53 − 5(5)2 + 2(5) + 8 = 18 > 0
3. f (x) = x(3x − 5)(x − 3)2 (x + 1)3 Thus the graph lies below the x-axis on (−∞, −1)
5 and on (2, 4) and above the x-axis on (−1, 2) and
The zeros of the function are 0, , 3, and −1. (4, ∞). We also know the points (−3, −70), (0, 8),
3
The factors x and 3x − 5 each occur once, so the zeros 0 (3, −4), and (5, 18) are on the graph.
5 4. From Step 3 we know that f (0) = 8, so the y-
and have multiplicity 1.
3 intercept is (0, 8).
The factor x − 3 occurs twice, so the zero 3 has multiplic-
ity 2. 5. We find additional points on the graph and draw
the graph.
The factor x + 1 occurs three times, so the zero −1 has y
multiplicity 3. x f (x)
8
4. In 1930, x = 1930 − 1900 = 30. −0.5 5.625
4
f (30) = −0.0000007623221(30)4 + 0.00021189064(30)3 −
0.5 7.875
0.016314058(30)2 + 0.2440779643(30) + 13.59260684 ≈ 4 2 2 4 x
11.3% 2.5 −2.625 4

In 1990, x = 1990 − 1900 = 90. 8


6 56
f (90) = −0.0000007623221(90) + 0.00021189064(90) −
4 3

0.016314058(90)2 + 0.2440779643(90) + 13.59260684 ≈ f(x )  x 3  5x 2  2x  8


7.9%

Copyright © 2013 Pearson Education, Inc.


176 Chapter 4: Polynomial and Rational Functions

6. Checking the graph as described on page 317 in the 5. We find additional points on the graph and draw
text, we see that it appears to be correct. the graph.
y
6. f (x) = −2x4 + x3 + 11x2 − 4x − 12 x f (x)
4
1. The leading term is −2x4 . The degree, 4, is even and −0.5 −7.5
the leading coefficient, −2, is negative so x → ∞, 4 2 2 4 x
f (x) → −∞ and as x → −∞, f (x) → −∞. 0.5 −11.25 4

2. We solve f (x) = 0. 8
2.5 −15.75
12
The possible rational zeros are ±1, ±2, ±3, ±4, ±6,
1 3
±12, ± , and ± . We try −1.
2 2 f(x )  2x 4  x 3  11x 2  4x  12

−1  −2 1 11 −4 −12
2 −3 −8 12 6. Checking the graph as described on page 317 in the
−2 3 8 −12 0 text, we see that it appears to be correct.
We have f (x) = (x + 1)(−2x3 + 3x2 + 8x − 12). Find 7. f (0) = −5 · 02 + 3 = 3
the other zeros. f (2) = −5 · 22 + 3 = −17
−2x3 + 3x2 + 8x − 12 = 0
By the intermediate value theorem, since f (0) and f (2)
2x3 − 3x2 − 8x + 12 = 0 Multiplying by −1 have opposite signs, f (x) has a zero between 0 and 2.
x2 (2x − 3) − 4(2x − 3) = 0
8. g(−2) = 2(−2)3 + 6(−2)2 − 3 = 5
(2x − 3)(x2 − 4) = 0
g(−1) = 2(−1)3 + 6(−1)2 − 3 = 1
(2x − 3)(x + 2)(x − 2) = 0
Since both g(−2) and g(−1) are positive, we cannot use
2x − 3 = 0 or x + 2 = 0 or x − 2 = 0 the intermediate value theorem to determine if there is a
2x = 3 or x = −2 or x=2 zero between −2 and −1.
3
x= or x = −2 or x=2 9. x3 + 4x2 + 4x + 6
2
x − 1 x4 + 3x3 + 0x2 + 2x − 5
3
The zeros of the function are −1, , −2, and 2, so x4 − x3
2   4x3 + 0x2
3
the x-intercepts are (−2, 0), (−1, 0), , 0 , and 4x3 − 4x2
2
4x2 + 2x
(2, 0).
4x2 − 4x
3. The zeros divide the x-axis
 into
 5 intervals,
 6x − 5
3 3 6x − 6
(−∞, −2), (−2, −1), − 1, , , 2 , and
2 2 1
(2, ∞). We choose a value for x in each interval and
find f (x). This tells us the sign of f (x) for all values The quotient is x3 + 4x2 + 4x + 6; the remainder is 1.
of x in that interval. P (x) = (x − 1)(x3 + 4x2 + 4x + 6) + 1
In (−∞, −2), test −3: f (−3) = −90 < 0 
10. 5  3 0 −12 7
In (−2, −1), test −1.5: f (−1.5) = 5.25 > 0 15 75 315
 
3 3 15 63 322
In − 1, , test 0: f (0) = −12 < 0
2 Q(x) = 3x2 + 15x + 63; R(x) = 322
 
3 
In , 2 , test 1.75: f (1.75) ≈ 1.29 > 0 11. −3  2 −6 1 −4
2
−6 36 −111
In (2, ∞), test 3: f (3) = −60 < 0 2 −12 37 −115
Thus  lies below the x-axis on (−∞, −2),
 the graph P (−3) = −115
3
on − 1, , and on (2, ∞) and above the x-axis on 
2   12. −2  1 4 1 −6
3 −2 −4 6
(−2, −1) and on , 2 . We also know the points
2 1 2 −3 0
(−3, −90), (−1.5, 5.25), (0, −12), (1.75, 1.29), and f (−2) = 0, so −2 is a zero of f (x).
(3, −60) are on the graph.
4. From Step 3 we know that f (0) = −12, so the y-
intercept is (0, −12).

Copyright © 2013 Pearson Education, Inc.


Chapter 4 Test 177

13. The function can be written in the form 20. g(x) = 2x4 − 11x3 + 16x2 − x − 6
f (x) = an (x + 3)2 (x)(x − 6). 1 3
a) Possibilities for p/q: ±1, ±2, ±3, ±6, ± , ±
The simplest polynomial is obtained if we let an = 1. 2 2
1
f (x) = (x + 3)2 (x)(x − 6) From the graph of y = g(x), we see that − , 1, 2,
2
= (x2 + 6x + 9)(x2 − 6x) 1
and 3 might be zeros. We try − .
= x4 + 6x3 + 9x2 − 6x3 − 36x2 − 54x 2

− 12  2 −11 16 −1 −6
= x4 − 27x2 − 54x
−1 6 −11 6
14. A polynomial function of degree 5 can have at most 5 zeros. 2 −12 22 −12 0
Since f (x) has rational coefficients, in addition to
√ the 3 Now we try 1.
given zeros,√the other zeros are the conjugates of 3 and 
1  2 −12 22 −12
2 − i, or − 3 and 2 + i.
2 −10 12
15. −3i is also a zero. 2 −10 12 0
 
f (x) = (x + 10)(x − 3i)(x + 3i) 1
We have g(x) = x + (x−1)(2x2 −10x + 12) =
= (x + 10)(x2 + 9) 2
 
= x3 + 10x2 + 9x + 90 1
2 x+ (x − 1)(x2 − 5x + 6). We find the other
√ 2
zeros.
16. 3 and 1 + i are also zeros.
√ √ x2 − 5x + 6 = 0
f (x) = (x − 0)(x + 3)(x − 3)[x − (1 − i)][x − (1 + i)]
(x − 2)(x − 3) = 0
= x(x2 − 3)[(x − 1) + i][(x − 1) − i]
x − 2 = 0 or x − 3 = 0
= (x3 − 3x)(x2 − 2x + 1 + 1)
x = 2 or x=3
= (x3 − 3x)(x2 − 2x + 2)
1
= x5 − 2x4 + 2x3 − 3x3 + 6x2 − 6x The rational zeros are − , 1, 2, and 3. These are
2
= x5 − 2x4 − x3 + 6x2 − 6x the only zeros.
 
1
17. f (x) = 2x3 + x2 − 2x + 12 b) g(x) = 2 x + (x − 1)(x − 2)(x − 3)
2
Possibilities for p ±1, ±2, ±3, ±4, ±6, ±12
: = (2x + 1)(x − 1)(x − 2)(x − 3)
Possibilities for q ±1, ±2
1 3 21. h(x) = x3 + 4x2 + 4x + 16
Possibilities for p/q: ±1, ±2, ±3, ±4, ±6, ±12, ± , ±
2 2 a) Possibilities for p/q: ±1, ±2, ±4, ±8, ±16
18. h(x) = 10x4 − x3 + 2x − 5 From the graph of h(x), we see that −4 might be a
Possibilities for p ±1, ±5 zero.
−4  1
:
Possibilities for q ±1, ±2, ±5, ±10 4 4 16
1 5 1 1 −4 0 −16
Possibilities for p/q: ±1, ±5, ± , ± , ± , ± 1 0 4 0
2 2 5 10
We have h(x) = (x + 4)(x2 + 4). We find the other
19. f (x) = x3 + x2 − 5x − 5
zeros.
a) Possibilities for p/q: ±1, ±5 x2 + 4 = 0
From the graph of y = f (x), we see that −1 might x2 = −4
be a zero.
x = ±2i
−1  1 1 −5 −5
−1 0 5 The rational zero is −4. The other zeros are ±2i.
1 0 −5 0 b) h(x) = (x + 4)(x + 2i)(x − 2i)
We have f (x) = (x + 1)(x − 5). We find the other
2
22. f (x) = 3x4 − 11x3 + 15x2 − 9x + 2
zeros.
1 2
x2 − 5 = 0 a) Possibilities for p/q: ±1, ±2, ± , ±
3 3
x2 = 5 2
√ From the graph of f (x), we see that and 1 might
x=± 5 3
√ 2
The rational zero is −1. The other zeros are ± 5. be zeros. We try .
3
√ √ 
b) f (x) = (x + 1)(x − 5)(x + 5) 2 
3 3 −11 15 −9 2
2 −6 6 −2
3 −9 9 −3 0

Copyright © 2013 Pearson Education, Inc.


178 Chapter 4: Polynomial and Rational Functions

Now we try 1. x+3 x+3


 25. f (x) = =
1  3 −9 9 −3 x2 − 3x − 4 (x + 1)(x − 4)
3 −6 3 1. The numerator and the denominator have no com-
3 −6 3 0 mon factors. The denominator is zero when x = −1
  or x = 4, so the domain excludes −1 and 4. It is
2
We have f (x) = x − (x−1)(3x2 −6x + 3) = (−∞, −1) ∪ (−1, 4) ∪ (4, ∞). The lines x = −1 and
3
  x = 4 are vertical asymptotes.
2
3 x− (x − 1)(x2 − 2x + 1). We find the other 2. Because the degree of the numerator is less than the
3
zeros. degree of the denominator, the x-axis, or y = 0, is
x2 − 2x + 1 = 0 the horizontal asymptote.
(x − 1)(x − 1) = 0 3. The numerator is zero at x = −3, so the x-intercept
is (−3, 0).
x − 1 = 0 or x − 1 = 0 0+3 3
4. f (0) = = − , so the y-intercept is
x = 1 or x=1
 (0
 + 1)(0 − 4) 4
2 3
The rational zeros are and 1. (The zero 1 has 0, − .
3 4
multiplicity 3.) These are the only zeros. 5. Find other function values to determine the shape
 
2 of the graph and then draw it.
b) f (x) = 3 x − (x − 1)(x − 1)(x − 1)
3
y
= (3x − 2)(x − 1)3
4
23. g(x) = −x8 + 2x6 − 4x3 − 1
2 y0
There are 2 variations in sign in g(x), so there are 2 or 0
positive real zeros. 4 2 2 4 6 x
2
g(−x) = −(−x)8 + 2(−x)6 − 4(−x)3 − 1 x  1 x 4
4
= −x8 + 2x6 + 4x3 − 1
There are 2 variations in sign in g(−x), so there are 2 or
x3
0 negative real zeros. f (x )  
x2  3x  4
2
24. f (x) =
(x − 3)2 26. Answers may vary. The numbers −1 and 2 must be zeros of
1. The numerator and the denominator have no com- the denominator, and −4 must be a zero of the numerator.
mon factors. The denominator is zero when x = 3, x+4 x+4
so the domain excludes 3. it is (−∞, 3) ∪ (3, ∞). f (x) = , or f (x) = 2
(x + 1)(x − 2) x −x−2
The line x = 3 is the vertical asymptote.
27. 2x2 > 5x + 3 Polynomial inequality
2. Because the degree of the numerator is less than the
degree of the denominator, the x-axis, or y = 0, is 2x − 5x − 3 > 0
2
Equivalent inequality
the horizontal asymptote. 2x2 − 5x − 3 = 0 Related equation
3. The numerator has no zeros, so there is no x- (2x + 1)(x − 3) = 0 Factoring
intercept.
  The solutions of the related equation are −
1
and 3. These
2 2 2 2
4. f (0) = = , so the y-intercept is 0, . 
(0 − 3)2 9 9 1
numbers divide the x-axis into the intervals − ∞, − ,
5. Find other function values to determine the shape   2
of the graph and then draw it. 1
− , 3 , and (3, ∞).
2
2
f (x)  We let f (x) = (2x + 1)(x − 3) and test a value in each
y (x  3)2
interval.
 
1
4
− ∞, − : f (−1) = 4 > 0
2
2
 
1
4 2 2 4 x − , 3 : f (0) = −3 < 0
y0 2
2
(3, ∞) : f (4) = 9 > 0
4 x3  
1
Function values are positive on − ∞, − and (3, ∞).
  2
1
The solution set is − ∞, − ∪ (3, ∞).
2

Copyright © 2013 Pearson Education, Inc.


Chapter 4 Test 179

x+1 f (t) = −16t2 + 64t − 48 and test a value in each


28. ≤ 3 Rational inequality
x−4 interval.
 
x+1 1
− 3 ≤ 0 Equivalent inequality [0, 1) : f = −20 < 0
x−4 2
x+1 (1, 3) : f (2) = 16 > 0
The denominator of f (x) = − 3 is zero when x = 4,
x−4 (3, 6] : f (4) = −48 < 0
so the function is not defined for this value of x. We solve
the related equation f (x) = 0. Function values are positive on (1, 3). The solution
x+1 set is (1, 3).
−3 = 0
x−4 30. f (x) = x3 − x2 − 2
 
x+1 The degree of the function is odd and the leading coeffi-
(x − 4) − 3 = (x − 4) · 0
x−4 cient is positive, so as x → ∞, f (x) → ∞ and as x → −∞,
x + 1 − 3(x − 4) = 0 f (x) → −∞. In addition, f (0) = −2, so the y-intercept is
x + 1 − 3x + 12 = 0 (0, −2). Thus D is the correct graph.

−2x + 13 = 0 31. f (x) = x2 + x − 12
2x = 13 Since we cannot take the square root of a negative number,
13 then x2 + x − 12 ≥ 0.
x=
2 x2 + x − 12 ≥ 0 Polynomial inequality
13
The critical values are 4 and . They divide the x-axis x2 + x − 12 = 0 Related equation
2   
13 13 (x + 4)(x − 3) = 0 Factoring
into the intervals (−∞, 4), 4, and , ∞ . We
2 2 The solutions of the related equation are −4 and 3. These
test a value in each interval. numbers divide the x-axis into the intervals (−∞, −4),
(−∞, 4) : f (3) = −7 < 0 (−4, 3) and (3, ∞).
 
4,
13
: f (5) = 3 > 0 We let f (x) = (x + 4)(x − 3) and test a value in each
2 interval.
 
13 (−∞, −4) : g(−5) = 8 > 0
, ∞ : f (9) = −1 < 0
2 (−4, 3) : g(0) = −12 < 0
 
13 (3, ∞) : g(4) = 8 > 0
Function values are negative for (−∞, 4) and ,∞ .
2 Function values are positive on (−∞, −4) and (3, ∞).
Since the
 inequality
 symbol is ≤, the endpoint of the in-
Since the inequality symbol is ≥, the endpoints of the
13
terval , ∞ must be included in the solution set. It is intervals must be included in the solution set. It is
2  (−∞, −4] ∪ [3, ∞).
13
(−∞, 4) ∪ ,∞ .
2
29. a) We write and solve a polynomial equation.
−16t2 + 64t + 192 = 0
−16(t2 − 4t − 12) = 0
−16(t + 2)(t − 6) = 0
The solutions are t = −2 and t = 6. Only t = 6 has
meaning in this application. The rocket reaches the
ground at t = 6 seconds.
b) We write and solve a polynomial inequality.
−16t2 + 64t + 192 > 240
−16t2 + 64t − 48 > 0
−16t2 + 64t − 48 = 0 Related equation
−16(t2 − 4t + 3) = 0
−16(t − 1)(t − 3) = 0
The solutions of the related equation are 1 and 3.
These numbers divide the t-axis into the intervals
(−∞, 1), (1, 3) and (3, ∞). Because the rocket re-
turns to the ground at t = 6, we restrict our discus-
sion to values of t such that 0 ≤ t ≤ 6. We consider
the intervals [0, 1), (1, 3) and (3, 6]. We let

Copyright © 2013 Pearson Education, Inc.


Copyright © 2013 Pearson Education, Inc.
Chapter 5
Exponential and Logarithmic Functions
15. Graph y = |x|. Some points on the graph are (0, 0),
Exercise Set 5.1 (−2, 2), (2, 2), (−5, 5), and (5, 5). Plot these points and
draw the graph. Then reflect the graph across the line
1. We interchange the first and second coordinates of each y = x.
ordered pair to find the inverse of the relation. It is
{(8, 7), (8, −2), (−4, 3), (−8, 8)}. y ⫽ 兩x 兩
y
3. We interchange the first and second coordinates of each 4
ordered pair to find the inverse of the relation. It is
2
{(−1, −1), (4, −3)}.
⫺4 ⫺2 2 4 x
5. Interchange x and y. ⫺2

y = 4x−5
  y⫽x ⫺4
 
x = 4y−5 x ⫽ 兩y兩

7. Interchange x and y.
17. We show that if f (a) = f (b), then a = b.
y = −5
x3  1 1
 a−6 = b−6
3 3
y 3 x = −5
1 1
a= b Adding 6
9. Interchange x and y. 3 3
x = y2 −2y a=b Multiplying by 3
 
   Thus f is one-to-one.
y = x2 −2x
19. We show that if f (a) = f (b), then a = b.
11. Graph x = y 2 − 3. Some points on the graph are (−3, 0), 1 1
(−2, −1), (−2, 1), (1, −2), and (1, 2). Plot these points a 3 + = b3 +
2 2
and draw the curve. Then reflect the graph across the line 1
y = x. a3 = b 3 Subtracting
2
a=b Taking cube roots
y
yx Thus f is one-to-one.
y x2 3 4
21. g(−1) = 1 − (−1)2 = 1 − 1 = 0 and
g(1) = 1 − 12 = 1 − 1 = 0, so g(−1) = g(1) but −1 = 1.
4 2 4 x Thus the function is not one-to-one.

23. g(−2) = (−2)4 − (−2)2 = 16 − 4 = 12 and


4
x  y2  3 g(2) = 24 − 22 = 16 − 4 = 12, so g(−2) = g(2) but −2 = 2.
  Thus the function is not one-to-one.
2
13. Graph y = 3x − 2. The intercepts are (0, −2) and ,0 . 25. The function is one-to-one, because no horizontal line
3
Plot these points and draw the line. Then reflect the graph crosses the graph more than once.
across the line y = x.
27. The function is not one-to-one, because there are many
y horizontal lines that cross the graph more than once.

4 29. The function is not one-to-one, because there are many


2 horizontal lines that cross the graph more than once.
x  3y  2
4 2 2 4 x 31. The function is one-to-one, because no horizontal line
2 y  3x  2 crosses the graph more than once.
yx 4

Copyright © 2013 Pearson Education, Inc.


182 Chapter 5: Exponential and Logarithmic Functions

33. The graph of f (x) = 5x − 8 is shown below. Since there is no horizontal line that crosses the graph
more than once, the function is one-to-one.
y
2
10 41. The graph of f (x) = is shown below.
8 3
6
y
4 f (x ) = 5x — 8
2 5

x 4 2
–10 –8 –6 –4 –2 2 4 6 8 10 f (x ) =
–2 3 3
–4 2
–6 1
–8
–5 –4 –3 –2 –1 1 2 3 4 5 x
–10 –1
–2
–3
Since there is no horizontal line that crosses the graph –4
more than once, the function is one-to-one. –5

35. The graph of f (x) = 1 − x2 is shown below.


2
Since the horizontal line y = crosses the graph more than
y 3
once, the function is not one-to-one.
5

4
43. The graph of f (x) = 25 − x2 is shown below.
3 f (x ) = 1 — x 2
2 y
1
5
–5 –4 –3 –2 –1 1 2 3 4 5 x
–1 4
–2 3
–3 2
–4 1
–5
–5 –4 –3 –2 –1 1 2 3 4 5 x
–1
–2 f (x ) = 25 x 2
Since there are many horizontal lines that cross the graph –3
more than once, the function is not one-to-one. –4
–5
37. The graph of f (x) = |x + 2| is shown below.
y Since there are many horizontal lines that cross the graph
5
more than once, the function is not one-to-one.

y1 ⫽ 0.8x ⫹ 1.7,
4
45.
f (x ) = | x + 2 |
x ⫺ 1.7
3
2
y2 ⫽
1 0.8
–5 –4 –3 –2 –1
–1
1 2 3 4 5 x
15 y2 y1
–2
–3
–4
–5
⫺5 25

Since there are many horizontal lines that cross the graph ⫺5
more than once, the function is not one-to-one.
Both the domain and the range of f are the set of all real
4
39. The graph of f (x) = − is shown below. numbers. Then both the domain and the range of f −1 are
x
also the set of all real numbers.
y

47. 1
5
y1 ⫽ ⫺x ⫺ 4,
4 4 2
3
f (x ) =
x y2 ⫽ 2x ⫹ 8
2 10
1

–5 –4 –3 –2 –1 1 2 3 4 5 x
–1
–2 ⫺15 15
–3
–4
–5
y1 y2 ⫺10

Copyright © 2013 Pearson Education, Inc.


Exercise Set 5.1 183

Both the domain and the range of f are the set of all real b) Replace f (x) with y: y = x + 4
numbers. Then both the domain and the range of f −1 are Interchange x and y: x = y + 4
also the set of all real numbers.
√ Solve for y: x − 4 = y
49. y1 = x − 3, y2 = x2 + 3, x ≥ 0 Replace y with f −1 (x): f −1 (x) = x − 4
57. a) The graph of f (x) = 2x−1 is shown below. It passes
the horizontal-line test, so it is one-to-one.
y

5
f (x ) = 2 x – 1
4
3
The domain of f is [3, ∞) and the range of f is [0, ∞). 2
Then the domain of f −1 is [0, ∞) and the range of f −1 is 1

[3, ∞). —5 —4 —3 —2 —1 1 2 3 4 5 x


—1

51. y1 = x2 − 4, x ≥ 0; y2 = 4 + x —2
—3
—4
—5

b) Replace f (x) with y: y = 2x − 1


Interchange x and y: x = 2y − 1
x+1
Solve for y: =y
Since it is specified that x ≥ 0, the domain of f is [0, ∞). 2
The range of f is [−4, ∞). Then the domain of f −1 is x+1
Replace y with f −1 (x): f −1 (x) =
[−4, ∞) and the range of f −1 is [0, ∞). 2
59. a) The graph of f (x) = 4
is shown below. It passes
√ x+7
3
x+9 the horizontal-line test, so the function is one-to-
53. y1 = (3x − 9)3 , y2 = one.
3
y

10 4
f (x ) = x–––
+7
8

6
4
2

—10 —8 —6 —4 —2 2 4 6 8 10 x
—2
Both the domain and the range of f are the set of all real —4

numbers. Then both the domain and the range of f −1 are —6


—8
also the set of all real numbers. —10

55. a) The graph of f (x) = x + 4 is shown below. It passes 4


b) Replace f (x) with y: y =
the horizontal-line test, so it is one-to-one. x+7
4
y Interchange x and y: x =
y+7
f (x ) = x + 4 5
4 Solve for y: x(y + 7) = 4
3 4
2 y+7 =
1
x
4
—5 —4 —3 —2 —1
—1
1 2 3 4 5 x
y = −7
x
—2
4
—3
Replace y with f −1 (x): f −1 (x) = −7
—4 x
—5

Copyright © 2013 Pearson Education, Inc.


184 Chapter 5: Exponential and Logarithmic Functions

x+4 y
61. a) The graph of f (x) = is shown below. It passes
x−3 5
the horizontal-line test, so the function is one-to- 4
f (x ) = x √ 4 – x 2
one. 3
2
y 1

—5 —4 —3 —2 —1 1 2 3 4 5 x
10
f (x ) = xx–––
+4
–3
—1
8
—2
6
—3
4
—4
2
—5
—10 —8 —6 —4 —2 2 4 6 8 10 x
—2
—4
—6
67. a) The graph of f (x) = 5x2 − 2, x ≥ 0 is shown below.
—8 It passes the horizontal-line test, so it is one-to-one.
—10

x+4 5
b) Replace f (x) with y: y =
x−3
4
3
y+4 2
Interchange x and y: x =
y−3 1

Solve for y: (y − 3)x = y + 4 —5 —4 —3 —2 —1


—1
1 2 3 4 5 x

xy − 3x = y + 4 —2
—3
xy − y = 3x + 4 —4
x 2 – 2, x > 0
y(x − 1) = 3x + 4 f (x ) = 5—5

3x + 4
y= b) Replace f (x) with y: y = 5x2 − 2
x−1
3x + 4 Interchange x and y: x = 5y 2 − 2
Replace y with f −1 (x): f −1 (x) =
x−1 Solve for y: x + 2 = 5y 2
63. a) The graph of f (x) = x3 −1 is shown below. It passes x+2
= y2
the horizontal-line test, so the function is one-to- 5

one. x+2
=y
5
y
(We take the principal square root, because
f (x ) = x 3 – 1
5
x ≥ 0 in the original equation.)
4

3
−1 −1 x+2
2 Replace y with f (x): f (x) = for
1 5
x+2
—5 —4 —3 —2 —1 1 2 3 4 5 x all x in the range of f (x), or f −1 (x) = ,
—1 5
—2 x ≥ −2

—3
—4 69. a) The graph of f (x) = x + 1 is shown below. It
—5
passes the horizontal-line test, so the function is one-
to-one.
b) Replace f (x) with y: y = x3 − 1
y
Interchange x and y: x = y 3 − 1
5 f (x ) = √ x + 1
Solve for y: x + 1 = y3

4
3
x+1 = y 3

√ 2
Replace y with f −1 (x): f −1 (x) = x + 1 3
1

√ —5 —4 —3 —2 —1 1 2 3 4 5 x
65. a) The graph of f (x) = x 4 − x2 is shown below. —1
—2
Since there are many horizontal lines that cross the —3
graph more than once, the function is not one-to-one —4
and thus does not have an inverse that is a function. —5

Copyright © 2013 Pearson Education, Inc.


Exercise Set 5.1 185


b) Replace f (x) with y: y =x+1 83. We find (f −1 ◦f )(x) and (f ◦f −1 )(x) and check to see that
√ each is x.
Interchange x and y: x = y + 1  
7
Solve for y: x2 = y + 1 (f −1 ◦ f )(x) = f −1 (f (x)) = f −1 x =
  8
x −1 = y
2
8 7
x =x
Replace y with f −1 (x): f −1 (x) = x2 − 1 for all x 7 8
   
in the range of f (x), or f −1 (x) = x2 − 1, x ≥ 0. 8 7 8
(f ◦ f −1 )(x) = f (f −1 (x)) = f x = x =x
71. f (x) = 3x 7 8 7
The function f multiplies an input by 3. Then to reverse 85. We find (f −1 ◦f )(x) and (f ◦f −1 )(x) and check to see that
this procedure, f −1 would divide each of its inputs by 3. each is x.
x 1  
Thus, f −1 (x) = , or f −1 (x) = x. 1−x
3 3 (f −1 ◦ f )(x) = f −1 (f (x)) = f −1 =
x
73. f (x) = −x 1 1 1
= = =x
The outputs of f are the opposites, or additive inverses, of 1−x 1−x+x 1
+1
the inputs. Then the outputs of f −1 are the opposites of x x x
its inputs. Thus, f −1 (x) = −x.  
−1 −1 1
√ (f ◦ f )(x) = f (f (x)) = f =
75. f (x) = 3 x − 5 x+1
1 x+1−1 x
The function f subtracts 5 from each input and then takes 1−
the cube root of the result. To reverse this procedure, f −1 x+1 = x+1 = x+1 =x
would raise each input to the third power and then add 5 1 1 1
to the result. Thus, f −1 (x) = x3 + 5. x+1 x+1 x+1
 
77. We reflect the graph of f across the line y = x. The re- 2
5 x+1 −5
flections of the labeled points are (−5, −5), (−3, 0), (1, 2), 87. (f −1 ◦ f )(x) = f −1 (f (x)) =
5
=
and (3, 5). 2
2x + 5 − 5 2x
y
= =x
(3, 5) 2 2
2  5x − 5 
4 (f ◦ f −1 )(x) = f (f −1 (x)) = +1=
5 2
2
(⫺3, 0) (1, 2) x−1+1=x
⫺4 ⫺2 2 4 x
⫺2
89. Replace f (x) with y: y = 5x − 3
⫺4 Interchange x and y: x = 5y − 3
(⫺5, ⫺5) Solve for y: x + 3 = 5y
x+3
79. We reflect the graph of f across the line y = x. The re- =y
flections of the labeled points are (−6, −2), (1, −1), (2, 0), 5
x+3 1 3
and (5.375, 1.5). Replace y with f −1 (x): f −1 (x) = , or x +
5 5 5
y The domain and range of f are (−∞, ∞), so the domain
and range of f −1 are also (−∞, ∞).
4

2 (5.375, 1.5) y
f
⫺4 ⫺2 2 4 x 4
⫺2 (2, 0) f ⫺1
(⫺6, ⫺2) 2
(1, ⫺1)
⫺4
⫺4 ⫺2 2 4 x
⫺2

81. We reflect the graph of f across the line y = x. ⫺4

y
2
91. Replace f (x) with y: y =
4 x
2
2 Interchange x and y: x =
y
⫺4 ⫺2 2 4 x Solve for y: xy = 2
⫺2
2
⫺4 y=
x

Copyright © 2013 Pearson Education, Inc.


186 Chapter 5: Exponential and Logarithmic Functions

2
Replace y with f −1 (x): f −1 (x) = 97. Since f (f −1 (x)) = f −1 (f (x)) = x, then f (f −1 (5)) = 5
x and f −1 (f (a)) = a.
The domain and range of f are (−∞, 0) ∪ (0, ∞), so the
domain and range of f −1 are also (−∞, 0) ∪ (0, ∞). 60 + 2x
99. Replace C(x) with y: y =
x
y 60 + 2y
Interchange x and y: x =
y
4

2
Solve for y: xy = 60 + 2y
xy − 2y = 60
4 2 2 4 x
y(x − 2) = 60
2
f f 1 60
4 y=
x−2
60
1 3 Replace y with C −1 (x): C −1 (x) =
93. Replace f (x) with y: y = x −2 x−2
3
C −1 (x) represents the number of people in the group,
1
Interchange x and y: x = y 3 − 2 where x is the cost per person in dollars.
3
1 3
Solve for y: x+2 = y 5 5
3 101. a) T (−13◦ ) = (−13◦ − 32◦ ) = (−45◦ ) = −25◦
3x + 6 = y 3 9 9
√ 5 5
3
3x + 6 = y T (86◦ ) = (86◦ − 32◦ ) = (54◦ ) = 30◦
√ 9 9
Replace y with f −1 (x): f −1 (x) = 3
3x + 6
b) Replace T (x) with y: y = 5 (x − 32)
The domain and range of f are (−∞, ∞), so the domain 9
and range of f −1 are also (−∞, ∞). 5
Interchange x and y: x = (y − 32)
9
y 9
f Solve for y: x = y − 32
4 5
2 f 1 9
x + 32 = y
5
4 2 2 4 x 9
2 Replace y with T −1 (x): T −1 (x) = x + 32
5
4
T −1 (x) represents the Fahrenheit temperature when
the Celsius temperature is x.
x+1
95. Replace f (x) with y: y =
x−3 103. The functions for which the coefficient of x2 is negative
y+1 have a maximum value. These are (b), (d), (f), and (h).
Interchange x and y: x =
y−3
105. Since |2| > 1 the graph of f (x) = 2x2 can be obtained
Solve for y: xy − 3x = y + 1 by stretching the graph of f (x) = x2 vertically. Since
1
xy − y = 3x + 1   1
0 <   < 1, the graph of f (x) = x2 can be obtained by
y(x − 1) = 3x + 1 4 4
shrinking the graph of y = x2 vertically. Thus the graph
3x + 1 of f (x) = 2x2 , or (a) is narrower.
y=
x−1
3x + 1 107. We can write (f) as f (x) = −2[x − (−3)]2 + 1. Thus the
Replace y with f −1 (x): f −1 (x) = graph of (f) has vertex (−3, 1).
x−1
The domain of f is (−∞, 3) ∪ (3, ∞) and the range of f is 109. Graph y1 = f (x) and y2 = g(x) and observe that the
(−∞, 1) ∪ (1, ∞). Thus the domain of f −1 is graphs are reflections of each other across the line y = x.
(−∞, 1) ∪ (1, ∞) and the range of f −1 is (−∞, 3) ∪ (3, ∞). Thus, the functions are inverses of each other.

y 111. The graph of f (x) = x2 − 3 is a parabola with vertex


(0, −3). If we consider x-values such that x ≥ 0, then the
4
f 1 graph is the right-hand side of the parabola and it passes
2 the horizontal line test. We find the inverse of f (x) =
f
4 2 2 4
x2 − 3, x ≥ 0.
x
2 Replace f (x) with y: y = x2 − 3
4 Interchange x and y: x = y 2 − 3

Copyright © 2013 Pearson Education, Inc.


Exercise Set 5.2 187

Solve for y: x + 3 = y2 13. Graph f (x) = 6x .



x+3 = y Compute some function values, plot the corresponding
(We take the principal square root, because x ≥ 0 in the points, and connect them with a smooth curve.
original equation.) x y = f (x) (x, y)

Replace y with f −1 (x): f −1√(x) = x + 3 for all x in the  
range of f (x), or f −1 (x) = x + 3, x ≥ −3. −3
1
− 3,
1
216 216
Answers may vary. There are other restrictions that also  
make f (x) one-to-one. 1 1
−2 − 2, y
36 36
3   6
113. Answers may vary. f (x) = , f (x) = 1 − x, f (x) = x. 1 1
x −1 − 1, 5
6 6 4
0 1 (0, 1) 3
Exercise Set 5.2 2
1 6 (1, 6) f (x)  6x
2 36 (2, 36)
1. e4 ≈ 54.5982 3 2 1 1 2 3 x
3 216 (3, 216) 1
−2.458
3. e ≈ 0.0856
 x
1
5. f (x) = −2x − 1 15. Graph f (x) = .
4
f (0) = −20 − 1 = −1 − 1 = −2
Compute some function values, plot the corresponding
The only graph with y-intercept (0, −2) is (f). points, and connect them with a smooth curve.

7. f (x) = ex + 3 x y = f (x) (x, y)

This is the graph of f (x) = ex shifted up 3 units. Then −3 64 (−3, 64)


(e) is the correct choice. −2 16 (−2, 16)
9. f (x) = 3−x − 2 −1 4 (−1, 4)
−0
f (0) = 3 − 2 = 1 − 2 = −1 0 1 (0, 1) y
 
Since the y-intercept is (0, −1), the correct graph is (a) or 1
1
1,
1 5
(c). Check another point on the graph. f (−1) = 4 4 4
3−(−1) − 2 = 3 − 2 = 1, so (−1, 1) is on the graph. Thus   3 f (x)  冢~冣
x
1 1
(a) is the correct choice. 2 2, 2
16 16
 
11. Graph f (x) = 3x . 1 1 3 2 1 1 2 3
3 3, x
64 64 1
Compute some function values, plot the corresponding
points, and connect them with a smooth curve.
17. Graph y = −2x .
x y = f (x) (x, y)
  x y (x, y)
1 1
−3 − 3,  
27 27 1 1
  −3 − − 3, −
1 1 8 8
−2 − 2,  
9 9 y 1 1
  −2 − − 2, −
1 1 4 4 y
−1 − 1, 8  
3 3 1 1
6 −1 − − 1, − 4
0 1 (0, 1) 2 2
4 2
1 3 (1, 3) 0 −1 (0, −1)
2 f (x)  3x
2 9 (2, 9) 1 −2 (1, −2) 4 2 2 4 x
2
3 27 (3, 27)
321
1
1 2 3 x 2 −4 (2, −4) y  2x
4
3 −8 (3, −8)

Copyright © 2013 Pearson Education, Inc.


188 Chapter 5: Exponential and Logarithmic Functions

19. Graph f (x) = −0.25x + 4. y


2
x y = f (x) (x, y) 1
3 2 1 1 2 3 x
−3 −60 (−3, −60)
2
y f (x)  1  ex
−2 −12 (−2, −12) 3
4 4
−1 0 (−1, 0) 5
2 f(x)  0.25x  4 6
0 3 (0, 3)
1 3.75 (1, 3.75) 4 2 2 4 x
2 27. Shift the graph of y = 2x left 1 unit.
2 3.94 (2, 3.94)
4
3 3.98 (3, 3.98) y

8
21. Graph f (x) = 1 + e−x . 6

x y = f (x) (x, y) 4

2 f (x) 2x  1
−3 21.1 (−3, 21.1)
y
−2 8.4 (−2, 8.4) 4 2 2 4 x
2
−1 3.7 (−1, 3.7) 4
f(x)  1  ex
2
0 2 (0, 2) 29. Shift the graph of y = 2x down 3 units.
1 1.4 (1, 1.4) 4 2 2 4 x
y
2
2 1.1 (2, 1.1) 6
4
3 1.0 (3, 1.0) 4

2
1
23. Graph y = ex . 4 2 2 4 x
4 2
f (x)  2x  3
Choose values for x and compute the corresponding y-
4
values. Plot the points (x, y) and connect them with a
smooth curve.
31. Shift the graph of y = 2x left 1 unit, reflect it across the
x y (x, y) y-axis, and shift it up 2 units.
−3 0.0124 (−3, 0.0124) y
y
−2 0.0338 (−2, 0.0338)
6
−1 0.0920 (−1, 0.0920) 5 4
y ~e x 4 2
0 0.25 (0, 0.25) 3
1 0.6796 (1, 0.6796) 2 ⫺4 ⫺2 2 4 x
1 ⫺2
2 1.8473 (2, 1.8473)
3 2 1 1 2 3 x ⫺4 f(x) ⫽ 21⫺x ⫹ 2
3 5.0214 (3, 5.0214) 1

25. Graph f (x) = 1 − e−x . 33. Reflect the graph of y = 3x across the y-axis, then across
Compute some function values, plot the corresponding the x-axis, and then shift it up 4 units.
points, and connect them with a smooth curve.
y
x y (x, y) 6 f (x) 4  3x
4
−3 −19.0855 (−3, −19.0855)
2
−2 −6.3891 (−2, −6.3891)
−1 −1.7183 (−1, −1.7183) 4 2 2 4 x
2
0 0 (0, 0)
4
1 0.6321 (1, 0.6321)
2 0.8647 (2, 0.8647)
3 0.9502 (3, 0.9502)

Copyright © 2013 Pearson Education, Inc.


Exercise Set 5.2 189

 x
3 47. Reflect the graph of y = ex across the y-axis and then
35. Shift the graph of y = right 1 unit.
2 across the x-axis; shift it up 1 unit and then stretch it
vertically.
y
f (x)  ( 32 )x  1 y
8
4
6
2
4
2 2 4 6 x
2
2

4 2 2 4 x 4

37. Shift the graph of y = 2x left 3 units and down 5 units. f (x) 2(1  ex )

y
49. We graph f (x) = e−x − 4 for x < −2, f (x) = x + 3 for
4 −2 ≤ x < 1, and f (x) = x2 for x ≥ 1.
f (x) 2x  3  5 2
y
8 6 4 2 x
2 4
4 2

4 2 2 4 x
39. Shift the graph of y = 2x right 1 unit, stretch it vertically, 2
and shift it up 1 unit. 4

y
ex  4, for x 2,
f (x)  x  3, for 2  x  1,
4
x2, for x  1
2
 nt
⫺4 ⫺2 2 4 x 51. a) We use the formula A = P 1 + r and substi-
⫺2 n
f(x) ⫽ 3 . 2 x⫺1 ⫹ 1 tute 82,000 for P , 0.045 for r, and 4 for n.
⫺4  4t
0.045
A(t) = 82, 000 1 + = 82, 000(1.01125)4t
4
41. Shrink the graph of y = ex horizontally.
b) y ⫽ 82,000(1.01125)4x
y 200,000

6 f (x)  e 2x
4 0 25
0
2
c) A(0) = 82, 000(1.01125)4·0 = $82, 000
4 2 2 4 x
A(2) = 82, 000(1.01125)4·2 ≈ $89, 677.22
43. Reflect the graph of y = ex across the x-axis, shift it up 1 A(5) = 82, 000(1.01125)4·5 ≈ $102, 561.54
unit, and shrink it vertically. The graph is in the answer A(10) = 82, 000(1.01125)4·10 ≈ $128, 278.90
section in the text.
d) Using the Intersect feature, we find that the first
45. Shift the graph of y = ex left 1 unit and reflect it across coordinate of the point of intersection of y1 =
the y-axis. 82, 000(1.01125)4x and y2 = 100, 000 is about 4.43,
so the amount in the account will reach $100,000 in
y
about 4.43 years. This is about 4 years, 5 months,
8 and 5 days.
6  nt
y  e x  1 r
4
53. We use the formula A = P 1 + and substitute 3000
n
2 for P , 0.05 for r, and 4 for n.
 4t
0.05
2 2 4 6 x A(t) = 3000 1 + = 3000(1.0125)4t
4

Copyright © 2013 Pearson Education, Inc.


190 Chapter 5: Exponential and Logarithmic Functions

On Willis’ sixteenth birthday, t = 16 − 6 = 10. 69. R(x) = 80, 000(1.1522)x


A(10) = 3000(1.0125)4·10 = 4930.86 In 1999, x = 1999 − 1996 = 3.
When the CD matures $4930.86 will be available. R(3) = 80, 000(1.1522)3 ≈ 122, 370 tons
 nt In 2007, x = 2007 − 1996 = 11.
r
55. We use the formula A = P 1 + and substitute 3000 R(11) = 80, 000(1.1522)11 ≈ 380, 099 tons
n
for P , 0.04 for r, 2 for n, and 2 for t. In 2012, x = 2012 − 1996 = 16.
 2·2
0.04 R(16) = 80, 000(1.1522)16 ≈ 771, 855 tons
A = 3000 1 + ≈ $3247.30
2 71. T (x) = 23.7624(1.0752)x
 nt
r In 2007, x = 2007 − 2006 = 1.
57. We use the formula A = P 1 + and substitute
n T (1) = 23.7624(1.0752)1 ≈ 25.5 million people, or
120,000 for P , 0.025 for r, 1 for n, and 10 for t. 25,500,000 people
 1·10
A = 120, 000 1 +
0.025
≈ $153, 610.15 In 2014, x = 2014 − 2006 = 8.
1 T (8) = 23.7624(1.0752)8 ≈ 42.4 million people, or
 nt 42,400,000 people
r
59. We use the formula A = P 1 + and substitute
n 73. a) y ⫽ 56,395(0.85)x
53,500 for P , 0.055 for r, 4 for n, and 6.5 for t. 60,000
 4(6.5)
0.055
A = 53, 500 1 + ≈ $76, 305.59
4
 nt
r 0 8
61. We use the formula A = P 1 + and substitute 0
n
17,400 for P , 0.081 for r, 365 for n, and 5 for t. b) V (t) = 56, 395(0.9)t
 365·5
0.081 V (0) = 56, 395(0.9)0 = $56, 395
A = 17, 400 1 + ≈ $26, 086.69
365 V (1) = 56, 395(0.9)1 ≈ $50, 756
63. W (x) = 23, 672.16(1.112)x V (3) = 56, 395(0.9)3 ≈ $41, 112
In 2000, x = 2000 − 1998 = 2. V (6) = 56, 395(0.9)6 ≈ $29, 971
W (2) = 23, 672.16(1.112)2 ≈ 29, 272 service members V (10) = 56, 395(0.9)10 ≈ $19, 664
In 2008, x = 2008 − 1998 = 10. c) Using the Intersect feature, we find that the first
W (10) = 23, 672.16(1.112)10
≈ 68, 436 service members coordinate of the point of intersection of
y1 = 56, 395(0.9)x and y2 = 30, 000 is approxi-
In 2013, x = 2013 − 1998 = 15. mately 6, so the machine will be replaced after 6
W (15) = 23, 672.16(1.112)15 ≈ 116, 362 service members years.

65. T (x) = 400(1.055)x 75. a)


In 1950, x = 1950 − 1913 = 37.
T (37) = 400(1.055)37 ≈ 2900 pages
In 1990, x = 1990 − 1913 = 77.
T (77) = 400(1.055)77 ≈ 24, 689 pages
b) f (25) = 100(1 − e−0.04(25) ) ≈ 63%.
In 2000, x = 2000 − 1913 = 87.
c) Using the Intersect feature, we find the first coordi-
T (87) = 400(1.055)87 ≈ 42, 172 pages
nate of the point of intersection of
67. M (x) = (200, 000)(1.1802)x y1 = 100(1 − e−0.04x ) and y2 = 90 is approximately
58, so 90% of the product market will have bought
In 2003, x = 2003 − 2001 = 2.
the product after 58 days.
M (2) = (200, 000)(1.1802)2 ≈ $278, 574
77. Graph (c) is the graph of y = 3x − 3−x .
In 2007, x = 2007 − 2001 = 6.
M (6) = (200, 000)(1.1802)6 ≈ $540, 460 79. Graph (a) is the graph of f (x) = −2.3x .
In 2013, x = 2013 − 2001 = 12. 81. Graph (l) is the graph of y = 2−|x| .
M (12) = (200, 000)(1.1802) 12
≈ $1, 460, 486
83. Graph (g) is the graph of f (x) = (0.58)x − 1.
85. Graph (i) is the graph of g(x) = e|x| .

Copyright © 2013 Pearson Education, Inc.


Exercise Set 5.3 191

87. Graph (k) is the graph of y = 2−x .


2
109. 7π ≈ 451.8078726 and π 7 ≈ 3020.293228, so π 7 is larger.
ex − e−x 7080 ≈ 4.054 × 10147 and 8070 ≈ 1.646 × 10133 , so 7080 is
89. Graph (m) is the graph of g(x) = . larger.
2
91. Graph y1 = |1 − 3x | and y2 = 4 + 3−x . Use the Intersect
2
111. a)
feature to find their point of intersection, (1.481, 4.090).
ex
93. Graph y1 = 2ex − 3 and y2 = . Use the Intersect feature
x
to find their points of intersection, (−0.402, −1.662) and
(1.051, 2.722).
b) There are no x-intercepts, so the function has no
95. Graph y1 = 5.3x − 4.2x and y2 = 1073. Use the Inter- zeros.
sect feature to find the first coordinate of their point of c) Relative minimum: none;
intersection. The solution is 4.448.
relative maximum: 1 at x = 0
97. Graph y1 = 2x and y2 = 1. Use the Intersect feature to
find their point of intersection, (0, 1). Note that the graph 113. f (x + h) − f (x) (2ex+h − 3) − (2ex − 3)
=
of y1 lies above the graph of y2 for all points to the right h h
of this point. Thus the solution set is (0, ∞). 2ex+h − 3 − 2ex + 3
=
99. Graph y1 = 2x + 3x and y2 = x2 + x3 . Use the Inter- h
sect feature to find the first coordinates of their points of 2ex+h − 2ex
intersection. The solutions are 2.294 and 3.228. =
h
101. (1 − 4i)(7 + 6i) = 7 + 6i − 28i − 24i2 2ex (eh − 1)
= (ex · eh = ex+h )
= 7 + 6i − 28i + 24 h
= 31 − 22i
Exercise Set 5.3
103. 2x2 − 13x − 7 = 0 Setting f (x) = 0
(2x + 1)(x − 7) = 0 1. Graph x = 3y .
2x + 1 = 0 or x − 7 = 0 Choose values for y and compute the corresponding x-
values. Plot the points (x, y) and connect them with a
2x = −1 or x=7
smooth curve.
1
x = − or x=7 x y (x, y)
2
1  
The zeros of the function are − and 7, and the x- 1 1
  2 −3 , −3
27 27
1
intercepts are − , 0 and (7, 0).  
2 1 1
−2 , −2
9 9 y
105. x4 − x2 = 0 Setting h(x) = 0  
1 1 3
x2 (x2 − 1) = 0 −1 , −1 2
x ⫽ 3y
3 3
x2 (x + 1)(x − 1) = 0 1 0 (1, 0) 1
2 4 6 8
x2 = 0 or x + 1 = 0 or x − 1 = 0 3 1 (3, 1) ⫺1
x

x = 0 or x = −1 or x=1 9 2 (9, 2) ⫺2
⫺3
The zeros of the function are 0, −1, and 1, and the x- 27 3 (27, 3)
intercepts are (0, 0), (−1, 0), and (1, 0).
 y
107. x3 + 6x2 − 16x = 0 1
3. Graph x = .
2
x(x2 + 6x − 16) = 0
Choose values for y and compute the corresponding x-
x(x + 8)(x − 2) = 0
values. Plot the points (x, y) and connect them with a
x = 0 or x + 8 = 0 or x − 2 = 0 smooth curve.
x = 0 or x = −8 or x=2
The solutions are 0, −8, and 2.

Copyright © 2013 Pearson Education, Inc.


192 Chapter 5: Exponential and Logarithmic Functions

x y (x, y) 9. log2 16 = 4 because the exponent to which we raise 2 to


get 16 is 4.
8 −3 (8, −3)
11. log5 125 = 3, because the exponent to which we raise 5 to
4 −2 (4, −2)
get 125 is 3.
2 −1 (2, −1)
y
13. log 0.001 = −3, because the exponent to which we raise 10
1 0 (1, 0)
  to get 0.001 is −3.
1 1 3
1 ,1 2 y 1
2 2 x ⫽ 冢q冣 15. log2 = −2, because the exponent to which we raise 2 to
  1 4
1 1 1
2 ,2 2 3 4 5 6 7 8 x get is −2
4 4 ⫺1
  4
⫺2
1 1 17. ln 1 = 0, because the exponent to which we raise e to get
3 ,3 ⫺3
8 8 1 is 0.
5. Graph y = log3 x. 19. log 10 = 1, because the exponent to which we raise 10 to
The equation y = log3 x is equivalent to x = 3y . We can get 10 is 1.
find ordered pairs that are solutions by choosing values for
21. log5 54 = 4, because the exponent to which we raise 5 to
y and computing the corresponding x-values.
get 54 is 4.
1
For y = −2, x = 3−2 = . √ 1
9 3 = log3 31/4 =
4
23. log3 , because the exponent to which
1 4
For y = −1, x = 3−1 = . we raise 3 to get 31/4
1
is .
3 4
For y = 0, x = 30 = 1.
25. log 10−7 = −7, because the exponent to which we raise 10
For y = 1, x = 31 = 3. to get 10−7 is −7.
For y = 2, x = 32 = 9.
1
x, or 3y y 27. log49 7 = , because the exponent to which we raise 49 to
2
y 1 √
1 get 7 is . (491/2 = 49 = 7)
−2 3 y ⫽ log3 x 2
9
2 3
1 29. ln e3/4 = , because the exponent to which we raise e to
−1 1 4
3 ⫺1
3
2 4 6 8 10 x get e3/4 is .
1 0 ⫺1 4
⫺2
3 1 31. log4 1 = 0, because the exponent to which we raise 4 to
⫺3
9 2 get 1 is 0.
√ 1
33. ln e = ln e1/2 = , because the exponent to which we
2
7. Graph f (x) = log x. 1
raise e to get e1/2 is .
Think of f (x) as y. The equation y = log x is equivalent 2
to x = 10y . We can find ordered pairs that are solutions 35. The exponent is the
by choosing values for y and computing the corresponding logarithm.
x-values. ↓ ↓
−2 103 = 1000⇒3 = log10 1000
For y = −2, x = 10 = 0.01.
↑ ↑ The base remains the
−1
For y = −1, x = 10 = 0.1. same.
For y = 0, x = 100 = 1. We could also say 3 = log 1000.
For y = 1, x = 101 = 10.
37. The exponent is
For y = 2, x = 102 = 100. the logarithm.
y ↓ ↓
x, or 10y y
1
3
f (x)  log x 81/3 = 2 ⇒ log 8 2 =
0.01 −2 2 3
↑ ↑ The base remains
0.1 −1 1
the same.
1 0 1 2 4 6 8 10 x
1
10 1 2
39. e3 = t ⇒ loge t = 3, or ln t = 3
3
100 2 41. e2 = 7.3891 ⇒ loge 7.3891 = 2, or ln 7.3891 = 2
43. pk = 3 ⇒ logp 3 = k

Copyright © 2013 Pearson Education, Inc.


Exercise Set 5.3 193

45. The logarithm is the 79. Graph y = 3x and then reflect this graph across the line
exponent. y = x to get the graph of y = log3 x.
↓ ↓
log 5 5 = 1⇒51 = 5 y
↑ ↑ The base remains the same.
4
f(x)  3x
47. log 0.01 = −2 is equivalent to log10 0.01 = −2. 2
The logarithm is
4 2 2 4 x
the exponent.
↓ ↓ 2
f 1(x)  log3 x
log10 0.01 = −2⇒10−2 = 0.01 4

↑ ↑ The base remains


the same.
81. Graph y = log x and then reflect this graph across the line
49. ln 30 = 3.4012 ⇒ e3.4012 = 30 y = x to get the graph of y = 10x .
51. loga M = −x ⇒ a−x = M y

53. loga T 3 = x ⇒ ax = T 3 f 1(x)  10 x


4

2
55. log 3 ≈ 0.4771
4 2 2 4 x
57. log 532 ≈ 2.7259 2
f (x)  log x
59. log 0.57 ≈ −0.2441 4

61. log(−2) does not exist. (The calculator gives an error mes-
sage.) 83. Shift the graph of y = log2 x left 3 units.
63. ln 2 ≈ 0.6931 y
65. ln 809.3 ≈ 6.6962 4

67. ln(−1.32) does not exist. (The calculator gives an error 2

message.) 2 2 4 6 x
2
69. Let a = 10, b = 4, and M = 100 and substitute in the f(x)  log 2 (x  3)
change-of-base formula. 4

log10 100
log4 100 = ≈ 3.3219
log10 4 Domain: (−3, ∞)
71. Let a = 10, b = 100, and M = 0.3 and substitute in the Vertical asymptote: x = −3
change-of-base formula.
85. Shift the graph of y = log3 x down 1 unit.
log10 0.3
log100 0.3 = ≈ −0.2614
log10 100 y

73. Let a = 10, b = 200, and M = 50 and substitute in the 4 y  log 3 x  1


change-of-base formula. 2
log10 50
log200 50 = ≈ 0.7384 2 4 6 8 x
log10 200
2

75. Let a = e, b = 3, and M = 12 and substitute in the 4


change-of-base formula.
ln 12
log3 12 = ≈ 2.2619 Domain: (0, ∞)
ln 3
Vertical asymptote: x = 0
77. Let a = e, b = 100, and M = 15 and substitute in the
change-of-base formula.
ln 15
log100 15 = ≈ 0.5880
ln 100

Copyright © 2013 Pearson Education, Inc.


194 Chapter 5: Exponential and Logarithmic Functions

87. Stretch the graph of y = ln x vertically. b) We substitute 3833.995 for P , since P is in thou-
sands.
y w(3833.995) = 0.37 ln 3833.995 + 0.05
8 ≈ 3.1 ft/sec
6 c) We substitute 433.746 for P , since P is in thousands.
4 w(433.746) = 0.37 ln 433.746 + 0.05
f(x)  4 ln x
2 ≈ 2.3 ft/sec
2 4 6 8 x d) We substitute 2242.193 for P , since P is in thou-
sands.
Domain: (0, ∞) w(2242.193) = 0.37 ln 2242.193 + 0.05
Vertical asymptote: x = 0 ≈ 2.9 ft/sec
e) We substitute 669.651 for P , since P is in thousands.
89. Reflect the graph of y = ln x across the x-axis and then
shift it up 2 units. w(669.651) = 0.37 ln 669.651 + 0.05
≈ 2.5 ft/sec
y
f) We substitute 340.882 for P , since P is in thousands.
4 w(340.882) = 0.37 ln 340.882 + 0.05
2 ≈ 2.2 ft/sec
2 4 6 8 x g) We substitute 798.382 for P , since P is in thousands.
2 w(798.382) = 0.37 ln 798.382 + 0.05
y  2  ln x
4 ≈ 2.5 ft/sec
h) We substitute 279.243 for P , since P is in thousands.
Domain: (0, ∞) w(279.243) = 0.37 ln 279.243 + 0.05
Vertical asymptote: x = 0 ≈ 2.1 ft/sec
91. Shift the graph of y = log x right 1 unit, shrink it verti- 107.7 · I0
cally, and shift it down 2 units. 97. a) R = log = log 107.7 = 7.7
I0
y 109.5 · I0
b) R = log = log 109.5 = 9.5
I0
4
106.6 · I0
2
1
f(x)  2 log (x  1)  2 c) R = log = log 106.6 = 6.6
I0
4 2 2 4 x 107.4 · I0
2 d) R = log = log 107.4 = 7.4
I0
4
108.0 · I0
e) R = log = log 108.0 = 8.0
I0
93. Graph g(x) = 5 for x ≤ 0 and g(x) = log x + 1 for x > 0. 107.9 · I0
f) R = log = log 107.9 = 7.9
y
I0
109.1 · I0
g) R = log = log 109.1 = 9.1
4 I0
2
109.3 · I0
h) R = log = log 109.3 = 9.3
4 2 2 4 x I0
2
99. a) 7 = − log[H+ ]
4
−7 = log[H+ ]
H+ = 10−7 Using the definition of
5, for x  0,
g (x)  logarithm
log x  1, for x  0
b) 5.4 = − log[H+ ]
95. a) We substitute 598.541 for P , since P is in thousands. −5.4 = log[H+ ]
w(598.541) = 0.37 ln 598.541 + 0.05 H+ = 10−5.4 Using the definition of
≈ 2.4 ft/sec logarithm
−6
H ≈ 4.0 × 10
+

Copyright © 2013 Pearson Education, Inc.


Chapter 5 Mid-Chapter Mixed Review 195

c) 3.2 = − log[H+ ] 113. f (x) = ln |x|


−3.2 = log[H ] + |x| must be positive. Since |x| > 0 for x = 0, the domain
is (−∞, 0) ∪ (0, ∞).
H+ = 10−3.2 Using the definition of
logarithm log(2x + 5)
115. Graph y = log2 (2x + 5) = . Observe
−4 log 2
H ≈ 6.3 × 10
+
5
d) 4.8 = − log[H+ ] that outputs are negative for inputs between − and −2.
  2
−4.8 = log[H+ ] 5
Thus, the solution set is − , −2 .
2
H+ = 10−4.8 Using the definition of
logarithm 117. Graph (d) is the graph of f (x) = ln |x|.
−5
H ≈ 1.6 × 10
+
119. Graph (b) is the graph of f (x) = ln x2 .
1014 · I0 121. a)
101. a) L = 10 log
I0
= 10 log 1014 = 10 · 14
≈ 140 decibels
1011.5 · I0
b) L = 10 log
I0
b) Use the Zero feature. The zero is 1.
= 10 log 1011.5 = 10 · 11.5
c) Use the Minimum feature. The relative minimum is
≈ 115 decibels
−0.368 at x = 0.368. There is no relative maximum.
109 · I0
c) L = 10 log 123. a)
I0
= 10 log 109 = 10 · 9
= 90 decibels
106.5 · I0
d) L = 10 log
I0
= 10 log 106.5 = 10 · 6.5 b) Use the Zero feature. The zero is 1.
= 65 decibels c) Use the Maximum feature. There is no relative min-
1010 · I0 imum. The relative maximum is 0.184 at x = 1.649.
e) L = 10 log
I0 4
125. Graph y1 = 4 ln x and y2 = x and use the Intersect
= 10 log 1010 · 10 · 10 e +1
feature to find the point(s) of intersection of the graphs.
= 100 decibels
The point of intersection is (1.250, 0.891).
1019.4 · I0
f) L = 10 log
I0 Chapter 5 Mid-Chapter Mixed Review
= 10 log 1019.4 · 10 · 19.4
= 194 decibels 1. The statement is false. The domain of y = log x, for in-
stance, is (0, ∞).
103. y = 6 = 0 · x + 6
Slope: 0; y-intercept (0, 6) 3. f (0) = e−0 = 1, so the y-intercept is (0, 1). The given
 statement is false.
105. −5  1 −6 3 10
−5 55 −290 5. The graph of f (x) = 3 + x2 is shown below. Since there
1 −11 58 −280 are many horizontal lines that cross the graph more than
once, the function is not one-to-one and thus does not have
The remainder is −280, so f (−5) = −280. an inverse that is a function.
√ √
107. f (x) = (x − 7)(x + 7)(x − 0) y

= (x − 7)(x)
2 5
4
= x − 7x
3
3 f (x ) = 3 + x 2
2
109. Using the change-of-base formula, we get 1
log5 8 x
= log2 8 = 3. —5 —4 —3 —2 —1
—1
1 2 3 4 5
log2 8
—2

111. f (x) = log5 x3 —3


—4
x3 must be positive. Since x3 > 0 for x > 0, the domain —5

is (0, ∞).

Copyright © 2013 Pearson Education, Inc.


196 Chapter 5: Exponential and Logarithmic Functions


7. (f −1 ◦ f )(x) = f −1 (f (x)) = ( x − 5)2 + 5 = x − 5 + 5 = x 11. Use the power rule.
√ √
(f ◦ f −1 )(x) = f (f −1 (x)) = x2 + 5 − 5 = x2 = x log y 8 = 8 log y
(This assumes that the domain of f −1 (x) is restricted to 13. Use the power rule.
{x|x ≥ 0}.)
logc K −6 = −6 logc K
Since (f −1 ◦ f )(x) = x = (f ◦ f −1 )(x), we know that
f −1 (x) = x2 + 5. 15. Use the power rule.
√ 1
ln 4 = ln 41/3 = ln 4
3
9. The graph of y = log2 x is (d).
3
11. The graph of f (x) = ex−1 is (c). 17. Use the quotient rule.
13. The graph of f (x) = ln (x − 2) is (b). logt
M
= logt M − logt 8
8
15. The graph of f (x) = | log x| is (e).
 nt 19. Use the quotient rule.
r x
17. A = P 1 + log = log x − log y
n y
 4·6
0.045 21. Use the quotient rule.
A = 3200 1 + ≈ $4185.57
4 r
ln = ln r − ln s
s
4
19. ln e−4/5 is − because the exponent to which we raise e 23. loga 6xy 5 z 4
5
4 = loga 6 + loga x + loga y 5 + loga z 4
to get e−4/5 is − .
5
Product rule
21. ln e2 = 2 because the exponent to which we raise e to get
= loga 6 + loga x + 5 loga y + 4 loga z
e2 is 2.
Power rule
1
23. log2 = −4 because the exponent to which we raise 2 to
16
25. p2 q 5
1 logb
get , or 2−4 , is −4. m4 b 9
16 = logb p2 q 5 − logb m4 b9 Quotient rule
25. log3 27 = 3 because the exponent to which we raise 3 to = logb p2 + logb q 5 − (logb m4 + logb b9 )
get 27 is 3.
Product rule
27. ln e = 1 because the exponent to which we raise e to get = logb p2 + logb q 5 − logb m4 − logb b9
e is 1.
= logb p2 + logb q 5 − logb m4 − 9 (logb b9 = 9)
r
29. log T = r is equivalent to 10 = T . = 2 logb p + 5 logb q − 4 logb m − 9 Power rule
log 10 1
31. logπ 10 = = ≈ 2.0115
log π log π 2
27. ln
33. The most interest will be earned the eighth year, because 3x3 y
the principle is greatest during that year. = ln 2 − ln 3x3 y Quotient rule

35. If log b < 0, then b < 1. = ln 2 − (ln 3 + ln x + ln y)


3
Product rule
= ln 2 − ln 3 − ln x3 − ln y
Exercise Set 5.4 = ln 2 − ln 3 − 3 ln x − ln y Power rule

29. log r3 t
1. Use the product rule.
log3 (81 · 27) = log3 81 + log3 27 = 4 + 3 = 7 = log(r t)1/2
3

1
3. Use the product rule. = log r3 t Power rule
2
log5 (5 · 125) = log5 5 + log5 125 = 1 + 3 = 4 1
= (log r3 + log t) Product rule
2
5. Use the product rule.
1
logt 8Y = logt 8 + logt Y = (3 log r + log t) Power rule
2
7. Use the product rule. 3 1
= log r + log t
2 2
ln xy = ln x + ln y
9. Use the power rule.
logb t3 = 3 logb t

Copyright © 2013 Pearson Education, Inc.


Exercise Set 5.4 197

x6 1
31. loga 41. loga x + 4 loga y − 3 loga x
p5 q 8 2
1 x6 = loga x1/2 + loga y 4 − loga x3 Power rule
= loga 5 8 = loga x1/2 y 4 − loga x3 Product rule
2 p q
1 1/2 4
x y
= [loga x6 − loga (p5 q 8 )] Quotient rule = loga Quotient rule
2 x3
1 y4
= [loga x6 − (loga p5 + loga q 8 )] Product rule = loga x−5/2 y 4 , or loga Simplifying
2 x5/2
1 √
= (loga x6 − loga p5 − loga q 8 ) 43. ln x2 − 2 ln x
2 √
= ln x2 − ln ( x)2 Power rule
1 √
= (6 loga x − 5 loga p − 8 loga q) Power rule = ln x2 − ln x [( x)2 = x]
2
5 x2
= 3 loga x − loga p − 4 loga q = ln Quotient rule
2 x
 = ln x
8 12
4 m n
33. loga
a3 b5 45. ln(x2 − 4) − ln(x + 2)
8 12
=
1 m n
loga 3 5 Power rule x2 − 4
4 a b = ln Quotient rule
x+2
1 (x + 2)(x − 2)
= (loga m8 n12 − loga a3 b5 ) Quotient rule = ln Factoring
4 x+2
=
1
[loga m8 + loga n12 − (loga a3 + loga b5 )] = ln(x − 2) Removing a factor of 1
4
Product rule 47. log(x2 − 5x − 14) − log(x2 − 4)
1 x2 − 5x − 14
= (loga m + loga n − loga a3 − loga b5 )
8 12
= log Quotient rule
4 x2 − 4
1 (x + 2)(x − 7)
= (loga m8 +loga n12 −3−loga b5 ) = log Factoring
4 (x + 2)(x − 2)
(loga a3 = 3)
x−7
1 = log Removing a factor of 1
= (8 loga m + 12 loga n − 3 − 5 loga b) x−2
4
Power rule
49. ln x − 3[ln(x − 5) + ln(x + 5)]
3 5 = ln x − 3 ln[(x − 5)(x + 5)] Product rule
= 2 loga m + 3 loga n − − loga b
4 4 = ln x − 3 ln(x2 − 25)
35. loga 75 + loga 2 = ln x − ln(x2 − 25)3 Power rule
x
= loga (75 · 2) Product rule = ln 2 Quotient rule
(x − 25)3
= loga 150
3 4
37. log 10, 000 − log 100 51. ln 4x6 − ln 2y 10
2 5
10, 000 3 4
= log Quotient rule = ln 22 x6 − ln 2y 10 Writing 4 as 22
100 2 5
= log 100 = ln(22 x6 )3/2 − ln(2y 10 )4/5 Power rule
=2 = ln(23 x9 ) − ln(24/5 y 8 )

39.
1 23 x9
log n + 3 log m = ln Quotient rule
2 24/5 y 8
= log n1/2 + log m3 Power rule 211/5 x9
= log n 1/2
m , or3
Product rule = ln
y8
3
√ √
log m n n1/2 = n
2
53. loga = loga 2 − loga 11 Quotient rule
11
≈ 0.301 − 1.041
≈ −0.74

Copyright © 2013 Pearson Education, Inc.


198 Chapter 5: Exponential and Logarithmic Functions

55. loga 98 = loga (72 · 2) 1


85. f (x) = x + 3 is of the form f (x) = mx + b, so it is a
2
= loga 72 + loga 2 Product rule linear function.
= 2 loga 7 + loga 2 Power rule
87. 5log5 8 = 2x
≈ 2(0.845) + 0.301
8 = 2x (aloga x = x)
≈ 1.991
4=x
loga 2 0.301 The solution is 4.
57. ≈ ≈ 0.356
loga 7 0.845 89. loga (x2 + xy + y 2 ) + loga (x − y)
59. logb 125 = logb 5 3
= loga [(x2 + xy + y 2 )(x − y)] Product rule
= 3 logb 5 Power rule = loga (x3 − y 3 ) Multiplying
≈ 3(1.609) x−y
≈ 4.827
91. loga 
x2 − y 2
1 x−y
61. logb = logb 1 − logb 6 Quotient rule = loga 2
6 (x − y 2 )1/2
= logb 1 − logb (2 · 3) = loga (x − y) − loga (x2 − y 2 )1/2 Quotient rule
1
= logb 1 − (logb 2 + logb 3) Product rule = loga (x − y) − loga (x2 − y 2 ) Power rule
2
= logb 1 − logb 2 − logb 3
1
≈ 0 − 0.693 − 1.099 = loga (x − y) − loga [(x + y)(x − y)]
2
≈ −1.792 1
= loga (x − y) − [loga (x + y) + loga (x − y)]
3 2
63. logb = logb 3 − logb b Quotient rule Product rule
b 1 1
≈ 1.099 − 1 = loga (x − y) − loga (x + y) − loga (x − y)
2 2
≈ 0.099
1 1
= loga (x − y) − loga (x + y)
65. logp p3 = 3 (loga ax = x) 2 2
4 2
y z 5
67. loge e|x−4| = |x − 4| (loga ax = x) 93. loga √
x3 z −2
4

69. 3log3 4x = 4x (aloga x = x) 


4 y2 z5
= loga
71. 10log w = w (aloga x = x) x3 z −2

2 7
73. ln e8t = 8t (loga ax = x) 4 y z
= loga
√ x3
75. logb b = logb b1/2  2 7 1/4
1 y z
= logb b Power rule = loga
2 x3
 2 7
1 1 y z
= ·1 (logb b = 1) =
4
loga
x3
Power rule
2
1 1
= = (loga y 2 z 7 − loga x3 ) Quotient rule
2 4
1
77. The degree of f (x) = 5 − x2 + x4 is 4, so the function is = (loga y 2 + loga z 7 − loga x3 ) Product rule
quartic. 4

3 
79. f (x) = − is of the form f (x) = mx + b with m = 0 and =
1
(2 loga y + 7 loga z − 3 loga x) Power rule
4
3 4
b = − , so it is a linear function. In fact, it is a constant 1
4 = (2 · 3 + 7 · 4 − 3 · 2)
function. 4
3 p(x) 1
81. f (x) = − is of the form f (x) = where p(x) and q(x) = · 28
x q(x) 4
are polynomials and q(x) is not the zero polynomial, so =7
f (x) is a rational function.
M
1 95. loga M − loga N = loga
83. The degree of f (x) = − x3 − 4x2 + 6x + 42 is 3, so the N
3 This is the quotient rule, so it is true.
function is cubic.

Copyright © 2013 Pearson Education, Inc.


Exercise Set 5.5 199

loga M 1 5. 2x = 33
97. = loga M = loga M 1/x . The statement is true
x x
by the power rule. log 2x = log 33 Taking the common
logarithm on both sides
99. loga 8x = loga 8 + loga x = loga x + loga 8. The statement x log 2 = log 33 Power rule
is true by the product rule and the commutative property log 33
of addition. x=
  log 2
1
101. loga = loga x−1 = −1 · loga x = −1 · 2 = −2 x≈
1.5185
x 0.3010
103. We use the change-of-base formula. x ≈ 5.044
log10 11·log11 12·log12 13 · · · The solution is 5.044.
log998 999·log999 1000 7. 54x−7 = 125
log 12 log 13 54x−7 = 53
= log10 11· 10 · 10 ···
log10 11 log10 12 4x − 7 = 3
log10 999 log10 1000
· 4x = 10
log10 998 log10 999
10 5
log10 11 log10 12 log10 999 x= =
= · ··· · log10 1000 4 2
log10 11 log10 12 log10 999 5
The solution is .
= log10 1000 2
2
=3 9. 27 = 35x · 9x
2
105. ln a − ln b + xy = 0 33 = 35x · (32 )x
ln a − ln b = −xy
2
33 = 35x · 32x
a 2
ln = −xy 33 = 35x+2x
b
Then, using the definition of a logarithm, we have 3 = 5x + 2x2
a
e−xy = . 0 = 2x2 + 5x − 3
b
 √  0 = (2x − 1)(x + 3)
107. x + x2 − 5 1
loga x= or x = −3
5 2
 √ √ 
x + x2 − 5 x − x2 − 5 1
= loga · √ The solutions are −3 and .
5 x − x2 − 5 2
   
5 1
= loga √ ) = loga √ 11. 84x = 70
5(x − x2 − 5 x − x2 − 5
√ log 84x = log 70
= loga 1 − loga (x − x2 − 5)
√ x log 84 = log 70
= − loga (x − x2 − 5)
log 70
x=
log 84
Exercise Set 5.5 1.8451
x≈
1.9243
1. 3x = 81 x ≈ 0.959
3x = 34 The solution is 0.959.
x=4 The exponents are the same. 13. 10−x = 52x
log 10−x = log 52x
The solution is 4.
−x = 2x log 5
3. 22x = 8 0 = x + 2x log 5
22x = 23 0 = x(1 + 2 log 5)
2x = 3 The exponents are the same. 0=x Dividing by 1 + 2 log 5
3 The solution is 0.
x=
2
3
The solution is .
2

Copyright © 2013 Pearson Education, Inc.


200 Chapter 5: Exponential and Logarithmic Functions

15. e−c = 52c 25. ex + e−x = 5


ln e−c = ln 52c e2x + 1 = 5ex Multiplying by ex
−c = 2c ln 5 e2x
− 5e + 1 = 0
x
This equation is quadratic
0 = c + 2c ln 5 in ex .

0 = c(1 + 2 ln 5) 5 ± 21
ex =
0=c Dividing by 1 + 2 ln 5 2
 √ 
The solution is 0. 5 ± 21
x = ln ≈ ±1.567
2
17. et = 1000 The solutions are −1.567 and 1.567.
ln et = ln 1000
27. 32x−1 = 5x
t = ln 1000 Using loga ax = x
log 32x−1 = log 5x
t ≈ 6.908
(2x − 1) log 3 = x log 5
The solution is 6.908.
2x log 3 − log 3 = x log 5
19. e−0.03t = 0.08 − log 3 = x log 5 − 2x log 3
ln e−0.03t = ln 0.08 − log 3 = x(log 5 − 2 log 3)
−0.03t = ln 0.08 − log 3
=x
t=
ln 0.08 log 5 − 2 log 3
−0.03 1.869 ≈ x
−2.5257 The solution is 1.869.
t≈
−0.03
t ≈ 84.191 29. 2ex = 5 − e−x
The solution is 84.191. 2e − 5 + e−x = 0
x

ex (2ex − 5 + e−x ) = ex · 0 Multiplying by ex


21. 3x = 2x−1 2x
2e − 5e + 1 = 0
x
ln 3x = ln 2x−1
Let u = ex .
x ln 3 = (x − 1) ln 2
2u2 − 5u + 1 = 0 Substituting
x ln 3 = x ln 2 − ln 2
a = 2, b = −5, c = 1
ln 2 = x ln 2 − x ln 3 √
−b ± b2 − 4ac
ln 2 = x(ln 2 − ln 3) u=
2a
ln 2 
ln 2 − ln 3
=x −(−5) ± (−5)2 − 4 · 2 · 1
u=
0.6931 2·2
≈x √
0.6931 − 1.0986 5 ± 17
u=
−1.710 ≈x 4
The solution is −1.710. Replace u with ex .
√ √
5 − 17 5 + 17
23. (3.9)x = 48 ex = or ex =
4 4
log(3.9)x = log 48  √   √ 
x 5 − 17 x 5 + 17
x log 3.9 = log 48 ln e = ln or ln e = ln
4 4
x=
log 48 x ≈ −1.518 or x ≈ 0.825
log 3.9 The solutions are −1.518 and 0.825.
1.6812
x≈ 31. log5 x = 4
0.5911
x ≈ 2.844 x = 54 Writing an equivalent
The solution is 2.844. exponential equation
x = 625
The solution is 625.

33. log x = −4 The base is 10.


−4
x = 10 , or 0.0001
The solution is 0.0001.

Copyright © 2013 Pearson Education, Inc.


Exercise Set 5.5 201

35. ln x = 1 The base is e. x+5 = 0 or x − 4 = 0


x=e =e1 x = −5 or x=4
The solution is e. Check: For −5:
log2 (x + 20) − log2 (x + 2) = log2 x
1
37. log64 =x
4 log2 (−5 + 20) − log2 (−5 + 2) ? log2 (−5)
1 |
= 64x
4 The number −5 does not check, because negative numbers
1 do not have logarithms.
= (43 )x
4 For 4:
4−1 = 43x
log2 (x + 20) − log2 (x + 2) = log2 x
−1 = 3x
1 log2 (4 + 20) − log2 (4 + 2) ? log2 4
− =x 
3 log2 24 − log2 6 

24 
1
The solution is − .
3 log2 
6 

39. log2 (10 + 3x) = 5 log2 4  log2 4 TRUE
25 = 10 + 3x The solution is 4.
32 = 10 + 3x 45. log8 (x + 1) − log8 x =2
22 = 3x  
x+1
22 log8 =2 Quotient rule
=x x
3 x+1
22 = 82
The answer checks. The solution is . x
3
x+1
41. log x + log(x − 9) = 1 The base is 10. = 64
x
log10 [x(x − 9)] = 1 x+1 = 64x
x(x − 9) = 101 1 = 63x
x2 − 9x = 10 1
=x
x2 − 9x − 10 = 0 63
1
(x − 10)(x + 1) = 0 The answer checks. The solution is .
63
x = 10 or x = −1
47. log x + log(x + 4) = log 12
Check: For 10:
log x + log(x − 9) = 1 log x(x + 4) = log 12
x(x + 4) = 12 Using the property of
log 10 + log(10 − 9) ? 1 logarithmic equality
log 10 + log 1  x2 + 4x = 12

1+0 
 x + 4x − 12 = 0
2
1  1 TRUE
(x + 6)(x − 2) = 0
For −1:
log x + log(x − 9) = 1 x+6 = 0 or x − 2 = 0
x = −6 or x=2
log(−1) + log(−1 − 9) ? 1
Check: For −6:
|
The number −1 does not check, because negative numbers log x + log(x + 4) = log 12
do not have logarithms. The solution is 10.
log(−6) + log(−6 + 4) ? log 12
43. log2 (x + 20) − log2 (x + 2) = log2 x |
x + 20 The number −6 does not check, because negative numbers
log2 = log2 x do not have logarithms.
x+2
x + 20
= x Using the property of
x+2 logarithmic equality
x + 20 = x2 + 2x Multiplying by
x+2
0 = x2 + x − 20
0 = (x + 5)(x − 4)

Copyright © 2013 Pearson Education, Inc.


202 Chapter 5: Exponential and Logarithmic Functions

For 2: 57. log6 x = 1 − log6 (x − 5)


log x + log(x + 4) = log 12 log6 x + log6 (x − 5) = 1
log 2 + log(2 + 4) ? log 12 log6 x(x − 5) = 1
 61 = x(x − 5)
log 2 + log 6 

log(2 · 6)  6 = x2 − 5x
 0 = x2 − 5x − 6
log 12  log 12 TRUE
The solution is 2. 0 = (x + 1)(x − 6)
x+1 = 0 or x − 6 = 0
49. log(x+8)−log(x+1) = log 6
x = −1 or x=6
x+8
log
x+1
= log 6 Quotient rule The number −1 does not check, but 6 does. The answer
x+8 is 6.
=6 Using the property of
x+1 logarithmic equality 59. 9x−1 = 100(3x )
x + 8 = 6x + 6 Multiplying by x+1 2 x−1
(3 ) = 100(3x )
2 = 5x
32x−2 = 100(3x )
2
=x 32x−2
5 = 100
2 3x
The answer checks. The solution is . 3 x−2
= 100
5
51. log4 (x + 3) + log4 (x − 3) = 2 log 3x−2 = log 100

log4 [(x + 3)(x − 3)] = 2 Product rule (x − 2) log 3 = 2


2
(x + 3)(x − 3) = 42 x−2 =
log 3
x2 − 9 = 16 2
x = 2+
x2 = 25 log 3
x = ±5 x ≈ 6.192
The number 5 checks, but −5 does not. The solution is 5. The solution is 6.192.

53. log(2x + 1) − log(x − 2) =1 61. ex − 2 = −e−x


  1
2x + 1 ex − 2 = − x
log =1 Quotient rule e
x−2
2x + 1 e2x − 2ex = −1 Multiplying by ex
= 101 = 10 x
e2x − 2e + 1 = 0
x−2
2x + 1 = 10x − 20 Let u = ex .
Multiplying by x − 2 u2 − 2u + 1 = 0
21 = 8x (u − 1)(u − 1) = 0
21 u − 1 = 0 or u − 1 = 0
=x
8 u = 1 or u=1
21
The answer checks. The solution is . ex = 1 or ex = 1 Replacing u with ex
8
x = 0 or x=0
55. ln(x + 8) + ln(x − 1) = 2 ln x
The solution is 0.
ln(x + 8)(x − 1) = ln x2
(x + 8)(x − 1) = x2 Using the property of 63. 2x − 5 = 3x + 1
logarithmic equality Graph y1 = 2x − 5 and y2 = 3x + 1 and find the first
x2 + 7x − 8 = x2 coordinates of the points of intersection using the Intersect
7x − 8 = 0 feature. The solutions are −1.911 and 4.222.
7x = 8 65. xe3x − 1 = 3
8
x= Graph y1 = xe3x − 1 and y2 = 3 and find the first coordi-
7 nate of the point of intersection using the Intersect feature.
8 The solution is 0.621.
The answer checks. The solution is .
7

Copyright © 2013 Pearson Education, Inc.


Exercise Set 5.5 203

67. 5e5x + 10 = 3x + 40 81. ex + e−x


=3
ex − e−x
Graph y1 = 5e5x + 10 and y2 = 3x + 40 and find the first
e + e = 3ex − 3e−x
x −x
Multiplying by ex − e−x
coordinates of the points of intersection using the Intersect
feature. The solutions are −10 and 0.366. 4e−x = 2ex Subtracting ex and adding 3e−x

69. log8 x + log8 (x + 2) = 2


2e−x = ex
log x log(x + 2)
Graph y1 = + and y2 = 2 and find the 2 = e2x Multiplying by ex
log 8 log 8 2x
first coordinate of the point of intersection using the inter- ln 2 = ln e
sect feature. The solution is 7.062. ln 2 = 2x
71. log5 (x + 7) − log5 (2x − 3) = 1 ln 2
=x
log(x + 7) log(2x − 3) 2
Graph y1 = − and y2 = 1 and find 0.347 ≈ x
log 5 log 5
the first coordinate of the point of intersection using the The solution is 0.347.
Intersect feature. The solution is 2.444. √ √
83. ln x = ln x
73. Solving the first equation for y, we get √ 1
12.4 − 2.3x 12.4 − 2.3x ln x = ln x Power rule
y= . Graph y1 = and 2
3.8 3.8 1
y2 = 1.1 ln(x − 2.05) and use the Intersect feature to find ln x = (ln x)2 Squaring both sides
the point of intersection. It is (4.093, 0.786). 4
1
75. Graph y1 = 2.3 ln(x + 10.7) and y2 = 10e−0.007x and use
2 0 = (ln x)2 − ln x
4
the Intersect feature to find the point of intersection. It is Let u = ln x and substitute.
(7.586, 6.684). 1 2
u −u = 0
77. g(x) = x2 − 6 4
 
1
a) −
b
=−
0
=0 u u−1 = 0
2a 2·1 4
g(0) = 02 − 6 = −6 1
u=0 or u − 1 = 0
4
The vertex is (0, −6).
1
b) The axis of symmetry is x = 0. u=0 or u=1
4
c) Since the coefficient of the x2 -term is positive, the u=0 or u=4
function has a minimum value. It is the second coor-
ln x = 0 or ln x = 4
dinate of the vertex, −6, and it occurs when x = 0.
x = e0 = 1 or x = e4 ≈ 54.598
79. G(x) = −2x − 4x − 7
2
Both answers check. The solutions are 1 and e4 , or 1 and
b −4 54.598.
a) − =− = −1
2a 2(−2)
85. (log3 x)2 − log3 x2 = 3
G(−1) = −2(−1)2 − 4(−1) − 7 = −5
(log3 x)2 − 2 log3 x − 3 = 0
The vertex is (−1, −5).
Let u = log3 x and substitute:
b) The axis of symmetry is x = −1.
u2 − 2u − 3 = 0
c) Since the coefficient of the x2 -term is negative, the
function has a maximum value. It is the second (u − 3)(u + 1) = 0
coordinate of the vertex, −5, and it occurs when u=3 or u = −1
x = −1. log3 x = 3 or log3 x = −1
x = 33 or x = 3−1
1
x = 27 or x=
3
1
Both answers check. The solutions are and 27.
3

Copyright © 2013 Pearson Education, Inc.


204 Chapter 5: Exponential and Logarithmic Functions

87. ln x2 = (ln x)2 95. a = log8 225, so 8a = 225 = 152 .


2 ln x = (ln x)2 b = log2 15, so 2b = 15.
0 = (ln x) − 2 ln x
2
Then 8a = (2b )2
Let u = ln x and substitute. (2 ) = 22b
3 a

0 = u2 − 2u 23a = 22b
0 = u(u − 2) 3a = 2b
u = 0 or u=2 2
a = b.
ln x = 0 or ln x = 2 3
x = 1 or x = e2 ≈ 7.389
Both answers check. The solutions are 1 and e2 , or 1 and Exercise Set 5.6
7.389.
1. a) Substitute 5.9 for P0 and 0.0176 for k in P (t) =
89. 52x − 3 · 5x + 2 = 0 P0 ekt . We have:
(5x − 1)(5x − 2) = 0 This equation is P (t) = 5.9e0.0176t , where P (t) is in millions and t is
quadratic in 5x . the number of years after 2010.
5x = 1 or 5x = 2 b) In 2016, t = 2016 − 2010 = 6.
x
log 5 = log 1 or log 5 = log 2 x P (6) = 5.9e0.0176(6) ≈ 6.6 million
x log 5 = 0 or x log 5 = log 2 c) Substitute 7 for P (t) and solve for t.
log 2 7 = 5.9e0.0176t
x=0 or x= ≈ 0.431
log 5 7
= e0.0176t
The solutions are 0 and 0.431. 5.9
7
ln = ln e0.0176t
91. ln xln x = 4 5.9
ln x · ln x = 4 7
ln = 0.0176t
(ln x)2 = 4 5.9
ln x = ±2 7
ln
5.9 =t
ln x = −2 or ln x = 2 0.0176
x = e−2 or x = e2 9.7 ≈t
x ≈ 0.135 or x ≈ 7.389 The population will be 7 million about 9.7 yr after
−2 2 2010.
Both answers check. The solutions are e and e , or 0.135
ln 2
and 7.389. d) T = ≈ 39.4 yr
0.0176

93. (e2x · e−5x )−4 ln 2
= e7 3. a) k = ≈ 0.98%
ex ÷ e−x 70.7
√ ln 2
e12x b) k= ≈ 1.51%
x−(−x)
= e7 45.9
e
ln 2
e6x c) T = ≈ 21.6 yr
= e7 0.0321
e2x ln 2
e4x = e7 d) T = ≈ 57.8 yr
0.012
4x = 7 ln 2
e) k= ≈ 0.28%
7 248
x=
4 ln 2
f) T = ≈ 29.9 per yr
7 0.0232
The solution is .
4 ln 2
g) T = ≈ 49.5 yr
0.014
ln 2
h) k= ≈ 0.66%
105.0
ln 2
i) k= ≈ 2.04%
34.0
ln 2
j) T = ≈ 37.7 yr
0.0184

Copyright © 2013 Pearson Education, Inc.


Exercise Set 5.6 205

5. First note that 32,961,561,600 square yards = 32,961,561.6 13. a) C(t) = C0 e−kt
thousand square yards.
2.92 = 4.85e−k·4
P (t) = P0 ekt 2.92
32, 961, 561.6 = 10, 032, 619e0.00787t = e−4k
4.85
32, 961, 561.6  
= e0.00787t 2.92
10, 032, 619 ln = ln e−4k
  4.85
32, 961, 561.6  
ln = ln e0.00787t 2.92
10, 032, 619 ln = −4k
  4.85
32, 961, 561.6  
ln = 0.00787t 2.92
10, 032, 619 ln
  4.85
=k
32, 961, 561.6 −4
ln
10, 032, 619
=t 0.1268 ≈k
0.00787
Then we have C(t) = 4.85e−0.1268t , where C is in
151 ≈t dollars and t is the number of years since 2009.
There will be one person for every 1000 square yards of b) In 2015, t = 2015 − 2009 = 6.
land about 151 yr after 2010.
C(6) = 4.85e−0.1268(6) ≈ $2.27
7. a) Substitute 10,000 for P0 and 5.4%, or 0.054 for k. In 2018, t = 2018 − 2009 = 9.
P (t) = 10, 000e0.054t C(9) = 4.85e−0.1268(9) ≈ $1.55
b) P (1) = 10, 000e0.054(1) ≈ $10, 554.85 c) 1.85 = 4.85e−0.1268t
P (2) = 10, 000e 0.054(2)
≈ $11, 140.48 1.85
= e−0.1268t
P (5) = 10, 000e 0.054(5)
≈ $13, 099.64 4.85
 
1.85
P (10) = 10, 000e0.054(10) ≈ $17, 160.07 ln = ln e−0.1268t
4.85
ln 2  
c) T = ≈ 12.8 yr 1.85
0.054 ln = −0.1268t
4.85
 
9. We use the function found in Example 5. If the bones have 1.85
lost 77.2% of their carbon-14 from an initial amount P0 , ln
4.85
then 22.8%P0 , or 0.228P0 remains. We substitute in the =t
−0.1268
function.
8 ≈t
0.228P0 = P0 e−0.00012t
At the given rate of decay, the average cost per watt
0.228 = e−0.00012t will be $1.85 about 8 yr after 2009, or in 2017.
ln 0.228 = ln e−0.00012t
15. a) In 2010, t = 2010 − 1980 = 30.
ln 0.228 = −0.00012t
C(t) = C0 ekt
ln 0.228
=t 565 = 28ek·30
−0.00012
12, 320 ≈ t 565
= e30k
28
The bones are about 12,320 years old.  
565
ln 2 ln = ln e30k
11. a) K = ≈ 0.224, or 22.4% per min 28
3.1  
565
ln 2 ln = 30k
b) k= ≈ 0.031, or 3.1% per yr 28
22.3  
ln 2 565
c) T = ≈ 60.3 days ln
28
0.0115 =k
ln 2 30
d) T = ≈ 10.7 yr 0.1002 ≈k
0.065
ln 2
e) k= ≈ 0.024, or 2.4% per yr C(t) = 28e0.1002t , where t is the number of years
29.1
ln 2 after 1980.
f) k= ≈ 0.010, or 1.0% per yr b) In 2013, t = 2013 − 1980 = 33.
70.0
ln 2 C(33) = 28e0.1002(33) ≈ 764 cable TV networks
g) k= ≈ 0.000029, or 0.0029% per yr
24, 100 In 2017, t = 2017 − 1980 = 37.
C(37) = 28e0.1002(37) ≈ 1141 cable TV networks

Copyright © 2013 Pearson Education, Inc.


206 Chapter 5: Exponential and Logarithmic Functions

c) 1500 = 28e0.1002t 21. To find k we substitute 43 for T1 , 68 for T0 , 12 for t, and


1500 55 for T (t) and solve for k.
= e0.1002t
28 55 = 68 + (43 − 68)e−12k
 
1500 −13 = −25e−12k
ln = ln e0.1002t
28 0.52 = e−12k
 
1500 ln 0.52 = ln e−12k
ln = 0.1002t
28 ln 0.52 = −12k
 
1500 0.0545 ≈ k
ln
28
=t The function is T (t) = 68 − 25e−0.0545t .
0.1002
40 ≈t Now we find T (20).
There will be 1500 cable TV networks about 40 yr T (20) = 68 − 25e−0.0545(20) ≈ 59.6◦ F
after 1980, or in 2020. 23. The data have the pattern of a decreasing exponential
function, so function (d) might be used as a model.
17. a)
25. The data have a parabolic pattern, so function (a) might
be used as a model.
27. The data have a logarithmic pattern, so function (e) might
be used as a model.
3500 29. a) y = 0.1377082721(1.023820625)x , where x is the
b) N (0) = ≈ 167
1 + 19.9e−0.6(0) number of years after 1900.
3500
≈ 500 r ≈ 0.9824, so the function is a good fit.
c) N (2) =
1 + 19.9e−0.6(2) b)
3500
N (5) = ≈ 1758
1 + 19.9e−0.6(5)
3500
N (8) = ≈ 3007
1 + 19.9e−0.6(8)
3500
N (12) = ≈ 3449
1 + 19.9e−0.6(12) c) In 2007, x = 2007 − 1900 = 107.
3500
N (16) = ≈ 3495 y = 0.1377082721(1.023820625)107 ≈ 1.7%
1 + 19.9e−0.6(16)
d) As t → ∞, N (t) → 3500; the number of people In 2015, x = 2015 − 1900 = 115.
infected approaches 3500 but never actually reaches y = 0.1377082721(1.023820625)115 ≈ 2.1%
it. In 2020, x = 2020 − 1900 = 120.
y = 0.1377082721(1.023820625)120 ≈ 2.3%
19. To find k we substitute 105 for T1 , 0 for T0 , 5 for t, and
70 for T (t) and solve for k. 31. a) y = 5.600477432(1.030576807)x , where y is in mil-
−5k
70 = 0 + (105 − 0)e lions and x is the number of years after 1980.
70 = 105e−5k b)
70
= e−5k
105
70
ln = ln e−5k
105
70
ln = −5k c) In 2020, x = 2020 − 1980 = 40.
105
70 y = 5.600477432(1.030576807)40 ≈ 18.7 million un-
ln occupied homes
105 =k
−5 33. a) y = 2079.930585(1.079796876)x , where x is the
0.081 ≈k number of years after 2000.
The function is T (t) = 105e−0.081t . b) In 2012, x = 2012 − 2000 = 12.
Now we find T (10). y = 2079.930585(1.079796876)12 ≈ $5226
−0.081(10) ◦
T (10) = 105e ≈ 46.7 F In 2015, x = 2015 − 2000 = 15.
y = 2079.930585(1.079796876)15 ≈ $6579

Copyright © 2013 Pearson Education, Inc.


Chapter 5 Review Exercises 207

c) Graph y1 = 2079.930585(1.079796876)x and y2 = 9. The graph of f (x) = −|x|+3 is shown below. The function
7300, and find the first coordinate of the point of is not one-to-one, because there are many horizontal lines
intersection of the graphs. We find that the price that cross the graph more than once.
per acre will exceed $7300 about 16 yr after 2000,
y
or in 2016.
5 f ( x ) = – |x | + 3
35. Multiplication principle for inequalities 4
3
37. Principle of zero products 2
1
39. Power rule —5 —4 —3 —2 —1 1 2 3 4 5 x
—1
41. P (t) = P0 ekt —2
—3
0.07(18)
50, 000 = P0 e —4

50, 000 —5
= P0
e0.07(18)
3
$14, 182.70 ≈ P0 11. The graph of f (x) = 2x − is shown below. The function
4
43. a) At 1 m: I = I0 e−1.4(1) ≈ 0.247I0 is one-to-one, because no horizontal line crosses the graph
more than once.
24.7% of I0 remains.
y
At 3 m: I = I0 e−1.4(3) ≈ 0.015I0
5
1.5% of I0 remains. f (x ) = 2 x – –34
4

−1.4(5)
≈ 0.0009I0
3
At 5 m: I = I0 e 2

0.09% of I0 remains. 1

−1.4(50) −31 x
≈ (3.98 × 10
—5 —4 —3 —2 —1 1 2 3 4 5
At 50 m: I = I0 e )I0 —1

Now, 3.98 × 10−31 = (3.98 × 10−29 ) × 10−2 , so


—2
—3

(3.98 × 10−29 )% remains. —4


—5
b) I = I0 e−1.4(10) ≈ 0.0000008I0
Thus, 0.00008% remains.
13. a) The graph of f (x) = 2−3x is shown below. It passes
the horizontal-line test, so the function is one-to-
45. y = aex one.
ln y = ln(aex )
y
ln y = ln a + ln ex
5
ln y = ln a + x 4 f (x ) = 2 – 3 x
3
Y = x + ln a 2
This function is of the form y = mx + b, so it is linear. 1

—5 —4 —3 —2 —1 1 2 3 4 5 x
—1
—2
Chapter 5 Review Exercises —3
—4
—5

1. The statement is true. See page 395 in the text.


b) Replace f (x) with y: y = 2 − 3x
3. The graph of f −1 is a reflection of the graph of f across
y = x, so the statement is false. Interchange x and y: x = 2 − 3y
−x + 2
5. The statement is false. The range of y = ax , for instance, Solve for y: y =
3
is (0, ∞). −x + 2
Replace y with f −1 (x): f −1 (x) =
7. We interchange the first and second coordinates of each 3
pair to find the inverse of the relation. It is
{(−2.7, 1.3), (−3, 8), (3, −5), (−3, 6), (−5, 7)}.

Copyright © 2013 Pearson Education, Inc.


208 Chapter 5: Exponential and Logarithmic Functions

√ y
15. a) The graph of f (x) = x − 6 is shown below. It
passes the horizontal-line test, so the function is one- 4
to-one. yx
f 1 2
y
x
4 2 2 4
10 2
8 f (x ) = √ x – 6
4
6
f
4
2

—10 —8 —6 —4 —2 2 4 6 8 10 x 23. Since f (f −1 (x)) = x, then f (f −1 (657)) = 657.


—2
—4 25.
—6
y
—8
—10
4 1
f(x)  
3 ( )x
2

b) Replace f (x) with y: y = x−6 4 2 2 4 x
√ 2
Interchange x and y: x = y−6
4
Solve for y: x2 = y − 6
x2 + 6 = y
27.
Replace y with f −1 (x): f −1 (x) = x2 + 6, for all x y
in the range of f (x), or f −1 (x) = x2 + 6, x ≥ 0. 4

17. a) The graph of f (x) = 3x2 + 2x − 1 is shown below. 2


It is not one-to-one since there are many horizon-
4 2 2 4 x
tal lines that cross the graph more than once. The 2
x
function does not have an inverse that is a function. f(x)  e
4
y

5
4 29.
y
3
2 4
1
2
—5 —4 —3 —2 —1 1 2 3 4 5 x
—1
4 2 2 4 x
—2 f (x ) = 3 x 2 + 2x – 1
—3 2
1
—4 f(x)  
2 ln x
4
—5

19. We find (f −1 ◦f )(x) and (f ◦f −1 )(x) and check to see that 31. f (x) = ex−3
each is x. This is the graph of f (x) = ex shifted right 3 units. The
(f −1 ◦ f )(x) = f −1 (f (x)) = f −1 (6x − 5) = correct choice is (c).
(6x − 5) + 5 6x 33. f (x) = − log3 (x + 1)
= =x
6 6
  This is the graph of log3 x shifted left 1 unit and reflected
x+5
(f ◦ f −1 )(x) = f (f −1 (x)) = f = across the y-axis. The correct choice is (b).
6
  35. f (x) = 3(1 − e−x ), x ≥ 0
x+5
6 −5=x+5−5=x This is the graph of f (x) = ex reflected across the y-axis,
6
reflected across the x-axis, shifted up 1 unit, and stretched
by a factor of 3. The correct choice is (e).
21. Replace f (x) with y: y = 2 − 5x 37. log5 125 = 3 because the exponent to which we raise 5 to
Interchange x and y: x = 2 − 5y get 125 is 3.
2−x
Solve for y: y = 39. ln e = 1 because the exponent to which we raise e to get e
5 is 1.
2−x
Replace y with f −1 (x): f −1 (x) = 1
5 41. log 101/4 = because the exponent to which we raise 10
4
The domain and range of f are (−∞, ∞), so the domain 1
and range of f −1 are also (−∞, ∞). to get 101/4 is .
4

Copyright © 2013 Pearson Education, Inc.


Chapter 5 Review Exercises 209

43. log 1 = 0 because the exponent to which we raise 10 to get 73. log16 4 = x
1 is 0. 16x = 4
√ 1
45. log2 2 = log2 21/3 = because the exponent to which we
3
(42 )x = 41
3
1 42x = 41
raise 2 to get 21/3 is .
3 2x = 1
47. log4 x = 2 ⇒ 4 = x 2
x=
1
2
1 1
49. 4−3 = ⇒ log4 = −3 The solution is
1
.
64 64 2
51. log 11 ≈ 1.0414 75. log2 x + log2 (x − 2) = 3
53. ln 3 ≈ 1.0986 log2 x(x − 2) = 3

55. log(−3) does not exist. (The calculator gives an error mes- x(x − 2) = 23
sage.) x2 − 2x = 8
log 24 x2 − 2x − 8 = 0
57. log5 24 = ≈ 1.9746
log 5 (x + 2)(x − 4) = 0
1 x+2 = 0 or x − 4 = 0
59. 3 logb x − 4 logb y + logb z
2 x = −2 or x=4
= logb x − logb y + logb z 1/2
3 4
The number 4 checks, but −2 does not. The solution is 4.

x3 z 1/2 x3 z
= logb , or log b 77. log x2 = log x
y4 y4
√ x2 = x
4
61. ln wr2 = ln(wr2 )1/4 x2 − x = 0
1 x(x − 1) = 0
= ln wr2
4
1 x = 0 or x − 1 = 0
= (ln w + ln r2 ) x = 0 or x=1
4
1 The number 1 checks, but 0 does not. The solution is 1.
= (ln w + 2 ln r)
4  nt
1
= ln w + ln r
1 79. a) A(t) = P 1 + r
4 2 n
 4t
  A(t) = 30, 000 1 +
0.042
Substituting
63. 6 4
loga 3 = loga
2 A(t) = 30, 000(1.0105) 4t

= loga 6 − loga 2
≈ 0.778 − 0.301 b) A(0) = 30, 000(1.0105)4·0 = $30, 000
≈ 0.477 A(6) = 30, 000(1.0105)4·6 ≈ $38, 547.20
A(12) = 30, 000(1.0105)4·12 ≈ $49, 529.56
1
65. loga = loga 5−1 A(18) = 30, 000(1.0105)4·18 ≈ $63, 640.87
5
= − loga 5 ln 2
81. T = ≈ 8.1 years
≈ −0.699 0.086
67. ln e−5k = −5k (loga ax = x) 83. P (t) = P0 ekt
69. log4 x = 2 0.73P0 = P0 e−0.00012t
x = 42 = 16 0.73 = e−0.00012t
The solution is 16. ln 0.73 = ln e−0.00012t
ln 0.73 = −0.00012t
71. ex = 80
ln 0.73
ln ex = ln 80 =t
−0.00012
x = ln 80 2623 ≈ t
x ≈ 4.382 The skeleton is about 2623 years old.
The solution is 4.382.

Copyright © 2013 Pearson Education, Inc.


210 Chapter 5: Exponential and Logarithmic Functions

106.3 · I0 97. | log4 x| = 3


85. R = log = log 106.3 = 6.3
I0 log4 x = −3 or log4 x = 3
87. a) We substitute 353.823 for P , since P is in thousands. x = 4−3 or x = 43
1
W (353.823) = 0.37 ln 353.823 + 0.05 x= or x = 64
64
≈ 2.2 ft/sec 1
Both answers check. The solutions are and 64.
b) We substitute 3.4 for W and solve for P . 64

3.4 = 3.7 ln P + 0.05 99. 5 x
= 625
3.35 = 0.37 ln P √
5 x = 54
3.35 √
= ln P x=4
0.37
e3.35/0.37 = P x = 16

P ≈ 8553.143 101. Reflect the graph of f (x) = ln x across the line y = x to


The population is about 8553.143 thousand, or obtain the graph of h(x) = ex . Then shift this graph 2
8,553,143. (Answers may vary due to rounding dif- units right to obtain the graph of g(x) = ex−2 .
ferences.) 103. loga ab3 = (loga a)(loga b3 ). If the first step had been cor-
89. a) P (t) = 6.188e0.01985t , where P (t) is in millions and rect, then so would the second step. The correct procedure
follows.
t is the number of years after 2011.
loga ab3 = loga a + loga b3 = 1 + 3 loga b
b) In 2013, t = 2013 − 2011 = 2.
P (2) = 6.188e0.01985(2) ≈ 6.439 million
In 2015, t = 2015 − 2011 = 4. Chapter 5 Test
P (4) = 6.188e0.01985(4) ≈ 6.699 million
c) 10 = 6.188e0.01985t 1. We interchange the first and second coordinates of each
10 0.01985t
pair to find the inverse of the relation. It is
=e {(5, −2), (3, 4)(−1, 0), (−3, −6)}.
6.188
 
10 2. The function is not one-to-one, because there are many
ln = ln e0.01985t
6.188 horizontal lines that cross the graph more than once.
 
10 3. The function is one-to-one, because no horizontal line
ln = 0.01985t
6.188 crosses the graph more than once.
 
10
ln 4. a) The graph of f (x) = x3 +1 is shown below. It passes
6.188
=t the horizontal-line test, so the function is one-to-
0.01985
one.
24 ≈t
y
The population will be 10 million about 24 yr after
2011. 5

ln 2 4
d) T = ≈ 34.9 yr 3
0.01985 2

4 + 3x x+4 1
91. We graph y1 = , y2 = , and y3 = x and ob-
x−2 x−3 —5 —4 —3 —2 —1
—1
1 2 3 4 5 x

serve that the graphs of y1 and y2 are not reflections of —2 f (x ) = x 3+1


each other across the third line, y = x. Thus f (x) and —3
g(x) are not inverses. —4
—5
x−3
93. The graph of f (x) = e + 2 is a translation of the graph
of y = ex right three units and up 2 units. The horizontal b) Replace f (x) with y: y = x3 + 1
asymptote of y = ex is y = 0, so the horizontal asymptote
of f (x) = ex−3 + 2 is translated up 2 units from y = 0. Interchange x and y: x = y 3 + 1
Thus, it is y = 2, and answer D is correct. Solve for y: y 3 = x − 1

95. The graph of f (x) = 2x−2 is the graph of g(x) = 2x shifted y = 3x−1

2 units to the right. Thus D is the correct graph. Replace y with f −1 (x): f −1 (x) = 3
x−1

Copyright © 2013 Pearson Education, Inc.


Chapter 5 Test 211

y
5. a) The graph of f (x) = 1 − x is shown below. It passes
the horizontal-line test, so the function is one-to- 5
4
one.
3

y 2
1
5 x
—5 —4 —3 —2 —1 1 2 3 4 5
4 —1
f (x ) = 1 – x
3 —2
2 —3
1 —4

—5 —4 —3 —2 —1 1 2 3 4 5 x x 2+x – 3
f (x ) = —5
—1
—2
—3 8. We find (f −1 ◦f )(x) and (f ◦f −1 )(x) and check to see that
—4
each is x.
—5
(f −1 ◦ f )(x) = f −1 (f (x)) = f −1 (−4x + 3) =
b) Replace f (x) with y: y = 1 − x 3 − (−4x + 3) 3 + 4x − 3 4x
= = =x
Interchange x and y: x = 1 − y 4 4 4
 
Solve for y: y = 1 − x 3−x
(f ◦ f −1 )(x) = f (f −1 (x)) = f =
4
Replace y with f −1 (x): f −1 (x) = 1 − x  
3−x
x −4 + 3 = −3 + x + 3 = x
6. a) The graph of f (x) = is shown below. It passes 4
2−x
the horizontal-line test, so the function is one-to- 1
9. Replace f (x) with y: y =
one. x−4
y
1
Interchange x and y: x =
y−4
5
4 Solve for y: x(y − 4) = 1
xy − 4x = 1
3
2
1 xy = 4x + 1
—5 —4 —3 —2 —1 1 2 3 4 5 x 4x + 1
—1
y=
—2 x
x —3 4x + 1
Replace y with f −1 (x): f −1 (x) =
f (x ) = 2–––
–x
—4
—5
x
The domain of f (x) is (−∞, 4) ∪ (4, ∞) and the range
of f (x) is (−∞, 0) ∪ (0, ∞). Thus, the domain of f −1 is
x
b) Replace f (x) with y: y = (−∞, 0) ∪ (0, ∞) and the range of f −1 is (−∞, 4) ∪ (4, ∞).
2−x
y
Interchange x and y: x = y y⫽x
2−y 6
f ⫺1
Solve for y: (2 − y)x = y 4

2x − xy = y 2

xy + y = 2x ⫺4 ⫺2 2 4 6 x
y(x + 1) = 2x ⫺2

2x ⫺4 f
y=
x+1
2x 10.
Replace y with f −1 (x): f −1 (x) = y
x+1
4
f (x)  4x
7. a) The graph of f (x) = x2 + x − 3 is shown below. It is
2
not one-to-one since there are many horizontal lines
that cross the graph more than once. The function 4 2 2 4 x
does not have an inverse that is a function. 2

4

Copyright © 2013 Pearson Education, Inc.


212 Chapter 5: Exponential and Logarithmic Functions


11.
y
24. ln x2 y = ln(x2 y)1/5
5

1
4
f(x)  log x
= ln x2 y
5
2 1
= (ln x2 + ln y)
4 2 2 4 x
5
2 1
= (2 ln x + ln y)
4
5
2 1
= ln x + ln y
5 5
12.  
y
25. loga 4 = loga 8
4
2
= loga 8 − loga 2
2
≈ 0.984 − 0.328
4 2 2 4 x ≈ 0.656
2
f (x)  e x  3
4 26. ln e−4t = −4t (loga ax = x)
27. log25 5 = x
13. 25x = 5
y

4
(52 )x = 51
2 52x = 51
2x = 1
4 2 2 4 x
2
1
f (x)  ln(x  2) x=
2
4
1
The solution is .
2
14. log 0.00001 = −5 because the exponent to which we raise 28. log3 x + log3 (x + 8) = 2
10 to get 0.00001 is −5.
log3 x(x + 8) = 2
15. ln e = 1 because the exponent to which we raise e to get e x(x + 8) = 32
is 1.
x2 + 8x = 9
16. ln 1 = 0 because the exponent to which we raise e to get 1 x2 + 8x − 9 = 0
is 0.
(x + 9)(x − 1) = 0
√ 1
17. log4 4 = log4 41/5 = because the exponent to which we
5
x = −9 or x = 1
5
1 The number 1 checks, but −9 does not. The solution is 1.
raise 4 to get 41/5 is .
5 29. 34−x = 27x
18. ln x = 4 ⇒ x = e 4
34−x = (33 )x
19. 3x = 5.4 ⇒ x = log3 5.4 34−x = 33x
4 − x = 3x
20. ln 16 ≈ 2.7726
4 = 4x
21. log 0.293 ≈ −0.5331 x=1
log 10 The solution is 1.
22. log6 10 = ≈ 1.2851
log 6
30. ex = 65
1
23. 2 loga x − loga y + loga z ln ex = ln 65
2
x = ln 65
= loga x2 − loga y + loga z 1/2
√ x ≈ 4.174
x2 z 1/2 x2 z
= loga , or loga The solution is 4.174.
y y
106.6 · I0
31. R = log = log 106.6 = 6.6
I0

Copyright © 2013 Pearson Education, Inc.


Chapter 5 Test 213

ln 2
32. k = ≈ 0.0154 ≈ 1.54%
45
33. a) 1144.54 = 1000e3k
1.14454 = e3k
ln 1.14454 = ln e3k
ln 1.14454 = 3k
ln 1.14454
=k
3
0.045 ≈ k
The interest rate is about 4.5%.
b) P (t) = 1000e0.045t
c) P (8) = 1000e0.045·8 ≈ $1433.33
ln 2
d) T = ≈ 15.4 yr
0.045
34. The graph of f (x) = 2x−1 + 1 is the graph of g(x) = 2x
shifted right 1 unit and up 1 unit. Thus C is the correct
graph.

3
35. x
4 =8

3
2 x
(2 ) = 23

3
22 x = 23

23x=3
√3
3
x=
2
 3
3
x=
2
27
x=
8
27
The solution is .
8

Copyright © 2013 Pearson Education, Inc.


Copyright © 2013 Pearson Education, Inc.
Chapter 6
Systems of Equations and Matrices
13. Graph x − y = −6 and y = −2x and find the coordinates
Exercise Set 6.1 of the point of intersection.

1. Graph (c) is the graph of this system y

5
3. Graph (f) is the graph of this system. (-2, 4) 4
3
5. Graph (b) is the graph of this system.
2

x — y = —6 1
7. Graph x + y = 2 and 3x + y = 0 and find the coordinates
of the point of intersection. –5 –4 –3 –2 –1 1 2 3 4 5 x
–1
–2 y = —2x
y –3

5 –4

4 –5

3
(-1, 3)
2 x+y=2 The solution is (−2, 4).
1

–5 –4 –3 –2 –1 1 2 3 4 5 x 15. Graph 2y = x−1 and 3x = 6y +3 and find the coordinates


–1
–2
of the point of intersection.
–3
y
–4
–5 3x + y = 0 5
4
3
The solution is (−1, 3). 2y = x — 1
2
1
9. Graph x + 2y = 1 and x + 4y = 3 and find the coordinates
of the point of intersection. –5 –4 –3 –2 –1 1 2 3 4 5 x
–1
–2
y –3
3x = 6y + 3
5 –4

4 –5

3
2
x + 4y = 3
The graphs coincide so there are infinitely many solutions.
(-1, 1) 1
x−1
x Solving either equation for y, we get y = , so the so-
 
–5 –4 –3 –2 –1 1 2 3 4 5
–1 2
–2 x−1
x + 2y = 1 lutions can be expressed as x, . Similarly, solving
–3 2
–4 either equation for x, we get x = 2y + 1, so the solutions
–5
can also be expressed as (2y + 1, y).

The solution is (−1, 1). 17. x + y = 9, (1)


2x − 3y = −2 (2)
11. Graph y + 1 = 2x and y − 1 = 2x and find the coordinates
of the point of intersection. Solve equation (1) for either x or y. We choose to solve for
y.
y
y =9−x
5
4
Then substitute 9 − x for y in equation (2) and solve the
3 resulting equation.
2
y + 1 = 2x 2x − 3(9 − x) = −2
1

–5 –4 –3 –2 –1 1 2 3 4 5 x 2x − 27 + 3x = −2
–1
–2
5x − 27 = −2
y — 1 = 2x
–3 5x = 25
–4
–5 x=5
Now substitute 5 for x in either equation (1) or (2) and
The graphs do not intersect, so there is no solution. solve for y.

Copyright © 2013 Pearson Education, Inc.


216 Chapter 6: Systems of Equations and Matrices

5+y = 9 Using equation (1). 27. x + 2y = 2, (1)


y=4 4x + 4y = 5 (2)
The solution is (5, 4). Solve one equation for either x or y. We choose to solve
equation (1) for x since x has a coefficient of 1 in that
19. x − 2y = 7, (1)
equation.
x = y + 4 (2)
x + 2y = 2
Use equation (2) and substitute y + 4 for x in equation (1).
x = −2y + 2
Then solve for y.
Substitute −2y + 2 for x in equation (2) and solve for y.
y + 4 − 2y = 7
4(−2y + 2) + 4y = 5
−y + 4 = 7
−8y + 8 + 4y = 5
−y = 3
−4y + 8 = 5
y = −3
−4y = −3
Substitute −3 for y in equation (2) to find x.
3
x = −3 + 4 = 1 y=
4
The solution is (1, −3). 3
Substitute for y in either equation (1) or equation (2)
21. y = 2x − 6, (1) 4
and solve for x.
5x − 3y = 16 (2) x + 2y = 2 Using equation (1)
Use equation (1) and substitute 2x − 6 for y in equation 3
(2). Then solve for x. x+2· = 2
4
5x − 3(2x − 6) = 16 3
x+ = 2
5x − 6x + 18 = 16 2
1
−x + 18 = 16 x=
2
−x = −2  
1 3
The solution is , .
x=2 2 4
Substitute 2 for x in equation (1) to find y. 29. 3x − y = 5, (1)
y = 2 · 2 − 6 = 4 − 6 = −2 3y = 9x − 15 (2)
The solution is (2, −2). Solve one equation for x or y. We will solve equation (1)
23. x + y = 3, (1) for y.
y =4−x (2) 3x − y = 5
Use equation (2) and substitute 4 − x for y in equation (1). 3x = y + 5
x+4−x = 3 3x − 5 = y
4=3 Substitute 3x − 5 for y in equation (2) and solve for x.
There are no values of x and y for which 4 = 3, so the 3(3x − 5) = 9x − 15
system of equations has no solution. 9x − 15 = 9x − 15
25. x − 5y = 4, (1) −15 = −15
y = 7 − 2x (2) The equation −15 = −15 is true for all values of x and y,
so the system of equations has infinitely many solutions.
Use equation (2) and substitute 7 − 2x for y in equation
We know that y = 3x − 5, so we can write the solutions in
(1). Then solve for x.
the form (x, 3x − 5).
x − 5(7 − 2x) = 4 1 5
If we solve either equation for x, we get x = y + , so we
x − 35 + 10x = 4  3 3
1 5
11x − 35 = 4 can also write the solutions in the form y + ,y .
3 3
11x = 39
39 31. x + 2y = 7, (1)
x=
11 x − 2y = −5 (2)
39 We add the equations to eliminate y.
Substitute for x in equation (2) to find y.
11 x + 2y = 7
39 78 1
y =7−2· =7− =− x − 2y = −5
11
 11
 11
39 1 2x =2 Adding
The solution is ,− .
11 11 x=1

Copyright © 2013 Pearson Education, Inc.


Exercise Set 6.1 217

Back-substitute in either equation and solve for y. 39. 2x = 5 − 3y, (1)


1 + 2y = 7 Using equation (1) 4x = 11 − 7y (2)
2y = 6 We rewrite the equations.
y=3 2x + 3y = 5, (1a)
The solution is (1, 3). Since the system of equations has 4x + 7y = 11 (2a)
exactly one solution it is consistent and the equations are Multiply equation (2a) by −2 and add to eliminate x.
independent.
−4x − 6y = −10
33. x − 3y = 2, (1) 4x + 7y = 11
6x + 5y = −34 (2) y=1
Multiply equation (1) by −6 and add it to equation (2) to Back-substitute to find x.
eliminate x. 2x = 5 − 3 · 1 Using equation (1)
−6x + 18y = −12
2x = 2
6x + 5y = −34
x=1
23y = −46
The solution is (1, 1). Since the system of equations has
y = −2 exactly one solution it is consistent and the equations are
Back-substitute to find x. independent.
x − 3(−2) = 2 Using equation (1)
41. 0.3x − 0.2y = −0.9,
x+6 = 2
0.2x − 0.3y = −0.6
x = −4
First, multiply each equation by 10 to clear the decimals.
The solution is (−4, −2). Since the system of equations
3x − 2y = −9 (1)
has exactly one solution it is consistent and the equations
are independent. 2x − 3y = −6 (2)
Now multiply equation (1) by 3 and equation (2) by −2
35. 3x − 12y = 6, (1)
and add to eliminate y.
2x − 8y = 4 (2)
9x − 6y = −27
Multiply equation (1) by 2 and equation (2) by −3 and −4x + 6y = 12
add.
5x = −15
6x − 24y = 12
x = −3
−6x + 24y = −12
Back-substitute to find y.
0=0
3(−3) − 2y = −9 Using equation (1)
The equation 0 = 0 is true for all values of x and y. Thus,
the system of equations has infinitely many solutions. −9 − 2y = −9
1 1 −2y = 0
Solving either equation for y, we can write y = x − so
 4 2
y=0
1 1
the solutions are ordered pairs of the form x, x − .
4 2 The solution is (−3, 0). Since the system of equations has
Equivalently, if we solve either equation for x we get exactly one solution it is consistent and the equations are
x = 4y + 2 so the solutions can also be expressed as independent.
(4y + 2, y). Since there are infinitely many solutions, the
system of equations is consistent and the equations are 1 1
43. x + y = 6, (1)
dependent. 5 2
3 1
37. 4x − 2y = 3, (1) x − y = 2 (2)
5 2
2x − y = 4 (2) We could multiply both equations by 10 to clear fractions,
Multiply equation (2) by −2 and add. but since the y-coefficients differ only by sign we will just
add to eliminate y.
4x − 2y = 3
1 1
−4x + 2y = −8 x+ y=6
5 2
0 = −5 3 1
We get a false equation so there is no solution. Since there x− y=2
5 2
is no solution the system of equations is inconsistent and 4
the equations are independent. x =8
5
x = 10
Back-substitute to find y.

Copyright © 2013 Pearson Education, Inc.


218 Chapter 6: Systems of Equations and Matrices

1 1 Carry out. We solve the system of equations


· 10 + y =6 Using equation (1)
5 2
x + y = 70, 093, (1)
1
2+ y =6 x − 4861 = y. (2)
2
1 Substitute x − 4861 for y in equation (1) and solve for x.
y =4 x + (x − 4861) = 70, 093
2
y =8 2x − 4861 = 70, 093
The solution is (10, 8). Since the system of equations has 2x = 74, 954
exactly one solution it is consistent and the equations are x = 37, 477
independent.
Back-substitute in equation (2) to find y.
45. The statement is true. See page 485 of the text. 37, 477 − 4861 = y
32, 616 = y
47. False; a consistent system of equations can have exactly
one solution or infinitely many solutions. See page 485 of Check. 37, 477 + 32, 616 = 70, 093 complaints and 32,616
the text. is 4861 less than 37,477. The answer checks.
State. There were 37,477 complaints about cell-phone
49. True; a system of equations that has infinitely many so- providers and 32,616 complaints about cable/satellite TV
lutions is consistent and dependent. See page 485 of the providers.
text.
55. Familiarize. Let x = the number of Amish living in Wis-
51. Familiarize. Let k = the number of knee replacements, consin and y = the number living in Ohio.
in millions, and h = the number of hip replacements, in
millions, in 2030. Translate. The total number of Amish living in Wiscon-
sin and Ohio is 73,950, so we have one equation:
Translate. The total number of knee and hip replace-
ments will be 4.072 million so we have one equation. x + y = 73, 950

k + h = 4.072 The number of Amish living in Ohio is 12,510 more than


three times the number living in Wisconsin, so we have a
There will be 2.982 million more knee replacements than second equation:
hip replacements, so we have a second equation.
y = 3x + 12, 510
k = h + 2.928
Carry out. We solve the system of equations
Carry out. We solve the system of equations
x + y = 73, 950, (1)
k + h = 4.072, (1)
y = 3x + 12, 510. (2)
k = h + 2.928. (2)
Substitute 3x + 12, 510 for y in equation (1) and solve for
Substitute h + 2.928 for k in equation (1) and solve for h. x.
h + 2.928 + h = 4.072 x + (3x + 12, 510) = 73, 950
2h + 2.928 = 4.072 4x + 12, 510 = 73, 950
2h = 1.144 4x = 61, 440
h = 0.572 x = 15, 360
Back-substitute in equation (2) to find k. Back-substitute in equation (2) to find y.
k = 0.572 + 2.928 = 3.5 y = 3(15, 360) + 12, 510 = 46, 080 + 12, 510 = 58, 590
Check. 3.5 + 0.572 = 4.072 replacements and 3.5 is 2.928 Check. 15, 360 + 58, 590 = 73, 950 and 58,590 is 12,510
more than 0.572. The answer checks. more than 3 times 15,360. The answer checks.
State. In 2030 there will be 3.5 million knee replacements State. There are 15,360 Amish living in Wisconsin, and
and 0.572 million, or 572,000, hip replacements. 58,590 Amish live in Ohio.
53. Familiarize. Let x = the number of complaints about 57. Familiarize. Let p = the number of calories in a piece
cell-phone providers and y = the number of complaints of pecan pie and l = the number of calories in a piece of
about cable/satellite TV providers. lemon meringue pie.
Translate. The total number of complaints was 70,093 so Translate. The number of calories in a piece of pecan pie
we have one equation. is 221 less than twice the number of calories in a piece of
x + y = 70, 093 lemon meringue pie, so we have one equation:
Cable/satellite TV providers received 4861 fewer com- p = 2l − 221
plaints than cell-phne providers so we have a second equa- The total number of calories in a piece of each kind of pie
tion. is 865, so we have a second equation:
x − 4861 = y p + l = 865

Copyright © 2013 Pearson Education, Inc.


Exercise Set 6.1 219

Carry out. We solve the system of equations: The total interest is $690.
p = 2l − 221, (1) 0.04x + 0.05y = 690
p + l = 865. (2) We have a system of equations:
Substitute 2l − 221 for p in equation (2) and solve for l. x+ y = 15, 000,
(2l − 221) + l = 865 0.04x + 0.05y = 690
3l − 221 = 865 Multiplying the second equation by 100 to clear the deci-
3l = 1086 mals, we have:
l = 362 x + y = 15, 000, (1)

Back-substitute in equation (1) to find p. 4x + 5y = 69, 000. (2)

p = 2 · 362 − 221 = 724 − 221 = 503 Carry out. We begin by multiplying equation (1) by −4
and adding.
Check. 503 is 221 less than 2 times 362, and 503 + 362 =
−4x − 4y = −60, 000
865. The answer checks.
4x + 5y = 69, 000
State. A piece of pecan pie has 503 calories, and a piece
of lemon meringue pie has 362 calories. y = 9000
Back-substitute to find x.
59. Familiarize. Let x = the number of standard-delivery
x + 9000 = 15, 000 Using equation (1)
packages and y = the number of express-delivery packages.
x = 6000
Translate. A total of 120 packages were shipped, so we
have one equation. Check. The total investment is $6000+$9000, or $15,000.
The total interest is 0.04($6000) + 0.05($9000), or $240 +
x + y = 120
$450, or $690. The solution checks.
Total shipping charges were $596, so we have a second
State. $6000 was invested at 4% and $9000 was invested
equation.
at 5%.
3.50x + 7.50y = 596
63. Familiarize. Let x = the number of pounds of French
Carry out. We solve the system of equations:
roast coffee used and y = the number of pounds of Kenyan
x+ y = 120, coffee. We organize the information in a table.
3.50x + 7.50y = 596. French
Kenyan Mixture
First we multiply both sides of the second equation by 10 roast
to clear the decimals. This gives us the system of equations Amount x y 10 lb
x+ y = 120, (1) Price
35x + 75y = 5960. (2) per $9.00 $7.50 $8.40
pound
Now multiply equation (1) by −35 and add.
Total $8.40(10), or
−35x − 35y = −4200 cost
$9x $7.50y
$84
35x + 75y = 5960
Translate. The first and third rows of the table give us a
40y = 1760 system of equations.
y = 44 x+ y = 10,
Back-substitute to find x. 9x + 7.5y = 84
x + 44 = 120 Using equation (1) Multiply the second equation by 10 to clear the decimals.
x = 76 x+ y = 10, (1)
Check. 76 + 44 = 120 packages. Total shipping charges 90x + 75y = 840 (2)
are $3.50(76) + $7.50(44) = $266 + $330 = $596. The
answer checks. Carry out. Begin by multiplying equation (1) by −75
and adding.
State. The business shipped 76 standard-delivery pack-
ages and 44 express-delivery packages. −75x − 75y = −750
90x + 75y = 840
61. Familiarize. Let x = the amount invested at 4% and y =
15x = 90
the amount invested at 5%. Then the interest from the
investments is 4%x and 5%y, or 0.04x and 0.05y. x=6
Translate. Back-substitute to find y.
The total investment is $15,000. 6 + y = 10 Using equation (1)
x + y = 15, 000 y=4

Copyright © 2013 Pearson Education, Inc.


220 Chapter 6: Systems of Equations and Matrices

Check. The total amount of coffee in the mixture is 6 + 4, 46 = (x + y)2 x + y = 23, (1)
or 10 lb. The total value of the mixture is 6($9)+4($7.50), or
or $54 + $30, or $84. The solution checks. 51 = (x − y)3 x − y = 17 (2)
State. 6 lb of French roast coffee and 4 lb of Kenyan coffee Carry out. We begin by adding equations (1) and (2).
should be used. x + y = 23
65. Familiarize. Let x = the number of servings of spaghetti x − y = 17
and meatballs required and y = the number of servings 2x = 40
of iceberg lettuce required. Then x servings of spaghetti x = 20
contain 260x Cal and 32x g of carbohydrates; y servings
of lettuce contain 5y Cal and 1 · y or y, g of carbohydrates. Back-substitute to find y.
20 + y = 23 Using equation (1)
Translate. One equation comes from the fact that 400 Cal
are desired: y=3
260x + 5y = 400. Check. The speed downstream is 20+3, or 23 km/h. The
distance traveled downstream in 2 hr is 23 · 2, or 46 km.
A second equation comes from the fact that 50g of carbo-
The speed upstream is 20 − 3, or 17 km/h. The distance
hydrates are required:
traveled upstream in 3 hr is 17 · 3, or 51 km. The solution
32x + y = 50. checks.
Carry out. We solve the system State. The speed of the boat is 20 km/h. The speed of
260x + 5y = 400, (1) the stream is 3 km/h.
32x + y = 50. (2) 69. Familiarize. Let d = the distance the slower plane trav-
Multiply equation (2) by −5 and add. els, in km. Then 780 − d = the distance the faster plane
260x + 5y = 400 travels. Let t = the number of hours each plane travels.
We organize the information in a table.
−160x − 5y = −250
100x = 150 Distance Speed Time
x = 1.5 Slower plane d 190 t
Back-substitute to find y. Faster plane 780 − d 200 t
32(1.5) + y = 50 Using equation (2)
Translate. Using d = rt in each row of the table, we get
48 + y = 50 a system of equations.
y=2 d = 190t, (1)
Check. 1.5 servings of spaghetti contain 260(1.5), or 390 780 − d = 200t (2)
Cal and 32(1.5), or 48 g of carbohydrates; 2 servings of
Carry out. We begin by adding the equations.
lettuce contain 5 · 2, or 10 Cal and 1 · 2, or 2 g of carbo-
hydrates. Together they contain 390 + 10, or 400 Cal and d = 190t
48 + 2, or 50 g of carbohydrates. The solution checks. 780 − d = 200t
State. 1.5 servings of spaghetti and meatballs and 2 serv- 780 = 390t
ings of iceberg lettuce are required. 2=t
67. Familiarize. It helps to make a drawing. Then organize Check. In 2 hr, the slower plane travels 190 · 2, or 380 km,
the information in a table. Let x = the speed of the boat and the faster plane travels 200 · 2, or 400 km. The total
and y = the speed of the stream. The speed upstream is distance traveled is 380 km + 400 km, or 780 km, so the
x − y. The speed downstream is x + y. answer checks.
State. The planes will meet in 2 hr.
46 km 2 hr (x + y) km/h
q ✲ 71. Familiarize and Translate. We use the system of equa-
Downstream tions given in the problem.
51 km 3 hr (x − y) km/h y = 70 + 2x (1)
✛ q
Upstream y = 175 − 5x, (2)
Carry out. Substitute 175 − 5x for y in equation (1) and
Distance Speed Time solve for x.
Downstream 46 x+y 2 175 − 5x = 70 + 2x
105 = 7x Adding 5x and subtracting 70
Upstream 51 x−y 3
15 = x
Translate. Using d = rt in each row of the table, we get
Back-substitute in either equation to find y. We choose
a system of equations.
equation (1).

Copyright © 2013 Pearson Education, Inc.


Exercise Set 6.1 221

y = 70 + 2 · 15 = 70 + 30 = 100 Check. A decrease of 8.2% from 838.3 million is


Check. Substituting 15 for x and 100 for y in both of the 0.918(838.3) = 769.5594 ≈ 769.6 million. The answer
original equations yields true equations, so the solution checks.
checks. State. There were about 838.3 million air travelers in
State. The equilibrium point is (15, $100). 2004.

73. Familiarize and Translate. We find the value of x for 81. Substituting 15 for f (x), we solve the following equation.
which C = R, where 15 = x2 − 4x + 3
C = 14x + 350, 0 = x2 − 4x − 12
R = 16.5x. 0 = (x − 6)(x + 2)
Carry out. When C = R we have: x − 6 = 0 or x + 2 = 0
14x + 350 = 16.5x x = 6 or x = −2
350 = 2.5x If the output is 15, the input is 6 or −2.
140 = x
83. f (−9) = (−9)2 − 4(−9) + 3 = 120
Check. When x = 140, C = 14 · 140 + 350, or 2310 and
If the input is −9, the output is 120.
R = 16.5(140), or 2310. Since C = R, the solution checks.
State. 140 units must be produced and sold in order to 85. Familiarize. Let x = the time spent jogging and y = the
break even. time spent walking. Then Nancy jogs 8x km and walks
4y km.
75. Familiarize and Translate. We find the value of x for
Translate.
which C = R, where
C = 15x + 12, 000, The total time is 1 hr.

R = 18x − 6000. x+y =1


Carry out. When C = R we have: The total distance is 6 km.
15x + 12, 000 = 18x − 6000 8x + 4y = 6
18, 000 = 3x Subtracting 15x and Carry out. Solve the system
adding 6000 x + y = 1, (1)
6000 = x 8x + 4y = 6. (2)
Check. When x = 6000, C = 15·6000+12, 000, or 102,000 Multiply equation (1) by −4 and add.
and R = 18 · 6000 − 6000, or 102,000. Since C = R, the
solution checks. −4x − 4y = −4
State. 6000 units must be produced and sold in order to 8x + 4y = 6
break even. 4x =2
1
77. a) r(x) = 0.2308826185x + 44.61085102, x=
2
p(x) = 0.6288961625x + 26.81925282 This is the time we need to find the distance spent jogging,
b) Graph y1 = r(x) and y2 = p(x) and find the first so we could stop here. However, we will not be able to
coordinate of the point of intersection of the graphs. check the solution unless we find y also so we continue.
It is approximately 45, so we estimate that poultry We back-substitute.
consumption will equal red meat consumption about 1
+ y = 1 Using equation (1)
45 yr after 1995. 2
1
79. Familiarize. Let t = the number of air travels in 2004, y=
2
in millions. Then a decrease of 8.2% from this number 1
is t − 8.2%t, or t − 0.082t, or 0.918t. This represents the Then the distance jogged is 8· , or 4 km, and the distance
2
number of air travelers in 2009. 1
walked is 4 · , or 2 km.
Translate. 2
Number of air 8.2% less than 1 1
is Check. The total time is hr + hr, or 1 hr. The total
travelers in 2009 number in 2004 2 2
 
    
  distance is 4 km + 2 km, or 6 km. The solution checks.
   State. Nancy jogged 4 km on each trip.
769.6 = 0.918t
87. Familiarize and Translate. We let x and y represent
Carry out. We solve the equation.
the speeds of the trains. Organize the information in a
769.6 = 0.918t table. Using d = rt, we let 3x, 2y, 1.5x, and 3y represent
838.3 ≈ t the distances the trains travel.

Copyright © 2013 Pearson Education, Inc.


222 Chapter 6: Systems of Equations and Matrices

 
First situation: 1
3 −B = 1 Using equation (1)
10
3 hours x km/h y km/h 3
q 2 hours −B = 1
10
Union Central
7
216 km −B =
10
Second situation: 7
B=−
10
1 7
1.5 hours x km/h
q
y km/h 3 hours We have A = and B = − .
10 10
Union Central
91. Familiarize. Let x and y represent the number of gallons
216 km of gasoline used in city driving and in highway driving,
respectively. Then 49x and 51y represent the number of
Distance Distance miles driven in the city and on the highway, respectively.
traveled traveled
in first in second Translate. The fact that 9 gal of gasoline were used gives
situation situation us one equation:
Train1 (from
3x 1.5x x + y = 9.
Union to Central)
A second equation comes from the fact that the car is
Train2 (from
2y 3y driven 447 mile:
Central to Union)
Total 216 216 49x + 51y = 447.
The total distance in each situation is 216 km. Thus, we Carry out. We solve the system of equations
have a system of equations. x+ y = 9, (1)
3x + 2y = 216, (1) 49x + 51y = 447. (2)
1.5x + 3y = 216 (2) Multiply equation (1) by −49 and add.
Carry out. Multiply equation (2) by −2 and add. −49x − 49y = −441
3x + 2y = 216 49x + 51y = 447
−3x − 6y = −432 2y = 6
−4y = −216 y=3
y = 54 Back-substitute to find x.
Back-substitute to find x. x+3 = 9
3x + 2 · 54 = 216 Using equation (1) x=6
3x + 108 = 216 Then in the city the car is driven 49(6), or 294 mi; on the
highway it is driven 51(3), or 153 mi.
3x = 108
Check. The number of gallons of gasoline used is 6 + 3,
x = 36
or 9. The number of miles driven is 294 + 153 = 447. The
Check. If x = 36 and y = 54, the total distance the trains answer checks.
travel in the first situation is 3 · 36 + 2 · 54, or 216 km.
State. The car was driven 294 mi in the city and 153 mi
The total distance they travel in the second situation is
on the highway.
1.5 · 36 + 3 · 54, or 216 km. The solution checks.
State. The speed of the first train is 36 km/h. The speed
of the second train is 54 km/h. Exercise Set 6.2
89. Substitute the given solutions in the equation 1. x + y + z = 2, (1)
Ax + By = 1 to get a system of equations.
6x − 4y + 5z = 31, (2)
3A − B = 1, (1)
5x + 2y + 2z = 13 (3)
−4A − 2B = 1 (2)
Multiply equation (1) by −6 and add it to equation (2).
Multiply equation (1) by −2 and add. We also multiply equation (1) by −5 and add it to equation
−6A + 2B = −2 (3).
−4A − 2B = 1 x+ y+ z=2 (1)
−10A = −1 − 10y − z = 19 (4)
1 − 3y − 3z = 3 (5)
A=
10 Multiply the last equation by 10 to make the y-coefficient
Back-substitute to find B. a multiple of the y-coefficient in equation (4).

Copyright © 2013 Pearson Education, Inc.


Exercise Set 6.2 223

x+ y+ z=2 (1) 5. x + 2y − z = 5, (1)


− 10y − z = 19 (4) 2x − 4y + z = 0, (2)
− 30y − 30z = 30 (6) 3x + 2y + 2z = 3 (3)
Multiply equation (4) by −3 and add it to equation (6). Multiply equation (1) by −2 and add it to equation (2).
x+ y+ z=2 (1) Also, multiply equation (1) by −3 and add it to equa-
tion (3).
− 10y − z = 19 (4)
x + 2y − z = 5, (1)
− 27z = −27 (7)
− 8y + 3z = −10, (4)
Solve equation (7) for z.
− 4y + 5z = −12 (5)
−27z = −27
Multiply equation (5) by 2 to make the y-coefficient a mul-
z=1
tiple of the y-coefficient of equation (4).
Back-substitute 1 for z in equation (4) and solve for y.
x + 2y − z = 5, (1)
−10y − 1 = 19
− 8y + 3z = −10, (4)
−10y = 20
− 8y + 10z = −24 (6)
y = −2
Multiply equation (4) by −1 and add it to equation (6).
Back-substitute 1 for z for −2 and y in equation (1) and
x + 2y − z = 5, (1)
solve for x.
x + (−2) + 1 = 2 − 8y + 3z = −10, (4)
x−1 = 2 7z = −14 (7)
x=3 Solve equation (7) for z.
The solution is (3, −2, 1). 7z = −14
z = −2
3. x − y + 2z = −3 (1)
Back-substitute −2 for z in equation (4) and solve for y.
x + 2y + 3z = 4 (2)
−8y + 3(−2) = −10
2x + y + z = −3 (3)
−8y − 6 = −10
Multiply equation (1) by −1 and add it to equation (2).
We also multiply equation (1) by −2 and add it to equation −8y = −4
(3). 1
y=
x − y + 2z = −3 (1) 2
1
3y + z = 7 (4) Back-substitute for y and −2 for z in equation (1) and
2
3y − 3z = 3 (5) solve for x.
1
Multiply equation (4) by −1 and add it to equation (5). x + 2 · − (−2) = 5
2
x − y + 2z = −3 (1) x+1+2 = 5
3y + z = 7 (4) x=2
− 4z = −4 (6)  
1
Solve equation (6) for z. The solution is 2, , −2 .
2
−4z = −4 7. x + 2y − z = −8, (1)
z=1 2x − y + z = 4, (2)
Back-substitute 1 for z in equation (4) and solve for y. 8x + y + z = 2 (3)
3y + 1 = 7 Multiply equation (1) by −2 and add it to equation (2).
3y = 6 Also, multiply equation (1) by −8 and add it to equa-
y=2 tion (3).
Back-substitute 1 for z and 2 for y in equation (1) and x + 2y − z = −8, (1)
solve for x. − 5y + 3z = 20, (4)
x − 2 + 2 · 1 = −3 − 15y + 9z = 66 (5)
x = −3 Multiply equation (4) by −3 and add it to equation (5).
The solution is (−3, 2, 1). x + 2y − z = −8, (1)
− 5y + 3z = 20, (4)
0=6 (6)

Copyright © 2013 Pearson Education, Inc.


224 Chapter 6: Systems of Equations and Matrices

Equation (6) is false, so the system of equations has no Multiply equation (3) by 3 to make the b-coefficient a mul-
solution. tiple of the b-coefficient in equation (4).

9. 2x + y − 3z = 1, (1) 4a + 9b = 8, (1)

x − 4y + z = 6, (2) − 18b + 6c = −17, (4)

4x − 7y − z = 13 (3) 18b + 18c = −3 (5)

Interchange equations (1) and (2). Add equation (4) to equation (5).

x − 4y + z = 6, (2) 4a + 9b = 8, (1)

2x + y − 3z = 1, (1) − 18b + 6c = −17, (4)

4x − 7y − z = 13 (3) 24c = −20 (6)

Multiply equation (2) by −2 and add it to equation (1). Solve equation (6) for c.
Also, multiply equation (2) by −4 and add it to equa- 24c = −20
tion (3). 20 5
c=− =−
x − 4y + z = 6, (2) 24 6
5
9y − 5z = −11, (4) Back-substitute − for c in equation (4) and solve for b.
6
9y − 5z = −11 (5)
−18b + 6c = −17
Multiply equation (4) by −1 and add it to equation (5).  
5
x − 4y + z = 6, (1) −18b + 6 − = −17
6
9y − 5z = −11, (4) −18b − 5 = −17
0=0 (6) −18b = −12
The equation 0 = 0 tells us that equation (3) of the original 12 2
b= =
system is dependent on the first two equations. The system 18 3
of equations has infinitely many solutions and is equivalent 2
Back-substitute for b in equation (1) and solve for a.
to 3
2x + y − 3z = 1, (1) 4a + 9b = 8
x − 4y + z = 6. (2) 2
4a + 9 · = 8
To find an expression for the solutions, we first solve equa- 3
tion (4) for either y or z. We choose to solve for z. 4a + 6 = 8
9y − 5z = −11 4a = 2
−5z = −9y − 11 1
a=
2
z=
9y + 11  
1 2 5
5 The solution is , ,− .
Back-substitute in equation (2) to find an expression for x 2 3 6
in terms of y. 13. 2x + z = 1, (1)
9y + 11
x − 4y + =6 3y − 2z = 6, (2)
5
x − 2y = −9 (3)
9 11
x − 4y + y + =6 Interchange equations (1) and (3).
5 5
11 19 11y + 19 x − 2y = −9, (3)
x= y+ =
5 5 5 3y − 2z = 6, (2)
 
11y + 19 9y + 11 2x + z=1 (1)
The solutions are given by , y, , where
5 5
Multiply equation (3) by −2 and add it to equation (1).
y is any real number.
x − 2y = −9, (3)
11. 4a + 9b = 8, (1)
3y − 2z = 6, (2)
8a + 6c = −1, (2)
4y + z = 19 (4)
6b + 6c = −1 (3)
Multiply equation (4) by 3 to make the y-coefficient a mul-
Multiply equation (1) by −2 and add it to equation (2). tiple of the y-coefficient in equation (2).
4a + 9b = 8, (1) x − 2y = −9, (3)
− 18b + 6c = −17, (4) 3y − 2z = 6, (2)
6b + 6c = −1 (3) 12y + 3z = 57 (5)

Copyright © 2013 Pearson Education, Inc.


Exercise Set 6.2 225

Multiply equation (2) by −4 and add it to equation (5). 17. Familiarize. Let x, y, and z represent the number of Win-
x − 2y = −9, (3) ter Olympics sites in North America, Europe, and Asia,
respectively.
3y − 2z = 6, (2)
Translate. The total number of sites is 21.
11z = 33 (6)
x + y + z = 21
Solve equation (6) for z.
The number of European sites is 5 more than the total
11z = 33 number of sites in North America and Asia.
z=3
y =x+z+5
Back-substitute 3 for z in equation (2) and solve for y.
There are 4 more sites in North America than in Asia.
3y − 2z = 6
x=z+4
3y − 2 · 3 = 6
We have
3y − 6 = 6 x + y + z = 21,
3y = 12 y = x + z + 5,
y=4 x=z+4
Back-substitute 4 for y in equation (3) and solve for x. or
x − 2y = −9 x + y+ z = 21,
x − 2 · 4 = −9 −x + y− z = 5,
x − 8 = −9 x − z = 4.
x = −1 Carry out. Solving the system of equations, we get
(6, 13, 2).
The solution is (−1, 4, 3).
Check. The total number of sites is 6 + 13 + 2, or 21. The
15. w+ x+ y+ z=2 (1) total number of sites in North America and Asia is 6+2, or
w + 2x + 2y + 4z = 1 (2) 8, and 5 more than this is 8 + 5, or 13, the number of sites
in Europe. Also, the number of sites in North America, 6,
−w + x − y − z = −6 (3)
is 4 more than 2, the number of sites in Asia. The answer
−w + 3x + y − z = −2 (4) checks.
Multiply equation (1) by −1 and add to equation (2). Add State. The Winter Olympics have been held in 6 North
equation (1) to equation (3) and to equation (4). American sites, 13 European sites, and 2 Asian sites.
w+ x+ y+ z=2 (1)
19. Familiarize. Let x, y, and z represent the number of
x + y + 3z = −1 (5) billions of pounds of apples prduced in China, the United
2x = −4 (6) States, and Turkey, respectively.
4x + 2y =0 (7) Translate. A total of 74 billion lb of apples were grown
Solve equation (6) for x. in three countries.

2x = −4 x + y + z = 74

x = −2 China produced 44 billion lb more than the combined pro-


ductions of the United States and Turkey.
Back-substitute −2 for x in equation (7) and solve for y.
x = y + z + 44
4(−2) + 2y = 0
The United States produced twice as many pounds as
−8 + 2y = 0 Turkey.
2y = 8 y = 2z
y=4
We have
Back-substitute −2 for x and 4 for y in equation (5) and x + y + z = 74,
solve for z.
x = y + z + 44,
−2 + 4 + 3z = −1
y = 2z.
3z = −3
or
z = −1 x + y + z = 74,
Back-substitute −2 for x, 4 for y, and −1 for z in equa- x − y − z = 44,
tion (1) and solve for w.
y − 2z = 0.
w−2+4−1 = 2
Carry out. Solving the system of equations, we get
w=1 (59, 10, 5).
The solution is (1, −2, 4, −1).

Copyright © 2013 Pearson Education, Inc.


226 Chapter 6: Systems of Equations and Matrices

Check. The total productions is 59+10+5, or 74 billion lb. x + y+ z = 355,


China’s production, 59 billion lb, is 44 billion lb more than x − y− z = 11,
the combined production of the United States and Turkey,
y− z = 16.
10 + 5, or 15 billion lb. The production in the United
States, 10 billion lb, is twice the production in Turkey, 5 Carry out. Solving the system of equations, we get
billion lb. The answer checks. (183, 94, 78).
State. The apple production in China, the United States, Check. The total number of fish, cats, and dogs is
and Turkey was 59 billion lb, 10 billion lb, and 5 billion 183 +94 +78, or 355 million. The total number of cats and
lb, respectively. dogs owned is 94 + 78, or 172 million, and 172 million +
11 million is 183 million, the number of fish owned. The
21. Familiarize. Let x y, and z represent the number of mil- number of cats owned, 94 million, is 16 million more than
ligrams of caffeine in an 8-oz serving of brewed coffee, Red 78 million, the number of dogs owned. The solution checks.
Bull energy drink, and Mountain Dew, respectively. State. Americans own 183 million fish, 94 million cats,
Translate. The total amount of caffeine in one serving of and 78 million dogs.
each beverage is 197 mg.
25. Familiarize. Let x, y, and z represent the amounts earned
x + y + z = 197 by The Dark Knight, Spider-Man 3, and The Twilight
One serving of brewed coffee has 6 mg more caffeine than Saga: New Moon, respectively, in millions of dollars.
two servings of Mountain Dew. Translate. Total earnings are $452 million.
x = 2z + 6 x + y + z = 452
One serving of Red Bull contains 37 mg less caffeine than Together, Spider-Man 3 and New Moon earned $136 mil-
one serving each of brewed coffee and Mountain Dew. lion more than The Dark Knight.
y = x + z − 37 y + z = x + 136
We have a system of equations New Moon earned $15 million less than The Dark Knight.
x + y + z = 197, x + y + z = 197, z = x − 15
x = 2z + 6, or x − 2z = 6, We have
y = x + z − 37 −x + y − z = −37 x + y + z = 452,
Carry out. Solving the system of equations, we get y + z = x + 136,
(80, 80, 37).
z = x − 15
Check. The total amount of caffeine is 80 + 80 + 37, or or
197 mg. Also, 80 mg is 6 mg more than twice 37 mg, and x + y+ z = 452,
80 mg is 37 mg less than the total of 80 mg and 37 mg, or
117 mg. The answer checks. −x + y+ z = 136,

State. One serving each of brewed coffee, Red Bull energy −x + z = −15.
drink, and Mountain Dew contains 80 mg, 80 mg, and Carry out. Solving the system of equations, we get
37 mg of caffeine, respectively. (158, 151, 143).
Check. The earnings total $158+$151+$143, or $452 mil-
23. Familiarize. Let x, y, and z represent the number of fish,
lion. Together, Spider-Man 3 and New Moon earned
cats, and dogs owned by Americans, in millions, respec-
$151 + $143, or $294 million. This is $136 million more
tively.
than $158 million, the earnings of The Dark Knight. Also,
Translate. The total number of fish, cats, and dogs owned $15 million less than $158 million, the earnings of The
is 355 million. Dark Knight is $158 − $15 or $143 million, the earnings of
x + y + z = 355 New Moon. The answer checks.
The number of fish owned is 11 million more than the total State. The Dark Knight, Spider-Man 3, and New Moon
number of cats and dogs owned. grossed $158 million, $151 million, and $143 million, re-
spectively, in a weekend.
x = y + z + 11
There are 16 million more cats than dogs. 27. Familiarize. Let x, y, and z represent the number of
servings of ground beef, baked potato, and strawberries
y = z + 16
required, respectively. One serving of ground beef con-
We have tains 245x Cal, 0x or 0 g of carbohydrates, and 9x mg
x + y + z = 355, of calcium. One baked potato contains 145y Cal, 34y g
x = y + z + 11, of carbohydrates, and 8y mg of calcium. One serving of
strawberries contains 45z Cal, 10z g of carbohydrates, and
y = z + 16
21z mg of calcium.
or
Translate.
The total number of calories is 485.

Copyright © 2013 Pearson Education, Inc.


Exercise Set 6.2 227

245x + 145y + 45z = 485 31. Familiarize. Let x, y, and z represent the prices of orange
A total of 41.5 g of carbohydrates is required. juice, a raisin bagel, and a cup of coffee, respectively. The
new price for orange juice is x + 25%x, or x + 0.25x, or
34y + 10z = 41.5 1.25x; the new price of a bagel is y + 20%y, or y + 0.2y, or
A total of 35 mg of calcium is required. 1.2y.
9x + 8y + 21z = 35 Translate.
We have a system of equations. Orange juice, a raisin bagel, and a cup of coffee cost $8.15.
245x + 145y + 45z = 485, x + y + z = 8.15
34y + 10z = 41.5, After the price increase, orange juice, a raisin bagel, and
a cup of coffee will cost $9.30.
9x + 8y + 21z = 35
1.25x + 1.2y + z = 9.30
Carry out. Solving the system of equations, we get
(1.25, 1, 0.75). After the price increases, a raisin bagel will cost 30/
c (or
$0.30) more than coffee.
Check. 1.25 servings of ground beef contains 306.25 Cal,
no carbohydrates, and 11.25 mg of calcium; 1 baked potato 1.2y = z + 0.30
contains 145 Cal, 34 g of carbohydrates, and 8 mg of cal- We have a system of equations.
cium; 0.75 servings of strawberries contains 33.75 Cal, 7.5 g x + y + z = 8.15, 100x + 100y + 100z = 815,
of carbohydrates, and 15.75 mg of calcium. Thus, there
1.25x+1.2y+z = 9.30, or 125x + 120y + 100z = 930,
are a total of 306.25 + 145 + 33.75, or 485 Cal, 34 + 7.5, or
41.5 g of carbohydrates, and 11.25 + 8 + 15.75, or 35 mg 1.2y = z + 0.30 12y − 10z = 3
of calcium. The solution checks. Carry out. Solving the system of equations, we get
State. 1.25 servings of ground beef, 1 baked potato, and (2.4, 2.75, 3).
0.75 serving of strawberries are required. Check. If orange juice costs $2.40, a bagel costs $2.75,
and a cup of coffee costs $3.00, then together they cost
29. Familiarize. Let x, y, and z represent the amounts in- $2.40 + $2.75 + $3.00, or $8.15. After the price increases
vested at 3%, 4%, and 6%, respectively. Then the annual orange juice will cost 1.25($2.40), or $3.00 and a bagel will
interest from the investments is 3%x, 4%y, and 6%z, or cost 1.2($2.75) or $3.30. Then orange juice, a bagel, and
0.03x, 0.04y, and 0.06z. coffee will cost $3.00 + $3.30 + $3.00, or $9.30. After the
Translate. price increase the price of a raisin bagel, $3.30, will be 30/
c
A total of $5000 was invested. more than the price of coffee, $3.00. The solution checks.
x + y + z = 5000 State. Before the increase orange juice cost $2.40, a raisin
bagel cost $2.75, and a cup of coffee cost $3.00.
The total interest is $243.
33. a) Substitute the data points (0, 16), (7, 9), and
0.03x + 0.04y + 0.06z = 243
(20, 21) in the function f (x) = ax2 + bx + c.
The amount invested at 6% is $1500 more than the amount
16 = a · 02 + b · 0 + c
invested at 3%.
9 = a · 72 + b · 7 + c
z = x + 1500
21 = a · 202 + b · 20 + c
We have a system of equations.
We have a system of equations.
x + y + z = 5000,
c = 16,
0.03x + 0.04y + 0.06z = 243,
49a + 7b + c = 9,
z = x + 1500
or 400a + 20b + c = 21.
x + y + z = 5000, Solving the system of equations, we get
 
3x + 4y + 6z = 24, 300, 5 87 5 2 87
, − , 16 so f (x) = x − x + 16,
−x + z = 1500 52 52 52 52
where x is the number of years after 1990 and f (x) is
Carry out. Solving the system of equations, we get
a percent.
(1300, 900, 2800).
b) In 2003, x = 2003 − 1990, or 13.
Check. The total investment was $1300+$900+$2800, or
5 87
$5000. The total interest was 0.03($1300) + 0.04($900) + f (13) = · 132 − · 13 + 16 = 10.5%
0.06($2800) = $39 + $36 + $168, or $243. The amount 52 52
invested at 6%, $2800, is $1500 more than the amount 35. a) Substitute the data points (0, 431), (5, 441), and
invested at 3%, $1300. The solution checks. (12, 418) in the function f (x) = ax2 + bx + c.
State. $1300 was invested at 3%, $900 at 4%, and $2800 431 = a · 02 + b · 0 + c
at 6%. 441 = a · 52 + b · 5 + c
418 = a · 122 + b · 12 + c

Copyright © 2013 Pearson Education, Inc.


228 Chapter 6: Systems of Equations and Matrices

We have a system of equations. 49. Substituting, we get


c = 431, 3
A + B + 3C = 12,
25a + 5b + c = 441, 4
4
144a + 12b + c = 418 A + B + 2C = 12,
3
Solving
 the system
 of equations, we get 2A + B + C = 12, or
37 353
− , , 431 , so
84 84 4A + 3B + 12C = 48,
37 353 4A + 3B + 6C = 36, Clearing fractions
f (x) = − x2 + x + 431,
84 84 2A + B + C = 12.
where x is the number of years after 1997.
Solving the system of equations, we get (3, 4, 2). The equa-
b) In 2000, x = 2000 − 1997, or 3. tion is 3x + 4y + 2z = 12.
37 353
f (3) = − · 32 + · 3 + 431 ≈ 440 acres 51. Substituting, we get
84 84
In 2010, x = 2010 − 1997 = 13. 59 = a(−2)3 + b(−2)2 + c(−2) + d,
37 353 13 = a(−1)3 + b(−1)2 + c(−1) + d,
f (13) = − · 132 + · 13 + 431 ≈ 411 acres
84 84 −1 = a · 13 + b · 12 + c · 1 + d,
37. a) f (x) = 0.143707483x −0.6921768707x + 5.882482993,
2
−17 = a · 23 + b · 22 + c · 2 + d, or
where x is the number of years after 2002.
b) In 2003, x = 2003 − 2002 = 1. −8a + 4b − 2c + d = 59,

f (1) ≈ 5.3% −a + b − c + d = 13,


a + b + c + d = −1,
In 2007, x = 2007 − 2002 = 5.
8a + 4b + 2c + d = −17.
f (5) ≈ 6.0%
Solving the system of equations, we get
In 2009, x = 2009 − 2002 = 7.
(−4, 5, −3, 1), so y = −4x3 + 5x2 − 3x + 1.
f (7) ≈ 8.1%
39. Perpendicular Exercise Set 6.3
41. A vertical line
1. The matrix has 3 rows and 2 columns, so its order is 3 × 2.
43. A rational function
3. The matrix has 1 row and 4 columns, so its order is 1 × 4.
45. A vertical asymptote
5. The matrix has 3 rows and 3 columns, so its order is 3 × 3.
2 2 3
47. + − = 3, 7. We omit the variables and replace the equals signs with a
x y z
1 2 3 vertical line.
− − = 9,  
x y z 2 −1 7
7 2 9  
− + = −39 1 4 −5
x y z
1 1 1
Substitute u for , v for , and w for and solve for u, 9. We omit the variables, writing zeros for the missing terms,
x y z
v, and w. and replace the equals signs with a vertical line.
 
2u + 2v − 3w = 3, 1 −2 3 12
 
u − 2v − 3w = 9,  2 0 −4 8 
7u − 2v + 9w = −39 0 3 1 7
Solving this system we get (−2, −1, −3).
11. Insert variables and replace the vertical line with equals
1 1 1
Then = −2, or x = − ; = −1, or y = −1; and signs.
x 2 y
1 1 3x − 5y = 1,
= −3, or z = − . The solution of the original system is
z 3 x + 4y = −2
 
1 1
− , −1, − . 13. Insert variables and replace the vertical line with equals
2 3 signs.
2x + y − 4z = 12,
3x + 5z = −1,
x−y+ z=2

Copyright © 2013 Pearson Education, Inc.


Exercise Set 6.3 229

15. 4x + 2y = 11, Multiply row 1 by −2 and add it to row 2.


3x − y = 2  
5 −2 −3
Write the augmented matrix. We will use Gaussian elimi-  
nation. 0 29 −114
 
4 2 11 1 1
  Multiply row 1 by and row 2 by .
5 29
3 −1 2
 
2 3
1 − −
Multiply row 2 by 4 to make the first number in row 2 a  5 5 
 
multiple of 4.  114 
  0 1 −
4 2 11 29
 
Write the system of equations that corresponds to the last
12 −4 8
matrix.
2 3
Multiply row 1 by −3 and add it to row 2. x− y = − , (1)
5 5
 
4 2 11 114
  y=− (2)
29
0 −10 −25 Back-substitute in equation (1) and solve for x.
 
1 1 2 114 3
Multiply row 1 by and row 2 by − . x− − =−
4 10 5 29 5
  x+
228
=−
3
1 11
1 145 5
 2 4 
  315 63
 5  x=− =−
0 1 145 29
2  
63 114
The solution is − , − .
Write the system of equations that corresponds to the last 29 29
matrix.
19. 3x + 4y = 7,
1 11
x+ y =
2 4
, (1) −5x + 2y = 10
5 Write the augmented matrix. We will use Gaussian elimi-
y= (2) nation.
2
 
Back-substitute in equation (1) and solve for x. 3 4 7
1 5 11  
x+ · = −5 2 10
2 2 4
5 11
x+ = Multiply row 2 by 3 to make the first number in row 2 a
4 4 multiple of 3.
6 3  
x= =
4 2 3 4 7
   
3 5
The solution is , . −15 6 30
2 2
17. 5x − 2y = −3, Multiply row 1 by 5 and add it to row 2.
2x + 5y = −24  
3 4 7
Write the augmented matrix. We will use Gaussian elimi-  
nation. 0 26 65
 
5 −2 −3 1 1
  Multiply row 1 by and row 2 by .
3 26
2 5 −24  4 7 
1
Multiply row 2 by 5 to make the first number in row 2 a  3 3 
 
multiple of 5.  
5
  0 1
5 −2 −3 2
 
Write the system of equations that corresponds to the last
10 25 −120 matrix.

Copyright © 2013 Pearson Education, Inc.


230 Chapter 6: Systems of Equations and Matrices

 
4 7 1 −3 8
x + y = , (1)
3 3  
5 2 −6 3
y= (2)
2
Multiply row 1 by −2 and add it to row 2.
Back-substitute in equation (1) and solve for x.
 
4 5
x+ · =
7 1 −3 8
3 2 3  
10 7 0 0 −13
x+ =
3 3
The last row corresponds to the false equation 0 = −13,
3 so there is no solution.
x = − = −1
3
  25. −2x + 6y = 4,
5
The solution is − 1, .
2 3x − 9y = −6
21. 3x + 2y = 6, Write the augmented matrix.
 
2x − 3y = −9 −2 6 4
Write the augmented matrix. We will use Gauss-Jordan  
elimination. 3 −9 −6
 
3 2 6 1
  Multiply row 1 by − .
2
2 −3 −9  
1 −3 −2
Multiply row 2 by 3 to make the first number in row 2 a  
multiple of 3. 3 −9 −6
 
3 2 6 Multiply row 1 by −3 and add it to row 2.
   
6 −9 −27 1 −3 −2
 
Multiply row 1 by −2 and add it to row 2. 0 0 0
 
3 2 6 The last row corresponds to the equation 0 = 0 which is
  true for all values of x and y. Thus, the system of equations
0 −13 −39 is dependent and is equivalent to the first equation −2x +
6y = 4, or x − 3y = −2. Solving for x, we get x = 3y − 2.
1 Then the solutions are of the form (3y − 2, y), where y is
Multiply row 2 by − .
13 any real number.
 
3 2 6 27. x + 2y − 3z = 9,
 
2x − y + 2z = −8,
0 1 3
3x − y − 4z = 3
Multiply row 2 by −2 and add it to row 1. Write the augmented matrix. We will use Gauss-Jordan
  elimination.
3 0 0
   
1 2 −3 9
0 1 3  
 2 −1 2 −8 
1 3 −1 −4 3
Multiply row 1 by .
3
  Multiply row 1 by −2 and add it to row 2. Also, multiply
1 0 0 row 1 by −3 and add it to row 3.
   
0 1 3 1 2 −3 9
 
 0 −5 8 −26 
We have x = 0, y = 3. The solution is (0, 3).
0 −7 5 −24
23. x − 3y = 8,
1
2x − 6y = 3 Multiply row 2 by − to get a 1 in the second row, second
5
Write the augmented matrix. column.

Copyright © 2013 Pearson Education, Inc.


Exercise Set 6.3 231

 
1 2 −3 9 Multiply row 2 by −1 and add it to row 3.
 8 26   
 0 1 − 
 5 5  2 1 2 5
 
0 −7 5 −24  0 −3 −9 −15 
0 0 2 6
Multiply row 2 by −2 and add it to row 1. Also, multiply
row 2 by 7 and add it to row 3. 1
Multiply row 2 by − to get a 1 in the second row, second
 1 7  3
1 0 − column.
 5 5   
 
 8 26  2 1 2 5
 0 1 −   
 5 5   0 1 3 5 
 
31 62
0 0 − 0 0 2 6
5 5
5 Multiply row 2 by −1 and add it to row 1.
Multiply row 3 by − to get a 1 in the third row, third  
31 2 0 −1 0
column.
   
1 7  0 1 3 5 
 1 0 −
 5 5 

0 0 2 6
 8 26 
 0 1 −  1
 5 5  Multiply row 3 by to get a 1 in the third row, third
0 0 1 −2 2
column.
 
1 2 0 −1 0
Multiply row 3 by − and add it to row 1. Also, multiply
5  
8  0 1 3 5 
row 3 by and add it to row 2.
5 0 0 1 3
 
1 0 0 −1 Add row 3 to row 1. Also multiply row 3 by −3 and add
  it to row 2.
 0 1 0 2 
 
0 0 1 −2 2 0 0 3
 
 0 1 0 −4 
We have x = −1, y = 2, z = −2. The solution is
(−1, 2, −2). 0 0 1 3

29. 4x − y − 3z = 1, 1
Finally, multiply row 1 by .
8x + y − z = 5, 2
 3 
2x + y + 2z = 5
1 0 0
 2 
Write the augmented matrix. We will use Gauss-Jordan  
elimination.  0 1 0 −4 
  0 0 1 3
4 −1 −3 1
 
 8 1 −1 5  3
We have x = , y = −4, z = 3. The solution is
2 1 2 5   2
3
, −4, 3 .
First interchange rows 1 and 3 so that each number be- 2
low the first number in the first row is a multiple of that 31. x − 2y + 3z = 4,
number.
  3x + y − z = 0,
2 1 2 5 2x + 3y − 5z = 1
 
 8 1 −1 5  Write the augmented matrix. We will use Gaussian elimi-
4 −1 −3 1 nation.
 
Multiply row 1 by −4 and add it to row 2. Also, multiply 1 −2 3 −4
row 1 by −2 and add it to row 3.  
 3 1 −1 0 
  2 3 −5 1
2 1 2 5
 
 0 −3 −9 −15  Multiply row 1 by −3 and add it to row 2. Also, multiply
0 −3 −7 −9 row 1 by −2 and add it to row 3.

Copyright © 2013 Pearson Education, Inc.


232 Chapter 6: Systems of Equations and Matrices

   
1 −2 3 −4 1 3 1 −1
   
 0 7 −10 12   0 −10 −5 5 
0 7 −11 9 0 −14 −7 7

Multiply row 2 by −1 and add it to row 3. 1


Multiply row 2 by − to get a 1 in the second row, second
  10
1 −2 3 −4 column.
   
 0 7 −10 12  1 3 1 −1
0 0 −1 −3  1 1 
 0 1 − 
 2 2 
1
Multiply row 2 by and multiply row 3 by −1. 0 −14 −7 7
7
  Multiply row 2 by 14 and add it to row 3.
1 −2 3 −4
 10 12   
 0 1 −  1 3 1 −1
 7  
7
 0 1 1 
 1 − 
0 0 1 3 2 2 
0 0 0 0
Now write the system of equations that corresponds to the
last matrix. The last row corresponds to the equation 0 = 0. This
x − 2y + 3z = −4, (1) indicates that the system of equations is dependent. It is
10 12 equivalent to
y− z= , (2)
7 7 x + 3y + z = −1,
z=3 (3) 1 1
y+ z=−
Back-substitute 3 for z in equation (2) and solve for y. 2 2
10 12 We solve the second equation for y.
y− ·3 =
7 7 1 1
y=− z−
30 12 2 2
y− = Substitute for y in the first equation and solve for x.
7 7  
42 1 1
y= =6 x+3 − z− + z = −1
7 2 2
Back-substitute 6 for y and 3 for z in equation (1) and 3 3
x − z − + z = −1
solve for x. 2 2
x − 2 · 6 + 3 · 3 = −4 1 1
x= z+
x − 3 = −4 
2

2
x = −1 1 1 1 1
The solution is z + , − z − , z , where z is any real
2 2 2 2
The solution is (−1, 6, 3). number.
33. 2x − 4y − 3z = 3, 35. p + q + r = 1,
x + 3y + z = −1, p + 2q + 3r = 4,
5x + y − 2z = 2 4p + 5q + 6r = 7
Write the augmented matrix. Write the augmented matrix.
   
2 −4 −3 3 1 1 1 1
   
 1 3 1 −1   1 2 3 4 
5 1 −2 2 4 5 6 7
Interchange the first two rows to get a 1 in the first row, Multiply row 1 by −1 and add it to row 2. Also, multiply
first column. row 1 by −4 and add it to row 3.
 
1 3 1 −1  
  1 1 1 1
 2 −4 −3 3   
 0 1 2 3 
5 1 −2 2
0 1 2 3
Multiply row 1 by −2 and add it to row 2. Also, multiply
Multiply row 2 by −1 and add it to row 3.
row 1 by −5 and add it to row 3.

Copyright © 2013 Pearson Education, Inc.


Exercise Set 6.3 233

 
1 1 1 1 Interchange rows 1 and 2.
 
 0 1 2 3   
1 1 1 1 −5
0 0 0 0  −2 2
 2 −2 −10 

 
The last row corresponds to the equation 0 = 0. This  3 1 −1 4 −2 
indicates that the system of equations is dependent. It is
1 3 −2 2 −6
equivalent to
p + q + r = 1, Multiply row 1 by 2 and add it to row 2. Multiply row 1
q + 2r = 3. by −3 and add it to row 3. Multiply row 1 by −1 and add
it to row 4.
We solve the second equation for q.
 
q = −2r + 3 1 1 1 1 −5
 0 4 0 −20 
Substitute for y in the first equation and solve for p.  4 
 
 0 −2 −4 1 13 
p − 2r + 3 + r = 1
p−r+3 = 1 0 2 −3 1 −1

p = r−2 Interchange rows 2 and 3.


The solution is (r − 2, −2r + 3, r), where r is any real  
1 1 1 1 −5
number.
 0 −2 −4 1 
 13 
37. a + b − c = 7,  
 0 4 4 0 −20 
a − b + c = 5,
0 2 −3 1 −1
3a + b − c = −1
Write the augmented matrix. Multiply row 2 by 2 and add it to row 3. Add row 2 to
row 4.
 
1 1 −1 7  
  1 1 1 1 −5
 1 −1 1 5   0 −2 −4 13 
 1 
3 1 −1 −1  
 0 0 −4 2 6 
Multiply row 1 by −1 and add it to row 2. Also, multiply 0 0 −7 2 12
row 1 by −3 and add it to row 3.
  Multiply row 4 by 4.
1 1 −1 7  
  1 1 1 1 −5
 0 −2 2 −2 
 0 −2
 −4 1 13 

0 −2 2 −22  
 0 0 −4 2 6 
Multiply row 2 by −1 and add it to row 3. 0 0 −28 8 48
 
1 1 −1 7 Multiply row 3 by −7 and add it to row 4.
 
 0 −2 2 −2   
1 1 1 1 −5
0 0 0 −20
 0 −2 −4 13 
 1 
The last row corresponds to the false equation 0 = −20.  
 0 0 −4 2 6 
Thus, the system of equations has no solution.
0 0 0 −6 6
39. −2w + 2x + 2y − 2z = −10,
1 1 1
w + x + y + z = −5, Multiply row 2 by − , row 3 by − , and row 6 by − .
2 4 6
3w + x − y + 4z = −2,  
1 1 1 1 −5
w + 3x − 2y + 2z = −6
 
 0 1 2 − 1 − 13 
Write the augmented matrix. We will use Gaussian elimi-  
 2 2 
nation.  1 3 
 0 0 1 − − 
   2 
−2 2 2 −2 −10 2
 1 1 −5  0 0 0 1 −1
 1 1 
 
 3 1 −1 4 −2  Write the system of equations that corresponds to the last
1 3 −2 2 −6 matrix.

Copyright © 2013 Pearson Education, Inc.


234 Chapter 6: Systems of Equations and Matrices

w+x+ y+ z = −5, (1) We have a system of equations.


1 13 x + y + z = 11, x + y + z = 11,
x + 2y − z = − , (2)
2 2 z = 3x, or −3x + z = 0,
1 3 y = z − 3, y − z = −3
y− z =− , (3)
2 2 Carry out. Using Gaussian elimination or Gauss-Jordan
z = −1 (4) elimination we find that the solution is (2, 3, 6).
Back-substitute in equation (3) and solve for y. Check. The total spending is 2 + 3 + 6, or $11 million.
1 3 The amount spent in 2012, $6 million, is three times the
y − (−1) =− amount spent in 2010, $2 million. The amount spent in
2 2
2011, $3 million, is $3 million less than the amount spent
1 3
y+ =− in 2012, $6 million. The answer checks.
2 2
State. The amounts spent on advertising in 2010, 2011,
y = −2
and 2012 were $2 million, $3 million, and $6 million, re-
Back-substitute in equation (2) and solve for x. spectively.
1 13
x + 2(−2) − (−1) = − 45. The function has a variable in the exponent, so it is an
2 2
exponential function.
1 13
x−4+ = − 47. The function is the quotient of two polynomials, so it is a
2 2
x = −3 rational function.

Back-substitute in equation (1) and solve for w. 49. The function is of the form f (x) = loga x, so it is logarith-
w − 3 − 2 − 1 = −5 mic.

w=1 51. The function is of the form f (x) = mx + b, so it is linear.


The solution is (1, −3, −2, −1). 53. Substitute to find three equations.
41. Familiarize. Let x and y represent the number of 44/
c and 12 = a(−3)2 + b(−3) + c
17/c stamps purchased, respectively. Then Otto spent −7 = a(−1)2 + b(−1) + c
0.44x on 44/c stamps and 0.17y on 17/
c stamps.
−2 = a · 12 + b · 1 + c
Translate. Otto spent a total of $22.35 on stamps. We have a system of equations.
0.44x + 0.17y = 22.35 9a − 3b + c = 12,
He purchased a total of 60 stamps. a − b + c = −7,
x + y = 60 a + b + c = −2
We have a system of equations. Write the augmented matrix. We will use Gaussian elimi-
0.44x + 0.17y = 22.35, 44x + 17y = 2235, nation.
or  
x+ y = 60, x+ y = 60 9 −3 1 12
 
Carry out. Using Gaussian elimination or Gauss-Jordan  1 −1 1 −7 
elimination, we find that the solution is (45, 15). 1 1 1 −2
Check. The total purchase price is $0.44(45) + $0.17(15),
Interchange the first two rows.
or $22.35. The number of stamps purchased is 45 + 15, or
60. The answer checks.  
1 −1 1 −7
State. Otto bought 45 44/
c stamps and 15 17/
c stamps.  
 9 −3 1 12 
43. Familiarize. Let x, y, and z represent the amount spent 1 1 1 −2
on advertising in fiscal years 2010, 2011, and 2012, respec-
tively, in millions of dollars. Multiply row 1 by −9 and add it to row 2. Also, multiply
row 1 by −1 and add it to row 3.
Translate. A total of $11 million was spent on advertis-
 
ing. 1 −1 1 −7
 
x + y + z = 11  0 6 −8 75 
The amount spent in 2012 was three times the amount 0 2 0 5
spent in 2010.
Interchange row 2 and row 3.
z = 3x
 
The amount spent in 2011 was $3 million less than the 1 −1 1 −7
 
amount spent in 2012.  0 2 0 5 
y =z−3 0 6 −8 75

Copyright © 2013 Pearson Education, Inc.


Exercise Set 6.3 235

Multiply row 2 by −3 and add it to row 3. 57. y = x + z,


  3y + 5z = 4,
1 −1 1 −7
  x + 4 = y + 3z, or
 0 2 0 5 
0 0 −8 60 x − y + z = 0,
3y + 5z = 4,
1 1
Multiply row 2 by and row 3 by − . x − y − 3z = −4
2 8
  Write the augmented matrix. We will use Gauss-Jordan
1 −1 1 −7 elimination.
 5   
 0  1 −1
 1 0  1 0
 2   
 15   0 3 5 4 
0 0 1 −
2 1 −1 −3 −4
Write the system of equations that corresponds to the last Multiply row 1 by −1 and add it to row 3.
matrix.  
x − y + z = −7, 1 −1 1 0
 
5  0 3 5 4 
y = ,
2 0 0 −4 −4
15
z=− 1
2 Multiply row 3 by − .
5 15 4
Back-substitute for y and − for z in the first equation  
2 2 1 −1 1 0
and solve for x.  
5 15  0 3 5 4 
x− − = −7
2 2 0 0 1 1
x − 10 = −7
Multiply row 3 by −1 and add it to row 1. Also, multiply
x=3 row 3 by −5 and add it to row 2.
 
5 15  
The solution is 3, , − , so the equation is 1 −1 0 −1
2 2  
 0 3 0 −1 
5 15
y = 3x2 + x − . 0 0 1 1
2 2
  1
1 5 Multiply row 2 by .
55.   3
3 2  
1 −1 0 −1
 1 
Multiply row 1 by −3 and add it to row 2.  0 1 0 − 
 3 
 
1 5 0 0 1 1
 
0 −13 Add row 2 to row 1.
 
4
1  1 0 0 − 
Multiply row 2 by − .  3 
13  1 
   0 1 0 − 
 3 
1 5
  Row-echelon form 0 0 1 1
0 1
Read the solution from the last matrix. It is
 
4 1
Multiply row 2 by −5 and add it to row 1. − ,− ,1 .
3 3
 
1 0 59. x − 4y + 2z = 7,
  Reduced row-echelon form
3x + y + 3z = −5
0 1
Write the augmented matrix.
 
1 −4 2 7
 
3 1 3 −5

Copyright © 2013 Pearson Education, Inc.


236 Chapter 6: Systems of Equations and Matrices

Multiply row 1 by −3 and add it to row 2. Multiply row 2 by −1 and add it to row 3.
   
1 −4 2 7 4 5 3
   
 0 1 3 
0 13 −3 −26
0 0 0
1
Multiply row 2 by . The last row corresponds to the equation 0 = 0. Thus we
13
  have a dependent system that is equivalent to
1 −4 2 7 4x + 5y = 3, (1)
 
  y = 3. (2)
3
0 1 − −2
13 Back-substitute in equation (1) to find x.
Write the system of equations that corresponds to the last 4x + 5 · 3 = 3
matrix. 4x + 15 = 3
x − 4y + 2z = 7,
4x = −12
3
y − z = −2 x = −3
13
Solve the second equation for y. The solution is (−3, 3).
3
y= z−2
13 Exercise Set 6.4
Substitute in the first equation and solve for x.
 
3    
x−4 z − 2 + 2z = 7 1. 5 x = y −3
13
12 Corresponding entries of the two matrices must be equal.
x − z + 8 + 2z = 7 Thus we have 5 = y and x = −3.
13
   
14
x = − z−1 3 2x 3 −2
3. =
 13  y −8 1 −8
14 3
The solution is − z − 1, z − 2, z , where z is any Corresponding entries of the two matrices must be equal.
13 13 Thus, we have:
real number.
2x = −2 and y = 1
61. 4x + 5y = 3, x = −1 and y = 1
−2x + y = 9,    
1 2 −3 5
3x − 2y = −15 5. A + B =
4 3
+
2 −1
Write the augmented matrix.  
  =
1 + (−3) 2 + 5
4 5 3 4+2 3 + (−1)
   
 −2 1 9 
−2 7
3 −2 −15 =
6 2
   
Multiply row 2 by 2 and row 3 by 4. 1 3 0 0
  7. E + 0 = +
4 5 3 2 6 0 0
   
 −4 2 18  1+0 3+0
=
12 −8 −60 2+0 6+0
 
Add row 1 to row 2. Also, multiply row 1 by −3 and add 1 3
=
it to row 3. 2 6
   
4 5 3 3 3
9. 3F = 3
  −1 −1
 0 7 21 
 
0 −23 −69 3·3 3·3
=
3 · (−1) 3 · (−1)
1 1  
Multiply row 2 by and row 3 by − .
7 23 9 9
=
  −3 −3
4 5 3
 
 0 1 3 
0 1 3

Copyright © 2013 Pearson Education, Inc.


Exercise Set 6.4 237
 
    −2 4  
3 3 9 9  3 −6
11. 3F = 3 = , 23. 5 1 
−1 −1 −3 −3 −1 4
    −1 −3
1 2 2 4  
2A = 2 = −2 · 3 + 4(−1) −2(−6) + 4 · 4
4 3 8 6 = 5 · 3 + 1(−1) 5(−6) + 1 · 4 
    −1 · 3 + (−3)(−1) −1(−6) + (−3) · 4
3F + 2A =
9 9
+
2 4  
−3 −3 8 6 −10 28
  =  14 −26 
9+2
=
9+4 0 −6
−3 + 8 −3 + 6
   
11 13 1  
= −6 5 8
5 3 25.  −5 
    0 4 −1
3
−3 5 1 2
13. B−A = −
2 −1 4 3 This product is not defined because the number of columns
    of the first matrix, 1, is not equal to the number of rows
−3 5 −1 −2 of the second matrix, 2.
= +
2 −1 −4 −3
  
[B − A = B + (−A)] 1 −4 3 3 0 0
 
27.  0 8 0   0 −4 0 
−3 + (−1) 5 + (−2) −2 −1 5 0 0 1
=  
2 + (−4) −1 + (−3)
3 + 0 + 0 0 + 16 + 0 0+0+3
 
−4 3 =  0 + 0 + 0 0 − 32 + 0 0+0+0 
= −6 + 0 + 0
−2 −4 0+4+0 0+0+5
 
   3 16 3
−3 5 1 2
15. BA = =  0 −32 0 
2 −1 4 3
−6 4 5
 
−3 · 1 + 5 · 4 −3 · 2 + 5 · 3  
= 29. a) A = 40 20 30
2 · 1 + (−1)4 2 · 2 + (−1)3
  b) 40 + 10% · 40 = 1.1(40) = 44
17 9
= 20 + 10% · 20 = 1.1(20) = 22
−2 1
   30 + 10% · 30 = 1.1(30) = 33
1 −1 1 1
17. CD = We write the matrix that corresponds to these
−1 1 1 1
amounts.
   
1 · 1 + (−1) · 1 1 · 1 + (−1) · 1 B = 44 22 33
=    
−1 · 1 + 1 · 1 −1 · 1 + 1 · 1
c) A + B = 40 20 30 + 44 22 33
   
0 0 = 84 42 63
=
0 0
The entries represent the total amount of each type
  
1 2 1 0 of produce ordered for both weeks.
19. AI =  
4 3 0 1
31. a) C = 140 27 3 13 64
   
1·1+2·0 1·0+2·1 P = 180 4 11 24 662
=
4·1+3·0 4·0+3·1  
B = 50 5 1 82 20
 
1 2 b) C + 2P + 3B
=
4 3  
  = 140
 27 3 13 64 + 
  6 360 8 22 48 1324 +
−1 0 7 
21. −4   
3 −5 2 150 15 3 246 60
1  
  = 650 50 28 307 1448
−1 · 6 + 0(−4) + 7 · 1
= The entries represent the total nutritional value of
3 · 6 + (−5)(−4) + 2 · 1
  one serving of chicken, 1 cup of potato salad, and 3
1 broccoli spears.
=
40  
1.50 0.15 0.26 0.23 0.64
 
33. a) M =  1.55 0.14 0.24 0.21 0.75 
 1.62 0.22 0.31 0.28 0.53 
1.70 0.20 0.29 0.33 0.68

Copyright © 2013 Pearson Education, Inc.


238 Chapter 6: Systems of Equations and Matrices

 
b) N = 65 48 93 57 45. −4w + x − y + 2z = 12,
 
c) NM = 419.46 48.33 73.78 69.88 165.65 w + 2x − y − z = 0,
d) The entries of NM represent the total cost, in dol- −w + x + 4y − 3z = 1,
lars, of each item for the day’s meals. 2w + 3x + 5y − 7z = 9
 
900 500 Write the coefficients on the left in a matrix. Then write
35. a) M =  450 1000  the product of that matrix and the column matrix contain-
600 700 ing the variables, and set the result equal to the column
  matrix containing the constants on the right.
b) P = 5 8 4     
  −4 1 −1 2 w 12
c) PM = 10, 500 13, 300  1 2 −1 −1   x   0 
  = 
d) The entries of PM represent the total profit from  −1 1 4 −3   y   1 
each distributor. 2 3 5 −7 z 9

  47. f (x) = x2 − x − 6
37. a) C = 20 25 15 b −1 1
  a) − =− =
 900  500 2a 2·1 2
b) CM = 20 25 15  450    2
1000  1 1 1 25
600 700 f = − −6=−
  2 2 2 4
= 38, 250 45, 500  
1 25
The total production costs for the products shipped The vertex is ,− .
2 4
to Distributors 1 and 2 are $38,250 and $45,500,
respectively. 1
b) The axis of symmetry is x = .
2
39. 2x − 3y = 7, 2
c) Since the coefficient of x is positive, the function
x + 5y = −6 has a minimum value. It is the second coordinate
25
Write the coefficients on the left in a matrix. Then write of the vertex, − .
the product of that matrix and the column matrix contain- 4
ing the variables, and set the result equal to the column d) Plot some points and draw the graph of the function.
matrix containing the constants on the right.
     y
2 −3 x 7
=
1 5 y −6
2
41. x + y − 2z = 6, 4 2 2 4 x
3x − y + z = 7, 2

2x + 5y − 3z = 8 4

Write the coefficients on the left in a matrix. Then write 6

the product of that matrix and the column matrix contain-


ing the variables, and set the result equal to the column f(x)  x 2  x  6
matrix containing the constants on the right.
    
1 1 −2 x 6 49. f (x) = −x2 − 3x + 2
 3 −1 1  y  =  7  b −3 3
2 5 −3 z 8 a) − =− =−
2a 2(−1) 2
43. 3x − 2y + 4z = 17,    2  
3 3 3 17
f − =− − −3 − +2=
2x + y − 5z = 13 2 2 2 4
 
Write the coefficients on the left in a matrix. Then write 3 17
the product of that matrix and the column matrix contain- The vertex is − , .
2 4
ing the variables, and set the result equal to the column
matrix containing the constants on the right. 3
  b) The axis of symmetry is x = − .
  x   2
3 −2 4  17
y = c) Since the coefficient of x2 is negative, the function
2 1 −5 13 has a maximum value. It is the second coordinate
z
17
of the vertex, .
4
d) Plot some points and draw the graph of the function.

Copyright © 2013 Pearson Education, Inc.


Chapter 6 Mid-Chapter Mixed Review 239

y 5. 2x + y = −4, (1)
x=y−5 (2)
4
Substitute y − 5 for x in equation (1) and solve for y.
2
2(y − 5) + y = −4
4 2 2 4 x
2
2y − 10 + y = −4
4
3y − 10 = −4
3y = 6
f(x)  x 2  3x  2 y=2

    Back-substitute in equation (2) to find x.


−1 0 −1 1 x = 2 − 5 = −3
51. A = ,B=
2 1 2 0
   The solution is (−3, 2).
0 −1 −2 1
(A + B)(A − B) = 7. 2x − 3y = 8,
2 3 2 −1 (1)
  3x + 2y = −1 (2)
−2 1
=
2 −1 Multiply equation (1) by 2 and equation (2) by 3 and add.
A2 − B2 4x − 6y = 16
      9x + 6y = −3
−1 0 −1 0 1 −1 1 −1
= −
2 1 2 1 0 2 0 2 13x = 13
    x=1
1 0 1 −3
= − Back-substitute and solve for y.
0 1 0 4
  3 · 1 + 2y = −1 Using equation (2)
0 3
=
0 −3 3 + 2y = −1

Thus (A + B)(A − B) = A2 − B2 . 2y = −4
y = −2
 Exercise 51 we found that (A + B)(A − B) =
53. In
The solution is (1, −2).
−2 1
2 −1 9. x + 2y + 3z = 4, (1)
and we also found A2 and B2 . x − 2y + z = 2, (2)
    
1 −1 −1 0 −3 −1 2x − 6y + 4z = 7 (3)
BA = =
0 2 2 1 4 2
Multiply equation (1) by −1 and add it to equation (2).
    
−1 0 1 −1 −1 1 Multiply equation (1) by −2 and add it to equation (3).
AB = =
2 1 0 2 2 0 x + 2y + 3z = 4 (1)
A + BA − AB − B
2 2
− 4y − 2z = −2 (4)
     
1 0 −3 −1 −1 1 − 10y − 2z = −1 (5)
= + − −
0 1 4 2 2 0
Multiply equation (5) by 2.
 
1 −3 x + 2y + 3z = 4 (1)
0 4
− 4y − 2z = −2 (4)
 
−2 1 − 20y − 4z = −2 (6)
=
2 −1
Multiply equation (4) by −5 and add it to equation (6).
Thus (A + B)(A − B) = A2 + BA − AB − B2 . x + 2y + 3z = 4 (1)
− 4y − 2z = −2 (4)
Chapter 6 Mid-Chapter Mixed Review 6z = 8 (7)
Solve equation (7) for z.
1. False; see page 485 in the text. 6z = 8
4
3. True; see pages 522 and 523 in the text. z=
3

Copyright © 2013 Pearson Education, Inc.


240 Chapter 6: Systems of Equations and Matrices

   
4
Back-substitute for z in equation (4) and solve for y. −2 3 0 −8 12 0
  3 15. 4D = 4  1 −1 2  =  4 −4 8 
4 −3 4 1 −12 16 4
−4y − 2 = −2
3   
3 −1 −2 6
8 17. AB =
−4y − = −2 5 4 1 −3
3
 
2 3(−2) + 9 + (−1) · 1 3 · 6 + (−1)(−3)
−4y = =
3 5(−2) + 4 · 1 5 · 6 + 4(−3)
1  
y=− −7 21
6 =
1 4 −6 18
Back-substitute − for y and for z in equation (1) and
6 3
 x. 
solve for 19. BC
  
1 4
x+2 − +3· = 4 −2 6 −4 1 −1
6 3 =
1 −3 2 3 −2
1  
x− +4 = 4
3 −2(−4)+6 ·2 −2·1 + 6·3 −2(−1) + 6(−2)
=
11 1(−4)+(−3)·2 1·1+(−3) · 3 1(−1)+(−3)(−2)
x+ =4  
3
20 16 −10
1 =
x= −10 −8 5
 3
1 1 4 21. A matrix equation equivalent to the given system is
The solution is ,− , .     
3 6 3 2 −1 3 x 7
11. 2x + y = 5,  1 2 −1   y  =  3 .
3 −4 2 z 5
3x + 2y = 6
Write the augmented matrix. We will use Gauss-Jordan 23. Add a non-zero multiple of one equation to a non-zero
elimination. multiple of the other equation, where the multiples are
 
2 1 5 not opposites.
 
3 2 6 25. No; see Exercise 17 on page 526 in the text, for example.

Multiply row 2 by 2.
  Exercise Set 6.5
2 1 5
 
    
6 4 12 7 3 1 −3 1 0
1. BA = =
2 1 −2 7 0 1
Multiply row 1 by −3 and add it to row 2.     
  1 −3 7 3 1 0
2 1 5 AB = =
  −2 7 2 1 0 1
0 1 −3 Since BA = I = AB, B is the inverse of A.
  
Multiply row 2 by −1 and add it to row 1. 2 3 2 −1 −1 6
  3. BA =  3 3 4   1 0 −2  =
2 0 8
  1 1 1 1 0 −3
 
0 1 −3 3 −2 0
 4 −3 0 
1
Multiply row 1 by . 1 −1 1
  2 Since BA = I, B is not the inverse of A.
1 0 4  
  3 2
5. A =
0 1 −3 5 3

The solution is (4, −3). Write the augmented matrix.


 
    3 2 1 0
3 −1 −2 6
13. A + B = + 5 3 0 1
5 4 1 −3
  Multiply row 2 by 3.
 
3 + (−2) −1 + 6 3 2 1 0
=
5+1 4 + (−3) 15 9 0 3
 
1 5 Multiply row 1 by −5 and add it to row 2.
=
6 1

Copyright © 2013 Pearson Education, Inc.


Exercise Set 6.5 241

 
3 2 1 0 1
Multiply row 3 by .
0 −1 −5 3 4
 1 1 1 
Multiply row 2 by 2 and add it to row 1.
  1 0 − − 0
 2 2 2 
3 0 −9 6  
 3 1 3 
0 −1 −5 3  0 1 − 0 
 2 2 2 
1  
Multiply row 1 by and row 2 by −1. 1 1 1
3 0 0 1 −
  4 2 4
1 0 −3 2 1
0 1 5 −3 Multiply row 3 by and add it to row 1. Also, multiply
2
  3
−3 2 row 3 by − and add it to row 2.
Then A−1 = . 2
5 −3  3 1 1 
  1 0 0 −
6 9  8 4 8 
7. A =  
4 6  1 3 3 
 0 1 0 − − 
 8 4 8 
Write the augmented matrix.
   
1 1 1
6 9 1 0 0 0 1 −
4 2 4
4 6 0 1  3 1 1 

Multiply row 2 by 3.
   8 4 8 
 
6 9 1 0 −1  1 3 3 
Then A =  − − .
12 18 0 3  8 4 8 
 
Multiply row 1 by −2 and add it to row 2. 1 1 1
  −
4 2 4
6 9 1 0  
0 0 −2 3 1 −4 8
We cannot obtain the identity matrix on the left since the 11. A =  1 −3 2 
second row contains only zeros to the left of the vertical 2 −7 10
line. Thus, A−1 does not exist. Write the augmented matrix.
   
3 1 0 1 −4 8 1 0 0
9. A =  1 1 1   1 −3 2 0 1 0 
1 −1 2 2 −7 10 0 0 1
Write the augmented matrix. Multiply row 1 by −1 and add it to row 2. Also, multiply
  row 1 by −2 and add it to row 3.
3 1 0 1 0 0  
 1 1 1 0 1 0  1 −4 8 1 0 0
1 −1 2 0 0 1  0 1 −6 −1 1 0 
0 1 −6 −2 0 1
Interchange the first two rows.
  Since the second and third rows are identical left of the
1 1 1 0 1 0
 3 1 0 1 0 0  vertical line, it will not be possible to obtain the identity
1 −1 2 0 0 1 matrix on the left side. Thus, A−1 does not exist.
 
Multiply row 1 by −3 and add it to row 2. Also, multiply 4 −3
13. A =
row 1 by −1 and add it to row 3. 1 −2
 
1 1 1 0 1 0 Write the augmented matrix.
 0 −2 −3 1 −3 0   
4 −3 1 0
0 −2 1 0 −1 1 1 −2 0 1
1
Multiply row 2 by − . Interchange the rows.
 
2
  1 −2 0 1
1 1 1 0 1 0 4 −3 1 0
 3 1 3 
 0 1 − 0  Multiply row 1 by −4 and add it to row 2.
 2 2 2   
0 −2 1 0 −1 1 1 −2 0 1
0 5 1 −4
Multiply row 2 by −1 and add it to row 1. Also, multiply
row 2 by 2 and add it to row 3. 1
  Multiply row 2 by .
5
1 0 −
1 1

1
0  
 2 2 2  1 −2 0 1
  
 3 1 3  1 4 
 0 1 − 0  0 1 −
 2 2 2  5 5
0 0 4 −1 2 1 Multiply row 2 by 2 and add it to row 1.

Copyright © 2013 Pearson Education, Inc.


242 Chapter 6: Systems of Equations and Matrices

 2 3 
1 0 − Multiply row 1 by 2 and add it to row 2. Also, multiply
 5 5  row 1 by −1 and add it to row 3.
   
1 4
0 1 − 1 2 −1 1 0 0
5 5  0 4 −1 2 1 0 
 2 3 
−   0 −3 1 −1 0 1
 5 5  0.4 −0.6
Then A−1 = , or 0.2 . Add row 3 to row 1 and also to row 2.
1 4 −0.8  
− 1 −1 0 0 0 1
5 5  0 1 0 1 1 1 
The x−1 key on a graphing calculator can also be used 0 −3 1 −1 0 1
to find A−1 .
  Add row 2 to row 1. Also, multiply row 2 by 3 and add it
2 3 2 to row 3.
 
15. A =  3 3 4  1 0 0 1 1 2
−1 −1 −1  0 1 0 1 1 1 
Write the augmented matrix. 0 0 1 2 3 4
   
2 3 2 1 0 0 1 1 2
 3 3 4 0 1 0  Then A−1 =  1 1 1 .
−1 −1 −1 0 0 1 2 3 4
Interchange rows 1 and 3. The x−1 key on a graphing calculator can also be used
 
−1 −1 −1 0 0 1 to find A−1 .
 3 3 4 0 1 0   
1 3 −1
2 3 2 1 0 0
19. A =  0 2 −1 
Multiply row 1 by 3 and add it to row 2. Also, multiply 1 1 0
row 1 by 2 and add it to row 3.
  Write the augmented matrix.
−1 −1 −1 0 0 1  
 0 1 3 −1 1 0 0
0 1 0 1 3   0 2 −1 0 1 0 
0 1 0 1 0 2
1 1 0 0 0 1
Multiply row 1 by −1.
  Multiply row 1 by −1 and add it to row 3.
1 1 1 0 0 −1  
 0 0 1 3 −1 1 0 0
1 0 1 3   0 2 −1 0 1 0 
0 1 0 1 0 2
0 −2 1 −1 0 1
Interchange rows 2 and 3.
  Add row 3

to row 1 and also to row 2.

1 1 1 0 0 −1 1 1 0 0 0 0
 0 1 0 2   0
1 0 0 0 −1 1 1 
0 0 1 0 1 3 0 −2 1 −1 0 1
Multiply row 2 by −1 and add it to row 1.
  Since the second row consists only of zeros to the left of the
1 0 1 −1 0 −3 vertical line, it will not be possible to obtain the identity
 0 1 0 1 0 2  matrix on the left side. Thus, A−1 does not exist. A
0 0 1 0 1 3 graphing calculator will return an error message when we
row 3 by −1 and add it to row 1.
Multiply try to find A−1 .
   
1 0 0 −1 −1 −6 1 2 3 4
 0 1 0 1 0 2   0 1 3 −5 
0 0 1 0 1 3 21. A =  0 0 1 −2 

 
−1 −1 −6 0 0 0 −1
Then A =  1
−1
0 2 . Write the augmented matrix.
0 1 3  
1 2 3 4 1 0 0 0
 0 1 3 −5 0 1 0 0 
The x−1 key on a graphing calculator can also be used  
 0 0 1 −2 0 0 1 0 
to find A−1 .
  0 0 0 −1 0 0 0 1
1 2 −1
Multiply row 4 by −1.
17. A =  −2 0 1   
1 −1 0 1 2 3 4 1 0 0 0
 0 1 3 −5 0 1 0 0 
Write the augmented matrix.  
   0 0 1 −2 0 0 1 0 
1 2 −1 1 0 0 0 0 0 1 0 0 0 −1
 −2 0 1 0 1 0 
1 −1 0 0 0 1

Copyright © 2013 Pearson Education, Inc.


Exercise Set 6.5 243

 19 19 
Multiply row 4 by −4 and add it to row 1. Multiply row 1 −14 −31 0 1 − 0 −
4 by 5 and add it to row 2. Also, multiply row 4 by 2 and  2 2 
 
add it to row 3.  0 −12 −28 0 1 −8 0 −9 
   
 0 16 36 0 −1 11 1 11 
1 2 3 0 1 0 0 4  
 0 1 3 0 0 1 0 −5  1 1
  0 0 1 1 0 0
 0 0 1 0 0 0 1 −2  4 4
0 0 0 1 0 0 0 −1 Multiply row 3 by
1
.
row 3 by −3 and add it to row 1 and to row 2. 36
Multiply  
  19 19
1 2 0 0 1 0 −3 10  1 −14 −31 0 1 −
2
0 −
2 
 0 1 0 0 0 1 −3 1   
   0 −12 −28 −8 −9 
 0 0 1 0 0 0 1 −2   0 1 0 
 4 1 11 1 11 
0 0 0 1 0 0 0 −1  0 0 − 
 1
36 
 9 36 36 36 
row 2 by −2 and add it to row 1.
Multiply  1 1 
  0 0 1 1 0 0
1 0 0 0 1 −2 3 8 4 4
 0 1 0 0 0 1 −3 1 
  Multiply row 3 by 31 and add it to row 1. Multiply row
 0 0 1 0 0 0 1 −2  3 by 28 and add it to row 2. Also, multiply row 3 by −1
0 0 0 1 0 0 0 −1 and add it to row 4.
   
1 −2 3 8 2 5 1 31 1
1 − 0 0 − −
 1 −3 1   9 36 36 36 36 
Then A−1 = 
0 .  
 0 1 −2   4 2 5 7 4 
0  0 0 0 − 
0 −1  9 
0 0  9 9 9 9 
 11 
 0 4
1 0 −
1 11 1 
The x−1 key on a graphing calculator can also be used 
 9 36 36 36 36  
to find A−1 . 
  4 1 1 1 1 
0 − 0 1 − − −
1 −14 7 38 9 36 18 36 18
 −1 2 1 −2 
23. A =   1
 1 2 −1 −6  Multiply row 2 by and add it to row 1. Also, multiply
2
1 −2 3 6 row 2 by −1 and add it to row 3. Add row 2 to row 4.
 
Write the augmented matrix. 1 1 5 1
  1 0 0 0 −
 4 4 4 4 
1 −14 7 38 1 0 0 0  
 −1  4 2 5 7 4 
 2 1 −2 0 1 0 0 
  0
 0 0 − 
 1 2 −1 −6 0 0 1 0   9 9 9 9 9 
 3 
1 −2 3 6 0 0 0 1  0 0 1 0 −1 −1 −3 
 4 
to row 2. Also, multiply row 1 by −1 and add  4 4 4 
Add row 1  1 1 3 1 
it to

row 3 and to row 4.
 0 0 0 1 −
4 2 4 2
1 −14 7 38 1 0 0 0
 0 9
 −12 8 36 1 1 0 0   Multiply row 2 by .
 0 −8 −44 −1 0 1 0  4
16  
1 1 5 1
0 12 −4 −32 −1 0 0 1 1 0 0 0 −
 4 4 4 4 
 
Add row 2 to row 4.
   1 5 7 
1 −14  0 1 0 0 −1 
7 38 1 0 0 0  
 0 −12 1 1 0 0   2 4 4 
 8 36   
 0 16 −8 −44 −1 0 1 0   0 0 1 0 − 1

1

3 3 
 4 
 4 4 4 
0 0 4 4 0 1 0 1  1 1 3 1 
1 0 0 0 1 −
Multiply row 4 by . 4 2 4 2
4
   
1 −14 7 38 1 0 0 0 1 1 5 1
 0 −12 1 0 0  −
 8 36 1   4 4 4 4 
 0  
16 −8 −44 −1 0 1 0   
   1 5 7
−1 
1 1  
= , or
0 0 1 1 0 0 2 4 4
Then A−1 
4 4
 1 1 3 3 

Multiply row 4 by −38 and add it to row 1. Multiply row  − − −
 4 4 4 4 

4 by −36 and add it to row 2. Also, multiply row 4 by 44  1 1 3 1 
and add it to row 3. −
4 2 4 2

Copyright © 2013 Pearson Education, Inc.


244 Chapter 6: Systems of Equations and Matrices

 
0.25 0.25 1.25 −0.25 33. x + z = 1,
 0.5 1.25 1.75 −1 
 . 2x + y = 3,
 −0.25 −0.25 −0.75 0.75 
0.25 0.5 0.75 −0.5 x−y+z=4
Write an equivalent matrix equation, AX = B.
The x−1 key on a graphing calculator can also be used     
to find A−1 . 1 0 1 x 1
 2 1 0  y  =  3 
25. Write an equivalent matrix equation, AX = B. 1 −1 1 z 4
      
11 3 x −4 1 1 1
= −    
7 2 y 5  2 2 2 1 3
 
Then we have X = A−1 B. Then X = A−1 B =    
 1 0 −1  3 = −3 .

        3 1 1 4 −2
x 2 −3 −4 −23
= = − −
y −7 11 5 83 2 2 2
The solution is (−23, 83). The solution is (3, −3, −2).

27. Write an equivalent matrix equation, AX = B. 35. 2x + 3y + 4z = 2,


    
3 1 0 x 2 x − 4y + 3z = 2,
 2 −1 2   y  =  −5  5x + y + z = −4
1 1 1 z 5 Write an equivalent matrix equation, AX = B.
Then we have X = A−1 B.     
       2 3 4 x 2
x 3 1 −2 2 −9  1 −4 3   y  =  2 
1 1
 y  =  0 −3 6   −5  =  45  = 5 1 1 z −4
9 9
z −3 2 5 5 9  1 1 25 
  −
−1  16 112 112     
 5    2 −1
 1 9 1
−1
Then X=A B =   − −   2  =  0 .
1
 8 56 56 
 −4 1
The solution is (−1, 5, 1).  
3 13 11

16 112 112
29. 4x + 3y = 2, The solution is (−1, 0, 1).
x − 2y = 6 37. 2w − 3x + 4y − 5z = 0,
Write an equivalent matrix equation, AX = B. 3w − 2x + 7y − 3z = 2,
    
4 3 x
=
2 w + x − y + z = 1,
1 −2 y 6
−w − 3x − 6y + 4z = 6
 2 3     Writean equivalent matrix equation, AX = B.
 11 11  2 2     
Then X = A−1 B =   6 = −2 . 2 −3 4 −5 w 0
1 4
−  3 −2 7 −3     
11 11   x  =  2 
 1 1 −1 1  y   1 
The solution is (2, −2).
−1 −3 −6 4 z 6
  
31. 5x + y = 2, 26 11 127 9 0
3x − 2y = −4 1  −8 −19 39 −34   
Then X = A−1 B =   2  =
Write an equivalent matrix equation, AX = B. 203 −37 39 −48 −5  1 
 2 −55 47 −11 20
1 
6
 
    1
5 1 x  13 13  2 −1 
=  =  
3 −2 y 3 5 −4  0 .

13 13
  1
2
The solution is (1, −1, 0, 1).
−4
 
0 39. Familiarize. Let x = the number of hot dogs sold and
Then X = A−1 B = . y = the number of sausages.
2
The solution is (0, 2). Translate.
The total number of items sold was 145.
x + y = 145

Copyright © 2013 Pearson Education, Inc.


Exercise Set 6.5 245

The number of hot dogs sold is 45 more than the number Check. Four tons of topsoil, 3 tons of mulch, and 6 tons
of sausages. of pea gravel costs 4 · $239 + 3 · $179 + 6 · $222, or $956 +
x = y + 45 $537 + $1332, or $2825. Five tons of topsoil, 2 tons of
mulch, and 5 tons of pea gravel costs 5 · $239 + 2 · $179 +
We have a system of equations: 5 · $222, or $1195 + $358 + $1110, or $2663. The price of
x + y = 145, x + y = 145, pea gravel, $222, is $17 less than the price of topsoil, $239.
or The solution checks.
x = y + 45, x − y = 45.
Carry out. Write an equivalent matrix equation, State. The price of topsoil is $239 per ton, of mulch is
AX = B. $179 per ton, and of pea gravel is $222 per ton.
     
1 1 x
=
145 43. −2  1 −6 4 −8
1 −1 y 45 −2 16 −40
 
1 1 1 −8 20 −48
   
 2 2  f (−2) = −48
Then X = A−1 B =   145 = 95 , so the
 1 1  45 50
− 45. 2x2 + x = 7
2 2
solution is (95, 50). 2x + x − 7 = 0
2

Check. The total number of items is 95 + 50, or 145. The a = 2, b = 1, c = −7


number of hot dogs, 95, is 45 more than the number of √
−b ± b2 − 4ac
sausages. The solution checks. x=
2a
State. Kayla sold 95 hot dogs and 50 Italian sausages.  √
−1 ± 12 − 4 · 2 · (−7) −1 ± 1 + 56
= =
41. Familiarize. Let x, y, and z represent the prices of one 2·2 4

ton of topsoil, mulch, and pea gravel, respectively. −1 ± 57
=
Translate. 4
√ √
Four tons of topsoil, 3 tons of mulch, and 6 tons of pea −1 + 57 −1 − 57
The solutions are and , or
gravel costs $2825. √ 4 4
−1 ± 57
4x + 3y + 6z = 2825 .
4
Five tons of topsoil, 2 tons of mulch, and 5 tons of pea √ √
gravel costs $2663. 47. 2x + 1 − 1 = 2x − 4
√ √
5x + 2y + 5z = 2663 ( 2x + 1 − 1)2 = ( 2x − 4)2 Squaring
√ both sides
Pea gravel costs $17 less per ton than topsoil. 2x + 1 − 2 2x + 1 + 1 = 2x − 4
z = x − 17 √
2x + 2 − 2 2x + 1 = 2x − 4
We have a system of equations. √
2 − 2 2x + 1 = −4 Subtracting 2x
4x + 3y + 6z = 2825, √
−2 2x + 1 = −6 Subtracting 2
5x + 2y + 5z = 2663, √
2x + 1 = 3 Dividing by −2
z = x − 17, or √ 2 2
( 2x + 1) = 3 Squaring both
sides
4x + 3y + 6z = 2825,
2x + 1 = 9
5x + 2y + 5z = 2663,
2x = 8
x − z = 17
x=4
Carry out. Write an equivalent matrix equation,
The number 4 checks. It is the solution.
AX = B.
    
4 3 6 x 2825 49. f (x) = x3 − 3x2 − 6x + 8
 5 2 5   y  =  2663  We use synthetic division to find one factor. We first try
1 0 −1 z 17 x − 1.
  

1 3 3
  1  1 −3 −6 8
 5 10 10  2825 1 −2 −8
 
Then X = A−1 B =   1 −1 1 
 2663
 = 1 −2 −8 0
 1  17

3

7 Since f (1) = 0, x − 1 is a factor of f (x). We have f (x) =
  5 10 10 (x − 1)(x2 − 2x − 8). Factoring the trinomial we get f (x) =
239 (x − 1)(x − 4)(x + 2).
 179 , so the solution is (239, 179, 222).
222

Copyright © 2013 Pearson Education, Inc.


246 Chapter 6: Systems of Equations and Matrices

 
 0 −3 
51. A = [x] M11 =   = 0(−5) − 2(−3) = 0 + 6 = 6
2 −5 
Write the augmented matrix.
  M32 is the determinant of the matrix formed by deleting
x 1
the third  second column of A:
 row and
1  7 −6 
Multiply by . M32 =    = 7(−3) − 2(−6) = −21 + 12 = −9
 
x 2 −3 
1 M22 is the determinant of the matrix formed by deleting
1
x the second row and second column of A:
  
1 7 −6 
Then A−1 exists if and only if x = 0. A−1 = M22 =  = 7(−5) − 1(−6) = −35 + 6 = −29
x 1 −5 
 
0 0 x 11. In Exercise 9 we found that M11 = 6.
53. A =  0 y 0  A11 = (−1)1+1 M11 = 1 · 6 = 6
z 0 0
In Exercise 9 we found that M32 = −9.
Write the augmented matrix.
  A32 = (−1)3+2 M32 = −1(−9) = 9
0 0 x 1 0 0
 0 y 0 0 1 0  In Exercise 9 we found that M22 = −29.
z 0 0 0 0 1 A22 = (−1)2+2 (−29) = 1(−29) = −29
Interchange row 1 and row 3.  
  7 −4 −6
z 0 0 0 0 1 13. A =  2 0 −3 
 0 y 0 0 1 0  1 2 −5
0 0 x 1 0 0
|A|
1 1 1
Multiply row 1 by , row 2 by , and row 3 by . = 2A21 + 0A22 + (−3)A23
z y x    
  −4 −6   −4 
1  = 2(−1)2+1   +0+(−3)(−1)2+3  7

1 0 0 0 0 2 −5  1 2 
z 
 
 1  = 2(−1)[−4(−5) − 2(−6)] + 0+
 0 1 0 0 0 
 y 
  (−3)(−1)[7 · 2 − 1(−4)]
1
0 0 1 0 0
x = −2(32) + 0 + 3(18) = −64 + 0 + 54
Then A−1 exists if and only if x = 0 and y = 0 and z = 0, = −10
or if and only if xyz = 0.
  
1  7 −4 −6
0 0
 z  15. A =  2 0 −3 
 
−1  1  1 2 −5
A = 0 0 
 y  |A|
 
1
0 0 = −6A13 + (−3)A23 + (−5)A33
x    
2 0  
= −6(−1)1+3   + (−3)(−1)2+3  7 −4  +
1 2 1 2 
Exercise Set 6.6  
 7 −4 
  (−5)(−1)3+3  
1.  5 3  2 0 
 −2 −4  = 5(−4) − (−2) · 3 = −20 + 6 = −14 = −6 · 1(2 · 2 − 1 · 0) + (−3)(−1)[7 · 2 − 1(−4)]+
  −5 · 1(7 · 0 − 2(−4))
3.  4 −7 
 −2 3  = 4 · 3 − (−2)(−7) = 12 − 14 = −2 = −6(4) + 3(18) − 5(8) = −24 + 54 − 40
 √  = −10
5.  −2
√ − 5  √ √
− 5 = −2 · 3 − (− 5)(− 5) = −6 − 5 = −11
3  17. Enter A and then use the determinant operation, det, from
  the MATRIX MATH menu to evaluate |A|. We find that
7.  x 4 
 x x  = x · x − x · 4 = x − 4x |A| = −10.
2 3
2
 
  1 0 0 −2
7 −4 −6  4 1 0 0 
9. A =  2 0 −3  19. A =  5

6 7 8 
1 2 −5
−2 −3 −1 0
M11 is the determinant of the matrix formed by deleting
M41 is the determinant of the matrix formed by deleting
the first row and first column of A:
the fourth row and the first column of A.

Copyright © 2013 Pearson Education, Inc.


Exercise Set 6.6 247

   
0 0 −2  1 0 −2 
 
M41 = 1 0 0  −1 ·  4 1 0 
6 7 8 5 6 8 
We will expand M41 across the first row.     
    1 0 
1+2 4 0

0 0   = −1 1(−1)1+1   + 0(−1)  +
M41 = 0(−1)1+1   + 0(−1)1+2  1 0  + 6 8 5 8

7 8   6 8
  
 
1 0 
1+3  4 1 

(−2)(−1)3+1  (−2)(−1) 
5 6
6 7
= 0 + 0 + (−2)(1)(1 · 7 − 6 · 0)
= −1[1·1(1·8−6 · 0) + 0 + (−2)·1(4·6−5·1)]
= 0 + 0 − 2(7) = −14
= −1[1(8) + 0 − 2(19)]
M33 is the determinant of the matrix formed by deleting
= −1(8 − 38) = −1(−30)
the third row and the third column of A.
  = 30
1 0 −2
M33 =  4 1 0   
1 0 0 −2
−2 −3 0  4 1 0 0 

23. A =  
We will expand M33 down the third column. 5 6 7 8 
   

1+3  4 1  
2+3  1 0  −2 −3 −1 0
M33 = −2(−1)  +0(−1)  +
−2 −3 −2 −3 |A| = 0 · A13 + 0 · A23 + 7A33 + (−1)A43
 
1 0 = 7A33 − A43
0(−1)3+3    
4 1  1
 0 −2 
= −2(1)[4(−3) − (−2)(1)] + 0 + 0 = 7(−1)3+3  4 1 0 −
 −2 −3 0 
= −2(−10) + 0 + 0 = 20  
  1 0 −2 
0 −2 
1 0 (−1)4+3  4 1 0 
 4 0  5
21. A = 
1 0  6 8 
 5 6 7 8 
−2 −3 −1 0 We will expand each determinant down the third column.
We have:
A24 = (−1)2+4 M24 = 1 · M24 = M24    

1+3  4 1 
  7 − 2(−1)
 1 0 0   −2 3  +0 + 0 +

=  5 6 7   







 −2 −3 −1  − 2(−1)1+3  4 1  3+3  1 0 
 5 6  + 0 + 8(−1)  4 1 
We will expand across the first row.
  = 7[−2(4(−3) − (−2) · 1)] + [−2(4 · 6 − 5 · 1)+
 1 0 0 

 5 6 7  8(1 · 1 − 4 · 0)]

 −2 −3 −1  = 7[−2(−10)] + [−2(19) + 8(1)]
    = 7(20) + (−30) = 140 − 30
6 7  
1+2  5 7 
= 1(−1)1+1  + 0(−1) +
−3 −1 −2 −1 = 110
 
 5 6  25. We will expand across the first row. We could have chosen
0(−1)1+3 
−2 −3  any other row or column just as well.
 
 
= 1 · 1[6(−1) − (−3)(7)] + 0 + 0  3 1 2
 
 
= 1(15) = 15  −2 3 1
 
 
A43 = (−1)4+3 M43 = −1 · M43  3 4 −6 
     
 1 0 −2  3 1   
   + 1 · (−1)1+2  −2 1  +
= −1 ·  4 1 0  = 3(−1)1+1   
4 −6 3 −6 
5 6 8 
 
We will expand across the first row.  −2 3 
2(−1)1+3  
3 4

= 3·1[3(−6)−4 · 1]+1(−1)[−2(−6) − 3 · 1]+


2 · 1(−2 · 4−3 · 3)
= 3(−22) − (9) + 2(−17)
= −109

Copyright © 2013 Pearson Education, Inc.


248 Chapter 6: Systems of Equations and Matrices

 
 −2 9 
27. We will expand down the second column. We could have Dx =   = −2(−3) − 3 · 9 = 6 − 27 = −21
chosen any other row or column just as well. 3 −3 
 
   
 x 0 −1   2 −2 
  Dy =   = 2 · 3 − 4(−2) = 6 + 8 = 14
  4 3 
 2 x x2 
 
 
 −3 x 1 Dx −21 1
    x= =
−42
=
 2 x2    D 2
= 0(−1)1+2   + x(−1)2+2  x −1  +
−3 1   −3 1 
y=
Dy
=
14
=−
1
D −42 3
   
 x −1  1 1
x(−1)3+2  
2  The solution is ,− .
2 x 2 3

= 0(−1)[2·1−(−3)x2 ]+ x · 1[x · 1−(−3)(−1)]+ 35. 2x + 5y = 7,


x(−1)[x · x2 − 2(−1)] 3x − 2y = 1
 
2 5 
= 0 + x(x − 3) − x(x3 + 2) D =   = 2(−2) − 3 · 5 = −4 − 15 = −19
3 −2 
= x2 − 3x − x4 − 2x = −x4 + x2 − 5x  
7 5 
Dx =    = 7(−2) − 1 · 5 = −14 − 5 = −19
29. −2x + 4y = 3, 1 −2 
3x − 7y = 1  
  2 7
 −2 4  
Dy =   = 2 · 1 − 3 · 7 = 2 − 21 = −19
D =   = −2(−7) − 3 · 4 = 14 − 12 = 2 3 1
3 −7 
  Dx −19
3 4  x= = =1
Dx =   = 3(−7) − 1 · 4 = −21 − 4 = −25 D −19
1 −7  Dy −19
  y= = =1
 −2 3  −19
Dy =   = −2 · 1 − 3 · 3 = −2 − 9 = −11 D
3 1 The solution is (1, 1).

Dx −25 25 37. 3x + 2y − z = 4,
x= = =−
D 2 2 3x − 2y + z = 5,
Dy −11 11 4x − 5y − z = −1
y= = =−  
D 2

2
  3 2 −1 
 
25 11
The solution is − , − . D =  3 −2 1  = 42
2 2  4 −5 −1 
31. 2x − y = 5,  
 4 2 −1 

x − 2y = 1 Dx =  5 −2 1  = 63
   −1 −5 −1 
 2 −1 
D =   = 2(−2) − 1(−1) = −4 + 1 = −3
1 −2  
 3 4 −1 

 
  Dy =  3 5 1  = 39
5 −1 
Dx =  = 5(−2) − 1(−1) = −10 + 1 = −9  4 −1 −1 
1 −2 
 
  3 2 4 
2 5  
Dy =  = 2 · 1 − 1 · 5 = 2 − 5 = −3 Dz =  3 −2 5  = 99
1 1  4 −5 −1 
Dx −9
x= = =3 Dx 63 3
D −3 x= = =
Dy −3 D 42 2
y= = =1
D −3 Dy 39 13
y= = =
The solution is (3, 1). D 42 14
33. 2x + 9y = −2, Dz 99 33
z= = =
4x − 3y = 3 D 42 14 
  3 13 33
2 9  The solution is , , .
D =   = 2(−3) − 4 · 9 = −6 − 36 = −42 2 14 14
4 −3 
(Note that we could have used Cramer’s rule to find only
two of the values and then used substitution to find the
remaining value.)

Copyright © 2013 Pearson Education, Inc.


Exercise Set 6.6 249

39. 3x + 5y − z = −2, (Note that we could have used Cramer’s rule to find only
x − 4y + 2z = 13, two of the values and then used substitution to find the
remaining value.)
2x + 4y + 3z = 1
  6y + 6z = −1,
 3 5 −1  43.
 
D =  1 −4 2  = −67 8x + 6z = −1,
2 4 3  4x + 9y =8
   
 −2 5 −1  0 6 6
   
Dx =  13 −4 2  = −201 D =  8 0 6  = 576
 1 4 3  4 9 0
   
 3 −2 −1   −1 6 6 
   
Dy =  1 13 2  = 134 Dx =  −1 0 6  = 288
2 1 3   8 9 0
   
 3 5 −2   0 −1 6 
   
Dz =  1 −4 13  = −67 Dy =  8 −1 6  = 384
2 4 1  4 8 0
 
Dx −201  0 6 −1 
 
Dz =  8 0 −1  = −480
x= = =3
D −67
4 9 8 
Dy 134
y= = = −2
D −67 Dx 288 1
x= = =
D 576 2
Dz −67
z= = =1
D −67 Dy 384 2
y= = =
The solution is (3, −2, 1). D 576 3
(Note that we could have used Cramer’s rule to find only Dz −480 5
two of the values and then used substitution to find the z= = =−
D 576 6
remaining value.)  
1 2 5
The solution is , ,− .
41. x − 3y − 7z = 6, 2 3 6
2x + 3y + z = 9, (Note that we could have used Cramer’s rule to find only
two of the values and then used substitution to find the
4x + y =7 remaining value.)
 
 1 −3 −7 
 
D =  2 3 1  = 57 45. The graph of f (x) = 3x + 2 is shown below. Since it passes
4 1 0  the horizontal-line test, the function is one-to-one.
 
 6 −3 −7  y
 
Dx =  9 3 1  = 57 5

7 1 0 
4
3
f (x ) = 3 x + 2
 
 1 6 −7  2
  1
Dy =  2 9 1  = 171 –5 –4 –3 –2 –1 1 2 3 4 5 x
4 7 0  –1
–2
 
 1 −3 6  –3
 
Dz =  2 3 9  = −114
–4
–5
4 1 7

Dx 57 We find a formula for f −1 (x).


x= = =1
D 57 Replace f (x) with y: y = 3x + 2
Dy 171 Interchange x and y: x = 3y + 2
y= = =3
D 57 x−2
Solve for y: y =
3
Dz −114
z= = = −2 x−2
D 57 Replace y with f −1 (x): f −1 (x) =
3
The solution is (1, 3, −2).

Copyright © 2013 Pearson Education, Inc.


250 Chapter 6: Systems of Equations and Matrices

47. The graph of f (x) = |x| + 3 is shown below. It fails the 9. Graph: y > 2x
horizontal-line test, so it is not one-to-one. 1. We first graph the related equation y = 2x. We
y
draw the line dashed since the inequality symbol is
>.
5
4 2. To determine which half-plane to shade, test a point
3 f (x ) = | x | + 3 not on the line. We try (1, 1) and substitute:
2
1 y > 2x
–5 –4 –3 –2 –1
–1
1 2 3 4 5 x
1 ? 2 · 1

–2 1  2 FALSE
–3
–4 Since 1 > 2 is false, (1, 1) is not a solution, nor are
–5
any points in the half-plane containing (1, 1). The
points in the opposite half-plane are solutions, so we
49. (3 − 4i) − (−2 − i) = 3 − 4i + 2 + i = shade that half-plane and obtain the graph.
(3 + 2) + (−4 + 1)i = 5 − 3i y
51. (1 − 2i)(6 + 2i) = 6 + 2i − 12i − 4i = 2
4
6 + 2i − 12i + 4 = 10 − 10i 2
 
y 2 4 2 2 4
53.   x
3 y = y 2
y  2x
y2 − 6 = y 4

y2 − y − 6 = 0
(y − 3)(y + 2) = 0 11. Graph: y + x ≥ 0
y − 3 = 0 or y + 2 = 0 1. First graph the related equation y + x = 0. Draw
the line solid since the inequality is ≥.
y = 3 or y = −2
The solutions are 3 and −2. 2. Next determine which half-plane to shade by testing
  a point not on the line. Here we use (2, 2) as a check.
 x 1 
55.  2 y+x ≥ 0
1 2 −1  = −6

3 4 −2  2 + 2 ? 0

−x − 2 = −6 Evaluating the 4  0 TRUE
determinant Since 4 ≥ 0 is true, (2, 2) is a solution. Thus shade
−x = −4 the half-plane containing (2, 2).
x=4
y
The solution is 4.
4
57. Answers may vary.
  2
 a b
 
 −b a  4 2 2 4 x
2
59. Answers may vary. yx0
  4
 2πr 2πr 
 
 −h r 
13. Graph: y > x − 3
1. We first graph the related equation y = x − 3. Draw
Exercise Set 6.7 the line dashed since the inequality symbol is >.
2. To determine which half-plane to shade, test a point
1. Graph (f ) is the graph of y > x.
not on the line. We try (0, 0).
3. Graph (h) is the graph of y ≤ x − 3. y > x−3
5. Graph (g) is the graph of 2x + y < 4. 0 ? 0 − 3

7. Graph (b) is the graph of 2x − 5y > 10. 0  −3 TRUE
Since 0 > −3 is true, (0, 0) is a solution. Thus we
shade the half-plane containing (0, 0).

Copyright © 2013 Pearson Education, Inc.


Exercise Set 6.7 251

y 2. To determine which half-plane to shade, test a point


4
not on the line. We try (0, 0).
2 3y + 2x ≥ 6

4 2 4 x
3 · 0 + 2 · 0 ? 6

2 0  6 FALSE
yx3
Since 0 ≥ 6 is false, (0, 0) is not a solution. We
shade the half-plane which does not contain (0, 0).
15. Graph: x + y < 4
y
1. First graph the related equation x + y = 4. Draw
the line dashed since the inequality is <. 4

2. To determine which half-plane to shade, test a point


not on the line. We try (0, 0). 4 2 2 x
x+y < 4 2

4 3y  2x  6
0 + 0 ? 4

0  4 TRUE
21. Graph: 3x − 2 ≤ 5x + y
Since 0 < 4 is true, (0, 0) is a solution. Thus shade
the half-plane containing (0, 0). −2 ≤ 2x + y Adding −3x
1. First graph the related equation 2x + y = −2. Draw
y the line solid since the inequality is ≤.
xy4 2. To determine which half-plane to shade, test a point
2
not on the line. We try (0, 0).
2x + y ≥ −2
4 2 2 4 x
2 2(0) + 0 ? −2

4 0  −2 TRUE
Since 0 ≥ −2 is true, (0, 0) is a solution. Thus shade
17. Graph: 3x − 2y ≤ 6 the half-plane containing the origin.
1. First graph the related equation 3x − 2y = 6. Draw y
the line solid since the inequality is ≤.
4
2. To determine which half-plane to shade, test a point
not on the line. We try (0, 0). 2

3x − 2y ≤ 6 4 2 2 4 x
2
3(0) − 2(0) ? 6
 4
0  6 TRUE
Since 0 ≤ 6 is true, (0, 0) is a solution. Thus shade 3x  2  5x  y
the half-plane containing (0, 0).
23. Graph: x < −4
y
1. We first graph the related equation x = −4. Draw
4 the line dashed since the inequality is <.
2 2. To determine which half-plane to shade, test a point
not on the line. We try (0, 0).
4 2 4 x
2 x < −4
3x  2y  6 0 ? −4 FALSE
Since 0 < −4 is false, (0, 0) is not a solution. Thus,
19. Graph: 3y + 2x ≥ 6 we shade the half-plane which does not contain the
origin.
1. First graph the related equation 3y + 2x = 6. Draw
the line solid since the inequality is ≥.

Copyright © 2013 Pearson Education, Inc.


252 Chapter 6: Systems of Equations and Matrices

y y

4 4
2 2 4  y  1

2 2 4 x 4 2 2 4 x
2 2
x  4
4

25. Graph: y ≥ 5 29. Graph: y ≥ |x|


1. First we graph the related equation y = 5. Draw 1. Graph the related equation y = |x|. Draw the line
the line solid since the inequality is ≥. solid since the inequality symbol is ≥.
2. To determine which half-plane to shade we test a 2. To determine the region to shade, observe that the
point not on the line. We try (0, 0). solution set consists of all ordered pairs (x, y) where
y≥5 the second coordinate is greater than or equal to the
absolute value of the first coordinate. We see that
0 ? 5 FALSE the solutions are the points on or above the graph
Since 0 ≥ 5 is false, (0, 0) is not a solution. We of y = |x|.
shade the half-plane that does not contain (0, 0).
y
y 4
8 y5
2
4
4 2 2 4 x
8 4 4 8 2 y  兩x兩
x
4 4
8

31. Graph (f ) is the correct graph.


27. Graph: −4 < y < −1
33. Graph (a) is the correct graph.
This is a conjunction of two inequalities
35. Graph (b) is the correct graph.
−4 < y and y < −1.
We can graph −4 < y and y < −1 separately and then 37. First we find the related equations. One line goes through
graph the intersection, or region in both solution sets. (0, 4) and (4, 0). We find its slope:
0−4 −4
m= = = −1
y
6 4−0 4
5 This line has slope −1 and y-intercept (0, 4), so its equation
4
is y = −x + 4.
3
2 The other line goes through (0, 0) and (1, 3). We find the
1 slope.
–6 –5 –4 –3 –2 –1
–1
1 2 3 4 5 6 x 3−0
m= =3
–2 1−0
–3
This line has slope 3 and y-intercept (0, 0), so its equation
–4
–5 —4 < y is y = 3x + 0, or y = 3x.
–6
Observing the shading on the graph and the fact that the
y
lines are solid, we can write the system of inqualities as
6
5
y ≤ −x + 4,
4 y ≤ 3x.
3
2
Answers may vary.
y < —1
1
39. The equation of the vertical line is x = 2 and the equation
6 x
of the horizontal line is y = −1. The lines are dashed and
–6 –5 –4 –3 –2 –1 1 2 3 4 5
–1
–2 the shaded area is to the left of the vertical line and above
–3
the horizontal line, so the system of inequalities can be
–4
–5
written
–6 x < 2,
y > −1.

Copyright © 2013 Pearson Education, Inc.


Exercise Set 6.7 253

41. First we find the related equations. One line goes through 47. Graph: y ≥ −3,
(0, 3) and (3, 0). We find its slope: x≥1
0−3 −3
m= = = −1 We graph the related equations y = −3 and x = 1 using
3−0 3 solid lines and determine the solution set for each inequal-
This line has slope −1 and y-intercept (0, 3), so its equation ity. Then we shade the region common to both solution
is y = −x + 3. sets.
The other line goes through (0, 1) and (1, 2). We find its
y
slope:
2−1 1 4
m= = =1
1−0 1 2
This line has slope 1 and y-intercept (0, 1), so its equation
⫺4 ⫺2 2 4 x
is y = x + 1. ⫺2
Observe that both lines are solid and that the shading lies ⫺4 (1, ⫺3)
below both lines, to the right of the y-axis, and above the
x-axis. We can write this system of inequalities as
We find the vertex (1, −3) by solving the system
y ≤ −x + 3,
y = −3,
y ≤ x + 1,
x = 1.
x ≥ 0,
y ≥ 0. 49. Graph: x ≤ 3,
y ≥ 2 − 3x
43. Graph: y ≤ x,
We graph the related equations x = 3 and y = 2 − 3x
y ≥ 3−x using solid lines and determine the half-plane containing
We graph the related equations y = x and y = 3 − x the solution set for each inequality. Then we shade the
using solid lines and determine the solution set for each region common to both solution sets.
inequality. Then we shade the region common to both
solution sets. y

y 4

2
4
⫺4 ⫺2 2 4 x
2
冣w, w冣 ⫺2
4 2 2 4 x ⫺4
2
⫺6
4 (3, ⫺7)
⫺8
 
3 3
We find the vertex , by solving the system We find the vertex (3, −7) by solving the system
2 2
x = 3,
y = x,
y = 2 − 3x.
y = 3 − x.
51. Graph: x + y ≤ 1,
45. Graph: y ≥ x,
x−y ≤ 2
y ≤ 4−x
We graph the related equations x + y = 1 and x − y = 2
We graph the related equations y = x and y = 4 − x using solid lines and determine the half-plane containing
using solid lines and determine the solution set for each the solution set for each inequality. Then we shade the
inequality. Then we shade the region common to both region common to both solution sets.
solution sets.
y y

4
4
2
2 (2, 2)
4 2 4 x
4 2 2 4 x 2
2
4
4 冣w, q冣
 
3 1
We find the vertex ,− by solving the system
2 2

Copyright © 2013 Pearson Education, Inc.


254 Chapter 6: Systems of Equations and Matrices

x + y = 1, y
x − y = 2. 8

53. Graph: 2y − x ≤ 2, (0, 0)


4

y + 3x ≥ −1 8 4 4 8 x
(0, 3)
We graph the related equations 2y−x = 2 and y+3x = −1 (12, 0)
using solid lines and determine the half-plane containing
the solution set for each inequality. Then we shade the
region common to both solution sets. (4, 6)

We find the vertex (−12, 0) by solving the system


y
4y + 3x = −36,
4
y = 0.
2
冣¢, ∞冣 We find the vertex (0, 0) by solving the system
4 2 2 4 x y = 0,
2
x = 0.
4
We find the vertex (0, −3) by solving the system
 
4 5 4y − 3x = −12,
We find the vertex − , by solving the system
7 7 x = 0.
2y − x = 2, We find the vertex (−4, −6) by solving the system
y + 3x = −1. 4y − 3x = −12,
55. Graph: x − y ≤ 2, 4y + 3x = −36.
x + 2y ≥ 8, 59. Graph: 3x + 4y ≥ 12,
y−4 ≤ 0 5x + 6y ≤ 30,
We graph the related equations x − y = 2, x + 2y = 8, 1≤x≤3
and y = 4 using solid lines and determine the half-plane Shade the intersection of the graphs of the given inequali-
containing the solution set for each inequality. Then we ties.
shade the region common to all three solution sets.
y
y 冢1, 256 冣
(0, 4) 4 冣3, e冣
冣1, #冣
2 (4, 2) (6, 4) 2 冣3, !冣
2 2 4 6 x
2 2 4 6 x 2

4
4
 
25
We find the vertex (0, 4) by solving the system We find the vertex 1, by solving the system
6
x + 2y = 8, 5x + 6y = 30,
y = 4. x = 1.
We find the vertex (6, 4) by solving the system  
5
x − y = 2, We find the vertex 3, by solving the system
2
y = 4. 5x + 6y = 30,
We find the vertex (4, 2) by solving the system x = 3.
x − y = 2,  
3
We find the vertex 3, by solving the system
x + 2y = 8. 4
57. Graph: 4x − 3y ≥ −12, 3x + 4y = 12,
4x + 3y ≥ −36, x = 3.
 
y ≤ 0, 9
We find the vertex 1, by solving the system
x≤0 4
Shade the intersection of the graphs of the four inequali- 3x + 4y = 12,
ties. x = 1.

Copyright © 2013 Pearson Education, Inc.


Exercise Set 6.7 255

61. Find the maximum and minimum values of y

P = 17x − 3y + 60, subject to


6x + 8y ≤ 48,
0 ≤ y ≤ 4,
0 ≤ x ≤ 7. B

Graph the system of inequalities and determine the ver- C


tices.
y A D x

Vertex A: (0, 0)
B C
Vertex B:
We solve the system 3x + 5y = 30 and x = 0. The
coordinates of point B are (0, 6).
D Vertex C:
A E x We solve the system 5x + 3y = 34 and 3x + 5y = 30.
The coordinates of point C are (5, 3).
Vertex A: (0, 0)
Vertex D:
Vertex B: We solve the system 5x + 3y  = 34and y = 0. The
We solve the system x = 0 and y = 4. The coordi- 34
coordinates of point D are ,0 .
nates of point B are (0, 4). 5
Vertex C: Evaluate the objective function F at each vertex.
We solve the system 6x + 8y  = 48 and y = 4. The Vertex F = 5x + 36y
8 A(0, 0) 5 · 0 + 36 · 0+ = 0
coordinates of point C are ,4 .
3
B(0, 6) 5 · 0 + 36 · 6 = 216
Vertex D:
C(5, 3) 5 · 5 + 39 · 3 = 133
We solve the system 6x + 8y  = 48 and x = 7. The  
3 34 34
coordinates of point D are 7,
4
. D ,0 5· + 36 · 0 = 34
5 5
Vertex E: The maximum value of F is 216 when x = 0 and y = 6.
We solve the system x = 7 and y = 0. The coordi-
nates of point E are (7, 0). The minimum value of F is 0 when x = 0 and y = 0.

Evaluate the objective function P at each vertex. 65. Let x = the number of jumbo biscuits and y = the number
of regular biscuits to be made per day. The income I is
Vertex P = 17x − 3y + 60
given by
A(0, 0) 17 · 0 − 3 · 0 + 60 = 60
I = 0.80x + 0.50y
B(0, 4) 17 · 0 − 3 · 4 + 60 = 48
  subject to the constraints
8 8 2
C ,4 17 · − 3 · 4 + 60 = 66 x + y ≤ 200,
3  3 3
3 3 3 2x + y ≤ 300,
D 7, 17 · 7 − 3 · + 60 = 176
4 4 4 x ≥ 0,
E(7, 0) 17 · 7 − 3 · 0 + 60 = 179 y ≥ 0.
The maximum value of P is 179 when x = 7 and y = 0. We graph the system of inequalities, determine the ver-
The minimum value of P is 48 when x = 0 and y = 4. tices, and find the value if I at each vertex.
63. Find the maximum and minimum values of y

F = 5x + 36y, subject to 300


(0, 200)
5x + 3y ≤ 34,
3x + 5y ≤ 30, 200

x ≥ 0,
(100, 100)
y ≥ 0. 100
(150, 0)
Graph the system of inequalities and find the vertices.
(0, 0) 100 200 300 x

Copyright © 2013 Pearson Education, Inc.


256 Chapter 6: Systems of Equations and Matrices

Vertex I = 0.10x + 0.08y y


12
(0, 0) 0.80(0) + 0.50(0) = 0
10 A
(0, 200) 0.80(0) + 0.50(200) = 100
8
(100, 100) 0.80(100) + 0.50(100) = 130
6
(150, 0) 0.80(150) + 0.50(0) = 120 B
4
The company will have a maximum income of $130 when C
2
100 of each type of biscuit are made. D
1 2 3 x
67. Let x = the number of units of lumber and y = the number
of units of plywood produced per week. The profit P is
Vertex C = 15x + 5y
given by
A(0, 10) 15 · 0 + 5 · 10 = 50
P = 20x + 30y  
6 6
subject to the constraints B ,4 + 5 · 4 = 38
15 ·
5 5
 
x + y ≤ 400, 24 24 24 24 12
C , 15 · +5· = 36
x ≥ 100, 13 13 13 13 13
y ≥ 150. D(3, 0) 15 · 3 + 5 · 0 = 45
We graph the system of inequalities, determine the vertices 12 24
The minimum cost of $36 is achieved by using , or
and find the value of P at each vertex. 13 13
11 24 11
1 sacks of soybean meal and , or 1 sacks of oats.
y 13 13 13
400 71. Let x = the amount invested in corporate bonds and y =
the amount invested in municipal bonds. The income I is
(100, 300)
300 given by
I = 0.08x + 0.075y
200
(250, 150)
subject to the constraints
(100, 150)
100 x + y ≤ 40, 000,
6000 ≤ x ≤ 22, 000,
400 x
100 200 300
0 ≤ y ≤ 30, 000.
We graph the system of inequalities, determine the ver-
Vertex P = 20x + 30y tices, and find the value of I at each vertex.
(100, 150) 20 · 100 + 30 · 150 = 6500
y
(100, 300) 20 · 100 + 30 · 300 = 11, 000 (6000, 30,000)
40,000
(250, 150) 20 · 250 + 30 · 150 = 9500 (10,000, 30,000)
30,000
The maximum profit of $11,000 is achieved by producing
100 units of lumber and 300 units of plywood.
20,000 (22,000, 18,000)
69. Let x = the number of sacks of soybean meal to be used
and y = the number of sacks of oats. The minimum cost 10,000
is given by (22,000, 0)

C = 15x + 5y 10,000 20,000 30,000 40,000 x


(6000, 0)
subject to the constraints
50x + 15y ≥ 120, Vertex I = 0.08x + 0.075y
(6000, 0) 480
8x + 5y ≥ 24,
(6000, 30, 000) 2730
5x + y ≥ 10,
x ≥ 0, (10, 000, 30, 000) 3050

y ≥ 0. (22, 000, 18, 000) 3110


Graph the system of inequalities, determine the vertices, (22, 000, 0) 1760
and find the value of C at each vertex. The maximum income of $3110 occurs when $22,000 is
invested in corporate bonds and $18,000 is invested in mu-
nicipal bonds.
73. Let x = the number of P1 airplanes and y = the num-
ber of P2 airplanes to be used. The operating cost C, in
thousands of dollars, is given by

Copyright © 2013 Pearson Education, Inc.


Exercise Set 6.7 257

C = 12x + 10y The maximum profit per day is $192 when 2 knit suits and
subject to the constraints 4 worsted suits are made.
40x + 80y ≥ 2000, 77. Let x = the number of pounds of meat and y = the number
40x + 30y ≥ 1500, of pounds of cheese in the diet in a week. The cost is given
by
120x + 40y ≥ 2400,
C = 3.50x + 4.60y
x ≥ 0,
subject to
y ≥ 0.
2x + 3y ≥ 12,
Graph the system of inequalities, determine the vertices,
and find the value of C at each vertex. 2x + y ≥ 6,

y
x ≥ 0,
60 (0, 60)
y ≥ 0.
50 Graph the system of inequalities, determine the vertices,
(6, 42) and find the value of C at each vertex.
40

30 y
20
(30, 10) 6 (0, 6)
10
(50, 0) 5
10 20 30 40 50 x 4
(1.5, 3)
3
Vertex C = 12x + 10y 2
(0, 60) 12 · 0 + 10 · 60 = 600 1 (6, 0)
(6, 42) 12 · 6 + 10 · 42 = 492 1 2 3 4 5 6 x
(30, 10) 12 · 30 + 10 · 10 = 460
Vertex C = 3.50x + 4.60y
(50, 0) 12 · 50 + 10 · 0 = 600
(0, 6) 3.50(0) + 4.60(6) = 27.60
The minimum cost of $460 thousand is achieved using 30
(1.5, 3) 3.50(1.5) + 4.60(3) = 19.05
P1 ’s and 10 P2 ’s.
(6, 0) 3.50(6) + 4.60(0) = 21.00
75. Let x = the number of knit suits and y = the number of
The minimum weekly cost of $19.05 is achieved when 1.5 lb
worsted suits made. The profit is given by
of meat and 3 lb of cheese are used.
P = 34x + 31y
79. Let x = the number of animal A and y = the number of
subject to
animal B. The total number of animals is given by
2x + 4y ≤ 20,
T =x+y
4x + 2y ≤ 16,
subject to
x ≥ 0,
x + 0.2y ≤ 600,
y ≥ 0.
0.5x + y ≤ 525,
Graph the system of inequalities, determine the vertices,
and find the value of P at each vertex. x ≥ 0,
y ≥ 0.
y
Graph the system of inequalities, determine the vertices,
and find the value of T at each vertex.
y
(0, 5)
(2, 4)
3000

2000

(0, 0) (4, 0) x 1000


(0, 525)
(550, 250)
Vertex P = 34x + 31y 250 500 750 1000 x
(0, 0)
(0, 0) 34 · 0 + 31 · 0 = 0 (600, 0)
(0, 5) 34 · 0 + 31 · 5 = 155
(2, 4) 34 · 2 + 31 · 4 = 192
(4, 0) 34 · 4 + 31 · 0 = 136

Copyright © 2013 Pearson Education, Inc.


258 Chapter 6: Systems of Equations and Matrices

Vertex T =x+y 89. y


(0, 0) 0+0=0
4
(0, 525) 0 + 525 = 525 2
(550, 250) 550 + 250 = 800
4 2 2 4 x
(600, 0) 600 + 0 = 600 2
The maximum total number of 800 is achieved when there 4
are 550 of A and 250 of B.

81. −5 ≤ x + 2 < 4 兩x兩  兩y兩

−7 ≤ x < 2 Subtracting 2
91. Let x = the number of chairs and y = the number of sofas
The solution set is {x| − 7 ≤ x < 2}, or [−7, 2). produced. Find the maximum value of
83. x2 − 2x ≤ 3 Polynomial inequality I = 120x + 575y
x2 − 2x − 3 ≤ 0 subject to
x − 2x − 3 = 0
2
Related equation 20x + 100y ≤ 1900,
(x + 1)(x − 3) = 0 Factoring x + 50y ≤ 500,
Using the principle of zero products or by observing the 2x + 20y ≤ 240,
graph of y = x2 − 2x − 3, we see that the solutions of x ≥ 0,
the related equation are −1 and 3. These numbers divide y ≥ 0.
the x-axis into the intervals (−∞, −1), (−1, 3), and (3, ∞).
We let f (x) = x2 − 2x − 3 and test a value in each interval. Graph the system of inequalities, determine the vertices,
and find the value of I at each vertex.
(−∞, −1): f (−2) = 5 > 0
(−1, 3): f (0) = −3 < 0 y
(3, ∞): f (4) = 5 > 0
Function values are negative on (−1, 3). This can also be 20
determined from the graph of y = x2 − 2x − 3. Since the
inequality symbol is ≤, the endpoints of the interval must (0, 10)
15
be included in the solution set. It is {x| − 1 ≤ x ≤ 3} or
(25, 9.5)
[−1, 3].
10
(70, 5)
85. Graph: y ≥ x − 2,2
5
y ≤ 2 − x2 (95, 0)
First graph the related equations y = x −2 and y = 2−x
2 2

using solid lines. The solution set consists of the region (0, 0) 100 200 300 400 500 x
above the graph of y = x2 − 2 and below the graph of
y = 2 − x2 . Vertex I = 120x + 575y
(0, 0) 120 · 0 + 575 · 0 = 0
y (0, 10) 120 · 0 + 575 · 10 = 5750
4 (25, 9.5) 120 · 25 + 575(9.5) = 8462.50
(70, 5) 120 · 70 + 575 · 5 = 11, 275
4 2 2 4 x (95, 0) 120 · 95 + 575 · 0 = 11, 400
The maximum income of $11,400 is achieved by making 95
4 chairs and 0 sofas.

87. y Exercise Set 6.8


4 x+7 A B
兩x  y兩  1 1. = +
2 (x − 3)(x + 2) x−3 x+2
4 2 2 4 x x+7 A(x + 2) + B(x − 3)
= Adding
2 (x − 3)(x + 2) (x − 3)(x + 2)
4 Equate the numerators:
x + 7 = A(x + 2) + B(x − 3)
Let x + 2 = 0, or x = −2. Then we get

Copyright © 2013 Pearson Education, Inc.


Exercise Set 6.8 259

−2 + 7 = 0 + B(−2 − 3) Let x + 2 = 0 or x = −2. Then we get


5 = −5B 3(−2)2 −11(−2)−26 = A(−2−2)(−2+1)+0+0
−1 = B 12 + 22 − 26 = A(−4)(−1)
Next let x − 3 = 0, or x = 3. Then we get 8 = 4A
3 + 7 = A(3 + 2) + 0 2=A
10 = 5A Next let x − 2 = 0, or x = 2. Then, we get
2=A 3 · 22 − 11 · 2 − 26 = 0 + B(2 + 2)(2 + 1) + 0
The decomposition is as follows: 12 − 22 − 26 = B · 4 · 3
2 1 −36 = 12B

x−3 x+2
−3 = B
7x − 1 Finally let x + 1 = 0, or x = −1. We get
3.
6x2 − 5x + 1
3(−1)2 −11(−1)−26 = 0 + 0 + C(−1 + 2)(−1 − 2)
7x − 1
= Factoring the denominator 3 + 11 − 26 = C(1)(−3)
(3x − 1)(2x − 1)
A B −12 = −3C
= +
3x − 1 2x − 1 4=C
A(2x − 1) + B(3x − 1) The decomposition is as follows:
= Adding
(3x − 1)(2x − 1) 2

3
+
4
Equate the numerators: x+2 x−2 x+1
7x − 1 = A(2x − 1) + B(3x − 1) 7.
9
1 (x + 2)2 (x − 1)
Let 2x − 1 = 0, or x = . Then we get A B C
2
    = + +
1 1 x + 2 (x + 2)2 x−1
7 −1 = 0+B 3· −1
2 2 A(x + 2)(x − 1) + B(x − 1) + C(x + 2)2
=
5 1 (x + 2)2 (x − 1)
= B Adding
2 2
5=B Equate the numerators:
1 9 = A(x + 2)(x − 1) + B(x − 1) + C(x + 2)2 (1)
Next let 3x − 1 = 0, or x = . We get
   
3 Let x − 1 = 0, or x = 1. Then, we get
1 1
7 −1 = A 2· −1 +0 9 = 0 + 0 + C(1 + 2)2
3 3
  9 = 9C
7 2
−1 = A −1 1=C
3 3
4 1 Next let x + 2 = 0, or x = −2. Then, we get
=− A 9 = 0 + B(−2 − 1) + 0
3 3
−4 = A 9 = −3B
The decomposition is as follows: −3 = B
4 5 To find A we first simplify equation (1).
− +
3x − 1 2x − 1 9 = A(x2 + x − 2) + B(x − 1) + C(x2 + 4x + 4)
5. 3x − 11x − 26
2
= Ax2 + Ax − 2A + Bx − B + Cx2 + 4Cx + 4C
(x2 − 4)(x + 1) = (A + C)x2 +(A + B + 4C)x+(−2A − B + 4C)
3x2 − 11x − 26 Then we equate the coefficients of x2 .
= Factoring the
(x + 2)(x − 2)(x + 1)
denominator 0 = A+C
A B C
= + + 0 = A+1 Substituting 1 for C
x+2 x−2 x+1
−1 = A
A(x−2)(x+1)+B(x+2)(x+1)+C(x+2)(x−2)
= The decomposition is as follows:
(x+2)(x−2)(x+1)
Adding 1 3 1
− − +
Equate the numerators: x + 2 (x + 2)2 x−1

3x2 −11x−26 = A(x − 2)(x + 1)+


B(x + 2)(x + 1)+C(x + 2)(x − 2)

Copyright © 2013 Pearson Education, Inc.


260 Chapter 6: Systems of Equations and Matrices

9. 2x2 + 3x + 1 −7x + 4
(x2 − 1)(2x − 1) (x + 1)2 (x − 3)
2x2 + 3x + 1 A B C
= Factoring the = + +
(x + 1)(x − 1)(2x − 1) x + 1 (x + 1)2 x−3
denominator
A B C A(x + 1)(x − 3) + B(x − 3) + C(x + 1)2
= + + =
x + 1 x − 1 2x − 1 (x + 1)2 (x − 3)
Adding
A(x−1)(2x−1)+B(x+1)(2x−1)+C(x+1)(x−1)
= Equate the numerators:
(x + 1)(x − 1)(2x − 1)
Adding −7x + 4 = A(x + 1)(x − 3) + B(x − 3)+
Equate the numerators: C(x + 1)2 (1)
2x + 3x + 1 = A(x − 1)(2x − 1)+
2
Let x − 3 = 0, or x = 3. Then, we get
B(x + 1)(2x − 1)+C(x + 1)(x − 1) −7 · 3 + 4 = 0 + 0 + C(3 + 1)2
Let x + 1 = 0, or x = −1. Then, we get −17 = 16C
2(−1)2 + 3(−1) + 1 = A(−1 − 1)[2(−1) − 1]+0+0 17
− =C
2 − 3 + 1 = A(−2)(−3) 16
0 = 6A Let x + 1 = 0, or x = −1. Then, we get
0=A −7(−1) + 4 = 0 + B(−1 − 3) + 0
Next let x − 1 = 0, or x = 1. Then, we get 11 = −4B
2 · 1 + 3 · 1 + 1 = 0 + B(1 + 1)(2 · 1 − 1) + 0
2 11
− =B
4
2+3+1 = B·2·1
To find A we first simplify equation (1).
6 = 2B
−7x + 4
3=B
= A(x2 − 2x − 3) + B(x − 3) + C(x2 + 2x + 1)
1
Finally we let 2x − 1 = 0, or x = . We get = Ax2 − 2Ax − 3A + Bx − 3B + Cx2 − 2Cx + C
2
 2      = (A+C)x2 + (−2A+B −2C)x + (−3A−3B +C)
1 1 1 1
2 +3 +1 = 0+0+C +1 −1 Then equate the coefficients of x2 .
2 2 2 2
 
1 3 3 1 0=A+C
+ +1 = C · · − 17 17
2 2 2 2 Substituting − for C, we get A = .
3 16 16
3=− C The decomposition is as follows:
4
−4 = C 17/16 11/4 17/16
− −
x+1 (x + 1)2 x−3
The decomposition is as follows:
The original expression is equivalent to the following:
3 4
− 17/16 11/4 17/16
x − 1 2x − 1 x−2+ − −
x+1 (x + 1)2 x−3
11. x4 − 3x3 − 3x2 + 10
(x + 1)2 (x − 3) 13. −x2 + 2x − 13
(x2 + 2)(x − 1)
x4 − 3x3 − 3x2 + 10
= Multiplying the Ax + B C
x3 − x2 − 5x − 3 denominator = 2 +
x +2 x−1
Since the degree of the numerator is greater than the de- (Ax + B)(x − 1) + C(x2 + 2)
gree of the denominator, we divide. = Adding
(x2 + 2)(x − 1)
x− 2
Equate the numerators:
x3 − x2 − 5x − 3 x4 −3x3 −3x2 + 0x+10
x4 − x3 −5x2 − 3x −x2 + 2x − 13 = (Ax + B)(x − 1) + C(x2 + 2) (1)
−2x3 +2x2 + 3x+10 Let x − 1 = 0, or x = 1. Then we get
−2x3 +2x2 +10x+ 6
−12 + 2 · 1 − 13 = 0 + C(12 + 2)
− 7x+ 4
−1 + 2 − 13 = C(1 + 2)
The original expression is thus equivalent to the following:
−7x + 4 −12 = 3C
x−2+ 3 −4 = C
x − x2 − 5x − 3
We proceed to decompose the fraction. To find A and B we first simplify equation (1).

Copyright © 2013 Pearson Education, Inc.


Exercise Set 6.8 261

−x2 + 2x − 13 6x3 + 5x2 + 6x − 2


17.
= Ax − Ax + Bx − B + Cx + 2C
2 2 2x2 + x − 1
Since the degree of the numerator is greater than the de-
= (A + C)x2 + (−A + B)x + (−B + 2C)
gree of the denominator, we divide.
Equate the coefficients of x2 :
3x+ 1
−1 = A + C 2x2 + x − 1 6x3 +5x2 +6x−2
Substituting −4 for C, we get A = 3. 6x3 +3x2 −3x
2x2 +9x−2
Equate the constant terms:
2x2 + x−1
−13 = −B + 2C 8x−1
Substituting −4 for C, we get B = 5. The original expression is equivalent to
The decomposition is as follows: 8x − 1
3x + 1 + 2
3x + 5 4 2x + x − 1

x2 + 2 x − 1 We proceed to decompose the fraction.
8x − 1 8x − 1
6 + 26x − x2 = Factoring the
15. 2x2 + x − 1 (2x − 1)(x + 1)
(2x − 1)(x + 2)2 denominator
A B C A B
= +
= + +
2x − 1 x + 2 (x + 2)2 2x − 1 x + 1
A(x + 1) + B(2x − 1)
A(x + 2)2 + B(2x − 1)(x + 2) + C(2x − 1) = Adding
= (2x − 1)(x + 1)
(2x − 1)(x + 2)2
Adding Equate the numerators:
Equate the numerators: 8x − 1 = A(x + 1) + B(2x − 1)
6 + 26x − x = A(x + 2) + B(2x − 1)(x + 2)+
2 2 Let x + 1 = 0, or x = −1. Then we get
C(2x − 1) (1) 8(−1) − 1 = 0 + B[2(−1) − 1]
1 −8 − 1 = B(−2 − 1)
Let 2x − 1 = 0, or x = . Then, we get
2 −9 = −3B
 2  2
1 1 1
6 + 26 · − =A +2 +0+0 3=B
2 2 2
 2 1
1 5 Next let 2x − 1 = 0, or x = . We get
6 + 13 − = A 2
   
4 2 1 1
8 −1 = A +1 +0
75 25 2 2
4
=
4
A  
3
3=A 4−1 = A
2
Let x + 2 = 0, or x = −2. We get 3
3= A
6 + 26(−2) − (−2)2 = 0 + 0 + C[2(−2) − 1] 2
6 − 52 − 4 = −5C 2=A
−50 = −5C The decomposition is
2 3
10 = C + .
2x − 1 x + 1
To find B we first simplify equation (1).
The original expression is equivalent to
6 + 26x − x2 2 3
3x + 1 + + .
= A(x2 + 4x + 4) + B(2x2 + 3x − 2) + C(2x − 1) 2x − 1 x + 1
= Ax2 +4Ax+4A+2Bx2 +3Bx−2B +2Cx−C 2x2 − 11x + 5
19.
= (A+2B)x + (4A+3B +2C)x + (4A−2B −C)
2
(x − 3)(x2 + 2x − 5)
Equate the coefficients of x2 : A Bx + C
= +
−1 = A + 2B x − 3 x2 + 2x − 5
Substituting 3 for A, we obtain B = −2. A(x2 +2x−5)+(Bx+C)(x−3)
= Adding
(x − 3)(x2 + 2x − 5)
The decomposition is as follows:
Equate the numerators:
3 2 10
− + 2x2 − 11x + 5 = A(x2 + 2x − 5)+
2x − 1 x + 2 (x + 2)2
(Bx + C)(x − 3) (1)
Let x − 3 = 0, or x = 3. Then, we get

Copyright © 2013 Pearson Education, Inc.


262 Chapter 6: Systems of Equations and Matrices

2 · 32 − 11 · 3 + 5 = A(32 + 2 · 3 − 5) + 0 36 = 3A + 4B Coefficients of x-terms


18 − 33 + 5 = A(9 + 6 − 5) 1 = 2A − 5B Constant terms
−10 = 10A We solve this system of equations and find
−1 = A A = 8 and B = 3.
To find B and C, we first simplify equation (1). The decomposition is
2x2 − 11x + 5 = Ax2 + 2Ax − 5A + Bx2 − 3Bx+ 8
+
3
.
Cx − 3C 4x − 5 3x + 2
= (A + B)x2 + (2A − 3B + C)x+ −4x2 − 9x + 8
25.
(−5A − 3C) (3x2 + 1)(x − 2)
The decomposition looks like
Equate the coefficients of x2 :
Ax + B C
2=A+B + .
3x2 + 1 x−2
Substituting −1 for A, we get B = 3. Add and equate the numerators.
Equate the constant terms: −4x2 − 9x + 8
5 = −5A − 3C = (Ax + B)(x − 2) + C(3x2 + 1)
Substituting −1 for A, we get C = 0. = Ax2 −2Ax+Bx−2B +3Cx2 +C
The decomposition is as follows: or

1
+
3x −4x2 − 9x + 8
x − 3 x2 + 2x − 5 = (A + 3C)x2 + (−2A + B)x + (−2B + C)
−4x2 − 2x + 10 Then equate corresponding coefficients.
21. −4 = A + 3C Coefficients of x2 -terms
(3x + 5)(x + 1)2
The decomposition looks like −9 = −2A + B Coefficients of x-terms
A B C 8 = −2B + C Constant terms
+ + .
3x + 5 x + 1 (x + 1)2 We solve this system of equations and find
Add and equate the numerators. A = 2, B = −5, C = −2.
−4x2 − 2x + 10 The decomposition is
= A(x + 1)2 + B(3x + 5)(x + 1) + C(3x + 5) 2x − 5 2
− .
= A(x2 + 2x + 1) + B(3x2 + 8x + 5) + C(3x + 5) 3x2 + 1 x − 2
or 27. x3 + x2 + 9x + 9 = 0
−4x2 − 2x + 10 2
x (x + 1) + 9(x + 1) = 0
= (A + 3B)x2 +(2A + 8B + 3C)x+(A + 5B + 5C)
(x + 1)(x2 + 9) = 0
Then equate corresponding coefficients.
x+1 = 0 or x2 + 9 = 0
−4 = A + 3B Coefficients of x2 -terms
x = −1 or x2 = −9
−2 = 2A + 8B + 3C Coefficients of x-terms
x = −1 or x = ±3i
10 = A + 5B + 5C Constant terms
The solutions are −1, 3i, and −3i.
We solve this system of three equations and find A = 5,
B = −3, C = 4. 29. f (x) = x3 + x2 − 3x − 2
The decomposition is We use synthetic division to factor the polynomial. Using
5 3 4 the possibilities found by the rational zeros theorem we
− + . find that x + 2 is a factor:
3x + 5 x + 1 (x + 1)2 
−2  1 1 −3 −2
36x + 1 36x + 1
23. = −2 2 2
12x2 − 7x − 10 (4x − 5)(3x + 2) 1 −1 −1 0
The decomposition looks like
We have x3 + x2 − 3x − 2 = (x + 2)(x2 − x − 1).
A B
+
4x − 5 3x + 2
. x3 + x2 − 3x − 2 = 0
Add and equate the numerators. (x + 2)(x2 − x − 1) = 0
36x + 1 = A(3x + 2) + B(4x − 5) x + 2 = 0 or x2 − x − 1 = 0
or 36x + 1 = (3A + 4B)x + (2A − 5B) The solution of the first equation is −2. We use the
Then equate corresponding coefficients. quadratic formula to solve the second equation.

Copyright © 2013 Pearson Education, Inc.


Exercise Set 6.8 263


−b ± b2 − 4ac Equate the coefficients of x3 :
x=
2a 0=A+C +D

−(−1) ± (−1)2 − 4 · 1 · (−1) 1
= Substituting 2 for C and for D, we get
2·1 4a

1± 5 A = − 2.
1
= 2a
2
√ √ Equate the constant terms:
1+ 5 1− 5
The solutions are −2, and . 0 = −Ba2 − Ca3 + Da3
2 2
1
31. f (x) = x3 + 5x2 + 5x − 3 Substitute 2 for C and for D. Then solve for B.
 4a
−3  1 5 5 −3 1 1
−3 −6 3 0 = −Ba2 − 2 · a3 + 2 · a3
4a 4a
1 2 −1 0
0 = −Ba2
x3 + 5x2 + 5x − 3 = 0 0=B
(x + 3)(x2 + 2x − 1) = 0 The decomposition is as follows:
x + 3 = 0 or x2 + 2x − 1 = 0 1 1 1
x
The solution of the first equation is −3. We use the 2a
− 2
2
+ 4a 2
+ 4a 2

quadratic formula to solve the second equation. x + a2 x+a x−a



−b ± b2 − 4ac 1 + ln x2 1 + 2 ln x
x= 35. =
2a (ln x + 2)(ln x − 3)2 (ln x + 2)(ln x − 3)2
 √
−2 ± 22 − 4 · 1 · (−1) −2 ± 8 Let u = ln x. Then we have:
= = 1 + 2u
2·1 2
√ √ (u + 2)(u − 3)2
−2 ± 2 2 2(−1 ± 2)
= = A B C
2√ 2 = + +
= −1 ± 2 u + 2 u − 3 (u − 3)2
√ √ A(u − 3)2 + B(u + 2)(u − 3) + C(u + 2)
The solutions are −3, −1 + 2, and −1 − 2. =
(u + 2)(u − 3)2
x Equate the numerators:
33.
x4 − a4
x 1 + 2u = A(u − 3)2 + B(u + 2)(u − 3) + C(u + 2)
= 2 Factoring the Let u − 3 = 0, or u = 3.
(x + a2 )(x + a)(x − a)
denominator
Ax + B C D 1 + 2 · 3 = 0 + 0 + C(5)
= + +
x2 + a2 x+a x−a 7 = 5C
=[(Ax + B)(x + a)(x − a) + C(x2 + a2 )(x − a)+ 7
=C
5
D(x2 + a2 )(x + a)]/[(x2 + a2 )(x + a)(x − a)]
Let u + 2 = 0, or u = −2.
Equate the numerators:
1 + 2(−2) = A(−2 − 3)2 + 0 + 0
x = (Ax + B)(x + a)(x − a) + C(x2 + a2 )(x − a)+
−3 = 25A
D(x2 + a2 )(x + a) 3
Let x − a = 0, or x = a. Then, we get − =A
25
a = 0 + 0 + D(a2 + a2 )(a + a) To find B, we equate the coefficients of u2 :
a = D(2a2 )(2a) 0=A+B
a = 4a3 D 3 3
Substituting − for A and solving for B, we get B = .
1 25 25
=D 1 + 2u
4a2 The decomposition of is as follows:
Let x + a = 0, or x = −a. We get (u + 2)(u − 3)2
−a = 0 + C[(−a)2 + a2 ](−a − a) + 0 3 3 7
− + +
25(u + 2) 25(u − 3) 5(u − 3)2
−a = C(2a2 )(−2a)
Substituting ln x for u we get
−a = −4a3 C
3 3 7
1 − + + .
=C 25(ln x + 2) 25(ln x − 3) 5(ln x − 3)2
4a2

Copyright © 2013 Pearson Education, Inc.


264 Chapter 6: Systems of Equations and Matrices

Multiply equation (4) by −7 and add it to equation (6).


Chapter 6 Review Exercises x + 5y + 3z = 4 (1)
− 17y + 13z = −9 (4)

1. The statement is true. See page 485 in the text. − 6z = 12 (7)


Now we solve equation (7) for z.
3. The statement is true. See page 521 in the text.
−6z = 12
5. (a) z = −2
7. (h) Back-substitute −2 for z in equation (4) and solve for y.
−17y + 13(−2) = −9
9. (b)
−17y − 26 = −9
11. (c)
−17y = 17
13. 5x − 3y = −4, (1) y = −1
3x − y = −4 (2) Finally, we back-substitute −1 for y and −2 for z in equa-
Multiply equation (2) by −3 and add. tion (1) and solve for x.
5x − 3y = −4 x + 5(−1) − 3(−2) = 4
−9x + 3y = 12 x−5+6 = 4
−4x =8 x+1 = 4
x = −2 x=3
Back-substitute to find y. The solution is (3, −1, −2).
3(−2) − y = −4 Using equation (2) 19. x − y = 5, (1)
−6 − y = −4 y−z = 6, (2)
−y = 2 z − w = 7, (3)
y = −2
x +w=8 (4)
The solution is (−2, −2).
Multiply equation (1) by −1 and add it to equation (4).
15. x + 5y = 12, (1) x−y =5 (1)
5x + 25y = 12 (2) y−z =6 (2)
Solve equation (1) for x. z−w=7 (3)
x = −5y + 12 y +w=3 (5)
Substitute in equation (2) and solve for y. Multiply equation (2) by −1 and add it to equation (5).
5(−5y + 12) + 25y = 12 x−y =5 (1)
−25y + 60 + 25y = 12 y−z =6 (2)
60 = 12 z−w=7 (3)
We get a false equation, so there is no solution. z + w = −3 (6)
17. x + 5y − 3z = 4, (1) Multiply equation (3) by −1 and add it to equation (6).
3x − 2y + 4z = 3, (2) x−y =5 (1)
2x + 3y − z = 5 (3) y−z =6 (2)
Multiply equation (1) by −3 and add it to equation (2). z− w=7 (3)
Multiply equation (1) by −2 and add it to equation (3). 2w = −10 (7)
x + 5y + 3z = 4 (1) Solve equation (7) for w.
− 17y + 13z = −9 (4) 2w = −10
− 7y + 5z = −3 (5) w = −5
Multiply equation (5) by 17. Back-substitute −5 for w in equation (3) and solve for z.
x+ 5y + 3z = 4 (1) z−w = 7
− 17y + 13z = −9 (4) z − (−5) = 7
− 119y + 85z = −51 (6) z+5 = 7
z=2

Copyright © 2013 Pearson Education, Inc.


Chapter 6 Review Exercises 265

Back-substitute 2 for z in equation (2) and solve for y. 4 2


x+ y+ z = 1 (1)
3 3
y−z = 6
49 3
y−2 = 6 y+ z =
26 13
(2)
y=8 z = −2
Back-substitute 8 for y in equation (1) and solve for x. Back-substitute in equation (2) and solve for y.
x−y = 5 49 3
y + (−2) =
x−8 = 5 26 13
49 3
x = 13 y− =
13 13
Writing the solution as (w, x, y, z), we have (−5, 13, 8, 2).
52
y= =4
21. Systems 13, 14, 15, 17, 18, and 19 each have either no 13
solution or exactly one solution, so the equations in those Back-substitute in equation (1) and solve for x.
systems are independent. System 16 has infinitely many 4 2
solutions, so the equations in that system are dependent. x + (4) + (−2) = 1
3 3
23. 3x + 4y + 2z = 3 16 4
x+ − =1
5x − 2y − 13z = 3 3 3
4x + 3y − 3z = 6 x = −3
Write the augmented matrix. We will use Gaussian elimi- The solution is (−3, 4, −2).
nation.
  25. w + x + y + z = −2,
3 4 2 3 −3w − 2x + 3y + 2z = 10,
 
 5 −2 −13 3  2w + 3x + 2y − z = −12,
4 3 −3 6 2w + 4x − y + z = 1
Multiply row 2 and row 3 by 3. Write the augmented matrix. We will use Gauss-Jordan
  elimination.
3 4 2 3  
  1 1 1 1 −2
 15 −6 −39 9 
 −3 −2
 3 2 10 
12 9 −9 18  
 2 3 2 −1 −12 
Multiply row 1 by −5 and add it to row 2. 2 4 −1 1 1
Multiply row 1 by −4 and add it to row 3.
  Multiply row 1 by 3 and add it to row 2.
3 4 2 3 Multiply row 1 by −2 and add it to row 3.
 
 0 −26 −49 −6  Multiply row 1 by −2 and add it to row 4.
0 −7 −17 6
 
1 1 1 1 −2
Multiply row 3 by 26.  0 1 4 
 6 5
   
3 4 2 3  0 1 0 −3 −8 
 
 0 −26 −49 −6  0 2 −3 −1 5
0 −182 −442 156
Multiply row 2 by −1 and add it to row 1.
Multiply row 2 by −7 and add it to row 3. Multiply row 2 by −1 and add it to row 3.
  Multiply row 2 by −2 and add it to row 4.
3 4 2 3
   
 0 −26 −49 −6  1 0 −5 −4 −6
0 0 −99 198  0 1 4 
 6 5 
 
1 1 1  0 0 −6 −8 −12 
Multiply row 1 by , row 2 by − , and row 3 by − .
3 26 99 0 0 −15 −11 −3
 
4 2 Multiply row 1 by 3.
1 1
 3 3 
  1
 49 3  Multiply row 3 by − .
 0 1  2
 26 13 
0 0 1 −2

Copyright © 2013 Pearson Education, Inc.


266 Chapter 6: Systems of Equations and Matrices

 
3 0 −15 −12 −18 Carry out. We begin by multiplying equation (1) by −30
 0 1 4  and adding.
 6 5 
  −30x − 30y = −150, 000
 0 0 3 4 6 
0 0 −15 −11 −3 30x + 35y = 167, 000
5y = 17, 000
Multiply row 3 by 5 and add it to row 1.
y = 3400
Multiply row 3 by −2 and add it to row 2.
Back-substitute to find x.
Multiply row 3 by 5 and add it to row 4.
x + 3400 = 5000 Using equation (1)
 
3 0 0 8 12 x = 1600
 0 1 0 −3 −8 
  Check. The total investment is $1600 + $3400, or $5000.
 
 0 0 3 4 6  The total interest is 0.03($1600) + 0.035($3400), or $48 +
0 0 0 9 27 $119, or $167. The solution checks.
State. $1600 was invested at 3% and $3400 was invested
1 at 3.5%.
Multiply row 4 by .
9
  29. Familiarize. Let x, y, and z represent the scores on the
3 0 0 8 12
first, second, and third tests, respectively.
 0 1 0 −3 −8 
  Translate.
 
 0 0 3 4 6 
The total score on the three tests is 226.
0 0 0 1 3
x + y + z = 226
Multiply row 4 by −8 and add it to row 1. The sum of the scores on the first and second tests exceeds
Multiply row 4 by 3 and add it to row 2. the score on the third test by 62.
Multiply row 4 by −4 and add it to row 3. x + y = z + 62
  The first score exceeds the second by 6.
3 0 0 0 −12
 0 1 1  x=y+6
 0 0 
 
 0 0 3 0 −6  We have a system of equations.
0 0 0 1 3 x + y + z = 226,
x + y = z + 62,
1
Multiply rows 1 and 3 by . x=y+6
3
  or x + y + z = 226,
1 0 0 0 −4
 0 1 0 0 x + y − z = 62,
 1 

  x−y = 6
 0 0 1 0 −2 
Carry out. Solving the system of equations, we get
0 0 0 1 3
(75, 69, 82).
The solution is (−4, 1, −2, 3). Check. The sum of the scores is 75 + 69 + 82, or 226. The
27. Familiarize. Let x = the amount invested at 3% and y = sum of the scores on the first two tests is 75 + 69, or 144.
the amount invested at 3.5%. Then the interest from the This exceeds the score on the third test, 82, by 62. The
investments is 3%x and 3.5%y, or 0.03x and 0.035y. score on the first test, 75, exceeds the score on the second
test, 69, by 6. The solution checks.
Translate.
State. The scores on the first, second, and third tests were
The total investment is $5000. 75, 69, and 82, respectively.
x + y = 5000
   
The total interest is $167. 1 −1 0 −1 0 6
0.03x + 0.035y = 167 31. A + B =  2 3 −2  +  1 −2 0 
−2 0 1 0 1 −3
We have a system of equations.  
x+ y = 5000, 1 + (−1) −1 + 0 0+6
=  2+1 3 + (−2) −2 + 0 
0.03x + 0.035y = 167
−2 + 0 0+1 1 + (−3)
Multiplying the second equation by 1000 to clear the dec-  
imals, we have: 0 −1 6
= 3 1 −2 
x + y = 5000, (1)
−2 1 −2
30x + 35y = 167, 000. (2)

Copyright © 2013 Pearson Education, Inc.


Chapter 6 Review Exercises 267
   
1 −1 0 −1 1 0
33. −A = −1  2 3 −2  =  −2 −3 2  43. 3x − 2y + 4z = 13,
−2 0 1 2 0 −1 x + 5y − 3z = 7,
2x − 3y + 7z = −8
35. B and C do not have the same order, so it is not possible Write the coefficients on the left in a matrix. Then write
to find B + C. the product of that matrix and the column matrix contain-
ing the variables, and set the result equal to the column
    matrix containing the constants on the right.
−1 0 6 1 −1 0     
37. BA =  1 −2 0 · 2 3 −2  3 −2 4 x 13
0 1 −3 −2 0 1  1 5 −3   y  =  7 
  2 −3 7 z −8
−1 + 0 − 12 1 + 0 + 0 0+0+6
= 1 − 4 + 0 −1 − 6 + 0 0 + 4 + 0  45. 5x − y + 2z = 17,
0+2+6 0+3+0 0−2−3 3x + 2y − 3z = −16,
 
−13 1 6 4x − 3y − z = 5
=  −3 −7 4  Write an equivalent matrix equation, AX = B.
8 3 −5     
5 −1 2 x 17
   3
0.98 0.23 0.30 0.28 0.45 2 −3   y  =  −16 
  4 −3 −1 z 5
39. a) M =  1.03 0.19 0.27 0.34 0.41 
 1.01 0.21 0.35 0.31 0.39  Then,
0.99 0.25 0.29 0.33 0.42  
11 7 1
 80
b) N = 32 19 43 38  80 80 

   
  17 1
9 13 21  
c) NM = 131.98 29.50 40.80 41.29 54.92 X = A−1 B =   80 − −16  =  −2 
 80 80 
 5 5
d) The entries of NM represent the total cost, in dol-  17 11 13 
lars, for each item for the day’s meal. − −
80 80 80
  The solution is (1, −2, 5).
0 0 3  
41. A =  0 −2 0   
47.  1 −2  = 1 · 4 − 3(−2) = 4 + 6 = 10
4 0 0 3 4 
Write the augmented matrix.
  49. We will expand across the first row.
0 0 3 1 0 0  
 0 −2 0 0 1 0   2 −1 1 
 
 1 2 −1 
4 0 0 0 0 1  
 3 4 −3 
Interchange rows 1 and 3.    
     
4 0 0 0 0 1 1+1  2 −1  1+2  1 −1 
= 2(−1)  4 −3  + (−1)(−1)  3 −3  +
 0 −2 0 0 1 0 
0 0 3 1 0 0  
1 2
1(−1)1+3  
1 1 1
Multiply row 1 by , row 2 by − , and row 3 by . 3 4
 4  2 3
1 = 2 · 1[2(−3) − 4(−1)] + (−1)(−1)[1(−3) − 3(−1)]+
 1 0 0 0 0
 4 

  1 · 1[1(4) − 3(2)]
 0 1 0 0 − 1
 0 
 = 2(−2) + 1(0) + 1(−2)
 2 
 1  = −6
0 0 1 0 0
3
 
1 51. 5x − 2y = 19,
 0 0
 4 
 7x + 3y = 15
 1   
A−1 =  0 − 0 

 5 −2 
  D =   = 5(3) − 7(−2) = 29
7 3 
2
 1 
0 0  
3  19 −2 

Dx =   = 19(3) − 15(−2) = 87
15 3 
 
 5 19 

Dy =   = 5(15) − 7(19) = −58
7 15 

Copyright © 2013 Pearson Education, Inc.


268 Chapter 6: Systems of Equations and Matrices

Dx 87 We find the vertex (0, 9) by solving the system


x= = =3
D 29
2x + y = 9,
Dy −58
y= = = −2 x = 0.
D 29
The solution is (3, −2). We find the vertex (2, 5) by solving the system

53. 3x − 2y + z = 5, 2x + y = 9,

4x − 5y − z = −1, 4x + 3y = 23.

3x + 2y − z = 4 We find the vertex (5, 1) by solving the system


 
 3 −2 1  4x + 3y = 23,
 
D =  4 −5 −1  = 42 x + 3y = 8.
 3 2 −1  We find the vertex (8, 0) by solving the system
 
 5 −2 1  x + 3y = 8,
 
Dx =  −1 −5 −1  = 63 y = 0.
 4 2 −1 
  59. Let x = the number of questions answered from group A
3 5 1 
 and y = the number of questions answered from group B.
Dy =  4 −1 −1  = 39
 3 4 −1  Find the maximum value of S = 7x + 12y subject to
  x + y ≥ 8,
 3 −2 5 
  8x + 10y ≤ 80,
Dz =  4 −5 −1  = 99
3 2 4  x ≥ 0,
Dx 63 3 y≥0
x= = =
D 42 2 Graph the system of inequalities, determine the vertices,
Dy 39 13 and find the value of T at each vertex.
y= = =
D 42 14 y
Dz 99 33 11
z= = =
D 42 14
 
10

3 13 33 9
The solution is , , . 8 (0, 8)
2 14 14 7
6
55.
y 5
4
y  3x  6 3
4 2
1 (8, 0)
2 (10, 0)
1 2 3 4 5 6 7 8 9 10 11 x
4 2 4 x
2
Vertex S = 7x + 12y
(0, 8) 7 · 0 + 12 · 8 = 96
57. Graph: 2x + y ≥ 9, (8, 0) 7 · 8 + 12 · 0 = 56
4x + 3y ≥ 23, (10, 0) 7 · 10 + 12 · 0 = 70
x + 3y ≥ 8, The maximum score of 96 occurs when 0 questions from
x ≥ 0, group A and 8 questions from group B are answered cor-
rectly.
y≥0
Shade the intersection of the graphs of the given inequali- −8x + 23 −8x + 23
61. =
ties. 2x2 + 5x − 12 (2x − 3)(x + 4)
A B
y = +
2x − 3 x + 4
A(x + 4) + B(2x − 3)
=
(2x − 3)(x + 4)
(0, 9)
Equate the numerators.
(2, 5)
4
(5, 1) −8x + 23 = A(x + 4) + B(2x − 3)
4 4 x
4 (8, 0)

Copyright © 2013 Pearson Education, Inc.


Chapter 6 Review Exercises 269

   
Let x =
3
: −8
3
+ 23 = A
3
+4 +0 Multiply equation (6) by −37 and add it to equation (8).
2 2 2 3 4 1
11 − + = −2 (1)
−12 + 23 = A x y z
2 23 11
11 − = 13 (6)
11 = A y z
2 384
2=A = 1152 (9)
z
Let x = −4 : −8(−4) + 23 = 0 + B[2(−4) − 3] Complete the solution.
32 + 23 = −11B 384
= 1152
z
55 = −11B
1
−5 = B 3
=z
2 5
The decomposition is − . 23 11
2x − 3 x + 4 − = 13
y 1/3
63. Interchanging columns of a matrix is not a row-equivalent 23
operation, so answer A is correct. (See page 511 in the − 33 = 13
y
text.) 23
= 46
65. Let x, y, and z represent the amounts invested at 4%, 5%, y
1 1
and 5 %, respectively. =y
2 2
Solve: x + y + z = 40, 000, 3 4 1
− + = −2
0.04x + 0.05y + 0.055z = 1990, x 1/2 1/3
0.055z = 0.04x + 590 3
− 8 + 3 = −2
x
x = $10, 000, y = $12, 000, z = $18, 000
3
− 5 = −2
3 4 1 x
67. − + = −2, (1)
x y z 3
=3
5 1 2 x
+ − = 1, (2) 1=x
x y z
 
7 3 2 1 1
+ + = 19 (3) The solution is 1, , .
x y z 2 3
Multiply equations (2) and (3) by 3. (We could also have solved this system of equations by first
3 4 1 1 1 1
− + = −2 (1) substituting a for , b for , and c for and proceeding
x y z x y z
as we did in Exercise 66 above.)
15 3 6
+ − =3 (4) 69.
x y z y
21 9 6
+ + = 57 (5)
x y z
Multiply equation (1) by −5 and add it to equation (2).
Multiply equation (1) by −7 and add it to equation (3). x
3 4 1
− + = −2 (1)
x y z
23 11
− = 13 (6) 兩xy兩  1
y z
37 1
− = 71 (7) 71. In general, (AB)2 = A2 B2 . (AB)2 = ABAB and
y z A2 B2 = AABB. Since matrix multiplication is not com-
Multiply equation (7) by 23. mutative, BA = AB, so (AB)2 = A2 B2 .
3 4 1
− + = −2 (1) 73. If a1 x + b1 y = c1 and a2 x + b2 y = c2 are parallel lines,
x y z
then a1 =  ka2 , b1 = kb2 , and c1 = kc2 , for
 some number
23 11  a1 b 1   c1 b1  a c 
− = 13 (6) k. Then    
= 0,   = 0, and  1 1  = 0.
y z a2 b2  c2 b2  a2 c2
851 23
− = 1633 (8) 75. The denominator of the second fraction, x2 − 5x + 6, can
y z
be factored into linear factors with real coefficients:

Copyright © 2013 Pearson Education, Inc.


270 Chapter 6: Systems of Equations and Matrices

(x − 3)(x − 2). Thus, the given expression is not a partial 4. 2x − 3y = 8, (1)


fraction decomposition. 5x − 2y = 9 (2)
Multiply equation (1) by 5 and equation (2) by −2 and
Chapter 6 Test add.
10x − 15y = 40
−10x + 4y = −18
1. 3x + 2y = 1, (1)
− 11y = 22
2x − y = −11 (2)
y = −2
Multiply equation (2) by 2 and add. Back-substitute to find x.
3x + 2y = 1 2x − 3(−2) = 8
4x − 2y = −22 2x + 6 = 8
7x = −21 2x = 2
x = −3 x=1
Back-substitute to find y. The solution is (1, −2). Since the system of equations has
2(−3) − y = −11 Using equation (2). exactly one solution, it is consistent and the equations are
independent.
−6 − y = −11
−y = −5 5. 4x + 2y + z = 4, (1)
y=5 3x − y + 5z = 4, (2)
The solution is (−3, 5). Since the system of equations has 5x + 3y − 3z = −2 (3)
exactly one solution, it is consistent and the equations are
Multiply equations (2) and (3) by 4.
independent.
4x + 2y + z=4 (1)
2. 2x − y = 3, (1) 12x − 4y + 20z = 16 (4)
2y = 4x − 6 (2) 20x + 12y − 12z = −8 (5)
Solve equation (1) for y. Multiply equation (1) by −3 and add it to equation (4).
y = 2x − 3 Multiply equation (1) by −5 and add it to equation (5).
Subsitute in equation (2) and solve for x. 4x + 2y + z=4 (1)
2(2x − 3) = 4x − 6 −10y + 17z = 4 (6)
4x − 6 = 4x − 6 2y − 17z = −28 (7)
0=0 Interchange equations (6) and (7).
The equation 0 = 0 is true for all values of x and y. 4x + 2y + z=4 (1)
Thus the system of equations has infinitely many solutions.
Solving either equation for y, we get y = 2x − 3, so the 2y − 17z = −28 (7)
solutions are ordered pairs of the form (x, 2x − 3). Equiva- −10y + 17z = 4 (6)
y+3
lently, if we solve either equation for x, we get x = , Multiply equation (7) by 5 and add it to equation (6).
  2
y+3 4x + 2y + z=4 (1)
so the solutions can also be expressed as , y . Since
2 2y − 17z = −28 (7)
there are infinitely many solutions, the system of equations
−68z = −136 (8)
is consistent and the equations are dependent.
Solve equation (8) for z.
3. x − y = 4, (1)
−68z = −136
3y = 3x − 8 (2)
z=2
Solve equation (1) for x.
Back-substitute 2 for z in equation (7) and solve for y.
x=y+4
2y − 17 · 2 = −28
Subsitute in equation (2) and solve for y.
2y − 34 = −28
3y = 3(y + 4) − 8
2y = 6
3y = 3y + 12 − 8
y=3
0=4
Back-substitute 3 for y and 2 for z in equation (1) and
We get a false equation so there is no solution. Since there solve for x.
is no solution the system of equations is inconsistent and
the equations are independent.

Copyright © 2013 Pearson Education, Inc.


Chapter 6 Test 271

4x + 2 · 3 + 2 = 4 can process 120 + 104, or 224, orders per day. Together,


4x + 8 = 4 Latonna and Sam can process 120+128, or 248, orders per
day. The solution checks.
4x = −4
State. Latonna can process 120 orders per day, Cole can
x = −1
process 104 orders per day, and Sam can process 128 orders
The solution is (−1, 3, 2). per day.
6. Familiarize. Let x and y represent the number of stu-  
−5 1 3 −4
dent and nonstudent tickets sold, respectively. Then the 8. B + C = +
−2 4 −1 0
receipts from the student tickets were 3x and the receipts
from the nonstudent tickets were 5y. 
−5 + 3 1 + (−4)
=
Translate. One equation comes from the fact that −2 + (−1) 4 + 0
750 tickets were sold. 
x + y = 750 −2 −3
=
−3 4
A second equation comes from the fact that the total re-
ceipts were $3066. 9. A and C do not have the same order, so it is not possible
3x + 5y = 3066 to find A − C.
 
Carry out. We solve the system of equations. 3 −4 −5 1
10. CB =
x + y = 750, (1) −1 0 −2 4
3x + 5y = 3066 (2) 
3(−5) + (−4)(−2) 3(1) + (−4)(4)
=
Multiply equation (1) by −3 and add. −1(−5) + 0(−2) −1(1) + 0(4)
−3x − 3y = −2250 
−7 −13
3x + 5y = 3066 =
5 −1
2y = 816
y = 408 11. The product AB is not defined because the number of
columns of A, 3, is not equal to the number of rows of B,
Substitute 408 for y in equation (1) and solve for x. 2.
x + 408 = 750  
1 −1 3 2 −2 6
x = 342 12. 2A = 2 =
−2 5 2 −4 10 4
Check. The number of tickets sold was 342 + 408, or 750. 
The total receipts were $3·342+$5·408 = $1026+$2040 = 3 −4
13. C =
$3066. The solution checks. −1 0
State. 342 student tickets and 408 nonstudent tickets were Write the augmented matrix.

sold. 3 −4 1 0
−1 0 0 1
7. Familiarize. Let x, y, and z represent the number of
Interchange rows.
orders that can be processed per day by Latonna, Cole, 
and Sam, respectively. −1 0 0 1
3 −4 1 0
Translate.
Multiply row 1 by 3 and add it to row 2.
Latonna, Cole, and Sam can process 352 orders per day. 
−1 0 0 1
x + y + z = 352 0 −4 1 3
Latonna and Cole together can process 224 orders per day. 1
Multiply row 1 by −1 and row 2 by − .
x + y = 224 4
 
Latonna and Sam together can process 248 orders per day. 1 0 0 −1
 1 3 
x + z = 248 0 1 − −
4 4
We have a system of equations:
 
x + y + z = 352, 0 −1
 
x+y = 224, C−1 =  1 3 
− −
x + z = 248. 4 4
Carry out. Solving the system of equations, we get  
(120, 104, 128). 0.95 0.40 0.39
14. a) M =  1.10 0.35 0.41 
Check. Latonna, Cole, and Sam can process 120 + 104 + 1.05 0.39 0.36
128, or 352, orders per day. Together, Latonna and Cole

Copyright © 2013 Pearson Education, Inc.


272 Chapter 6: Systems of Equations and Matrices


b) N = 26 18 23 20.
y
c) NM = 68.65 25.67 25.80
4
d) The entries of NM represent the total cost, in dol- 3x  4y  12
2
lars, for each type of menu item served on the given
day. 4 2 2 4 x
     2
3 −4 2 x −8
15.  2 3 1  y  =  7  4

1 −5 −3 z 3
16. 3x + 2y + 6z = 2, 21. Find the maximum value and the minimum value of
x + y + 2z = 1, Q = 2x + 3y subject to
2x + 2y + 5z = 3 x + y ≥ 6,
Write an equivalent matrix equation, AX = B. 2x − 3y ≥ −3,
    
3 2 6 x 2 x ≥ 1,
 1 1 2  y  =  1  y ≥ 0.
2 2 5 z 3
Graph the system of inequalities and determine the ver-
Then, tices.
    
1 2 −2 2 −2
X = A−1 B =  −1 3 0  1  =  1  y

0 −2 1 3 1 4

The solution is (−2, 1, 1). C


3
 
 
17.  3 −5  = 3 · 7 − 8(−5) = 21 + 40 = 61
8 7 
2
B

18. We will expand across the first row.


  A D
 2 −1 4  1 2 3 4 5 6 x
 
 −3 1 −2 
  Vertex A:
 5 3 −1 
    We solve the system x = 1 and y = 0. The coordi-
 
1+1  1 −2 

1+2  −3 −2  nates of point A are (1, 0).
= 2(−1)  3 −1  + (−1)(−1)  5 +
−1 
Vertex B:
  We solve the system 2x − 3y
 −3 1   = −3 and x = 1. The
4(−1)1+3   5
5 3 coordinates of point B are 1, .
3
= 2 · 1[1(−1) − 3(−2)] + (−1)(−1)[−3(−1) − 5(−2)]+ Vertex C:
4 · 1[−3(3) − 5(1)] We solve the system x + y = 6 and 2x − 3y = −3.
The coordinates of point C are (3, 3).
= 2(5) + 1(13) + 4(−14)
Vertex D:
= −33
We solve the system x + y = 6 and y = 0. The
19. 5x + 2y = −1, coordinates of point D are (6, 0).
7x + 6y = 1 Evaluate the objective function Q at each vertex.
 
5 2
D=   = 5(6) − 7(2) = 16 Vertex Q = 2x + 3y
7 6 (1, 0) 2·1+3·0=2
 
  5 5
 −1 2  1, 2·1+3· =7

Dx =   = −1(6) − (1)(2) = −8
1 6
3 3
(3, 3) 2 · 3 + 3 · 3 = 15
 
5 −1  (6, 0) 2 · 6 + 3 · 0 = 12
Dy =  = 5(1) − 7(−1) = 12
7 1  The maximum value of Q is 15 when x = 3 and y = 3.
Dx −8 1 The minimum value of Q is 2 when x = 1 and y = 0.
x= = =−
D 16 2
Dy 12 3
y= = =
D 16 4 
1 3
The solution is − , .
2 4

Copyright © 2013 Pearson Education, Inc.


Chapter 6 Test 273

22. Let x = the number of coffee cakes prepared and y = 25. Solve:
the number of cheesecakes. Find the maximum value of A(2) − B(−2) = C(2) − 8
P = 8x + 17y subject to
A(−3) − B(−1) = C(1) − 8
x + y ≤ 100,
A(4) − B(2) = C(9) − 8
x ≥ 25, or
y ≥ 15 2A + 2B − 2C = −8
Graph the system of inequalities, determine the vertices, −3A + B − C = −8
and find the value of P at each vertex. 4A − 2B − 9C = −8
y The solution is (1, −3, 2), so A = 1, B = −3, and C = 2.

100
90
80 (25, 75)
70
60
50
40
30
20 (85, 15)
10 (25, 15)

10 20 30 40 50 60 70 80 90 100 x

Vertex P = 8x + 17y
(25, 15) 8 · 25 + 17 · 15 = 455
(25, 75) 8 · 25 + 17 · 75 = 1475
(85, 15) 8 · 85 + 17 · 15 = 935
The maximum profit of $1475 occurs when 25 coffee cakes
and 75 cheesecakes are prepared.

3x − 11 3x − 11
23. =
x2 + 2x − 3 (x − 1)(x + 3)
A B
= +
x−1 x+3
A(x + 3) + B(x − 1)
=
(x − 1)(x + 3)
Equate the numerators.
3x − 11 = A(x + 3) + B(x − 1)
Let x = −3 : 3(−3) − 11 = 0 + B(−3 − 1)
−20 = −4B
5=B
Let x = 1 : 3(1) − 11 = A(1 + 3) + 0
−8 = 4A
−2 = A
2 5
The decomposition is − + .
x−1 x+3
24. Graph the system of inequalities. We see that D is the
correct graph.

Copyright © 2013 Pearson Education, Inc.


Copyright © 2013 Pearson Education, Inc.
Chapter 7
Conic Sections
y

Exercise Set 7.1 4

2
1. Graph (f) is the graph of x2 = 8y.
⫺4 ⫺2 2 4 x
⫺2
3. Graph (b) is the graph of (y − 2) = −3(x + 4). 2
x 2 ⫺ 4y ⫽ 0
⫺4
5. Graph (d) is the graph of 13x2 − 8y − 9 = 0.
7. x2 = 20y 13. x = 2y 2
x2 = 4 · 5 · y Writing x2 = 4py 1
y2 = x
2
Vertex: (0, 0)
1
Focus: (0, 5) [(0, p)] y2 = 4 · · x Writing y 2 = 4px
8
Directrix: y = −5 (y = −p) Vertex: (0, 0)
 
1
y Focus: ,0
8
4
x 2 ⫽ 20y 1
2 Directrix: x = −
8
⫺4 ⫺2 2 4 x
⫺2 y
⫺4 2 x ⫽ 2y 2
1
9. y = −6x
2
⫺2 ⫺1
  1 2 x
3 ⫺1
y =4 −
2
x 2
Writing y = 4px
2 ⫺2
Vertex: (0, 0)
 
3 15. The vertex is at the origin and the focus is (−3, 0), so the
Focus: − ,0 [(p, 0)]
2 equation is of the form y 2 = 4px where p = −3.
 
3 3 y 2 = 4px
Directrix: x = − − = (x = −p)
2 2 y 2 = 4(−3)x
y 2 = −12x
y

4
17. Since the directrix, x = −4, is a vertical line, the equation
y 2 ⫽ ⫺6x is of the form (y − k)2 = 4p(x − h). The focus, (4, 0),
2
is on the x-axis so the axis of symmetry is the x-axis and
⫺4 ⫺2 2 4 x p = 4. The vertex, (h, k), is the point on the x-axis midway
⫺2 between the directrix and the focus. Thus, it is (0, 0). We
⫺4 have
(y − k)2 = 4p(x − h)
11. x2 − 4y = 0 (y − 0)2 = 4 · 4(x − 0) Substituting
2
x = 4y y 2 = 16x.
x2 = 4 · 1 · y Writing x2 = 4py 19. Since the directrix, y = π, is a horizontal line, the equation
is of the form (x − h)2 = 4p(y − k). The focus, (0, −π), is
Vertex: (0, 0)
on the y-axis so the axis of symmetry is the y-axis and p =
Focus: (0, 1) [(0, p)] −π. The vertex (h, k) is the point on the y-axis midway
Directrix: y = −1 (y = −p) between the directrix and the focus. Thus, it is (0, 0). We
have
(x − h)2 = 4p(y − k)
(x − 0)2 = 4(−π)(y − 0) Substituting
x = −4πy
2

Copyright © 2013 Pearson Education, Inc.


276 Chapter 7: Conic Sections

21. Since the directrix, x = −4, is a vertical line, the equation Vertex: (−1, −3) [(h, k)]
    
is of the form (y − k)2 = 4p(x − h). The focus, (3, 2), is 1 7
on the horizontal line y = 2, so the axis of symmetry is Focus: − 1, −3+ − , or − 1, −
2 2
y = 2. The vertex is the point on the line y = 2 that
[(h, k+p)]
is midway between the directrix and the focus. That is,  
it is the midpoint 1 5
  of the
 segment
 from (−4, 2) to (3, 2): Directrix: y = −3 − − =− (y = k − p)
−4 + 3 2 + 2 1 1 2 2
, , or − , 2 . Then h = − and the
2 2 2 2
directrix is x = h − p, so we have y

x = h−p 2
1
−4 = − − p ⫺4 ⫺2 2 4 x
2 ⫺2
7 ⫺4
− = −p
2
⫺6
7
= p.
2
x 2 ⫹ 2x ⫹ 2y ⫹ 7 ⫽ 0
Now we find the equation of the parabola.
(y − k)2 = 4p(x − h) 27. x2 − y − 2 = 0
   
7 1
(y − 2)2 = 4 x− − x2 = y + 2
2 2
  1
(x − 0)2 = 4 · · [y − (−2)]
1 4
(y − 2)2 = 14 x +
2 [(x − h)2 = 4p(y − k)]
23. (x + 2)2 =−6(y − 1) Vertex: (0, −2) [(h, k)]
     
3 1 7
[x−(−2)]2 =4 − (y−1) [(x−h)2 = 4p(y−k)] Focus: 0, −2 + , or 0, − [(h, k + p)]
2 4 4
Vertex: (−2, 1) [(h, k)] 1 9
     Directrix: y = −2 − = − (y = k − p)
3 1 4 4
Focus: − 2, 1 + − , or − 2, −
2 2
y
[(h, k + p)]
  4
3 5
Directrix: y = 1 − − = (y = k − p) 2
2 2
⫺4 ⫺2 2 4 x
y

4 ⫺4 x2 ⫺ y ⫺ 2 ⫽ 0
2

⫺4 2 4 x 29. y = x2 + 4x + 3
⫺2 y − 3 = x2 + 4x
⫺4
y − 3 + 4 = x2 + 4x + 4
y + 1 = (x + 2)2
(x ⫹ 2)2 ⫽ ⫺6(y ⫺ 1)
1
4 · · [y − (−1)] = [x − (−2)]2
4
25. x2 + 2x + 2y + 7 = 0
[(x − h)2 = 4p(y − k)]
x2 + 2x = −2y − 7
Vertex: (−2, −1) [(h, k)]
(x + 2x + 1) = −2y − 7 + 1 = −2y − 6
2    
1 3
(x + 1)2 = −2(y + 3) Focus: − 2, −1 + , or − 2, − [(h, k + p)]
  4 4
1
[x − (−1)] = 4 −
2
[y − (−3)] 1
Directrix: y = −1− = −
5
(y = k−p)
2 4 4
[(x − h)2 = 4p(y − k)]

Copyright © 2013 Pearson Education, Inc.


Exercise Set 7.1 277

y The depth
 ofthe satellite dish at the vertex is x
15
4 where x, is a point on the parabola.
2
2
y2 = 16x
⫺4 2 4 x
 2
15 15
⫺2 = 16x Substituting for y
2 2
⫺4
225
= 16x
4
y ⫽ x 2 ⫹ 4x ⫹ 3
225
= x, or
64
31. y2 − y − x + 6 = 0
33
y2 − y = x−6 3 = x
64
1 1
y2 − y + = x−6+ The depth of the satellite dish at the vertex is 3
33
ft.
4 4 64
 2
1 23 35. We position a coordinate system with the origin at the
y− = x−
2 4 vertex and the x-axis on the parabola’s axis of symmetry.
 2   The parabola is of the form y 2 = 4px with p = 3.287.
1 1 23
y− = 4· x− Because the parabola is 2.625 in. deep, a point on the
2 4 4
parabola is (2.625, y), where y = one-half the diameter of
[(y − k)2 = 4p(x − h)] the outside edge of the receiver.
 
23 1 y 2 = 4px
Vertex: , [(h, k)]
4 2
    y 2 = 4 · 3.287 · 2.625
23 1 1 1
Focus: + , , or 6, [(h + p, k)] y 2 = 34.5135
4 4 2 2
y ≈ 5.8748
23 1 22 11
Directrix: x = − = or (x = h − p) The width at the opening is about 2 · 5.8748 in., or about
4 4 4 2
11.75 in.
y
37. When we let y = 0 and solve for x, the only equation for
4 2
which x = is (h), so only equation (h) has x-intercept
2   3
2
,0 .
2 4 6 8 x 3
⫺2
7
⫺4 39. Note that equation (g) is equivalent to y = 2x − and
4
1 1
equation (h) is equivalent to y = − x + . When we look
y2 ⫺ y ⫺ x ⫹ 6 ⫽ 0 2 3
at the equations in the form y = mx+b, we see that m > 0
33. a) The vertex is (0, 0). The focus is (4, 0), so p = 4. for (a), (b), (f), and (g) so these equations have positive
slope, or slant up front left to right.
The parabola has a horizontal axis of symmetry so
the equation is of the form y 2 = 4px. We have 41. When we look at the equations in the form y = mx+b (See
y 2 = 4px 1 1
Exercise 39.), only (b) has m = so only (b) has slope .
y2 = 4 · 4 · x 3 3

y 2 = 16x 43. Parallel lines have the same slope and different y-
intercepts. When we look at the equations in the form
b) We make a drawing. y = mx + b (See Exercise 39.), we see that (a) and (g)
represent parallel lines.
y
15 45. A parabola with a vertical axis of symmetry has an equa-
( x, — )
2
tion of the type (x − h)2 = 4p(y − k).
Solve for p substituting (−1, 2) for (h, k) and (−3, 1) for
x x (x, y).
15 ft
[−3 − (−1)]2 = 4p(1 − 2)
15 4 = −4p
( x, – — )
2
−1 = p
The equation of the parabola is

Copyright © 2013 Pearson Education, Inc.


278 Chapter 7: Conic Sections

[x − (−1)]2 = 4(−1)(y − 2), or The lengths of the vertical cables are 10 ft, 11.6 ft, 16.4 ft,
(x + 1)2 = −4(y − 2). 24.4 ft, 35.6 ft, and 50 ft.

47. Vertex: (0.867, 0.348)


Exercise Set 7.2
Focus: (0.867, −0.190)
Directrix: y = 0.887 1. Graph (b) is the graph of x2 + y 2 = 5.
49. Position a coordinate system as shown below with the y- 3. Graph (d) is the graph of x2 + y 2 − 6x + 2y = 6.
axis on the parabola’s axis of symmetry.
5. Graph (a) is the graph of x2 + y 2 − 5x + 3y = 0.
y 7. Complete the square twice.
(–100, 50) (100, 50)
x2 + y 2 − 14x + 4y = 11
(0, 10) x2 − 14x + y 2 + 4y = 11
x x2 − 14x + 49 + y 2 + 4y + 4 = 11 + 49 + 4
20 40 60 80 100
(x − 7)2 + (y + 2)2 = 64
The equation of the parabola is of the form (x − h)2 = (x − 7)2 + [y − (−2)]2 = 82
4p(y − k). Substitute 100 for x, 50 for y, 0 for h, and 10 Center: (7, −2)
for k and solve for p. Radius: 8
(x − h)2 = 4p(y − k)
(100 − 0)2 = 4p(50 − 10) y

10, 000 = 160p 8


250 4
=p
4
⫺8 ⫺4 4 8 12 x
Then the equation is
 
250
x2 = 4 (y − 10), or ⫺8
4
x2 = 250(y − 10).
x 2 ⫹ y 2 ⫺ 14x ⫹ 4y ⫽ 11
To find the lengths of the vertical cables, find y when x = 0,
20, 40, 60, 80, and 100. 9. Complete the square twice.
When x = 0 : 02 = 250(y − 10) x2 + y 2 + 6x − 2y = 6
0 = y − 10 x2 + 6x + y 2 − 2y = 6
10 = y x2 + 6x + 9 + y 2 − 2y + 1 = 6 + 9 + 1
When x = 20 : 20 = 250(y − 10)
2
(x + 3)2 + (y − 1)2 = 16
400 = 250(y − 10) [x − (−3)]2 + (y − 1)2 = 42
1.6 = y − 10 Center: (−3, 1)
11.6 = y Radius: 4
When x = 40 : 402 = 250(y − 10)
y
1600 = 250(y − 10)
4
6.4 = y − 10
2
16.4 = y
⫺6 ⫺4 ⫺2 2
When x = 60 : 602 = 250(y − 10) x
⫺2
3600 = 250(y − 10)
⫺4
14.4 = y − 10
24.4 = y x 2 ⫹ y 2 ⫹ 6x ⫺ 2y ⫽ 6
When x = 80 : 802 = 250(y − 10)
11. Complete the square twice.
6400 = 250(y − 10)
x2 + y 2 + 4x − 6y − 12 = 0
25.6 = y − 10
x2 + 4x + y 2 − 6y = 12
35.6 = y
x2 + 4x + 4 + y 2 − 6y + 9 = 12 + 4 + 9
When x = 100, we know from the given information that
(x + 2)2 + (y − 3)2 = 25
y = 50.
[x − (−2)]2 + (y − 3)2 = 52

Copyright © 2013 Pearson Education, Inc.


Exercise Set 7.2 279

Center: (−2, 3) 17. Complete the square twice.


Radius: 5 x2 + y 2 − 9x = 7 − 4y
x − 9x + y 2 + 4y
2
=7
y
81 81
x2 − 9x + + y 2 + 4y + 4 = 7+ +4
4 4
6  2
9 125
4 x− + (y + 2)2 =
2 4
2  2  √ 2
9 5 5
⫺4 ⫺2 2 4
x− + [y − (−2)]2 =
x 2 2
 
9
⫺4 Center: , −2
2

5 5
x 2 ⫹ y 2 ⫹ 4x ⫺ 6y ⫺ 12 ⫽ 0 Radius:
2
13. Complete the square twice. y
x2 + y 2 − 6x − 8y + 16 = 0
8
x2 − 6x + y 2 − 8y = −16 4
x2 − 6x + 9 + y 2 − 8y + 16 = −16 + 9 + 16
⫺8 ⫺4 4 8 x
(x − 3)2 + (y − 4)2 = 9
(x − 3)2 + (y − 4)2 = 32 ⫺8
Center: (3, 4)
Radius: 3 x 2 ⫹ y 2 ⫺ 9x ⫽ 7 ⫺ 4y

y 19. Graph (c) is the graph of 16x2 + 4y 2 = 64.


6 21. Graph (d) is the graph of x2 + 9y 2 − 6x + 90y = −225.
4
23. x2 y2
2 + =1
4 1
⫺2 2 4 6 x x2 y2
⫺2
2
+ 2 =1 Standard form
2 1
a = 2, b = 1
x 2 ⫹ y 2 ⫺ 6x ⫺ 8y ⫹ 16 ⫽ 0
The major axis is horizontal, so the vertices are (−2, 0)
√ that c = a − b √
2 2 2
15. Complete the square twice. and (2, 0). Since we know , we have
c√= 4 − 1 = 3, so c = 3 and the foci are (− 3, 0) and
2
x2 + y 2 + 6x − 10y = 0
( 3, 0).
x2 + 6x + y 2 − 10y = 0
To graph the ellipse, plot the vertices. Note also that since
x + 6x + 9 + y 2 − 10y + 25 = 0 + 9 + 25
2
b = 1, the y-intercepts are (0, −1) and (0, 1). Plot these
(x + 3)2 + (y − 5)2 = 34 points as well and connect the four plotted points with a
√ smooth curve.
[x − (−3)]2 + (y − 5)2 = ( 34)2
Center: (−3, 5) y

Radius: 34 4

2
y
⫺4 4 x
8
⫺2
4
⫺4
⫺8 4 8 x
⫺4 x2 y2
⫹ ⫽1
⫺8 4 1

x 2 ⫹ y 2 ⫹ 6x ⫺ 10y ⫽ 0

Copyright © 2013 Pearson Education, Inc.


280 Chapter 7: Conic Sections

25. 16x2 + 9y 2 = 144 1 1


a= ,b=
2 3
x2 y2  
+ =1 Dividing by 144 1
9 16 The major axis is horizontal, so the vertices are − , 0
  2
x2 y2 1 1 1 5
32
+
42
=1 Standard form and , 0 . Since c = a −b , we have c = − =
2 2 2 2
,
2√ 4 9 36
a = 4, b = 3  √  √ 
5 5 5
so c = and the foci are − , 0 and ,0 .
The major axis is vertical, so the vertices are (0, −4) and 6 6 6
√ Since c = a − b , we √ have c =√16 − 9 = 7, so
2 2 2 2
(0, 4). To graph the ellipse, plot the 
vertices. Note 
also that
 since
c = 7 and the foci are (0, − 7) and (0, 7). 1 1 1
b = , the y-intercepts are 0, − and 0, . Plot
To graph the ellipse, plot the vertices. Note also that since 3 3 3
b = 3, the x-intercepts are (−3, 0) and (3, 0). Plot these these points as well and connect the four plotted points
points as well and connect the four plotted points with a with a smooth curve.
smooth curve.
y
y 1

2 ⫺1 1 x
⫺4 ⫺2 2 4 x
⫺2 ⫺1

4x 2 ⫹ 9y 2 ⫽ 1
16x ⫹ 9y ⫽ 144
2 2

31. The vertices are on the x-axis, so the major axis is hori-
27. 2 2
2x + 3y = 6 zontal. We have a = 7 and c = 3, so we can find b2 :
c2 = a2 − b2
x2 y2
+ =1 32 = 72 − b2
3 2
x2 y2 b2 = 49 − 9 = 40
√ + √ =1
( 3)2 ( 2)2 Write the equation:
√ √ x2 y2
a = 3, b = 2 + 2 =1
√ a 2 b
The major
√ axis is horizontal, so the vertices are (− 3, 0)
and ( 3, 0). Since c = a − b , we have c = 3 − 2 = 1,
2 2 2 2 x2 y2
+ =1
so c = 1 and the foci are (−1, 0) and (1, 0). 49 40
To graph 33. The vertices, (0, −8) and (0, 8), are on the y-axis, so the
√ the ellipse, plot the vertices.
√ Note also√that since
b = 2, the y-intercepts are (0, − 2) and (0, 2). Plot major axis is vertical and a = 8. Since the vertices are
these points as well and connect the four plotted points equidistant from the origin, the center of the ellipse is at
with a smooth curve. the origin. The length of the minor axis is 10, so b = 10/2,
or 5.
y
Write the equation:
4 x2 y2
2
+ 2 =1
2 b a
⫺4 ⫺2 2 4 x
x2 y2
+ =1
⫺2 52 82
⫺4 x2 y2
+ =1
25 64
2x 2 ⫹ 3y 2 ⫽ 6 35. The foci, (−2, 0) and (2, 0) are on the x-axis, so the major
axis is horizontal and c = 2. Since the foci are equidistant
29. 4x2 + 9y 2 = 1 from the origin, the center of the ellipse is at the origin.
x2 y2 The length of the major axis is 6, so a = 6/2, or 3. Now
1
+
1
=1 we find b2 :
4 9 c2 = a2 − b2
x2 y2 22 = 32 − b2
 2 +  2 = 1 4 = 9 − b2
1 1
2 3 b2 = 5

Copyright © 2013 Pearson Education, Inc.


Exercise Set 7.2 281

Write the equation: y

x2 y2 8
2
+ 2 =1
a b 4
x2 y2
+ =1 ⫺8 4 8 x
9 5 ⫺4

37. (x − 1)2 (y − 2)2 ⫺8


+ =1
9 4
(x − 1)2 (y − 2)2 (x ⫹ 3)2 (y ⫺ 5)2
+ =1 Standard form ⫹ ⫽1
2 25 36
3 22

The center is (1, 2). Note that a = 3 and b = 2. The major 41. 3(x + 2)2 + 4(y − 1)2 = 192
axis is horizontal so the vertices are 3 units left and right (x + 2)2 (y − 1)2
+ =1 Dividing by 192
of the center: 64 48
(1 − 3, 2) and (1 + 3, 2), or (−2, 2) and (4, 2). [x − (−2)]2 (y − 1)2
√ + √ =1 Standard form
We know that c =√a − b , so c = 9 − 4 = 5 and c = 5.
2 2 2 2 8 2
( 48)2
Then the foci are 5 units left and right of the center: √
√ √ √ center is (−2, 1). Note that a = 8 and b = 48, or
The
(1 − 5, 2) and (1 + 5, 2). 4 3. The major axis is horizontal so the vertices are 8
To graph the ellipse, plot the vertices. Since b = 2, two units left and right of the center:
other points on the graph are 2 units below and above the (−2 − 8, 1) and (−2 + 8, 1), or (−10, 1) and (6, 1).
center: We know that c2 = a2 −b2 , so c2 = 64−48 = 16 and c = 4.
(1, 2 − 2) and (1, 2 + 2) or (1, 0) and (1, 4) Then the foci are 4 units left and right of the center:
Plot these points also and connect the four plotted points (−2 − 4, 1) and (−2 + 4, 1) or (−6, 1) and (2, 1).
with a smooth curve. √
To graph the ellipse, plot the vertices. Since b = 4 3 ≈
y
6.928, two other points on the graph are about 6.928 units
(x ⫺ 1)2 (y ⫺ 2)2
below and above the center:
6 ⫹ ⫽1
9 4 (−2, 1 − 6.928) and (−2, 1 + 6.928), or
(−2, −5.928) and (−2, 7.928).
2
Plot these points also and connect the four plotted points
⫺4 ⫺2 2 4 x with a smooth curve.
⫺2
y

39. (x + 3)2 (y − 5)2


+ =1 4
25 36
⫺8 ⫺4
[x − (−3)]2 (y − 5)2 4 8 x
+ =1 Standard form ⫺4
5 2 62
⫺8
The center is (−3, 5). Note that a = 6 and b = 5. The
major axis is vertical so the vertices are 6 units below and 3(x ⫹ 2)2 ⫹ 4(y ⫺ 1)2 ⫽ 192
above the center:
(−3, 5 − 6) and (−3, 5 + 6), or (−3, −1) and (−3, 11). 43. Begin by completing the square twice.
We know
√ that c2 = a2 − b2 , √
so c2 = 36 − 25 = 11 and 4x2 + 9y 2 − 16x + 18y − 11 = 0
c = 11. Then the foci are 11 units below and above 4x2 − 16x + 9y 2 + 18y = 11
the vertex:
√ √ 4(x2 − 4x) + 9(y 2 + 2y) = 11
(−3, 5 − 11) and (−3, 5 + 11).
4(x2 − 4x + 4) + 9(y 2 + 2y + 1) = 11 + 4 · 4 + 9 · 1
To graph the ellipse, plot the vertices. Since b = 5, two
other points on the graph are 5 units left and right of the 4(x − 2)2 + 9(y + 1)2 = 36
center: (x − 2)2 (y + 1)2
+ =1
(−3 − 5, 5) and (−3 + 5, 5), or (−8, 5) and (2, 5) 9 4
Plot these points also and connect the four plotted points (x − 2)2 [y − (−1)]2
+ =1
with a smooth curve. 32 22
The center is (2, −1). Note that a = 3 and b = 2. The
major axis is horizontal so the vertices are 3 units left and
right of the center:

Copyright © 2013 Pearson Education, Inc.


282 Chapter 7: Conic Sections

(2 − 3, −1) and (2 + 3, −1), or (−1, −1) and (5, −1). 47. The ellipse in Example 4 is flatter than the one in Example
√ 2, so the ellipse in Example 2 has the smaller eccentricity.
We know that c =√a − b , so c = 9 − 4 = 5 and c = 5.
2 2 2 2

Then the foci are 5 units left and right of the center: We compute the eccentricities: In Example 2, c = 3 and
√ √ a√= 5, so e = c/a = 3/5 = 0.6.√ In Example 4, c =
(2 − 5, −1) and (2 + 5, −1).
2 3 and a = 4, so e = c/a = 2 3/4 ≈ 0.866. These
To graph the ellipse, plot the vertices. Since b = 2, two computations confirm that the ellipse in Example 2 has
other points on the graph are 2 units below and above the the smaller eccentricity.
center:
(2, −1 − 2) and (2, −1 + 2), or (2, −3) and (2, 1). 49. Since the vertices, (0, −4) and (0, 4) are on the y-axis and
are equidistant from the origin, we know that the major
Plot these points also and connect the four plotted points axis of the ellipse is vertical, its center is at the origin, and
with a smooth curve. a = 4. Use the information that e = 1/4 to find c:
c
y e=
a
4 1 c
4x 2 ⫹ 9y 2 ⫺ 16x ⫹ 18y ⫺ 11 ⫽ 0 = Substituting
2 4 4
⫺4 ⫺2 2 4 x
c=1
Now c2 = a2 − b2 , so we can find b2 :
⫺4 12 = 42 − b2
1 = 16 − b2
45. Begin by completing the square twice. b2 = 15
4x + y − 8x − 2y + 1 = 0
2 2 Write the equation of the ellipse:
4x2 − 8x + y 2 − 2y = −1 x2 y2
+ =1
4(x2 − 2x) + y 2 − 2y = −1 b2 a2
4(x2 − 2x + 1) + y 2 − 2y + 1 = −1 + 4 · 1 + 1 x2 y2
+ =1
15 16
4(x − 1)2 + (y − 1)2 = 4
51. From the figure in the text we see that the center of the
(x − 1)2 (y − 1)2
+ =1 ellipse is (0, 0), the major axis is horizontal, the vertices are
1 4
(−50, 0) and (50, 0), and one y-intercept is (0, 12). Then
(x − 1)2 (y − 1)2 a = 50 and b = 12. The equation is
2
+ =1
1 22 x2 y2
The center is (1, 1). Note that a = 2 and b = 1. The major 2
+ 2 =1
a b
axis is vertical so the vertices are 2 units below and above
the center: x2 y
+ 2 =1
502 12
(1, 1 − 2) and (1, 1 + 2), or (1, −1) and (1, 3).
√ x2 y2
We know that c2 =√a2 − b2 , so c2 = 4 − 1 = 3 and c = 3. + = 1.
2500 144
Then the foci are 3 units below and above the center:
√ √ 53. Position a coordinate system as shown below where
(1, 1 − 3) and (1, 1 + 3).
1 unit = 107 mi.
To graph the ellipse, plot the vertices. Since b = 1, two
other points on the graph are 1 unit left and right of the y

center:
(1 − 1, 1) and (1 + 1, 1) or (0, 1) and (2, 1).
9.3 9.1 x
Plot these points also and connect the four plotted points Focus Sun V
with a smooth curve.

4
The length of the major axis is 9.3 + 9.1, or 18.4. Then
the distance from the center of the ellipse (the origin) to
2
V is 18.4/2, or 9.2. Since the distance from the sun to V
⫺4 ⫺2 2 4 x is 9.1, the distance from the sun to the center is 9.2 − 9.1,
⫺2 or 0.1. Then the distance from the sun to the other focus
⫺4 is twice this distance:
2(0.1 × 107 mi) = 0.2 × 107 mi
4x 2 ⫹ y 2 ⫺ 8x ⫺ 2y ⫹ 1 ⫽ 0 = 2 × 106 mi
55. midpoint

Copyright © 2013 Pearson Education, Inc.


Chapter 7 Mid-Chapter Mixed Review 283

57. y-intercept The equation of the ellipse is

59. remainder x2 y2
2
+ 2 =1
25 14
61. parabola x2 y2
+ = 1.
63. The center of the ellipse is the midpoint of the segment 625 196
connecting the vertices: A point 6 ft from the riverbank corresponds to (25 − 6, 0),
  or (19, 0) or to (−25 + 6, 0), or (−19, 0). Substitute either
3 + 3 −4 + 6
, , or (3, 1). 19 or −19 for x and solve for y, the clearance.
2 2
192 y2
Now a is the distance from the origin to a vertex. We use + =1
the vertex (3, 6). 625 196
 y2 361
a = (3 − 3)2 + (6 − 1)2 = 5 = 1−
196 625
Also b is one-half the length of the minor axis.  
 361
(5 − 1)2 + (1 − 1)2 4 y 2 = 196 1 −
b= = =2 625
2 2
y ≈ 9.1
The vertices lie on the vertical line x = 3, so the major
axis is vertical. We write the equation of the ellipse. The clearance 6 ft from the riverbank is about 9.1 ft.
(x − h) (y − k)
2 2
+ =1
b2 a2 Chapter 7 Mid-Chapter Mixed Review
(x − 3) (y − 1)
2 2
+ =1
4 25
1. The equation (x + 3)2 = 8(y − 2) is equivalent to the equa-
65. The center is the midpoint of the segment connecting the
tion [x − (−3)]2 = 4 · 2(y − 2), so the given statement is
vertices:
  true. See page 581 in the text.
−3 + 3 0 + 0
, , or (0, 0).
2 2 3. The equation (x − 4)2 + (y + 1)2 = 9 is equivalent to the
Then a = 3 and since the vertices are on the x-axis, the equation (x − 4)2 + [y − (−1)]2 = 32 . This is the equation
major axis is horizontal. The equation is of the form of a circle with center (4, −1) and radius 3, so the given
x2 y2 statement is false.
+ 2 = 1.
a2 b 5. Graph (c) is the graph of x2 = −4y.
22
Substitute 3 for a, 2 for x, and for y and solve for b2 .
3 7. Graph (d) is the graph of 16x2 + 9y 2 = 144.
484
4 9. Graph (b) is the graph of (x − 1)2 = 2(y + 3).
+ 92 = 1
9 b 11. Graph (g) is the graph of (x − 2)2 + (y + 3)2 = 4.
4 484
+ 2 =1 13. y 2 = 12x
9 9b
4b2 + 484 = 9b2 y2 = 4 · 3 · x
484 = 5b2 Vertex: (0, 0)
484 Focus: (3, 0)
= b2
5 Directrix: x = −3
x2 y2
Then the equation is + = 1.
9 484/5 y

67. Center: (2.003, −1.005)


4
Vertices: (−1.017, −1.005), (5.023, −1.005) 2

69. Position a coordinate system as shown. 4 2 2 4 x


2
y y 2  12x
4
(0, 14)

(–25, 0) (25, 0) x
15. Since the directrix, y = 1, is a horizontal line, the equation
is of the form (x − h)2 = 4p(y − k). The focus, (0, 3), is
on the y-axis so the axis of symmetry is the y-axis. The
vertex is the point on the y-axis that is midway between
the directrix and the focus. That is, it is the midpoint of

Copyright © 2013 Pearson Education, Inc.


284 Chapter 7: Conic Sections

 
the segment from (0, 1) to (0, 3):
0+0 1+3
, , or (0, 2). (2 − 4, −1) and (2 + 4, −1), or (−2, −1) and (6, −1).
2 2 We √know that√ c2 = a2 − b2 so c2 =√16 − 4 = 12 and
Then k = 2 and the directrix is y = k − p, so we have
c = 12, or 2 3. Then the foci are 2 3 units to the left
y = k−p and right of the center:
√ √
1 = 2−p (2 − 2 3, −1) and (2 + 2 3, −1).
p = 1.
y
Now we find the equation of the parabola.
(x − h)2 = 4p(y − k) 4
(x − 0)2 = 4 · 1 · (y − 2) 2
x2 = 4(y − 2)
2 2 4 6 x
17. x2 + y 2 + 4x − 8y = 5 2

x2 + 4x + y 2 − 8y = 5 4

x + 4x + 4 + y − 8y + 16 = 5 + 4 + 16
2 2
(x  2) 2 (y  1) 2
(x + 2)2 + (y − 4)2 = 25 16  4 1

[x − (−2)]2 + (y − 4)2 = 52
Center: (−2, 4); radius: 5 23. The vertices, (−5, 0) and (5, 0) are on the x-axis, so the
major axis is horizontal and a = 5. Since the vertices are
y equidistant from the origin, the center of the ellipse is the
origin. The foci are (−2, 0) and (2, 0), so we know that
8 c = 2. Then we have
6 c2 = a2 − b2
4 4 = 25 − b2
2 b2 = 21.
Now we write the equation of the ellipse.
6 2 2 x
x2 y2
+ =1
25 21
x 2  y 2  4x  8y  5
25. The foci, (−3, 0) and (3, 0) are on the x-axis, so the major
axis is horizontal. The foci are equidistant from the origin,
19. x2 y2 so the center of the ellipse is the origin. The length of the
+ =1
1 9 major axis is 8, so a = 8/2, or 4. Now we find b2 .
x2 y2 c2 = a2 − b2
2
+ 2 =1
1 3 9 = 16 − b2
a = 3, b = 1
b2 = 7
The major axis is vertical, so the vertices are (0, −3) and
We write the equation of the ellipse.
√ Since√c = a − b we have c √= 9 − 1 = √
2 2 2 2
(0, 3). 8, so
c = 8, or 2 2, and the foci are (0, −2 2) and (0, 2 2). x2 y2
+ =1
16 7
y
27. See page 579 of the text.
4 29. No, the center of an ellipse is not part of the graph of the
2 ellipse. Its coordinates do not satisfy the equation of the
ellipse.
4 2 2 4 x
2

4 x2 y2 Exercise Set 7.3


1  9 1

x2 y2
(x − 2)2
(y + 1) 2 1. Graph (b) is the graph of − = 1.
21. + =1 25 9
16 4
(y − 1)2 (x + 3)2
(x − 2)2 [y − (−1)]2 3. Graph (c) is the graph of − = 1.
2
+ =1 16 1
4 22
The center is (2, −1). Note that a = 4 and b = 2. The 5. Graph (a) is the graph of 25x2 − 16y 2 = 400.
major axis is horizontal, so the vertices are 4 units to the
left and right of the center:

Copyright © 2013 Pearson Education, Inc.


Exercise Set 7.3 285

7. The vertices are equidistant from the origin and are on the (2 − 3, −5) and (2 + 3, −5), or (−1, −5) and (5, −5).
y-axis, so the center is at the origin and the transverse axis √
Since c2 = a2 + b2 √
, we have c2 = 9 + 1 = 10 and c = 10.
is vertical. Since c2 = a2 + b2 , we have 52 = 32 + b2 so Then the foci are 10 units left and right of the center:
b2 = 16. √ √
(2 − 10, −5) and (2 + 10, −5).
y2 x2
The equation is of the form 2 − 2 = 1, so we have Find the asymptotes:
a b
y2 x2 b b
− = 1. y − k= (x − h) and y − k=− (x − h)
9 16 a a
9. The asymptotes pass through the origin, so the center is 1 1
y − (−5)= (x − 2) and y − (−5)=− (x − 2)
the origin. The given vertex is on the x-axis, so the trans- 3 3
b 3
verse axis is horizontal. Since x = x and a = 2, we 1 1
a 2 y + 5= (x − 2) and y + 5=− (x − 2), or
x2 y2 3 3
have b = 3. The equation is of the form 2 − 2 = 1, so 1 17 1 13
a b y= x − and y=− x −
x2 y2 x2 y2 3 3 3 3
we have 2 − 2 = 1, or − = 1.
2 3 4 9 Sketch the asymptotes, plot the vertices, and draw the
graph.
x2 y2
11. − =1
4 4 y
x2 y2
− 2 = 1 Standard form 2
22 2
The center is (0, 0); a = 2 and b = 2. The transverse axis ⫺4 ⫺2 2 4 6 x
⫺2
is horizontal so the vertices are (−2, 0) and (2,
√ 0). Since

c2 = a2 + b2 , we have √c2 = 4 + 4 = √
8 and c = 8, or 2 2.
Then the foci are (−2 2, 0) and (2 2, 0).
Find the asymptotes:
b b (x ⫺ 2)2 (y ⫹ 5)2
y = x and y = − x ⫺ ⫽1
9 1
a a
2 2 (y + 3)2 (x + 1)2
y = x and y = − x 15. − =1
2 2 4 16
y=x and y = −x [y − (−3)]2 [x − (−1)]2
− =1 Standard form
To draw the graph sketch the asymptotes, plot the vertices, 22 42
and draw the branches of the hyperbola outward from the
vertices toward the asymptotes. The center is (−1, −3); a = 2 and b = 4. The transverse
axis is vertical, so the vertices are 2 units below and above
y the center:
4 (−1, −3 − 2) and (1, −3 + 2), or (−1, −5) and (−1, −1).
2 2 2 2

2 √c = a + b , we have c√= 4 + 16 = 20 and c = 20,
Since
or 2 5. Then the foci are 2 5 units below and above of
⫺4 4 x the center:
⫺2 √ √
(−1, −3 − 2 5) and (−1, −3 + 2 5).
⫺4
Find the asymptotes:
a a
x2

y2
⫽1
y − k= (x − h) and y − k=− (x − h)
4 4 b b
2 2
(x − 2)2 (y + 5)2 y−(−3)= (x−(−1)) and y−(−3)=− (x−(−1))
13. − =1 4 4
9 1 1 1
(x − 2)2 [y − (−5)]2 y+3= (x+1) and y+3=− (x+1), or
− =1 Standard form 2 2
32 12 1 5 1 7
y= x − and y=− x −
The center is (2, −5); a = 3 and b = 1. The transverse 2 2 2 2
axis is horizontal, so the vertices are 3 units left and right Sketch the asymptotes, plot the vertices, and draw the
of the center: graph.

Copyright © 2013 Pearson Education, Inc.


286 Chapter 7: Conic Sections

y Sketch the asymptotes, plot the vertices, and draw the


2 graph.

6 x y
⫺2 16
⫺4 12
8
⫺6
4
⫺16 ⫺8 8 12 16 x
(y ⫹ 3)2 (x ⫹ 1)2 ⫺4
⫺ ⫽1
4 16 ⫺8
⫺12
17. x2 − 4y 2 = 4 ⫺16

x2 y2
− =1 9y 2 ⫺ x 2 ⫽ 81
4 1
x2 y2 21. x2 − y 2 = 2
2
− 2 = 1 Standard form
2 1 x2 y2
The center is (0, 0); a = 2 and b = 1. The transverse axis − =1
2 2
is horizontal, so the vertices are (−2, 0) and (2,√0). Since
c2 = a2 + b2 , we 2 x2 y2
√ have c =√4 + 1 = 5 and c = 5. Then √ − √ =1 Standard form
the foci are (− 5, 0) and ( 5, 0). ( 2)2 ( 2)2
√ √
Find the asymptotes: The center is (0, 0); a = 2 and b = √2. The transverse√
b b axis is horizontal, so the vertices are (− 2, 0) and ( 2, 0).
y = x and y = − x Since c2 = a2 + b2 , we have c2 = 2 + 2 = 4 and c = 2.
a a
Then the foci are (−2, 0) and (2, 0).
1 1
y = x and y = − x Find the asymptotes:
2 2
Sketch the asymptotes, plot the vertices, and draw the b b
y = x and y = − x
graph. a a
√ √
2 2
y y = √ x and y = − √ x
x 2 ⫺ 4y 2 ⫽ 4 2 2
4
y=x and y = −x
2
Sketch the asymptotes, plot the vertices, and draw the
⫺4 4 x graph.
⫺2
y
⫺4
4
2 x2 ⫺ y2 ⫽ 2
19. 9y 2 − x2 = 81
y2 x2 ⫺4 ⫺2 2 4 x
− =1 ⫺2
9 81
⫺4
y2 x2
2
− 2 = 1 Standard form
3 9
1
The center is (0, 0); a = 3 and b = 9. The transverse 23. y 2 − x2 =
axis is vertical, so the vertices are (0, −3) and (0, 3). 4
√ Since
c2√= a2 + b2 , we have c2 = 9 +√81 = 90 and √ c = 90, or y2 x2
3 10. Then the foci are (0, −3 10) and (0, 3 10). − =1
1/4 1/4
Find the asymptotes: y2 x2
a a 2
− =1 Standard form
y = x and y = − x (1/2) (1/2)2
b b 1 1
3 3 The center is (0, 0); a = and b = . The transverse axis
y = x and y = − x 2  2  
9 9 1 1
is vertical, so the vertices are 0, − and 0, . Since
1 1 2 2 
y = x and y = − x
3 3 1 1 1 1
c2 = a2 + b2 , we have c2 = + = and c = , or
√  √ 
4 4 2 √  2
2 2 2
. Then the foci are 0, − and 0, .
2 2 2

Copyright © 2013 Pearson Education, Inc.


Exercise Set 7.3 287

Find the asymptotes: 27. Begin by completing the square twice.


a a 36x2 − y 2 − 24x + 6y − 41 = 0
y= x and y = − x
b b
(36x2 − 24x) − (y 2 − 6y) = 41
1/2 1/2  
y= x and y = − x 2
1/2 1/2 36 x2 − x − (y 2 − 6y) = 41
3
y=x and y = −x  
2 1 1
Sketch the asymptotes, plot the vertices, and draw the 36 x2 − x+ −(y 2 −6y+9) = 41+36· −1 · 9
3 9 9
graph.  2
1
y 36 x − − (y − 3)2 = 36
3
1  2
y 2 ⫺ x 2 ⫽ –14 x−
1
3 (y − 3)2
− =1
⫺1 1 x 1 36
 2
1
⫺1 x−
3 (y − 3)2
2
− = 1 Standard
1 62 form
 
25. Begin by completing the square twice. 1
The center is , 3 ; a = 1 and b = 6. The transverse
x2 − y 2 − 2x − 4y − 4 = 0 3
axis is horizontal, so the vertices are 1 unit left and right
(x2 − 2x) − (y 2 + 4y) = 4 of the center:
     
(x2 − 2x + 1) − (y 2 + 4y + 4) = 4 + 1 − 1 · 4 1 1 2
− 1, 3 and + 1, 3 or − , 3 and
(x − 1)2 − (y + 2)2 = 1 3  3 3
4
(x − 1)2 [y − (−2)]2 ,3 .
2
− =1 Standard 3
1 12 form √
Since c2 = a2 + b2 ,√we have c2 = 1 + 36 = 37 and c = 37.
The center is (1, −2); a = 1 and b = 1. The transverse Then the foci are 37 units left and right of the center:
   
axis is horizontal, so the vertices are 1 unit left and right 1 √ 1 √
of the center: − 37, 3 and + 37, 3 .
3 3
(1 − 1, −2) and (1 + 1, −2) or (0, −2) and (2, −2) Find the asymptotes:

Since c2 = a2 + b2√
, we have c2 = 1 + 1 = 2 and c = 2. b b
Then the foci are 2 units left and right of the center: y − k = (x − h) and y − k = − (x − h)
√ √ a a
(1 − 2, −2) and (1 + 2, −2).    
6 1 6 1
y−3 = x− and y − 3 = − x −
Find the asymptotes: 1 3 1 3
b b    
y − k = (x − h) and y − k = − (x − h) 1 1
a a y−3 = 6 x− and y − 3 = −6 x − , or
3 3
1 1
y − (−2) = (x − 1) and y − (−2) = − (x − 1) y = 6x + 1 and y = −6x + 5
1 1 Sketch the asymptotes, plot the vertices, and draw the
y+2 = x−1 and y + 2 = −(x − 1), or graph.
y = x−3 and y = −x − 1
y
Sketch the asymptotes, plot the vertices, and draw the
graph. 36x 2 ⫺ y 2 ⫺ 24x ⫹ 6y ⫺ 41 ⫽ 0

2 ⫺8 ⫺4 4 8 x
⫺4 ⫺2 4 6 x

x 2 ⫺ y 2 ⫺ 2x ⫺ 4y ⫺ 4 ⫽ 0
⫺6

Copyright © 2013 Pearson Education, Inc.


288 Chapter 7: Conic Sections


29. Begin by completing the square twice. Since c2 = a2 + b2√
, we have c2 = 1 + 1 = 2 and c = 2.
9y 2 − 4x2 − 18y + 24x − 63 = 0 Then the foci are 2 units left and right of the center:
√ √
9(y 2 − 2y) − 4(x2 − 6x) = 63 (1 − 2, −2) and (1 + 2, −2).
9(y 2 − 2y + 1) − 4(x2 − 6x + 9) = 63+9·1−4·9 Find the asymptotes:
9(y − 1) − 4(x − 3) = 36
2 2 b b
y − k = (x − h) and y − k = − (x − h)
a a
(y − 1)2 (x − 3)2
− =1 1 1
4 9 y − (−2) = (x − 1) and y − (−2) = − (x − 1)
1 1
(y − 1)2 (x − 3)2
2
− =1 Standard y+2 = x−1 and y + 2 = −(x − 1), or
2 32 form y = x−3 and y = −x − 1
The center is (3, 1); a = 2 and b = 3. The transverse axis Sketch the asymptotes, plot the vertices, and draw the
is vertical, so the vertices are 2 units below and above the graph.
center:
(3, 1 − 2) and (3, 1 + 2), or (3, −1) and (3, 3). y

Since c2 = a2 + b2 √
, we have c2 = 4 + 9 = 13 and c = 13. 4
Then the foci are 13 units below and above the center: 2
√ √
(3, 1 − 13) and (3, 1 + 13). ⫺4 ⫺2 2 4 x
Find the asymptotes:
a a
y − k = (x − h) and y − k = − (x − h)
b b
2 2 x 2 ⫺ y 2 ⫺ 2x ⫺ 4y ⫽ 4
y − 1 = (x − 3) and y − 1 = − (x − 3), or
3 3
2 2 33. Begin by completing the square twice.
y = x − 1 and y = − x+3
3 3 y 2 − x2 − 6x − 8y − 29 = 0
Sketch the asymptotes, plot the vertices, and draw the
(y 2 − 8y + 16) − (x2 + 6x + 9) = 29 + 16 − 9
graph.
(y − 4)2 − (x + 3)2 = 36
y (y − 4)2 (x + 3)2
− =1
6 36 36
(y − 4)2 [x − (−3)]2
2
− =1 Standard
2 6 62 form
⫺2 2 4 6 8 x The center is (−3, 4); a = 6 and b = 6. The transverse
axis is vertical, so the vertices are 6 units below and above
⫺4 the center:
(−3, 4 − 6) and (−3, 4 + 6), or (−3, −2) and (−3, 10).

9y 2 ⫺ 4x 2 ⫺ 18y ⫹ 24x ⫺ 63 ⫽ 0 2 2 2 2
√c = a +b , we have c√= 36+36 = 72 and c = 72,
Since
or 6 2. Then the foci are 6 2 units below and above the
31. Begin by completing the square twice. center:
√ √
x2 − y 2 − 2x − 4y = 4 (−3, 4 − 6 2) and (−3, 4 + 6 2).
(x2 − 2x + 1) − (y 2 + 4y + 4) = 4 + 1 − 4 Find the asymptotes:
(x − 1)2 − (y + 2)2 = 1 a a
y − k = (x − h) and y − k = − (x − h)
b b
(x − 1)2 [y − (−2)]2
− =1 Standard 6 6
12 12 form y − 4 = (x − (−3)) and y − 4 = − (x − (−3))
6 6
The center is (1, −2); a = 1 and b = 1. The transverse y−4 = x+3 and y − 4 = −(x + 3), or
axis is horizontal, so the vertices are 1 unit left and right
y = x+7 and y = −x + 1
of the center:
Sketch the asymptotes, plot the vertices, and draw the
(1 − 1, −2) and (1 + 1, −2), or (0, −2) and (2, −2). graph.

Copyright © 2013 Pearson Education, Inc.


Exercise Set 7.3 289

y c2 = a2 + b2
12 62 = 52 + b2
8 11 = b2
4 Write the equation:
y2 x2
⫺12 ⫺4 4 8 x 2
− 2 =1
⫺4 a b
⫺8 y2 x2
− =1
25 11
y 2 ⫺ x 2 ⫺ 6x ⫺ 8y ⫺ 29 ⫽ 0
41. a) The graph of f (x) = 2x − 3 is shown below.
35. The hyperbola in Example 3 is wider than the one in Ex- y
ample 2, so the hyperbola in Example 3 has the larger
5
eccentricity.
4
Compute the eccentricities: In Example 2,√c = 5 and a = 4, 3
= 5/4, or 1.25. In Example 3, c = 5 and a = 1, so
so e √ 2
e = 5/1 ≈ 2.24. These computations confirm that the 1

hyperbola in Example 3 has the larger eccentricity. –5 –4 –3 –2 –1 1 2 3 4 5 x


–1

37. The center is the midpoint of the segment connecting the –2 f (x ) = 2x — 3


–3
vertices:
  –4
3−3 7+7 –5
, , or (0, 7).
2 2
The vertices are on the horizontal line y = 7, so the trans- Since there is no horizontal line that crosses the
verse axis is horizontal. Since the vertices are 3 units left graph more than once, the function is one-to-one.
and right of the center, a = 3. b) Replace f (x) with y: y = 2x − 3
Find c: Interchange x and y: x = 2y − 3
c 5
e= = Solve for y: x + 3 = 2y
a 3
x+3
c 5 =y
= Substituting 3 for a 2
3 3 x+3
c=5 Replace y with f −1 (x): f −1 (x) =
2
Now find b2 :
c2 = a2 + b2 5
43. a) The graph of f (x) = is shown below.
x−1
52 = 32 + b2
16 = b2 y

Write the equation: 5


4
(x − h) (y − k)
2 2 5
− =1 f (x ) = x 1 3
a2 b2 2

x2 (y − 7)2 1
− =1 –5 –4 –3 –2 –1 1 2 3 4 5 x
9 16 –1

39. y
–2
–3
(0, 6) F1 Hyperbola –4
(0, 5) –5

x
Since there is no horizontal line that crosses the
graph more than once, the function is one-to-one.
5
(0, –6) F2 b) Replace f (x) with y: y =
Parabola x−1
(0, –8) 5
Interchange x and y: x =
y−1
One focus is 6 units above the center of the hyperbola, so
c = 6. One vertex is 5 units above the center, so a = 5.
Find b2 :

Copyright © 2013 Pearson Education, Inc.


290 Chapter 7: Conic Sections

Solve for y: x(y − 1) = 5 53. S and T are the foci of the hyperbola, so c = 300/2 = 150.
5 0.186 mi
y−1 = 200 microseconds · = 37.2 mi, the
x 1 microsecond
5 difference of the ships’ distances from the foci. That is,
y = +1 2a = 37.2, so a = 18.6.
x
−1 −1 5 5+x Find b2 :
Replace y with f (x): f = + 1, or
x x c2 = a2 + b2
45. x + y = 5, (1) 1502 = 18.62 + b2
x−y=7 (2)
22, 154.04 = b2
2x = 12 Adding
x=6 Then the equation of the hyperbola is
Back-substitute in either equation (1) or (2) and solve for x2 y2 x2 y2
− = 1, or − = 1.
y. We use equation (1). 18.62 22, 154.04 345.96 22, 154.04
6+y = 5
y = −1 Exercise Set 7.4
The solution is (6, −1).
1. The correct graph is (e).
47. 2x − 3y = 7, (1)
3x + 5y = 1 (2) 3. The correct graph is (c).
Multiply equation (1) by 5 and equation (2) by 3 and add 5. The correct graph is (b).
to eliminate y.
7. x2 + y 2 = 25, (1)
10x − 15y = 35
9x + 15y = 3 y−x=1 (2)
19x = 38 First solve equation (2) for y.
x=2
y =x+1 (3)
Back-substitute and solve for y.
Then substitute x + 1 for y in equation (1) and solve for
3 · 2 + 5y = 1 Using equation (2) x.
5y = −5 x2 + y 2 = 25
y = −1 x2 + (x + 1)2 = 25
The solution is (2, −1). x + x2 + 2x + 1 = 25
2

49. The center is the midpoint of the segment connecting 2x2 + 2x − 24 = 0


(3, −8) and (3, −2): 1
  x2 + x − 12 = 0 Multiplying by
3 + 3 −8 − 2 2
, , or (3, −5). (x + 4)(x − 3) = 0 Factoring
2 2
The vertices are on the vertical line x = 3 and are 3 units x+4 = 0 or x − 3 = 0 Principle of zero
above and below the center so the transverse axis is vertical products
and a = 3. Use the equation of an asymptote to find b: x = −4 or x=3
a
y − k = (x − h) Now substitute these numbers into equation (3) and solve
b for y.
3 y = −4 + 1 = −3
y + 5 = (x − 3)
b
y = 3+1=4
3 9
y = x− −5 The pairs (−4, −3) and (3, 4) check, so they are the solu-
b b
This equation corresponds to the asymptote y = 3x − 14, tions.
3
so = 3 and b = 1. 9. 4x2 + 9y 2 = 36, (1)
b
Write the equation of the hyperbola: 3y + 2x = 6 (2)
(y − k)2 (x − h)2 First solve equation (2) for y.
− =1
a2 b2 3y = −2x + 6
(y + 5)2 (x − 3)2 2
− =1 y = − x+2 (3)
9 1 3
2
51. Center: (−1.460, −0.957) Then substitute − x + 2 for y in equation (1) and solve
3
Vertices: (−2.360, −0.957), (−0.560, −0.957) for x.
Asymptotes: y = −1.20x − 2.70, y = 1.20x + 0.79

Copyright © 2013 Pearson Education, Inc.


Exercise Set 7.4 291

4x2 + 9y 2 = 36 15. y 2 − x2 = 9 (1)


 2 2
4x2 + 9 − x + 2 = 36 2x − 3 = y (2)
3
4  Substitute 2x − 3 for y in equation (1) and solve for x.
8
4x + 9 x − x + 4
2 2
= 36
9 3 y 2 − x2 = 9
4x2 + 4x2 − 24x + 36 = 36
(2x − 3)2 − x2 = 9
8x2 − 24x = 0
4x2 − 12x + 9 − x2 = 9
x2 − 3x = 0
3x2 − 12x = 0
x(x − 3) = 0
x2 − 4x = 0
x = 0 or x = 3
x(x − 4) = 0
Now substitute these numbers in equation (3) and solve
x = 0 or x = 4
for y.
2 Now substitute these numbers into equation (2) and solve
y = − ·0+2=2 for y.
3
2 If x = 0, y = 2 · 0 − 3 = −3.
y = − ·3+2=0
3 If x = 4, y = 2 · 4 − 3 = 5.
The pairs (0, 2) and (3, 0) check, so they are the solutions.
The pairs (0, −3) and (4, 5) check. They are the solutions.
11. 2 2
x + y = 25, (1) 17. y 2 = x + 3, (1)
y2 = x + 5 (2)
2y = x + 4 (2)
We substitute x + 5 for y 2 in equation (1) and solve for x. First solve equation (2) for x.
x2 + y 2 = 25 2y − 4 = x (3)
x2 + (x + 5) = 25 Then substitute 2y − 4 for x in equation (1) and solve for
x2 + x − 20 = 0 y.
(x + 5)(x − 4) = 0 y2 = x + 3
x+5 = 0 or x−4 = 0 y 2 = (2y − 4) + 3
x = −5 or x=4 y 2 = 2y − 1
We substitute these numbers for x in either equation (1) y 2 − 2y + 1 = 0
or equation (2) and solve for y. Here we use equation (2). (y − 1)(y − 1) = 0
y 2 = −5 + 5 = 0 and y = 0. y − 1 = 0 or y−1 = 0
y 2 = 4 + 5 = 9 and y = ±3. y = 1 or y=1
The pairs (−5, 0), (4, 3) and (4, −3) check. They are the Now substitute 1 for y in equation (3) and solve for x.
solutions. 2·1−4 = x
13. x2 + y 2 = 9, (1) −2 = x
x −y =9
2 2
(2) The pair (−2, 1) checks. It is the solution.
Here we use the elimination method. 19. x2 + y 2 = 25, (1)
x2 + y 2 = 9 (1)
xy = 12 (2)
x2 − y 2 = 9 (2)
First we solve equation (2) for y.
2x2 = 18 Adding
xy = 12
x2 = 9 12
y=
x = ±3 x
If x = 3, x2 = 9, and if x = −3, x2 = 9, so substituting 3 12
Then we substitute for y in equation (1) and solve for
or −3 in equation (1) gives us x.
x
x2 + y 2 = 9
9 + y2 = 9
y2 = 0
y = 0.
The pairs (3, 0) and (−3, 0) check. They are the solutions.

Copyright © 2013 Pearson Education, Inc.


292 Chapter 7: Conic Sections

x2 + y 2 = 25 Then substitute −2y + 7 for x in equation (1) and solve


 12 2 for y.
x2 + = 25
x x2 + 4y 2 = 25
2 144 (−2y + 7)2 + 4y 2 = 25
x + 2 = 25
x
4y − 28y + 49 + 4y 2 = 25
2
x4 + 144 = 25x2 Multiplying by x2
8y 2 − 28y + 24 = 0
x4 − 25x2 + 144 = 0
2y 2 − 7y + 6 = 0
u2 − 25u + 144 = 0 Letting u = x2
(2y − 3)(y − 2) = 0
(u − 9)(u − 16) = 0
3
u = 9 or u = 16 y = or y = 2
2
We now substitute x2 for u and solve for x. Now substitute these numbers in equation (3) and solve
x2 = 9 or x2 = 16 for x.
x = ±3 or x = ±4 3
x = −2 · + 7 = 4
Since y = 12/x, if x = 3, y = 4; if x = −3, y = −4; 2
if x = 4, y = 3; and if x = −4, y = −3. The pairs x = −2 · 2 + 7 = 3
 3
(3, 4), (−3, −4), (4, 3), and (−4, −3) check. They are the
The pairs 4, and (3, 2) check, so they are the solutions.
solutions. 2
25. x2 − xy + 3y 2 = 27, (1)
21. x2 + y 2 = 4, (1)
16x2 + 9y 2 = 144 (2) x−y =2 (2)
First solve equation (2) for y.
−9x2 − 9y 2 = −36 Multiplying (1) by −9
16x2 + 9y 2 = 144 x−2=y (3)
7x2 = 108 Adding Then substitute x − 2 for y in equation (1) and solve for
108 x.
x2 = x2 − xy + 3y 2 = 27
7
 
108 3 x2 − x(x − 2) + 3(x − 2)2 = 27
x=± = ±6
7 7 x2 − x2 + 2x + 3x2 − 12x + 12 = 27

x=±
6 21 Rationalizing the de- 3x2 − 10x − 15 = 0
7 nominator 
√ √ −(−10) ± (−10)2 − 4(3)(−15)
x=
6 21
or −
6 21 2·3
Substituting for x in equation (1) gives √ √
7 7 10 ± 100 + 180 10 ± 280
us x= =
36 · 21 6 6
+ y2 = 4 √ √
49 10 ± 2 70 5 ± 70
108 x= =
y2 = 4 − 6 3
7 Now substitute these numbers in equation (3) and solve
80 for y.
y2 = − √ √

7
 5 + 70 −1 + 70
y= −2=
80 5 3 3
y=± − = ±4i √ √

7 7 5 − 70 −1 − 70
y= −2=
4i 35 3 3
y=±
7
. Rationalizing the  5 + √70 −1 + √70 
denominator The pairs , and
 6√21 4i√35 
3 3

 5 − 70 −1 − 70  √
The pairs , , , check, so they are the solutions.
7 7
 6√21 4i√35   6√21 4i√35 
3 3
,− , − , , and 27. x2 + y 2 = 16, x2 + y 2 = 16, (1)
7 7 7 7

 6 21 4i 35  √ or
y 2 − 2x2 = 10 −2x2 + y 2 = 10 (2)
− ,− check. They are the solutions.
7 7 Here we use the elimination method.
23. x2 + 4y 2 = 25, (1)
2x2 + 2y 2 = 32 Multiplying (1) by 2
x + 2y = 7 (2) −2x2 + y 2 = 10
First solve equation (2) for x. 3y 2 = 42 Adding
y2 = 14√
x = −2y + 7 (3)
y = ± 14

Copyright © 2013 Pearson Education, Inc.


Exercise Set 7.4 293

√ √ √  √ 
Substituting 14 or − 14 for y in equation (1) gives us 15 + 561 15 + 561
If x = , y =7−3 , or
x2 + 14 = 16 8 8

2
x =2 11 − 3 561
√ .
x=± 2 8
 √ √ 
√ √ √ √ √ √ 15 − 561 11 + 3 561
The
√ √pairs (− 2, − 14), (− 2, 14), ( 2, − 14), and The pairs , and
8 8
( 2, 14) check. They are the solutions.  √ √ 
15 + 561 11 − 3 561
29. x2 + y 2 = 5, (1) , check and are the solutions.
8 8
xy = 2 (2)
33. a + b = 7, (1)
First we solve equation (2) for y.
ab = 4 (2)
xy = 2
2 First solve equation (1) for a.
y= a = −b + 7 (3)
x
2 Then substitute −b + 7 for a in equation (2) and solve for
Then we substitute for y in equation (1) and solve for
x b.
x.
(−b + 7)b = 4
x2 + y 2 = 5
 2 2 −b2 + 7b = 4
x2 + =5
x 0 = b2 − 7b + 4
4 
x2 + 2 = 5 −(−7) ± (−7)2 − 4 · 1 · 4
x b=
2·1
x4 + 4 = 5x2 Multiplying by x2 √
7 ± 33
x4 − 5x2 + 4 = 0 b=
2
u2 − 5u + 4 = 0 Letting u = x2 Now substitute these numbers in equation (3) and solve
for a.
(u − 4)(u − 1) = 0  7 + √33  √
7 − 33
u = 4 or u = 1 a=− +7=
2 2
We now substitute x2 for u and solve for x.  7 − √33  √
7 + 33
x2 = 4 or x2 = 1 a=− +7=
2 2
x = ±2 x = ±1  7 − √33 7 + √33 
Since y = 2/x, if x = 2, y = 1; if x = −2, y = −1; if x = 1, The pairs , and
2 2
y = 2; and if x = −1, y = −2. The pairs (2, 1), (−2, −1), √
 7 + 33 7 − 33 √
(1, 2), and (−1, −2) check. They are the solutions. , check, so they are the
2 2
31. 3x + y = 7 (1) solutions.
4x2 + 5y = 56 (2)
35. x2 + y 2 = 13, (1)
First solve equation (1) for y.
xy = 6 (2)
3x + y = 7
First we solve equation (2) for y.
y = 7 − 3x (3)
xy = 6
Next substitute 7 − 3x for y in equation (2) and solve for 6
x. y=
x
4x2 + 5y = 56 6
Then we substitute for y in equation (1) and solve for
4x2 + 5(7 − 3x) = 56 x
x.
4x2 + 35 − 15x = 56 x2 + y 2 = 13
4x2 − 15x − 21 = 0  6 2
x2 + = 13
Using the quadratic formula, we find that x
√ √
15 − 561 15 + 561 36
x= or x = . x2 + 2 = 13
8 8 x
Now substitute these numbers into equation (3) and solve x4 + 36 = 13x2 Multiplying by x2
for y.
√  √  x4 − 13x2 + 36 = 0
15 − 561 15 − 561
If x = , y =7−3 , or u2 − 13u + 36 = 0 Letting u = x2
8 8
√ (u − 9)(u − 4) = 0
11 + 3 561
. u=9 or u=4
8

Copyright © 2013 Pearson Education, Inc.


294 Chapter 7: Conic Sections

We now substitute x2 for u and solve for x. 2a + 4 − a2 = 1


x2 = 9 or x2 = 4 0 = a2 − 2a − 3
x = ±3 or x = ±2 0 = (a − 3)(a + 1)
Since y = 6/x, if x = 3, y = 2; if x = −3, y = −2; a=3 or a = −1
if x = 2, y = 3; and if x = −2, y = −3. The pairs
Substitute these numbers in equation (2) and solve for b.
(3, 2), (−3, −2), (2, 3), and (−2, −3) check. They are the
solutions. b = 4 − 32 = −5
b = 4 − (−1)2 = 3
37. x2 + y 2 + 6y + 5 = 0 (1)
The pairs (3, −5) and (−1, 3) check. They are the solu-
x2 + y 2 − 2x − 8 = 0 (2)
tions.
Using the elimination method, multiply equation (2) by
−1 and add the result to equation (1). 41. a2 + b2 = 89, (1)
x2 + y 2 + 6y + 5 = 0 (1) a−b=3 (2)
−x2 − y 2 + 2x + 8 = 0 (2) First solve equation (2) for a.
2x + 6y + 13 = 0 (3) a=b+3 (3)
Solve equation (3) for x. Then substitute b + 3 for a in equation (1) and solve for b.
2x + 6y + 13 = 0 (b + 3)2 + b2 = 89
2x = −6y − 13 b2 + 6b + 9 + b2 = 89
−6y − 13
x= 2b2 + 6b − 80 = 0
2
−6y − 13 b2 + 3b − 40 = 0
Substitute for x in equation (1) and solve for y.
2 (b + 8)(b − 5) = 0
x2 + y 2 + 6y + 5 = 0 b = −8 or b = 5
 2
−6y − 13 Substitute these numbers in equation (3) and solve for a.
+ y 2 + 6y + 5 = 0
2 a = −8 + 3 = −5
36y 2 + 156y + 169 a = 5+3=8
+ y 2 + 6y + 5 = 0
4 The pairs (−5, −8) and (8, 5) check. They are the solu-
36y 2 + 156y + 169 + 4y 2 + 24y + 20 = 0 tions.
40y 2 + 180y + 189 = 0
43. xy − y 2 = 2, (1)
Using the quadratic formula, we find that 2xy − 3y 2 = 0 (2)
√ √
−45 ± 3 15 −45 ± 3 15 −2xy + 2y 2 = −4 Multiplying (1) by −2
y= . Substitute for y in
20 20 2xy − 3y 2 = 0
−6y − 13
x= and solve for x. −y 2 = −4 Adding
2 y2
√ = 4
−45 + 3 15 y = ±2
If y = , then
20
 √  We substitute for y in equation (1) and solve for x.
−45 + 3 15
−6 − 13 √ When y = 2 : x · 2 − 22 = 2
20 5 − 9 15
x= = . 2x − 4 = 2
2 20
√ 2x = 6
−45 − 3 15
If y = , then x=3
20
 √ 
−45 − 3 15 When y = −2 : x(−2) − (−2)2 = 2
−6 − 13 √
x=
20
=
5 + 9 15
. −2x − 4 = 2
2 20 −2x = 6
 √ √ 
5 + 9 15 −45 − 3 15
The pairs , and x = −3
20 20
 √ √  The pairs (3, 2) and (−3, −2) check. They are the solu-
5 − 9 15 −45 + 3 15 tions.
, check and are the solutions.
20 20
45. m2 − 3mn + n2 + 1 = 0, (1)
39. 2a + b = 1, (1)
3m − mn + 3n
2 2
= 13 (2)
b = 4 − a2 (2)
Equation (2) is already solved for b. Substitute 4 − a2 for m2 − 3mn + n2 = −1 (3) Rewriting (1)
b in equation (1) and solve for a. 3m − mn + 3n = 13
2 2
(2)

Copyright © 2013 Pearson Education, Inc.


Exercise Set 7.4 295

−3m2 +9mn− 3n2 = 3 Multiplying (3) by −3 Now substitute these numbers in equation (3) and solve
3m2 − mn+ 3n2 = 13 for x.
3 √ 3 √
8mn = 16 x = −4 + i 6 + 8 = 4 + i 6
mn = 2 2 2
2 3 √ 3 √
n= (4) x = −4 − i 6 + 8 = 4 − i 6
m 2 2
 
2 3 √ 3 √
Substitute for n in equation (1) and solve for m. The pairs 4 + i 6, −4 + i 6 and
m 2 2
   2  
2 2 3 √ 3 √
m2 − 3m + +1 = 0 4 − i 6, −4 − i 6 check. They are the solutions.
m m 2 2
4
m2 − 6 + 2 + 1 = 0 49. a2 + b2 = 14, (1)
m √
ab = 3 5 (2)
4
m2 − 5 + 2 = 0 Solve equation (2) for b.
m

m4 − 5m2 + 4 = 0 Multiplying 3 5
b=
by m2 a

Substitute u for m2 . 3 5
Substitute for b in equation (1) and solve for a.
u2 − 5u + 4 = 0 a
 √ 2
(u − 4)(u − 1) = 0 3 5
a2 + = 14
u=4 or u=1 a
45
m2 = 4 or m2 = 1 a2 + 2 = 14
a
m = ±2 or m = ±1
a4 + 45 = 14a2
Substitute for m in equation (4) and solve for n. a4 − 14a2 + 45 = 0
2
When m = 2, n = = 1. u2 − 14u + 45 = 0 Letting u = a2
2
2 (u − 9)(u − 5) = 0
When m = −2, n = = −1.
−2 u=9 or u=5
2 2
a =9 or a2 = 5
When m = 1, n = = 2.
1 √
2 a = ±3 or a = ± 5
When m = −1, n = = −2. √ √ √
−1 = 3 5/a, if a = 3, b = √5; if a = −3, b = − 5;
Since b √
The pairs (2, 1), (−2, −1), (1, 2), and (−1, −2) check. They if a√= 5, b =√3; and √ if a = −
√ 5, b = −3. The pairs
are the solutions. (3, 5), (−3, − 5), ( 5, 3), (− 5, −3) check. They are
the solutions.
47. x2 + y 2 = 5, (1)
51. x2 + y 2 = 25, (1)
x−y =8 (2)
2 2
9x + 4y = 36 (2)
First solve equation (2) for x.
x=y+8 (3) −4x − 4y = −100
2 2
Multiplying (1) by −4
Then substitute y + 8 for x in equation (1) and solve for
9x2 + 4y 2 = 36
y.
5x2 = −64
(y + 8)2 + y 2 = 5 64
x2 = −
y + 16y + 64 + y 2 = 5
2
5

2y 2 + 16y + 59 = 0 −64 8i
 x=± = ±√
−16 ± (16)2 − 4(2)(59) 5 5

y = 8i 5
2·2 x=± Rationalizing the
√ 5 denominator
−16 ± −216
y = √ √
4 8i 5 8i 5
√ Substituting or − for x in equation (1) and
−16 ± 6i 6 5 5
y = solving for y gives us
4
3 √
y = −4 ± i 6
2

Copyright © 2013 Pearson Education, Inc.


296 Chapter 7: Conic Sections

64 55. The statement is true. See Example 4, for instance.


− + y 2 = 25
5
189 57. The statement is true because a line and a circle can in-
y2 = tersect in at most two points.
5
 
189 21 59. Familiarize. We first make a drawing. We let l and w
y=± = ±3 represent the length and width, respectively.
5 5

3 105 ✚
y=± . Rationalizing the ✚
10✚✚
5 denominator
 √ √   √ √  ✚
w
8i 5 3 105 8i 5 3 105 ✚
The pairs , , − , , ✚
5 5 5 5
 √ √   √ √  ✚
8i 5 3 105 8i 5 3 105 l
,− , and − ,−
5 5 5 5 Translate. The perimeter is 28 cm.
check.
2l + 2w = 28, or l + w = 14
They are the solutions.
Using the Pythagorean theorem we have another equation.
53. 5y 2 − x2 = 1, (1) l2 + w2 = 102 , or l2 + w2 = 100
xy = 2 (2) Carry out. We solve the system:
Solve equation (2) for x. l + w = 14, (1)
2 2 2
x= l + w = 100 (2)
y
First solve equation (1) for w.
2
Substitute for x in equation (1) and solve for y. w = 14 − l (3)
y
 2 Then substitute 14 − l for w in equation (2) and solve for
2
5y 2 − =1 l.
y
l2 + w2 = 100
4
5y 2 − 2 = 1 l2 + (14 − l)2 = 100
y
5y 4 − 4 = y 2 l + 196 − 28l + l2 = 100
2

5y 4 − y 2 − 4 = 0 2l2 − 28l + 96 = 0
5u2 − u − 4 = 0 Letting u = y 2 l2 − 14l + 48 = 0
(5u + 4)(u − 1) = 0 (l − 8)(l − 6) = 0
5u + 4 = 0 or u − 1 = 0 l = 8 or l = 6
4 If l = 8, then w = 14 − 8, or 6. If l = 6, then w = 14 − 6,
u= − or u=1 or 8. Since the length is usually considered to be longer
5
than the width, we have the solution l = 8 and w = 6, or
4
y2 = − or y2 = 1 (8, 6).
5
2i Check. If l = 8 and w = 6, then the√perimeter is 2·8+2·6,

y= ±√ or y = ±1 or 28. The length of a diagonal is 82 + 62 , or 100, or
5 10. The numbers check.

2i 5
y= ± or y = ±1 State. The length is 8 cm, and the width is 6 cm.
5

2i 5 2 5 61. Familiarize. We first make a drawing. Let l = the length
Since x = 2/y, if y = ,x= √ = √ = and w = the width of the brochure.
5 2i 5 i 5
√ √5
5 −i 5 √ 2i 5
√ · √ = −i 5; if y = − ,
i 5 −i 5 5 w
2 √
x= √ = i 5;
2i 5
− l
5
if y = 1, x = 2/1 = 2; if y = −1, x = 2/ − 1 = −2. Translate.
 √   √ 
√ 2i 5 √ 2i 5 Area: lw = 20
The pairs − i 5, , i 5, − , (2, 1) and
5 5 Perimeter: 2l + 2w = 18, or l + w = 9
(−2, −1) check. They are the solutions.
Carry out. We solve the system:

Copyright © 2013 Pearson Education, Inc.


Exercise Set 7.4 297

Solve the second equation for l: l = 9 − w Translate.



Substitute 9 − w for l in the first equation and solve for w. Area: lw = 3 (1)
(9 − w)w = 20 From the Pythagorean theorem: l2 + w2 = 22 (2)
9w − w2 = 20 Carry out. We solve the system of equations.
0 = w2 − 9w + 20 We first solve equation (1) for w.

0 = (w − 5)(w − 4) lw = 3
w = 5 or w = 4 √
3
If w = 5, then l = 9 − w, or 4. If w = 4, then l = 9 − 4, w=
l
or 5. Since length is usually considered to be longer than √
3
width, we have the solution l = 5 and w = 4, or (5, 4). Then we substitute for w in equation 2 and solve for
l
Check. If l = 5 and w = 4, the area is 5 · 4, or 20. The l.
perimeter is 2 · 5 + 2 · 4, or 18. The numbers check.  √3 2
2
l + =4
State. The length of the brochure is 5 in. and the width l
is 4 in. 3
l2 + 2 = 4
l
63. Familiarize. We make a drawing of the dog run. Let l =
l4 + 3 = 4l2
the length and w = the width.
l4 − 4l2 + 3 = 0
u2 − 4u + 3 = 0 Letting u = l2
w (u − 3)(u − 1) = 0
u = 3 or u = 1
We now substitute l2 for u and solve for l.
l l2 = 3 or l2 = 1

Since it takes 210 yd of fencing to enclose the run, we know l = ± 3 or l = ±1
that the perimeter is 210 yd. Measurements cannot be negative, so
√ √ we only need
√ to con-
Translate. sider l = 3 and l√ = 1. Since w = 3/l, if l = 3, w = 1
Perimeter: 2l + 2w = 210, or l + w = 105 and if l = 1, w = 3. Length is usually considered √ to be
longer than√width, so we have the solution l = 3 and
Area: lw = 2250
w = 1, or ( 3, 1).
Carry out. We solve the system: √ √ √
Solve the first equation for l: l = 105 − w
Check. √ If2 l =2 3 and w = 1, 2the area is 3 · 1 = 3.
Also ( 3) + 1 = 3 + 1 = 4 = 2 . The numbers check.
Substitute 105 − w for l in the second equation and solve √
State. The length is 3 m, and the width is 1 m.
for w.
(105 − w)w = 2250 67. Familiarize. We let x = the length of a side of one test
105w − w = 2250
2 plot and y = the length of a side of the other plot. Make
a drawing.
0 = w2 − 105w + 2250
0 = (w − 30)(w − 75)
w = 30 or w = 75 x y
If w = 30, then l = 105 − 30, or 75. If w = 75, then
l = 105 − 75, or 30. Since length is usually considered y
to be longer than width, we have the solution l = 75 and x
w = 30, or (75, 30). Area: x2 Area: y 2
Check. If l = 75 and w = 30, the perimeter is 2·75+2·30, Translate.
or 210. The area is 75(30), or 2250. The numbers check. The sum of the areas is 832 ft2 .
 

 

State. The length is 75 yd and the width is 30 yd. ↓ ↓ ↓


65. Familiarize. We first make a drawing. Let l = the length x2 + y 2 = 832
and w = the width. The difference of the areas is 320 ft2 .
 

 

✚ ↓ ↓ ↓

✚ x2 − y 2 = 320
2 ✚ w
✚ Carry out. We solve the system of equations.



l

Copyright © 2013 Pearson Education, Inc.


298 Chapter 7: Conic Sections

x2 + y 2 = 832 y

x2 − y 2 = 320 (⫺2, 4)
5
4
3
2x2 = 1152 Adding 2
1 (1, 1)
2
x = 576 ⫺5 ⫺4 ⫺3 ⫺2 ⫺1 1 2 3 4 5 x
⫺1
x = ±24 ⫺2
⫺3
Since measurements cannot be negative, we consider only ⫺4
⫺5
x = 24. Substitute 24 for x in the first equation and solve
for y. To find the points of intersection of the graphs we solve
242 + y 2 = 832 the system of equations
576 + y 2 = 832 x2 = y,
y 2 = 256 x + y = 2.
y = ±16 The points of intersection are (−2, 4) and (1, 1).
Again, we consider only the positive value, 16. The possi- 79. Graph: x2 + y 2 ≤ 25,
ble solution is (24, 16).
x−y >5
Check. The areas of the test plots are 242 , or 576, and
The solution set of x2 + y 2 ≤ 25 is the circle x2 + y 2 = 25
162 , or 256. The sum of the areas is 576 + 256, or 832.
and the region inside it. The solution set of x − y > 5
The difference of the areas is 576 − 256, or 320. The values
is the half-plane below the line x − y = 5. We shade the
check.
region common to the two solution sets.
State. The lengths of the test plots are 24 ft and 16 ft.
y
69. The correct graph is (b).
4
71. The correct graph is (d). 3
2
1
(5, 0)
73. The correct graph is (a).
⫺4 ⫺3 ⫺2 ⫺1 1 2 3 4 x
⫺1

75. Graph: x2 + y 2 ≤ 16, ⫺2


⫺3
⫺4
y<x
(0, ⫺5)
The solution set of x2 + y 2 ≤ 16 is the circle x2 + y 2 = 16
and the region inside it. The solution set of y < x is To find the points of intersection of the graphs we solve
the half-plane below the line y = x. We shade the region the system of equations
common to the two solution sets. x2 + y 2 = 25,

y
x − y = 5.
5 The points of intersection are (0, −5) and (5, 0).
3 (2兹2, 2兹2) 81. Graph: y ≥ x2 − 3,
2
1
y ≤ 2x
⫺5 ⫺3 ⫺2 ⫺1 1 2 3 5 x
⫺1
⫺2
The solution set of y ≥ x2 − 3 is the parabola y = x2 − 3
(⫺2兹2, ⫺2兹2) ⫺3 and the region inside it. The solution set of y ≤ 2x is the
⫺5 line y = 2x and the half-plane below it. We shade the
region common to the two solution sets.
To find the points of intersection of the graphs we solve
the system of equations y

2 2 7
x + y = 16, 6
(3, 6)
5
y = x. 4
√ √ 3
The √ of intersection are (−2 2, −2 2) and
√ points 2
1
(2 2, 2 2).
⫺5 ⫺4 ⫺3 ⫺2 ⫺1 1 2 3 4 5 x
77. Graph: x2 ≤ y, (⫺1, ⫺2) ⫺2

x+y ≥2
To find the points of intersection of the graphs we solve
The solution set of x2 ≤ y is the parabola x2 = y and the
the system of equations
region inside it. The solution set of x + y ≥ 2 is the line
x + y = 2 and the half-plane above the line. We shade the y = x2 − 3,
region common to the two solution sets. y = 2x.
The points of intersection are (−1, −2) and (3, 6).

Copyright © 2013 Pearson Education, Inc.


Exercise Set 7.4 299

83. Graph: y ≥ x2 , (x − h)2 + (y − k)2 = r2


y <x+2 (2 − 2)2 + (4 − 3)2 = r2
The solution set of y ≥ x2 is the parabola y = x2 and 0 + 1 = r2
the region inside it. The solution set of y < x + 2 is the 1 = r2
half-plane below the line y = x + 2. We shade the region
The equation of the circle is (x − 2)2 + (y − 3)2 = 1.
common to the two solution sets.
91. The equation of the ellipse  is √
of the
 form  
y
x2 y2 3 √ 1
5 + = 1. Substitute 1, and 3, for (x, y)
4 (2, 4) a2 b2 2 2
3 to get two equations.
2
(⫺1, 1) 1  √ 2
3
⫺5 ⫺4 ⫺3 ⫺2 ⫺1 1 2 3 4 5 x
⫺1 12 2 1 3
⫺2 + = 1, or 2 + 2 = 1
⫺3 a2 b2 a 4b
⫺4
⫺5
 2
1
√ 2
( 3) 2 3 1
To find the points of intersection of the graphs we solve + = 1, or 2 + 2 = 1
the system of equations a2 b2 a 4b
1 1
y = x2 , Substitute u for 2 and v for 2 .
a b
y = x + 2. 3
u + v = 1, 4u + 3v = 4,
The points of intersection are (−1, 1) and (2, 4). 4
or
85. 23x = 64 1
3u + v = 1 12u + v = 4
23x = 26 4
1
3x = 6 Solving for u and v, we get u = , v = 1. Then
4
x=2 1 1 1
u = 2 = , so a = 4; v = 2 = 1, so b2 = 1.
2
a 4 b
The solution is 2.
Then the equation of the ellipse is
87. log3 x = 4 x2 y2 x2
+ = 1, or + y 2 = 1.
x = 34 4 1 4
x = 81 93. See the answer section in the text.
The solution is 81. 95. See the answer section in the text.
89. (x − h) + (y − k) = r
2 2 2
97. x3 + y 3 = 72, (1)
If (2, 4) is a point on the circle, then x+y =6 (2)
(2 − h)2 + (4 − k)2 = r2 . Solve equation (2) for y: y = 6 − x
If (3, 3) is a point on the circle, then Substitute for y in equation (1) and solve for x.
(3 − h)2 + (3 − k)2 = r2 . x3 + (6 − x)3 = 72
Thus x3 + 216 − 108x + 18x2 − x3 = 72
(2 − h)2 + (4 − k)2 = (3 − h)2 + (3 − k)2
18x2 − 108x + 144 = 0
4 − 4h + h2 + 16 − 8k + k 2 =
x2 − 6x + 8 = 0
9 − 6h + h2 + 9 − 6k + k 2 1
Multiplying by
−4h − 8k + 20 = −6h − 6k + 18 18
(x − 4)(x − 2) = 0
2h − 2k = −2
x = 4 or x = 2
h − k = −1
If x = 4, then y = 6 − 4 = 2.
If the center (h, k) is on the line 3x−y = 3, then 3h−k = 3.
If x = 2, then y = 6 − 2 = 4.
Solving the system
The pairs (4, 2) and (2, 4) check.
h − k = −1,
3h − k = 3
99. p2 + q 2 = 13, (1)
we find that (h, k) = (2, 3). 1 1
=− (2)
Find r 2 , substituting (2, 3) for (h, k) and (2, 4) for (x, y). pq 6
We could also use (3, 3) for (x, y). Solve equation (2) for p.

Copyright © 2013 Pearson Education, Inc.


300 Chapter 7: Conic Sections

1 p 7. Graph (a) is the graph of y 2 = 9 − x2 .


=−
q 6
9. Graph (g) is the graph of 9y 2 − 4x2 = 36.
6
− =p
q 11. Graph (f) is the graph of 4x2 + y 2 − 16x − 6y = 15.
Substitute −6/q for p in equation (1) and solve for q. (x + 3)2 (y − 1)2
 6 2 13. Graph (c) is the graph of − = 1.
− + q 2 = 13 16 25
q 15. y 2 = −12x
36
+ q 2 = 13 y 2 = 4(−3)x
q2
36 + q 4 = 13q 2 F : (−3, 0), V : (0, 0), D: x = 3

q 4 − 13q 2 + 36 = 0 17. Begin by completing the square twice.


u − 13u + 36 = 0
2
Letting u = q 2 16x2 + 25y 2 − 64x + 50y − 311 = 0
(u − 9)(u − 4) = 0 16(x2 − 4x) + 25(y 2 + 2y) = 311

u=9 or u=4 16(x2 − 4x + 4) + 25(y 2 + 2y + 1) = 311 + 16 · 4 + 25 · 1


16(x − 2)2 + 25(y + 1)2 = 400
x2 = 9 or x2 = 4
(x − 2)2 [y − (−1)]2
x = ±3 or x = ±2 + =1
25 16
Since p = −6/q, if q = 3, p = −2; if q = −3, p = 2; The center is (2, −1). Note that a = 5 and b = 4. The
if q = 2, p = −3; and if q = −2, p = 3. The pairs major axis is horizontal so the vertices are 5 units left and
(−2, 3), (2, −3), (−3, 2), and (3, −2) check. They are the right of the center: (2 − 5, −1) and (2 + 5, −1), or (−3, −1)
solutions. and (7, −1).
√ We know that c = a − b = 25 − 16 = 9
2 2 2

and c = 9 = 3. Then the foci are 3 units left and right


101. Find the points of intersection of y1 = ln x+2 and y2 = x2 . of the center: (2 − 3, −1) and (2 + 3, −1), or (−1, −1) and
They are (1.564, 2.448) and (0.138, 0.019). (5, −1).
103. Find the points of intersection of y1 = ex and y2 = x + 2.
They are (1.146, 3.146) and (−1.841, 0.159).
 
14.5x2 − 64.5 14.5x2 − 64.5
105. Graph y1 = , y2 = − , and
13.5 13.5
y3 = (5.5x − 12.3)/6.3 and find the points of intersection.
They are (2.112, −0.109) and (−13.041, −13.337).

56, 548 − 0.319x2
107. Graph y1 = ,
2688.7
  19. Begin by completing the square twice.
56, 548 − 0.319x2 0.306x2 − 43, 452
y2 = − , y3 = , 1
2688.7 2688.7 x2 − 2y 2 + 4x + y − = 0
 8
0.306x2 − 43, 452  
and y4 = − and find the points of in- 1 1
2688.7 (x2 + 4x) − 2 y 2 − y =
tersection. They are (400, 1.431), (−400, 1.431), 2 8
 
(400, −1.431), and (−400, −1.431). 1 1 1 1
(x2 + 4x + 4) − 2 y 2 − y + = +4−2·
2 16 8 16
 2
Chapter 7 Review Exercises 1
(x + 2)2 − 2 y − =4
4
 2
1. x + y2 = 1 1
y−
y2 = x − 1 [x − (−2)] 2
4
− =1
1 4  2
(y − 0)2 = 4 · (x − 1) 1
4 The center is − 2, . The transverse axis is horizon-
This parabola has a horizontal axis of symmetry, the focus 4
  tal, so the vertices are 2
5 3   units left and
 right
 of the cen-
is , 0 , and the directrix is x = . Thus it opens to the 1 1 1
4 4 ter: − 2 − 2, and − 2 + 2, , or − 4, and
left and the statement is true.   4 4 4
1
0, . Since c2 = a2 + b2 , we have c2 = 4 + 2 = 6 and
3. The statement is true. See page 597 in the text. 4√ √
c = 6.Then the foci  are 6 units left and right of the
5. The statement is false. See Example 4 on page 610 in the √ 1 √ 1
center: − 2 − 6, and − 2 + 6, .
text. 4 4

Copyright © 2013 Pearson Education, Inc.


Chapter 7 Review Exercises 301

Find the asymptotes: y = 0 or y = 1


b b
y − k = (x − h) and y − k = − (x − h) x2 = 0 + 3
a a
√ √ x2 = 3
y− =
1 2 1
(x + 2) and y − = −
2
(x + 2) √
4 2 4 2 x=± 3
21. x2 − 16y = 0, (1)
x2 = 1 + 3
x − y = 64
2 2
(2)
x2 = 4
2
From equation (1) we have x = 16y. Substitute in equa-
x = ±2
tion (2). √ √
16y − y 2 = 64 The pairs ( 3, 0), (− 3, 0), (2, 1), and (−2, 1) check.

0 = y 2 − 16y + 64 29. x2 + y 2 = 100, (1)


0 = (y − 8) 2
2x − 3y = −120
2 2
(2)
0 = y−8 2 2
3x + 3y = 300 Multiplying (1) by 3
8=y 2x2 − 3y 2 = −120
x2 − (8)2 = 64 Substituting in equation (2) 5x2 = 180 Adding
2
x = 128 x2 = 36
√ √ x= ±6
x = ± 128 = ±8 2
√ √ (±6)2 + y 2 = 100
The pairs (−8 2, 8) and (8 2, 8) check.
y 2 = 64
23. x2 − y 2 = 33, (1)
y = ±8
x + y = 11 (2)
The pairs (6, 8), (−6, 8), (6, −8), and (−6, −8) check.
y = −x + 11
31. Familiarize. Let x and y represent the numbers.
x2 − (−x + 11)2 = 33 Substituting in (1)
Translate. The sum of the numbers is 11.
x2 − (x2 − 22x + 121) = 33
x2 − x2 + 22x − 121 = 33 x + y = 11

22x = 154 The sum of the squares of the numbers is 65.

x=7 x2 + y 2 = 65
y = −7 + 11 = 4 Carry out. We solve the system of equations.
The pair (7, 4) checks. x + y = 11, (1)
x2 + y 2 = 65 (2)
25. x2 − y = 3, (1)
First we solve equation (1) for y.
2x − y = 3 (2)
y = 11 − x
From equation (1) we have y = x2 − 3. Substitute in
equation (2). Then substitute 11 − x for y in equation (2) and solve for
x.
2x − (x2 − 3) = 3
x2 + (11 − x)2 = 65
2x − x2 + 3 = 3
x2 + 121 − 22x + x2 = 65
0 = x2 − 2x
2x2 − 22x + 121 = 65
0 = x(x − 2)
2x2 − 22x + 56 = 0
x = 0 or x = 2
x2 − 11x + 28 = 0 Dividing by 2
y = 02 − 3 = −3
(x − 4)(x − 7) = 0
y = 22 − 3 = 1
x − 4 = 0 or x − 7 = 0
The pairs (0, −3) and (2, 1) check.
x = 4 or x=7
27. x2 − y 2 = 3, (1) If x = 4, then y = 11 − 4 = 7.
y = x2 − 3 (2) If x = 7, then y = 11 − 7 = 4.
From equation (2) we have x2 = y + 3. Substitute in In either case, the possible numbers are 4 and 7.
equation (1).
Check. 4 + 7 = 11 and 42 + 72 = 16 + 49 = 65. The
y + 3 − y2 = 3 answer checks.
0 = y2 − y State. The numbers are 4 and 7.
0 = y(y − 1)

Copyright © 2013 Pearson Education, Inc.


302 Chapter 7: Conic Sections

33. Familiarize. Let x and y represent the positive integers. Since the length of a radius cannot be negative, we consider
Translate. The sum of the numbers is 12. only x = 11. Substitute 11 for x in equation (1) and solve
for y.
x + y = 12
π · 112 + πy 2 = 130π
3
The sum of the reciprocals is . 121π + πy 2 = 130π
8
1 1 3 πy 2 = 9π
+ =
x y 8 y2 = 9
Carry out. We solve the system of equations. y = ±3
x + y = 12, (1) Again, we consider only the positive solution.
1 1 3
+ = (2) Check. If the radii are 11 ft and 3 ft, the sum of the areas
x y 8 is π · 112 + π · 32 = 121π + 9π = 130π ft2 . The difference
First solve equation (1) for y. of the areas is 121π − 9π = 112π ft2 . The answer checks.
y = 12 − x State. The radius of the larger circle is 11 ft, and the
Then substitute 12 − x for y in equation (2) and solve for radius of the smaller circle is 3 ft.
x. 37. Graph: x2 + y 2 ≤ 16,
1 1 3
+ = , LCD is 8x(12 − x) x+y <4
x 12 − x 8
  The solution set of x2 + y 2 ≤ 16 is the circle x2 + y 2 = 16
1 1 3
8x(12 − x) + = 8x(12 − x) · and the region inside it. The solution set of x + y < 4
x 12 − x 8
is the half-plane below the line x + y = 4. We shade the
8(12 − x) + 8x = x(12 − x) · 3 region common to the two solution sets.
96 − 8x + 8x = 36x − 3x2
y
96 = 36x − 3x2
5
(0, 4)
3x2 − 36x + 96 = 0 3
x2 − 12x + 32 = 0 Dividing by 3 2
1
(4, 0)
(x − 4)(x − 8) = 0 ⫺5 ⫺3 ⫺2 ⫺1
⫺1
1 2 3 5 x
⫺2
x − 4 = 0 or x − 8 = 0 ⫺3

x = 4 or x=8 ⫺5

If x = 4, y = 12 − 4 = 8.
39. Graph: x2 + y 2 ≤ 9,
If x = 8, y = 12 − 8 = 4.
x ≤ −1
In either case, the possible numbers are 4 and 8.
1 1 2 1 3 The solution set of x2 + y 2 ≤ 9 is the circle x2 + y 2 = 9
Check. 4 + 8 = 12; + = + = . The answer
4 8 8 8 8 and the region inside it. The solution set of x ≤ −1 is the
checks. line x = −1 and the half-plane to the left of it. We shade
State. The numbers are 4 and 8. the region common to the two solution sets.

y
35. Familiarize. Let x = the radius of the larger circle and
5
let y = the radius of the smaller circle. We will use the 4
formula for the area of a circle, A = πr2 . (⫺1, 2兹2)
2
Translate. The sum of the areas is 130π ft2 . 1
⫺5 ⫺4 ⫺2 ⫺1 1 2 4 5 x
πx2 + πy 2 = 130π ⫺1
⫺2

The difference of the areas is 112π ft2 . (⫺1, ⫺2兹2)


⫺4

πx2 − πy 2 = 112π ⫺5

We have a system of equations.


41. A straight line can intersect an ellipse at 0 points, 1 point,
πx2 + πy 2 = 130π, (1) or 2 points but not at 4 points, so answer D is correct.
πx2 − πy 2 = 112π (2)
43. Familiarize. Let x and y represent the numbers.
Carry out. We add.
Translate. The product of the numbers is 4.
πx2 + πy 2 = 130π
xy = 4
πx2 − πy 2 = 112π 65
2πx2 = 242π The sum of the reciprocals is .
56
x2 = 121 Dividing by 2π 1 1 65
x= ±11 + =
x y 56

Copyright © 2013 Pearson Education, Inc.


Chapter 7 Test 303

Carry out. We solve the system of equations. 49. No, the asymptotes of a hyperbola are not part of the
xy = 4, (1) graph of the hyperbola. The coordinates of points on the
1 1 65 asymptotes do not satisfy the equation of the hyperbola.
+ = (2)
x y 56
First solve equation (1) for y. Chapter 7 Test
4
y=
x
4 1. Graph (c) is the graph of 4x2 − y 2 = 4.
Then substitute for y in equation (2) and solve for x.
x 2. Graph (b) is the graph of x2 − 2x − 3y = 5.
1 1 65
+ =
x 4/x 56 3. Graph (a) is the graph of x2 + 4x + y 2 − 2y − 4 = 0.
1 x 65
+ = 4. Graph (d) is the graph of 9x2 + 4y 2 = 36.
x 4 56
 
1 x 65 5. x2 = 12y
56x + = 56x ·
x 4 56 x2 = 4 · 3y
2
56 + 14x = 65x V : (0, 0), F : (0, 3), D : y = −3
14x2 − 65x + 56 = 0
y
(2x − 7)(7x − 8) = 0
4
2x − 7 = 0 or 7x − 8 = 0
2
2x = 7 or 7x = 8
7 8 ⫺4 ⫺2 2 4 x
x= or x= ⫺2
2 7
7 4 2 8 ⫺4
x 2 ⫽ 12y
If x = , y = =4· = .
2 7/2 7 7
8 4 7 7
If x = , y = =4· = . 6. y 2 + 2y − 8x − 7 = 0
7 8/7 8 2
7 8 y 2 + 2y = 8x + 7
In either case the possible numbers are and . 2
2 7 y + 2y + 1 = 8x + 7 + 1
7 8 1 1 2 7 16 49 65 (y + 1)2 = 8x + 8
Check. · = 4; + = + = + = .
2 7 7/2 8/7 7 8 56 56 56
[y − (−1)]2 = 4(2)[x − (−1)]
The answer checks.
7 8 V : (−1, −1)
State. The numbers are and .
2 7 F : (−1 + 2, −1) or (1, −1)
45. The vertices are (0, −3) and (0, 3), so the center is (0, 0), D : x = −1 − 2 = −3
and the major axis is vertical.
x2 y2 y
2
+ 2 =1
a 3
 √  4
1 3 3
Substitute − , and solve for a. 2
2 2
 2  √ 2 ⫺4 ⫺2 2 4 x
1 3 3 ⫺2

2 2 ⫺4
+ =1
a2 32
1 3 y 2 ⫹ 2y ⫺ 8x ⫺ 7 ⫽ 0
+ =1
4a2 4
1 + 3a2 = 4a2 Multiplying by 4a2 7. (x − h)2 = 4p(y − k)
1 = a2 (x − 0)2 = 4 · 2(y − 0)
1=a x2 = 8y
y2 8. Begin by completing the square twice.
The equation of the ellipse is x2 + = 1.
9 x2 + y 2 + 2x − 6y − 15 = 0
47. The equation of a circle can be written as x2 + 2x + y 2 − 6y = 15
(x − h)2 (y − k)2
+ =1 (x2 + 2x + 1) + (y 2 − 6y + 9) = 15 + 1 + 9
a2 b2
where a = b = r, the radius of the circle. In an ellipse, (x + 1)2 + (y − 3)2 = 25
a > b, so a circle is not a special type of ellipse. [x − (−1)]2 + (y − 3)2 = 52

Copyright © 2013 Pearson Education, Inc.


304 Chapter 7: Conic Sections

Center: (−1, 3), radius: 5 11. The vertices (0, −5) and (0, 5) are on the y-axis, so the
major axis is vertical and a = 5. Since the vertices are
equidistant from the center, the center of the ellipse is at
4
the origin. The length of the minor axis is 4, so b = = 2.
2
x2 y2
The equation is + = 1.
4 25
12. 4x2 − y 2 = 4
x2 y2
− =1
1 4
x2 y2
2
− 2 =1
1 2
9. 9x2 + 16y 2 = 144 The center is (0, 0); a = 1 and b = 2.
x2 y2
+ =1 The transverse axis is horizontal, so the vertices are (−1, 0)
16 9 (1, 0). Since c2 = a2 + b2 ,√we have c2 √
and √ = 1 + 4 = 5 and
2 2
x y c = 5. Then the foci are (− 5, 0) and ( 5, 0).
+ 2 =1
42 3 Find the asymptotes:
a = 4, b = 3
b b
The center is (0, 0). The major axis is horizontal, so the y = x and y = − x
a a
√ 0). Since c = a − b√, we have
2 2 2
vertices are (−4, 0) and (4, 2 2
c√= 16 − 9 = 7, so c = 7 and the foci are (− 7, 0) and
2 y = x and y = − x
1 1
( 7, 0).
y = 2x and y = −2x

⫺4 ⫺2 2 4 x
⫺2

⫺4

4x 2 ⫺ y 2 ⫽ 4

10. (x + 1)2 (y − 2)2


+ =1 (y − 2)2 (x + 1)2
4 9 13. − =1
4 9
[x − (−1)]2 (y − 2)2
+ =1 (y − 2)2 [x − (−1)]2
2 2 32
2
− =1
The center is (−1, 2). Note that a = 3 and b = 2. The 2 32
major axis is vertical, so the vertices are 3 units below and The center is (−1, 2); a = 2 and b = 3.
above the center: The transverse axis is vertical, so the vertices are 2 units
(−1, 2 − 3) and (−1, 2 + 3) or (−1, −1) and (−1, 5). below and above the center:

We know that c2 =√a2 − b2 , so c2 = 9 − 4 = 5 and c = 5. (−1, 2 − 2) and (−1, 2 + 2) or (−1, 0) and (−1, 4).

Then the foci are 5 units below and above the center: Since c2 = a2 + b2 √
, we have c2 = 4 + 9 = 13 and c = 13.
√ √
(−1, 2 − 5) and (−1, 2 + 5). Then the foci are 13 units below and above the center:
√ √
(−1, 2 − 13) and (−1, 2 + 13).
Find the asymptotes:
a a
y − k = (x − h) and y − k = − (x − h)
b b
2 2
y − 2 = (x − (−1)) and y − 2 = − (x − (−1))
3 3
2 2
y − 2 = (x + 1) and y − 2 = − (x + 1)
3 3
2 8 2 4
y = x+ and y= − x+
3 3 3 3

Copyright © 2013 Pearson Education, Inc.


Chapter 7 Test 305

y Then substitute 1 − x for y in equation (1) and solve for


4 x.
2
x2 + (1 − x)2 = 13
x2 + 1 − 2x + x2 = 13
4 2 2 4 x
2 2x2 − 2x − 12 = 0
4 2(x2 − x − 6) = 0
2(x − 3)(x + 2) = 0
(y  2) 2 (x  1) 2
4  9  1 x = 3 or x = −2
If x = 3, y = 1 − 3 = −2. If x = −2, y = 1 − (−2) = 3.
14. 2y 2 − x2 = 18
The pairs (3, −2) and (−2, 3) check.
y2 x2
− =1 18. x + y = 5, (1)
9 18
y2 x2 xy = 6 (2)
2
− √ =1
3 (3 2)2 First solve equation (1) for y.

h = 0, k = 0, a = 3, b = 3 2 y = −x + 5
a a Then substitute −x + 5 for y in equation (2) and solve for
y − k = (x − h) and y − k = − (x − h)
b b x.
3 3
y − 0 = √ (x − 0) and y − 0 = − √ (x − 0) x(−x + 5) = 6
3 2 3 2
√ √ −x2 + 5x − 6 = 0
2 2
y= x and y=− x −1(x2 − 5x + 6) = 0
2 2
2 −1(x − 2)(x − 3) = 0
15. The parabola
 is ofthe form y = 4px. A point on the
18 x = 2 or x = 3
parabola is 6, , or (6, 9).
2 If x = 2, y = −2 + 5 = 3. If x = 3, y = −3 + 5 = 2.
y 2 = 4px The pairs (2, 3) and (3, 2) check.
92 = 4 · p · 6
19. Familiarize. Let l and w represent the length and width
81 = 24p of the rectangle, in feet, respectively.
27 Translate. The perimeter is 18 ft.
=p
8
  2l + 2w = 18 (1)
27 27
Since the focus is at (p, 0) = , 0 , the focus is in. From the Pythagorean theorem, we have
8 8 √
from the vertex. l2 + w2 = ( 41)2 (2)
16. 2x2 − 3y 2 = −10, (1) Carry out. We solve the system of equations. We first
2 2 solve equation (1) for w.
x + 2y = 9 (2)
2l + 2w = 18
2x − 3y = −10
2 2
2w = 18 − 2l
−2x2 − 4y 2 = −18 Multiplying (2) by −2
w = 9−l
− 7y 2 = −28 Adding
Then substitute 9 − l for w in equation (2) and solve for l.
y2 = 4 √
y = ±2 l2 + (9 − l)2 = ( 41)2
l2 + 81 − 18l + l2 = 41
x2 + 2(±2)2 = 9 Substituting into (2)
2l2 − 18l + 40 = 0
x2 + 8 = 9
2(l2 − 9l + 20) = 0
x2 = 1
2(l − 4)(l − 5) = 0
x = ±1
l = 4 or l = 5
The pairs (1, 2), (1, −2), (−1, 2) and (−1, −2) check.
If l = 4, then w = 9 − 4 = 5. If l = 5, then w = 9 − 5 = 4.
17. x2 + y 2 = 13, (1) Since length is usually considered to be longer than width,
x+y =1 (2) we have l = 5 and w = 4.
First solve equation (2) for y. Check. The perimeter
√ is 2 · 5√+ 2 · 4, or 18 ft. The length
of a diagonal is 52 + 42 , or 41 ft. The solution checks.
y =1−x
State. The dimensions of the garden are 5 ft by 4 ft.

Copyright © 2013 Pearson Education, Inc.


306 Chapter 7: Conic Sections

20. Familiarize. Let l and w represent the length and width 23. Use the midpoint formula to find the center.
of the playground, in feet, respectively.  
1 + 5 1 + (−3)
(h, k) = , = (3, −1)
Translate. 2 2
Perimeter: 2l + 2w = 210 (1) Use the distance formula to find the radius.
1 1 √
Area: lw = 2700 (2) r= (1 − 5)2 + (1−(−3))2 = (−4)2 + (4)2 = 2 2
2 2
Carry out. We solve the system of equations. First solve Write the equation of the circle.
equation (2) for w.
(x − h)2 + (y − k)2 = r2
2700 √
w=
l (x − 3)2 + [y − (−1)]2 = (2 2)2
2700 (x − 3)2 + (y + 1)2 = 8
Then substitute for w in equation (1) and solve for
l
l.
2700
2l + 2 · = 210
l
5400
2l + = 210
l
2l2 + 5400 = 210l Multiplying by l
2l2 − 210l + 5400 = 0
2(l2 − 105l + 2700) = 0
2(l − 45)(l − 60) = 0
l = 45 or l = 60
2700 2700
If l = 45, then w = = 60. If l = 60, then w = =
45 60
45. Since length is usually considered to be longer than
width, we have l = 60 and w = 45.
Check. Perimeter: 2 · 60 + 2 · 45 = 210 ft
Area: 60 · 45 = 2700 ft2
The solution checks.
State. The dimensions of the playground are 60 ft by
45 ft.
21. Graph: y ≥ x2 − 4,
y < 2x − 1
The solution set of y ≥ x2 − 4 is the parabola y = x2 − 4
and the region inside it. The solution set of y < 2x − 1
is the half-plane below the line y = 2x − 1. We shade the
region common to the two solution sets.

y
5 (3, 5)
4
3
2
1
5 4 3 1 1 3 4 5 x
1

(1, 3) 3

5

To find the points of intersection of the graphs of the re-


lated equations we solve the system of equations
y = x2 − 4,
y = 2x − 1.
The points of intersection are (−1, −3) and (3, 5).

22. (y−1)2 = 4(x+1) represents a parabola with vertex (−1, 1)


that opens to the right. Thus the correct answer is A.

Copyright © 2013 Pearson Education, Inc.


Chapter 8
Sequences, Series, and Combinatorics
7. an = (−1)n n2
Exercise Set 8.1 a1 = (−1)1 12 = −1,
1. an = 4n − 1 a2 = (−1)2 22 = 4,

a1 = 4 · 1 − 1 = 3, a3 = (−1)3 32 = −9,

a2 = 4 · 2 − 1 = 7, a4 = (−1)4 42 = 16;

a3 = 4 · 3 − 1 = 11, a10 = (−1)10 102 = 100;

a4 = 4 · 4 − 1 = 15; a15 = (−1)15 152 = −225

a10 = 4 · 10 − 1 = 39; (−2)n+1


9. an = 5 +
a15 = 4 · 15 − 1 = 59 2n

n (−2)1+1 4
3. an = ,n≥2 a1 = 5 + = 5 + = 7,
n−1 21 2
The first 4 terms are a2 , a3 , a4 , and a5 : (−2)2+1 −8
a2 = 5 + =5+ = 3,
2 22 4
a2 = = 2,
2−1 (−2)3+1 16
a3 = 5 + =5+ = 7,
3 3 23 8
a3 = = ,
3−1 2 (−2)4+1 −32
4 4 a4 = 5 + =5+ = 3;
a4 = = , 24 16
4−1 3 (−2) 10+1
−1 · 211
a10 = 5+ = 5 + = 3;
5 5 210 210
a5 = = ;
5−1 4 (−2)15+1 216
10 10 a15 = 5 + 15
= 5 + 15 = 7
a10 = = ; 2 2
10 − 1 9
11. an = 5n − 6
15 15
a15 =
15 − 1
=
14 a8 = 5 · 8 − 6 = 40 − 6 = 34

n2 − 1 13. an = (2n + 3)2


5. an = ,
n2 + 1 a6 = (2 · 6 + 3)2 = 225
12 − 1 15. an = 5n2 (4n − 100)
a1 = = 0,
12 + 1 a11 = 5(11)2 (4 · 11 − 100) = 5(121)(−56) =
2 −1
2
3 −33, 880
a2 = = ,
22 + 1 5
17. an = ln en
32 − 1 8 4
a3 = 2 = = , a67 = ln e67 = 67
3 +1 10 5
19. n Un
42 − 1 15
a4 = 2 = ; 1 2
4 +1 17
102 − 1 99 2 2.25
a10 = = ;
102 + 1 101 3 2.3704
152 − 1 224 112 4 2.4414
a15 = = =
152 + 1 226 113 5 2.4883
6 2.5216 3
7 2.5465
8 2.5658
9 2.5812
10 2.5937 0 10
0

Copyright © 2013 Pearson Education, Inc.


308 Chapter 8: Sequences, Series, and Combinatorics


6
21. 39. 2i = 20 + 21 + 22 + 23 + 24 + 25 + 26
n Un
i=0
1 2 = 1 + 2 + 4 + 8 + 16 + 32 + 64
2 1.5538 = 127
3 1.4998

10
4 1.4914 41. ln k = ln 7 + ln 8 + ln 9 + ln 10 =
k=7
5 1.4904
ln(7 · 8 · 9 · 10) = ln 5040 ≈ 8.5252
6 1.4902 3

8
k 1 2 3 4
7 1.4902 43. = + + + +
8 1.4902 k+1 1+1 2+1 3+1 4+1
k=1
5 6 7 8
9 1.4902 + + +
5+1 6+1 7+1 8+1
10 1.4902 0 10 1 2 3 4 5 6 7 8
0 = + + + + + + +
2 3 4 5 6 7 8 9
23. 2, 4, 6, 8, 10,. . . 15, 551
=
2520
These are the even integers, so the general term might be
 5
2n. 45. (−1)i

25. −2, 6, −18, 54, . . . i=1


= (−1)1 + (−1)2 + (−1)3 + (−1)4 + (−1)5
We can see a pattern if we write the sequence as = −1 + 1 − 1 + 1 − 1
−1 · 2 · 1, 1 · 2 · 3, −1 · 2 · 9, 1 · 2 · 27, . . . = −1
The general term might be (−1) 2(3) n n−1
. 
8
47. (−1)k+1 3k
2 3 4 5 6 k=1
27. , , , , , . . .
3 4 5 6 7 = (−1)2 3 · 1 + (−1)3 3 · 2 + (−1)4 3 · 3+
These are fractions in which the denominator is 1 greater (−1)5 3 · 4 + (−1)6 3 · 5 + (−1)7 3 · 6+
than the numerator. Also, each numerator is 1 greater
than the preceding numerator. The general term might be (−1)8 3 · 7 + (−1)9 3 · 8
n+1
. = 3 − 6 + 9 − 12 + 15 − 18 + 21 − 24
n+2 = −12
29. 1 · 2, 2 · 3, 3 · 4, 4 · 5, . . . 
6
2 2 2 2 2
These are the products of pairs of consecutive natural num- 49. = 2 + + + +
k 2 +1 0 + 1 12 + 1 22 + 1 32 + 1
bers. The general term might be n(n + 1). k=0
2 2 2
+ +
31. 0, log 10, log 100, log 1000, . . . 42 + 1 52 + 1 62 + 1
We can see a pattern if we write the sequence as 2 2 2 2 2
= 2+1+ + + + +
log 1, log 10, log 100, log 1000, . . . 5 10 17 26 37
2 1 2 1 2
The general term might be log 10n−1 . This is equivalent = 2+1+ + + + +
to n − 1. 5 5 17 13 37
157, 351
33. 1, 2, 3, 4, 5, 6, 7, . . . =
40, 885
S3 = 1 + 2 + 3 = 6  5

S7 = 1 + 2 + 3 + 4 + 5 + 6 + 7 = 28 51. (k 2 − 2k + 3)
k=0
35. 2, 4, 6, 8, . . . = (02 − 2 · 0 + 3) + (12 − 2 · 1 + 3)+
S4 = 2 + 4 + 6 + 8 = 20 (22 − 2 · 2 + 3) + (32 − 2 · 3 + 3)+
S5 = 2 + 4 + 6 + 8 + 10 = 30 (42 − 2 · 4 + 3) + (52 − 2 · 5 + 3)
5 = 3 + 2 + 3 + 6 + 11 + 18
1 1 1 1 1 1
37. = + + + + = 43
2k 2·1 2·2 2·3 2·4 2·5
k=1
1 1 1 1 1
= + + + +
2 4 6 8 10
60 30 20 15 12
= + + + +
120 120 120 120 120
137
=
120

Copyright © 2013 Pearson Education, Inc.


Exercise Set 8.1 309


10
2i
53. 1 4 4 9
2i + 1 a3 = 1 + =1+ = 1 , or
i=0 5 5 5 5
0
2 21 22 23 24 4
= + + + + +
20 +1 21 +1 22 +1 23 +1 24 +1 1 5 5 14
a4 = 1 + = 1 + = 1 , or
2 5
2 2 6
2 7
2 8 9 9 9 9 9
+ + + + + 5
25 +1 26 + 1 27 + 1 28 + 1 29 + 1 √
210
67. a1 = 6561, ak+1 = (−1)k ak

210 + 1 a2 = (−1)1 6561 = −81

1 2 4 8 16 32 64 128 a3 = (−1)2 −81 = 9i
= + + + + + + + + √ √
2 3 5 9 17 33 65 129 a4 = (−1)3 9i = −3 i
256 512 1024
+ + 69. a1 = 2, ak+1 = ak + ak−1
257 513 1025
≈ 9.736 a2 = 3
a3 = 3 + 2 = 5
55. 5 + 10 + 15 + 20 + 25+ . . .
a4 = 5 + 3 = 8
This is a sum of multiples of 5, and it is an infinite series.
Sigma notation is 71. a) a1 = $1000(1.062)1 = $1062
∞
5k. a2 = $1000(1.062)2 ≈ $1127.84
k=1 a3 = $1000(1.062)3 ≈ $1197.77
57. 2 − 4 + 8 − 16 + 32 − 64 a4 = $1000(1.062)4 ≈ $1272.03
This is a sum of powers of 2 with alternating signs. Sigma a5 = $1000(1.062)5 ≈ $1350.90
notation is a6 = $1000(1.062)6 ≈ $1434.65
6 
6
(−1)k+1 2k, or (−1)k−1 2k a7 = $1000(1.062)7 ≈ $1523.60
k=1 k=1 a8 = $1000(1.062)8 ≈ $1618.07
1 2 3 4 5 6 a9 = $1000(1.062)9 ≈ $1718.39
59. − + − + − +
2 3 4 5 6 7 a10 = $1000(1.062)10 ≈ $1824.93
This is a sum of fractions in which the denominator is
b) a20 = $1000(1.062)20 ≈ $3330.35
one greater than the numerator. Also, each numerator
is 1 greater than the preceding numerator and the signs 73. Find each term by adding $1.10 to the preceding term:
alternate. Sigma notation is
$9.80, $10.90, $12, $13.10, $14.20, $15.30, $16.40, $17.50,

6
k $18.60, $19.70
k
(−1) .
k+1
k=1 75. a1 = 1 (Given)
61. 4 − 9 + 16 − 25 + . . . + (−1) n n 2 a2 = 1 (Given)
k 2
This is a sum of terms of the form (−1) k , beginning with a3 = a2 + a1 = 1 + 1 = 2
k = 2 and continuing through k = n. Sigma notation is a4 = a3 + a2 = 2 + 1 = 3
n
a5 = a4 + a3 = 3 + 2 = 5
(−1)k k 2 .
k=2 a6 = a5 + a4 = 5 + 3 = 8
1 1 1 1 a7 = a6 + a5 = 8 + 5 = 13
63. + + + +. . .
1·2 2·3 3·4 4·5 77. a) an = −659.8950216n2 + 6297.77684n + 54, 737.29091
This is a sum of fractions in which the numerator is 1 and
the denominator is a product of two consecutive integers. b) In 2010, n = 2010 − 2001 = 9.
The larger integer in each product is the smaller integer a9 ≈ 57, 966 thousand, 57,966,000 vehicles
in the succeeding product. It is an infinite series. Sigma In 2011, n = 2011 − 2001 = 10.
notation is
∞ a10 ≈ 51, 726 thousand, or 51,726,000 vehicles
1
k(k + 1)
. In 2012, n = 2012 − 2001 = 11.
k=1
a11 ≈ 44, 166 thousand, or 44,166,000 vehicles
1
65. a1 = 4, ak+1 = 1 + 79. Familiarize. Let x, y and z represent the number
ak
1 1 5 of patent applications in the United States, Japan, and
a2 = 1 + = 1 , or China, respectively.
4 4 4

Copyright © 2013 Pearson Education, Inc.


310 Chapter 8: Sequences, Series, and Combinatorics

Translate. The total number of applications was 89,348,


so we have one equation. Exercise Set 8.2
x + y + z = 89, 348
The number of applications in China was 3741 less than 1. 3, 8, 13, 18, . . .
one-half the number of applications in Japan, so we have a1 = 3
a second equation. d=5 (8 − 3 = 5, 13 − 8 = 5, 18 − 13 = 5)
1
z = y − 3741. 3. 9, 5, 1, −3, . . .
2
The number of applications in the United States was a1 = 9
32,518 more than the number of applications in China, d = −4 (5−9 = −4, 1−5 = −4, −3−1 = −4)
so we have a third equation.
3 9 15
x = z + 32, 518 5. , , 3, ,. . .
2 4 4
We have a system of equations. 3
a1 =
x + y + z = 89, 348, 2
 
1 3 9 3 3 9 3
z = y − 3741, d= − = ,3 − =
2 4 4 2 4 4 4
x = z + 32, 518.
7. a1 = $316
Carry out. Solving the system of equations, we get
(44, 855, 32, 156, 12, 337). d = −$3 ($313 − $316 = −$3,
$310 − $313 = −$3, $307 − $310 = −$3)
Check. The total number of applications was
44, 855 + 32, 156 + 12, 337 = 89, 348. The number of 9. 2, 6, 10, . . .
applications in Japan was 32,156. We see that a1 = 2, d = 4, and n = 12
1 an = a1 + (n − 1)d
· 32, 156 − 3741 = 16, 078 − 3741 = 12, 337, which was
2
a12 = 2 + (12 − 1)4 = 2 + 11 · 4 = 2 + 44 = 46
the number of applications in China. We also see that
32,518 more than this number is 12, 337+32, 518, or 44,855, 7 5
which was the number of applications in the United States. 11. 3, , ,. . .
3 3
The answer checks. 2
a1 = 3, d = − , and n = 14
State. The number of patent applications in the United 3
States, Japan, and China was 44,855, 32,156, and 12,337, an = a1 + (n − 1)d
respectively.  2 26 17
a14 = 3 + (14 − 1) − =3− =−
81. We complete the square twice. 3 3 3
x2 + y 2 + 5x − 8y = 2 13. $2345.78, $2967.54, $3589.30, . . .
x2 + 5x + y 2 − 8y = 2 a1 = $2345.78, d = $621.76, and n = 10
25 25 an = a1 + (n − 1)d
x2 + 5x + + y 2 − 8y + 16 = 2+ + 16
4 4 a10 = $2345.78 + (10 − 1)($621.76) = $7941.62
 2
5 97 15. 106 = 2 + (n − 1)(4)
x+ + (y − 4)2 =
2 4
  2 √ 2 106 = 2 + 4n − 4
5 97
x− − + (y − 4)2 = 108 = 4n
2 2
  √ 27 = n
5 97
The center is − , 4 and the radius is . The 27th term is 106.
2 2
83. an = in 2
17. a1 = 3, d = −
3
a1 = i
an = a1 + (n − 1)d
a2 = i2 = −1
Let an = −27, and solve for n.
a3 = i3 = −i  2
a4 = i4 = 1 −27 = 3 + (n − 1) −
3
a5 = i5 = i4 · i = i; −81 = 9 + (n − 1)(−2)
S5 = i − 1 − i + 1 + i = i −81 = 9 − 2n + 2
85. Sn = ln 1 + ln 2 + ln 3 + · · · + ln n −92 = −2n
= ln(1 · 2 · 3 · · · n) 46 = n
The 46th term is −27.

Copyright © 2013 Pearson Education, Inc.


Exercise Set 8.2 311

19. an = a1 + (n − 1)d Then


n
33 = a1 + (8 − 1)4 Substituting 33 for a8 , Sn = (a1 + an )
2
8 for n, and 4 for d
33 = a1 + 28 20
S20 = (2 + 97)
2
5 = a1
= 10(99) = 990.
(Note that this procedure is equivalent to subtracting d
from a8 seven times to get a1 : 33 − 7(4) = 33 − 28 = 5) 
40
33. (2k + 3)
21. an = a1 + (n − 1)d k=1

−507 = 25 + (n − 1)(−14) Write a few terms of the sum:


−507 = 25 − 14n + 14 5 + 7 + 9+ . . . +83
−546 = −14n This is a series coming from an arithmetic sequence with
39 = n a1 = 5, n = 40, and a40 = 83. Then
n
Sn = (a1 + an )
25 95 2
23. + 15d =
3 6 40
45 S40 = (5 + 83)
15d = 2
6
= 20(88) = 1760
1
d=
2 
19
k−3
25  1  25 1
35.
4
a1 = − 16 = −8= k=0
3 2 3 3
Write a few terms of the sum:
1 5 4 11 7
The first five terms of the sequence are , , , , . 3 1 1 1
3 6 3 6 3 − − − + 0 + + . . . +4
4 2 4 4
25. 5 + 8 + 11 + 14 + . . . Since k goes from 0 through 19, there are 20 terms. Thus,
Note that a1 = 5, d = 3, and n = 20. First we find a20 : this is equivalent to a series coming from an arithmetic
3
an = a1 + (n − 1)d sequence with a1 = − , n = 20, and a20 = 4. Then
4
a20 = 5 + (20 − 1)3 n
Sn = (a1 + an )
= 5 + 19 · 3 = 62 2
 
Then 20 3
S20 = − +4
n 2 4
Sn = (a1 + an )
2 13 65
= 10 · = .
20 4 2
S20 = (5 + 62)
2

57
7 − 4k
= 10(67) = 670. 37.
13
k=12
27. The sum is 2 + 4 + 6+ . . . +798 + 800. This is the sum of
the arithmetic sequence for which a1 = 2, an = 800, and Write a few terms of the sum:
41 45 49 221
n = 400. − − − −. . . −
n 13 13 13 13
Sn = (a1 + an ) Since k goes from 12 through 57, there are 46 terms. Thus,
2
400 this is equivalent to a series coming from an arithmetic
S400 = (2 + 800) = 200(802) = 160, 400 41 221
2 sequence with a1 = − , n = 46, and a46 = − . Then
13 13
29. The sum is 7 + 14 + 21 + . . . + 91 + 98. This is the sum n
Sn = (a1 + an )
of the arithmetic sequence for which a1 = 7, an = 98, and 2
n = 14.  
46 41 221
n S46 = − −
Sn = (a1 + an ) 2 13 13
2  
14 262 6026
S14 = (7 + 98) = 7(105) = 735 = 23 − =− .
2 13 13
31. First we find a20 : 39. Familiarize. We have a sequence $5000, $6125, $7250, . . ..
an = a1 + (n − 1)d It is an arithmetic sequence with a1 = 5000, d = 1125, and
n = 25.
a20 = 2 + (20 − 1)5 n
Translate. We want to find Sn = (a1 + an ) where
= 2 + 19 · 5 = 97 2
an = a1 + (n − 1)d, a1 = 5000, d = 1125, and n = 25.

Copyright © 2013 Pearson Education, Inc.


312 Chapter 8: Sequences, Series, and Combinatorics

 
Carry out. First we find a25 . 2 1 3 12
 
a25 = 5000 + (25 − 1)1125 = 5000 + 27, 000 = 32, 000  1 −3 −2 −1 
25 25 5 2 −4 −4
Then S25 = (5000 + 32, 000) = · 37, 000 = 462, 500.
2 2
Check. We can do the calculations again, or we can do Next we interchange the first two rows.
the entire addition:  
1 −3 −2 −1
5000 + 6125 + 7250 + . . . + 32, 000.  
 2 1 3 12 
The answer checks. 5 2 −4 −4
State. The total amount received from the investment
was $462,500. Now multiply the first row by −2 and add it to the second
row. Also multiply the first row by −5 and add it to the
41. Familiarize. We have arithmetic sequence with a1 = 28, third row.
d = 4, and n = 20.  
n 1 −3 −2 −1
Translate. We want to find Sn = (a1 + an ) where an =  
2  0 7 7 14 
a1 + (n − 1)d, a1 = 28, d = 4, and n = 20.
0 17 6 1
Carry out. First we find a20 .
a20 = 28 + (20 − 1)4 = 104 1
Multiply the second row by .
20 7
Then S20 = (28 + 104) = 10 · 132 = 1320.  
2 1 −3 −2 −1
Check. We can do the calculations again, or we can do  
 0 1 1 2 
the entire addition:
0 17 6 1
28 + 32 + 36+ . . . 104.
The answer checks. Multiply the second row by 3 and add it to the first row.
Also multiply the second row by −17 and add it to the
State. There are 1320 seats in the first balcony.
third row.
43. Yes; d = 48 − 16 = 80 − 48 = 112 − 80 = 144 − 112 = 32.  
1 0 1 5
a10 = 16 + (10 − 1)32 = 304  
 0 1 1 2 
10
S10 = (16 + 304) = 1600 ft 0 0 −11 −33
2
45. Familiarize. We have a sequence 10, 12, 14, . . .. It is an 1
Multiply the third row by − .
arithmetic sequence with a1 = 10, d = 2, and n = 8. 11
n
Translate. We want to find Sn = (a1 + an ), where an =  
2 1 0 1 5
a1 + (n − 1)d, a1 = 10, d = 2, and n = 8.  
 0 1 1 2 
Carry out. First we find a8 . 0 0 1 3
a8 = 10 + (8 − 1)(2) = 10 + 7 · 2 = 24
Multiply the third row by −1 and add it to the first row
8
Then S8 = (10 + 24) = 4 · 34 = 136 and also to the second row.
2  
Check. We can do the calculations again, or we can do 1 0 0 2
the entire addition:  
 0 1 0 −1 
10 + 12 + 14 + . . . + 24. 0 0 1 3
The answer checks.
Now we can read the solution from the matrix. It is
State. There are 24 marchers in the last row, and there (2, −1, 3).
are 136 marchers altogether.
51. The vertices are on the y-axis, so the transverse axis is
47. Yes; d = 6 − 3 = 9 − 6 = 3n − 3(n − 1) = 3. vertical and a = 5. The length of the minor axis is 4, so
49. 2x + y + 3z = 12 b = 4/2 = 2. The equation is
x − 3y − 2z = −1 x2 y2
+ = 1.
4 25
5x + 2y − 4z = −4
We will use Gauss-Jordan elimination with matrices. First n
53. Sn = (1 + 2n − 1) = n2
we write the augmented matrix. 2

Copyright © 2013 Pearson Education, Inc.


Exercise Set 8.3 313

55. Let d = the common difference. Then a4 = a2 + 2d, or 5a 5a2 5a3


a a a
10p + q = 40 − 3q + 2d 9. 2 = , 4 = , 8 =
5 2 5a 2 5a 2
10p + 4q − 40 = 2d 2 4
5p + 2q − 20 = d. r=
a
Also, a1 = a2 − d, so we have 2
a1 = 40 − 3q − (5p + 2q − 20) 11. 2, 4, 8, 16, . . .
4
= 40 − 3q − 5p − 2q + 20 a1 = 2, n = 7, and r = , or 2.
2
= 60 − 5p − 5q. We use the formula an = a1 rn−1 .
57. 4, m1 , m2 , m3 , m4 , 13 a7 = 2(2)7−1 = 2 · 26 = 2 · 64 = 128
We look for m1 , m2 , m3 , and m4 such that 4, m1 , m2 , √
13. 2, 2 3, 6, . . .
m3 , m4 , 13 is an arithmetic sequence. In this case a1 = 4, √
n = 6, and a6 = 13. First we find d. 2 3 √
a1 = 2, n = 9, and r = , or 3
an = a1 + (n − 1)d 2
an = a1 rn−1
13 = 4 + (6 − 1)d √ √
a9 = 2( 3)9−1 = 2( 3)8 = 2 · 81 = 162
9 = 5d
4 7 7
1 =d 15. ,− ,. . .
5 625 25
Then we have 7
7 −
4 4 a1 = , n = 23, and r = 25 = −25.
m1 = a1 + d = 4 + 1 = 5 , 625 7
5 5
4 4 8 3 625
m2 = m1 + d = 5 + 1 = 6 = 7 , an = a1 rn−1
5 5 5 5
7 7
3 4 7 2 a23 = (−25)23−1 = (−25)22
m3 = m2 + d = 7 + 1 = 8 = 9 , 625 625
5 5 5 5
7
2 4 6 1 = 2 · 252 · 2520 = 7(25)20 , or 7(5)40
m4 = m3 + d = 9 + 1 = 10 = 11 . 25
5 5 5 5
17. 1, 3, 9, . . .
3
Exercise Set 8.3 a1 = 1 and r = , or 3
1
an = a1 rn−1
1. 2, 4, 8, 16, . . .
an = 1(3)n−1 = 3n−1
4 8 16
= 2, = 2, =2
2 4 8 19. 1, −1, 1, −1, . . .
r=2 −1
a1 = 1 and r = = −1
3. 1, −1, 1, −1, . . . 1
an = a1 rn−1
−1 1 −1
= −1, = −1, = −1 an = 1(−1)n−1 = (−1)n−1
1 −1 1
r = −1 1 1 1
21. , , ,. . .
2 4 8 16 x x2 x3
5. , − , , − , . . . 1
3 3 3 3
4 8 16 1 x2 1
− − a1 = and r =
x 1
=
x
3 = −2, 3 = −2, 3 = −2
2 4 8 x
− an = a1 rn−1
3 3 3
r = −2 1  1 n−1 1 1 1 1
an = = · n−1 = 1+n−1 = n
0.6275 0.06275 x x x x x x
7. = 0.1, = 0.1
6.275 0.6275
r = 0.1

Copyright © 2013 Pearson Education, Inc.


314 Chapter 8: Sequences, Series, and Combinatorics


11  k
23. 6 + 12 + 24 + . . . 2
12 41. 15
a1 = 6, n = 7, and r = , or 2 3
k=1
6 
2 2 2
a1 (1 − r ) or 10; |r| =   = , n = 11
n
a1 = 15 ·
Sn = 3 3 3
1−r   11   
6(1 − 27 ) 6(1 − 128) 6(−127) 2 2048
S7 = = = = 762 10 1 − 10 1 −
1−2 −1 −1 3 177, 147
S11 = =
2 1
1 1 1 1−
25. − + −. . . 3 3
18 6 2 175, 099
1 = 10 · ·3
− 177, 147
1 1 18
a1 = , n = 9, and r = 6 = − · = −3 1, 750, 990 38, 569
18 1 6 1 = , or 29
59, 049 59, 049
18
a1 (1 − rn ) ∞  k−1
Sn =  1
1−r 43.
1  1 k=1
2
1−(−3)9 (1 + 19, 683)  
18 18 1 1
S9 =
1 − (−3)
=
4 a1 = 1, |r| =   =
2 2
1
(19, 684) 1  1  4921 a1 1 1
18 S∞ = = = =2
= (19, 684) = 1−r 1 1
4 18 4 18 1−
√ 2 2
27. Multiplying each term of the sequence by − 2 produces ∞

the next term, so it is true that the sequence is geometric. 45. 12.5k
k=1
2n+1
29. Since n = 2, the sequence has a common ratio so it is
2 Since |r| = 12.5 > 1, the sum does not exist.
true that the sequence is geometric. ∞

31. Since | − 0.75| < 1, it is true that the series has a sum. 47. $500(1.11)−k
k=1
33. 4+2+1+. . . $500 1
2 1 1 a1 = $500(1.11)−1 , or ; |r| = |1.11−1 | =
    1.11 1.11
|r| =   =   = , and since |r| < 1, the series
4 2 2 $500 $500
does have a sum. a1 1.11
S∞ = = = 1.11 ≈ $4545.45
a1 4 4 2 1−r 1 0.11
S∞ = = = =4· =8 1−
1−r 1 1 1 1.11 1.11
1− ∞
2 2 
49. 16(0.1)k−1
35. 25 + 20 + 16 + . . .
 20   4  4 k=1
   
|r| =   =   = , and since |r| < 1, the a1 = 16, |r| = |0.1| = 0.1
25 5 5
a1 16 16 160
series does have a sum. S∞ = = = =
1−r 1 − 0.1 0.9 9
a1 25 25 5
S∞ = = = = 25 · = 125
1−r 4 1 1 51. 0.131313 . . . = 0.13 + 0.0013 + 0.000013+ . . .
1−
5 5 This is an infinite geometric series with a1 = 0.13.
 
37. 8 + 40 + 200 + . . .  0.0013 
 40  
|r| =   = |0.01| = 0.01 < 1, so the series has a limit.
 
|r| =   = |5| = 5, and since |r| > 1 the series does not 0.13 
8 a1 0.13 0.13 13
have a sum. S∞ = = = =
1−r 1 − 0.01 0.99 99
39. 0.6 + 0.06 + 0.006+ . . .
  53. We will find fraction notation for 0.9999 and then add 8.
 0.06 

|r| =   = |0.1| = 0.1, and since |r| < 1, the series does
0.6  0.9999 = 0.9 + 0.09 + 0.009 + 0.0009+. . .
have a sum. This is an infinite geometric series with a1 = 0.9.
 
a1 0.6 0.6 6 2  0.09 
S∞ = = = = = 
|r| =   = |0.1| = 0.1 < 1, so the series has a limit.
1−r 1 − 0.1 0.9 9 3 0.9 
a1 0.9 0.9
S∞ = = = =1
1−r 1 − 0.1 0.9
Then 8.9999 = 8 + 1 = 9.

Copyright © 2013 Pearson Education, Inc.


Exercise Set 8.3 315

55. 3.4125125 = 3.4 + 0.0125125 $3200[1 − (1.046)10 ]


S10 = .
We will find fraction notation for 0.0125125 and then add 1 − 1.046
34 17 Carry out. We carry out the computation and get S10 ≈
3.4, or , or . $39, 505.71.
10 5
0.0125125 = 0.0125 + 0.0000125+ . . . Check. Repeat the calculations.
This is an infinite geometric series with a1 = 0.0125. State. The amount of the annuity is $39,505.71.
 
 0.0000125  63. Familiarize. We have a sequence 0.01, 2(0.01), 22 (0.01),

|r| =   = |0.001| = 0.001 < 1, so the series has
0.0125  23 (0.01), . . . . The thickness after 20 folds is given by the
a limit. 21st term of the sequence.
a1 0.0125 0.0125 125
S∞ = = = = Translate. Using the formula
1−r 1 − 0.001 0.999 9990
17 125 33, 966 125 34, 091 an = a1 rn−1 ,
Then + = + =
5 9990 9990 9990 9990 where a1 = 0.01, r = 2, and n = 21, we have
57. Familiarize. The total earnings are represented by the a21 = 0.01(2)21−1 .
geometric series Carry out. We carry out the computation and get
$0.01 + $0.01(2) + $0.01(2)2 + . . . + $0.01(2)27 , where 10,485.76.
a1 = $0.01, r = 2, and n = 28. Check. Repeat the calculations.
Translate. Using the formula State. The result is 10,485.76 in. thick.
a1 (1 − rn )
Sn = 65. We use the formula
1−r  nN 
we have i
P 1+ −1
$0.01(1 − 228 ) n
S28 = . V =
1−2 i/n
Carry out. We carry out the computation and get with P = 300, i = 5.1%, or 0.051, n = 4, and N = 12.
$2,684,354.55.  4·12 
0.051
300 1 + −1
Check. Repeat the calculation. 4
V =
State. You would earn $2,684,354.55. 0.051/4
V ≈ $19, 694.01
59. a) Familiarize. The rebound distances form a geo-
metric sequence: The amount of the annuity is $19,694.01.

0.6 × 200, (0.6)2 × 200, (0.6)3 × 200, . . . , 67. We first use the formula
a1
or 120, 0.6 × 120, (0.6)2 × 120, . . . S∞ = ,
1−r
The total rebound distance after 9 rebounds is the where a1 = 30% of 5,000,000 or 0.3(5,000,000), and r =
sum of the first 9 terms of this sequence. 30%, or 0.3.
Translate. We will use the formula 0.3(5, 000, 000)
a1 (1 − rn ) S∞ = ≈ 2, 142, 857
Sn = with a1 = 120, r = 0.6, and n = 9. 1 − 0.3
1−r In all, the advertising campaign will reach about 2,142,857
Carry out. people.
120[1 − (0.6)9 ] 2, 142, 857
S9 = ≈ 297 Since ≈ 0.429, or 42.9%, we see that about
1 − 0.6 5, 000, 000
Check. We repeat the calculation. 42.9% of the population will buy the product.
State. The bungee jumper has traveled about 297 ft 69. f (x) = x2 , g(x) = 4x + 5
upward after 9 rebounds. (f ◦ g)(x) = f (g(x)) = f (4x + 5) = (4x + 5)2 =
a1 120 16x2 + 40x + 25
b) S∞ = = = 300 ft
1−r 1 − 0.6
(g ◦ f )(x) = g(f (x)) = g(x2 ) = 4x2 + 5
61. Familiarize. The amount of the annuity is the geometric
series 71. 5x = 35
$3200 + $3200(1.046) + $3200(1.046)2 + . . . + ln 5x = ln 35
$3200(1.046)9 , where a1 = $3200, r = 1.046, and n = 10. x ln 5 = ln 35
Translate. Using the formula ln 35
x=
a1 (1 − rn ) ln 5
Sn = x ≈ 2.209
1−r
we have 73. See the answer section in the text.

Copyright © 2013 Pearson Education, Inc.


316 Chapter 8: Sequences, Series, and Combinatorics

75. a) If the sequence is arithmetic, then a2 − a1 = a3 − a2 . 25. Familiarize. Let x, y, and z represent the amounts in-
x + 7 − (x + 3) = 4x − 2 − (x + 7) vested at 1.5%, 2%, and 3%, respectively.
13 Translate. We know that simple interest for one year was
x= $104. This gives us one equation:
3
13 22 0.015x + 0.02y + 0.03z = 104
The three given terms are +3= ,
3 3 The amount invested at 2% is twice the amount invested
13 34 13 46
+7= , and 4 · −2= . at 1.5%:
3 3 3 3
12 y = 2x, or −2x + y = 0
Then d = , or 4, so the fourth term is
3 There is $400 more invested at 3% than at 2%:
46 12 58 z = y + 400, or −y + z = 400
+ = .
3 3 3
We have a system of equations:
b) If the sequence is geometric, then a2 /a1 = a3 /a2 .
0.015x + 0.02y + 0.03z = 104,
x+7 4x − 2
x+3
=
x+7 −2x + y = 0
11 − y+ z = 400
x=− or x = 5
3 Carry out. Solving the system of equations, we get
11 (800, 1600, 2000).
For x = − : The three given terms are
3 Check. Simple interest for one year would be
11 2 11 10 0.015($800)+0.02($1600)+0.03($2000), or $12+$32+$60,
− +3=− , − +7= , and
3 3 3 3 or $104. The amount invested at 2%, $1600, is twice $800,
 
11 50 the amount invested at 1.5%. The amount invested at 3%,
4 − −2=− . $2000, is $400 more than $1600, the amount invested at
3 3
Then r = −5, so the fourth term is 2%. The answer checks.
50 250 State. Martin invested $800 at 1.5%, $1600 at 2%, and
− (−5) = . $2000 at 3%.
3 3
For x = 5: The three given terms are 5 + 3 = 8, 27. See the answer section in the text.
3
5 + 7 = 12, and 4 · 5 − 2 = 18. Then r = , so the
2 29. See the answer section in the text.
3
fourth term is 18 · = 27.
2 Chapter 8 Mid-Chapter Mixed Review
77. x2 − x3 + x4 − x5 + . . .
This is a geometric series with a1 = x2 and r = −x. 1. All of the terms of a sequence with general term an = n
a1 (1 − rn ) x2 (1 − (−x)n ) x2 (1 − (−x)n ) are positive. Since the given sequence has negative terms,
Sn = = =
1−r 1 − (−x) 1+x the given statement is false.
3. a2 /a1 = 7/3; a3 /a2 = 3/ − 1 = −3; since 7/3
= −3, the
Exercise Set 8.4 sequence is not geometric. The given statement is false.
5. an = 3n + 5
1. n2 < n3 a1 = 3 · 1 + 5 = 8,
12 < 13 , 22 < 23 , 32 < 33 , 42 < 43 , 52 < 53 a2 = 3 · 2 + 5 = 11,
The first statement is false, and the others are true. a3 = 3 · 3 + 5 = 14,
n(n − 3) a4 = 3 · 4 + 5 = 17;
3. A polygon of n sides has diagonals.
2 a9 = 3 · 9 + 5 = 32;
3(3 − 3)
A polygon of 3 sides has diagonals. a14 = 3 · 14 + 5 = 47
2
4(4 − 3) 7. 3, 6, 9, 12, 15, . . .
A polygon of 4 sides has diagonals.
2 These are multiples of 3, so the general term could be 3n.
5(5 − 3)
A polygon of 5 sides has diagonals. 1 1 1 7 15
2 9. S4 = 1 + + + = 1 , or
2 4 8 8 8
6(6 − 3)
A polygon of 6 sides has diagonals. 11. −4 + 8 − 12 + 16 − 20 + . . .
2
7(7 − 3) This is an infinite sum of multiples of 4 with alternating
A polygon of 7 sides has diagonals. signs. Sigma notation is
2
∞
Each of these statements is true.
(−1)k 4k.
5. - 23. See the answer section in the text. k=1

Copyright © 2013 Pearson Education, Inc.


Exercise Set 8.5 317

13. 7 − 12 = −5; 2 − 7 = −5; −3 − 2 = −5


The common difference is −5. Exercise Set 8.5
15. In Exercise 14 we found that d = 2. 1. 6 P6 = 6! = 6 · 5 · 4 · 3 · 2 · 1 = 720
an = a1 + (n − 1)d
44 = 4 + (n − 1)2 3. Using formula (1), we have

44 = 4 + 2n − 2 10 P7 = 10 · 9 · 8 · 7 · 6 · 5 · 4 = 604, 800.
44 = 2 + 2n Using formula (2), we have
10! 10! 10 · 9 · 8 · 7 · 6 · 5 · 4 · 3!
42 = 2n 10 P7 = = = =
(10 − 7)! 3! 3!
21 = n
604, 800.
The 21st term is 44.
−8 1 4 1 −2 1 5. 5! = 5 · 4 · 3 · 2 · 1 = 120
17. =− ; =− ; =−
−16 2 −8 2 4 2 7. 0! is defined to be 1.
1 9 · 8 · 7 · 6 · 5!
The common ratio is − . 9.
9!
= = 9 · 8 · 7 · 6 = 3024
2 5! 5!
   
 4   1 1

19. |r| =   = −  = < 1, so the series has a sum.
 11. (8 − 3)! = 5! = 5 · 4 · 3 · 2 · 1 = 120
−8   2  2
10! 10 · 9 · 8 · 7! 10 · 3 · 3 · 4 · 2
a1 −8 −8 2 16 13. = = =
S∞ = =  = = −8 · = − 7!3! 7!3 · 2 · 1 3·2·1
1−r 1 3 3 3
1− − 10 · 3 · 4 = 120
2 2
15. Using formula (2), we have
21. Familiarize. The number of plants is represented by the
arithmetic series 36 + 30 + 24 + . . . with a1 = 36, 8! 8!
8 P0 = = = 1.
d = 30 − 36 = −6, and n = 6. (8 − 0)! 8!
n 17. Using a calculator, we find
Translate. We want to find Sn = (a1 + an ) where
2 52 P4 = 6, 497, 400
an = a1 + (n − 1)d.
Carry out. 19. Using formula (1), we have n P3 = n(n − 1)(n − 2).
a6 = 36 + (6 − 1)(−6) = 36 + 5(−6) = 36 − 30 = 6 Using formula (2), we have
6 n! n(n − 1)(n − 2)(n − 3)!
S6 = (36 + 6) = 3 · 42 = 126 n P3 = = =
2 (n − 3)! (n − 3)!
Check. We can do the calculations again or we can do n(n − 1)(n − 2).
the entire addition 36 + 30 + 24 + 18 + 12 + 6. The answer
checks. 21. Using formula (1), we have n P1 = n.
State. In all, there will be 126 plants. Using formula (2), we have
n! n(n − 1)!
23. See the answer section in the text. n P1 = = = n.
(n − 1)! (n − 1)!
25. 1 + 2 + 3 + . . . + 100 23. 6 P6 = 6! = 720
= (1 + 100) + (2 + 99) + (3 + 98) + . . .+
25. 9 P9 = 9! = 362, 880
(50 + 51)
= 101 + 101 + 101 + . . . + 101 27. 9 P4 = 9 · 8 · 7 · 6 = 3024
  
50 addends of 101 29. Without repetition: 5 P5 = 5! = 120
= 50 · 101 With repetition: 55 = 3125
= 5050 31. There are 5 P5 choices for the order of the rock numbers
A formula for the first n natural numbers is and 4 P4 choices for the order of the speeches, so we have
n 5 P5 ·4 P4 = 5!4! = 2880.
(1 + n).
2
33. The first number can be any of the eight digits other than
27. We can prove an infinite sequence of statements Sn by 0 and 1. The remaining 6 numbers can each be any of the
showing that a basis statement S1 is true and then that ten digits 0 through 9. We have
for all natural numbers k, if Sk is true, then Sk+1 is true.
8 · 106 = 8, 000, 000
Accordingly, there can be 8,000,000 telephone numbers
within a given area code before the area needs to be split
with a new area code.

Copyright © 2013 Pearson Education, Inc.


318 Chapter 8: Sequences, Series, and Combinatorics

35. a2 b3 c4 = a · a · b · b · b · c · c · c · c 47. n P4 = 8 ·n−1 P3


There are 2 a’s, 3 b’s, and 4 c’s, for a total of 9. We have n! (n − 1)!
= 8·
9! (n − 4)! (n − 1 − 3)!
2! · 3! · 4! n! (n − 1)!
= 8·
9 · 8 · 7 · 6 · 5 · 4! 9·8·7·6·5 (n − 4)! (n − 4)!
=
2 · 1 · 3 · 2 · 1 · 4!
=
2·3·2
= 1260. n! = 8 · (n − 1)! Multiplying by (n−4)!

37. a) 6 P5 = 6 · 5 · 4 · 3 · 2 = 720 n(n − 1)! = 8 · (n − 1)!


n=8 Dividing by (n − 1)!
b) 65 = 7776
c) The first letter can only be D. The other four letters 49. n P4 = 8 ·n P3

are chosen from A, B, C, E, F without repetition. n! n!


= 8·
We have (n − 4)! (n − 3)!
1 ·5 P4 = 1 · 5 · 4 · 3 · 2 = 120. (n − 3)! = 8(n − 4)! Multiplying by
d) The first letter can only be D. The second letter (n − 4)!(n − 3)!
can only be E. The other three letters are chosen n!
(n − 3)(n − 4)! = 8(n − 4)!
from A, B, C, F without repetition. We have
n−3 = 8 Dividing by (n − 4)!
1 · 1 ·4 P3 = 1 · 1 · 4 · 3 · 2 = 24.
n = 11
39. a) Since repetition is allowed, each of the 5 digits can
be chosen in 10 ways. The number of zip-codes pos- 51. There is one losing team per game. In order to leave one
sible is 10 · 10 · 10 · 10 · 10, or 100,000. tournament winner there must be n − 1 losers produced in
n − 1 games.
b) Since there are 100,000 possible zip-codes, there
could be 100,000 post offices.
Exercise Set 8.6
41. a) Since repetition is allowed, each digit can be chosen
in 10 ways. There can be 13!
10 · 10 · 10 · 10 · 10 · 10 · 10 · 10 · 10, or 1. 13 C2 =
2!(13 − 2)!
1,000,000,000 social security numbers.
13! 13 · 12 · 11!
b) Since more than 303 million social security numbers = =
2!11! 2 · 1 · 11!
are possible, each person can have a social security
13 · 12 13 · 6 · 2
number. = =
2·1 2·1
43. x2 + x − 6 = 0 = 78
(x + 3)(x − 2) = 0  13 
3. 13!
=
x+3 = 0 or x − 2 = 0 11 11!(13 − 11)!
x = −3 or x=2 13!
=
The solutions are −3 and 2. 11!2!
= 78 (See Exercise 1.)
45. f (x) = x3 − 4x2 − 7x + 10
7 7!
We use synthetic division to find one factor of the polyno- 5. =
mial. We try x − 1.
1 1!(7 − 1)!
 7 · 6!
1  1 −4
7!
−7 10 = =
1 −3 −10 1!6! 1 · 6!
1 −3 −10 0 =7
x3 − 4x2 − 7x + 10 = 0 5 P3 5·4·3
7. =
(x − 1)(x − 3x − 10) = 0
2 3! 3!
5·4·3 5·2·2·3
(x − 1)(x − 5)(x + 2) = 0 = =
3·2·1 3·2·1
x − 1 = 0 or x − 5 = 0 or x + 2 = 0 = 5 · 2 = 10
x = 1 or x = 5 or x = −2 6 6!
The solutions are −2, 1, and 5. 9. =
0 0!(6 − 0)!
6! 6!
= =
0!6! 6! · 1
=1

Copyright © 2013 Pearson Education, Inc.


Exercise Set 8.6 319
   
  6·5 10 5
6 27. 10 C7 ·5 C3 = · Using the fundamen-
11. = = 15 7 3 tal counting principle
2 2·1
    10! 5!
n n = ·
13. = , so
r n−r 7!(10 − 7)! 3!(5 − 3)!
 7   7   7   7   7  10 · 9 · 8 · 7! 5 · 4 · 3!
+ + + + + = ·
0 1 2 3 4 7! · 3! 3! · 2!
 7   7
  7  10 · 9 · 8 5 · 4
+ + = · = 120 · 10 = 1200
5 6 7 3·2·1 2·1
  
 
7 7 7   7  29. 52 C5 = 2, 598, 960
=2 + + +
0 1 2 3 31. a)
  31 P2 = 930
7! 7! 7! 7! b) 31 · 31 = 961
=2 + + +
7!0! 6!1! 5!2! 4!3!
c) 31 C2 = 465
= 2(1 + 7 + 21 + 35) = 2 · 64 = 128
33. 2x2 − x = 3
15. We will use form (1). 2x − x − 3 = 0
2

52! (2x − 3)(x + 1) = 0


52 C4 =
4!(52 − 4)!
2x − 3 = 0 or x + 1 = 0
52 · 51 · 50 · 49 · 48!
= 2x = 3 or x = −1
4 · 3 · 2 · 1 · 48!
3
52 · 51 · 50 · 49 x= or x = −1
= 2
4·3·2·1
3
= 270, 725 The solutions are and −1.
2
17. We will use form (2). 35. x3 + 3x2 − 10x = 24
 27 
x3 + 3x2 − 10x − 24 = 0
11 We use synthetic division to find one factor of the polyno-
27 · 26 · 25 · 24 · 23 · 22 · 21 · 20 · 19 · 18 · 17 mial on the left side of the equation. We try x − 3.
= 
11 · 10 · 9 · 8 · 7 · 6 · 5 · 4 · 3 · 2 · 1 3  1 3 −10 −24
= 13, 037, 895 3 18 24
 n  n! n(n − 1)!
1 6 8 0
19. = = =n Now we have:
1 1!(n − 1)! 1!(n − 1)!
(x − 3)(x2 + 6x + 8) = 0
 m  m! m!
21. = = =1 (x − 3)(x + 2)(x + 4) = 0
m m!(m − m)! m!0!
x − 3 = 0 or x + 2 = 0 or x + 4 = 0
23!
23. 23 C4 = x = 3 or x = −2 or x = −4
4!(23 − 4)!
23! 23 · 22 · 21 · 20 · 19! The solutions are −4, −2, and 3.
= =
4!19! 4 · 3 · 2 · 1 · 19! 37. There are 13 diamonds, and we choose 5. We have 13 C5 =
23 · 22 · 21 · 20 23 · 2 · 11 · 3 · 7 · 4 · 5 1287.
= =
4·3·2·1 4·3·2·1
39. Playing once: n C2
= 8855
Playing twice: 2 ·n C2
13!
25. 13 C10 =
10!(13 − 10)!
13! 13 · 12 · 11 · 10!
= =
10!3! 10! · 3 · 2 · 1
13 · 12 · 11 13 · 3 · 2 · 2 · 11
= =
3·2·1 3·2·1
= 286

Copyright © 2013 Pearson Education, Inc.


320 Chapter 8: Sequences, Series, and Combinatorics
 
n
41. =6 Combination notation method:
n−2
(x − 3)5
n!
=6      
5 5 5
(n − (n − 2))!(n − 2)! = x5 + x4 (−3) + x3 (−3)2 +
0 1 2
n!  5   5   5 
=6
2!(n − 2)! x2 (−3)3 + x(−3)4 + (−3)5
3 4 5
n(n − 1)(n − 2)!
=6 5! 5 5! 4 5! 3
2 · 1 · (n − 2)! = x + x (−3) + x (9)+
0!5! 1!4! 2!3!
n(n − 1)
=6 5! 2 5! 5!
2 x (−27) + x(81) + (−243)
3!2! 4!1! 5!0!
n(n − 1) = 12
= x5 − 15x4 + 90x3 − 270x2 + 405x − 243
n2 − n = 12
n − n − 12 = 0
2 5. Expand: (x − y)5 .
We have a = x, b = −y, and n = 5.
(n − 4)(n + 3) = 0
n = 4 or n = −3 Pascal’s triangle method: We use the sixth row of Pascal’s
triangle.
Only 4 checks. The solution is 4.
1 5 10 10 5 1
(x − y)5
Exercise Set 8.7 = 1 · x5 + 5x4 (−y) + 10x3 (−y)2 + 10x2 (−y)3 +
5x(−y)4 + 1 · (−y)5
1. Expand: (x + 5)4 . = x5 − 5x4 y + 10x3 y 2 − 10x2 y 3 + 5xy 4 − y 5
We have a = x, b = 5, and n = 4. Combination notation method:
Pascal’s triangle method: Use the fifth row of Pascal’s (x − y)5
triangle.      
5 5 5
1 4 6 4 1 = x5 + x4 (−y) + x3 (−y)2 +
0 1 2
(x + 5)4      
5 5 5
= 1 · x4 + 4 · x3 · 5 + 6 · x2 · 52 + x2 (−y)3 + x(−y)4 + (−y)5
3 4 5
4 · x · 53 + 1 · 54 5! 5 5! 4 5! 3 2
4 3 2 = x + x (−y) + x (y )+
= x + 20x + 150x + 500x + 625 0!5! 1!4! 2!3!
Combination notation method: 5! 2 5! 5!
x (−y 3 ) + x(y 4 ) + (−y 5 )
(x + 5)4 3!2! 4!1! 5!0!
 4   4   4  = x5 − 5x4 y + 10x3 y 2 − 10x2 y 3 + 5xy 4 − y 5
= x4 + x3 · 5 + x2 · 52 +
0 1 2 7. Expand: (5x + 4y)6 .
 4   4 
x · 53 + 54 We have a = 5x, b = 4y, and n = 6.
3 4
Pascal’s triangle method: Use the seventh row of Pascal’s
4! 4 4! 3 4! 2 2 triangle.
= x + x ·5+ x ·5 +
0!4! 1!3! 2!2! 1 6 15 20 15 6 1
4! 4! 4
x · 53 + 5 (5x + 4y)6
3!1! 4!0!
= 1 · (5x)6 + 6 · (5x)5 (4y) + 15(5x)4 (4y)2 +
= x4 + 20x3 + 150x2 + 500x + 625
20(5x)3 (4y)3 + 15(5x)2 (4y)4 + 6(5x)(4y)5 +
3. Expand: (x − 3)5 . 1 · (4y)6
We have a = x, b = −3, and n = 5. = 15, 625x6 + 75, 000x5 y + 150, 000x4 y 2 +
Pascal’s triangle method: Use the sixth row of Pascal’s 160, 000x3 y 3 +96, 000x2 y 4 +30, 720xy 5 +4096y 6
triangle.
Combination notation method:
1 5 10 10 5 1 (5x + 4y)6
(x − 3)5  6   6 
= (5x)6 + (5x)5 (4y)+
= 1 · x5 + 5x4 (−3) + 10x3 (−3)2 + 10x2 (−3)3 + 0 1
5x(−3)4 + 1 · (−3)5  6   6 
(5x)4 (4y)2 + (5x)3 (4y)3 +
= x5 − 15x4 + 90x3 − 270x2 + 405x − 243 2 3
 6   6   6 
(5x)2 (4y)4 + (5x)(4y)5 + (4y)6
4 5 6

Copyright © 2013 Pearson Education, Inc.


Exercise Set 8.7 321

   
6! 6! 7! 7! 1 7! 1
= (15, 625x6 ) + (3125x5 )(4y)+ = (128t7 )+ (64t6 ) + (32t5 ) 2 +
0!6! 1!5! 0!7! 1!6! t 2!5! t
   
6! 6! 7! 1 7! 1
(625x4 )(16y 2 ) + (125x3 )(64y 3 )+ 4
(16t ) 3 + 3
(8t ) 4 +
2!4! 3!3! 3!4! t 4!3! t
6! 6!      
(25x2 )(256y 4 ) + (5x)(1024y 5 )+ 7! 1 7! 1 7! 1
4!2! 5!1! (4t2 ) 5 + (2t) 6 +
5!2! t 6!1! t 7!0! t7
6! = 128t7 + 448t5 + 672t3 + 560t + 280t−1 +
(4096y 6 )
6!0!
84t−3 + 14t−5 + t−7
= 15, 625x6 + 75, 000x5 y + 150, 000x4 y 2 +
11. Expand: (x2 − 1)5 .
160, 000x3 y 3 + 96, 000x2 y 4 + 30, 720xy 5 +
4096y 6 We have a = x2 , b = −1, and n = 5.
 7 Pascal’s triangle method: Use the sixth row of Pascal’s
1 triangle.
9. Expand: 2t + .
t 1 5 10 10 5 1
1 (x2 − 1)5
We have a = 2t, b = , and n = 7.
t
= 1 · (x2 )5 + 5(x2 )4 (−1) + 10(x2 )3 (−1)2 +
Pascal’s triangle method: Use the eighth row of Pascal’s
triangle. 10(x2 )2 (−1)3 + 5(x2 )(−1)4 + 1 · (−1)5
1 7 21 35 35 21 7 1 = x10 − 5x8 + 10x6 − 10x4 + 5x2 − 1
 7 Combination notation method:
1
2t + (x2 − 1)5
t
   2  5   5 
1 1 = (x2 )5 + (x2 )4 (−1)+
= 1 · (2t)7 + 7(2t)6 + 21(2t)5 + 0 1
t t  5   5 
 3  4  5 (x2 )3 (−1)2 + (x2 )2 (−1)3 +
1 1 1 2 3
35(2t)4 + 35(2t)3 + 21(2t)2 +
t t t  5   5 
 6  7 (x2 )(−1)4 + (−1)5
1 1 4 5
7(2t) +1·
t t 5! 10 5! 8 5! 6
1 1 = (x ) + (x )(−1) + (x )(1)+
= 128t7 + 7 · 64t6 · + 21 · 32t5 · 2 + 0!5! 1!4! 2!3!
t t 5! 4 5! 2 5!
1 1 1 (x )(−1) + (x )(1) + (−1)
35 · 16t4 · 3 + 35 · 8t3 · 4 + 21 · 4t2 · 5 + 3!2! 4!1! 5!0!
t t t = x − 5x + 10x − 10x + 5x − 1
10 8 6 4 2

1
7 · 2t · 6 + 7
1 √
t t 13. Expand: ( 5 + t)6 .

= 128t7 + 448t5 + 672t3 + 560t + 280t−1 + We have a = 5, b = t, and n = 6.
84t−3 + 14t−5 + t−7 Pascal’s triangle method: We use the seventh row of Pas-
Combination notation method: cal’s triangle:
 7 1 6 15 20 15 6 1
2t +
1 √ √ √
t ( 5 + t)6 = 1 · ( 5)6 + 6( 5)5 (t)+
  √ √
 7   7  1 15( 5)4 (t2 ) + 20( 5)3 (t3 )+
= (2t)7 + (2t)6 + √ √
0 1 t 15( 5)2 (t4 ) + 6 5t5 + 1 · t6
 7   2   3 √ √
1 7  1 = 125 + 150 5 t + 375t2 + 100 5 t3 +
(2t)5 + (2t)4 + √
2 t 3 t 75t4 + 6 5 t5 + t6
 7   4    5
1 7 1 Combination notation method:
(2t)3 + (2t)2 +    
4 t 5 t √ 6 √ 6 6 √ 5
 6   7 ( 5 + t)6 = ( 5) + ( 5) (t)+
 7  1 7  1
0 1
(2t) +    
6 t 7 t 6 √ 4 2 6 √ 3 3
( 5) (t ) + ( 5) (t )+
2 3
   
6 √ 2 4 6 √
( 5) (t ) + ( 5)(t5 )+
4 5

Copyright © 2013 Pearson Education, Inc.


322 Chapter 8: Sequences, Series, and Combinatorics

 
6 = a9 − 9(2a7 ) + 36(4a5 ) − 84(8a3 ) + 126(16a)−
(t6 )
6 126(32a−1 ) + 84(64a−3 ) − 36(128a−5 )+
6! 6! √ 6!
= (125) + (25 5)t + (25)(t2 )+ 9(256a−7 ) − 512a−9
0!6! 1!5! 2!4! = a9 − 18a7 + 144a5 − 672a3 + 2016a − 4032a−1 +
6! √ 6! 5376a−3 − 4608a−5 + 2304a−7 − 512a−9
(5 5)(t3 ) + (5)(t4 )+
3!3! 4!2! √ √
6! √ 6! 6 17. ( 2 + 1)6 − ( 2 − 1)6
( 5)(t5 ) + (t ) √
5!1! 6!0! First, expand ( 2 + 1)6 .
√ √  6 √  6 √
= 125 + 150 5 t + 375t2 + 100 5 t3 + √
√ ( 2+1)6 = ( 2)6 + ( 2)5 (1)+
75t4 + 6 5 t5 + t6 0 1
 6 √  6 √
 2 9 ( 2)4 (1)2 + ( 2)3 (1)3 +
15. Expand: a− . 2 3
a  6 √  6 √
2 ( 2)2 (1)4 + ( 2)(1)5 +
We have a = a, b = − , and n = 9. 4 5
a
Pascal’s triangle method: Use the tenth row of Pascal’s  6 
triangle. (1)6
6
1 9 36 84 126 126 84 36 9 1 6! 6! √ 6!
 2 9  2  2 2 = ·8+ ·4 2+ · 4+
a− = 1 · a9 + 9a8 − + 36a7 − + 6!0! 5!1! 4!2!
a a a 6! √ 6! 6! √ 6!
 2 3  2 4 ·2 2+ ·2+ · 2+
84a6 − + 126a5 − + 3!3! 2!4! 1!5! 0!6!
a a √ √ √
 2 5  2 6 = 8 + 24 2 + 60 + 40 2 + 30 + 6 2 + 1

126a4 − + 84a3 − + = 99 + 70 2
a a √
 2 7  2 8  2 9 Next, expand ( 2 − 1)6 .
36a2 − +9a − +1 · − √
a a a ( 2 − 1)6
= a9 − 18a7 + 144a5 − 672a3 + 2016a−  6 √  6 √
= ( 2)6 + ( 2)5 (−1)+
4032a−1 + 5376a−3 − 4608a−5 + 0 1
 6 √  6 √
2304a−7 − 512a−9
( 2)4 (−1)2 + ( 2)3 (−1)3 +
Combination notation method: 2 3
 2 9  6 √  6 √
a− ( 2)2 (−1)4 + ( 2)(−1)5 +
a 4 5
       
9 9 2 9 2 2  6 
= a9 + a8 − + a7 − + (−1)6
0 1 a 2 a 6
    
9 2 3 9
 2 4 6! 6! √ 6! 6! √
a6 − + a5 − + = ·8− ·4 2+ ·4− · 2 2+
3 a 4 a 6!0! 5!1! 4!2! 3!3!
    
9 2 5 9
 2 6 6! 6! √ 6!
a4 − + a3 − + ·2− · 2+
5 a 6 a 2!4! 1!5! 0!6!
     √ √ √
9 2 7 9
 2 8 = 8 − 24 2 + 60 − 40 2 + 30 − 6 2 + 1
a2 − + a − + √
7 a 8 a = 99 − 70 2
 
9 2 9 √ √
− ( 2 + 1)6 − ( 2 − 1)6
9 a √ √
= (99 + 70 2) − (99 − 70 2)
9! 9 9! 8  2  9! 7  4  √ √
= a + a − + a + = 99 + 70 2 − 99 + 70 2
9!0! 8!1! a 7!2! a2 √
9! 6  8 9! 5  16 
= 140 2
a − 3 + a +
6!3! a 5!4! a4 19. Expand: (x−2 + x2 )4 .
   
9! 4
a − 5 +
32 9! 3 64
a + We have a = x−2 , b = x2 , and n = 4.
4!5! a 3!6! a6
Pascal’s triangle method: Use the fifth row of Pascal’s
9! 2  128  9!  256  triangle.
a − 7 + a +
2!7! a 1!8! a8
1 4 6 4 1.
9!  512 
− 9
0!9! a

Copyright © 2013 Pearson Education, Inc.


Exercise Set 8.7 323

(x−2 + x2 )4 29. The expansion of (2u − 3v 2 )10 has 11 terms so the 6th
−2 4 −2 3 −2 2
= 1 · (x ) + 4(x 2
) (x ) + 6(x 2 2
) (x ) + term is the middle term. Note that 6 = 5 + 1, a = 2u,
−2 b = −3v 2 , and n = 10. Then the 6th term of the expansion
4(x )(x ) + 1 · (x )
2 3 2 4
of (2u − 3v 2 )10 is
= x−8 + 4x−4 + 6 + 4x4 + x8  
10
Combination notation method: (2u)10−5 (−3v 2 )5
5
(x−2 + x2 )4
10!
    = (2u)5 (−3v 2 )5
4 4 5!5!
= (x−2 )4 + (x−2 )3 (x2 )+ = 252(32u5 )(−243v 10 )
0 1
    = −1, 959, 552u5 v 10
4 4
(x−2 )2 (x2 )2 + (x−2 )(x2 )3 +
2 3
  31. The number of subsets is 27 , or 128
4
(x2 )4 33. The number of subsets is 224 , or 16,777,216.
4
4! −8 4! −6 2 4! −4 4 35. The term of highest degree of (x5 + 3)4 is the first term,
= (x ) + (x )(x ) + (x )(x )+
4!0! 3!1! 2!2! or
 4  4! 20
4! −2 6 4! 8 (x5 )4−0 30 = x = x20 .
(x )(x ) + (x ) 0 4!0!
1!3! 0!4!
= x−8 + 4x−4 + 6 + 4x4 + x8 Therefore, the degree of (x5 + 3)4 is 20.

21. Find the 3rd term of (a + b)7 . 37. We use combination notation. Note that
First, we note that 3 = 2 + 1, a = a, b = b, and n = 7. a = 3, b = i, and n = 5.
Then the 3rd term of the expansion of (a + b)7 is (3 + i)5
   5   5   5 
7 7! 5 2
a7−2 b2 , or a b , or 21a5 b2 . = (35 ) + (34 )(i) + (33 )(i2 )+
2 2!5! 0 1 2
 5   5   5 
23. Find the 6th term of (x − y)10 . (32 )(i3 ) + (3)(i4 ) + (i5 )
3 4 5
First, we note that 6 = 5 + 1, a = x, b = −y, and n = 10.
Then the 6th term of the expansion of (x − y)10 is =
5!
(243) +
5!
(81)(i) +
5!
(27)(−1)+
 10  0!5! 1!4! 2!3!
x5 (−y)5 , or −252x5 y 5 . 5! 5! 5!
5 (9)(−i) + (3)(1) + (i)
3!2! 4!1! 5!0!
25. Find the 12th term of (a − 2)14 . = 243 + 405i − 270 − 90i + 15 + i
First, we note that 12 = 11+1, a = a, b = −2, and n = 14. = −12 + 316i
Then the 12th term of the expansion of (a − 2)14 is
  39. We use combination notation. Note that
14 14! 3 √
a14−11 · (−2)11 = a (−2048)
11 3!11! a = 2, b = −i, and n = 4.
3
= 364a (−2048) √  4 √  4 √
( 2−i)4 = ( 2)4 + ( 2)3 (−i)+
= −745, 472a3 0 1
 4 √  4 √

27. Find the 5th term of (2x3 − y)8 . 2
( 2)2 (−i)2 +
3
( 2)(−i)3 +

First, we note that 5 = 4 + 1, a = 2x3 , b = − y, and  
√ 4
n = 8. Then the 5th term of the expansion of (2x − y)8
3 (−i)4
4
is
  4! 4! √
8 √ = (4) + (2 2)(−i)+
(2x3 )8−4 (− y)4 0!4! 1!3!
4
8! √ 4! 4! √
= (2x3 )4 (− y)4 (2)(−1)+ ( 2)(i)+
4!4! 2!2! 3!1!
= 70(16x12 )(y 2 ) 4!
(1)
= 1120x12 y 2 4!0!
√ √
= 4 − 8 2i − 12 + 4 2i + 1

= −7 − 4 2i

Copyright © 2013 Pearson Education, Inc.


324 Chapter 8: Sequences, Series, and Combinatorics

   
n n term in which the sum of the exponents is −1/6. That is,
41. (a − b)n = an (−b)0 + an−1 (−b)1+
0 1 1  1 1
  (7 − k) + − (k) = −
n 3 2 6
an−2 (−b)2 +· · ·+
2 7 k k 1
 − − =−
n  1
 n 
a (−b)n−1 + a0 (−b)n 3 3 2 6
n−1 n 5k 15
 n   n  − =−
6 6
= (−1)0 an b0 + (−1)1 an−1 b1+
0 1 k=3
 n 
Find the (3 + 1)st, or 4th term.
(−1)2 an−2 b2 +· · ·+  
2 7 √  1 3 7! 4/3
 n  ( 3 x)4 − √ = (x )(−x−3/2 ) =
3 x 4!3!
(−1)n−1 a1 bn−1 +
n−1 35
 n  −35x−1/6 , or − 1/6 .
x
(−1)n a0 bn
n 53. 100 C0 +100 C1 + · · · +100 C100 is the total number of subsets
n 
n 
 of a set with 100 members, or 2100 .
= (−1)k an−k bk 23  
k=0
k  23
55. (loga x)23−k (loga t)k =
k
43. (x + h) − xn
n k=0

h (loga x + loga t)23 = [loga (xt)]23


 n   n   n 
xn + xn−1 h+· · ·+ hn −xn 57. See the answer section in the text.
0 1 n
=
h
 n   n   n  Exercise Set 8.8
= xn−1 + xn−2 h + · · · + hn−1
1 2 n 1. a) We use Principle P .
n 
 n  n−k k−1 For 1: P =
18
, or 0.18
= x h 100
k
k=1
24
45. (f g)(x) = f (x)g(x) = (x2 + 1)(2x − 3) = For 2: P = , or 0.24
100
2x3 − 3x2 + 2x − 3 23
For 3: P = , or 0.23
47. (g ◦ f )(x) = g(f (x)) = g(x2 + 1) = 2(x2 + 1) − 3 = 100
23
2x2 + 2 − 3 = 2x2 − 1 For 4: P = , or 0.23
100
4 
4 
4 
  4  4−k For 5: P =
12
, or 0.12
49. (−1)k x4−k 6k = x (−6)k , so 100
k k
k=0 k=0
b) Opinions may vary, but it seems that people tend
the left side of the equation is sigma notation for (x − 6)4 . not to select the first or last numbers.
We have:
(x − 6)4 = 81 3. The company can expect 78% of the 15,000 pieces of ad-
vertising to be opened and read. We have:
x − 6 = ±3 Taking the 4th root on both
sides 78%(15, 000) = 0.78(15, 000) = 11, 700 pieces.

x−6 = 3 or x − 6 = −3 5. a) Since there are 14 equally likely ways of selecting a


marble from a bag containing 4 red marbles and 10
x=9 or x=3 green marbles, we have, by Principle P ,
The solutions are 9 and 3. 4 2
P (selecting a red marble) = = .
If we also observe that (3i)4 = 81, we also find the imagi- 14 7
nary solutions 6 ± 3i. b) Since there are 14 equally likely ways of selecting a
marble from a bag containing 4 red marbles and 10
√ 1 7 green marbles, we have, by Principle P ,
51. The (k + 1)st term of 3 x − √ is 10 5
x P (selecting a green marble) = = .
    14 7
7 √ 1 k 1
( 3 x)7−k − √ . The term containing 1/6 is the c) Since there are 14 equally likely ways of selecting a
k x x marble from a bag containing 4 red marbles and 10
green marbles, we have, by Principle P ,
0
P (selecting a purple marble) = = 0.
14

Copyright © 2013 Pearson Education, Inc.


Exercise Set 8.8 325

d) Since there are 14 equally likely ways of selecting a d) Seven of the 8 outcomes have 1, 2, or 3 heads. Thus,
marble from a bag containing 4 red marbles and 10 7
P (at least one head) = .
green marbles, we have, by Principle P , 8
e) Three of the 8 outcomes have exactly two tails.
P (selecting a red or a green marble) = 3
4 + 10 Thus, P (exactly two tails) = .
= 1. 8
14
21. The roulette wheel contains 38 equally likely slots. Eigh-
7. There are 6 possible outcomes. There are 3 numbers less teen of the 38 slots are colored black. Thus, by Principle
3 1 P,
than 4, so the probability is , or .
6 2 18 9
4 1 P (the ball falls in a black slot) = = .
9. a) There are 4 queens, so the probability is , or . 38 19
52 13 23. The roulette wheel contains 38 equally likely slots. Only 1
b) There are 4 aces and 4 tens, so the probability is slot is numbered 0. Then, by Principle P ,
4+4 8 2
, or , or . 1
52 52 13 P (the ball falls in the 0 slot) = .
13 1 38
c) There are 13 hearts, so the probability is , or . 25. The roulette wheel contains 38 equally likely slots. Thirty-
52 4
2 six of the slots are colored red or black. Then, by Principle
d) There are two black 6’s, so the probability is , or P,
52
1 36 18
. P (the ball falls in a red or a black slot) = = .
26 38 19
11. The number of ways of drawing 3 cards from a deck of 52 27. The roulette wheel contains 38 equally likely slots. Eigh-
is 52 C3 . The number of ways of drawing 3 aces is 4 C3 . The teen of the slots are odd-numbered. Then, by Principle
probability is P,
4 C3 4 1 P (the ball falls in a an odd-numbered slot) =
= = .
52 C3 22, 100 5525 18 9
= .
13. The total number of people on the sales force is 10 + 10, or 38 19
20. The number of ways to choose 4 people from a group 29. zero
of 20 is 20 C4 . The number of ways of selecting 2 people
from a group of 10 is 10 C2 . This is done for both the men 31. function; domain; range; domain; range
and the women. 33. combination
10 C2 ·10 C2
P (choosing 2 men and 2 women) = =
20 C4 35. factor
45 · 45 135  
= 37. a) There are 13
4845 323 ways to select 2 denominations
2
15. The number of ways of selecting 5 cards from a deck of 52  
from the 13 denominations. Then in each denomina-
cards is 52 C5 . Three sevens can be selected in 4 C3 ways 4
tion there are ways to choose 2 of the 4 cards.
and 2 kings in 4 C2 ways. 2 
4 C3 ·4 C2
44
P (drawing 3 sevens and 2 kings) = , or Finally there are ways to choose the fifth card
1
52 C5
from the 11 remaining denominations (4 · 11, or 44
1
. cards). Thus the number of two pairs hands is
108, 290        
13 4 4 44
17. The number of ways of selecting 5 cards from a deck of · · · , or 123,552.
2 2 2 1
52 cards is 52 C5 . Since 13 of the cards are spades, then 5
123, 552 123, 552
spades can be drawn in 13 C5 ways b) = ≈ 0.0475
52 C5 2, 598, 960
13 C5 1287
P (drawing 5 spades) = = = 39. a) There
52 C5 2, 598, 960   are 13 ways to select a denomination and then
33 4
ways to choose 3 of the 4 cards in that de-
3  
66, 640
48
nomination. Now there are ways to choose 2
19. a) HHH, HHT, HTH, HTT, THH, THT, TTH, TTT 2
cards from the 12 remaining denominations (4 · 12,
b) Three of the 8 outcomes have exactly one head.
3 or 48 cards). But these combinations include the
Thus, P (exactly one head) = . 3744 hands in a full house like Q-Q-Q-4-4 (Exercise
8
38), so these must be subtracted.  Thus
 the number
c) Seven of the 8 outcomes have exactly 0, 1, or 2
7 4 48
heads. Thus, P (at most two heads) = . of three of a kind hands is 13 · · − 3744,
8 3 2
or 54,912.

Copyright © 2013 Pearson Education, Inc.


326 Chapter 8: Sequences, Series, and Combinatorics

54, 912 54, 912   5 


b) = ≈ 0.0211 1
52 C5 2, 598, 960 a1 1 −
31 2
=
2 1
1−
Chapter 8 Review Exercises  2
1
a1 1 −
31 32
=
1. The statement is true. See page 628 in the text. 2 1
2
3. The statement is true. See page 663 in the text. 31
  31 a1
n2 = 32
5. an = (−1)n 2 1
n4 + 1
 2  2
1 1
a1 = (−1)1 4 =− 31
=
31
a1 ·
2
1 +1 2 2 32 1
 2 
2 4 31 31
a2 = (−1)2 4 = = a1
2 +1 17 2 16
 2 
3 9 8= a1
a3 = (−1)3 4 =−
3 +1 82 an = a1 rn−1
 2   5−1  4
4 16 1 1 1 1
a4 = (−1)4 4 = a5 = 8 =8 =8· =
4 +1 257 2 2 16 2
 
112 121  
a11 = (−1)11 =−  0.0027   
114 + 1 14, 642 
19. Since |r| =   = 0.01 = 0.01 < 1, the series has a
  0.27 
232 529 sum.
a23 = (−1) 23
=− 0.27 0.27 27 3
234 + 1 279, 842 S∞ = = = =
4 1 − 0.01 0.99 99 11
(−1)k+1 3k
7.
3k − 1 21. 2.43 = 2 + 0.43 + 0.0043 + 0.000043+ . . . .
k=1
(−1)1+1 31 (−1)2+1 32 (−1)3+1 33 (−1)4+1 34 We will find fraction
 notation for 0.43 and then add 2.
= + + +  0.0043 
31 − 1 32 − 1 33 − 1 34 − 1 |r| =   = |0.01| = 0.01 < 1, so the series has a limit.
3 9 27 81 0.43 
= − + − 0.43 0.43 43
2 8 26 80 S∞ = = =
417 1 − 0.01 0.99 99
=
1040 43 198 43 241
Then 2 + = + =
99 99 99 99

7
9. (k 2 − 1) 23. Familiarize. The distances the ball drops are given by a
k=1 geometric sequence:
 2  3  4  5
11. an = a1 + (n − 1)d 3 3 3 3 3
30, · 30, · 30, · 30, · 30, · 30.
a1 = a − b, d = a − (a − b) = b 4 4 4 4 4
a6 = a − b + (6 − 1)b = a − b + 5b = a + 4b Then the total distance the ball drops is the sum of these
6 terms. The total rebound distance is this sum less 30 ft,
13. 1 + 2 + 3 + . . . +199 + 200 the distance the ball drops initially.
n
Sn = (a1 + an ) Translate. To find the total distance the ball drops we
2
200 will use the formula
S200 = (1 + 200) = 20, 100 a1 (1 − rn ) 3
2 Sn = with a1 = 30, r = , and n = 6.
1−r 4
15. d = 3, a10 = 23 Carry out.
23 = a1 + (10 − 1)3   6 
3
23 = a1 + 27 30 1 −
4 50, 505
S6 = =
−4 = a1 3 512
1−
4
1 31
17. r= , n = 5, Sn = 50, 505
2 2 Then the total rebound distance is − 30, or
12
a1 (1 − rn ) 35, 145
, and the total distance the ball has traveled is
Sn =
1−r 512
50, 505 34, 145 42, 825
+ , or , or about 167.3 ft.
512 512 256

Copyright © 2013 Pearson Education, Inc.


Chapter 8 Review Exercises 327

     
Check. We can perform the calculations again. 1 1 1 1
1− 1− ... 1 − 1−
2 3 k k+1
State. The ball will have traveled about 167.3 ft when it    
hits the pavement for the sixth time. 1 1
= · 1− By Sk
k k+1
25. a), b) Familiarize. We have an arithmetic sequence with  
1 k+1−1
a1 = 10, d = 2, and n = 365. = ·
k k+1
Translate. We want to find an = a1 + (n − 1)d and
n 1 k
Sn = (a1 + an ) where a1 = 10, d = 2, and n = 365. = ·
2 k k+1
Carry out. First we find a365 . 1
= . Simplifying
a365 = 10 + (365 − 1)(2) = 738/c, or $7.38 k+1
365 31. 9 · 8 · 7 · 6 = 3024
Then S365 = (10 + 738) = 136, 510/c or $1365.10.
2 33. 24 · 23 · 22 = 12, 144
Check. We can repeat the calculations.
35. 3 · 4 · 3 = 36
State. a) You will receive $7.38 on the 365th day.
37. 28 = 256
b) The sum of all the gifts is $1365.10. √
39. Expand: (x − 2)5
n(3n − 1)
27. Sn : 1 + 4 + 7 + . . . + (3n − 2) = Pascal’s triangle method: Use the 6th row.
2
1(3 − 1) 1 5 √10 10 5 1 √ √
S1 : 1 = (x − 2)5 = x5 + 5x4 (− 2) + 10x3 (− 2)2 +
2
√ √ √
k(3k − 1) 10x2 (− 2)3 + 5x(− 2)4 + (− 2)5
Sk : 1 + 4 + 7 + . . . + (3k − 2) = √ √ √
2 = x5 − 5 2x4 + 20x3 − 20 2x2 + 20x − 4 2
Sk+1 : 1 + 4 + 7 + . . . + (3k − 2) + [3(k + 1) − 2]
Combination notation
 method:
  
= 1 + 4 + 7 + . . . + (3k − 2) + (3k + 1) √ 5 5 5 5 4 √ 5 3 √ 2
(x− 2) = x + x (− 2)+ x (− 2) +
(k + 1)(3k + 2) 0 1 2
=      
2 5 2 √ 3 5 √ 5 √
x (− 2) + x(− 2)4 + (− 2)5
1(3 − 1) 2 3 4 5
1. Basis step: = = 1 is true. √ √ √
2 2 = x5 − 5 2x4 + 20x3 − 20 2x2 + 20x − 4 2
2. Induction step: Assume Sk . Add 3k + 1 to both sides.  8
1
1 + 4 + 7 + . . . + (3k − 2) + (3k + 1) 41. Expand: a +
a
k(3k − 1)
= + (3k + 1) Pascal’s triangle method: Use the 9th row.
2
1 8 28 56 70 56 28 8 1
k(3k − 1) 2(3k + 1)  8    2  3
= + 1 1 1 1
2 2 a+ = a8 + 8a7 + 28a6 + 56a5 +
3k 2 − k + 6k + 2 a a a a
=  4  5  6
2 1 1 1
70a4 + 56a3 + 28a2 +
3k 2 + 5k + 2 a a a
=
2  7  8
1 1
(k + 1)(3k + 2) 8a +
= a a
2 = a8 + 8a6 + 28a4 + 56a2 + 70+
    
29. Sn : 1 − 1 1 1 1 56a−2 + 28a−4 + 8a−6 + a−8
1− ... 1 − =
2 3 n n Combination notation method:
   8          2
1 1 1 8 8 1 8 1
S2 : 1 − = a+ = 8
a + a7 + a6 +
2 2 a 0 1 a 2 a
        3    4
1 1 1 1 8 1 8 1
Sk : 1 − 1− ... 1 − = a5 + a4 +
2 3 k k 3 a 4 a
         5    6
1 1 1 1 1 1 1
Sk+1 : 1− 1− . . . 1− 1− = 8 8
2 3 k k+1 k+1 a3 + a2 +
5 a 6 a
1. Basis step: S2 is true by substitution.    7   8
8 1 8 1
2. Induction step: Assume Sk . Deduce Sk+1 . Starting with a +
7 a 8 a
the left side of Sk+1 , we have
= a8 + 8a6 + 28a4 + 56a2 + 70+
56a−2 + 28a−4 + 8a−6 + a−8

Copyright © 2013 Pearson Education, Inc.


328 Chapter 8: Sequences, Series, and Combinatorics

43. Find 4th term of (a + x)12 . 63. Choosing k objects from a set of n objects is equivalent to
  not choosing the other n − k objects.
12
a9 x3 = 220a9 x3
3 65. In expanding (a + b)n , it would probably be better to use
45. Of 36 possible combinations, we can get a 10 with (4, 6), Pascal’s triangle when n is relatively small. When n is
(6, 4) or (5, 5). large, and many rows of Pascal’s triangle must be com-
3 1 puted to get to the (n + 1)st row, it would probably be
Probability = = better to use combination notation. In addition, combina-
36 12
Since we cannot get a 10 on one die, the probability of tion notation allows us to write a particular term of the
getting a 10 on one die is 0. expansion more efficiently than Pascal’s triangle.

4 C2 ·4 C1 6·4 6
47. = =
52 C3 22, 100 5525 Chapter 8 Test
49. a) an = 0.1285179017n + 2.870691091
b) In 2010, n = 2010 − 1930 = 80. 1. an = (−1)n (2n + 1)
a80 ≈ 13.2 lb of American cheese a21 = (−1)21 [2(21) + 1]
51. There are 3 pairs that total 4: 1 and 3, 2 and 2, 3 and 1. = −43
There are 6 · 6, or 36, possible outcomes. Thus, we have n+1
3 1 2. an =
, or . Answer A is correct. n+2
36 12
1+1 2
ak+1 bk+1 ak+1 bk+1 a1 = =
53. = r1 , = r2 , so = r1 r2 , a constant. 1+2 3
ak bk ak bk
2+1 3
55. f (x) = x4 − 4x3 − 4x2 + 16x = x(x3 − 4x2 − 4x + 16) a2 =
2+2
=
4
We know that 0 is a zero. 3+1 4
Consider x3 − 4x2 − 4x + 16. a3 = =
3+2 5
Possibilities for p/q: ±1, ±2, ±4, ±8, ±16 4+1 5
 a4 = =
−2  1 −4 −4 16 4+2 6
−2 12 −16 5+1 6
1 −6 8 0 a5 = =
5+2 7
f (x) = x(x + 2)(x2 − 6x + 8) 
4
3. (k 2 + 1) = (12 + 1) + (22 + 1) + (32 + 1) + (42 + 1)
= x(x + 2)(x − 2)(x − 4)
k=1
The zeros are −2, 0, 2, and 4. = 2 + 5 + 10 + 7

10   = 34
57. k 10
(−1) (log x)10−k (log y)k
k 4.
k=0 n Un
= (log x − log y) 10
1 .66667
 10
x 2 .75
= log
y 3 .8
 
59. n 4 .83333
= 36
n−1
5 .85714
n! 2
= 36 6 .875
(n − 1)![n − (n − 1)]!
7 .88889
n(n − 1)!
= 36 8 .9
(n − 1)!1!
n = 36 9 .90909
0 10
10 .91667 0
61. Put the following in the form of a paragraph.
First find the number of seconds in a year (365 days): 
6


✦ · 124day ✦ · 60 sec =
hr 60 min
5. 4k
365 days
✦· 1✧ ✦
k=1
hr 1 min


31,536,000 sec.
6. 2k
The number of arrangements possible is 15!. k=1
15!
The time is ≈ 41, 466 yr.
31, 536, 000

Copyright © 2013 Pearson Education, Inc.


Chapter 8 Test 329

1 
8
7. an+1 = 2 + 14. 2k
an
k=1
1 a1 = 21 = 2, r = 2, n = 8
a1 = 2 + =2+1=3
1
2(1 − 28 )
1 1 S8 = = 510
a2 = 2 + = 2 1−2
3 3
6 1
1 3 3 15. a1 = 18, r = =
a3 = 2 + = 2 + = 2 18 3
7 7 7
1
3 Since |r| = < 1, the series has a sum.
3
1 7 7
a4 = 2 + =2+ =2 18 18 3
17 17 17 S∞ = = = 18 · = 27
1 2 2
7 1−
3 3
8. d=5−2=3
16. 0.56 = 0.56 + 0.0056 + 0.000056 + . . .
an = a1 + (n − 1)d  
 0.0056 
a15 = 2 + (15 − 1)3 = 44 |r| =   = |0.01| = 0.01 < 1, so the series has a sum.
0.56 
9. a1 = 8, a21 = 108, n = 21 0.56 0.56 56
an = a1 + (n − 1)d S∞ = = =
1 − 0.01 0.99 99
108 = 8 + (21 − 1)d 17. a1 = $10, 000
100 = 20d a2 = $10, 000 · 0.80 = $8000
5=d a3 = $8000 · 0.80 = $6400
Use an = a1 + (n − 1)d again to find a7 . a4 = $6400 · 0.80 = $5120
a7 = 8 + (7 − 1)(5) = 8 + 30 = 38 a5 = $5120 · 0.80 = $4096
10. a1 = 17, d = 13 − 17 = −4, n = 20 a6 = $4096 · 0.80 = $3276.80
First find a20 :
18. We have an arithmetic sequence $8.50, $8.75, $9.00, $9.25,
an = a1 + (n − 1)d and so on with d = $0.25. Each year there are 12/3, or
a20 = 17 + (20 − 1)(−4) = 17 − 76 = −59 4 raises, so after 4 years the sequence will have the origi-
Now find S20 : nal hourly wage plus the 4 · 4, or 16, raises for a total of
n 17 terms. We use the formula an = a1 + (n − 1)d with
Sn = (a1 + an )
2 a1 = $8.50, d = $0.25, and n = 17.
20 a17 = $8.50+(17−1)($0.25) = $8.50+16($0.25) = $8.50+
S20 = (17 − 59) = 10(−42) = −420
2 $4.00 = $12.50

25
11. (2k + 1) At the end of 4 years Tamika’s hourly wage will be $12.50.
k=1
a1 (1 − rn )
a1 = 2 · 1 + 1 = 3 19. We use the formula Sn = with a1 = $2500,
1−r
a25 = 2 · 25 + 1 = 51 r = 1.056, and n = 18.
n 2500[1 − (1.056)18 ]
Sn = (a1 + an ) S18 = = $74, 399.77
2 1 − 1.056
25 25
S25 = (3 + 51) = · 54 = 675 20. n(3n + 1)
2 2 Sn : 2 + 5 + 8 + . . . + (3n − 1) =
2
−5 1
12. a1 = 10, r = =− 1(3 · 1 + 1)
10 2 S1 : 2 =
an = a1 rn−1 2
 11−1 k(3k + 1)
1 5 Sk : 2 + 5 + 8 + . . . + (3k − 1) =
a11 = 10 − = 2
2 512
Sk+1 : 2 + 5 + 8 + . . . + (3k − 1) + [3(k + 1) − 1] =
13. r = 0.2, S4 = 1248
(k + 1)[3(k + 1) + 1]
a1 (1 − rn )
Sn = 2
1−r
1(3 · 1 + 1) 1·4
a1 (1 − 0.24 ) 1) Basis step: = = 2, so S1 is true.
1248 = 2 2
1 − 0.2
0.9984a1
1248 =
0.8
a1 = 1000

Copyright © 2013 Pearson Education, Inc.


330 Chapter 8: Sequences, Series, and Combinatorics

2) Induction step: 8 8 4
31. = =
6+8 14 7
2 + 5 + 8 + . . . + (3k − 1) +[3(k + 1) − 1]
  
6 C1 ·5 C2 ·4 C5 6 · 10 · 4 48
k(3k + 1) 32. = =
= + [3k + 3 − 1] By Sk 15 C6 5005 1001
2
33. an = 2n − 2
3k 2 k
= + + 3k + 2 Only integers n ≥ 1 are inputs.
2 2
a1 = 2 · 1 − 2 = 0, a2 = 2 · 2 − 2 = 2, a3 = 2 · 3 − 2 = 4,
3k 2 7k a4 = 2 · 4 − 2 = 6
= + +2
2 2
Some points on the graph are (1, 0), (2, 2), (3, 4), and (4, 6).
3k 2 + 7k + 4 Thus the correct answer is B.
=
2
(k + 1)(3k + 4) 34. n P7 = 9 ·n P6
=
2 n! n!
= 9·
(k + 1)[3(k + 1) + 1] (n − 7)! (n − 6)!
=
2 n! (n − 6)! n! (n − 6)!
· = 9· ·
15! (n − 7)! n! (n − 6)! n!
21. 15 P6 = = 3, 603, 600 (n − 6)(n − 7)!
(15 − 6)! =9
(n − 7)!
21!
22. 21 C10 = = 352, 716 n−6 = 9
10!(21 − 10)!
  n = 15
n n!
23. =
4 4!(n − 4)!
n(n − 1)(n − 2)(n − 3)(n − 4)!
=
4!(n − 4)!
n(n − 1)(n − 2)(n − 3)
=
24
6!
24. 6 P4 = = 360
(6 − 4)!
25. a) 64 = 1296
5!
b) 5 P3 = = 60
(5 − 3)!
28!
26. 28 C4 = = 20, 475
4!(28 − 4)!
12! 8!
27. 12 C8 ·8 C4 = · = 34, 650
8!(12 − 8)! 4!(8 − 4)!
28. Expand: (x + 1)5 .
Pascal’s triangle method: Use the 6th row.
1 5 10 10 5 1
(x+1)5 = x5 +5x4 ·1+10x3 ·12 +10x2 ·13 +5x·14 +15
= x5 + 5x4 + 10x3 + 10x2 + 5x + 1
Combination notation method:
     
5 5 5
(x + 1)5 = x5 + x4 · 1 + x3 · 12 +
0 1 2
     
5 5 5
x2 · 13 + x · 14 + 15
3 4 5
= x5 + 5x4 + 10x3 + 10x2 + 5x + 1

29. Find 5th term of (x − y)7 .


 
7
x3 (−y)4 = 35x3 y 4
4

30. 29 = 512

Copyright © 2013 Pearson Education, Inc.

Das könnte Ihnen auch gefallen